Business Math and Stat Nots by Sir Abdul Ahad

You might also like

Download as pdf or txt
Download as pdf or txt
You are on page 1of 903

1/1/2022 CHAPTER-1

MATHEMATICAL EQUATIONS AND


COORDIANTE SYSTEM

ABDUL AHAD BUTT


Expression:
An expression is a number, a variable or a combination of number and variables
Example:
5+7, or 8÷3
Polynomial: (algebraic expression)
A polynomial is an algebraic expression consisting of one or more terms in each of which the
exponent of the variable is zero or a positive integer.
Example:
13, -x, 5x + 3y, x2 -3x + 1 are all polynomial

Equation:
When two mathematical expressions are considered equal by (=sign). They consider an equation.
Equation consists of coefficients, variables as well as constants

Variable:
A variable is a value that may change within the scope of a given problem or set of operations-
often represented by letters likes x, y and z

Constant:
A constant is a value that remain unchanged

Linear equation:
A linear equation is one in which each term is either a constant or the product of a constant and a
single variable. The equation of straight line is a linear equation with two variables

OPERATIONS ON POLYNOMIALS

Addition of polynomial

Question.1 Add 3x3 + 5x2 - 4x, x3 - 6 + 3x2 and 6 - x2 - x

A. B. C. D.

Solution
Given
3x3 + 5x2 - 4x + 0
x3 + 3x2 + 0x - 6
0x3 - x2 - x + 6
4x3 + 7x2 - 5x

1|Page
Question.2 Add 1 + 2x + 3x2, 3x - 4 - 2x2, x2 - 5x + 4.

A. B. C. D.

Solution
Given
3x2 + 2x + 1
-2x2 + 3x - 4
x2 - 5x + 4
2x2 + 1

Question.3 a3 + 2a2 – 6a + 7, a3 + 2a + 5
2a3 + 2a – a2 - 8
A. B. C. D.

Solution
Given
a3 + 2a2 - 6a + 7
a3 + 2a + 5
3 2
2x - a + 2a - 8
4a3 + a2 - 2a + 4

Question.4 a3 - 2a2b + b3, 4a3 + 2ab2 + 6a2b,


2b3 - 5a3 - 4a2b
A. B. C. D.

Solution
Given
a3 - 2a2b + b3
4a3 + 6a2b + 2ab2
-5a3 - 4x2b + 2b3
oa3 + oa2b + 3b3 + 2ab2

Subtraction of algebraic expression

Question.5 Subtract 2x3 – 4x2 + 8 - x, from


5x4 + x – 3x2 - 9

A. B. C. D.

2|Page
Solution
Given
5x4 + 0x3 - 3x2 + x - 9
+0x4 -2x3 + 4x2 + x - 8
5x4 - 2x3 + x2 + 2x - 17

Question.6 Subtract P from Q when


P = 3x4 + 6x3 + 2x2 - x, Q = 4x4 + 2x2 + x3 - x + 1

A. B. C. D.

Solution
Given
4x4 + x3 + 2x2 - x + 1
- 3x4 - 6x3 -2x2 + x
x4 -5x3 + 0x2 + 0x + 1

Question.7 Subtract 2x + 3y - 4z - 1, from 2y + 3x – 4z + 1

A. B. C. D.

Solution
Given
3x + 2y – 4z + 1
-2x - 3y + 4z + 1
x - y + 0z + 2

Question.8 Subtract a3 + 2a2b + 3ab2 + b3, from


a3 – 3a2b + 3ab2 - b3

A. B. C. D.

Solution
Given
a3 - 3a2b + 3ab2 - b3
- a3 - 2a2b - 3ab2 - b3
oa3 – 5a2b + oab2 - 2b3

3|Page
Multiplication of polynomials
Question.9 Find the product of
3x2 + 2x -4 and 5x2 - 3x + 3 by Horizontal method

A. B. C. D.

Solution
Given
(3x2 + 2x – 4) (5x2 - 3x + 3)
= 15x4 - 9x3 + 9x2 + 10x3 – 6x2 + 6x - 20x2+12x - 12
= 15x4 + (-9 + 10) x3 + (9 – 6 – 20) x2 + (6 + 12) x - 12
= 15x4 + x3 - 17x2 + 18x – 12

Question.10 Multiply
2x - 3 with 5x + 6 by vertical method

A. B. C. D.

Solution
Given
5x + 6
2x - 3

10x2 + 12x
-15x - 18
2
10x - 3x - 18

Question.11 Find product of the following polynomial


i) (x + 3) (x2 - 3x + 9)
ii) (3x2 -7x + 5) (4x2 - 2x + 1)
iii) (a + b + c) (a2 + b2 + c2 - ab - bc - ca)

Division of polynomial

Question.12 Divide x3 - 2x + 4 by x + 2

A. B. C. D.
Solution
x2 - 2x + 2
x+2
√x 3 − 2x + 4
3 2
-x -2x
-2x2 - 2x + 4
4|Page
+2x2 + 4x
2x + 4
- 2x - 4
0
Note: If a polynomial is exactly divisible by another polynomial, then the remainder is zero

Question.13 Simplify (x5 – y5) ÷ (x – y)

A. B. C. D.

Solution
x4 + x3y + x2y2 + xy3 + y4
x− y
√x 5 − y5
- x5 +x4y
x4 y - y5
-x4y + x3y2
x3y2 – y5
- x3y2 + x3y2
x3y2 – y5
3 2 4
- x y + xy

xy4 – y5
-xy4 +y5
x
Question.14 Simplify (16a + 4a – 4a + 3a - 1) ÷ (4a2 – 2a + 1)
5 3 2

A. B. C. D.

Solution
Given
4a3 + 2a2 + a - 1
4a2 −2a+1
√16a5 + 4a3 − 4a2 + 3a − 1
- 16a5 + 8a4 - 4a3
8a4 - 4a2 + 3a - 1
- 8a4 -2a2 +4a3
4a3 - 6a2 +3a -1
- 4a3 + 2a2 -a
-4a2 + 2a -1
-4a2 – 2a +1
x

5|Page
LINEAR EQUATION
Graph

6|Page
(Find Value of x)
Question.15 Evaluate
2x−3 x+1
= x+6 + 1
x−3

A. B. C. D.

Solution
Given
2x−3 x+1
= x+6 + 1
x−3
2x−3 x+1+x+6
=
x−3 x+6
2x−3 2x+7
=
x−3 x+6

(2x - 3) (x + 6) = (x - 3) (2x + 7)
2x2 + 12x - 3x – 18 = 2x2 + 7x – 6x – 21
9x - 18 = x - 21
9x - x = -21 + 18
8x = -3
x = -3/8
Solution set = {-3/8}

Question.16 Evaluate
2x−5 x−1
+1 = x−3 + 2
x+4

A. B. C. D.

Solution
Given
2x−5 x−1
+1 = +2
x+4 x−3
2x−5+x+4 x−1+2x−6
=
x+4 x−3
3x−1 3x−7
=
x+4 x−3
(3x - 1) (x - 3) = (3x - 7) (x + 4)
3x2 - 9x - x + 3 = 3x2 + 12x - 7x - 28
-10x + 3 = 5x - 28
-10x - 5x = -28 - 3
-15x = -31
x = 31/15

7|Page
Question.17 Solve
1 1 2
+ x+1 = x+3
x

A. B. C. D.

Solution
Given
1 1 2
+ x+1 = x+3
x
x+1+x 2
= x+3
x(x+1)
2x+1 2
= x+3
x2 +x

(2x + 1) (x + 3) = 2(x2 + x)
2x2 + 6x + x + 3 = 2x2 + 2x
7x + 3 = 2x
7x – 2x = -3
5x = -3
x = -3/5

Question.18 Simplify
2x−10 x−2
+3 = x−3 + 4
x+4

A. B. C. D.

Solution
Given
2x−10 x−2
+3 = x−3 + 4
x+4
2x−10+3(x+4) x−2+4(x−3)
=
x+4 x−3
2x−10+3x+12 x−2+4x−12
=
x+4 x−3
5x+2 5x−14
=
x+4 x−3

(5x + 2) (x - 3) = (5x - 14) (x + 4)


5x2 - 15x + 2x - 6 = 5x2 + 20x - 14x - 56
-13x - 6 = 6x - 56
-13x - 6x = -56 + 6
-19x = -50
x = 50/19

8|Page
Question.19 Solve the following
4−x 8(x−2)
2x + 2 = 3x - 4

A. B. C. D.

Solution
Given
4−x 8(x−2)
2x + = 3x -
2 4
4x+4−x 12x−8x+16
=
2 4
3x+4 4x+16
=
2 4

4(3x + 4) = 2(4x + 16)


12x + 16 = 8x + 32
12x - 8x = 32 - 16
4x = 16
x=4

Question.20 Solve the equation


12x − 5 4x+8
= 4
3

A. B. C. D.

Solution
Given
12x − 5 4x+8
=
3 4

4(12x - 5) = 3(4x + 8)
48x - 20 = 12x + 24
48x – 12x = 24 + 20
36x = 44
x = 44/36
11
x= 9

Question.21 Solve the following equation


7x+ 8 5
=
3x+1 3

A. B. C. D.
Solution

9|Page
Given
7x+ 8 5
=3
3x+1

3(7x + 8) = 5(3x + 1)
21x + 24 = 15x + 5
21x - 15x = 5 - 24
6x = -19
19
x=− 6

Question.22 Solve the following


1 1 2
+ x+1 = x+3
x

A. B. C. D.

Solution
Given
1 1 2
+ x+1 = x+3
x
x+1+x 2
=
x(x+1) x+3
2x+1 2
=
x2 +x x+3

(x + 3) (2x + 1) = 2(x2 + x)
2x2 + x + 6x + 3 = 2x2 + 2x
7x + 3 = 2x
7x - 2x = -3
5x = -3
x = -3/5

Question.23 Solve for x the following equation


1 x 5x 5x x−12 x+3
(4 − 3) + - = -
3 6 4 5 3

A. B. C. D.

Solution
Given
1 x 5x 5x x−12 x+3
( − 3) +
3 4 6
- 4
= 5
- 3

10 | P a g e
x 5x 5x x−12 x+3
-1 + - = -
12 6 4 5 3
x−12+10x−15x 3(x−12)−5(x+3)
=
12 15
−4x−12 −2x − 51
=
12 15
15(-4x - 12) = 12(-2x - 51)
-60x - 180 = -24x - 612
-60x + 24x = -612 + 180
-36x = -432
432
x= 36
x = 12

Question.24 Solve the following equation


3 5 2 3
(x − 1) - (x-4) = 5 (x-6) + 4
16 12

A. B. C. D.

Solution Given
3 5 2 3
(x − 1) - (x-4) = 5 (x-6) + 4
16 12
3x−3 5x−20 2x−12 3
- = +4
16 12 5
3(3x−3)−4(5x−20) 4(2x−12)+3(5)
=
48 20
9x−9−20x+80 8x−48+15
=
48 20
−11x+71 8x−33
=
48 20
20(-11x + 71) = 48(8x - 33)
-220x + 1420 = 384x - 1584
-604x = - 3004
3004
x= 604
751
x = 151

K-1
Question.25 Exact value of √𝟎. 𝟎𝟒 is

A. B. C. D.
Solution
Given
√0.04

11 | P a g e
= 0.2
K-2
𝟏
Question.26 Starting with the smallest, following numbers in order of size are 𝟑 , 0.32, 0.32

A. B. C. D.

Solution
Given
1
, 0.32, 0.32
3
0.33, 0.09, 0.32
Arrange
0.09, 0.32, 0.33

K-3
𝟏𝟐𝐱 𝟐 𝐲 𝟑
Question.27 Simplified from of is
𝟒𝐱 𝟑 𝐲

A. B. C. D.

Solution
Given
12x 2 y 3
4x 3 y
3y2
= x
K-4
𝟐 𝟐⁄ 𝟐⁄
Question.28 Evaluation of 𝟔𝟑 x 𝟔 𝟑 x𝟔 𝟑 is

A. B. C. D.

Solution
Given
2 2⁄ 2⁄
63 x 6 3 x6 3
2 2 2
= 63+3+3
2+2+2
=6 3
6
= 6 ⁄3
= 62
= 36

12 | P a g e
K-6
Question.29 If x = 3 and y = 1/6, then find value of x = 1/2y

A. B. C. D.

Solution
Given
1
x = 2y
1
x = 2(1⁄
6)
1
x = 1⁄
3
x=3

K-7
Question.30 An Airliner has seats for 120 passengers. The number of passengers on board
when 7/15 of the seats are occupied is:

A. B. C. D.

Solution
Given
7
x=120 x 15
x= 8 x 7
x = 56
K-19
𝟏𝟑
Question.31 Simplest form of the fraction which is exactly half way(average) between 𝟏𝟕
𝟏𝟒
and 𝟏𝟕 is:

A. B. C. D.

Solution
Given
13 14
and 17
17
Then to find half way. Find average.
13 14
+
17 17
= 2
27⁄
17
= 2

13 | P a g e
27
= 34

K-20
Question.32 if V = X2H + AL, then x is term if V, H, A and L is:

A. B. C. D.

Solution
Given
V = X2H + AL
V - AL = X2H
V−AL
=X2
H
V−AL
x=√ H

K-12
Question.33 Factors of expression ax2 + bx2 - ay2 - by2 are

A. B. C. D.

Solution
Given
ax2 + bx2 - ay2 - by2
= x2(a + b) - y2(a + b)
= (x2 - y2) (a + b)
= (x + y) (x - y) (a + b)

K-23
Question.34 Simplified
𝐱 𝐱
(𝐱−𝟏 − 𝟏) ÷ (𝟏 + 𝟏−𝐱) is

A. B. C. D.

Solution
Given
x x
(x−1 − 1) ÷ (1 + 1−x)
x−x+1 1−x+x
=( x−1
)÷( 1−x
)
1 1
= (x−1) ÷ (1−x)

14 | P a g e
1 1−x
= x−1 x 1
1 −(x−1)
= x−1 x 1
= -1

K-16
Question.35 A number lies between 90 and 100. When it is divided by 7 these is a remainder
of 5 then required number is:

A. B. C. D.

Solution

Given
Number between 90 and 100 divided by 7 and remainder is 5 is
13
7
√96
91
5

K-25
Question.36 Factors of expression a2b2 + b2 - 2ab2 - 4 are

A. B. C. D.

Solution
Given
a2b2 + b2 - 2ab2 – 4
= a2b2 -2ab2 + b2 - 4
= (ab - b)2 - (2)2
= (ab - b + 2) (ab - b - 2)

K-18
𝟐𝒚𝟐 +𝟐
Question.37 If x2y = (√𝒙)

A. B. C. D.

Solution
Given
2y2 +2
x2y = (√x)
2y2 +2
2y
x = (𝑥) 2

15 | P a g e
Compare
2y = y2 + 1
y2 - 2y + 1 = 0
(y - 1)2 = 0
y-1=0
y=1
K-19
𝟏
Question.38 Factors of 3x2 - 𝟑 y2 are

A. B. C. D.

Solution
Given
1
3x2 - 3 y2
1
= 3 (9x2 - y2)
1
= 3 [(3x)2 - (y)2]
1
= 3 (3x + y) (3x - y)
K-28
Question.39 If √𝐲 = 64x3 then 𝟑√𝐲 =

A. B. C. D.

Solution
Given
√y = 64x3
1⁄
(y) 2= 64x3
1⁄
y 2= 43x3
Taking power 2/3 on both side
1⁄
y 2= (43x3)2/3
1
y = (4x)2
3
3
√y = 16x2

16 | P a g e
2003
Question.40 Solve the following exponential function
(25)x+2 = 53x-4

A. B. C. D.

Solution
Given
(25)x+2 = 53x-4
(52)x+2 = 53x-4
52x+4 = 53x-4
2x + 4 = 3x - 4
4 + 4 = 3x - 2x
8=x

17 | P a g e
CO-ORDINATE SYSTEM:
Coordinate Geometry is widely used in social-sciences and business where we want to create
linear-relations if we have two coordinate systems. We can predict the future outcomes through
liner relation.

1-Coordinate system/Rectangular System/xy plane (x, y) = (abscissa, ordinate)


is a method to represent numbers in the xy plane.

(−𝑥, +y) (+𝑥, +y)


4 P(2, 4)

(−𝑥, -y) (+x, -y)

How to make an equation (we are discussing about straight line) or liner equation: -?

2-Distance formula between two points

d = √(x1 − x2 )2 + (y1 − y2 )2
or
d = √(x2 − x1 )2 + (y2 − y1 )2

3-Slope: or gradient –
(i) Slope measure the change
change in dependent variable 𝐲 −𝐲
(ii) Slope is the ratio between change in independent variable
= 𝐱𝟐 −𝐱𝟏
𝟐 𝟏
Note:
(i) It is denoted by m
y −y
(ii) It two points are given then slope is computed by using formula m = x2 −x1
2 1

18 | P a g e
4-Requirement to find an equation
To make an equation or straight-line or generating liner relation.
1. Two points are given
2. One point and slope are given

5- SLOPE INTERCEPT FORM


Slope can also find by making
y = mx + c
Where
m = slope
c = y- intercept
6-Equation of line
y – y1 = m (x - x1)

7-Slope:
(i) Slop of parallel line are “Same”
(ii) Slope of perpendicular liner are negative reciprocal
(iii)Slope of collinear points are always same
A B C D E
Slope of AB = Slope CD = Slope of BC

8- Intercept:

(i) x-intercept:
If a line cut x-axis then it is called x-intercept
(ii) y-intercept:

If a line cut y-axis then it is called y-intercept. A line that passes through origin (0, 0) having no
intercept.

9- Two intercept form:


x y
+ =1
a b

10- satisfy Equation


The point (2, 3) lies on the line or passes through the point we should put that point on the given
line 2x – 3y = -2 must satisfy the equation.

19 | P a g e
Question.1 Find the length of the straight-line connecting points A and B having co-
ordinates (2, -5) and (-4, 3) respectively.

A. B. C. D.

Solution:
Given points are
(2, -5) and (-4, 3)
The required distance is
d = √(x2 − x1 )2 + (y2 − y1 )2
d = √(−4 − 2)2 + (3 + 5)2
d = √(−6)2 + (8)2
d = √36 + 64
d = √100
d = 10
Question.2 In the diagram B is the point (0, 16) and C is the point (0, 6). The sloping line
through B and the horizontal line through C meet at the point A. then equation
of the line AC is.
Y

B (0, 16)
A
C (0, 6)
x-asix
0

A. B. C. D.

Solution:
Since line AC is parallel to x-axis and slope of x-axis is Zero.
Slope of line AC= m = 0
Now equation of line passes through C (0, 6) and having slope is
y - y1 = m(x – x1)
y - 6 = 0(x - 0)
y-6=0
y=6

20 | P a g e
Question.3 A line passes through the point (0, 5) and has gradient - 2. The equation of the
line is:

A. B. C. D.

Solution:
Equation of line passing through (0, 5) and having slop -2 is
y - y1 = m(x - x1)
y - 5 = -2(x - 0)
y = -2x + 5
y = 5 - 2x

Question.4 A slope of the line perpendicular to the line 3x - 4y + 5 = 0 is.

A. B. C. D.

Solution:
Given line
3x - 4y + 5 = 0 where a =3 b = -4 line 2 (slope)
a
Slope of line = - b
3
= 4
3
m = slope of line = 3x - 4y + 5 = 0 (line-1)
4
1
Slope of perpendicular line = - m
−1
= 3⁄
4
= -4/3
Question.5 Whether the part of lines 3x = y + 7 and x + 3y = 7 are parallel, perpendiculars
or neither:

A. B. C. D.

Solution:
Given line
3x = y + 7
3x - y = 7 (i)
Also
x + 3y = 7 (ii)
Slope of line (i)
−a −3
m1 = = −1 = 3
b

21 | P a g e
Slope of line (ii)
−a −1
m2 = =
b 3
If product of slopes are -1 then lines are perpendicular
−1
m1 m2 = (3) ( 3 )
m1 m2 = -1
Lines are perpendicular

Question.6 Find y - intercept and slope of equation 3y = 9 - 12x are:

A. B. C. D.

Solution:
Given
3y = 9 - 12x
3y = -12x + 9
y = -4x + 3
Compare with
y = mx + c
m = slope = -4, y – intercept = c = 3

Question.7 y = c is equation of straight line parallel to

A. B. C. D.

Solution:
Given
y=c
When y = c is constant then line is parallel to x-axis
Slope of x-axis = 0
Slope of required line is
dy
m = dx = tan (0) = 0

Question.8 A firm’s fixed costs are Rs. 50,000 per week and the variable cost is Rs. 10 per
unit. The total cost function for the firm is:

A. B. C. D.
Solution:
Given
Total cost = Fixed cost + variable cost

22 | P a g e
y = a + bx
y = 50,000 + 10x
y = 10x + 50,000

Question.9 The slope of the straight-line y = 2 - 3x is

A. B. C. D.

Solution:
Given
y = 2 - 3x
y = -3x + 2
Compare with
y = mx + c
Slope = m = -3

Question.10 The total cost curve of the number of copies of a particular photograph is
linear. The total cost for 5 and 8 copies of a photograph are Rs. 80 and Rs. 116
respectively the total cost for ten copies of the photograph will be:

A. B. C. D.

Solution:
let
The required number of copies = x
The cost of copies = y
Then
P1(Copies, cost) and P2(Copies, cost)
(5, 80) and (8, 116)]
Now required linear equation is
y −y
y – y1 = x2−x1 (x - x1)
2 1
36
y - 80 = (x - 5)
3
y - 80 = 12 (x - 5)
Now put x = 10
y - 80 = 12 (10 - 5)
y - 80 = 12 (5)
y - 80 = 60
y = 60 + 80
y = 140

23 | P a g e
Question.11 A manufacturer produces 80 T.V. sets at a cost Rs. 220000 and 125 T.V. sets
at a cost of Rs. 287500. Assuming the cost curve to be linear. The cost of 95 sets
with the help of equation of the line is:

A. B. C. D.

Solution:
let
The required number of T.V. sets = x
The required cost of T.V. = y
Than the required points to from a linear equation is
(80,220,000) and (125,287,500)
Required linear equation is
y −y
y - y1 = x2−x1 (x - x1)
2 1
287500−220000
y - 220000 = (x - 80)
125−80
67500
y - 220000 = (x - 80)
45
y - 220000 = 1500 (x - 80)
Put x = 95
y - 220000 = 1500 (95 - 80)
y - 220000 = 1500 (15)
y - 220000 = 22500
y = 242500

Question.12 Pair of points (x, y) that satisfy the equation 4x + 3y = 29 are


i) (2, 7) and (3, 5) ii) (7, 2) and (5, 3)
iii) (2, 7) and (5, -3) iv) (2, 7) and (5, 3)

A. B. C. D.

Solution:
If given points satisfy the required equation then that pair of points are called required points.
Option (iv) (2, 7) and (5, 3) are the required points

Question.13 Line has equation y = 3x + 7 and passes through the point (h, h + 15), then the
value of h is:

A. B. C. D.

24 | P a g e
Solution:
Since the given line y = 3x + 7
Pass through the point (h, h + 15)
Point (h, h + 15) satisfy the equation
So put (h1 h + 15) in equation
y = 3x + 7
h + 15 = 3(h) + 7
h + 15 = 3h + 7
15 - 7 = 3h - h
8 = 2h
h = 8/2
h=4

Question.14 The point (p, 2p) lies on the straight-line x + 4y = 36. The value of p is.

A. B. C. D.

Solution:
Since the point (p, 2p) lies on the line so it must satisfy the equation.
x + 4y = 36
p + 4(2p) = 36
p + 8p = 36
9p = 36
p=4

Question.15 For the profit function


P = -q2 + 17q - 42
Break even points are

A. B. C. D.

Solution:
Breakeven is a point where there is no profit and no loss.
i-e
p=0
-q2 + 17q - 42 = 0
q2 - 17q + 42 = 0
q2 - 14q - 3q + 42 = 0
q(q - 14) -3 (q - 14) = 0
Either q - 3 = 0 or q - 14 = 0
q=3 or q = 14

25 | P a g e
Question.16 The equation of line joining the points (3, 5) to the point of intersection of the
lines 4x + y - 1 = 0 and 7x – 3y – 35 = 0
l2 l1

(3, 5)

P1 P2

A. B. C. D.

Solution:
Since the point lie on the line and line is passes through the point of intersection of two lines, so
first we find the point of intersection of two line.
4x + y - 1 = 0
7x - 3y - 35 = 0
7(i) – 4(ii)
28x + 7y = 7
-28x + 12y = -140
19y = -133
y = -7
Putting value of ‘y = -7’ in (i)
4x - 7 = 1
4x = 1 + 7
4x = 8
x=2
Point of intersection is (2, -7)
Now equation of line passing through (3, 5) and (2, -7) is
y −y
y - y1 = x2−x1 (x - x1)
2 1
−7−5
y-5= (x - 3)
2−3
−12
y-5= (x - 3)
−1
y - 5 = 12 (x - 3)
y - 5 = 12x - 36
12x - y - 36 + 5 = 0
12x - y - 31 = 0

26 | P a g e
Question.17 The sum of the intercepts of a straight line on the axis is 5 and product of the
intercepts is 6. Then equation of one of the lines is:

A. B. C. D.

Solution:
Let the x intercept = a
y-intercept = b
Then
Sum of intercept = a + b = 5 ----------(i)
Product of intercept = ab = 6 ----------(ii)
From (i)
a+b=5
a=5–b ---------- (iii)
Putting value of ‘a’ in (ii)
(5 - b)b = 6
5b - b2 = 6
b2 - 5b + 6 = 0
b2 - 3b-2b + 6 = 0
b(b - 3) -2 (b - 3) = 0
(b - 2) (b - 3) = 0
b-2=0 or b-3=0
b=2 or b=3
Putting value of b = 2 and b = 3 in (iii)
When When
a=5-b a=5-b
than b = 2 than b = 3
a=5-2 a=5-3
a=3 a=2
So (a, b) = (3, 2) and (a, b) = (2, 3)
And equation is
x y x y
+ =1 and +b=1
a b a
x y x y
+ =1 and +3=1
3 2 2

Question.18 The equation of the line passing through the point of intersection of 2x + 3y -
5 = 0 and 7x - 5y - 2 = 0 and parallel to the line 2x - 3y + 14 = 0

A. B. C. D.

Solution:
First we find point of intersection of l1 and l2

27 | P a g e
2x + 3y - 5 = 0 ------- (i)
7x - 5y - 2 = 0 ------- (ii)
7(i) -2 (ii)
14x + 21y - 35 = 0
-14x + 10y + 4 = 0
31y - 31 = 0
31y = 31
y=1
Putting value of ‘y’ in (i)
2x + 3(1) - 5 = 0
2x + 3 - 5 = 0
2x - 2 = 0
2x = 2
x=1
Also
2x - 3y + 14 = 0 is a line parallel to the required line
Slop of parallel lines are same
Slop of parallel line = 2/3
Equation of required line having
Slope 2/3 and passing through point (1, 1) is
y - y1 = m(x - x1)
2
y - 1 = 3 (x - 1)
3(y - 1) = 2(x - 1)
3y - 3 = 2x - 2
2x - 3y + -2 + 3 = 0
2x - 3y + 1 = 0

Question.19 The cost of a product in rupees is c = 15x + 9750 where x is the number of items
produced. If selling price of each item is Rs. 30. The sales quantity at which
there would be no profit is.

A. B. C. D.

Solution:

Note: Revenue = Selling price x No of units

The sales quantity at which there is no profit and no loss is called breakeven point, where
R(x) = C(x)
30(x) = 15x + 9750
30x - 15x = 9750
15x = 9750
x = 650

28 | P a g e
Question.20 A manufacturer sells a product at Rs. 8 per unit. Fixed cost is Rs. 5000 and the
variable cost is Rs. 22/9 per unit the total output at breakeven point is:

A. B. C. D.

Solution:
Breakeven point is a point when there is no profit and no loss i-e
R(x) = C(x)
Selling price x Number of units = variable cost + Fixed cost
22
8x = x + 5000
9
22
8x - x = 5000
9
72x−22x
= 5000
9
72x - 22x = 45000
50x = 45000
x = 900 units

Question.21 A firm is introducing a new washing detergent. The firm plans to sell the
family size box for Rs. 24. Production estimates have shown that the variable cost of
producing one unit of the product is Rs. 21.60. Fixed cost of production is Rs. 36,000. It is
assumed that both the total revenue and total cost functions are linear over the relevant sales
quantity range. Then the break-even volume of sales is.

A. B. C. D.

Solution:
As we know break-even is a point where
R(x) = C(x)
24x = 21.60x + 36000
24x - 21.60x = 36000
2.40x = 36000
x = 15000 boxes

2001
Question.22 A transport firm is planning to purchase new buses. The budget allocated for
this purpose is Rs. 50 million. Two types of buses are to be purchased. One
costing
Rs. 300,000 each and other Rs. 500,000 each. If x equals the number of type I
buses and y the number of type 2 buses.
i) Determine the linear equation which states that the total amount spent
this purchase equals to Rs. 50 million.

29 | P a g e
ii) Rewrite the equation in slope intercept form (take y as dependent
variable)
iii) Interpret the meaning of slope and y-intercept.

A. B. C. D.

Solution:
(i) let
Number of type-1 buses = x
Number of type-2 buses = y
According to given condition
Total cost = Type 1 buses cost + Type II buses cost
50,000,000 = 300,000x + 500,000 y
ii) Slope-intercept form
Given equation is
50,000,000 = 300,000x + 500,000 y
y = - 0.6x + 100

iii) Interpretation:
Slope: Since the value 0.6 is representing slope. That will change the value of
independent variable by 0.6 every time.

Y-intercept: Since the value of y-intercept is 100, therefore, the firm can
purchase100 more type-II buses as compared to type-I buses

2002
Question.23 The real estate association in an urban area has determined that a linear
function can be used to relate monthly housing sales (the dependent variable
y) and the average monthly mortgage interest rate (the dependent variable x).
The function is of the form.
y = 4500 - 90x
i) Interpret the y-intercept and slope for this function.
ii) If the monthly mortgage interest rate is 14. What are the expected
housing sales for the month?
iii) If the mortgage interest rate declines from 15 to 12.5. What is the
expected change in housing sales?

A. B. C. D.

Solution:
i) Given function

30 | P a g e
y = 4500 – 90x
Compare with
y = mx + c
Slope = m = -90
y-intercept = c = 4500
The value of slope, interprets that as the value of x increases the total sales
of houses will be decrease.
ii) Given
x = 14
Put in
y = 4500 - 90x
y = 4500 - 90(14)
= 4500 - 1260
y = 3240

If the rate of interest is 14% then the number of houses ready to sale is 3240.
iii) Put x = 15 in
y = 4500 - 90x
y = 4500 - 90(15)
y = 4500 - 1350
y = 3150
Also
Put x = 12.5 in
y = 4500 - 90x
= 4500 - 90(12.5)
= 4500 - 1125
y = 3375
So if rate of interest decreases the sale of houses will be increase.

Question.24 Determine which pair of given lines are parallel, perpendicular or neither.
Given reason in each case and show your calculation.
(i) 3x = y + 7 and x + 3y = 7
(ii) x+y=4 and x - 2y = 3
(iii) 2x + 3y = 5 and 4x + 5 = -6y

A. B. C. D.

Solution:
(i) Given equations are
3x = y + 7 x + 3y = 7
3x - y – 7 = 0 x + 3y -7 = 0

31 | P a g e
−a −3 −a
Slope of line I = m = = −1 Slope of line II = m2 =
b b
−1
Slope = m1 = 3 = m2 = 3
Now Product of slopes = m1 m2
1
= (3) - 3
= -1
Since the product of slopes are -1, lines are perpendicular
(ii) Line- I Line- II
x+y=4 x - 2y = 3
a a
Slope of line I = m1 = − b Slope of line II = m2 = − b
−1 −1
m1 = = -1 m2 = −2
1
m2 = 1/2
Since m1 and m2 are not equal and also their product is also not equal to -1. So lines are
neither parallel nor perpendicular
(iii) Line- I Line- II
2x + 3y = 5 4x + 5 = -6y
4x + 5 = -6y
a a
Slope of line I = m1 = − b Slope of line II = m2 = − b
−2 4
m1 = m2 = -6
3
m2 = -2/3
Since slope are equal i-e m1 = m2
Hence lines are parallel to each other.

Question.25 The demand of a product at a price of Rs. 5 per unit in 80 units. When the
price is increased to Rs. 12 per unit? The demand function is reduce to 45
units. Determine the linear equation of the demand function in the form
Q = f(p).

A. B. C. D.
Solution:
Let
The number of unit = x
The price = y
Then required points are
(5, 80) and (12, 45)
For linear equation, we use
y −y
y - y1 = x2−x1 (x - x1)
2 1
45−80
y - 80 = (x - 5)
12−5
35
y - 80 = − (x - 5)
7
y - 80 = -5 (x - 5)
y - 80 = -5x + 25
32 | P a g e
y = -5x + 25 + 80
y = -5x +105
is the required linear function

Question.26 Find the equation of a straight line which intersect the y-axis at y = 4 and the
x-axis at x = 8

A. B. C. D.

Solution:
Equation of straight- line which cuts y-axis at y = 4 and x-axis at x = 8 is
x y
+ =1
a b
x y
8 4
+ =1
x+2y
=1
8
x + 2y = 8
−1 8
y= x+2
2
is the required equation of line.

Question.27 The average price of an item ‘e’ is directly related to the quantity ordered. The
average price is Rs. 35 when 250 items are ordered and Rs. 55 if only 50 items
are ordered. Identify the linear price function and calculate the price per item
if a quantity of 115 is ordered.

A. B. C. D.
Solution:
let
The required price = p
The required quantity = q
Then the linear function passes through points (250, 35) and (50, 55) is
p - p1 = m (q - q1)
p −p
p - p1 = q2 −q1 (q - q1)
2 1
55 −35
p - 35 = 50−250 (q - 250)
20
p - 35 = −200 (q - 250)
p - 35 = 0.1(q - 250)
Now put q = 115 then
p - 35 = -0.1 (115 - 250)
p - 35 = -0.1 (-135)
p - 35 = 13.5

33 | P a g e
p = 35 + 13.5
p = Rs. 48.50

Note: Slope of line having same intercept is always equal to ‘-1’

Question.28 A line passes through the point (3, 5) and has same value of x - intercept and
y - intercept. Find equation of the line.

(o, a)

(a, 0)

A. B. C. D.

Solution:
Equation of line passing through (3, 5) and having slope ‘-1’ is:
y - y1 = m (x - x1)
y - 5 = -1(x - 3)
y - 5 = -x + 3
x + y - 8 = 0 is the required equation of line.

Question.29 The y-intercept of a line has the same value as its gradient. If this line cuts the
curve y = x3 - 2x2 - 3x - 8 at x = 3. Find the equation of the line.

A. B. C. D.

Solution:
Let the value of y-intercept = t
Since the slope and y - intercept are same
m = t ,c= t
Now equation of line having y - intercept and slope is.
y = mx + c
y = tx + t -------- (i)
Also
Given
y = x3 - 2x2 - 3x - 8
Put x = 3, to find value of y
y = (3)3 -2 (3)2 - 3(3) - 8

34 | P a g e
y = 27 - 18 - 9 - 8
y = -8
Required point is (3, -8)
Since the line cut the curve at point (3, -8). So this point satisfies the equation
y = tx + t
-8 = t(3) + t
-8 = 3t + t
-8 = 4t
t = -2
putting value of ‘t’ in (i), we get
y = -2x - 2
is the required equation

Question.30 The equation y = 50,000x + 1,000,000 represents the value of


i) Find the slope of the line and intercept its meaning.
ii) Find the y-intercept and explain what it represent?

A. B. C. D.

Solution:
Given
y = 50,000x + 1,000,000
Here
Slope = 50,000
Y-intercept = 1,000,000
Slope:
The value of slope represents the change in price.
Y-intercept
The value of intercept represents. The actual value of the product.

Question.31 A buyer spent 30,151 on 550units of a particular item. The first 100 units cost
50 each . The next 150 units cost 8250 in total. The next batch cost 11200 in total and final
100 cost x each. The value of x is
A. B. C. D.

Solution:
Total cost =30151
30151 =100(50) +8250 +11200+100(x)
5701 =100x
x=57.01

35 | P a g e
Question.32 The line joining (-1, 1) and (2, -2) and the line joining (1, 2) and (2, k) are
perpendicular to each other for the following value of k

Solution:
Slop of (-1, 1) (2, -2) and (1, 2) (2, k)
−2 −1 −3
Slope= m1 = = = −1
2+1 3

Slope of (1, 2) and (2, k) is


𝑘−2
Slope=m2 = 2−1 = 𝑘 − 2

Since lines are perpendicular

(-1) (k-2) = -1

-k + 2 = -1

-k = - 3

k=3

Question.33 A factory produces 200 bulbs for a total cost of Rs 800/- and 400 bulbs for Rs
1200. The equation of total cost line

Solution:
Required points are (200, 800) & (400, 1200)

Slope of line is
1200−800
m = 400−200

400
m = 200

m=2

Equation of line having slope 2 and point (200, 800 is

𝑦 – 800 = 2(𝑥-200)

𝑦 – 800 = 2 𝑥 – 400

2 𝑥 − 𝑦 + 400 = 0

36 | P a g e
Question.34 The equation of straight line passing through the points (-5, 2) and (6, -4) is

Solution:
Slope of line is
𝑦2−y1
m = 𝑥2−𝑥1 ,

−4−2
m= ,
6+5

−6
m = 11

Required equation is 𝑦 − 𝑦1 = 𝑚 (𝑥 − 𝑥1)


−6
𝑦−2= (𝑥 + 5)
11

11(𝑦 − 2) = −6(𝑥 + 5)

11𝑦 − 22 = −6𝑥 − 30

6𝑥 + 11𝑦 = −8

6𝑥 + 11𝑦 + 8 = 0

Question.35 The equation of line through (-1,3) and parallel to the line joining (6, 3) and (2, -
3) is

Solution:
Slope of line
−3−3
m= 2−6

−6
m = −4

3
m= 2

3
Equation of line passing through (-1, 3) and having slope is
2

y- y1 = m(x-x1)
3
𝑦 − 3 = 2 (𝑥 + 1)

2𝑦 − 6 = 3𝑥 + 3

37 | P a g e
3𝑥 − 2𝑦 + 9 = 0

Question.36 If the lines 3𝒙 − 𝟒𝒚 + 𝟏𝟑 = 𝟎, 𝟖𝒙 − 𝟏𝟏𝒚 + 𝟑𝟑 = 𝟎, 𝟐𝒙 − 𝟑𝒚 + 𝒎 = 𝟎

are concurrent. Then value of m is

Solution:
If line are concurrent then value of determinants must be equal to zero

i.e

3 −4
|8 −11 13
33| = 0
2 −3 𝑚

3(−11
−3
33
𝑚
)+ 4(82 33
𝑚
) + 13(82 −11
−3
) =0

3(-11𝑚 + 99)+4(8𝑚 − 66)+13(−24+22)

-33m +297 +32m -264-26=0

m = -7

Question.37 A firm produces 50 unit of product for Rs 320 and 80 units for Rs380
Considering the cost curve to be straight line. The cost of producing 110 units be estimated as

Solution:
Required points are

(50, 320) & (80, 380)

Equation of line passing through two points is

𝑦 − 𝑦1 = 𝑚(𝑥 − 𝑥1)
380−320
𝑦 − 320 = (𝑥 − 50)
80−50

Put 𝑥 = 110
60
𝑦 − 320 = 30 (110 − 50)

𝑦 − 320 = 2(60)

𝑦 − 320 = 120

𝑦 = 440

38 | P a g e
Question.38 The distance from origin to the point of intersection of two straight lines
having equation 3𝒙 − 𝟐𝒚 = 𝟔 𝒂𝒏𝒅 𝟑𝒙 + 𝟐𝒚 = 𝟏𝟖

Solution:
point of intersection

3𝑥 − 2𝑦 = 6

3𝑥 + 2𝑦 = 18

-4𝑦 = −12

𝑦=3

Put in

3𝑥 − 2𝑦 = 6

3𝑥 − 2(3) = 6

𝑥 = 12

Now Distance between (4, 3) and (0, 0) is

d=√(0 − 3)2 + (0 − 4)2

d= √9 + 16

d= √25

d= 5

Question 39 The point of intersection between the straight line 𝟑𝒙 + 𝟐𝒀 = 𝟔 𝒂𝒏𝒅

3𝑥 − 𝑦 = 12 𝑙𝑖𝑒 𝑖𝑛

Solution:
Point of intersection

3𝑥 + 2𝑦 = 6

3𝑥 − 𝑦 = 12

3𝑦 = −6

𝑦 = −2

39 | P a g e
Put in

3𝑥 + 2𝑦 = 6

3𝑥 + 2(−2) = 6

3𝑥 − 4 = 6

3𝑥 = 6 + 4
10
𝑥= 3

Required point is

(10⁄3 , −2) = (𝑥 + 𝑣𝑒, 𝑦 − 𝑣𝑒)

Hence required point’s lies in fourth quad rout.

Question.40 The point whose abscissa is -5 and lies on 𝒙 − 𝒂𝒙𝒊𝒔 is

Solution:
Required Point is

(-5, 0)

Question.41 If an investment of Rs 1000 and Rs 100 yield on income of Rs 90 and Rs 20. The
equation of investment.

Solution:
Required points for equation of investment is

(90, 1000) & (20, 100)

Required equation is

𝑦 − 𝑦1 = 𝑚(𝑥 − 𝑥1)

100 − 1000
𝑦 − 1000 = (𝑥 − 90)
20 − 90
−900
𝑦 − 100 = (𝑥 − 90)
−70

90
𝑦 − 100 = (𝑥 − 90)
7

40 | P a g e
7𝑦 − 7000 = 90𝑥 − 8100

7𝑦 − 90𝑥 + 1100 = 0

Question.42 A manger has observed that when expenditure on advertisement is 8000


rupees sales comes to 18 lac rupees and when expenditure on advertisement is 15 thousand
rupees. The sales come to 20 lac rupees. What will be the estimate sales if expenditure on
advertisement is 10000 rupees?

Solution:
Required points are

(8000, 1800,000) and (15,000 280,000)

Required line is
𝑦2−𝑦1
𝑦 − 𝑦1 = (𝑥 − 𝑥1)
𝑥2−𝑥1

2,800,000−1,800,000
𝑦 − 1,800,000 = (𝑥 − 8,000)
15,000−8,000

1,000,000
𝑦 − 1,800,000 = (𝑥 − 8,000)
7,000

Put 𝑥 = 10,000
1,000
𝑦 − 1,800,000 = (10,000 − 8,000)
7

𝑦 − 1,800,000 = 142.85(2,000)

𝑦 − 1,800,000 = 285714.28

𝑦 = 2085714.28⁄1,000,000

𝑦 = 2.08 𝑚𝑖𝑙𝑙𝑖𝑜𝑛

MCQ:
1- There is a (indirect) relationship between break -even point and selling price
2- If break-even point of any company increase then it would be (bad) news for that
company
3- If break-even point of an industry increases by 10% then this might occur due to the
following reason (increase in fixed cost)
4- In break -even analysis, we assume (linear) relationship

41 | P a g e
Check yourself:
Question:1

A company sells two product A and B. Total profit earned by the company in one particular period is Rs.
150,000 by selling 4 units of A and 10 units of B. The ratio of profits between A is to B is 1.5:1. Compute
profit per unit from product B.

(a)RS.9,375 (b)Rs. 14,062.5


(c)Rs. 13,062.5 (d)10,375
Question:2

Sana distributed Rs. 1,800 amongst her three brothers in such a way that each one of them received Rs.
150 at least. What can be maximum difference between amounts received by each brother?

(a)Rs. 1,500 (b) Rs. 1,350


(c)Rs. 1,650 (d)Not possible to determine
Question:3

A manufacture has three production workers working eight hours per day and producing twelve units in
total. How many hours will this manufacture need to operate per day if one worker leaves the company
and company has to produce sixteen units in total.

(a)8 hours (b)16 hours


(c)24 hours (d)10 hours
Question:4

A company has total budget of Rs.105,000 to purchase tables and chairs. The cost of each table is twice
that of a chair and company wants to have 3 table and 15 chairs. If the company uses its entire budget,
how much more will be spent on buying chairs as compared to tables.

(a) Rs. 30,000 (b) Rs. 75,000


(c) Rs. 45,000 (d)Rs. 50,000
Question:5

y = 3x + c is the equation of a straight line. The line passes through the point (2,11). Determine value of
y- intercept

(a)5 (b)6
(c)8 (d)10
Question:6

Find the equation of a line which passes through the point (2,1) and (4,9)

(a)y = 4x-7 (b)y = 2x + 7


(c) y = 6x – 7 (d)y = 4x + 7

42 | P a g e
Question:7

The product of two positive integers is 28 and when added together the result is 11. Find the integers

(a) 2 and 14 (b) 4 and 7


(c)28 and 1 (d)Not possible to determine
Question:8

A restaurant sells fries for Rs. 120 each, burger for Rs. 200 each and sandwiches for Rs. 250 each. If the
profit on fries is 1/3rd of its selling price, profit on burgers is Rs. 60 more than the profit on fries and
profit on sandwiches is 125% of profit on burgers, what will be the total profit for the sale of 5 fries, 10
burgers and 15 sandwiches

(a)Rs. 3,075 (b)Rs. 2,075


(c)Rs. 5,075 (d)Rs. 2,175
Question:9

Determine x-intercept of a line passing through the points (1,7) and (2,10)

(a)3⁄4 (b) - 3⁄4


(c)- 4/3 (d)4/3
Question:10

A person buys 20 units of a product for Rs. 950 in total, this includes discount of 5% which is applicable
if more than 10 units are purchased. Compute cost per unit if 5 units are purchased

(a)Rs. 250 (b)Rs. 45


(c)Rs. 50 (d)Rs. 200
Question:11

A father gives his son certain amount of pocket money every week. Son saves 30% of amount and
spends the rest, with spending on Saturday and Sunday being twice than what is spent on other days of
the week. If total savings in the week are Rs. 420. How much is spent each Saturday?

(a)Rs, 217.78 (b)Rs. 317.78


(c)Rs. 117.78 (d)Rs. 17.78
Question:12

The slope of the line parallel to x-axis is:

(a)0 (b) cannot be determined


(c)+1 (d)-1
Question:13

The slope of the line which passes through origin and such that every coordinate has equal x and y
values is:

43 | P a g e
(a) o (b)1
(c)-1 (d)infinity

Question :14

Find slope of x, and intercept of x. of the following 2x-y-9=0

(a)0.5, 4.5 (b)2, 4.5


(c)0.5, 9 (d)2, 9
Question :15

Two lines having slope m1 and m2 are perpendicular if

(a)m1 = m2 (b)m1 .m 2 = 1
(c)m1 = m2 (d)m1. m2 = -1
Question :16

Which of the following is incorrect?

(a)Slope is always positive before and after (b)Slope is positive before maximum but negative
point of inflection afterwards
(c)Slope is negative before minima but positive (d)Slope is always constant before and after point
afterward of inflection
Question :17

4y=-3 + 2x find slope

(a)2 (b)3
(c)1/2 (d)3/2
Question :18

Find the Equation of the line which passes through the intersection of 5x + 2y = 44 and 3x-y=22, and
perpendicular to 9x+y-5 =0

(a)9x-y-10=0 (b)9y-x-10=0
(c)9y-x=-10 (d)x=-10+9y
Question :19

Define contribution

(a)Difference of sales and fixed cost (b)Difference of sales and variable cost
(c)Profit per unit (d)None of these

Question :20

point (-6,12) lies in which quadrant?

44 | P a g e
Question :21

x+2y+3=0, which statement is wrong about this equation

(a)This equation is representing the relationship (b)Slope of Y is -0.5


between X and Y
(c)Intercept of X is 3 (d)None of these
Question :22

Which of the following is the equation of the straight line whose slope is -3 and y intercept is 4?

(a)3x+y=4 (b)x-4y=-3
(c)-4x+y=-3 (d)4x+y=-3
Question :23

Which of the following is the equation of the straight line whose slope is 4 and y intercept is – 3?

(a) -3x+y=-4 (b) x-4y=-3


(c) -4x+y=-3 (d) 4x+y=-3
Question :24

(a)Only maxima or minima can be found (b)line of symmetry will pass through maxima or
minima
(c)It is related to quadratic equation only (d)All of these
Question :25

Which of the following have positive slope?

a) 2x+y=20 b) x-2y=20
c) 2x =-y +10 d) All of these
Question :26

Which of the following have negative slope?

a) 2x+y=20 b) x-2y=20
c) 2x= -y+10 d) Both a and c
Question :27

Which of the following have 0 slope?

a) Y=20 b) X=20
c) 2x= -y+10 d) None of these
Question :28

Which of the following have infinite slope?

a) Y=20 b) X=20
c) 2x= -y+10 d) None of these

45 | P a g e
Question :29

Which of the following has no solution?

a) x + y=20 and x + y=20 b) x + y=20 and x - y=20


c) 2x-2y=40 and x + y=20 d) x-y = 20 and x + y=20
Question :30

x+2y+3=0, which statement is correct about this equation

a) Slope of line is 1 b) Slope of line is -1


c) Degree of equation is 1 d) Intercept of x axis is 3

46 | P a g e
SOLUTION OF ICAP BOOK:

Question:1

A firm introducing a new washing detergent. The firm plans to sell the family size box for
Rs.24. Production estimates have shown that the variable cost of producing one unit of the
product is Rs.21.60. Fixed cost of production is Rs.36,000. It is assumed that both the total
revenue and total cost functions are linear over the relevant sale quantity range. Then the break-
even volume of sales is:

a) 150,000 boxes b) 1,500 boxes


c) 150 boxes d) 15,000 boxes

Solution:

Revenue = 24x

Total cost = Variable cost (x) + Fixed cost

TC = 21.60x + 36000

Break-even

R (x) = C (x)

24x = 21.60 x + 36000

2.4x = 36000

x = 15000

Question:2

A table manufacture has to pay Rs.30,000 as a fixed cost and Rs.10,000 as variable cost per table
manufactured. If the selling price of each table is Rs. 15,000 compute break-even point.

a) 6 units b) 4 units
c) 8 units d) 2 units

Solution:

Break-even point

R (x) = C (x)

15000x = 10,000x + 30,000

47 | P a g e
5000x = 30000

x = 6 units

Question:3

A company sells two products A and B. Total profit earned by the company in one particular
period is Rs. 150,000 by selling 4 units of A and 10 units of B. The ratio of profit between A is
to B is 1.5:1. Compute profit per unit from product B.

a) Rs. 9,375 b) Rs. 14,062.5


c) Rs. 13,062.5 d) Rs. 10,375

Solution:

Total Profit

4x + loy= 150,000
𝑥 1⋅5
Ratio =
𝑦 1

x = 1.5y

Put in

4x + 10y = 150,000

6y + 10y = 150,000

16y = 150,000

y= 9375

Question:4

Sana disturbed Rs. 1,800 amongst her three brothers in such a way that each one of them
received Rs. 150 at least. What can be maximum difference between amount received by each
brother?

a) Rs. 1,500 b) Rs. 1,350


c) Rs. 1,650 d) Not possible to determine

48 | P a g e
Solution:

Case-1 Case-2

Let: Let:

A = 150 A = 200

B = 150 B = 200

C = 1500 C = 1400

So maximum difference So maximum difference

= 1500 – 150 =1400 – 200

= 1350 = 1200

Case 1 is true

Question:5

A manufacture has three production workers working eight hours per day and producing twelve
units in total. How many hours will this manufacture need to operate per day if one worker
leaves the company and company has to produce sixteen units in total.

a) 8 hours b) 16 hours
c) 24 hours d) 10 hours

Solution:

Total working hour =3 x 8 = 24 hours

Number of units = 12
24
Time for one unit = 12 = 2 hours

Total number of hours = 16 x 2 = 32

Each person work = 16 hours

49 | P a g e
Question:6

A company has total budget of Rs. 105,000 to purchase tables and chairs. The cost of each table
is twice that of a chair and company wants to have 3 tables and 15 chairs. If the company uses its
entire budget. How much more will be spent on buying chairs as compared to tables

a) Rs. 30,000 b) Rs. 75,000


c) Rs. 45,000 d) Rs. 50,000

Solution:

Total budget = 105,000

Let:

Cost of chair = x

Cost of table = 2x

Also:

Unit x Price + unit x price = Total cost

3(2x) + 15(x) = 105,000

21x = 105,000

x= 5000

Now

Cost of chair = 5000 x 15 = 75000

Cost of table = 10000 x 3 = 30000

Difference in cost = 75000 – 30000 = 45000

Question:7

A restaurant sells fries for Rs. 120 each, burgers for Rs. 200 each and sandwiches for Rs. 250
each. If the profit on fries is 1/3rd of its selling price, Profit on burgers is Rs. 60 more than the
profit on fries and profit on sandwiches is 125% OF profit on burgers. What will be the total
profit for the sale of 5 fries, 10 burgers and 15 sandwiches?

a) Rs. 3,075 b) Rs. 2,075


c) Rs. 5,075 d) Rs. 2,175

50 | P a g e
Solution:

Sale price = Fries + burgers + Sandwiches

Sale price = 120 + 200 + 250

Also:

Profit = Fries + Burgers + Sandwiches

= 40 x 5 + 100 x 10 + 125 x 15

= 200 + 1000 + 1875

Hence profit = 3075

Question:8

A person buys 20 units of a product for Rs. 950 in total, this includes discount of 5% which is
applicable if more than 10 units are purchased. Compute cost per unit if 5 units are purchased

a) Rs. 250 b) Rs. 45


c) Rs. 50 d) Rs. 200

Solution:

Price of per unit if less than 10 units are purchased = x

Price per unit if more than 10 units are purchased = 0.95x

Also:

Cost of 20 units = 0.95x x 20

Given 0.95x x 20 = 950

x = 50

If 5 units are purchased then cost = 50

Question:9

A person buys 20 units of product for Rs. 950 in total, this includes discount of 5% which is
applicable if more than 10 units are purchased. Compute cost per units are purchased

51 | P a g e
Solution:

Let

Price per unit if less than 10 units are purchased =x

Price per unit if more than 10 units are purchased =0.95x

Also

Cost of 20 units = 0.95x * 20 =

Given

0.95x *20 =950

X=50

If 5 units are purchased then cost =50

Question:10

A father gives his son certain amount of pocket money every week. Son saves 30% of amount
and spends the rest, with spending on Saturday and Sunday being twice than what is spent on
other days of the week. If total savings in the week are Rs. 420. How much is spent each
Saturday?

a) Rs. 217.78 b) Rs. 317.78


c) Rs. 117.78 d) Rs. 17.78

Solution:

Let:

Total pocket money = x

Saving of pocket money = 0.3 x

Spending of pocket money = 0.7 x

Saving = Saving

0.3x = 420

x = 1400

52 | P a g e
Now:

Saving of pocket money = 420

Spending of pocket money = 980

Also:

Whole week spending = 980

y+y+y+y+y+2y+2y = 980

Remaining days = y = 108.88

Saturday day = 2y = 217.76

Question:11

The slope of the line which passes through origin and such that every coordinate has equal x and
y value is:

a) 0 b) 1
c) -1 d) Infinity
Solution:

Since (1,1) (0,0) in which x and y has same value

If x and y co-ordinate is same then value of m = 1


0−1
Note: m = 0−1 = 1

53 | P a g e
EXTRA PRACTICE QUESTION

Percentage/profit and loss


Percent: Cent means 100. Per is dividing or out of. Percent - how many out of 100.

Percent to Decimal or Fraction


40% as a decimal 35% as a fraction is
number is To change a percent to either a decimal 35 1 7
=  =
40 100 = 0.40 = 0.4 or fraction, divide by 100. 1 100 20

Formula:

part percentage
i) =
whole 100

(ii) Profit = Selling price - Cost price


profit/loss
(iii) Profit/loss % = x 100
cost price
(iv) S.P = C.P + Percentage (C.P)
K-18

Question.1 165 meters as a percentage of 3 kilometers (3000 meter) is:

A. B. C. D.

Solution
Given
using
part percentage
=
whole 100
165 percentage
=
3000 100
165 x 100
= percentage
3000
5.5% = percentage

Question.2 If 8.4% of a book consists of 42 pages. Find total number of pages in the book.

A. B. C. D.

Solution
To find the whole from a part using the formula
Part Percent
=
whole 100

54 | P a g e
42 8.4
=
whole 100

42x100
= whole = total pages
8.4
Total pages = 500

Question.3 The number of employees in 2012 was 5% higher than in 2011. The number
in 2013 was 5% higher than in 2012. The total percentage increase in the
number of employees from 2011 to 2013 is:

A. B. C. D.

Solution
Number of employees in 2011 =x
Number of employees in 2012 = x + 5%x
= x + 0.05x
= (1 + 0.05) x
= 1.05x
Number of employees in 2013 = 1.05x + 0.05(1.05x)
= 1.05x (1 + 0.05)
= 1.05x (1.05)
= 1.1025x
Percentage increase = 1.1025x x 100%
= 110.25x%
Increase = 110.25 - 100 = 10.25%

Question.4 If 45% of students in a school are girls. What percentage are boy?

A. B. C. D.

Solution
Given
Number of Girls = 45%
Number of boys = (100 - 45) %
= 55%

Question.5 If 82% of the houses have a television what percentage do not have?

A. B. C. D.

Solution
Percentage of homes having television = 82%

55 | P a g e
Percentage of homes do not have television = (100 - 82) %
= 18%

Question.6 A hockey team won 62% of their matches and 26% of them were ended in
draw. What percentage of the matches they lost?

A. B. C. D.

Solution
Given
Team won matches = 62%
Team draw matches = 26%
Team loss matches = (100 - 62 - 26) %
= 12%

Question.7 An aero plane carries 400 passengers, 52% of the passengers were Pakistani,
17% were Chinese, 12% were from Iran and the rest were from British?
i) How many people of each nationality were on the plane?
ii) What percentage were British?

A. B. C. D.

Solution
Given
Total number of passengers = 400
Percentage of Pakistani passenger = 52% x 400
= 208
Percentage of Chinese passenger = 17% x 400
= 68
Percentage of Iranian passenger = 12% x 400
= 48
Number of British passengers = 400 - 208 - 68 - 48
= 76
ii) Percentage of British passenger = (100 - 52 - 12 -17)%
= 19%
Or
76
= 400 x 100%
= 19%

Question.8 Amna scored 46 out of 50 in a mathematics test, 64 out of 75 in chemistry test


and 72 out of 80 in a physics test. In which subject did she perform best?

56 | P a g e
A. B. C. D.

Solution
Given
46
Percentage score in math test = 50 x 100%
= 92%
64
Percentage score in chemistry test = 75 x 100%
= 85.3%
72
Percentage score in Physics test = 80 x 100%
= 90%
Amna obtain the best percentage in mathematics

Question.9 A table costs a carpenter Rs. 720 to make. He sells it for Rs. 920. What
percentage of profit did he earn?

A. B. C. D.

Solution
Given
C.P = Cost price = 720
S.P = Sale price = 920
Profit = S.P - C.P
= 920 - 720
= 200
Profit
Profit percentage = x 100%
C.P
200
= 720 x 100%
= 27.78%

Question.10 The cost of wood used by a furniture manufacturer for making a table
amounts to Rs. 7000. Other cost incurred by sum amount to 30% of the cost
of wood. What price shall be charge if he wishes to earn a profit of 12.5% of
the selling price?

A. B. C. D.

Solution
Given let the required selling price = x
Also
Cost of wood = 7,000

57 | P a g e
Other cost = 30% of cost of wood
= 30% x 7,000
30
= 100 x 7,000
= 2100
Total cost incurred = 7000 + 2100 = 9100

Now
Total cost = Selling price – 12.5% of S.P
9100 = x – 0.125x
9100 = 0.875x
x = 10,400

2007
Question.11 A brief case selling for Rs. 1500 was marked down by 20% for a special
promotion. It was later marked down further by 10% of the promotional
sales price. Compute the profit/loss in percentage if the cost is Rs. 1000.

A. B. C. D.

Solution
Given
Cost price = Rs. 1000
Sale price = Rs. 1500
Promotional decrease in
Sale price first time = 1500 (1 - 20%)
Promotional decrease in
Sale price 2nd time = 1500 (1 - 20%) (1-10%)
= 1500 (0.8) (0.9)
= 1080
Profit = S.P - C.P
= 1080 - 1000
= Rs. 80
profit
Profit Percentage = x 100
C.P
80
= x 100
1000
= 8%
2006
Question.12 The selling price of an item is Rs. 300 and the gross profit is 15% of cost.
Compute the cost.

A. B. C. D.

Solution

58 | P a g e
Let
The required cost = x
Selling price = 300
We know
Cost + profit = selling price
x + 15% of x = 300
x + 0.15x = 300
1.15 x = 300
x = Rs. 260.86

2007
Question.13 Suppose there were 150 Traffic accidents last year. What is the number of
Traffic accidents if the number goes down 20%? What happens if in the
following year, the number of accidents goes up by 20%?

A. B. C. D.

Solution
Total number of accidents = 150
Total decrease in traffic = 20%
Accident in this year
Number of accidents = 150 (1 - 20%)
= 150 (1 - 0.2)
= 150 (0.8)
= 120
Number of accidents
Increase in following = 20%
The expected number of
Accident in following year = 120 (1 + 20%)
= 120(1.20)
= 144

Question.14 A railway train carries 800 passengers, 55% passenger are men, 15% children.
What is the percentage of women?

A. B. C. D.

Solution
Percentage of men = 55%
Percentage of children = 15%
Total percentage of men and children = (55 + 15)%
= 70%

59 | P a g e
Percentage of women = (100 -70)% = 30%

Question.15
i) 180 is 20% of what number?

A. B. C. D.

Solution
Let the required number
20%x = 180
20
x = 180
100
0.20x = 180
180
x = 0.20
x = 900

ii) What is 25% of 640?

A. B. C. D.

Solution
Given
= 25% x 640
= 0.25 x 640
= 160

iii) Rs. 260 is what percentage of Rs. 10,000?

A. B. C. D.

Solution
Let the required number = x%
𝑥 260
= 10,000
100
260
x = 100
x =2.6%

Question. 16 Subtracting 40% of a number from the number. We get result as 30. The
number is:

Solution:

60 | P a g e
𝑥 − 40% = 30
𝑥 − 0.4%𝑥 = 30
0.6𝑥 = 30
𝑥 = 50
Question. 17 If the selling price of an article is 4/3 times its cost price, the profit percent is:
Solution:
𝐶. 𝑃 = 𝑥
𝑆. 𝑃 = 4/3 𝑥
𝑃𝑟𝑖𝑓𝑡 = 𝑆. 𝑃 – 𝐶. 𝑃
𝑃𝑟𝑜𝑓𝑖𝑡 = 4/3 𝑥 – 𝑥 = 𝑥/3
𝑥
1
𝑃𝑟𝑜𝑓𝑖𝑡% = 3 × 100 = × 100 = 33.35
𝑥 3

Question. 18 A school has only three classes which contain 40, 50 and 60 students
respectively. The pass percentage of these classes is 10, 20 and 10 respectively. The pass
percentage of the school is:
Solution:
10 2 10
Total Number of passed candidate = × 40 + 100 × 50 + 100 × 60
100

= 4 + 10 + 6 = 20
Total students in school = 40 + 50 + 60 = 150
𝑜𝑏𝑡𝑎𝑖𝑛 20 40 1
So, required percentage = 𝑇𝑜𝑡𝑎𝑙
𝑋 100 = 150 × 100 = 3
= 13 3 %

Question: 19 Two students were appeared at an examination. One of them secured 9 marks
more than the others and his marks were 56% of the sum of their marks. The marks
obtained by them are

Solution:
Marks of Ist = x
Marks of 2nd = x + 9
Sum of Marks = x + x +9
Now
x+9 = 56%(2x+9)
x = 33

61 | P a g e
Marks of Ist = x =33
Marks of 2nd = x + 9 =33+9 =42

Question: 20 If sale price of a shirt after giving 8% discount is Rs.161. then written price is.

Solution:
Let required written price = x
Then
x = 161 + 8%x
x = 161 +0.08x
0.92x = 161
x = 175

Question: 21 A and B together have Rs.1210. if 50% of A’s amount is equal to 75% of B’s
amount. How much amount does B have?

Solution:
Total amount = 1210
Then 50% A = 75% of B
𝐴 3
=4B
2
𝐴 3
=2
𝐵
A: B = 3: 2
2
B’s share = 5 * 1210
= 484

Question: 22 In an examination, 80% of candidate passed in English, 85% in math, 75%


in both. If 40 fail in both subject. Find total number of candidates.
Solution:
Fail in English =100 -80 =20%
Fail in math = 100-85=15%
Fail in both = 100-75 =25%
Total fail = 40
(20+15-25)% x =40
10% x =40
x = 4000/10
x = 400

Question: 23 Out of 1000 persons, 25% will industrial workers and the rest were
agricultural workers. 300 persons employed world cup match on TV. 30% of the people

62 | P a g e
who had not watched world cup matches were industrial workers. What is the number of
agricultural workers who had enjoyed world cup matches on T.V?
Solution:
Total workers = 1000
Industrial worker = 250
Non-industrial worker = 750
Also
Persons watch match = 300
Persons do not watch match = 700
So 700*30% = 210
Industrial worker did not watch = 210
Industrial persons who watched = 250 – 210 = 40
Non-industrial persons who watched = 300 – 40 = 260

Question: 24 A sample study of the people of an Area revealed that total numbers of
women were 40% and the percentage of coffee drinks were 45 as a whole and the
percentage of male coffee drinkers was 20%. What was the percentage of female non-coffee
drinkers?
Solution:
Total population = 100
No. of women = 40%
No. of men = 60%
Total population drink coffee = 45%
No. of male person who drink coffee = 20%
So No. of female drink coffee = (45 -20) % = 25%
No. of female who do not drink coffee = (40% -25%) = 15%

Question: 25 Total number of students in a school is 1200. If 65% students are belongs to
rural areas, then the number of urban areas students is.

Solution:
Total students = 1200
Rural Areas = 65%
Rural Areas students = 1200 x 65%
= 780
Urban Area = 1200-780 = 420

63 | P a g e
Question 26 How many pieces of 0.85 meters can be cut from a rod 42.5 meters long?

Solution
42.50
= 0.85
4250
= = 50
85

Question 27 A student has to secures 40% marks to pass. He gets 178 marks and fails by
22 marks. The maximum marks are

Solution
Let the total marks = x
40% x-22 = 178
0.4x = 200
x = 500

Question 28 A number on subtracting 15 from it, reduces to its 80%. What is 40% of that
numbers.

Solution
80
x – 15 = 100 x
4𝑥
x – 15 =
5
4𝑥
x- = 15
5
x = 75
Required answer = 75 * 40% = 30

𝟕
Question 29 A, B and C contract a work for Rs 1100, A and B together is to do 𝟏𝟏 of the
work. C’& share is
Solution
Let total work to be done = 100%=1
A+B+C=1
7
+C=1
11
4
C = 11
C’s share
4
C= 11 * 1100
C= 400

64 | P a g e
Question 30 Buy how much is two third of 57 more than one third of 90?

Solution
2 1
= (57) - 3 (90)
3
= 38 – 30
=8

Question: 31 35% of what number is 70.


Solution:
𝑃𝑎𝑟𝑡 𝑃𝑒𝑟𝑐𝑒𝑛𝑡𝑎𝑔𝑒
=
𝑊ℎ𝑜𝑙𝑒 100
70 35
= 100
𝑋
X = 200

Question: 32 what number is 5% of 2000.


Solution:
𝑃𝑎𝑟𝑡 𝑃𝑒𝑟𝑐𝑒𝑛𝑡𝑎𝑔𝑒
=
𝑊ℎ𝑜𝑙𝑒 100
𝑥 5
= 100
2000
x = 100

Question: 33 if a discount of 25% off the retail price of a desk save Rs.45. How much did
he pay for the desk?

Solution:
𝑃𝑎𝑟𝑡 𝑃𝑒𝑟𝑐𝑒𝑛𝑡𝑎𝑔𝑒
=
𝑊ℎ𝑜𝑙𝑒 100
45 25
= 100
𝑥
x =180
Actual pay
= 180 – 45
= 135

Question: 34 A customers pay Rs.1100 in state taxes on a newly purchased car. What is the
value of the car if state taxes are 8.9% of the value?

Solution:
𝑃𝑎𝑟𝑡 𝑃𝑒𝑟𝑐𝑒𝑛𝑡𝑎𝑔𝑒
=
𝑊ℎ𝑜𝑙𝑒 100
1100 8.9
= 100
𝑥
x = 12359.55

65 | P a g e
Question: 35 Mr. Ahsan boss state that he will increase his salary from Rs.12, 000 to RS.14,
000 per year. If he enrolls in business course at a local community college. What percent
increase in salary will result from Ahsan taking the business course?

Solution:
Actual = 12,000
Increase = 14,000
Difference = 2000
2000
% increase = 12000 ∗ 100
% increase = 16.7%

Question: 36 A computers are on sale for Rs.1600, which is a 20% discount off the
regular price. What is the regular price?
Solution:
let the required regular price = x
then
x-20%x =1600
x(1-0.2) =1600
x(0.8) =1600
x=1600/0.8
x =1600/0.8
x=2000
Question 37 There would be 10% loss of a toy if it is sold at RS 10.80 per piece. At what
price should it be sold to earn a 20% profit
Solution:
Method:1
According to question 10.80 is the 90% of the original price
Let the original price is x
Then 90%x = 10.80
90= 10.80 * 100/x
x = 12
We need 120% of x So
x= 12+ 1.2(12)
x =14.40
Method:2
x -10%x =10.80
x -10%x =10.80

66 | P a g e
0.9x =10.80
Also
x+20%x =S.P
12+20% (12) =S.P
12+2.4 =S.P
14.4 =S.P

67 | P a g e
SALES AND PURCHASE PRICE
Formula:

(i) S.P = C.P + profit% (cost price)


(ii) C.P = S.P – profit% (sale price)

Question.38 Find the S.P, when


i) C.P = Rs. 950 , profit = 10%
ii) C.P = Rs. 1540, loss = 5%

A. B. C. D.
Solution
(i)Given
C.P = Rs. 950 , Profit = 10%
Then
S.P =950+950x.10=1045

(ii) C.P = Rs.1540 Loss = 5%

S.P =1540-1540x.5%=1463

Question.39 Find the C.P, when


i) S.P = Rs. 672, Profit = 5%
ii) S.P = Rs. 851, loss = 8%
1
iii) S.P = Rs. 100, Profit = 332 %

A. B. C. D.
Solution
(i) C.P = 672- (672* 0.05)= 638.5
(ii) C.P = 851 + 851*.08=919.08
(iii) C.P =100 -100*0.335= 66.5

Question.40 A man spend 94% of his income and save Rs. 460. What is his income?

A. B. C. D.

Solution
Let the required income = x
Than according to statement
x = 94%x + 460
x = 0.94x + 460

68 | P a g e
x - 0.94x = 460
x(1 - 0.94) = 460
x(0.06) = 460
x = 7666.67

Question.41 A firm increased his assets by 20% as compared to its previous year assets. If
the value of firm’s current assets is Rs. 432,525. Find the value of his previous
year assets.

A. B. C. D.

Solution
Let the value of previous years assets = x
Then according to statement
x + 0.20x = 432525
x(1+0.20) = 432525
x(1.20) = 432525
x = 360437.50

Question.42 Haris purchased a car for Rs. 248,000 and spend Rs. 12,000 on its denting
and painting. He sold that at a profit of 5%. What did the customer pay to
Haris?

A. B. C. D.

Solution
Given
Purchase price = 248,000
Amount spend on denting and painting = 12000
Total cost of car = 248,000 + 12000
= Rs. 260,000
Profit = 5%
S.P = ?
Using
S.P = C.P + profit % of C.P
= 260,000 + 0.05 * 260,000
= 260,000 + 13000
S.P = Rs. 273,000

Question.43 A shop-keeper gains a profit of 7% by selling a dinner set for Rs. 3,852. If he
sells it for Rs. 4,050. Find his profit percentage.

69 | P a g e
A. B. C. D.

Solution
Given
Profit = 7%
S.P = 3852
C.P = ?
Let
C.P = x
S.P = x + 7% x
3852=1.07x
C.P = Rs. 3600
Also
S.P = 4050
C.P = 3600
Profit = 4050 - 3600 = Rs. 450
Profit
Profit% = x 100%
C.P
450 25 1
= 3600 x 100% = % = 12 2 %
2

Question.44 The selling price of 12 articles is equal to the cost price of 15 Article. Find
profit percentage.

A. B. C. D.

Solution
Given
C.P of 15 article = 100
S.P of 12 article = 100
Now we find selling price of 15 article
S.P of 12 article = Rs. 100
100
S.P of 1 article =
12
100
S.P of 15 article = x 15
12
= Rs. 125
Profit = S.P - C.P
= 125 - 100
= 25
Now

70 | P a g e
Profit
Profit% = x 100
C.P
25
= 100 x 100
= 25%

Question.45 Find cost price, if a fan is sold for Rs. 1,470, to get a profit 1/6th of its cost
price.

A. B. C. D.

Solution
Let
C.P = x
S.P = C.P + Profit
1
1470 = x + 6x
1470 = 7x/6
8820 = 7x
C.P=x = Rs. 1,260

𝟏
Question.46 A man sold an almirah at a profit of 7 𝟐%, had he sold it for Rs. 209, he would
have lost 2%. For how much the man purchased it?

A. B. C. D.

Solution
S.P = 209
Loss = 2%
Let
C.P = 100
S.P = 100 - 2
= 98
If S.P is 98 then C.P = 100
100
If S.P is 1 then C.P = 98
100
If S.P is 209 then C.P = x 209
98
= Rs. 213.30

Question.47 Find the selling price, when


i) M.P = Rs. 728, Discount = 6%
ii) M.P = Rs. 2,760, Discount = 5%

A. B. C. D.

71 | P a g e
Solution
(i)
S.P = Marked price - Marked price x Discount
= 728 - 728 x 6%
S.P = 728 - 43.68
S.P = Rs. 684.32
(ii)
S.P = Marked price - Marked price x Discount
= 2760 - 2760 x 5%
= 2760 – 138
=2622

Question:48 After having to pay increased income taxes this year. Ali has to sell his BMW
Ali bought the car for Rs.490, 000, but he sold it for a 20% loss. What did Ali sell the car
for?

Solution:
S.P = C.P - loss % of C.P
S.P = 4, 90,000 - 490,000 * 20%
S.P= 3, 92,000
𝟏
Question 49. How many 𝟏/𝟖 𝒔 are there in 𝟑𝟕 𝟐 ?

Solution. = (37 1/2)( 8/1)


= (75/2(8/1)
= 300

𝐐𝐮𝐞𝐬𝐭𝐢𝐨𝐧: 𝟓𝟎 A man gains 10% by selling a certain article for a certain price. If he sells at
double the price. The profit made is
Solution:
Let the C.P = x
S.P by 10 % increase = 110% x = 1.1x
If S.P is double then
S.P = 2.2x
Profit = 2.2x – x = 1.2x
𝑝𝑟𝑜𝑓𝑖𝑡
Profit% = * 100%
𝑐𝑜𝑠𝑡
1.2𝑥
= * 100%
𝑥
= 120%

Question: 51 A single discount equivalent to a discount series of 30%, 20% and 10% is:

72 | P a g e
Solution
A single discount equivalent of 3 % and 20% is.
30∗20
= 30 + 20 - 100
= 50 – 6
= 44%
Also a single discount of 44% and 10% is
44∗10
= 44 + 10 - 100
= 54 – 4.4
= 49. 6%

Question 52 A fruit seller purchases orange at the rate of 3 of Rs 5 and sells them at 2 for
Rs 4. His profit in the transaction is:

Solution:
Cost of 3 oranges = Rs 5
5
Cost of 1 orange = Rs 3
Also
Sale of 2 oranges = RS 4
4
Sale of 1 orange = 2 = 2
Profit = S.P – C.P
5
=2-3
1
=3
𝑝
Profit % = * 100
𝑐.𝑝
1
3
= 5 * 100 = 20%
3

Question 53 5 out of 2250 parts of earth is Sulphur. What is the percentage of surplus in
earth?

Solution
5
Required percentage = (2250 * 100) %

2
= 9%
Question 54. A retailer buys a radio for Rs. 225 his overhead expenses are Rs. 15. He sells
the radio for Rs. 300. The profit percent of the retailer is:
Solution.

73 | P a g e
𝐶. 𝑃 = 225 + 15 = 240
𝑆. 𝑃 = 300
𝑃𝑟𝑜𝑓𝑖𝑡% = 300 − 240 = 60
𝑃𝑟𝑜𝑓𝑖𝑡 % = 60/240 × 100% = 25%

Question 55. By selling a pen for Rs.15 a man loses one-sixteenth of what is costs him. The
cost price of the pen is
Solution:
𝐿𝑒𝑡 𝐶. 𝑃 𝑜𝑓 𝑝𝑒𝑛 = 𝑥
𝑥 − 1/16 𝑥 = 15
(16𝑥 − 𝑥)/16 = 15
𝑥 = 16
Question 56. A producer of tea blends two varieties of tea from two tea gardens. One
costing Rs 18 per kg and another Rs 20 per kg in the ratio 5∶ 𝟑. If he sells the blended
variety at Rs. 21 per kg. then is gain percent is:
Solution.
𝐶𝑜𝑠𝑡 𝑝𝑟𝑖𝑐𝑒 = 5 × 18 + 3 × 20 = 𝑅𝑠. 150
𝑆𝑒𝑙𝑙𝑖𝑛𝑔 𝑃𝑟𝑖𝑐𝑒 = 8 × 21 = 𝑅𝑠. 168
𝑃𝑟𝑜𝑓𝑖𝑡 = 168 − 150 = 18
𝑃𝑟𝑜𝑓𝑖𝑡 % = 18/150 × 100 = 12%

Question 57. A book was sold for Rs. 27.50 with a profit of 10% if it was sold for Rs. 27.75
then what would have been percentage of profit and loss?
Solution:
x+ profit =27.50
x+10%x =27.50
x+0.1x =27.50
1.1x =27.50
x=25
Profit =s.p – c.p
=27.75 -25

74 | P a g e
=2.75
Profit =(2.75/25) x100
=11%
Question 58. A fruit seller sells mangoes at the rate of Rs. 9 per kg and thereby losses 20%.
At what price per kg, he should have sold them to make a profit of 5%?
Solution:
Method :1
Let the required selling price be x ,then
C.P -loss =S.P
x -20%x =9
x = 11.25
new selling price
S.P=x+ 5%x
S.P =11.25 +0.05(11.25)
S.P =11.81
Method :2
Gain:Sp = Gain:Sp
80 : 9 = 105 : x
105
𝑥 = 9×
80
𝑥 = 𝑅𝑠. 11.81
Question 59. A shopkeeper sold an article for Rs. 2564.36. approximately, what was his
profit percent if the cost of price of the article was Rs. 2400?
Solution.
Profit = S.P –C.P
=2564.36-2400
= 164.36
164.36
𝐺𝑎𝑖𝑛% = × 100
2400

= 6.84%

75 | P a g e
Question. 60 A man sells 320 mangoes at the cost price of 400 mangoes. His gain percent is
Solution:
𝐿𝑒𝑡 𝑡ℎ𝑒 𝑠𝑒𝑙𝑙𝑖𝑛𝑔 𝑝𝑟𝑖𝑐𝑒 𝑜𝑓 𝑒𝑎𝑐ℎ 𝑚𝑎𝑛𝑔𝑜 = 1 = 100%
𝐶𝑜𝑠𝑡 𝑝𝑟𝑖𝑐𝑒 𝑜𝑓 400 𝑚𝑎𝑛𝑔𝑜𝑒𝑠 = 𝑆. 𝑃 𝑜𝑓 320 𝑚𝑎𝑛𝑔𝑜𝑒𝑠
320
𝐶. 𝑃 𝑜𝑓 1 𝑚𝑎𝑛𝑔𝑜𝑒𝑠 𝑖𝑠 = = 0.8
400
Profit = S.P – C.P
= 1 − 0.8
𝑃𝑟𝑜𝑓𝑖𝑡 = 0.2
𝑃𝑟𝑜𝑓𝑖𝑡
𝑃𝑟𝑜𝑓𝑖𝑡 % = × 100%
𝐶. 𝑃

0.2
= × 100%
0.8
= 25%

76 | P a g e
Ratio and Proportion
Ratio

a
Definition: A ratio is an ordered pair of numbers, denoted as either a : b or and read as the
b
ratio of a to b.

Question. 61. The ratio of boys to girls in a class is 3:4. The class has 35 students.
How many boys and girls are in the class?

Solution: The ratio 3:4 is equivalent to the ratio 3n : 4n where 3n represents the number of
boys and 4n represents the number of girls.

3n + 4n = 35

7n = 35

n=5

The number of boys is 3n = 3 ∙ 5 = 15.

The number of girls is 4n = 4 ∙ 5 = 20.

Question: 62 In a mixture of 60 litters, the ratio of milk and water is 2: 1. If this ratio be 1:2
then quantity of water further added is.

Solution:
2
Quantity of milk = 60*3 = 40
Quantity of water = 60 -40 = 20
If the new ratio is 1:2 then how much water is added more is
40 1
=2
20+𝑥
20+x = 80
x = 60

Question. 63 Four milk men rented a pasture. A grated 24 cows for 3 months, B 10 cows for
5 months, C 35 cows for 4 months and D 21 cows for 3 months. If A’s share of rent is 720,
total rent of the field is:
Solution:
𝐴∶ 𝐵 ∶ 𝐶 ∶ 𝐷
24 𝑥 3 ∶ 10𝑥 5 ∶ 35𝑥4 ∶ 21𝑥 3

77 | P a g e
72: 50: 140 ∶ 63
𝑇𝑜𝑡𝑎𝑙 𝑟𝑎𝑡𝑖𝑜′ 𝑠 = 72 + 50 + 140 + 63 = 325
𝑙𝑒𝑡 𝐴′ 𝑠𝑠ℎ𝑎𝑟𝑒 𝑏𝑒 ′𝑥 ′ , 𝑡ℎ𝑒𝑛
72 ∗ 𝑥
= 720
325
72𝑥 = 720 × 325
𝑥 = 3250

Question 64 The ratio of three numbers is 3: 4: 7 and their product is 18144. The numbers
are

Solution:
Let the numbers are 3x, 4x, 7x
Then
(3x) (4x) (7x) = 18144
84𝑥 3 = 18144
𝑥 3 = 216
x=6
Required numbers are
18:24:42

Question 65 The ratio of three numbers is 3: 4: 5: and the sum of their squares is 1250. The
sum of the numbers is

Solution
Let the required numbers be
3x, 4x, 5x
Also
9𝑥 2 + 16𝑥 2 + 25𝑥 2 = 1250
50𝑥 2 = 1250
𝑥 2 = 25
x=5
Sum of numbers = 3x + 4x + 5x = 12x = 12(5) = 60

Question 66 The salaries of A, B , C are in the ratio 2:3:5. If the increments of 15% , 10%
and 20% are allowed respectively in their salaries. Then what will be new ratio of their
salaries?

78 | P a g e
Solution
Let A = 2k, B = 3K and C = 5k
115 23𝑘
A’& new salary = 100 * 2k = 10
110 33
B’& new salary = 100 * 3k = 10 k
120
C’& new salary = 100 * 5k = 6k
23𝑘 33𝑘
New ratio = : : 6k
10 10
= 23: 33: 60
Question. 67. If the ratio of A to B is 9 times the ratio of B to A, then A/B could be:
Solution:
𝐴 𝐵
= 9( )
𝐵 𝐴
𝐴 9𝐵
=
𝐵 𝐴
𝐴2 = 9𝐵 2
𝐴2
=9
𝐵2
𝐴
=3
𝐵

Question. 68 Two numbers are in the ratio 3∶ 𝟓. If each number is increased by 10, the
ratio becomes 5∶ 𝟕. The ratio between numbers is:
Solution:
Let the required numbers are 𝑎, 𝑏
𝑎 3
𝑇ℎ𝑒𝑛 =
𝑏 5
𝑎 = 3/5 𝑏 _________ (𝑖)
Also (𝑎 + 10)/(𝑏 + 10) = 5/7 _____________(ii)
7 (𝑎 + 10) = 5 (𝑏 + 10)
7 (3/5 𝑏 + 10) = 5𝑏 + 50
21/5 𝑏 + 70 = 5𝑏 + 50
21𝑏/5 − 5𝑏 = 50 − 70
(21𝑏 − 25𝑏)/5 = −20

79 | P a g e
−4𝑏 = −100
𝑏 = 25
𝑃𝑢𝑡 𝑖𝑛 (𝑖)
3
𝑎 = (25)
5
𝑎 = 15
Hence, required numbers are 15,25

Question. 69 If a box containing a dozen mirrors is dropped, which of the following cannot
be the ratio of broken mirrors to unbroken mirrors? (sum of ratio divide the number so be
part of ratio)
Solution:
(i) 2:1 (ii) 3 ∶ 1 (iii) 3:2 (iv) 7:5
Proportion

Definition: A proportion is a statement that gives the equality of two ratios, denoted as either a :
a c
b :: c : d or = .
b d
The a and d are called the extremes and the b and c are called the means.

Question.70 Find value of x in the proportion


20 : 50 :: 8 : x?

A. B. C. D.

Solution
Given
20 : 50 :: 8 : x
Product of extremes = product of means
20x = 50 x 8
20x = 400
x = 20
2001
Question.71 If 4 : a :: 5 : 8
A. B. C. D.

Solution

80 | P a g e
Given
4 : a :: 5 : 8
Product of extremes = product of means
4x8=5xa
32 = 5a
a = 32/5

Question: 72 A sum of Rs.312 was divided among 100 boys and girls in such a way that the
boys gets Rs.3.60 and each girl Rs.2.40 the number of girls is.
Solution:
3.60x + 2.40 (100- x) = 312
3.60x + 240 + 2.40x = 312
1.2x = 72
Number of boys x = 60
Number of girls = 100 – 60 = 40

81 | P a g e
DIRECT PROPORTION:
When two quantities are so related that increase in one brings increase in the second or
decrease in one brings decrease in the second. Then they are in direct proportion.
INVERSE PROPORTION:
If two quantities are so related that increase in one brings decrease in other or vice versa
than they inverse proportion.

DIRECT AND INVERSE PROPORTION

Direct proportion Indirect Proportion


Example:1 Example:1

Minutes No. of words No.of Worker Days


30 540 52 35

6 x 6 x
540 𝑋 6 52 𝑋 35
x= 30
=108 x= 28
=65
Example:2 Example:2

No. of Books Cost (Rs) Time Speed


12 156 20 12

7 x 15 x
156 𝑋 7 20 𝑋 12
x= =91 x= 15
=16
12
Example:3 Example:3

Time(min) Distance (m) No. of men No.of days


125 100 120 195

315 x 90 x
315 𝑋 100 120 𝑋 195
x= 125
=252 x= 90
=260
Example:4 Example:4

Distance(min) Fare (Men) Days


240 15 6 4

139.2 x 3 x
15 𝑋 139.2 6𝑋4
x= =252 x=3
=8
240
Example:5 Example:5

Wage No. of days Hours Speed


210 6 5 45

82 | P a g e
875 x 3 x
6 𝑋875 5 𝑋45
x= =25 x= =75
210 3

Question.73 If a pole of height 30 feet casts a shadow of 34 feet, how long a shadow would
be for a pole of height 40 feet?

A. B. C. D.

Solution

Let the required length of shadow = x


Than
Pole height Shadow length
30 34

40 x
It is a case of direct proportion
Hence
30 : 40 :: 34 : x
30 34
=
40 x
34 x 40
x= 30
x = 45.3 feet

Question.74 If the price of 50 fans is Rs. 36500 then what will be the price of 85 such fans?

A. B. C. D.

Solution
Let the required price = x
Then
Fans Price
50 36500

85 x
It is a case of direct proportion
Hence
50 : 85 :: 36500 : x

83 | P a g e
50 36500
=
85 x
36500 x 85
x= 50
x = Rs.62,050

Question.75 The price of 15 suits is Rs. 6750. How many such suits can be purchased by
an amount of Rs. 4050?

A. B. C. D.

Solution Let the required suits = x

Price Suits
6750 15

4050 x
It is a case of direct proportion
Hence
6750 15
=
4050 x
15 x 4050
x= 6750
x=9
Question.76 A motor cycle covers 90km in 2 liters of petrol. In how many liters of petrol
will it cover 225km?

A. B. C. D.

Solution
Let the required quantity of petrol = x
Then
Distance cover Quantity
90 2

225 x
It is a case of direct proportion
Hence
90 : 225 :: 2 : x
90 2
=x
225
2 x 225
x= 90
x=5

84 | P a g e
INVERSE PROPORTION

Question.77 A certain Journey by train takes 5 hours at the speed of 45 km/h. What will be
the speed of the train to complete the same Journey in 3 hours?

A. B. C. D.

Solution
Let the required speed = x
Then
Time Train speed
5 45

3 x
It is a case of inverse proportion
Hence
5 : 3 :: x : 43
5 x
=
3 45
5 x 45
=x
3
x = 75

Question.78 Six men can paint a house in four days. How long it would take to paint the
house if three men are employed?

A. B. C. D.

Solution
Let the required speed = x
Then
Mens Time
6 4

3 x
It is a case of inverse proportion
Hence
6 : 3 :: x : 4
6 x
=
3 4
2x4
=x
3
x = 8 days

85 | P a g e
Question.79 8 persons can do a job in 24 days if 4 more persons joined them, how much
time they will take to complete the same job?

A. B. C. D.

Solution
Let the required time = x

Then
Persons Days
8 24

8 + 4 = 12 x
It is a case of inverse proportion
Hence
8 : 12 :: x : 24
8 x
=
12 24
8 x 24
=x
12
16 = x

SIMPLE PROPORTION:

Question 80 If 76 is divided into four parts proportional to 7 , 5 , 3 , 4 then smallest part


is

Solution
Given ratio = 7 + 5 +3 +4
Sum of ratio = 19
3
Smallest part = 76 * 19 = 12

Question: 81 A sum of money is to be distributed among A, B, C, and D in the proportion


of 5: 2: 4: 3 if C gets Rs.1000 more than D, what is the B’s share?

Solution:
Note: Ratio means common factor each of them.
Let common factor is x.
A = 5x
B = 2x
C = 4x
D =3x

86 | P a g e
According to statement
C = D +1000
4x = 3x + 1000
x =1000
Share of B = 2(1000) = 2000

AVERAGE
Question. 82 The average of 7 consecutive numbers is 33. The largest of these numbers is:
Solution:
Let the required numbers are
(𝑥 + 𝑥 + 1 + 𝑥 + 2 + 𝑥 + 3 + 𝑥 + 4 + 𝑥 + 5 + 𝑥 + 6)/7 = 33
7𝑥 + 21 = 231
7𝑥 = 210
𝑥 = 30
The largest numbers is 𝑥 + 6 = 30 + 6 = 36
Question.83 On a sports day if 30 children were made to stand in a column, 16 columns
could be formed. If 24 children were made to stand in a column. How many columns be
formed?
Solution:

Total number of students = Column x Student =16 x 30 = 480


480
If 24 children stand in a column then = = 20 columns
24

Question. 84 A factory produces on average of one defective unit after 100 non- defective
units. The cost of producing one unit is Rs. 15, the defective units also cost 15 but sold as
scrape for Rs. 5. Determine the selling prices of non-defective unit, if manufacturer wants
to earn a profit of 25% of total sale:
Solution:
TR=100x +5(1)
Tc= 15(101) =1515
P= 100x +5-1515
P =100x -1510
0.25(TR) = Profit

87 | P a g e
25x +1.25 =100x -1510
75x =1511.25
x =20.15
Question 85. A boy was asked to multiply a certain number by 25. He multiplied it by 52
and got the answer more than the correct one by 324. The number to be multiplied Was?
Solution.
𝐿𝑒𝑡 𝑡ℎ𝑒 𝑟𝑒𝑞𝑢𝑖𝑟𝑑 𝑛𝑢𝑚𝑏𝑒𝑟 = 𝑥
𝑥 × 52 = 𝑥 × 25 + 324
27𝑥 = 324
𝑥 = 12

Question: 86 The average monthly income of P and Q is Rs.5050. the average monthly
income of Q and R is Rs.6250 and average monthly income of P and R is 5200. The
monthly income of P is.

Solution:
𝑃+𝑄
= 5050
2
P +Q = 10100 ___ (i)
𝑄+𝑅
= 6250
2
Q+R= 12500 ___ (ii)
𝑃+𝑅
= 5200
2
P + R = 10400 ___(iii)
Adding (i), (ii), (iii)
2(P+Q+R) = 33000
P +Q +R = 16500 ___ (IV)
Putting value of Q + R in (IV)
P + 12500 = 16500
P = 4000

Question: 87 The average age of husband and wife and their children 3 years ago was 27
and that of wife and child 5 years ago was 20 years. Find present age of husband is

Solution:
Sum of present age of husband, wife and child 3 years age = 27 x 3 + 3x 3 =90 years

88 | P a g e
Sum of present age of wife and child 5 years age = 20 x 2 + 5x 2 =50 years
Age of husband is 90 – 50 = 40 years

Question: 88 Mr. Ahsan spent Rs. 7.70 on 22 paisa and 40 Paisa stamps. If he bought 26
stamps in all, then find how many of them each type he bought is

Solution:
Method:1
Total spending = 15 x 0.22 +11 x0.40
Total spending = 3.3 +4.4
Total spending = 7.7
Method:2
Let the type one =x
Let the type two =y
Then
x+y =26
0.22x +0.40y = 7.70
Using calculator
x= 15
y =11

Question.89 A tanks fills completely in 2 hours if both the taps are open. If only one of the
taps is open at the given time, the smaller tap takes 3 hours more than the larger one to fill
the tank. How much time does each tap take to fill the tank completely?

Solution:
Let the volume of the tank be V
Together two taps take 2 hours to fill it completely.
Rate of both the taps together = V/2
Let the two taps be A and B.
Time taken by tap A = t hours
So, rate = V/t
Time taken by tap B = (t + 3) hours
So, rate = V/(t + 3)
Combined rate = V/t + V/(t + 3)
We already know that combined rate = V/2

89 | P a g e
⇒ V/t + V/(t + 3) = V/2 ..................(1)
Dividing this equation by V, we get.

⇒1/t + 1/(t + 3) = 1/2


Taking LCM of the denominators and then solving it.

⇒ (t + 3 + t)/t(t + 3) = 1/2
Now, cross multiplying.

⇒ 2t + 6 + 2t = t² + 3t

⇒ t² + 3t - 4t - 6 = 0

⇒ t² - t - 6 = 0

⇒ t² - 3t + 2t - 6 = 0

⇒ t(t - 3) + 2(t - 3) = 0

⇒ (t + 2) (t - 3) = 0

⇒ t = - 2 or t = 3
Since, time cannot be negative.
So, time taken by tap A is 3 hours
And, time taken by tap B = 3 + 3 = 6 hours.
Question.90 A customer pays Rs 1100 in state taxes on a newly purchased car. What is the value of
car. If state taxes are 8.9% of the values?

Solution:
1100 8.9
𝑥
= 100

x = 12359.55

Question.91 What is 35% of a number if 12 is 15% of that number?

Solution:
let the required number = x

Then 15% x =12

= 80

90 | P a g e
Also 35 %( 80) = 28

Question.92 After having to pay increased income tax this year. Ali has to sell hi BMW. Ali bought
the car for Rs 490,000 but he sold it for a 20% loss. What did Ali sell the car for?

Solution:
Selling price = 490,000 – 20%(490,000)

= 392,000

Question.93 Total number of students in a school is 1200. If 65% students belong to rural areas
then the number of urban students is

Solution:
Total students = 1200

Rural Area = 1200 × 65% = 780

Urban Area = 1200-780 = 420

Question.94 Anwar got 85% marks of the total marks and obtained 765 marks then the total marks
is

Solution:
Let the required total marks = x
765 85
=
𝑥 100

x = 900

Question.95 Two numbers are respectively 20% and 50% more than a third number. The ratio of
two numbers are

Solution:
let the third number be x

Then
120𝑥 6𝑥
First number = 120%, x = =
100 5

150 3𝑥
Second number = 150%, x = 100 = 2

Ratio of first two numbers is


6𝑥 3𝑥
:
5 2

12x : 15x

91 | P a g e
4:5

Question.96 Seats for mathematics, physics and biology in a school are in the ratio 5 : 7 : 8. There is
a proposal to increase these seats by 40% , 50% and 75% respectively. What will be the ratio of
increased seats?

Solution:
let the seats for mathematics, physics and biology in a school are 5x, 7x and 8x

Also increase in seats 40%, 50% and 75% respectively then


140 150 175
( × 5𝑥) : ( × 7𝑥) : × 8𝑥
100 100 100

2 :3 :4

Question.97 A batsman scored 110 runs which included 3 boundaries and 8 sixes. What percent of
total score did he make by running between the wickets?

Solution:
Total score = 110

Score from boundaries = 3×4 + 8×6 = 60

Remaining score = 110-60 = 50


50
Percentage of score with running = × 100%
110

= 45.45%=

5
= 45 %
11

Question.98 A Fruit seller had some apple. He sells 40% apples and still has 420 apples. Originally
he had

Solution:
Let the total number of apple = 𝑥

Then 𝑥 − 40% 𝑥 = 420

𝑥 − 0.40𝑥 = 420

0.60 𝑥 = 420

𝑥 = 70

92 | P a g e
Question.99 Electricity tax is increased by 20% but its consumption is decreased by 20%. Then
increase or decrease in the amount of bill is

Solution:
20%×20%
= 20% - 20% - 100

400%
=- 100

= -4% (Decrease)

Question.100 The ratio of boys to girl in a school is 7:6. If the number of boys and girls in the school
increases by 10% and 15% respectively in one year. The number of boys will exceed the number of
girls by 120. What is the total number of students in the school?

Solution:
Given ratio 7:6

110 15
Increase in ratio = × 7𝑥 : × 6𝑥
100 100

= 7.70 𝑥 ∶ 6.9𝑥

According to statement 7.70 𝑥 = 6.90 𝑥 + 120 = 𝑥 = 150

Total number of students = 7 × 150 + 6 × 150

= 1050 + 900

= 1950

Question.101 Chemical A & B are formed by mixing 𝒙, 𝒚, 𝒛 in the ratio of 8:3:4 and 4:3:8
respectively. The cost per liter of x, y,z is Rs 300,Rs 240 and Rs180 respectively. Chemical C is
produced by mixing Chemicals A& B in a certain ratio. If Total cost of producing 1000 liters of Chemical
C is Rs 252,000. Find the ratio of Chemicals A & B in chemical C

Solution:
8 3 4
1 liter of A = 15 𝑥 + 15
𝑦 + 15 𝑧

4 3 8
1 liter of B = 𝑥 + 𝑦 + 𝑧
15 15 15

93 | P a g e
8 3 4
Cost of 1 liter of A = 15 × 300 + 15 × 240 + 15 × 180 = 256 liter

4 3 8
Cost of 1 liter of B = 15 × 300 + 15 × 240 + 15 × 180 = 240 liter

25000
Cost of 1 liter of C = 1000
= 252

256 ×𝐴 224 ×𝐵
Cost of 1 liter of C = 𝐴+𝐵
+ 𝐴+𝐵

256 ×𝐴 224 ×𝐵
252= +
𝐴+𝐵 𝐴+𝐵

252 A + 252B = 256A = 224b

28B = 4A

B=A in ratio 7:1

The ratio of A to B in chemical is 7 to 1

Question.102 The salaries of A to B and C are in the ratio 1:2:3. The salary of B and C together is Rs
6000. By what percent is the salary of C more than that of A?

Solution:
Let the salaries of A,B and C be 𝑥, 2𝑥 𝑎𝑛𝑑 3𝑥

Then 2𝑥 + 3𝑥 = 6000

𝑥 = 1200

Now A’s Salary = 1200

B’s Salary = 2400

C’s Salary = 3600

2400
Excess of c’s Salary over A’s = × 100% = 200%
1200

Question.103 The average age of three boys is 25 and their ages are in the ratio 3:5:7 the age of the
eldest is

Solution:

94 | P a g e
Let the ratio are

3𝑥 + 5𝑥 + 7𝑥 = 15𝑥

Average age = 25

3𝑥+5𝑥+7𝑥
Then = 25
3

15 𝑥= 75

𝑥=5

Age of the eldest =7 𝑥 = 7 × 5 = 35

𝟐
Question.104 A dealer of household appliance paid Rs 200000 for 15 refrigerators. He sold 𝟓 of them
at 15000 each and remaining at 14000. What was his profit.

Solution:
Total cost = 200,000

2
Number of refrigerators to be sold = × 15 = 6
5

Price of each sold refrigerator = 15,000

Price of 6 refrigerator = 6 × 15,000 = 90,000

Remaining Refrigerator price = 9 × 14,000 = 126,000

Total sale price = 90,000+126,000 = 216,000

Cost price = 200,000

Profit = S.P – C.P

= 216,000 – 200,000

= 16,000

Question.105 In a certain store, The profit is 320%of cost if the cost increases by 25% but the selling
price remain constant. What % of selling price is profit

95 | P a g e
Solution:
Let C.P = 100

Profit = 320

S.P = 420

New C.P = 125% of 100 = 125

New S.P = 420

Profit = 420 – 125 = 295

295
Profit percentage = ( × 100)%
420

Profit percentage = 70.2%

Question.106 Rs 700 is divided among A,B and C such that A receives half as much as B and B half as
much as C, then C’s share is

Solution:
Given A+B+C = 700……………….(i)

Also

1 1 1
A=2 𝐵 𝐵 = 2𝐶 Then A=2 𝐵 2𝐵 = 𝐶

Put in (i)

1
2
𝐵 + 𝐵 + 2𝐵 + 700

B = 200

Hence C = 2(200) = 400

Question.107 An inspector rejects 0.08% of the meter as defective. How many meters he
examines to reject 2 meters?

Solution:
0.08 x= 2 x 100
x =200/0.08
96 | P a g e
x =2500

Question :108

In a certain examination, the number of students who passed, was three times the number
of those who were rejected. If there had been 16 fewer candidates and if six more
had been rejected, the number of those who passed and of those who were rejected
would have been in the ratio 2: 1. Find the number of candidates?

Solution:

Let number of rejected students = x

So, number of passed students = 3x

Total number of students = 4x

With 16 fewer candidates and 6 more rejected, 22 (16+6) less would have passed.

3x − 22 2
=1
x+6

3x - 22 = 2(x + 6)

3x - 22 = 2x + 12

x = 34

Number of candidates = 4x = 136

97 | P a g e
1/1/2022 CHAPTER-2
QUADRATIC EQUATIONS

COMPILED BY
ABDUL AHAD
Algebraic Expression

A collection of letters (called variables) and real numbers (called constants) Combined using
the operations of addition, subtraction, multiplication, division, and exponentiation is called an
algebraic expression.

FACTORIZATION

Solve the following equations by factorization.

Question.1 Solve x2 + 7x + 12 = 0

A. B. C. D.

Solution
Given
x2 + 7x + 12 = 0
x2 + 4x + 3x + 12 = 0
x(x + 4) +3(x + 4) = 0
(x + 3) (x + 4) = 0
x+3=0 or x+4=0
x-3 or x = -4
Solution set = {-3, -4}

Question.2 Solve x2 - x = 2

A. B. C. D.

Solution
Given
x2 - x = 2
x2 - x - 2 = 0
x2 - 2x + x - 2 = 0
x(x - 2) +1 (x - 2) = 0
(x + 1) (x - 2) = 0
x+1=0 or x-2=0
x = -1 or x=2
Solution set = {-1, 2}

1|Page
Question.3 Solve 3x2 + 4x + 1 = 0

A. B. C. D.

Solution
Given
3x2 + 4x + 1 = 0
3x2 +3x +x +1 =0
3x(x + 1) +1 (x + 1) = 0
(3x + 1) (x + 1) = 0
3x + 1 = 0 or x+1=0
3x = -1 or x = -1
x = -1/3
Solution set = {-1/3, -1}

Question.4 Solve 9x2 - 12x - 5 = 0

A. B. C. D.

Solution
Given
9x2 - 12x - 5 = 0
9x2 - 15x + 3x - 5 = 0
3x(3x - 5) +1(3x - 5) = 0
(3x + 1) (3x - 5) = 0
Either
3x + 1 = 0 or 3x - 5 = 0
3x = -1 or 3x = 5
x = -1/3 x = 5/3
1 5
Solution set = {− 3 , 3}

Question.5 Solve x(x + 7) = (2x - 1) (x + 4)

A. B. C. D.

Solution
Given
x(x + 7) = (2x - 1) (x + 4)
x2 + 7x = 2x2 + 8x – x - 4

2|Page
x2 + 7x = 2x2 + 7x – 4
x2 – 4 = 0
(x + 2) (x - 2) = 0
Either
x+2=0 or x-2=0
x = -2 x=2
Solution set = {-2, 2}

K-13
Question.6 Factorization of expression x2 – 4x – 21 are

A. B. C. D.

Solution
Given
x2 - 4x - 21
= x2 - 7x + 3x - 21
= x(x - 7) + 3 (x - 7)
= (x + 3) (x - 7)
Question.7 Factorize the following expression to its simplest form (completing square)
x4 - 22x2y2 + 9y4

A. B. C. D.

Solution
Given
= (x2)2 - 2(x2) (3y2) + (3y2) 2 - 16 x2y2
= (x2-3y2)2 - (4xy)2
= (x2-3y2 + 4xy) (x2 - 3y2 - 4xy)

Question.8 Factorize the expression (completing square)


16x4 + 4x2y2 + y4

A. B. C. D.

Solution
Given
16x4 + 4x2y2 + y4
= [(4x2)2 + 2 x 4x2xy2 + (y2)2] - 4x2y2
= (4x2 + y2)2 - (2xy)2
= (4x2 + y2 + 2xy) - (4x2 + y2 - 2xy)

3|Page
Question.9 Factorize the following expression to its simplest form
ax2 - ay2 + bx2 - by2

A. B. C. D.
Solution
Given
= ax2 - ay2 + bx2 - by2
= a(x2 - y2) + b(x2 - y2)
= (a + b) (x2 - y2)
= (a + b) (x + y) (x - y)

Question.10 Factorize the expression (completing square)


4x4 + 81y4

A. B. C. D.

Solution
Given
4x4 + 81y4
= (2x2)2 + (9y2)2 + 2 x 2x2 x 9y2 - 36 x2 y2
= (2x2 +9y2)2 - (6xy)2
= (2x2 + 9y2 + 6xy) (2x2 + 9y2 - 6xy)

Question.11 Factorize f(x) by completing the square


f(x) = x4 - 2x2y2 - 8y4
Find the possible value of x, where f(x) = 0 and y = 5

A. B. C. D.

Solution
Given
f(x) = x4 - 2x2y2 - 8y4
put f(x) = 0, Given
x4 - 2x2y2 - 8y4 = 0
(x2)2 - 2x2y2 = 8y4
(x2)2 - 2x2y2 + y4 = 8y4 + y4
(x2)2 - 2x2y2 + (y2)2 = 9y4
(x2 - y2)2 = (3y2)2
(x2 - y2) = + 3y2
x2 = + 3y2 + y2
x2 = 3y2 + y2 or x2 = -3y2 + y2
x2 = 4y2 or x2 = -2y2 (Neglect)

4|Page
x = + 2y
Now
Put y = 5
x = +2(5) = +10
x = +10

Question.12 Solve the following equation

(2e5x + 5ex) (2e2x - 11) = -55ex

A. B. C. D.

Solution
Given
(2e5x + 5ex) (2e2x - 11) = -55ex

Let
ex = y
e2x = y2
e5x = y5
(2y5 + 5y) (2y2 - 11) = -55y
4y7 - 22y5 + 10y3 - 55y = -55y
4y7 - 22y5 + 10y3 = 0
y3(4y4 - 22y2 + 10) = 0
y3(4y4 - 20y2 -2y2 + 10) = 0
y3[4y2(y2-5) - 2(y2 -5)] = 0
y3 [(4y2-2) (y2 - 5)] = 0
y3 = 0 or 4y2 - 2 = 0, or y2 - 5 = 0
e3x = 0 or 4e2x - 2 = 0 or e2x - 5 = 0
As e3x cannot be zero (neglect e3x)
Now
4e2x - 2 = 0 e2x - 5 = 0
e2x = 2/4 e2x = 5
e2x = 1/2 2x Ine = Ih5
2x
e = 0.5 2x(1) = 1.6094
x = 1.6094/2
2x lne = In (0.5)
2x(1) = -0.6931 x = 0.8047
x = -0.6931/2
x = -0.3465

5|Page
SIMPLIFIY
(Find value of x)

𝐱 𝐱+𝟏 𝟓
Question.13 + =𝟐
𝐱+𝟏 𝐱

A. B. C. D.

Solution
x x+1 5
+ =2
x+1 x
x2 + (x+1)2 5
=
x(x+1) 2

x2 + x2 +2x+1 5
=
x2 +x 2

2(2x + 2x + 1) = 5(x2 + x)
2

4x2 + 4x + 2 = 5x2 + 5x
x2 + x - 2 = 0
x2 + 2x - x - 2 = 0
x(x + 2) -1 (x + 2) = 0
(x - 1) (x + 2) = 0
Either
x-1=0 or x+2=0
x=1 x = -2
Solution set = {1, -2}

𝟏 𝟐 𝟕
Question.14 + 𝐱+𝟐 = 𝐱+𝟓
𝐱+𝟏

A. B. C. D.

Solution
1 2 7
+ =
x+1 x+2 x+5
x+2+2(x+1) 7
(x+1)(x+2)
= x+5
x + 2 + 2x +2 7
= x+5
x2 +3x+2
3x + 4 7
= x+5
x2 +3x+2

(3x + 4) (x + 5) = 7(x2 + 3x + 2)
3x2 + 15x + 4x + 20 = 7x2 + 21x + 14
3x2 + 19x + 20 = 7x2 + 21x + 14
4x2 + 2x - 6 = 0
6|Page
2(2x2 + x – 3) = 0
2x2 + x - 3 = 0
2x2 + 3x - 2x - 3 = 0
x (2x + 3) -1 (2x + 3) = 0
(x - 1) (2x + 3) = 0
Either
x-1=0 or 2x + 3 = 0
x=1 2x = -3
x = -3/2
Question.15 Simplify form of fraction
𝟐𝐱 𝟐 −𝟐𝐱−𝟏𝟐
is
𝟐𝐱 𝟐 −𝟏𝟖

A. B. C. D.

Solution
Given
2x2 − 2x − 12
2x2 − 18
2x2 − 6x + 4x − 12
= 2x2 −18
2x(x− 3) + 4(x − 3)
= 2(x2 −9)
(2x + 4) (x− 3)
= 2(x+3)(x−3)
2(x + 2)
= 2(x+3)
x+2
= x+3

Question.16 Simplify the following


𝐱+𝟏 (𝐱−𝟐)𝟐 𝐱 𝟐 −𝟏
x ÷ 𝐱𝟐 −𝟗
𝐱 𝟐 −𝟒 𝐱+𝟑

A. B. C. D.

Solution
Given
x+1 (x−2)2 x2 −1
x ÷ x2−9
x2 −4 x+3
x+1 (x−2)2 (x+1)(x−1)
= (x+2)(x−2) x ÷
x+3 (x+3)(x−3)
x+1 x−2 (x+3)(x−3)
= x+2 x x+3 x (x+1)(x−1)
(x−2)(x−3)
= (x+ 2)(x − 1)

7|Page
𝐱 𝟑 + 𝐚𝟑 𝐱 𝟐 +𝐚𝐱+𝐛𝐱+𝐚𝐛 𝐱 𝟐 − 𝐚𝟐
Question.17 x ÷ 𝐱𝟑 − 𝐚𝟑
𝐱𝟑+ 𝐛𝟑 𝐱 𝟒 + 𝐚𝟐 𝐱 𝟐 +𝐚𝟒

A. B. C. D.

Solution
Given
x3 + a3 x2 +ax + bx+ab x2 − a2
x ÷ x3 − a3
x3 + b3 x4 + a2 x2 +a4
(x+a)(x2 − ax + a2 ) x(x +a)+b (x+a) (x+a)(x−a)
x (x4+2a2x2+a4)−a2x2 ÷ (x−a) (x2+ax+
(x+b)(x2 − bx + b2 ) a2 )

(x+a)(x2 − ax + a2 ) (x + b) (x + a) (x−a)( (x2 +ax + a2 )


x (x2 x
(x+b)(x2 − bx+ b2 ) +a2 +ax)(x2 +a2 −ax) (x+a)(x−a)
x+a
x2 −bx+ b2

Question.18 Simplify
𝟐 𝟏
(𝟐𝟏𝟔) ⁄𝟑 𝐱 (𝟐𝟓) ⁄𝟐
√ 𝟒 −𝟑⁄
( ) 𝟐
𝟏𝟎𝟎

A. B. C. D.

Solution
Given
2 1
(216) ⁄3 x (25) ⁄1
√ 4 −3⁄
( ) 2
100

2 1
(63 ) ⁄3 x (52 ) ⁄2
=√ 1 −3⁄
( ) 2
25

62 x 51
=√ −3⁄
(5−2 ) 2

62 x 51
=√ 53

62
=√52

= 6/5

8|Page
𝟏 𝟏
Question.19 If x + 𝐱 = 2p, x - 𝐱 = 2q + 1

Compute a relation on between p and independent of x.


A. B. C. D.

Solution
Given
1
x + x = 2p (i)
1
x - x = 2q + 1 (ii)
Squaring (i) and (ii) and subtract
1
x2 + + 2 = 4p2
x2
1
-x2 - x2 + 2 = -4q2 - 1 - 4q

4 = 4p2 - 4q2 - 4q - 1
4p2 - 4q2 - 4q = 5

𝟏 1
Question.20 if x = √𝟓 + 2, Find the values of x + 𝐱 and x2 + x2

A. B. C. D.

Solution
Given
x = √5 + 2
1 1
=
x √5+2
1 1 √5−2
= x
x √5+2 √5−2
1 √5−2
x
= (√5)2 −(2)2
1 √5−2
=
x 5−4
1 √5−2
=
x 1
1
= √5 - 2
x
Now
1
x + x = √5 + 2 + √5 - 2
1
x + x = 2√5
1
(x + x)2 = (2 √5)2
1
x2 + x2 + 2 = 4 x 5

9|Page
1
x2 + x2 + 2 = 20
1
x2 + x2 = 20 - 2
1
x2 + x2 = 18

𝟏 𝟏
Question.21 If x - 𝐱 = 4, Find the values of x3 - 𝐱𝟑

A. B. C. D.

Solution
Given
1
x-x=4
1
(x − x)3 = (4)3
1 1 1
x3 - x3 +3 (x) (x ) (x - x) = 64
1 1
x3 - x3 -3(x + x) = 64
1
x3 - x3 -3(4) = 64
1
x3 - x3 -12 = 64
1
x3 - x3 = 64 +12
1
x3 - = 76
x3

Question.22 if x = a2b and y = a3√𝐛, Express b in term of x and y.

A. B. C. D.

Solution
Given
x = a2b and y = a3√b
Take
x = a2b
x
= a2
b
3⁄
x 2 3⁄
(b ) = (a)2 x 2
3⁄
x 2
(b ) = a3
Put in
y = a3√b

10 | P a g e
3⁄
x 2
y = (b) √b
3
x ⁄2 1⁄
y= 3 b 2
b ⁄2

3
x ⁄2
y= 3 1
b ⁄2− ⁄2
3
x ⁄2
y= b
3
x ⁄2
b= y
.

Question.23 Simplify the following to find values of x.


𝐱+𝟏 𝐱+𝟑 𝟐(𝐱+𝟏)
+ 𝐱+𝟒 = (𝐱+𝟐)(𝐱+𝟒)
𝐱+𝟐

A. B. C. D.

Solution
x+1 x+3 2x(x+1)
+ x+4 = x(x+2)(x+4)
x+2
(x+1)(x+4)+(x+3)(x+2) 2(x+1)
(x+2)(x+4)
+ (x+2)(x+4)

x2 + 5x + 4 + x2 + 5x + 6 = 2x + 2
2x2 + 10x + 10 = 2x + 2
2x2 + 10x - 2x + 10-2 = 0
2x2 + 8x + 8 = 0
2(x2 + 4x + 4) = 0
x2 + 4x + 4 = 0
x2 + 2x + 2x + 4 = 0
x (x + 2) + 2 (x + 2) = 0
(x + 2) (x + 2) = 0
(x + 2)2 = 0
x+2=0
x = -2

Question.24 Find the value of x if 5 + 2x+2 = 261

A. B. C. D.

11 | P a g e
Solution
Given
5 + 2x+2 = 261
2x+2 = 261 - 5
2x ( 22)= 256
4(2x) = 256
2x = 256/4
2x = 64
2x = 26
x=6
𝟐𝐲 𝟐 +𝟐
Question.25 If x2y = (√𝐱)

A. B. C. D.

Solution
2y2 +2
x2y = (√x)
2y2 +2
x2y = (x1/2 )
2y2 +2
2y
x =x 2
2
x2y = x y +1
2y = y2 + 1
y2 - 2y + 1 = 0
(y - 1)2 = 0
y-1=0
y=1

Question.26 Find the value of x in given equation 82x-4 = 16x

A. B. C. D.

Solution
Given
82x-4 = 16x
(23)2x-4 = (24)x
26x-12 = 24x
6x -12= 4x
6x - 4x = 12
2x = 12
x=6

12 | P a g e
Question.27 Find the value of x
𝐱+𝟑 𝟖 𝐱+𝟑
- = 𝐱−𝟐
𝐱−𝟐 𝟑

A. B. C. D.

Solution
Given
x+3 8 x+3
- = x−2
x−2 3
3(x+3)−8(x−2) x+3
= x−2
3(x−2)
3x - 9 - 8x + 16 = 3x + 9
-8x + 25 = 9
-8x = -16
8x = 16
x=2

(𝐱 𝟑 −𝐲 𝟑 )(𝐱+𝐲)𝟐
Question.28 Simplify the following (𝐱 𝟐 +𝐱𝐲+ 𝐲 𝟐 )(𝐱 𝟐 −𝐲 𝟐 )

A. B. C. D.

Solution
(x3 −y3 )(x+y)2
(x2 +xy+ y2 )(x2 −y2 )

(x−y)(x2 +xy+y2 ) (x+y)2


= (x2 +xy+ y2 )(x+y) (x−y)

=x+y

𝟐𝐱 𝟐 −𝟖 𝟐𝐱 𝟐 −𝟏𝟎𝐱 + 𝟏𝟐
Question.29 ÷
𝐱 𝟐 +𝟔𝐱+ 𝟖 𝐱 𝟐 +𝐱− 𝟏𝟐

A. B. C. D.

Solution
Given
2x2 −8 2x2 −10x + 12
÷
x2 +6x+ 8 x2 +x− 12
2(x2 −4) 2(x2 −5x + 6)
= x2 +4x+ 2x + 8 ÷ x2 +4x −3x− 12
2(x+2)(x−2) 2(x2 −3x− 2x+6)
= x(x+4)+2(x + 4) ÷ x(x+4)−3(x+4)
2(x+2)(x−2) 2(x−3) (x−2)
= (x+2)(x + 4) ÷ (x−3)(x+4)

13 | P a g e
2(x−2) 2(x−2)
= ÷
x+4 x+4
2(x−2) x+4)
= x 2(x−2)
x+4

=1
Question.30 Simplify the following
𝐱−𝟏 𝟐
+ 𝐱+𝟏
𝐱 𝟐 +𝟑𝐱+ 𝟐

A. B. C. D.

Solution
Given
x−1 2
+ x+1
x2 +3x+ 2
x−1 2
= + x+1
x2 +2x+x+ 2
x−1 2
= + x+1
x(x+2)+1(x+ 2)
x−1 2
= +
(x+1)(x+ 2) x + 1
x−1+2(x+2)
= (x+1)(x+ 2)
x−1+2x + 4
= (x+1)(x+ 2)
3x+3
= (x+1)(x+ 2)
3(x+1)
= (x+1)(x+ 2)
3
= x+ 2

Question.31 e2x - 1 = 0

A. B. C. D.

Solution
Given
e2x = 1
taking log on both side
In e2x = In1
2x lne = In1
2x(1) = 0
x= 0

14 | P a g e
Question.32 Find the value of x such that 4x+1 = 2 - 7(2x)

A. B. C. D.

Solution
Given
4x+1 = 2 - 7(2x)
(22)x+1 = 2 - 7(2x)
22x+2 = 2 - 7(2x)
22x . 22 = 2 - 7(2x)
Let
2x = y
22x = y2
y2 . 4 = 2 - 7 (y)
4y2 = 2 - 7 (y)
4y2 + 7y -2 = 0
4y2 + 8y - y - 2 = 0
4y(y + 2) - 1 (y + 2) = 0
(4y - 1) (y + 2) = 0
4y – 1 = 0 or y+2=0
y=¼ y = -2

When when
x
y=2 y = 2x
then then
y=¼ y = -2
2x = ¼ 2x = -2
2x = ½ = 2-2 (Neglect -ve value)
x = -2

Question.33 Given that 8x2 - 2xy - 3y2 = 0 express ‘y’ in terms of ‘x’.

A. B. C. D.

Solution
Given
8x2 - 2xy - 3y2 = 0
8x2 - 6xy + 4xy - 3y2 = 0
2x(4x - 3y) + y (4x - 3y) = 0
(2x + y) (4x - 3y) = 0
15 | P a g e
2x + y = 0 or 4x - 3y = 0
y = -2x or +3y = +4x
y = 4/3x

Question.34 For positive integers x and y, Find all possible set of values satisfying the
following equations 2xy (xy - 11) = 420

A. B. C. D.

Solution
Given
2xy (xy - 11) = 420
Let
xy = t
2t (t - 11) = 420
2t2 -22t = 420
2t2 -22t - 420 = 0
2(t2 - 11t - 210) = 0
t2 - 11t-210 = 0
t2 - 21t + 10t - 210 = 0
t(t-21) + 10(t - 21) = 0
(t + 10) (t - 21) =
t + 10 = 0 or t - 21 = 0
t = -10 or t = 21
xy = -10 xy = 21
As x and y are positive Integers (Neglect -ve value)
Hence all value for x and y are
{(7, 3) , (3, 7) , (21, 1), (1, 21)}

Question.35 Solve the following equation


4m4 - 9m2 + 2 = 0

A. B. C. D.

Solution
Given
4m4 - 9m2 + 2 = 0
4m4 - 8m2 - m2 + 2 = 0
4m2 (m2 - 2) -1 (m2 - 2) = 0
(4m2 - 1) (m2 - 2) = 0
Either
16 | P a g e
4m2 -1 = 0 or m2 = 2
m2 = 1/4 m = +√2
m = +1/2 m = +1.414

Question.36 If x-1 = 5-1 + 7-1 then x is

A. B. C. D.

Solution
Given
x-1 = 5-1 + 7-1
1 1 1
=5+7
x
1 7+5
=
x 35
1 12
= 35
x
35
x = 12

Question.37 Roots of the equation (4y - 3) (2y + 5) = 0 are

A. B. C. D.

Solution
Given
(4x - 3) (2y + 5) = 0
(4y - 3) = 0 or (2y + 5) = 0
4y = 3 or 2y = -5
y=¾ or y = -5/2

Question.38 The equation x2 + kx - 18 = 0 where k is a constant, is satisfied by x = 2 then


value of k is.

A. B. C. D.

Solution
Given
x2 + kx - 18 = 0
put x = 2
then
(2)2 + k(2) - 18 = 0
4 + 2k - 18 = 0
2k - 14 = 0
2k = 14

17 | P a g e
k=7

Question.39 Solution of equation (x + 5)2 = 16

A. B. C. D.

Solution
Given
(x+5)2 = 16
√(x + 5)2 = √16
x+5=±4
x = -5 ± 4
x = -5 + 4, -5 -4
x = -1, -9

Question.40 Pick up the correct value of x for which


𝐱 𝟏 𝐱 𝟏
- 𝟎.𝟎𝟓 + 𝟎.𝟎𝟎𝟓 - 𝟎.𝟎𝟎𝟎𝟓 = 0
𝟎.𝟓

A. B. C. D.

Solution
Given
x 1 x 1
- 0.05 + 0.005 - 0.0005 = 0
0.5
10x 100 1000x 10000
- + - =0
5 5 5 5
10x−100+1000x−1000
=0
5

1010x - 10100 = 0
1010x = 10100
10100
x= 1010

x = 10

18 | P a g e
QUADRATIC FORMULA

Question.41 Find roots of the following equations by using quadratic formula


5x2 - 13x + 6 = 0

A. B. C. D.

Solution
Given
5x2 - 13x + 6 = 0
Here
a = 5, b = -13, c = 6
Using quadratic formula
−b ± √b2 −4ac
x= 2a
−(−13) ± √(−13)2 −4(5)(6)
x= 2(5)
13 ± √169 −120
x= 10
13 ± √49
x= 10
13 ± 7
x= 10
13+ 7 13 − 7
x= ,
10 10
20 6
x = 10 , 10
x = 2, x = 3/5
Solution set = {3/5, 2}

Question.42 4x2 + 7x - 1 = 0

A. B. C. D.

Solution
Given
4x2 + 7x - 1 = 0
Here
a = 4, b = 7, c = -1

Using quadratic formula


−b ± √b2 −4ac
x= 2a
−7 ± √(7)2 −4(4)(1)
x= 2(4)

19 | P a g e
−4 ± √49+16
x= 8
−7 ± √65
x= 8
−7±√65
Solution set = { }
8

Question.43 15x2 + 2ax - a2 = 0

A. B. C. D.

Solution
Given
15x2 + 2ax - a2 = 0
Here
a = 15, b = 2a, c = -a2
Using quadratic formula
−b ± √b2 −4ac
x= 2a
−2a ± √(2a)2 −4(15)(a2 )
x= 2(15)
−2a ± √4a2 + 60a2
x= 30
−2a ± √64a2
x= 30
−2a ± 8a
x=
30
−2a+ 8a −2a − 8a
x= ,
30 30
6a −10a
x = 30 , 30
−a
x = a/5 , x= 3
Solution set = {a/5, -a/3}

Question.44 16x2 + 8x + 1 = 0

A. B. C. D.

Solution
Given
16x2 + 8x + 1 = 0
Here
a = 16, b = 8, c = 1
Using quadratic formula
−b ± √b2 −4ac
x= 2a

20 | P a g e
−8 ± √(8)2 −4(15)(1)
x= 2(16)
−8 ± √64−64
x= 32
8±0
x=- 32
8+0 8−0
x=- , x=-
32 32
−8 −8
x = 32 , 32
x = -1/4 , -1/4
Solution set = {-1/4, -1/4}

Question.45 Using quadratic formula, solve the following equation


x + 10 = 11x2 - x + 1

A. B. C. D.

Solution
Given
x + 10 = 11x2 - x + 1
11x2 - 2x - 9 = 0
Here
a = 11, b = -2, c = -9
Using
−b±√b2 −4ac
x= 2a

−(−2)±√(−2)2 −4x 11 x−9


x= 2(11)
2±√4+396
x= 22
2±20
x= 22
2+20 2+20
x= or
22 22
22 18
x= or -
22 22
19
x = 1 or -11

Question.46 Determine the value of x by using quadratic formula where 9x2 - 81 = 0

A. B. C. D.

21 | P a g e
Solution

Given
9x2 - 81 = 0
Here
a = 9, b = 0, c = -81
Using
−b+√b2 −4ac
x= 2a
−0+√0−4(a)(−81)
x= 2(9)
x = ±√2916)/18
x = + ±54/18
x = ±3

Question.47 Solve the following equation by using quadratic formula,


2 + 6√𝟐 𝒚 + 9y2 = 0

A. B. C. D.

Solution

Given
9y2 + 6√2 y + 2 = 0
Here
a = 9, b = 6√2 , c = 2

Using
−b ±√b2 −4ac
y= 2a
2
−6√2±√(6√2) −4x9x2
y= 2(9)
−6√2±√72−72
y= 18
−6√2±√0
y= 18
−6√2±0
y= 18
−6√2
y= 18

√2
y= 3

22 | P a g e
Question.48 Solve the following equation
2.5x2 + 3.25x – 15.6 = 0

A. B. C. D.

Solution
Given
2.5x2 + 3.25x - 15.6 = 0
25x2 3.25 15.6
+ x- =0
10 100 10

250x2 + 325x - 1560 = 0


Here
a = 250, b = 325 , c = -1560

Using
−b ± √b2 −4ac
x= 2a

−325±√(325)2 −4 x 250 (−1560)


x= 2(250)

−325 ±√105,625+1,560,000
x= 500
−325 ±√1665625
x= 500
−325 ±√1290.59
x= 500
−325+ 1290.59 −325 + 1290.59
x= or
500 500
965.59 1615.59
x= or x=-
500 500

x = 1.93 x = -3.23

23 | P a g e
Completing square

Question.49 Solve the following equations by completing the square


x2 - 2x - 899 = 0

A. B. C. D.

Solution
Given
x2 - 2x - 899 = 0
x2 - 2x = 899
x2 - 2x + 1 = 899 + 1
(x - 1)2 = 900
√(x − 1)2 = √900
x – 1 = + 30
x = 1 + 30
x = 1 + 30 , 1 – 30
x = 31, -29

Hence
Solution set = {31, -29}

Question.50 x2 - 4x - 1085 = 0

A. B. C. D.

Solution
Given
x2 - 4x - 1085 = 0
x2 - 4x = 1085
x2 - 4x + (2)2 = 1085 + (2)2
(x - 2)2 = 1085 + 4
(x - 2)2 = 1089
√(x − 2)2 = √1089
x + 2 = + 33
x + 2 = 33 , x + 2 = -33
x = 33 - 2 , x = -33 -2
x = 31, x = -35
Solution set = {31, -35}

24 | P a g e
Question.51 x2 + 6x - 567 = 0

A. B. C. D.

Solution
Given
x2 + 6x - 567 = 0
x2 - 6x = 567
x2 + 6x + (3)2 = 567 + (3)2
x2 + 6x + (3)2 = 567 + 9
(x - 3)2 = 576
√(x + 3)2 = √576
x + 3 = + 24
x = -3 + 24
x = -3 + 24 , -33 -24
x = 21, -27
Solution set = {21, -27}

Question.52 x2 - 3x - 648 = 0

A. B. C. D.

Solution
Given
x2 - 3x - 648 = 0
x2 - 3x = 648
3 3
x2 - 3x + (2)2 = 648 + (2)2
3 9
x2 - 3x + (2)2 = 648 + 4
3 2592+ 9
(x - 2)2 = 4
3 2 2601
(x - ) =
2 4

2
√(𝑥 − 3) = √2601
2 4

3 51
x-2=+ 2
3 51
x=2 + 2
3 51 3 51
x=2+ ,2-
2 2

25 | P a g e
3+51 3−51
x= ,
2 2
54 −48
x= ,
2 2

x = 27, -24
Solution set = {27, -24}

Question.53 x2 - x - 1806 = 0

A. B. C. D.

Solution
Given
x2 - x - 1806 = 0
x2 - x = 1806
1 1
x2 - x + (2)2 = 1806 + (2)2
1 1
(x - 2)2 = 1806 + 4
1 7224+1
(x - 2)2 = 4
1 2 7225
(x - ) =
2 4

2
√(𝑥 − 1) = √7225
2 4

1 85
x-2=+ 2
1 85
x=2 + 2
1 85 1 85
x=2+ ,2-
2 2
1+ 85 1−85
x= ,
2 2
86 −84
x= ,
2 2

x = 43, -42
Solution set = {43, -42}

Question.54 2x2 + 12x - 110 = 0

A. B. C. D.

26 | P a g e
Solution
Given
2x2 + 12x - 110 = 0
Divided by ‘2’
x2 + 6x – 55 = 0
x2 + 6x = 55
x2 + 6x + (3)2 = 55 + (3)2
x2 + 6x + (3)2 = 55 + 9
(x +3)2 = 64
√(x + 3)2 = √64
x+3=+8
x=+8-3
x = +8 - 3, -8 -3
x = 5 , -11
Solution set = {5, -11}

27 | P a g e
SIMULTANEOUS EQUATION

Question.55 Solve the following simultaneous equation.


2x + y = -7
3x + 2y = -12

A. B. C. D.

Solution
Given
2x + y = -7 ---- (i)
3x + 2y = -12 --- (ii)
3(i) -2 (ii)
6x + 3y = -21
-6x - 4y = +24
-y = +3
y = -3
Putting y = -3 in (i)
2x + (-3) = -7
2x - 3 = -7
2x = -7 + 3
2x = -4
x = -2
Solution set = {(-2, -3)}

Question.56 93x + 15y = 123


15x + 93y = 201

A. B. C. D.

Solution
Given
93x + 15y = 123 (i)
15x + 93y = 201 (ii)
15(i) -93 (ii)
1395x + 8649y = 18693
-1395x + 225y = +1845
8424y = 16848
y=2

Putting ‘y = 2’ in (i)
93x + 15(2) = 123
93x + 30 = 123
93x = 123 - 30
93x = 93
28 | P a g e
x=1
Solution set = {(1, 2)}

Question.57 465x + 75y = 615


75x + 465y = 1005

A. B. C. D.

Solution
Given
465x + 75y = 615
75x + 465y = 1005
75(i) -465 (ii)
34875x + 216225y = 467325
-34875x + 5625y = + 467325
210600y = 421200
y=2
Putting ‘y = 2’ in (i)
465x + 75(2) = 615
465x + 150 = 615
465x = 615 - 150
465x = 465
x=1
Solution set = {(1, 2)}
𝟐 𝟑
Question.58 +𝐲=2
𝐱
𝟖 𝟗
+𝐲=7
𝐱

A. B. C. D.

Solution
Given
2 3
+y=2 (i)
x
8 9
+y=7 (ii)
x
4(i) – (ii)
8 12
+ y =8
x
8 9
− x − y = +7

3/y = 1
y=3
Putting ‘y = 3’ in (i)
2 3
+3=2
x

29 | P a g e
2
+1=2
x
2
=2–1
x
2
=1
x
x=2
Solution set = {(2, 3)}

Question.59 Find the values of x and y in the given set of Equations


𝐱−𝟏 𝟒 𝐱−𝟓 𝟏
= 𝟓 and 𝐲+𝟓 = 𝟐
𝐲+𝟏

A. B. C. D.

Solution
x−1 4
=5 ------ (i)
y+1
x−5 1
and =2 (ii)
y+5
From (i)
x−1 4
=5
y+1
5(x-1) = 4(y + 1)
5x - 5 = 4y + 4
5x - 4y = 9 ----- (iii)
From (ii)
x−5 1
=2
y+5
2(x - 5) = 1(y + 5)
2x - 10 = y + 5
2x - y = 15 -----(iv)
Solving (iii) and (iv)
5x - 4y = 9
2x - y = 15
2(i) – 5(ii)
10x - 8y = 18
-10x + 5y = -75
-3y = -57
3y = 57
y = 19
Putting value of y = 19 in (iii)
5x - 4(19) = 9
5x - 76 = 9
5x = 9 + 76
5x = 85
x = 17

30 | P a g e
Hence solution set = [(17, 19)]

Question.60 Solution of the equations


𝟒 𝟓 𝐱+𝐲 𝟑
- = + 𝟏𝟎
𝐱 𝐲 𝐱𝐲
and 3xy = 10(y-x) is

A. B. C. D.

Solution
Given
4 5 x+y 3
- = + ------ (i)
x y xy 10
Also
3xy = 10(y - x) -------- (ii)
From ( i)
4y−5x 10x+10y+3xy
=
xy 10xy
10(4y - 5x) = 10x + 10y + 3xy
40y - 50x = 10x + 10y + 3xy
30y - 50x = 10x + 10y + 3xy
30y - 60x = 3xy ------- (iii)

Compare (ii) and (iii)


10(y - x) = 30y - 60x
10y - 10x = 30y - 60x
10y - 30x = -60x + 10x
-20y = -50x
50
y = 20x
5
y = 2x -------(iv)
Put in (ii)
5x 5x
3x( 2 ) =10( 2 − x)
15x2 5x−2x
= 10 ( )
2 2
15x2
= 25x - 10x
2
15x2 = (15x)(2)
15x = 30x
x = 30/15
Put x = 2 in (iv)
5
y=2x2

31 | P a g e
y=5
Hence solution set is
(x, y) = {(2 , 5)}

Question.61 Solution set for simultaneous equations 2x - 3y = 19 and


3x + 2y = -4 is

A. B. C. D.

Solution
Given
2x - 3y = 19 ---------(i)
3x + 2y = -14 ---------(ii)
3(i) - 2(ii)
6x – 9y = 57
-6x - 4y = +8
-13y = 65
−65
y = 13
y = -5
Put in (i)
2x - 3(-5) = 19
2x + 15 = 19
2x = 19 - 15
2x = 4
x=2
Hence solution set is
(x, y) = (2, -5)

Question.62 The solutions of the equation


𝟔𝐱 𝟔(𝐱+𝟏)
+ = 13 are
𝐱+𝟏 𝐱

A. B. C. D.

Solution
Given
6x 6(x+1)
+ = 13
x+1 x
x(6x)+6(x+1)(x+1)
= 13
x(x+1)
6x2 +6(x+1)2
= 13
x2 +x
6x2 + 6(x2 + 2x + 1) = 13(x2 + x)
6x2 + 6x2 + 12x + 6 = 13x2 + 13x

32 | P a g e
12x2 + 12x + 6 = 13x2 + 13x
x2 + x - 6 = 0
x2 + 3x - 2x - 6 = 0
x(x+3) -2(x+3) = 0
(x-2)(x+3) = 0
Either
x-2=0 or x+3=0
x=2 x = -3
Hence solution set is
(x, y) = (2, -3)

Question.63 If 7x – 5y = 13 ; 2x - 7y = 26 then 5x + 2y is

A. B. C. D.

Solution
Given
7x - 5y = 13 (i)
-2x +7y = 26 (ii)
(i) – (ii)
5x + 2y = -13
Question.64 4y - x = 10 and 3x = 2y then xy is

A. B. C. D.

Solution
Given
-x + 4y = 10 (i)
3x - 2y = 0 (ii)
3(i) + (ii)
-3x + 12y = 30
3x _- 2y = 0
10y = 30
y=3
Putting value of ‘y = 3’ in (ii),
3x - 2(3) = 0
3x - 6 = 0
3x = 6
x=2
Now
xy = (2) (3) = 6
33 | P a g e
STATEMENTS PROBLEM
Type:1 (Number)

Question.65 A student chose a number, multiplied it by 2, then subtracted 138 from the
result and got 102. What was the number he chooses?

A. B. C. D.

Solution
Let
The required number = x
Than
According to statement
2x – 138 = 102
2x = 102 + 138
2x = 240
x = 120

Question.66 In a cricket match bye runs were double of the no-balls. The remaining score
was greater by 6 than twelve times the number of byes runs. If total score was 276. How
were the runs obtained?

A. B. C. D.

Solution
Let
The bye run = x
No ball runs = y
Remaining score = z
Then
According to statement
x + y + z = 276 -------(i)
x = 2y -------(ii)
z = 6 + 12x -------(iii)

Putting value of y and z from (ii) and (iii) in (i)


x
x + 2 + 6 + 12x = 276
2x+x+12+24x
= 276
2
27x + 12 = 552
27x = 552 - 12

34 | P a g e
27x = 540
x =20
Putting value of x in (ii)
20 = 2y
20
=y
2
y = 10
Putting value of ‘x = 20’ in (iii)
z = 6 + 12(20)
z = 6 + 240
z = 246

Question.67 The number of colour T.V sets sold by a firm was three times the combined
sale of C.D players and radios. If the sales include 72 T.V sets and 8 radios,
how many C.D players are sold?

A. B. C. D.

Solution
Let
Number of T.V sold = x
Number of C.D players sold = y
Number of radios sold = z
According to statement
x = 3 (y + z) ------- (i)
Also
x = 72 --------- (ii)
z=8 ------------ (iii)
putting value of x and z in (i)
72 = 3(y + 8)
72 = 3y + 24
3y = 48
y = 16

Question.68 A customer paid Rs. 800 for shampoo, hair conditioner and hair spray. The
conditioner cost twice as much as the shampoo and the hair spray cost Rs.
100 less than the shampoo. How much did each item cost?

A. B. C. D.

Solution
Let
Cost of shampoo = x
Cost of conditioner = 2x
Cost of hair spray = x - 100
35 | P a g e
Also
Total cost = 800
According to statement
x + 2x + x – 100= 800
4x – 100 = 800
4x = 900
Cost of shampoo x = 225
Cost of conditioner = 2x = 2 x 225 = 45
Cost of hair spray = x – 100 = 225 - 100 = 125

Question.69 Three times the square of a number when added to seven times the number,
result is 26. The number is.

A. B. C. D.

Solution

Let
The required number = x
Then according to statement
3x2 + 7x = 26
3x2 + 7x - 26 = 0
3x2 + 13x - 6x - 26 = 0
x(3x + 13) -2 (3x + 13) = 0
(x – 2) (3x + 13) = 0
Either
x–2=0 or 3x + 13 = 0
x=2 or 3x = -13
x = -13/3
Hence solution is
x = 2 and x = -13/3
Question.70 There are two consecutive numbers such that the difference of their
𝟏
reciprocal is 𝟐𝟒𝟎. The numbers are.

A. B. C. D.

Solution
Let two consecutive numbers are
x, x+1
According to statement

36 | P a g e
1 1 1
- =
x x + 1 240
x+1−x 1
=
x(x+1) 240
1 1
= 240
x2 + x
2
x + x = 240
x2 + x –240 = 0
x2 + 16x - 15x - 240 = 0
x (x + 16) -15( x + 16) = 0
(x - 15) (x + 16) = 0
Either
x - 15 = 0 or x + 16 = 0
x = 15 or x = -16 (neglect -ve sign)

First consecutive number = x = 15


2nd consecutive number = x + 1 = 15 + 1 = 16

Question.71 Two numbers are such that twice the greater number exceeds twice the
𝟏 𝟏
smaller one by 18 and 𝟑 of smaller and 𝟓 of greater numbers are together 21.

A. B. C. D.

Solution
Let
The smaller number = x
Greater number = y
According to statement
2y = 18 + 2x
2x – 2y = -18 (i)
And
1 1
x + 5 y = 21
3
5x+3y
= 21
15
5x + 3y = 315 (ii)
5(i) -2 (ii)
10x – 10y = -90
-10x- 6y = -630
-16y = -720
16y = 720
y = 45
Greater number = y = 45
Putting value of ‘y = 45’ in (i)
2x – 2(45) = -18
2x - 90 = -18
2x = -18 + 90

37 | P a g e
2x = 72
Smaller number = x = 36

Question.72 On Pakistan day 9 kilogram of sweets were distributed among children. Each
child received as many grams of sweets as the number of children and still
164 grams of sweets left over. Find number of children.

A. B. C. D.

Solution
Total weight of sweets = 9000 grams
Total sweets distributed = (No. of children) (No. of sweets)
= (x)(x)
= x2
According to statements
x2 + 164 = 9000
x2 = 9000 – 164
x2 = 8835
No. of children = x = 94

Question.73 A sum of two numbers is 18 and the sum of the square of the numbers is 194.
Find the numbers.

A. B. C. D.

Solution
Let
First number = x
2nd number = y
According to statement
x + y = 18 (i)
x2 + y2 = 194 (ii)

From (i)
x + y = 18
x = 18 - y (iii)
Putting value of x in (ii)
(18 - y) 2 + y2 = 194
324 + y2 - 36y + y2 = 194
2y2 - 36y + 130 = 0
2(y2 - 18y + 65) = 0
y2 - 18y + 65 = 0
y2 - 13y - 5y + 65 = 0

38 | P a g e
y(y - 13) -5 (y - 13) = 0
(y - 5) (y -13) = 0

Either y - 5 = 0 or y - 13 = 0
y=5 or y = 13

When When
y=5 y = 13
then x = 18 – 5 then
x = 13 x = 18 - 13
x=5
Hence required numbers are (5,13)

Question.74 The product of four greater than a certain negative integer and sixteen less
than four times the integer is equal to 36. The integer is.

A. B. C. D.

Solution
Let the required negative integer = x
According to statement
(x + 4) (4x – 16) = 36
4x2 - 16x + 16x - 64 = 36
4x2 = 36 + 64
4x2 = 100
x2 = 25
x = +5
Hence required negative integer is -5

Question.75 The length of rectangle is 4 times its width. The perimeter of the rectangle is
150 cm. Find the dimensions of the rectangle.

A. B. C. D.

Solution
Let
Length or rectangle = x
Width or rectangle = y
According to statement
4x = y ------ (i)
Also perimeter = 2(x + y) = 150

39 | P a g e
2x + 2y = 150 ------(ii)
Putting value of ‘y = 4x’ in (ii)
2x + 2(4x) = 150
2x + 8x = 150
10x = 150
Length of rectangle = x = 15
Putting value of ‘x’ = 15’ in (i)
y = 4(15)
Width of rectangle = y = 60

Question.76 The third angle of an isosceles Triangle is 16 less than the sum of two equal
angles. Find three angles of the Triangle.

A. B. C. D.

Solution
Let
The equal angle = x
The Third angle = y
Than According to statement
2x - 16 = y
2x - y = 16 ---- (i)

(i) + (ii)
Also
x+x+y =180
2x + y = 180
4x = 196
x = 49
Putting value of ‘x = 49’ in (i)
2(49) - y = 16
98 - y = 16
98 - 16 = y
y = 82

Question.77 The difference between two numbers is 2 and difference between their
square is 32. Find the number.

A. B. C. D.

Solution

40 | P a g e
Let
First number = x
2nd number = y
Than
According to statement
x-y=2 (i)
x2 - y2 = 32 (ii)
From (i)
x-y=2
x=2+y (iii)
Putting value of x in (ii)
(2 + y)2 – y2 = 32
4 + y2 + 4y – y2 = 32
4y = 32 – 4
4y = 28
2 nd number = y = 7
Putting value of ‘y = 7’ in (iii)
x=2+7
First number = x = 9

Question.78 A certain number when added to the square of the number results in 8.75.
What is the number?

A. B. C. D.

Solution

Let the required number = x

Then
According to statement
x + x2 = 8.75
x2 + x – 8.75 = 0
Here
a = 1 , b = 1 , c = -8.75
using
−b ± √b2 −4ac
x= 2a

−1 ± √1−4x1 x (−8.75)
x= 2(1)

41 | P a g e
−1 ± √1+35
x= 2
−1 ± √36
x= 2
−1 ±6
x= 2
−1 + 6 −1−6
x= ,
2 2
5 −7
x=2, 2

Question.79 The sum of digits in a three digits number is 12. If the digits are reversed the
number increased by 495 but reversing only of the ten’s and unit digits increase the
number by 36. The number is

A. B. C. D.

Solution

Let required numbers are

a= 100 digit
b= 10’s digit
c= unit digit
then the number is
100a + 10b +c
when numbers are reversed
100a +10b +c = 100c +10b +a-495
99a = 99c - 495
a = c-5 ---------(i)
Also

100a +10 b +c =100a +10c +b -36


b= c-4 -------(ii)
Also
a+b+c =12
Putting value of a and b
c-5 +c-4 +c =12
3c = 12+9
c= 21/3
c=7

42 | P a g e
Question: 80 A numbers consist of two digits whose sum is 10 if 18 is subtracted from the number
if its digits are reversed. Find the number

Solution:
Let
The unit number = y
10th = x
According to statement
x + y = 10 ___ (i)
If numbers are reversed. Than
10x + y -18 = 10y +x
9x – 9y = 18 ____ (ii)
Using calculator
x = 6 and y = 4
Hence the number is 64

Question: 81 if from twice the greater of two numbers 20 is subtracted. The result is the other
number. If from twice the smaller number 5 is subtracted. The result is the first
number. The greater number is.
Solution:
Let larger number = x
Smaller number = y
According to statement
2x – 20 = y
2x – y = 20 ___ (i)
Also
2y – 5 = x
-x + 2y = 5 ____ (ii)
Using calculator
x = 15 and y = 10

Question: 82 The sum of 2 numbers is 80. If the larger number exceeds four times the smaller one
by 5, then the smaller number is.
Solution:
Let x and y be two number and x > y
Then
X + y = 80 __ (i)
Also
X = 4y -5 ___ (ii)
Solving (i) and (ii)
80 – Y = 4y -5
75 = 5y
Y = 15

43 | P a g e
Question: 83 A two digit number is such that the product of the digit is 8 when 18 is added to the
number digits are reversed. The number is
Solution:
Let
The digits be x and y
X = 10th unit
Then
Y = one unit
Then
x y = 8 __ (i)
Also
Digit
10 x + y
Then according to statement
10x + y +18 = 10y + x
9x – 9y = -18 __ (ii)
From (i)
8
X=
𝑦
Put in (ii)
8
9 ( ) – 9y = -18
𝑦
72
− 9𝑦 = −18
𝑦
72 - 9𝑦 2 = −18𝑦
9𝑦 2 – 18y – 72 = 0

Using calculator
Y=

44 | P a g e
Type: 2 (Mixture)
Question.84 A company manufactures two products A and B. Each unit of A takes 20
hours in assembling department and 5 hours in finishing department. Each
unit of B takes 10 hours for assembling and 3 hours for finishing. If the total
number of hours available in assembling and finishing departments are 450
and 130 respectively. Calculate the number of each product that can be
produced provided both departments are fully utilized.

A. B. C. D.

Solution
Let
Number of units produced for product A = x

Numbers of units produced for product B = y


Then According to statement
20x + 10y = 450 (i)
5x + 3y = 130 (ii)
(i) -4 (ii)
20x + 10y = 450
-20x - 12y =- 520
-2y = -70
y = 35
Putting value of ‘y=35’ in (i)
20x + 10(350) = 450
20x + 350 = 450
20x = 450 – 350
20x = 100
x=5

Hence
Units produced for product A = 5
Units produced for product B = 35

Question.85 Two ingredient x and y, each measured in kilograms, are used to make two
different batches of vitamin C tablets, For batch I, x and y must be mixed in
the weight ratio of 5x to 3y. For batch II, the weight ratio is 6x to 2y
production requirements are 4000 kg of batch I and 3000 kg of batch II. How
much of each ingredient should be purchased to satisfy the production
requirements?

45 | P a g e
A. B. C. D.

Solution
According to given statement
For batch I 5x + 3y = 4000 (i)
For batch II 6x + 2y = 3000 (i)
6(i) - 5(ii)
30x + 18y = 2400
30x + 10y = 15000
8y = 9000
y = 1125
Putting value of y = 1125 in (i)
5x + 3(1125) = 4000
5x + 3375 = 4000
5x = 4000 - 3375
5x = 625
x = 125

Question.86 A grocer wants to mix nuts which sell for Rs. 4 per kilo with nuts which sells
for Rs. 7 per kilo in order to make a mixture which could sell for Rs. 5 per
kilo. How many kilos of each should be mix to get mixture of 42 kilos?

A. B. C. D.

Solution
Let
Quantity of nuts which sells for Rs. 4 per kilo = x
Quantity of nuts which sells for Rs. 7 per kilo = y
Than
According to given statement
x + y = 42 (i)
and
4x + 7y = 5 x 42
4x + 7y = 210 (ii)
Now
4(i) – (ii)
4x + 4y = 168
-4x -7y = -210
-3y = - 42
3y = 42
Quantity for = y = 14

46 | P a g e
As per kilo
Putting value of ‘y = 14’ in (i)
x + 14 = 42
x = 42 – 14
x = 28

Quantity of Rs. 4 per kilo = x = 28

47 | P a g e
Type:3 (To Find Age difference)
Question.87 Zain is x years old and his sister Ifrah is (5x - 12) years old. Given that Ifrah
is twice as old as Zain, then the age of Ifrah is

A. B. C. D.

Solution
Given
Age of Zain = x
Age of Ifrah = 5x - 12
According to statement
5x - 12 = 2x
5x - 2x = 12
3x = 12
Zain age = x = 4 years
Ifrah age = 5x - 12 = 5(4) - 12 = 20 - 12 = 8 years

Question.88 Ten years ago the age of a father was four times of his son. Ten years hence
the age of the father will be twice that of his son. The present ages of the
father and son are.

A. B. C. D.

Solution
Let the age of son = x
Age of father = y
According to statement

10 years before 10 years after


Age of son = x - 10 Age of son = x + 10
Age of father = y - 10 Age of father = y + 10
Then Then
y - 10 = 4 (x - 10) y + 10 = 2 (x + 10)
y - 10 = 4x - 40 y + 10 = 2x + 20
4x - y = -10 + 40 2x - y = 10 - 20
4x - y = 30 - (i) 2x - y = -10 (ii)
(i) -2 (ii)
4x - y = 30
-4x +2y = +20
y = 50

48 | P a g e
Age of father = y = 50 years
Putting value of y = 50’ in (i)
4x - 50 = 30
4x = 30 + 50
4x = 80
Age of son = x = 20

Question.89 A boy is 24 years younger than his father in 2 years’ time, the sum of their
ages will be 40. How old is the father and how old is the son now?

A. B. C. D.

Solution
Let
Age of father = x
Age of son = y
then
According to statement
x – 24 = y
x – y = 24 (i)
After two years
(x + 2) + (y + 2) = 40
x + y + 4 = 40
x + y = 36 (ii)
(i) - (ii)
x – y = 24
-x - y = -36
-2y = - 12
2y = 12
Age of son = y = 6
Putting value of ‘y = 5 in (i)
x – 6 = 24
x = 24 + 6
Age of father = x = 30

Question.90 Present age of a father is three times that of his son but after 22 years, it will
be only twice of his son. Find his present ages.

A. B. C. D.

Solution
49 | P a g e
Let
Age of father = x
Age of son = y
then
According to statement
x = 3y
x - 3y = 0 (i)
After 22 years
Age of father = x + 22
Age of son = y + 22
Then according to statement
x + 22 = 2(y + 22)
x + 22 = 2y + 44
x - 2y = 44 - 22
x - 2y = 22 (i)
(i) – (ii)
x – 3y = 0
-x +2y = -22
-y = -22
Age of son = y = 22
Putting value of y = 22 in (i)
x – 3(22) = 0
x – 66 = 0
Age of father = x = 66

Question.91 Age of a father and his son are in the ratio 10 : 3. Eight year after their ages
will be in the ratios 12 : 5. Find their ages.

A. B. C. D.

Solution
Let
Age of father = x
Age of son = y
According to statement
x 10
=
y 3
3x = 10y
3x - 10y = 0 (i)
After 8 years

50 | P a g e
Age of father = x + 8
Age of son = y + 8

According to statement
x+8 12
=
y+8 5
5(x + 8) = 12(y + 8)
5x + 40 = 12y + 96
5x – 12y = 96 – 40
5x – 12y = 96 – 40
5x – 12y = 56 (ii)
5(i) -3 (ii)
15x - 50y = 0
-15x +36y = 168
-14y =-168
168
y= 14
Age of son = y = 12
Putting value of y = 12 in (i)
3x – 10(12) = 0
3x – 120 = 0
3x = 120
Age of father = x = 40

Question.92 A man is four times as old as his son and five years ago the product of their
ages were 234. Find their present ages.

A. B. C. D.

Solution
Let
Age of father = x
Age of son = y
According to statement
x = 4y ------ (i)
5 years ago
Age of father = x - 5
Age of son = y - 5

Then according to statement


(x - 5) (y - 5) = 234 (ii)

51 | P a g e
4y2 - 20y - 5y + 25 = 234
4y2 - 25y + 25 = 234
4y2 - 25y + 25 - 234 = 0
4y2 - 25y + 25 - 209 = 0
4y2 - 44y + 19y - 20 = 0
4y(y - 11) + 19 (y - 11) = 0
(4y + 16) (y - 11) = 0
Either 4y + 19 = 0 or y - 11 = 0
4y = -19 y = 11
y = -19/4
(Age cannot be negative)
Putting value of ‘y = 11’ in (i)
x = 4(11)
Age of father = x = 44
Age of son = y = 11

Question.93 The ages of a person and his son add up to 45 years. Five years ago the
person was six times as old as his son. Find the age of the person when his
son was born.

A. B. C. D.

Solution
Let
Age of father = x
Age of son = y
According to statement
x + y = 45 (i)
Five year ago
Age of father = x - 5
Age of son = y - 5
According to statement
x - 5 = 6(y - 5)
x - 5 = 6y - 30
x - 6y = -30 + 5
x - 6y = -25 (ii)
(i) - (ii)
x + y = 45
-x +6y = 25
7y = 70
Age of son = y = 10

52 | P a g e
Putting value of ‘y = 10’ in (i)
x + 10 = 45
x = 45 - 10
Age of father = x = 35

Question.94 The sum of ages of a girl and his brother is 26 years. Three years ago her age
was 4 times the age of her brother. Find out the present ages of the girl and
her brother.

A. B. C. D.

Solution
Let
Age of girl = x
Age of brother = y
Than
According to statement
x + y = 26 (i)
Three years ago

Age of girl = x - 3
Age of brother = y - 3
Than
According to statement
x - 3 = 4(y-3)
x - 3 = 4y - 12
x – 4y = -12 + 3
x – 4y = -9 (ii)
(i) – (ii)
x + y = 26
x + 4y = +9
5y = 35
Age of brother = y = 7
Putting value of ‘y = 7’ in (i)
x + 7 = 26
x = 26 - 7
Age of girl = x = 19

53 | P a g e
Type:4 (Cost Determination)

Question.95 Cost of 3 pens and 8 pencils is Rs. 100 whereas cost of 5 pens and 6 pencils is
Rs. 130. Find the cost of a pen and a pencil.

A. B. C. D.

Solution
Let
Cost of pen = x
Cost of pencil = y
According to given condition
3x + 8y = 100 ------- (i)
5x + 6y = 130 -------- (ii)

5 (i) -3 (ii)

15x + 40y = 500


-15x - 18y =-390
22y = 110
y=5
Putting value of y is (i)
3x + 8(5) = 100
3x + 40 = 100
3x = 60
x = 20
Hence Cost of pen = x = 20
Cost of pencil = y = 5

Question.96 Heat and electricity together cost a company Rs.1080 for the month of
January. If the consumption for heating purposes is three times as much as light,
how much each expense cost to the company?

A. B. C. D.

Solution
Total cost = 1080
Let
Lighting expense = x
Heating expense = 3x
According to statement
x + 3x = 1080
4x = 1080
Lighting expense = x = 270

54 | P a g e
Heat expense = 3x = 3(270) = 810
𝟐
Question.97 A type writer is sold for Rs.1500. The gross profit is of the Cost. Find the
𝟑
cost and the gross profit.

A. B. C. D.

Solution
Given
Selling price = 1500
Let
Cost price = x
2
Gross profit = 3 x
Than
According to statement
Selling price = Cost price + Gross profit
2
1500 = x + 3 x
3x+2x
1500 = 3
4500 = 5x
Cost price = x = 900
2 2
Gross profit = 3 x = 3 (900) = 600

Question.98 A piece of iron rod costs Rs. 60. If the rod was 2 meter shorter and each
meter cost Rs. 1.00 more, the cost would remain unchanged. The length of
the rod is.

A. B. C. D.

Solution
Let the cost of rod = x
Length of rod = y
According to statement
xy = 60 ------- (i)
and (x + 1) (y – 2) = 60 -------- (ii)

From (i)
xy = 60
60
y= ------ (iii)
x
Put in (ii)

55 | P a g e
60
(x + 1) ( x -2) = 60
60−2x
(x + 1) ( ) = 60
x
(x + 1) (60 - 2x) = 60
60x - 2x2 + 90 - 2x = 60x
-2x2 - 2x + 60 = 0
-2(x2 + x - 30) = 0
x2 + x - 30 = 0
x2 + 6x - 5x - 30 = 0
x(x + 6) -5 (x + 6) = 0
(x - 5) (x + 6) = 0
Either x - 5 = 0 or x+6=0
x=5 , x = -6 (neglect -ve high)
Put x = 5 in (iii)
60
y= 2
y = 12

Question.99 Rs. 1870 is divided into three parts in such a way that half of the first part,
one third of second part, and one sixth of the third part are equal. Find out
the value of each part.

A. B. C. D.

Solution
Given
Total amount = 1870
Let First part = x
2nd part = y
3rd part = x
Than According to statement
x + y + z = 1870 ----- (i)
Also
1 1 1
x=3y=6z --------- (ii)
2
From (ii)
Compare
1 1 1 1
x=3y x=6z
2 2
3x 6x
y= z= 2
2
z = 3x
Putting values in (i)
3x
x+= + 3x = 1870
2

56 | P a g e
2x+3x+6x
= 1870
2
11x = 3740
First part = x = 340
Then
3x 3
2nd part = y = 2 = 2 (340) = 510
3nd part = x = 3x = 3(340) = 1020

Question.100 A well-known publishing house agreed to sell a stated number of copies of


their book for Rs. 4500. Had they sold 5 copies less for the same amount, they
have received Rs. 30 more for each copy. Find the number of copies sold.

A. B. C. D.

Solution
Let
The number of copies = x
The price of each copies = y
According to statement
xy = 4500
4500
y= (i)
x
Also
(x – 5) (y + 30) = 4500
xy + 30x - 5y - 150 = 4500
xy + 30x - 5y = 4500 + 150
xy + 30x - 5y = 4650 (ii)
putting value of ‘y’ in (ii)
4500 4500
x( ) + 30x - 5( ) = 4650
x x
22500
4500 + 30x - = 4650
x
22500
30x - = 150
x
30x2 - 22500 = 150x
30x2 – 150x - 22500 = 0
30(x2 - 5x - 750) = 0
x2 - 5x - 750 = 0

Here
a = 1, b = -5 , c = -750
using
−b+ √b2 −4ac
x= 2a

57 | P a g e
√(−5)2 −4 x 1 x (−750)
x = (−(−5) ± )
2(1)

5 ±√25+ 3000
x= 2
5 ± √3025
x= 2
5 ± 55
x= 2
5 + 55 5− 55
x= ,
2 2
60 −50
x= ,
2 2
x = 30 , -25 (x cannot negative)
Put in (i)
4500
y= = 150
30

Question.101 The charges of a hired car are Rs. 8 per km for the first 60 km, Rs. 6 per km
for the next 60 km and Rs. 4 per km for any further Journey. If the balance
amount left over with Mr. Rashid is ¼ of what he paid towards the charges of
the hired car for traveling 450 km, How much money did he have initially
with him.

A. B. C. D.

Solution
Total Journey = 450
Total charges
= 8(60) + 6(60) +4 (450 - 120)
= 480 + 360 + 1320
Total charges = 2160
Balance amount = ¼ (total charges)
= ¼ (2160)
= 540
Hence
Total amount = 2160 + 540
= 2700

Question.102 Iqbal bought a certain number of pens for Rs. 60. Had he got 5 pens more for
the same price, each pen would have cost him one rupees less. Find the
number of pens purchased and the price of each pen.

58 | P a g e
A. B. C. D.

Solution
Let
Required no of pens = x
Price of each pen = y
Than
According to statement
xy = 60 ------- (i)
(x + 5) (y – 1) = 60 -------- (ii)

From (i)
xy = 60
y = 60/x (iii)
Putting value of y in (ii)
60
(x + 5) ( x − 1) = 60
300
60 - x + -5 = 60
x
300
-x + =5
x
−x2 + 300
=5
x
2
-x + 300 = 5x
x2 + 5x – 300 = 0
x2 + 20x – 15x - 300 = 0
x(x + 20) -15(x + 20) = 0
(x - 15) (x + 20) = 0
Either x – 15 = 0 or x + 20 = 0
Number of pens = x = 15 x = -20
(neglect -ve value)
Putting value of ‘x’ in (iii)
60
y = 15
Price of pen = y = 4

Question.103 A shop keep bought a certain number of exercise books at Rs. 15 and others
Rs. 17 each. Had he bought half as many at Rs. 15 and twice as many at Rs. 17, his
bill would have been Rs. 1960 instead of Rs. 1880 which he actually paid. Find the
number of exercise books he bought altogether.

A. B. C. D.

Solution
Let

59 | P a g e
Type I exercise books = x
Type II exercise books = y

Than
According to statement
15x + 17y = 1880 (i)
15
x + 34y = 1960 (ii)
2
(i) -2 (ii)
15x + 17y = 1880
-15x - 68y = -3920
-51y = -2040
nd
Type 2 exercise book = y = 40
Putting value of ‘y = 40’ in (i)
15x + 17(40) = 1880
15x + 680 = 1880
15x = 1880 – 680
15x = 1200
x = 1200/15
Type I exercise books x = 80

Question.104 A carpenter can make a cabinet in 30 hours. If he works for 21 hours at the
normal rate of pay and 9 hours at the overtime rate, his working time will
cost 69 rupees. But if he works for 27 hours of normal rate and 3 hours at the
overtime rate his working time will cost 63 rupees. Find his normal and over
time rate of pay.

A. B. C. D.

Solution
Let
Normal rate of pay = x
Overtime rate of pay = y
Than According to statement
21x + 9y = 69 (i)
and 27x + 3y = 63 (ii)
(i) -3 (ii)
21x + 9y = 69
-81x - 9y = -189
-60x = -120
Normal rate of pay = x = 2

60 | P a g e
Putting value of ‘x = 2’ in (i)
21(2) + 9y = 69
42 + 9y = 69
9y = 69 - 42
9y = 27
Overtime pay = y = 3

Question.105 A person bought 2 kilogram of onion and 10 kilograms of tomatoes for 24


rupees, another person spends 27 rupees for 1 kilogram of onion and 20
kilograms of tomatoes. Calculate the price for each kilogram of onion and
tomatoes.

A. B. C. D.

Solution
Let
Price of onion = x
Price of onion = y
Then according to statement
2x + 10y = 24 (i)
x + 20y = 27 (ii)
(i) -2 (ii)
2x + 10y = 24
-2x - 40y =- 54
-30y = -30
Price of Onion = y = 1
Putting value of ‘y = 1’ in (i)
2x + 10(1) = 24
2x + 10 = 24
2x = 24 – 10
2x = 14
Price of Tomatoes = x = 7

Question.106 Find the whole number such that twice of its square is 11 more than 21 time
of the original number.

A. B. C. D.

Solution
Let
The required number = x
According to statement
61 | P a g e
2x2 = 11 + 21x
2x2 -21x - 11 = 0
2x2 -21x + x - 11 = 0
2x (x – 11) +1 (x – 11) = 0
(2x + 1) (x - 11) = 0

Either 2x + 1 = 0 or x – 11 = 0
2x = -1 x = 11
x = -1/2

Question.107 An investor receives a total of Rs. 5700 per annum in interest from 3 stocks
yielding 4%, 5% and 8% per annum respectively. The amount at 4% is 20000 more than
the amount invested at 5% and the interest from 8% investment is 8 times the interest from
the 5% investment. How much money is invested in each stock?

A. B. C. D.

Solution
Let
Investment at 5% = x
Investment at 4% = x + 20,000
Also
I1= Interest earned from amount Invested at 5% = 0.05x
I2=Interest earned from amount Invested at 4% = 0.04 (x + 20,000) = 0.04x + 800
I3 = Interest earned at 8% = 8(0.05x) = 0.40x

Total interest earned


0.05x + 0.04x + 800 + 0.40x = 5700
0.49x = 4900
x = Rs. 10,000
Hence amount invested
At 5% rate = Rs. 10,000
At 4% rate = Rs. x + 20,000 = 10,000 + 20,000
= Rs. 30,000
At 8% rate
I3= Prt
0.40x =P x 0.08 x 1
0.40(10000) = 0.08 P
P =50,000

62 | P a g e
Type:5 (Distance)

Question.108 Two cars start from the same place and at the same time but travel in
opposite direction. After 6 hours, they are 540 meters apart. If one car travels 10 meter per
hour slower than the other. What is their speed per hour?

A. B. C. D.

Solution
Let
Speed of first car = x
Speed of first car = y
According to statement
6x + 6y = 540
6(x + y) = 540
x + y = 90 ----- (i)
and one car is slower than other
then
x = y - 10 ------ (ii)
Putting value of ‘x’ in (i)
y - 10 + y = 90
2y – 10 = 90
2y = 90 + 10
2y = 100
Speed of 2nd car = y = 50
Putting value of ‘y’ in (ii)
x = 50 - 10
Speed of 1st car = x = 40

Question.109 In six hour A walks 10 kilometers more than B does in five hours and in 8
hours B walks seven kilometer more than A does in 5 hours. What are the
speeds of each person?

A. B. C. D.

Solution
Let
Speed of car A = x
Speed of car B = y

Then according to statement


6x - 10 = 5y
6x - 5y = 10 (i)

63 | P a g e
And 8y - 7 = 5x
5x - 8y = -7 (ii)
5(i) - 6(ii)
30x - 25y = 50
30x + 48y =+42
23y = 92
Speed of car is = y = 4
Putting value of ‘y = 4’ in (i)
6x - 5(4) = 10
6x - 20 = 10
6x = 10 + 20
6x = 30
Speed of car A = x = 5

Question.110 Two cars that are 600 km apart are moving towards each other. Their speed
𝟏
differ by 6km per hour and cars are 123 km apart after 4𝟐 hours. Find speed
of each car.

A. B. C. D.

Solution
Let Speed of 1st car = x
Speed of 2nd car = y
Than According to statement
(difference) x - y = 6 ----- (i)
9 9
And x + 2 y = 600 - 123
2
9 9
x + 2 y = 477
2
9
(x + y) = 477
2
2
x + y = 477 x 9
x + y = 106------ (ii)
(i) + (ii)
x-y=6
x + y = 106
2x = 112
x = 56
Putting value of ‘x = 56’ in (i)
56 - y = 6
56 - 6 = y
50 = y

64 | P a g e
EXTRA PRACTICE QUESTIONS

65 | P a g e
Question.1 The condition for which the system of equation kx –y =2 and 6x-2y =3 has a
unique solution is
Solution
Given
Kx – y = 2
6x -2y = 3
Are has a unique solution if
𝑎 𝑏
𝑐
≠𝑑
Given
a = k , b = -1, c =6, d = -2
So
𝑘 −1
6
≠ −2
K≠3

Question: 2 The value of k for which the system of equation 2x + 3y = 5 and 4x + ky = 10 has an
infinite number of solution is.
Solution:
Consider the equation
ax + by = c and dx + ey = f
for infinite solution
𝑎 𝑐
=
𝑑 𝑒
2 3
=
4 𝑘
K=6

Question: 3 (𝒙 − 𝒚)𝟑 + (𝒚 − 𝒛)𝟑 + (𝒛 − 𝒙)𝟑


Solution:
= [𝑥 3 − 𝑦 3 − 3𝑥𝑦 (𝑥 − 𝑦)] + [𝑦 3 − 𝑧 3 − 3𝑦𝑧 (𝑦 − 𝑧)] + [𝑧 3 − 𝑥 3 − 3𝑧𝑥 (𝑧 − 𝑥)]
= 𝑥 3 − 𝑦 3 − 3𝑥𝑦 (𝑥 − 𝑦) + 𝑦 3 − 𝑧 3 − 3𝑦𝑧 (𝑦 − 𝑧) + 𝑧 3 − 𝑥 3 − 3𝑧𝑥 (𝑧 − 𝑥)
= 3xy (x-y) – 3yz (y-z) – 3zx (z -x)
= 3𝑥 2 𝑦 − 3𝑥𝑦 2 − 3𝑦 2 𝑧 + 3𝑦𝑧 2 − 3𝑧 2 𝑥 + 3𝑧𝑥 2
= 3 (𝑥 2 𝑦 − 𝑥𝑦 2 − 𝑦 2 𝑧 + 𝑦𝑧 2 − 𝑧 2 𝑥 + 𝑧𝑥 2 )
= 3(x-y) (y-z) (z -x)

Question: 4 𝑥 3 − 1 + 𝑦 3 + 3𝑥𝑦

Solution:
𝑥 3 − 1 + 𝑦 3 + 3𝑥𝑦
= (𝑥)3 − (1)3 + (𝑦)3 − 3(𝑥)(−1)(𝑦)
= (x -1 + y) (𝑥 2 + 1 + 𝑦 2 + 𝑥 + 𝑦 − 𝑥𝑦)

66 | P a g e
Formula:
(𝑎 + 𝑏)2 = 𝑎2 + 𝑏 2 + 2𝑎𝑏
(𝑎 − 𝑏)2 = 𝑎2 + 𝑏 2 − 2𝑎𝑏
(𝑎 + 𝑏)3 = 𝑎3 + 𝑏 3 + 3𝑎𝑏(𝑎 + 𝑏)
(𝑎 − 𝑏)3 = 𝑎3 − 𝑏 3 − 3𝑎𝑏 (𝑎 − 𝑏)
𝑎3 + 𝑏 3 = (𝑎 + 𝑏)(𝑎2 − 𝑎𝑏 + 𝑏 2 )
𝑎3 − 𝑏 3 = (𝑎 − 𝑏)(𝑎2 + 𝑎𝑏 + 𝑏 2 )
𝑎2 − 𝑏 2 = (𝑎 + 𝑏)(𝑎 − 𝑏)

Question: 5 𝒙𝟑 − 𝒙𝟐 𝒚 - x𝒚𝟐 + 𝒚𝟑

Solution:
𝑥 3 + 𝑦 3 − 𝑥 2 𝑦 - x𝑦 2
= (𝑥)3 + (𝑦)3 − 𝑥𝑦 (𝑥 + 𝑦)
= (x+y) (𝑥 2 − 𝑥𝑦 + 𝑦 2) – xy (x+y)
=x+y (𝑥 2 − 𝑥𝑦 + 𝑦 2 − 𝑥𝑦)
= (x+ y)(𝑥 2 − 2𝑥𝑦 + 𝑦 2 )
= (x + y) (𝑥 − 𝑦)2

Question: 6 𝟖𝒙𝟑 − 𝟐𝟕𝒚𝟑


Solution:
= (2𝑥)3 − (3𝑦)3
= (2x – 3y) (4𝑥 2 + 6𝑥𝑦 + 9𝑦 2 )

Question: 7 (𝒙𝟐 + 𝟐𝒙 + 𝟐) (𝒙𝟐 − 𝟐𝒙 + 𝟐)

Solution:
= (𝑥 2 + 2 + 2𝑥) (𝑥 2 + 2 − 2𝑥)
= [(𝑥 2 + 2) + 2𝑥] [(𝑥 2 + 2) − 2𝑥]
= (𝑥 2 + 2)2 − (2𝑥)2
= (𝑥 4 + 4 + 2 ∗ 𝑥 2 ∗ 2) - 4𝑥 2 = 𝑥 4 + 4

Question: 8 A purse contain only 25 paise and 10 paise coins. The total amount in the purse is
Rs.8.25. if the number of 25 paise coins is one-third of the number of 10 paise coins
in the purse then total number of coins in the purse is.

Solution:
Let
Number of 10 paise coins = x
𝑥
Number of 25 paise coins = 3
Then
𝑥
10x + 25 * = 825 paise
3

67 | P a g e
30𝑥+25𝑥
3
= 825
55x = 825 * 3
X = 45
No of 10p coins = 45
No of 25 coins = 15

Question: 9 A man have some hens and cows. If the numbers of heads be 48 and number of feet
equal to 140 the number of hens will be

Solution:
Let
Cows = x
Hens = y
Then
(Heads), x + y = 48 ___ (i)
(Legs) 4x +2y = 140 ___ (ii)
Using calculator
X = 22, y = 26

Question: 10 There are two examination hall p and q. if ten students are sent from p and q then
number of students in each room is same. If 20 students are sent from q to p then
number of students in p is double of that in Q. the number of students in p and q
respectively is.
Solution:
Let the number of students in examination halls p and Q is x and y respectively.
First condition
x – 10 = y + 10
x - Y = 20 ___ (i)
2nd condition
X + 20 = 2 (y -20)
X + 20 = 2y -40
X – 2y = -60 ___ (ii)
Use calculator
Y = 80, x = 100

Question: 11 Points A and B are 60 km apart. A bus starts from A and another from B at the
same time. If they go in the same direction, they meet in 6 hours and if they go in
opposite direction. They meet in 2 hours. The speed of the bus with greater speed is.

Solution:
Let the faster bus travels at x km/h and slower at y km/h according to statement
6 (x-y) = 60
X – y = 60 __ (i)

68 | P a g e
2 (x +y) = 60
X + y = 30 ___ (ii)
Using calculator
X = 20, y = 10

Question: 12 Rs.49 was divided among 150 children. Each girl got 50 paise and a boy 25 paise.
How many boys were there?
Solution:
Let
Number of girls = x
Number of boys = y
Then
Firsts condition x + y = 150 ___ (i)
(Quantity)
2nd condition 50x + 25 y = 4900 (49 rupees)
(Cash)
Using calculator
X = 46, y = 104

Question: 13 In covering a distance of 30km, ahmad takes two hours more than shahid. If ahmad
doubles his speed, he would take 1 hour less than shahid. Ahmad’s speed is.
Solution:
let the speed of A be x and that of B be y.
So
30 30
𝑥
− 𝑦
= 2 __ (i)
Also
30 30
− = −1 __ (ii)
2𝑥 𝑦
Using calculator
x = 5km/h and y = 7.5 km/h

Question: 14 The area of a rectangle gets reduced by 9𝒎𝟐 . If its length is reduced by 5m and
breadth increased by 3m. if we increased the length by 3m and breath by 2m. the
area is increased by 67𝒎𝟐 . The length of the rectangle is.
Solution:
Let
length of rectangle = x
Width of rectangle = y
Area of rectangle = xy
According to statement
(x - 5) (y + 3) = xy – 9
Xy + 3x – 5y – 15 = xy – 9
3x -5y = 6 __ (i)
Also

69 | P a g e
(x+3)(y+2) = xy + 67
Xy + 2x + 3y + 6 = xy + 67
2x + 3y = 61 __ (ii)
Using calculator
x = 17, and y = 9

Question: 15 A two-digit number is obtained either by multiplying the sum of the digits by 8 and
adding 1 or by multiplying the difference of digits by 13 and adding 2. The number
is.
Solution:
Let
x = tenth digit
y = unit digit
Also
8 (x + y) + 1 = 10x + y
8x + 8y + 1 = 10x + y
2x – 7y = 1 ___ (i)
Also
13 (x-y) + 2 = 10x + y
-3x + 14y = 2
Using calculator
X = 4 and y = 1

Question: 16 How many gallons of a 15% acid solution must be mixed with 5 gallons of a 20%
acid solution to make a 16% acid solution?
Solution:
0.15x + 0.2 * 5 = 0.16 (x+5)
0.01x = 0.2
x = 20

Question: 17 Find the number if 7 is subtracted from number and result multiplied by 12 giving a
result 6 times the number.
Solution:
Let the required number = x
(a - 7). 12 = 6a
12a – 84 = 6a
6a = 84
a = 14

Question: 18 A tank can be filled by a big pump in 20 hours and by a small pump in 80 hours,
how long will it take to fill the tank by both pump together.
Solution:
1 1
(20 + 80
) ∗ 𝑡𝑖𝑚𝑒 = 1
(0.05 + 0.0125) * time = 1

70 | P a g e
(0.0325) * time = 1
Time = 16hours

Question: 19 In a charity show tickets were sold at Rs.180 for 1st class, at Rs.120 for second class
and Rs.60 for the general enclosure. The no. of seats in general enclosure was double of that in the
first class and number of seats in second class was triple of that in first class. If all seats were
occupied expect 20 in first class and show fetches Rs.62,400 find no. of persons who attend the show

Solution:
Let x be the number of seats
Seats for first class = x
Seats for 2nd class = 3x
Seats for 3rd class = 2x

180 (x-2) + 120 (3x) + 60 (2x) = 62400


660x = 62400
X = 100
Total number of persons
= x -20 + 3x + 2x
= 100- 20 + 3 * 100 + 2 * 100
= 80 + 300 + 200
= 580

Question: 20 Seven years back age of father was 5 time the age of his son it will be twice after 14
years. Find father age at present.
Solution:
Let
age of father = x
Son = y
7 years back
X -7 = 5 (y -7)
X – 5 = 5y -35
X -5y = -30 ___ (i)
After 14 years
x + 14 = 2 (y+14)
x+14 = 2y + 28
x – 2y = 14 ___ (ii)
Using calculator
x =, y =

Question: 21 A railway half tickets costs half the full fare but reservation charges is the same on
half tickets as on full tickets. One reversed first class tickets is Rs.362 and one full
and one half reversed forts class tickets cost Rs.554. reservation charges is.

71 | P a g e
Solution:
Let
The reservation charges = x
First class charges for full ticket = y
Then
x + y = 362 ___ (i)
Also
1
1 + 2 𝑦 + 2x = 554
3𝑦
+ 2𝑥 = 554
2
3y + 4x = 1108
Using calculator, we get
y = 340
x = 362 – 340 = 22

Question: 22 Perimeter of a rectangle is 82m and its area is 400. Breadth of the rectangle is.

Solution:
Perimeter = 2(L + B)
82 = L+ B
L + B = 41 __ (i)
Area = 400
LB = 400 ___ (ii)
Putting value of (ii)
400
B= 𝐿
in (i)
400
L+ 𝐿
= 41
𝐿2 + 400 = 41 L
𝐿2 + 400 – 41 L= 0
Using calculator
L = 25 or 16
B = 26 or 9

Question: 23 40kg broken rice at the rate of Rs.5 per kg less than the fine quality rice is mixed in
100 kg rice of fine quality. Price of mixture is Rs. 90 per kg. find the price per kg of
fine quality.
Solution:
Total price = 40+ 100 = 140
Mixture – broken rice = fine quality Rice
140 (90) – 40 (x-5) = 100x
12600 – 40x + 200 =100x
12800 = 140x
X = 91.43

72 | P a g e
Question: 24 Arslan has only Rs.1 and Re. 2 coins with him. If the total number of coins that he
has is 50 and the amount of money with him is Rs.75, then the number of Re.1 and
Re.2 coins is respectively.
Solution:
Let
Re.2 number of coins = x
Re. 1 number of coins = 50 – x
Then
1 (50 - x) + 2x = 75
50 – X + 2x = 75
Re. coins = x = 25
Re. coins = 50 –x = 2 5

Question: 25 How many gallons of 45% acid solution and how many gallons of 15% acid solution
shall be mixed to obtain 60 gallons of a 25% solution.

Solution:
Let first solution = x
2nd solution = y
X +y = 60 __(i)
Also
0.45 x + 0.15 y = 25% * 60
0.45x + 0.15 y = 1 5 __ (ii)
Using calculator, we get
x = 40 and 20

Question: 26 An airline wishes to purchase the following cue crafts to meet an estimated demand
for 2000 seats.
Company Aircraft Price of each (million) Seating capacity
Boing 747 150 400
Boeing 777 115 300
Airbus A 320 60 200

The airline wishes to buy equal number of 747‘s and 777’s. How many should it order to meet the
demand for seats within a purchasing budget of & 710 million?

Solution:
Let number of Boeing 747 & 777 = x
Number of airbus = y
Then
150x + 115x + 60y = 710
265x + 60y = 710 ___ (i)
Number of seats
400x +300x + 200y = 2000

73 | P a g e
700x + 200y = 2000 __ (ii)
Solving equation (i) (ii)
Using calculator
X=2
Y=3

Question.27 Ahmed’s total investment in bonds is Rs 50,000. He made two investments.


The first one four years before at 8% and the 2nd two years before at 10%. If his
income through these investments this year is Rs 4600.what was the amount of each
investment?

Solution:

Rs 20,000 at 8% and Rs 30,000 at 10%

x+y =50,000 --------------------(i)

0.08x +0.10y =4600 --------------(ii)

Solve (i) & (ii)

x=20,000 and y=30000

Question.28 Two square have sides p can and (p + 5) can.

The sum of their square is 625sq. the sides of the square are

Solution:
𝑝2 + (𝑝 + 5)2 = 625

𝑝2 + 𝑝2 + 10𝑝 + 25 = 625

2𝑝2 + 10𝑝 = 600

2𝑝2 + 10𝑝 − 600 = 0

(𝑝 + 20)(𝑝 − 15) = 0

Hence 𝑝 = −20, 𝑝 = 15

Neglect (-ve) value 𝑝 = 15

Question.29 The sum of ages of 3 persons is 150 years. Ten years ago their ages were in
the ratio 7 : 8 : 9. What are their present age.

74 | P a g e
Solution:
𝑥 + 𝑦 + 𝑧 = 150

10 year ago sum of their ages are

= 150 – (10 + 10 +10)

= 120

Ten years ago:

Ten year ago they are in ratio 7:8:9

Sum of ratio = 24
7
Age of 1st = 24 × 120 = 35

8
Age of 2nd = 24 × 120 = 40

9
Age of 3rd = 24 × 120 = 45

Present age

1st person = 35 + 10 = 45

2nd person = 40 + 10 = 50

3rd person = 45 + 10 = 55

Question.30 If three of A’s age 6 year ago be subtracted from twice his present age. The result
would be equal to his present age. Find A’s present age.

Solution:
2𝑥 − 3(𝑥 − 6) = 𝑥

2𝑥 − 3𝑥 + 18 = 𝑥

18 = 2𝑥

𝑥=9

Question.31 if roots of equation

x2-5x +9 are equal, then α is

75 | P a g e
Solution:
b2 -4ac =0
25-4(1)(α)=0
25=4α
α=25/4

Question.32 The distance between two is 380km. at the same moment, a passenger car and a
truck start moving towards each other from different town. They meet 4 hours later. If
the car drives 5 km the faster than the truck. What are their speed.

Solution:
let the speed of a truck = 𝑥

the speed a car = 𝑥 + 5

then 𝐷⁄𝑇 = 𝑠𝑝𝑒𝑒𝑑

380
=4
𝑥+𝑥+5
380
=4
2𝑥 + 5
380 = 8𝑥 + 20

𝑥 = 45

𝑥 + 5 = 50

Question.33 A car owner covers 15 kilometers to a liter of petrol and 600 kilometers to a
liter of motor oil. He estimates that an annual distance of 6,000 kilometers would cost him
Rs. 5030 in petrol and motor oil. In fact, he used twice as much motor oil as he estimated
and the GST was 5340. Find the cost of a liter of petrol.

Solution:
Cost of motor oil = 𝑥

Cost of petrol = 𝑦

Estimated quantity on 6,000 kilometers


6,000
Petrol = = 400 𝑙𝑖𝑡𝑒𝑟.
15

6,000
Motor oil = = 10 𝑙𝑖𝑡𝑒𝑟.
600

76 | P a g e
So, 10𝑥 + 400𝑦 = 5030…………… (i)

20𝑥 + 400𝑦 = 5340………………… (ii)

Hence 𝑥 = 31 , 𝑦 = 11.8

Question.34 a shop sells bicycles and tricycles.in total there are 7 cycles(cycles both
included bicycle an tricycles) and 19 wheels. Determine how many of each there are ,if
bicycle has two wheels and a tricycle has three wheels

Solution:
x +y 7

2x +3y =19

x =2,y 5

Question.35 a father is three times more than his son zahid. After 8 years, he would be two
and a half times of zahid’s age. After further 8 years, how many times would he be of
zahid’s age?

Solution:
Zahid’s age =x

Father age =3x

After 8 years

3x +8 =5/2(x+8)

x=24

3x =72

After 8 more year= x=32

After 8 year more year =3x=80

Question: 36 The equation ax + b = 0 and cx + d = 0 are consistent if.


Solution:
ad = b c

Question:37

Two square have sides pcm and (p+5)cm. the sum of their square is 625 Sq. the sides of the
square are.

Solution:
77 | P a g e
𝑝2 + (𝑝 + 5)2 = 625

𝑝2 + 𝑝2 + 10𝑝 + 25 = 625

2𝑝2 = 10𝑝 = 600

2𝑝2 + 10𝑝 − 600 = 0

(𝑝 + 20)(𝑝 − 15) = 0

Hence

𝑃 = −20, 𝑝 = 15

(Neglect - ve)

So

𝑝 = 15

Question:38

The sum of ages of 3 persons is 150 (2 years. Ten year ago their ages were in the
ratio 7:8:9.

What are their present age?

Solution:
𝑥 + 𝑦 + 𝑧 = 150

10 YEARS AGO SUM OF THEIR AGES ARE

= 150 − (10 + 10 + 10)

= 120

Ten years ago sum of their ages are in ratio

7:8:9

Sum of the ratio = 24


7
Age of 1st = 24 × 120 = 35

8
Age of 2nd = 24 × 120 = 40

78 | P a g e
9
Age of 3rd = 24 × 120 = 45

Present age 1st person = 35 + 10 = 45

Present age 2nd person = 40 + 10 = 50

Present age 3rd person = 45 + 10 = 55

Question:39

If trice of A’s age 6 year ago be subtracted from twice his present age. The result would be
equal to his present age. Find A’s present age.

Solution:
2𝑥 − 3(𝑥 − 6) = 𝑥

2𝑥 − 3𝑥 + 18 = 𝑥

18 = 2𝑥

𝑥=9

Question:40

The area of a plot of land is 126 square meters. If the breadth of the plot is 5 meters less
than the length. What will be its length and breadth?

Solution:
let the require length and breadth is

𝑥 𝑎𝑛𝑑 (𝑥 − 5)

Then

𝐴𝑟𝑒𝑎 = 𝑥(𝑥 − 5)

126 = 𝑥 2 − 5𝑥

𝑥 2 − 5𝑥 − 126 = 0

𝑥 2 − 14𝑥 + 9𝑥 − 126 = 0

𝑥 = −9, 14 (𝑁𝑒𝑔𝑙𝑒𝑐𝑡 − 𝑣𝑒)

Hence

𝑥 = 14
79 | P a g e
𝑥−5=9

Question:41

A car owner covers 15 kilometers to a liter of petrol and 600 kilometers to a liter of motor
oil. He estimated that an annual distance of 6,000 kilometer would cost him Rs. 50 30 in
petrol and motor oil. In fact, he used twice as much motor oil as he estimated and cost was
5340. Find the cost of a liter of petrol.

Solution:
Cost of motor oil = 𝑥

Cost of petrol = 𝑦

Estimated quantity on 6,000 kilometers


6,000
Petrol = = 400 𝑙𝑖𝑡𝑒𝑟𝑠
15

6000
Motor oil = = 10 𝑙𝑖𝑡𝑒𝑟
600

So,

10𝑥 + 400𝑦 = 5030 … … … … . . (𝑖)

20𝑥 + 400𝑦 = 5340 … … … … … . . (𝑖𝑖)

Hence

𝑥 = 31, 𝑦 = 11.8

Question:42

The distance between two towns is 380 km. at the same moment a passenger car and truck
start moving towards each other from different town. They meet 4 hour later. If the car
drives 5 km/hr faster than the truck. What are their speed.

Solution:
let the speed of truck = 𝑥

let the speed of car = 𝑥 + 5

then
𝐷
= 𝑠𝑝𝑒𝑒𝑑
𝑇

380
=4
𝑥+𝑥+5

80 | P a g e
380
=4
2𝑥+5

380 = 8𝑥 + 20

𝑥 = 45

𝑥 + 5 = 50

Question:43

Find the value of 𝒙

(𝒙 − 𝟐)(𝒙 − 𝟒)(𝒙 − 𝟔)(𝒙 − 𝟖) = 𝟗

Solution:
(𝑥 − 2)(𝑥 − 8)(𝑥 − 4)(𝑥 − 6) = 9

(𝑥 2 − 10𝑥 + 16)(𝑥 2 − 10𝑥 + 24) = 9

Let 𝑥 2 − 10𝑥 = 𝑦

Then

(𝑦 + 16)(𝑦 + 24) = 9

𝑦 2 + 40𝑦 + 384 = 9

𝑦 2 + 40𝑦 + 375 = 0

𝑦 = −15, −25

When When

𝑦 = 𝑥 2 − 10𝑥 𝑦 = 𝑥 2 − 10𝑥

Then 𝑦 = −15 then 𝑦 = −25

So So

𝑥 2 − 10𝑥 = −15 𝑥 2 − 10𝑥 = −256

𝑥 = 5 ± √10 𝑥=5

Question:44

If roots of equation

𝒙𝟐 − 𝟖𝒙 + 𝒎 = 𝟎

81 | P a g e
Solution:
Let roots of equation be

𝑥 𝑎𝑛𝑑 𝑥 + 4

Then
−(−8)
𝑥+𝑥+4= =8
1

2𝑥 + 5 = 8 … … … . . (𝑖
𝑐 𝑚
Also 𝑥(𝑥 + 4) = 𝑎 = 1

𝑥 2 + 4𝑥 = 𝑚______________(𝑖𝑖

From (i)

2𝑥 + 4 = 8

2𝑥 = 4

𝑥=2

Put in (𝑖𝑖)

(2)2 + 4(2) = 𝑚

4+8=𝑚

𝑚 = 12

Question:45

Divide 25 into two parts so that sum of their reciprocal is 𝟏⁄𝟔

Solution:
let the required number are 𝑥, 𝑎𝑛𝑑 25 − 𝑥

Then
1 𝑥 1
+ 25−𝑥 = 6
𝑥

25−𝑥+𝑥 1
=6
𝑥(25−𝑥)

25𝑥 − 𝑥 2 = 150

82 | P a g e
𝑥 2 − 25𝑥 + 150 = 0

𝑥 = 15, 10

Question:46

The fourth part of a number exceeds the sixth part by 4.

Solution:
According to statement
𝑥 𝑥+24
𝑥=
4 6

6𝑥 = 4𝑥 + 96

2𝑥 = 96

𝑥 = 48

Question:47

𝟐𝒙−𝟐 + 𝟐𝟑−𝒙 = 𝟑

Solution:

2𝑥 . 2−2 + 23 . 2−𝑥 = 3
2𝑥 8
+ 2𝑥 = 3
4

Let 2𝑥 = 𝑦
𝑦 8
+𝑦 =3
4

𝑦 2 + 32 = 12𝑦

𝑦 2 − 12𝑦 + 32 = 0

𝑦 2 − 8𝑦 − 4𝑦 + 32 = 0

(𝑦 − 8)(𝑦 − 4) = 0

𝑦 = 8, 4

When When

𝑦 = 2𝑥 𝑦 = 2𝑥

83 | P a g e
Then 𝑦 = 8 then 𝑦=4

So, 8 = 2𝑥 So, 4 = 2𝑥

23 = 2𝑥 22 = 2𝑥

𝑥=3 𝑥=2

Question:48

Given that a, b and c are successive terms in an arithmetic sequence. Calculate 𝒙 if

(𝒃 − 𝒄) 𝒙𝟐 + (𝒄 − 𝒂)𝒙 + (𝒂 − 𝒃) = 𝟎

Solution:
. Here 𝑎𝐼 = 𝑏 − 𝑐, 𝑏 𝐼 = 𝑐 − 𝑎, 𝑐 𝐼 = 𝑎 − 𝑏

−𝑏±√𝑏 2 −4𝑎𝑐
Using 𝑥 = 2𝑎

−(𝑐−𝑎)±√(𝑐−𝑎)2 −4(𝑏−𝑐)(𝑎−𝑏)
𝑥= 2𝑎

−𝑐+𝑎±√𝑐 2 +𝑎2 −2𝑎𝑐−4(𝑎𝑏−𝑏 2 −𝑎𝑐+𝑏𝑐)


𝑥= 2(𝑏−𝑐)

−𝑐+𝑎±√𝑎2 +4𝑏2 −𝑐 2 −4𝑎𝑏+2𝑎𝑐−4𝑏𝑐


𝑥=
2(𝑏−𝑐)

−𝑐+𝑎±√(𝑎−2𝑏+𝑐)2
𝑥= 2(𝑏−𝑐)

−𝑐+𝑎±(𝑎−2𝑏+𝑐) −𝑐+𝑎−𝑎+2𝑏−𝑐
𝑥= ,
2(𝑏−𝑐) 2(𝑏−𝑐)

2(𝑎−𝑏) 2(𝑏−𝑐)
= ,
2(𝑏−𝑐) 2(𝑏−𝑐)

𝑎−𝑏
𝑥= ,1
𝑏−𝑐

Note: Nature of roots

Discrimination = 𝒃𝟐 − 𝟒𝒂𝒄

Rules: 𝑖) 𝐷 > 0 𝑏 2 − 4𝑎𝑐 > 0 Real, distinct


Two distinct i) perfect square(Rational roots)
Real roots ii) not perfect square (irrational roots
ii) 𝐷=0 𝑏 2 − 4𝑎𝑐 = 0 Real, equal

84 | P a g e
two equal roots

iii) D < 0 𝑏 2 − 4𝑎𝑞𝑐 < 0 Complex, distinct


no real roots

Question :49

Factorize 4x4 + 81y4

a) (2x2 + 9y2 + 6xy) (2x2- 9y2 - 6xy)

Question :50

Factorize y=3x2-6x-9

a) (3x-9)(x+1) (3x-9)(x+1) c) (3x-9)(x-1) d) none

Question :51

Factorize x4 + 324y4

Question :52

Factorize y=3x2-9x+6

a) (3x-3)(x-2)

Question: 53

Find x, y, z

3x + 2y + 3y = 3

x + y + z = 12

–y–z=4
a) X = 10, y = 22, z = -29 b) X = 8, y = 33, z = -29
c) X=8, y = 44, z = 29 d) None of these

Question: 54

x+10 = 11x2 – x + 1 Find x

(a) x = -1 x = 0.818 b) x=1 x = -0.818

85 | P a g e
(c) x = -1 x =-0.818 d) x=1 x = 0.818

Question: 55

Sajid and Hamid have some amount to invest in such a way that 1 time of Hamid and 7 times of

Sajid’s investment is 9 million and thrice of Hamid and twice of Sajid is 4 million Find investment of

both respectively

a) 5 million & 5.05 million b) 4 million each

c) 0.52 million &1.21 ,million d) None of these

Question:56
Hamid and Majid invest in a project. If sum of 4 time of Hamid and 5 times of Majid is 32 million and
difference of 3 times of Majid and 2 times of Hamid is 6 million Find investment of both respectively

a) 3 million & 4 million. b) 1.05 million & 1 million

No
e) of
c) 570,000 & 1.25 million these

Question:57

x2+7x+12=0 find the value of x.

a) (3,4) b) (-3, 4)

c) (-3,-4) d) (3,-4)

Question: 58

Which of the following is not linear?

a) 2x+y=1 b) 2+√2 y =3

c) (x +1)2 = 2 d) All of these

86 | P a g e
Question: 59

The equation of a line parallel to y-axis and at a distance of 4 units to the right of y-axis, is:

a) X=4 b) Y=4

c) X=4Y d) Y=4X

Question :60 Find the value of x in equation

9(2x-3)= 5√3

a) 1.26 b) 1.35 √3 c) 1.25 d) 1.55

Question :61 Y= 3x2 +5x-3

Above equation have graph

a) Linear b) Symmetric (parabolic)

c) Both d) None

Question :62 (ICAP-Book Questions)

If the solution of a quadratic equation has two values 0 and 4, the equation is:

(a)x2 – 4 = 0 (b)x2 + 16 = 0
(c)x2 – 16 = 0 (d)x2 – 4x = 0
Question :63

The equation x2 + kx – 18 = 0, where k is a constant, is satisfied by x = 2. Then the value of k is


:

(a)-7 (b)5
(c)±7 (d)7
Question :64

Solution of the equation (x + 5)2 = 16 is:

(a)1 or 9 (b)-1 or -9
(c)-1 or 9 (d)1 or -9
Question :65

Given the 8x2 – 2xy – 3y2 = 0; then y in term of x is

87 | P a g e
(a)4x/3 and 2x (b) 4x/3 and -2x
(c)-4x/3 and -2x (d)4/3x and -2x
Question :66

A quadric function can have:

(a)Only one x-intercept (b)Two x- intercept


(c)No x-intercepts (d)All options are possible
Question :67

The profit function of a company is given a P (x) = -0.05x2 + 4x -30. What will be the profit of
the company if company sells 25 units?

(a)38.75 (b)48.75
(c)58.75 (d)68.75
Question :68

The profit function of a company is given as p(x) = -0.05x2 + 4x -30. What will be the profit of
the company if company is not able to sell any unit?

(a)-30 (b)0
(c)30 (d)Cannot be determined
Question :69

Which of the following statement is/are correct?

1. Quadric equations when plotted on a graph are always U shaped.

2. Quadric equations when plotted on a graph are always inverted U shaped.

3. Quadric equations when plotted on a graph are can either be U shaped or


inverted U shaped.

4. Quadric equations when plotted on a graph can never be inverted U shaped.

(a) 1 (b) 1 and 2


(c)3 (d)4
Question :70

A quadratic equation is one which takes the form:

(a)y= mx + c (b) y = ax2 + bx + c


(c)y = ax3 + bx2 + c (d) Any of the above
Question :71

Solve: (5x - 1) (x-4) =0

88 | P a g e
(a)-1/5 and 4 (b)-1/5 and -4
(c) 1/5 and 4 (d)4 only
Question :72

Expand the following equation:

(x+2) (x+3) = 0

(a)x+6 = 0 (b)x2 + 5x +6 = 0
(c)x2 + 3x + 4 = 0 (d)x2+6x = 0
Question :73

A quadratic expression is one in which the degree of variable is ____.

(a)One (b)Two
(c)Three (d)Four
Question :74

If the solution of the equation (Kx - 3) (Kx + 1) = 0 has two roots 3/2 and -1/2, the value of K is:

(a)3 (b)-1
(c)2 (d)-2
Question :75

Roots of the equation (ax + b) (cx + d) = 0 are:

(a)-b/a and -d/c (b)-a/b and -c/d


(c)a/b and c/d (d)b and d
Question :76

For the equation Kx2+2K2x+K=0. The possible value of K and K2 are:

(a)1 and 3 (b)2 and 3


(c)2 and 5 (d)2 and 4
Question :77

By comparing 5x2+ 3x-6 = 0 with ax2 + bx+ c = 0 the value of a, b and c are

(a)5, 3 and -6 (b)3, 5 and 6


(c)5, 3 and 6 (d)-5, -3 and -6
Question :78

By comparing -5x2 -3x-6 = 0 with ax2 + bx + c = 0 the values of a, b and c are:

(a)-5, -3 and -6 (b)-5 3 and 6


(c)5, 3 and 6 (d)-5, 3 and -6

89 | P a g e
Question :79

Number of real roots for the quadratic equation 3x2 + 2x + 1 = 0 are:

(a)one (b)Two
(c)Three (d)None
Question :80

Number of real roots for the quadratic equation -3x2 + 2x + 1 = 0 are:

(a)Two (b)one
(c)Three (d)None
Question :81

Number of real roots for the quadratic equation x2 + 2x + 1 = 0 are:

(a)One (b)Two
(c)Three (d)Four
Question :82

Which of the following statement is/are correct?

1. A quadratic equation will always have two equal roots.

2. A quadratic equation will always have two roots.

3. A quadratic equation will always have one roots.

4. A quadratic equation will either have one or two roots.

(a) 1 (b) 1 and 2


(c)All (d)None
Question :83

which of the following statement is/are correct:

1. The roots of a quadratic equation can never be negative.

2. The roots of a quadratic equation can never be positive.

3. One of the roots of a quadratic equation is always zero.

4. A quadratic equation can have no real roots.

(a) 1 (b) 2
(c) 3 (d) 4

90 | P a g e
Question :84

Solve x2 + 7x + 12 = 0

(a)-3 and 4 (b)3 and -4


(c)-3 and -4 (d)3 and 4

Question :85

Which of the following statement is/are correct?

1. Graph of a quadratic equation will always intersect x-axis.

2. Graph of a quadratic equation may intersect x-axis.

3. Graph of a quadratic equation will always have two x-intercepts.

4. Graph of a quadratic equation will never have two x-intercepts.

(a)1 (b)2
(c)4 (d)3 and 4
Question :86

Which of the following statement is/are correct?

1. Graph of a quadratic equation will always have a negative slope.

2. Graph of a quadratic equation never intersect y-axis.

3. Graph of a quadratic equation will always be U shaped.

4. Graph of a quadratic equation may have two x-intercepts

(a) 4 (b)3
(c)2 (d)1

91 | P a g e
Note: Quadratic equation graph is u shaped but not always u shaped

92 | P a g e
Solution of ICAP book

Solution:

Put 0 and 4 in

Put x = 0 Put x = 4 in

X2 – 4x =0 X 2 - 4x = 0

0=0 16 – 16

L.H.S = R.H.S 0=0

L.H.S=R.H.S

Solution:

Given:

X 2 + Kx – 18 = 0

Put x = 2

4 + 2x – 18 = 0

2x = 14

X=7

Solution:

Given

(x + 5) 2 = 16

X+5=±4

X+5=4 , x + 5 = -4

X = -1 , X = -9

Solution set = {(−1 , −9)}

93 | P a g e
Solution:

Given

8x2 – 2xy – 3y2 = 0

8x 2 – 6xy + 4xy – 3y 2 = 0

2x (4x -3y) +y (4x – 3y) = 0

(2x + y) (4x – 3y) = 0


4𝑥
Y = -2x , y = 3

Solution:

All option are possible

Solution:

Given

P(x) = - 0.05x2 + 4x – 30

Put x = 25

P (25) = -0.05 (25)2 + 4 (25) – 30

P (25) = 38.75

Solution:

Given

P (x) = -0.05x 2 + 4x – 30

Put x = 0

P (0) = - 30

Solution:

Quadratic equations when plotted on a graph can either be U shaped or inverted U shaped

Solution:

Given:

(5x - 1) (x - 4) = 0

94 | P a g e
5x – 1 = 0, x – 4 = 0

5x = 1

X = 1/5 , x=4

Solution:

Given:

(x + 2) (x + 3) = 0

x2 + 5x +6 = 0

Solution:

Given:

(Kx – 3) (Kx + 1) = 0

Kx – 3 = 0 Kx + 1 = 0

Kx= 3 Kx = -1
3 −1
X= X=
𝑘 𝑘

compare with Compare with

x= 3/2 X = -1/2

Hence k=2

Solution:

Given (ax + b) (cx + d) = 0

ax= -b , cx= -d

−𝑏 −𝑑
x= , x=
𝑎 𝑐

Solution:

Given:

Kx2 + 2k2x + k = 0

Hence:

K = 2 , K2 = 4

95 | P a g e
By using answer.

Solution:

Given:

5x2 + 3x – 6 = 0

Compare with

ax2 + bx + c = 0

Hence:

a = 5, b = 3, c = -6

Solution:

Given:

-5x2 – 3x – 6 = 0

Compare with

ax2 + bx + c = 0

Hence:

a = -5, b = -3, c = -6

Solution:

Number of real roots

Discriminant = b2 – 4ac

= 4 – 12

=-8

Since discriminant is negative

So there is no real roots

Solution:

Given:

-3x2 + 2x + 1 = 0

96 | P a g e
Using:

Disc = b2 – 4ac

= 4 + 12

= 16

Two real roots are possible

Solution:

Given:

x2 + 2x + 1 = 0

Using

Disc = b2 – 4ac

=4–4

=0

One real root possible.

97 | P a g e
1/1/2022 CHAPTER-3
MATHEMATICAL PROGRESSION
ARITHMETIC SEQUENCE OR PROGRESSION

Arithmetic sequence: a sequence in which a constant d (common difference) can be added to


each term to get the next term.

Common difference: the constant (common) difference, usually denoted as d

Geometric Sequence: a sequence in which a constant r can be multiplied by each term to get
the next term

Common ratio: the constant (common) ratio, usually denoted by r.

(i) Arithmetic sequence:

an =a +(n-1) d
(ii) Arithmetic Mean: The average between 2 numbers

𝒂+𝒃
A.M= 𝟐

(iii) Geometric Sequence:

an=arn-1
(iv) Geometric Mean: the term between two given terms of a geometric sequence as
defined by the following formula:

GM=√𝒂𝒃

1|Page
Series
Note: Arithmetic series: The sum of the terms of an arithmetic sequence.

Geometric Series: The sum of the terms of a geometric sequence.

Sums of arithmetic and geometric series

(v) Sum of an Arithmetic series:


𝒏
, 𝒔𝒏 = 𝟐 [𝟐𝒂 + (𝒏 − 𝟏)𝒅]

Or
𝒏
𝒔𝒏 = [𝒂 + 𝒂𝒏]
𝟐

(vi) Sum of a geometric series:

𝒂(𝒓𝒏 − 𝟏)
𝒔𝒏 =
𝒓−𝟏
(vii) Infinite Geometric Series

Theorem: an infinite geometric series is convergent and has a sum “S” if and only if its common
ratio, r meets the following condition: | r | < 1

𝒂
𝑺=
𝟏−𝒓

2|Page
Question 1.
What is the 8th term? When a = 10 and d = 5

Solution
8th term = 10 + (8 – 1) 5 = 45

Question 2.
. What is the 19th term? When a = 10 and d = -6

Solution

19th term = 10 + (19 – 1) (-6) = -98

Question 3.
Calculate the 6th and 11th term of a series which starts at 7 and has a common difference of 6

Solution

a = 7 and d = 6

6th term = 7 + (6 – 1) 6 = 37

11th term = 7 + (11 – 1) 6 = 67

Question 4.
Calculate the 5th and 12th term of a series which starts at 8 and has a common difference of -7.

Solution

a = 8 and d= -7

5th term = 8 + (5-1) (-7) = -20

12th term = 8 + (12-1) (-7) = -69

Question 5.
What is the sum of the first 8 terms? When a = 10; d=5

Solution

8
S=2 {2 × 10 + (8 − 1)5}

S = 4 (20 + 35)

S = 220

2012

3|Page
Question 6. Find the sum of all integers between 170 and 1000 which are exactly divisible
by 8.

A. B. C. D.

Solution
Given
The number of terms between 170 and 1000 exactly divisible by 8 are
176, 184, 192 , …….. 992
It is an A.P.
Here
a = 176
d=8
an = 992
n =?
Using
an = a + (n – 1) d
992 = 176 + (n – 1) 8
816 = (n-1)8
102 = n ‒ 1
n = 103
Now find S103 (Sum of 103 terms)
Using
n
Sn = 2 [2a + (n – 1) d]
103
S103 = [2 (176) + (102) 8]
2
S103 = 60152

2012
Question.7 Sidiqi has recently appointed as an accountant in XYZ Ltd. He intends to set
aside an amount of Rs.10,000 from his salary received in the first month.
Which would be increased by Rs 500 in each of the subsequent months. You
are required to determine the number of months it would take for his
aggregate savings to exceed Rs 500, 000

A. B. C. D.

Solution
Given
a = 10,000
d = 500
Sn = 500,000
n =?
4|Page
Using
n
Sn = 2 [2a + (n – 1) d]
n
500,000 = [2 (10,000) + (n – 1) 500]
2
n
500,000 = 2 [20,000 + 500n – 500]
1,000,000 = n [19500 + 500n]
1,000,000 = 19500n + 500n2
500n2 + 19500n – 1,000,000 = 0
500 (n2 + 39n – 2000) = 0
n2 + 39n – 2000 = 0(using calculator)
Here
a = 1, b = 39, C= ‒2000
Using
−b ± √b 2 − 4ac
n=
2a
−39 ± √(39)2 − 4(1)(−2000)
n=
2(1)
−39 ± 97.573
n=
2
−39 + 97.573 −39 − 97.575
n= , or n =
2 2
n = 29.287 n = -68.25 (Neglect –ve value)

2009
Question.8 Ali walked a certain distance on the first of February 2009. On each
successive day he walked 100 meters more than the previous day. If the total
distance covered by him in 28 days is 51.8 km, find the distance covered by
him on
(i) The first day
(ii) The last day

A. B. C. D.

Solution
Given
a =?
d = 100m
S28 = 51.8 km = 51800m
a28 =?
n = 28
Using
n
Sn = 2 [2a + (n – 1) d]
28
51800 = [2 (a) + (28 – 1) 100]
2

5|Page
51800 = 14 [2a + 27 (100)]
3700 = 2a + 2700
3700 – 2700 = 2a
1000 = 2a
500 = a = First term of sequence

Now last term of sequence 5


α28 = a + 27d
= 500 + 27 (100) = 3200m = 3.2 km

2007
Question.9 A manufacturer produced 1200 units of a product in the first week. He
wishes to plan his production and is considering the following options.
Option 1: Increase production by 80 units each week.
Option 2: Increase production by 5% each week. Under each of the above
option, compute the following
(a) Output in the 20th week
(b) Total output of first 20 weeks

A. B. C. D.

Solution
Given

Option 1 Option 2
Given Given 1200,1260,1323….. 20th weeks
1200, 1280, 1360 ……… 20th weeks a = 1200
Here 1+r=r = 1.05
a = 1200 n = 20
d = 80 Using
n = 20 an = arn-1
Using a20 = a + 19d a20 = 1200 (1.05)19
= 1200 + 19(80)=2720 = 3032.34
Also Also
20 [rn −1]
S20 = 2 [2a + (n – 1)d] Sn = a r−1
S20 = 10 [2(1200) + (19)( 80)] 1200[(1.05)20 − 1]
S20 =
1.05 − 1
= 10 [2400 + 1520]
= 39200 units S20 = 39679 units

6|Page
2007
Question.10 The sum of 12 terms of an arithmetic progression is 234 and the last term is
36 find the common difference and first term.

A. B. C. D.

Solution
Given
S12 = 234
an = 36
d =?
a =?
Using
n
Sn = 2 [2a + (n – 1) d]
n
Sn = 2 [a + a + (n – 1) d]
n
S12 = 2 [a + an]
12
234 = [a + 36]
2
234
= a+36
6
39 = a + 36
3= a
Also
an = a + (n – 1) d
36 = 3 + (12 – 1)d]
36 = 3 + 11d
33 = 11d
3=d

2005
Question.11 The Sum of 16 terms of an Arithmetic progression, who’s last term is 250, is
2800. Find the first term and common difference.

A. B. C. D.

Solution
Given
n = 16
l = a16 = 250
s16 = 2800
a =?

7|Page
d =?
Using
n
Sn = 2 [a + l]
16
S16 = [a + l]
2
2800 = 8[a + 250]
2800
= a + 250
8
350 = a + 250
350 – 250 = a
…100 = a…
Now
an = a + (n -1 ) d
a16 = 100 + (16 – 1) d
250 = 100 + 15d
150 = 15d
…d = 10…

2005-k-34
Question.12 An auditorium has 20 seats in the front row, 25 seats in the second row, 30
seats in the third row and so on for 13 rows. How many seats are there in the
thirteen row?

A. B. C. D.

Solution
Given
According to given statement
20, 25, 30, ……….. 13th term
The given sequence is in A.P
a = 20, d = 5, n = 13
a13 = ?
Using
an = a + (n – 1)d
a13 = 20 + (13 – 1)5
a13 = 20 + (12)5
a13 = 20 + 60
…a13 = 80…

8|Page
2001
Question.13 A company earned a profit of Rs.37000 in its first year of operation. It is
estimated that the profit will increase by Rs.7000 each year. Find the total
profit of first 10 years using the method of arithmetic progression.

A. B. C. D.

Solution
According to given statement
37,000, 44,000, 51,000, 10th term
Given
a = 37000
d = 7000
n = 10
s10 = ?
Using
n
Sn = 2 [2a + (n – 1)d]
10
= [2 (37000) + (9)(7000)]
2
= 5 [74000 + 63000]
= 5[137,000]
S10= 6,85,000
K-1
Question.14 7th term of an A.P 8, 5, 2, ‒1, ‒4 is

A. B. C. D.

Solution
Given
8, 5, 2, ‒1, ‒4 are in AP
Here
a=8
d = ‒3
a7 = ?
n=7
Using
an = a + (n ‒1) d
a7 = 8 + (7 – 1) (‒3)
a7 = 8 + (6) (‒3)
a7 = 8 ‒ 18
a7 = ‒10

9|Page
K-2
Question.15 Prove that a-b, a, a + b are in A.P

A. B. C. D.

Solution
Given
a – b, a, a + b
Now we prove they are in A.P
d = 2nd term – 1st term
d= a – (a – b)
d=a–a+b
d = b ----- (i)
Also
d = 3rd term – 2nd term
d=a+b-a
d = b --- (ii)
From (i) and (ii), common difference is same.
Given sequence is an A.P

K-3
Question.16 The sum of the series 1 + 2 + 3 + ……+ n is.

A. B. C. D.

Solution
Given
1 + 2 + 3 + …. + n
Here
a=1
d = 2 -1 = 1
n=n
Sn =?
an =n
Using
n
Sn = 2 [a + an]
n
= 2 [1 + n]
n
Sn = 2 (n + 1)

10 | P a g e
Question.17 2 + 4 + 6 + ….. + 100,Find sum of the series

A. B. C. D.

Solution
Given
2 + 4 + 6 + ……. + 100
Here
a=2
d = 4-2 = 2
Sn =?
an = 100
Using
an = a + ( n – 1)d
100 = 2 + (n – 1) 2
100 = 2 + 2n - 2
100 = 2n
…50 = n…

Also
n
Sn = 2 [2a + (n – 1) d]
50
= 2 [2(2) + (50 – 1) 2]
= 25 [4 + 98]
= 25 (102)
Sn = 2550

K-5
3 4 5 17
Question.18 Which term of the A.P , , , … … . is
√7 √7 √7 √7

A. B. C. D.

Solution
Given
3 4 5
, , . is in A. P
√7 √7 √7
Here
3
a=
√7
1
d=
√7
n=?
17
an =
√7
Using

11 | P a g e
an = a + (n – 1) d
17 3 1
= + (n − 1) ( )
√7 √7 √7
17 1
= (3 + n − 1)
√7 √7
17 = 2 + n
17 ‒ 2 = n
…n = 15…

K-6
Question.19 The value of x such that 8x + 4, 6x – 2, 2x + 7 will form an A.P is

A. B. C. D.

Solution
Given
8x + 4, 6x – 2, 2x + 7
Now we will find the value of d to form an A.P.

d = 2nd term – 1st term


d = 6x – 2 – 8x - 4
d = -2x – 6 ----- (i)
Also
d = 3rd term – 2nd term
d= 2x + 7 – 6x + 2
d = ‒4x + 9 ------ (ii)

Compare (i) and (ii)


‒2x – 6 = -4x + 9
‒2x + 4x = 9 + 6
2x = 15
x = 15/2
K-7
Question.20 The sum of a certain number of terms of an A.P series -8, -6, -4, …… is 52.
The number of terms is.

A. B. C. D.

Solution
Given
A.P is
‒8, ‒6, ‒4, ……….. 52
Here
a = ‒8
d=2

12 | P a g e
Sn = 52
n =?
Using
n
Sn = 2 [2a + (n – 1)d]
n
52= 2 [2 (‒8) + (n – 1)2]
n
52 = 2 [‒16 + 2n – 2]
n
52 = 2 [‒18 + 2n]
52 = n[‒9 + n]
52 = ‒9n + n2
n2 – 9n – 52 = 0
n2 – 13n + 4n – 52 = 0
n(n – 13) + 4(n – 13) = 0
(n + 4) (n – 13) = 0
Either n + 4 = 0 or n – 13 = 0
n = ‒4 or n = 13
(Neglect –ve value)

Required number of term is 13

K-10
Question.21 If the terms 2x, (x + 10) and (3x + 2) be in A.P, the value of x is.

A. B. C. D.
Solution
Given
2x, (x + 10) and (3x + 2) are in A.P
∴ Common difference must be same
i.e., 2nd term – 1st term = 3rd term – 2nd term
(x + 10) – 2x = (3x + 2) – (x + 10)
x + 10 – 2x = 3x + 2 – x ‒ 10
‒x + 10 = 2x ‒ 8
‒x – 2x = ‒8 ‒ 10
‒3x = ‒18
x =6
K-11
Question.22 The sum of all odd numbers between 200 and 300 is.

A. B. C. D.

Solution
Given
Required odd number between 200 and 300 are

13 | P a g e
201, 203, 205, ………. 299
Here
a = 201
d=2
n =?
Sn=?
an=299
Using
an = a + (n – 1) d
299 = 201 + 2n ‒ 2
299 – 201 = 2n ‒ 2
98 = 2n ‒ 2
98 + 2 = 2n
100 = 2n
50 = n
Also
n
Sn = 2 [2a + (n – 1) d]
50
S50 = [2(201) + (50 – 1) 2]
2
= 25 [402 + (49)2]
= 25 [402 + 98]
= 25 [500]
S50 = 12,500

K-16
Question.23 The first term of an A.P is 14 and the sum of the first five terms and the first
ten terms are equal in magnitude but opposite in sign. The 3rd term of the
A.P is

A. B. C. D.

Solution
Given
a = 14
a3 =?
Also
S5 = ‒S10
5 10
[2a + (5 ‒ 1) d] = − [20 + (10 – 1) d]
2 2
5 −10
[2a + 4d] = [2a + 9d]
2 2
Put a = 14

14 | P a g e
5
[2(14) + 4d] = ‒5[2(14) + 9d]
2
5
[28 + 4d] = ‒5 [28 + 9d]
2
5 [14 + 2d] = ‒5 [28 + 9d]
14 + 2d = ‒ [28 + 9d]
14 + 2d = ‒28 – 9d
2d + 9d = ‒28 ‒ 14
11d = ‒42
42
…..d = − 11…..
Also
a3 = a + 2d
−42
a3 = 14 + 2 ( 11 )
84
𝑎3 = 14 −
11
154 − 84
𝑎3 =
11
70
𝑎3 =
11
4
a3 = 6
11

K-17
Question.24 The first and last term of an A.P is -4 and 146. The sum of the terms is 7171.
The number of terms is.

A. B. C. D.

Solution
Given
a = ‒4
an = l = 146
Sn = 7171
Using
n
Sn = 2 [a + an]
n
7171 = 2 [‒4 + 146]
n
7171 = 2 [142]
7171 = n(71)
n = 101

15 | P a g e
K-19
Question.25 A person is employed in a company at Rs.3000 per month and he would get
an increase of Rs.100 per month. The total amounts which he receives in 25 years and the
monthly salary in the last month is.

A. B. C. D.

Solution
Given
a = 3000
d = 100
n = 25
a300 =?
S300 =?
First we find the base year salary
an = a + (n – 1)d
a300 = 3000 + (300-1) 100
a300 = 3000 + (299) 100
= 3000 + 29900
a300 = 32900
Using
n
S300 = 2 [2a + (n-1) d]
300
= [6000 + (300-1)(100)]
2
= 150 (35900)
S300 = 5385000

K-21
Question.26 Three numbers are in A.P and their sum is 21. If 1, 5, 15 are added to them
respectively they from G.P the number are.

A. B. C. D.

Solution
Given
Let the three numbers in A.P
a – d, a, a + d
Then according to statement
a -d + a + a + d = 21
3a = 21

16 | P a g e
….a = 7….
Also
a – d + 1, a + 5, a + d + 15 are in G.P
Put a = 7
7 – d + 1, 7 + 5, 7 + d + 15
8 – d, 12, 22 + d are in G.P, common ratio must be equal
12 22 + d
=
8−d 12
144 = (8 – d)( 22 + d)
144 = 176 + 8d – 22d – d2
d2 + 14d – 176 + 144 = 0
d2 + 14d – 32 = 0
d2 + 16a – 2d – 32 = 0
d(d + 16) -2 (d + 16) = 0
(d-2) (d + 16) = 0
d – 2 = 0, or d + 16 = 0
d = 2, d= -16 (neglect – ve sign)
Now required numbers are
When
a=7
Than d = 2
∴ 7 – 2, 7, 7 + 2
5, 7, 9

K-22
Question.27 The sum of three numbers in G.P is 70. If the two extremes by multiplied
each by 4 and the geometric mean by 5, the product are in A.P the numbers
are.

A. B. C. D.

Solution
Given
Let the required three numbers in G.P are
a, ar, ar2
Than according to statement
a + ar + ar2 = 7 0 (i)
Also
4a , 5ar , 4ar2 are in A.P
∴ Common difference must be equal
5ar – 4a = 4ar2 – 5ar

17 | P a g e
a (5r – 4) = a(4r2 ‒ 5r)
5r – 4 = (4r2 – 5)
5r – 4 = 4r2 – 5r
4r2 – 5r – 5r + 4 = 0
4r2 – 10r + 4 = 0
2(2r2 – 5r + 2) = 0
2r2 – 5r + 2 = 0
2r2 – 4r – r + 2 = 0
2r (r – 2) -1(r – 2) = 0
(2r – 1) (r – 2) = 0
Either
2r -1 = 0 or r – 2 = 0
2r = 1
r = 1/2 r=2
Putting value of r = 1/2 in (i)
a + a (1/2) + a (1/2)2 = 70
a a
a + + = 70
2 4
4a + 2a + a
= 70
4
7a = 280
….a = 40….

Also putting value of r = 2 in (i)


2
a + a (2) + a(2) = 70
a + 2a + 4a = 70
7a = 70
…a = 10…
Now required numbers are
When r = ½ When r = 2
Than a = 40 Than a = 10
a = 40 ∴ a = 10
ar = 40 (1/2) = 20 ar = 10(2) = 20
2 2
ar = 40(1/2) = 10 ar2 = 10 (2)2 = 40
Hence numbers ax
40, 20, 10 or 10, 20, 40
K-23
Question.28 The sum of all natural numbers between 500 and 1000 which are divisible by
13 is

A. B. C. D.

18 | P a g e
Solution
Given
Required number which are divisible by 13 and between 500 and 1000 are 507, 520, 533,
… 988
Here
a = 507
an = 988
Sn = ?
d = 13
n=?
First find the value of n
Using
an = a + (n – 1) d
988 = 507 + (n – 1) 13
988 = 507 + 13n - 13
988 = 494 + 13n
494 = 13n
n = 38…
Now. We will find sum of term by using
n
Sn = 2 [a + l]
38
S38 = [507 + 988]
2
= 19 (1495)
…S38 = 28405…
K-24
Question.29 If unity is added to the sum of any number of term of the A.P 3, 5, 7, 9, …
The resulting sum is.

A. B. C. D.

Solution
Given
3,5,7,9,-------
Then
3+1=4(perfect square)
3+5+1=9(perfect square)
3+5+7+1=16(perfect square)
3+5+7+9+1=25(perfect square)

19 | P a g e
K-25
Question.30 A person has to pay Rs.975 by monthly installments each less then the former
by Rs.5 the first installment is Rs.100. The time by which the entire amount
will be paid is.

A. B. C. D.

Solution
Given
Sn = 975
a = 100
d = ‒5
n=?
Using
n
Sn = 2 [2a + (n – 1) d]
n
975 = 2 [2 (100) + (n – 1) (‒5)]
n
975 = 2 [200 – 5n + 5]
n
975 = 2 [205 – 5n]
1950 = n [205 – 5n]
1950 = 205n – 5n2
5n2 – 205n + 1950 = 0
5(n2 – 41n + 390) = 0
n2 – 41n + 390 = 0
n2 – 26n – 15n + 390 = 0
n(n – 26) – 15 (n – 26) = 0
(n – 15) (n – 26) = 0
either n ‒15 = 0 or n ‒ 26 = 0
n = 15 or n = 26
Required time to pay off the loan is 15 months
K-27
Question.31 Divide 12.50 into five parts in A.P. Such that the first part and the last part
are in the ratio of 2 : 3.

A. B. C. D.

Solution
Given
Let the five numbers in A.P. are a - 2d, a - d , a, a + d , a –+2d
According to statement
a -2d + a - d + a + a + d + a + 2d = 12.50

20 | P a g e
5a = 12.50
a = 2.5
Also The first part and last part are in ratio 2 : 3
∴ a + 2d : a – 2d = 2 : 3
a − 2d 2
=
a + 2d 3
3a - 6d = 2a + 4d
3a – 2a = 4d + 6d
a = 10d
Put a = 2.5
∴ 2.5 = 10d
2.5
= d
10
…d = 0.25…

Now required numbers are


a - 2d = 2.5 - 2 (0.25) = 2
a - d = 2.5 - 0.25 = 2.25
a = 2.5
a + d = 2.5 +(0.25) = 2.75
a +2d = 2.5 + 2 (0.25) = 3

K-28
𝟏
Question.32 The pth term of an A.P is 𝐪 and qth term is 1/p. The sum of pqth term is.

A. B. C. D.

Solution
Given
1
aP = q ------ (i)
aq = 1/p -------- (ii)
Spq = ?
a=?
d=?
From (i)
1
ap = q
a + (p - 1) d = 1/q (iii)
From (ii)
aq = 1/p

21 | P a g e
a + (q – 1)d = 1/p (iv)
(iv) – (iii)
a + (q – 1)d = 1/p
a + (p – 1)d = 1/q
-- - .
1 1
(q – 1 - p + 1)d = p − q
q−p
(q – p)d = pq
1
...d = pq…
1
Putting value of d = pq in (iii)
1 1
a + (p – 1) (pq) = q
p −1 1
a+ =
pq q
1 p− 1
a=q− pq
p−p+1
a= pq
1
a = pq

Now we will find the sum of pqth term


Using
n
Sn = 2 [2a + (n - 1)d]
pq 1 1
Spq = [2 (pq) + (pq − 1) ]
2 pq
pq 2 pq − 1
Spq = [pq + ]
2 qp
pq 2+pq− 1
Spq = [ ]
2 pq
1+pq
Spq = 2

Question.33 The nth term of an A.P is (3n+5). Its 7th term is

Solution
Given
an = 3n + 5
𝑎7 = 3(7) +5
= 21 +5= 26

Question: 34 if nth term of a G.P is 𝟐𝒏 , then its 6th term is .


Solution:

22 | P a g e
Given
𝑎𝑛 = 2𝑛
𝑎6 = 26
= 64

Question: 35 the 5th and 13th term of an A.P are 5 and -3 respectively. The 9th term of the
A.P is.
Solution:
Given:
𝑎5 = 5 𝑎13 = -3
a + 4d = 5 ___ (i) a + 12d = -3 ___ (ii)

Solving (i) and (ii)


a + 4d = 5
-a ± 12d = ∓3
-8d = 8
d = -1
Put in (i)
a + 4d = 5
a + 4(-1) = 5
a–4=5
a=5+4
a=9

Now
𝑎9 = a + 8d
= 9 + 8(-1)
= 9 -8
=1

Question: 36 if a clock strikes once at one o’clock, twice at two o’clock, thrice at 3 o’clock
and so on and again once at one o’clock and so on. Then how many times will the bell are
struck in the course of 2 days?
Solution:
Given circle of clock
1 + 2 + 3 + 4……. 12 = 78
Now total time the bell struck.
= 78 * 4 = 312

23 | P a g e
Question: 37 Mr. Ali has been offered appointment simultaneously from two firms MCB
and silk bank MCB offers a pay scale of 500 – 50 -1200, while silk bank offers a scale of
600-25-1250. He wants to select that offer which results in greatest total receipts drawn in
20 years of services. He should accept the offer of.
Solution:
Offer – I – MCB Offer –II – silk bank
n = 20 n = 20
a = 500*12 = 6000 a = 600 * 12 = 7200
d = 50 *12 = 600 d = 25 * 12 = 300
𝑆𝑛 = ? 𝑠𝑛 = ?

𝑛 𝑛
𝑆𝑛 = 2 [2𝑎 + (𝑛 − 1)𝑑] 𝑆𝑛 = 2 [2𝑎 + (𝑛 − 1)𝑑]
20 20
𝑆20 = [2(6000) + (20 − 1)600] 𝑆20 = [2(7200) + (20 − 1)300]
2 2
𝑆20 = 2, 34,000 Offer –I is Acceptable 𝑆20 = 201,000

Question: 38 A T.V company in its 3rd year existence produces 6000 sets and 9000 sets in 5th
year. What is production of company in first year.

Solution:
𝑎3 = 6000 𝑎5 = 9000
a + 2d = 6000 ___ (i) a + 4d = 9000 ___ (ii)
From (i) and (ii)
a + 2d = 6000
± 𝑎 ±4d = 9000
-2d = -3000
d = 1500

Put in (i)
a + 2 (1500) = 6000
a + 3000 = 6000
a = 6000 – 3000
a = 3000

Question: 39 A drilling company contracted to drill a well at a cost of Rs.30 for first foot,
Rs.35 for the second foot and Rs.40 for the third foot and so on. How deep a
well can be drilled for Rs.3075.
Solution:
30, 35, 40……..3075
a = 30, d = 5, 𝑠𝑛 = 3075

24 | P a g e
Using
𝑛
𝑠𝑛 = 2 [2a + (n-1) d]
𝑛
3075 = 2 [2(30) + (n-1)5]
6150 = n (60+5n-5)
6150 = n (55+5n)
6150 = 55n + 5𝑛2
5𝑛2 + 55n – 6150 = 0
𝑛2 + 11n – 1230= 0
Using calculator
n = 30

Question: 40 Find 5th term from the end of the G.P. 3, 6, 12, 24,…………………………
12288.

Solution:
First we find n. a9 (5th term from end) 13

12288 = 3(2)n-1

4096 = 2n-1

212 = 2n-1

n = 13

Also

a9 = arn-1 = 3(2)8 = 768

Question: 41 Find middle term of A.P

(i) 2, A, 4

Solution:
4+2
A= =3
2

(ii) 2, A1, A2 , 4

Solution:
a=2

A4 = a + 3d = 4

2 + 3d = 4

25 | P a g e
2
d=3

2 8⁄
A1 = a + d = 2 + 3 = 3

A2 = a + 2d = 2 + 2 (2⁄3) = 10⁄3

Question: 42 If nth term of A.P is 𝒙 and 2n (th) term is y and 3n (th) term is

Solution:
nth term is

a + (𝑛 - 1)d = 𝑥 ………….. (i)

2n (th) term is

a + (2𝑛 - 1)d = y …………….(ii)


𝑦−𝑥
d= 𝑛

put in (i)

a = 𝑥 − (𝑛 − 1)𝑑
𝑦−𝑥 𝑛𝑥−(𝑛𝑦−𝑛𝑥−𝑦+𝑥) 2𝑛𝑥−𝑛𝑦+𝑦−𝑥
𝑎 = 𝑥 − (𝑛 − 1) ( )= =
𝑛 𝑛 𝑛

Put in

a3n = 𝑎 + (3𝑛 − 1)𝑑


2𝑛𝑥−𝑛𝑦+𝑦−𝑥 (3𝑛−1)(𝑦−𝑥)
= +
𝑛 𝑛

2𝑛𝑥−𝑛𝑦+𝑦−𝑥+3𝑛𝑦−3𝑛𝑥−𝑦+𝑥
= 𝑛

−𝑛𝑥+2𝑛𝑦
= = 2𝑦 − 𝑥
𝑛

Question: 43 Obtain the sum of all first 10,000 integers which are neither divisible by 2
nor by 5

Solution:
10,000
Divisible by 2 = = 5,000
2

10,000
Divisible by 5 = = 2,000
5

26 | P a g e
10,000
Divisible by 5 = = 1,000
10

Divisible by 2 and 5 = 5,000 + 2,000 – 1,000 = 6,000

Not divisible by 2 and 5= 4,000

Also 1,3,7, …………………………….. 9,999


4,000
𝑆𝑛 = (1 + 9,9999)
2

𝑆𝑛 = 2,000(10,000) = 20,000,000

Question: 44 The 35th term of an A.P is 69. Find the sum of its 69 terms

Solution:
𝑎35 = 𝑎 + 34𝑑

69 = 𝑎 + 34𝑑 … … … … . . (𝑖)
69
Also S69 = [2𝑎 + (69 − 1)𝑑]
2

69
S69 = [(2𝑎 + 68𝑑)]
2

69
= × 2[𝑎 + 34𝑑]
2

S69 = 69(69)

S69 = 4761

Question: 45 What is the sum of all two digits’ number which are divided by 7 gives a
remainder of 3?

Solution:

10, 17, 24. ………………….94 𝑎𝑛 = 𝑎 + (𝑛 − 1)𝑑


𝑛
Sn = 2 (𝑎 + 𝑎𝑛) 94 = 10 + (𝑛 − 1)7

13
= (10 + 94) 94 = 3 + 7𝑛
2

= 676 91 = 7𝑛

𝑛 = 13

27 | P a g e
Question: 46 The number of terms between 74 and 25,556 divisible by 5 is

Solution:
75, 80, …………………………. 25,555

Here 𝑎 = 75, 𝑎𝑛 = 25,555, 𝑑=5

Using

𝑎𝑛 = 𝑎 + (𝑛 − 1)𝑑

25,555= 75 + (𝑛 − 1)𝑑

25,480 = (𝑛 − 1)5

5,096 = 𝑛 − 1

𝑛 = 5,097

Question: 47 The number of terms taken so that 𝟏 + 𝟐 + 𝟒 + 𝟖 … … … . 𝒘𝒊𝒍𝒍 𝒃𝒆 𝟖, 𝟏𝟗𝟏

Solution:
Using.
𝑎(𝑟 𝑛 −1)
𝑆𝑛 = 𝑟−1

8,191 = 2n -1

8,192 = 2n

213 = 2𝑛

𝑛 = 13

Question:48 The 5th and 13th term of an A.P are 5 and -3 respectively. The 9th term of A.P
is

Solution:
a5 = a+4d=55 ……………….(i) a + 4d = 5

a13= a + 12d + -3 …………(ii) -a ± 12d = ±3

put in (i) __________

a+4(-1) = 5 -8d = 8

a=9 (d = -1)

28 | P a g e
Also

a9 = a + 8d = 9 + 8(-1) = 9 – 8 = 1

Question: 49 which term of the A.P 24,21,18, ……is the second negative term
Solution:
Given a=24 , d=-3
Since
an<0
a+(n-1)d<0
24+(n-1)(-3)<0
-3(n-1) <-24
3(n-1)>8
n-1>8
n >9
10th term will be first negative term and 11th term will be 2nd negative number
Question: 50 divide 25 into five parts in A.P. Such that the first and last term are in the
ration 2:3 then what will be its 10th term
Solution:
Let the required terms are
a-2d, a-d, a, a+d,a+2d
then
a-2d+ a-d+ a+a+d+a+2d=25
a=5
Also
𝑎 − 2𝑑 2
=
𝑎 + 2𝑑 3

d=1/2
and
a10 =a+9d
a10 =5+9(1/2)
a10=9.5

29 | P a g e
Question:51 A person has salary scale of 10000-100-20000.Find his total salary after 6
years and 10 months
Solution:
Yearly salary
Given
a=10000 x12 =120000 , d=100 x12 =1200 ,n=6

sn=n/2{2a+(n-1)d}
sn= 6/2{2(120000) +(6-1)1200}
sn= 738,000

Monthly salary
sn=n/2{2a+(n-1)d}
sn=10/2{2(10000)+(10-1)100}
sn=5(20900)
sn=104500

Total salary =738000+104500=842,500

30 | P a g e
EXTRA PRACTICE QUESTION FOR A.P

Question:1
Solve for n when the first term is 10 and the series has a common difference of 2. How many
terms are needed to produce a sum of 252?

Question:2
The first term is 10 and the series has a common difference of 5. How many terms are needed to
produce a sum of 325?

Question:3
A person sets aside Rs 1000 in the first month, 1100 in the second, 1200 in the third and so on.
How many months will it take to save Rs 9100?

Question:4
A company is funding a major expansion which will take place over a period of 18 months. It
will commit Rs 100,000 to the project in the first month followed by a series of monthly
payments increasing by Rs.10000 per month. What is the monthly payment in the last month of
the project? What is the total amount of funds invested by the end of the project?

Question:5
Ali intends to start saving in July. He will set aside Rs 5000 in the first month and increase this
by Rs 300 in each of the subsequent months. How much will he set aside in March next year?

Question:6

A property owner has to repay his load of Rs 425000. He can pay Rs 20,000 in the first year and
then increase payment by Rs 5000 in every installment. How many installments will it take for
him to clear his loan?

Question:7
A firm agrees to hire an employee at the pay of Rs 65000 per month. Annual increase in salary
would be 78,000. However, it does not agree to increase the salary after it reaches Rs1638,000
per year. How long would it take the employee to exhaust his increment?

31 | P a g e
GEOMETRIC PROGRESSION

Question.1

What is the 9th term? When a = 3; r = 2

Solution
nth term = a r n-1

9th term = 3× 2 9 -1

9th term = 3 × 256 = 768

K-8
Question.2 The 7th terms of the series 6, 12, 24, … is.

A. B. C. D.

Solution
Given
6, 12, 24 , ... 7th term
Here
a=6
r=2
n=7
a7 = ?
Using
an = ar4-1
a7 = 6(2)7-1
a7 = 6(2)6
a7 = 6(64)
a7 =384

K-9
Question.3 The last term of the series x2, x, 1 to 31 term is.

A. B. C. D.

Solution
Given
x2 , x, 1, …… 31 term
Here

32 | P a g e
a = x2
r = 1/x
n = 31
a31 =?
Using
an = arn-1
a31 = ar31-1
a31 = x2 (1/x)30
1
a31 = 28
x

K-12
𝟑 𝟗 𝟐𝟕
Question.4 The sum of the series 𝟏, , 𝟐𝟓 , 𝟏𝟐𝟓 …….. to infinity is
𝟓

A. B. C. D.

Solution
Given
3 9 27
1, , , ………
5 25 125
Here
a=1
r = 3/5
S∞ =?

Using
a
S∞ =
1− r
1
𝑆∞ =
1 − 3⁄5
1
𝑆∞ =
2⁄
5
5
𝑆∞ = ⁄2

K-13
𝟑 𝟑
Question.5 Sum of the series 𝟔 + 𝟐 + + ……… to infinity is
𝟖

A. B. C. D.

Solution

33 | P a g e
Given
3 3
6 + + +. … ….
2 8
Here
a=6
r = 1/4
S∞ =?
Using
a
S∞ =
1− r
6
S∞ =
1 − 1/4
6
S∞ =
4 − 1/4
6
S∞ =
3/4
24
S∞ =
3
S∞ = 8

K-14
𝟏 𝟏
Question.6 The sum of the infinite series 𝟏 + + … … … . 𝐢𝐬
𝟐 𝟒

A. B. C. D.

Solution
Given
1 1
1+ +
2 4
Here
a=1
r = 1/2
S∞ = ?
Using
a
S∞ =
1− r
1
S∞ =
1
1−2
1
S∞ =
2−1
2
34 | P a g e
1
S∞ =
1/2
S∞ = 2

K-15-2004
𝟏 𝟏 𝟏
Question.7 Sum of infinity of the following geometric progression 𝟏.𝟏 + (𝟏.𝟏)𝟐
+ (𝟏.𝟏)𝟑
+
⋯ 𝐢𝐬

A. B. C. D.

Solution
Given
1 1 1
+ + + ⋯ 𝑖𝑠
1.1 (1.1)2 (1.1)3
Here
1 10
a= =
1.1 11
1 10
r= =
1.1 11
S∞ =?
Using
a
S∞ =
1− r
10⁄
= 11
10
1 − 11
10
S∞ = 11
11 − 10
11
10/11
S∞ =
1/11
10 11
S∞ = ×
11 1
S∞ = 10
K-18
Question.8 If you save 1 paisa today, 2 paisa the next day, 4 paisa the succeeding day
and so on, then your total saving in two weeks will be.

A. B. C. D.

Solution
Given
35 | P a g e
1, 2, 4, …………. 14th term
Here
a=1
r=2
S14 =?
Using
a(r 𝑛 − 1)
Sn =
r−1
1 (214 − 1)
S14 =
2−1
214 − 1
S14 =
1
S14 = 2 ‒ 1
14

S14 = 16384 ‒ 1
= 16383

K-33
𝟏 𝟏 𝟏
Question.9 How many terms are there in the sequence of 𝟏𝟐𝟖 , 𝟔𝟒 , 𝟑𝟐 , …... 32, 64?

A. B. C. D.

Solution
Given
1 1 1
, 64 , 32 , ………. 32, 64
128
Here
1
a=
128
r=2
an = 64
Using
an = arn-1
1
64 = (2)n−1
128
8192 = 2n-1
213 = 2n-1
 13 = n-1
13 + 1 = n
…n = 14…

K-35
𝟏 𝟏 𝟏
Question.10 Sum of the series = 𝟏, 𝟑 , 𝟗 , 𝟐𝟕 , ………. To infinity is?

A. B. C. D.

36 | P a g e
Solution
Given
1 1 1
1, , , , … … ….
3 9 27
Here
a=1
1
r=3
S∞ = ?
Using
1
S∞ =
1−r
1
S∞ =
1 − 1/3
1
S∞ =
3−1
3
1
S∞ =
2/3
S∞ = 3/2

K-36
Question.11 The sum of the infinite series 2 + √𝟐 + 1 + …… is?

A. B. C. D.

Solution
Given
2 + √2 + 1 + ……
Here
a=2
r = (√𝟐/𝟐)
S∞ = ?
Using
a
S∞ =
1−r
2
S∞ =
1 − √𝟐/𝟐

2
S∞ =
2 − √2
2
4
S∞ =
2 − √2

37 | P a g e
4 2 + √2
S∞ = ×
2 − √2 2 + √2
4 (2 + √2)
S∞ = 2
(2)2 − (√2)
4 (2 + √2
S∞ =
4−2
4 (2 + √2)
S∞ =
2
S∞ = 2 (2 + √2)
S∞ = 4 + 2√2

2011
Question.12 Bashir has chosen to receive pockets money of Rs.1200 per week. He was also
offered an alternative by which he would have received Rs.1 on the first day
of the week, Rs.3 on the second day, Rs.9 on the third day and so on. Has he
taken a wise decision?

A. B. C. D.

Solution
Given
1, 3, 9, …………. 7th term
Here
a=1
r=3
n=7
Using
a (r n − 1)
Sn =
r−1
1 (37 − 1)
S7 =
3−1
37 − 1
S7 =
2
2186
S7 =
2
S7 = 1093
Bashir has chosen a wise decision by accepting a pocket money of 1200

2009
Question.13 A shopkeeper sold goods worth Rs.3.0 million during 2008. If he is able to
increase his sale by 15% annually, determine the year in which he would
achieve annual sale of Rs.25 million.

38 | P a g e
A. B. C. D.

Solution
Given
a = 3million
r = 1 +r% = 1 + 15% = 1.5
an = 25 million
n=?
Using
an = a rn-1
25 = 3 (1.15)n-1
25 = 3 (1.15)n-1
25
= (1.15)n−1
3
8.33 = (1.15)n-1
Taking log on both side
log 8.33 = log (1.13)n-1
log 8.33 = (n-1) log (1.15)
0.9208 – (n – 1) (0.0606)
15.19 = n-1
n = 16.19
…n = 16… approximately

2008
Question.14 A gardener has been given the task of digging an area of 800 square meters.
It is expected that he will dig 10 square meters on the first day and on each
successive day he will dig 1.2 times more than the area he day on the previous
day. Find the number of days the gardener will take to complete the task.

A. B. C.0+ D.

Solution
Given
10, 22, 48.4 ………….. are in G.P
Here
a = 10
r = 1 + r = 1+1.2 = 2.2
n=?
Sn = 800
Using
a (r n − 1)
Sn =
r−1
10 [(2.2)n − 1]
800 =
2.2 − 1

39 | P a g e
10 [(2.2)n − 1]
800 =
1.2
960 = 10 [(2.2)n – 1]
96 = (2.2)n ‒1
97 = (2.2)n
Taking log on both side
Log 97 = log (2.2)n
Log 97 = n log 2.2
1.9867 = n (0.3424)
n = 5.80
n = 6 days (approximately)

2006
Question.15 A man got a lottery of Rs.7,000,000 on 15th February. He paid income tax of
25% on his amount. From the remaining amount he decided to donate one
paisa on the first day, 2 paisa on 2nd day , 4 paisas on third day and so on
commencing from March 1, till the end of March. Determine whether the
amount of money available with him is enough to pay these donations?

A. B. C.0+ D.

Solution
Given
Cash received = 7,000,000
Tax paid = 0.25 (7000,000) = 1,75,0000
Remaining amount = 7,000,000 – 175,0000
= 5,25,0000
Also Given
1, 2, 4, …………31st term
Here
a=1
r=2
n = 31
Using
a (r n − 1)
Sn =
r−1
1 (231 − 1)
S31 =
2−1
231 − 1
S31 =
1
S31 = 2147483648 - 1
S31 = 2147483647 paisa
S31 = Rs.2147483647
S31 = Rs.2147483647/100=21474836.47
So the amount 5250,000 is not enough to pay the donation in the following sequence.

40 | P a g e
2002
Question.16 A company offers two alternatives for the payment of salary to an executive.
Either one may receive Rs.300,000 per month or 1 paisa on the first day of
the month, 2 paisa’s on the second day of the month, 4 paisa on the third day,
8 paisa on the fourth day and so on for the rest of the month i.e., each day
getting double of what he has received one day before which of the two
alternatives should be prefer?

A. B. C.0+ D.

Solution
First offer of salary = 300,000

2nd offer:

Given
1, 2, 4, 8, ………….. 30th term
Here
a=1
r=2
n = 30
Sn = ?
Using
a (r n − 1)
Sn =
r−1
1 (230 − 1)
S30 =
2−1
S30 = 230 -1
S30 = 10,73, 741824 paisas
S30 = Rs.1073741824/100 = 10,73,7418.24
Hence 2nd offer is more preferable?

2004
Question.17 The arithmetic mean of profit earned by two companies ‘X’ and ‘Y’ is Rs.34
millions whereas the geometric mean is Rs.16 million. Find out the profit
earned by each company. It is known that company ‘X’ earned more than
‘Y’.

A. B. C. D.

Solution
Given
Let the profit earned by company X = x the profit earned by company Y = y according to
Statement.

41 | P a g e
x+y
Mean =
2
x+y
34 =
2
x + y = 68 (i)
Also
G. M = √xy
16 = √xy
xy = 256 (ii)

From (i)
x + y = 68
x = 68 – y (iii)
Putting value of ‘x’ in (ii)
(68 – y) y = 256
68y – y2 = 256
y2 – 68y + 256 = 0
y2 – 64y – 4y + 256 = 0
y(y – 64) –4 (y – 64) = 0
(y – 4) (y – 64) = 0
Either
y–4=0 or y – 64 =0
y=4 or y = 64
Put y = 4 in (iii) Put y = 64 in (iii)
x = 68 – 4 x = 68 – 64
x = 64 x=4
Hence required profit is x = 64 and y = 4

K-29
Question.18 The least value of n for which the sum of n terms of the series 1 + 3 + 32 + …
is greater than 7000 is.

A. B. C. D.

Solution
Given
1 + 3 + 32 + ……..
Here
a=1
r=3
Sn > 7000

42 | P a g e
Using
a (r n − 1)
> 7000
r−1
1 (3n − 1)
> 7000
3−1
3n − 1
> 7000
2
3n – 1 > 14000
3n > 140001
Taking log on both side
log 3n > log 14001
n log 3 > log 14001
n (0.4771) 4.14615
n >8.69
or n = 9 (approximately)

Question.19 The sum of infinite terms in a G.P is 2 and sum of their squares is 4/3 the
series is.

A. B. C. D.

Solution
Given
a
S∞ = 1−r = 2
a= 2(1-r) -------- (i)

And
a2
S∞ = 1−r2 = 4/3 ---------(ii)
Putting value of a in (ii)
4 (1 − r)2 4
× =
(1 + r) (1 − 𝑟) 1 3
1−r 1
=
1+r 3
3+3r = 1 + r
3 – 2 = 2r
2 = 4r
r = 1/2
Putting value of r = 1/2 in (i)
a
=2
1 − 1/2

43 | P a g e
a
=2
1/2
2a = 2
…a = 1..,.
∴ required series is
1, 1/2, 1/4, ….
K-31
Question.20 The infinite G.P with first term 1/4 and sum 1/3 is

A. B. C. D.

Solution
Given
a = 1/4
S∞ = 1/3
r=?
Using
a
S∞ = 1−r
1 1/4
=
3 1−r
1−r 1
=
3 4
4 – 4r = 3
4r = 4-3
4r = 1
r = 1/4
∴ Required G.P is
1/4 , 1/16 , 1/64 , ……..

k-32
Question.21 The number x, 8 and y are in G.P and the numbers x, y, - 8 are in A.P. The
value of x and y are .

A. B. C. D.

Solution
Given
x, 8 , y are in G.P
8 𝑦
=8
𝑥

xy = 64 (i)

44 | P a g e
Also x, y, ‒ 8 are in A.P
∴ common difference must be equal
y – x = ‒8 ‒ y
y + y = ‒8 + x
2y = ‒8 + x
x = 2y + 8 (ii)
Putting value of x in (i)
(2y + 8) y = 64
2y2 + 8y = 64
2y2 + 8y – 64 = 0
2(y2 + 4y – 32) = 0
y2 + 4y – 32 = 0
y2 + 8y – 4y – 32 = 0
y(y + 8) ‒4 (y + 8) = 0
(y – 4) (y + 8) = 0
Either
y – 4 = 0, or y + 8 = 0
y=4 or y = ‒8
When
y=4 y = -8
Put in (i) Put in (i)
x (4) = 64 x (-8) = 64
…x = 16…. …x = -8…
K-20
Question.22 A person borrows Rs. 8000 at 2.76% simple interest per annum. The
principal and the interest are to be paid in 10 monthly installments. If each
installment is double the preceding one, the value of the first and the last
installments are.

A. B. C. D.

Solution
Given
P = 8000
r = 2.76% = 0.0276
t = 10
Using
I= Prt
I = 8000 x 0.0276 x 10/12
I= 184
So the total amount paid in 10 installment is

45 | P a g e
S10= Amount + interest
S10= 8000 + 184
S10 = 8184
Payment plan
x, 2x, 4x, ………….. 10th term
Here
a=x
r=2
S10 = 8184
a10 =?

First we fill find ‘a’

Using
a (r n − 1)
Sn =
r−1
x (210 − 1)
8184 =
2−1
8183 = x (1024 – 1)
8184 = x (1023)
a= x = 8
Also
a10 = ar10-1
a10= 8 (2)10-1
a10= 8 (29)
a10 = 8 (512)
a10 = 4096

Question:23 A Gardner brought 5 rabbits, after 2 months’ rabbit become 10 and after 4
months they become 20. If the growth continuous on the same ratio. What would be the
number of rabbits after 1 year?
Solution:
Given
5 , 10, 20 ……
a=5
r=2
n = 6(times)
Using
𝑎 (𝑟 𝑛 −1 )
𝑆𝑛 = 𝑟−1
5(26 −1 )
𝑆6 = 2−1

46 | P a g e
𝑆6 = 315
Question:24 Find G.P of the middle terms
(i) 2, G, 4

Solution:
G = √2 × 4 = √8

(ii) 2, G1 , G2 , 4

Solution:
a = 2, a4 = 4

a4 = a𝑟3 = 4

a𝑟3 = 4

r (3) 1/3 = (2)1/3

r= 2 1/3

Now

G1 = a𝑟 = 2(2)1/3 = 24/3

G2 = a𝑟2 = 2(2)2/3 = 25/2

Question: 25 In a G.P the product of first three terms is 𝟐𝟕⁄𝟖. The middle term is

Solution:
Required terms are

𝑎, 𝑎𝑟, 𝑎𝑟 2

Then 𝑎(𝑎𝑟)(𝑎𝑟 2 ) = 27⁄8

𝑎3 𝑟 3 = 27⁄8

Middle term = 𝑎𝑟 = 3⁄2

Question: 26 Find the first term if G.P whose sum of infinite term is 8 and second term is
2.

Solution:
𝑎
𝑎𝑟 = 2 … … … … … (𝑖𝑖) 𝑆∞ = 1−𝑟

47 | P a g e
Put in (ii) 8(1 − 𝑟) = a …………(i)

8(1 − 𝑟)𝑟 = 2

8𝑟 − 8𝑟 2 = 2

8𝑟 2 − 8𝑟 + 2 = 0

8𝑟 2 − 4𝑟 + 4𝑟 + 2 = 0 (solve yourself)

Question: 27
The Hart company Limited has decided to change their mix of sales of two products Reds
and Blues. Sales of Blues are to be gradually reduced to zero and replaced by increased
sales of Reds. Blues sold in December amount to 4096 units. Sales volume will be halved
each month, beginnings in January, the 1000 units of Reds will be sold in January, and
sales will be increased by 20% per month. How many units of Reds will be sold in October
next year, to the nearest whole units?
Solution: Unit Of Red In The Month Of October
Time= jan to oct=n =10
a=1000
r =20%
using
an=a(1+r)10-1
an =1000(1.20)9

an =5160
Question: 28
The Hart company Limited has decided to change their mix of sales of two products Reds
and Blues. Sales of Blues are to be gradually reduced to zero and replaced by increased
sales of Reds. Blues sold in January amount to 4096 units. Sales volume will be halved each
month, beginnings in January, the first month when only one unit is sold, will be the last
month of sales of blues. The 1000 units of Reds will be sold in January, and sales will be
increased by 20% per month. How many units of Blues will be sold in total over the period
January –December?
Solution: Total Unit Of Blues
Time= Jan to Dec= n =12
4096,2048,…….12th term
a=4096

48 | P a g e
r =1/2=0.5
using
4096(0.512 −1)
Sn= 0.5−1

Sn =8190
Question:29
(Drop/Fall) 14 A ball drops from the height of 2 meter each time. It hits the ground, it
rebound to 75% of its height. Find total distance it travels before coming to a stop
Solution:

=2+2[2*3/4+2*3/4*3/4+….]
=2+2[3/2+9/8+….]
𝑎
Distance of rise and drop =s∞ =1−𝑟
6
4
= 2[ 1−0.75 ]

=12
Distance of rise and drop =12+2 =14
Question: 30
(Throw) A ball is thrown 7.8m into the air. The ball falls and rebounds to 68% of its
previous height the fall again
(i) Find the total distance the ball travel until it hits the ground for the 8th time.
(ii) If the ball were allowed to continue bouncing forever, how far it would it travel
Solution:
(i) 2[7.8+7.8(0.68)+7.8(0.68)2+7.8(0.68)3+…..+7.8(0.68)7 ]
𝑎(1−𝑟 𝑛 )
Sn=2[ ]
1−𝑟

7.8(1−0.688 )]
=2[ ]
1−0.68

=46.5
(ii)
2(𝑎)
Sn=
1−𝑟

49 | P a g e
2[7.8]
=
1−0.68

=48.75m
Question: 31
A ball bounces two-third of the distance it fall. If it is dropped from a height 10 meter, how
far does it move before hitting the floor for the fourth time
Solution:
10+2[10*2/3 +10*2/3*2/3+….]
=10+2[20/3+40/9+80/27]
=10+2[20/3 +40/9 +80/27]
=886/27
=38.1
Question: 32

Find the first term if G.P whose sum of infinite term is 8 and second term is 2.

Solution:
𝑎
𝑎𝑟 = 2 … … … … … (𝑖𝑖) 𝑆∞ = 1−𝑟

Put in (ii) 8(1 − 𝑟) = a …………(i)

8(1 − 𝑟)𝑟 = 2

8𝑟 − 8𝑟 2 = 2

8𝑟 2 − 8𝑟 + 2 = 0

8𝑟 2 − 4𝑟 + 4𝑟 + 2 = 0 (solve yourself)

Question: 33
if a rubber ball consistently bounces back 2/3 of the height from which it is dropped, what
fraction of its original height will the ball bounce after being dropped and bounced four
times without being stopped?
Solution:
Each time the ball is dropped and it bounces back, it reaches 2/3 of the height it was dropped
from so
=2/3 *2/3 *2/3 *2/3 =16/81

50 | P a g e
EXTRA PRACTICE QUESTION
Question:1

What is the 9th term and their sum? When a = 100; r = 0.5

Question:2

Find the 10th term of the geometric progression, 5, 10, 20…. and their sum

Question:3

Find the 12th term of the geometric progression, 5, -10, 20, -40 ………

Question:4

Find the 6th term of the geometric progression, 1,000, 800, 640….

Question:5

Find the sum of the given geometric series:


1 1 1 1 1
+ + + + …
2 4 8 16 𝑛

Question:6

Find the sum of the given geometric series:

Question:7

What is the last term in the sequence 1.2, 1.44, …, 2.0736? Also find its sum.

Question:8

What is the last term in the sequence 240,144, …, 4.0310784? Also find its sum.

Question:9

A company earns a profit of Rs. 75000 in its first year. In the second year, it made Rs. 86250. If the
profit continues to grow in geometric sequence, what will be the profit in its 4th year. What will be the
total earned profit?

51 | P a g e
TEST: SEQUENCE AND SERIES-PRACTICE
QUESTION :1 In a G.P a5 = 24, a13 = 39,936, a10 =?

(a)2470.72 (b)9474.872
(c)2744.2448 (d)None of these
Question :2 Which one of the following is not from Geometric Progression?

(a)10,000 (b)2,4,6
(c)5, 7, 5 (d)1/ 21, 2
Question: 3 Arithmetic progression a1 = x + 7 , a3 = x + 49 , a7= ?

(a) x + 14 (b) x +133


(c) x + 3 (d) x - 11
Question: 4 Mr. A has salary scale of 2000 -250-25000 and Mr B has salary scale of 1500-360-
80,000 find their total salary after 4 years and 10 months

(a)144,000 (b)244,000
(c)444,111 (d)None of these
Question: 5 A person has monthly income of Rs. 1000 and his income increased by Rs. 50 of
the previous month income. On which term his total salary will exceed 100,000.

(a)46th team (b)47th team


th
(c)86 team (d)85th team
Question: 6 A person has salary scale of 10000-100-20000, Find his total salary after 5 years
and 8 months

(a)964,000 (b)696,000
(c)396,296 (d)None of these
Question: 7 In an AP the 3 term is x + 7 and 7th term is x + 49, the 1st term is:
rd

(a)x – 7 (b)x + 7
(c)x - 14 (d)x – 49
Question: 8 The sum of the three numbers in A.P is 21 and the product of the first and third
number of the sequence is 45. What are the three numbers?

(a)5,7,9 (b)3,7,11
(c)9,5,7 (d) Both a and c
Question: 9 Which term of the A.P. 24, 21, 18, ……… is the second negative term?

(a)8th (b)9th
th
(c)10 (d)11th
Question: 10 Divide 25 into five parts in A.P. such that the first and the last term are in the ratio
2:3. Then what will be its 10th term?

(a) 11.5 (b)8.5


52 | P a g e
(c)7.5 (d)9.5
Question: 11 Find the 15 term of the G.P. 3, 6, 12. 24 ……, 12,288
th

(a)384 (b)49,152
(c)15,360 (d)768
Question: 12 The two geometric means between the num 1 and 64 are:

(a)1 and 64 (b)2 and 16


(c)4 and 16 (d)4 and 32
Question: 13 The sum of all odd between 150 and 300 is:

(a)72,501 (b)34,421
(c)85,001 (d)16,875
Question: 14 If the second term of G.P. is 2 and the of its infinite term is 8, then its 5th term is:

(a) 𝟏⁄𝟒 (b)1⁄2


(c)2 (d)4
Question:15 If a rubber ball consistently bounces back2⁄3 of the height from which it is
dropped, what fraction of its original height will the ball bounce after being dropped and
bounced four times without being stopped?

(a)16/81 (b)16/27
(c)4/9 (d)37/81
Question:16 Shaheer bought equipment for RS, 450,000. The amount is Payable in 11 annual
installments, where first 10 installment are in arithmetic progression and 11th installment with
settle the remaining balance. If the first two installment are RS. 10,000 and RS. 12,000
respectively compute the amount of 11th installment.

(a)240,000 (b)230,000
(c)260,000 (d)280,000
Question:17 Saqib will invest Rs. 2 on the first day of January, Rs. 4 on the second day of
January, Rs. 6 on the third day of January and so on for the first five days of January. He will
then spend Rs 1 per day from sixth to tenth day of January. Compute what amount will he have
with him on the eleventh day of January

(a)25 (b)30
(c)40 (d)45
Question:18

A company produces 100 units in first week. It is expected that re-engineering production
process will increase output by 10% every week of the number of units produced in previous
week till fifth week, after which the output per week will be constant. If re-engineering is carried
out compute the number of total units produced in ninth and tenth week (to the nearest whole
number)

53 | P a g e
(a)293 (b)280
(c)275 (d)288
Question:19 The first and last terms of an arithmetic progression are 5 and 25 respectively. Find
the middle term of the series

(a)20 (b)25
(c)5 (d)15
Question:20 Sami borrowed a loan of Rs. 11,000 with the condition that the repayment will be
made in seven annual installments. The first and the last installment will be equal to each other
and the first six installments will be in arithmetic progression. If common difference between
first six installments is Rs 500. Compute fifth installment.

(a)2,500 (b)500
(c)3,000 (d)1,000
Question:21 A company earned a profit of Rs. 50,000 in first year. The profits is expected to
increase by Rs 5,000 each year: How many years will it take the company to earn a total profit of
Rs. 165,000

(a)4 (b)3
(c)5 (d)2
Question:22 Find the value of Y such that Y+2, 2Y+3, 6Y+1 will form an AP

(a)2 (b)3
(c)1 (d)5
Question:23 If the first and the last terms of an arithmetic progression are 10 and 50
respectively and the total number of terms is 10. Find the sum of all the terms

(a)200 (b)400
(c)300 (d)100
Question:24 A worker produces 1,092 units in three days. If the production is expected to
increase by 20% percent each day compute the total number of units produced on first and last
day.

(a)732 (b)432
(c)300 (d)360
Question:25 The first and the last term of an arithmetic progression are in the ratio of 3:4. The
sum of all term is 210 and there are 6 terms in total. Compute common difference.

(a)2 (b)3
(c)5 (d)10
Question:26 Rizwan borrowed a loan of Rs 6,620 to be paid in three annual payments in
geometric progression if the first installment is Rs. 2,000 compute the common ratio.

(a)1.15 (b)1.1
(c)1.2 (d)1.22

54 | P a g e
Question:27 The sum of 5 terms of an AP, whose last term is 17 is 65. The first term and
common difference are:

(a)9 and 2 (b)13 and 2


(c)11 and 7 (d)11 and 2
Question:28 The last term of the sequence, 1, -4, 16, …… to 5th term is

(a)64 (b)256
(c)-256 (d)-64
Question:29 Find the value of x such that x+3, 2x+11, 2x+41 are in the G.P.

(a)3 (b)4
(c)2 (d)1
Question:30 If the first term of a G.P is a and the common ratio is 2. What will be the value of
third term?

(a) a (b) 2a
(c)3a (d) 4a
Question:31 If the fourth term of a G.P is 8 times its first term compute common ratio

(a)3 (b)4
(c)2 (d)5
Question:32 The first term of an A.P. is 5 and fourth term is 17. Compute the value of common
difference

(a)4 (b)12
(c)3 (d)5
Question:33 A particular A.P. has first term as 3 and third term as 7. The common difference of
this A.P. is equivalent to the first term of a G.P. with common ratio of 5. Compute third term of
G.P.

(a)60 (b)50
(c)40 (d)20
Question:34 Sum of G.P. is 175, if the total number of terms is 3 and common ratio is 2
compute the value of first term

(a)25 (b)50
(c)40 (d)70
Question:35 The infinite G.P. with common ratio of ½ and sum 10 is

(a)5,2.5, 1.25 ….. (b)1.25, 2.5, 5….


(c)2.5, 1.25, 5 ….. (d)None of these

55 | P a g e
Solution

Given:

10,000, 12,000, …… 10th term

a = 10,000, d = 2000, n = 10

Using
𝑛
S 10 = 2 [2𝑎 + (𝑛 − 1)𝑑]

10
S 10 = [2(10,000) + (10 − 1)(2000)]
2

S 10 = 5 [20,000 + 18000]

S 10 = 190,000

Remaining amount= 450,000 – 190,000 = 260,000

Solution

Investment for first five days

2, 4, 6 ….. 5th term

a=2, d=2, n=5


𝑛
Sn= [2𝑎 + (𝑛 − 1)𝑑]
2

5
S5= [4 + 8]
2

S 5 = 30

Spending in five days = -5

Remaining amount in 11th day = 30 – 5 = 25

Solution

110,121,,…. 5th week

Given

a=110 ,r =1+0.1=1.1 n=5

using

an =arn-1

56 | P a g e
a6 =ar6-1

an =110(1.1)5

an =177.15

for constant increase

6th week 7th week 8th week 9th week 10 week


177.15 *1.1=195 215 237 261 287

Hence in 10th week total out put

=287

Solution

Given: a=5, an =25, middle term = ?


𝑎 + 𝑎𝑛
Using = 2

5 + 25
=
2

= 15

Solution

57 | P a g e
Solution

a=50,000 d= 5000 Sn =165,000

using
𝑛
S 10 = 2 [2𝑎 + (𝑛 − 1)𝑑]

𝑛
165000 = 2 [100000 + (𝑛 − 1)5000]

330,000 = 𝑛 [100000 + 5000𝑛 − 5000]

S 10 = 𝑛 [95000 + 5000𝑛]

5000n2 +95000n -330,000=0

n=3,-22

n=3

Solution

Given

Y + 2, 2y +3, 6y+1 are in A.P

Then

d= 2y+3 – y – 2 = y+1

d = 6y + 1- 2y – 3 = 4y – 2

Now

d=d

y+1 = 4y – 2

3y = 3

Y=1

Solution

Given

a = 10, an = 50, n = 10

using

58 | P a g e
𝑛
Sn = [2𝑎 + (𝑛 − 1)𝑑]
2

𝑛
Sn = 2 [𝑎 + 𝑎𝑛]

10
Sn = [10 + 50]
2

Sn = 300

Solution

S3 = 1092, r = 1 + 20% = 1.20, a = ?, an = ?

Using:

a(r4 −1)
Sn=
r−1

a(1.23 −1)
1092 =
1.2−1

a(1.23 −1)
1092 =
0−2

a = 300

Also:

a3 = a r 3 -1

a3 = a r 2

a3 = 300 (1.2)2

a3 = 432

Total units = 300 + 432 = 732

Solution

Given:

a = ?, an = ? ratio = 3:4, S6 = 210


a 3
=
an 4

a 3
=4
a6

59 | P a g e
a 3
=
a+5d 4

4a = 3a + 15d

a – 15d = 0 ____________ (1)

S6 = 210
𝑛
[2𝑎 + (𝑛 − 1)𝑑] = 210
2

6
[2𝑎 + (6 − 1)𝑑] = 210
2

3 [2𝑎 + 5𝑑] = 210

6a + 15 d = 210 __________ (2)

From (1) and (2)

a = 30, d = 2

Solution

S 3 = 6620, a = 2000, r= ? , n=3

Using

a(rn −1)
Sn =
r−1

2000(r3 −1)
6620 =
r−1

(r−1)(r2 +r+1)
3.31 =
r−1

r2 + r – 3.30 = 0

r = 1.1

Solution

Given

S5 = 65, a 5 = 17, a=?, d=?

Using
𝑛
S 5 = 2 [𝑎 + 𝑎5 ]

60 | P a g e
5
S 5 = 2 [𝑎 + 𝑎5 ]

5
65 = [𝑎 + 17]
2

1
13 = [𝑎 + 17]
2

26 = a + 17

a=9

Also

an = a +(n - 1) d

a5 = a + (4) d

17 = a + 4d

8 = 4d

d=2

Solution

Given

a = 1, r = - 4, a5 = ?

a 5 = ar 4

a 5 = 1 ( -4) 4

a 5 = 256

Solution

Given

X + 3, 2x+11, 2x+41 are in G.P

Since term are in G.P

2nd term 3rd term


= 2nd term
1st term

2x+11 2x+41
=
x+3 2x+11

(2x + 11)(2x + 11) = (2x + 41) (x + 3)

61 | P a g e
4x 2 + 22x + 22x + 121 = 2x 2 + 6x + 41x + 123

4x 2 + 44x+121 = 2x 2 + 47x + 123

2x 2 – 3x – 2 = 0

Using calculator

x=2

Solution

Given:

a = a, r = 2, a3 = ?

a 3 = ar 2

a 3 = a (2) 2

a3=4a

Solution

Given:

a 4 = 8a

ar 3 = 8 a

r3=8

r3=23

r=2

Solution

Given:

a = 5, a 4 = 17, d =?

a4 = a + 3d

17 = 5 + 3d

12 = 3d

d=4

62 | P a g e
Solution

Given:

a = 3, a3 =7, d = a, r =5, a3 = ?

Now

a3 = 7
a + 2d = 7

3 + 2d = 7

2d = 4

d=2

Also

d=a

d=2

a3 = ar 2

a3 = 2 (5)2

a3= 50

Solution

Sn = 175, n = 3, r = 2, a= 2

Using:

a(r4 −1)
Sn =
r−1

a(23 −1)
175 =
2−1

a(8−1)
175 =
1

175 = a (7)

25 = a

63 | P a g e
Solution

S∞ = 10, r = 1/2
𝑎
S∞ =
1−𝑟

𝑎
10 =
1−1⁄2

𝑎
10 = 1
⁄2

a=5

Required sequence is

S, 5/2, 5/4, ………

5, 2.5, 1.25, …………

64 | P a g e
1/1/2022 Chapter-4
LINEAR PROGRAMMING

ABDUL AHAD BUTT


Different Parts of Coordinate System
The three main parts of a rectangular coordinate system are discussed below.

(i) Axes
There are two axes in rectangular coordinate system. These are mutually perpendicular to each
other. The horizontal axis is called X axis, while the vertical axis is referred to as Y axis.

(ii) Origin
On rectangular coordinate system, the origin is a point where X and Y axis intersect each other. It
is denoted by the letter O.

(iii) Quadrants
The two axes divide the rectangular coordinate system in four sections that are known as
quadrants. They are denoted by Roman symbols I, II, III and IV. All the coordinates that have
positive abscissa and ordinate (+, +) do belong to quadrant I. In quadrant II, former is negative and
latter is positive (-, +). Quadrant III include coordinates with negative abscissa and ordinate (-, -)
and quadrant IV has former positive and latter negative. They are demonstrated below.

1|Page
Question.1

For the function y=2x, calculate the values of y for x =-2,-1,0,1,2. Plot the points on a graph
paper and draw the graph of the function

Solution:

x Y
-2 ¼
-1 ½
0 1
1 2
2 4

Question.2

Sketch the feasible region for the following set of constraints:

x ≤5 y≥2 x+y≤8

Solution:

x≤5 (Restriction line)

y≥2 (Restriction line)

x+y≤8 ( Find intercept and zero test)

X -INTERCEPT ZERO TEST


Put y=0 in Put
x+y =8 x=0 and y=0 in
x+0 =8 x+y < 8
x=8 0+0 ,8
(x,y)=(8,0) 0<8
Y-INTERCEPT True zero test
put x=0 Shade towards origin
x+y =8
0+y =8
y=8
(x,y)= (0,8)

2|Page
Question.3

For the following set of inequalities. Draw the graph and highlight the feasible region
clearly indicating its boundaries.

Solution:

x+y≥7 (Find intercept and zero test)

2x+y≥10 (Find intercept and zero test)

x+y≤ 8 (Find intercept and zero test)

x,y≥0 (First Quadrant)

x+y=7 2x+y=10 x+y= 8 Zero Test


X-INTERCEPT X-INTERCEPT X-INTERCEPT x+y>7
put y=0 put y=0 put y=0 0>7(false)
x=7 x=5 x=8
(x,y)=(7,0) (x,y)=(5,0) (x,y)=(8,0) 2x+y>10
0>10 (false)
Y- INTERCEPT Y INTERCEPT Y INTERCEPT
put x =0 put x=0 put x =0 x+y< 8
y=7 y=10 y=8 0<8 (True)
(x,y)=(0,7) (x,y)=(0,10) (x,y)=(0,8)

3|Page
Question.4

Plot the following inequalities on graph and identify the feasible region

x, y≥ 1 , x, y ≤ 4 and x ≥ y

Solution:

x ≥ 1 (Restriction line)

y ≥ 1 (Restriction line)

x ≤ 4 (Restriction line)

y ≤ 4 (Restriction line)

x ≥ y (line passes through origin and

first and third quadrant)

4|Page
Question.5

Sketch the feasible region and identify the redundant constraints from the following set of
inequalities

x+ y ≤ 6 5x +3y ≤ 15 X≤2 x.y ≥ 0

Solution:

x+y ≤ 6 (Find intercept and zero test)

5x +3y ≤ 15 (Find intercept and zero test)

X≤2 (Restriction line)

x.y ≥ 0 (First Quadrant)

x+y = 6 5x +3y = 15 ZERO TEST


X-INTERCEPT X-INTERCEPT x+y < 6
put y=0 put y=0 0<6(True)
x=6 x=3
(x,y)=(6,0) (x,y)=(3,0) 5x +3y < 15
0<15 (True)
Y- INTERCEPT Y- INTERCEPT
put x =0 put x =0
y=6 y=5
(x,y)=(0,6) (x,y)=(0,5)

5|Page
Question.6

Sketch the feasible region and identity The point of optimal solution for the Function

Z =2X +5Y

Subject to the following constraints

x-3y ≤ 0 (line passes through origin)

x+3y ≥ 150 (intercept and zero test)

y ≤ 50 (Restriction line)

x,y ≥ 0 (First Quadrant)

x+3y =150 ZERO TEST


X-INTERCEPT x+3y >150
put y=0 0>150(False)
x=150
(x,y)=(150,0)

Y- INTERCEPT
put x =0
y=50
(x,y)=(0,50)

Co-ordinates values

(150,0) 2x150 +5x0 =300

(75,25) 2x75 +5x25 = 275

(150,50) 2x150 +5x50 =550

150 units of x and 50 units of y will optimize the function

Z =2x +5y

6|Page
Question.7

Solve the following set of inequalities by graphical method and find the feasible region

x+y≥ 8 , 2x+y ≥ 12 , x+y ≤ 10 , x ≥ 0 and y ≥ 0

Solution:

x+y=8 2x+y=12 x+y=10 Zero Test


X-INTERCEPT X-INTERCEPT X-INTERCEPT x+y>8
put y=0 put y=0 put y=0
x=8 x=6 x=10 0>7(false)
(x,y)=(8,0) (x,y)=(6,0) (x,y)=(10,0)
2x+y>12
Y- INTERCEPT Y INTERCEPT Y INTERCEPT
put x =0 put x=0 put x =0 0>10 (false)
y=8 x=12 y=10
(x,y)=(0,8) (x,y)=(0,12) (x,y)=(0,10) x+y< 8
0<8 (True)

7|Page
Question.8 The graph to express the inequality x + y < 9 is: a

Solution:

Answer

Note: if y is less then x, the shade tends towards x-axis

Question.9 The graph to express the inequality 5x + 3y > 30 is: c

Solution:

Answer

8|Page
𝟏
Question.10 The graph to express the inequality y < (𝟐) x is indicated by: d

Solution:

Answer

Question.11
(i) An employer recruits experienced (x) and fresh workmen (y) for his firm under the
condition that he cannot employ more than 9 people, x and y can be related by the
inequality:
Solution:

(a) x+y≠9 (b) x + y < 9, x > 0, y > 0


(c) x + y > 9, x > 0, y > 0 (d) None of these

(ii) On the average, an experienced person does 5 units of work while a fresh recruit
does 3 units of work daily and the employer has to maintain an output of at least 30
units of work per day. This situation can be expressed as:
Solution:

(a) 5x + 3y < 30 (b) 5x + 3y > 30


(c) 5x + 3y > 30, x > 0, y > 0 (d) None of these

(iii) The rules and regulations demand that the employer should employ not more than 5
experienced hands to 1 fresh one. This fact can be expressed as:
Solution:

(a) y > x/5 (b) 5y < x


(c) x > 5y (d) None of these

9|Page
(iv) The union however forbids him to employ less than 2 experienced person to each
fresh person. This situation can be expressed as:

Solution:

(a) x < y/2 (b) y < x/2


(c) y > x/2 (d) x > 2y

Question.12

L1: 5x + 3y = 30, L2: x + y = 9, L3: y = x/3, L4: y = x/2


The common region (shaded part) shown in the diagram refers to:

Solution:

(a) 5x + 3y < 30 (b) 5x + 3y > 30


x+y<9 x+y<9
y < 1/5x y > x/3
y < x/2 y < x/2
x > 0, y > 0
(c) 5x + 3y > 30

x+y<9
y < x/3 (d) None of these
y > x/2
x > 0, y > 0

10 | P a g e
Question.13 The region indicated by the shading in the graph is expressed by inequalities:

Solution:

(a) x1 + x2 < 2 (b) x1 + x2 < 2


2x1 + 2x2 > 8 x1 + x2 < 4
x1 < 0, x2 < 0

(c) x1 + x2 > 2 (d) x1 + x2 < 2


2x1 + 2x2 > 8 2x1 + 2x2 > 8

Question.14 If A is the number of batsmen and B is the number of bowlers, the inequality
constraint that the number of batsmen must be no more than 50% of the total
players is:

Solution: A ≤50% (A +B)

(a) A>B (b) A>B (c) B < A (d) A<B

Question.15 Graphs of four equations are drawn below:

L1: 2x + y = 9, L2: x + y = 7, L3: x + 2y = 10, L4: x + 3y = 12

The common region (shaded part) indicated on the diagram is expressed by the set of
inequalities.

11 | P a g e
(a) 2x + y < 9 (b) 2x + y > 9
x+y>7 x+y<7
x + 2y > 10 x + 2y > 10
x + 3y > 12 x + 3y > 12

(c) 2x + y < 9 (d) None of these


x+y>7
x + 2y > 10
x + 3y > 12
x > 0, y > 0

Question.16 The common region satisfied by the inequalities L1: 3x + y > 6, L2: x + y > 4,
L3: x + 3y > 6 and L4: x + y > 6 is indicated by:

Solution:
Answer

Question.17 The set of inequalities L1: x1 + x2 < 12, L2: 5x1 + 2x2 < 50, L3: x1 + 3x2 < 30,
x1 > 0 and x2 > 0 is represented by:

12 | P a g e
Solution: Answer

Question.18 The common region satisfying the set of inequalities x > 0, y > 0, L1: x + y <
5, L2: x + 2y < 8 and L3: 4x + 3y > 12 is indicated by:

Answer

13 | P a g e
Statements:

Question.1
Stable limited manufactures two models of refrigerators. Deluxe model requires 14 hours
of department A and 40 hours of department B. standard model requires 7 hours of
department A and 30 hours of department B. each month a maximum of 2100 hours are
available in department A and 8400 hours in department B. the company makes a profit of
Rs. 5,000 on each deluxe model and Rs. 4,000 on each standard model.

(a) Construct the set of constraints and the objective function for profit
maximization.
(b) Draw the graph and identify the feasible region, clearly indicating its
boundaries.
(c) How many refrigerators of each model should the company manufacture per
day maximize its profit?

Solution:
Let the number of units of Deluxe model =x
Let the number of units of Standard model =y

Product Department Department Profit


A B
Deluxe model 14 40 5000
Standard model 7 30 4000

Objective function
P=5000x + 4000y
Constraints
14x +7y ≤ 2100
40x +30y ≤ 8400
x,y ≥ 0

14 +7y = 2100 40x +30y = 8400 Zero Test

X-INTERCEPT X-INTERCEPT 14 +7y < 2100


put y=0 put y=0
x=150 x=210 0 <2100(True)
(x,y)=(150,0) (x,y)=(210,0)
40x +30y < 8400
Y- INTERCEPT Y INTERCEPT
put x =0 put x=0 0<8400 (True)
y=300 y=280
(x,y)=(0,300) (x,y)=(0,280)

14 | P a g e
Co-ordinate Profit (Rs)

(0,280) 0(5000) +280(4000) =1,12,000

(150,0) 150(5000) +0(4000) =750,000

(30,240) 30(5000) +240(4000) =1,110,000

If 280 units of standard model but no unit of Deluxe model are produced profit will be maximized
at Rs 1,120,000

Question.2
A pharmaceutical company has developed a formula to prepare a herbal medicine. The
medicine can be produced by using either 𝒙 product or product 𝒚 or combination of both.
From each milligram (mg) of 𝒙 if can extract one unit of iron and two units of calcium and
from each mg of 𝒀 it can extract one unit of iron and one unit of calcium. Each tablet of the
medicine is required to contain:

➢ 5 to 7 units of iron
➢ 8 to 10 units of calcium
The cost of 𝑿 is Rs. 6 per mg whereas 𝒀 costs Rs. 4 per mg.

You are required to:

15 | P a g e
i. Construct the set of constraints and the objective function for cost
minimization.
ii. Draw the graph and identify the feasible region, clearly indicating it’s
boundaries.
iii. How many mg of each product should be used to produce the tablets at the
lowest cost?
Solution:(i)
Iron Calcium Cost
X 1 2 6
Y 1 1 4

Let the number of mg of x product =x


Let the number of mg of y product =y

Objective function
C =6x +4y
Constraint

x+y ≥ 5
x +y ≤ 7
Also
2x +y ≥ 8
2x + y ≤ 10
(ii)
x+y=5 x +y =7 2x+y= 8 2x+y= 10 Zero Test
X-INTERCEPT X-INTERCEPT X-INTERCEPT X-INTERCEPT x+y>5
put y=0 put y=0 put y=0 put y=0
x=5 x=7 x=4 x=5 0>5(false)
(x,y)=(5,0) (x,y)=(7,0) (x,y)=(4,0) (x,y)=(5,0)
x+y<7
Y- INTERCEPT Y INTERCEPT Y INTERCEPT Y INTERCEPT
put x =0 put x=0 put x =0 put x =0 0<7 (True)
y=5 y=7 y=8 y=10
(x,y)=(0,5) (x,y)=(0,7) (x,y)=(0,8) (x,y)=(0,10) 2x+y> 8
0>8 (False)

2x+y< 10
0<10(True)

(iii)
Points cost
(1,6)= 1x6 + 6x 4 =30
(3,2)= 3x6 + 2x 4 =26
(5,4)= 5x6 + 0x 4 =30
The lowest cost is 3mg of x product and 2mg of y product

16 | P a g e
Question.3

A furniture firm makes chairs and tables on three machines M1, M2, and M3. The detail of
number of hours required per unit and total hours available on each machine is as follows:

Machine Time in hours


Chair Table Available
time
M1 3 3 36
M2 5 2 50
M3 2 6 60
Draw the graph of linear inequalities and indicate the feasible region by proper
shading.

Solution:
Let the number of chairs =x

Let the number of tables =y

3x +3y ≤ 36

5x +2y ≤ 50

2x +6y ≤ 60

3x+3y=36 5x +2y =50 2x+6y= 60 Zero Test


X-INTERCEPT X-INTERCEPT X-INTERCEPT 3x+3y<36
put y=0 put y=0 put y=0

17 | P a g e
x=12 x=10 x=30 0<36(True)
(x,y)=(12,0) (x,y)=(10,0) (x,y)=(30,0)
5x+2y<50
Y- INTERCEPT Y INTERCEPT Y INTERCEPT
put x =0 put x=0 put x =0 0<50 (True)
y=12 y=25 y=10
(x,y)=(0,12) (x,y)=(0,25) (x,y)=(0,10) 2x+6y< 60
0<60 (True)

Question.4
A manufacturer produces two products P and Q which must pass through the same
processes in the departments A and B having weekly production capacities of 240 hours
and 100 hours respectively. Product p needs 4 hours in department A and 2 hours in
department B and product Q required 3 hours in department A and 1 hour in department
B. profit yields for product P is Rs. 700 and for product Q is Rs. 500. The manufacturer
wants to maximize the profit within the given sets of limitations.

You are required to:

a) Construct the objective function and all the constraints.


b) Plot the above constraints graphically and identify the feasible region.
c) Determine the combination of units which will give the maximum profit.

Solution:
Product Department A Department B Profit
P 4 2 700
Q 3 1 500

18 | P a g e
Let the number of units of product p=x
Let the number of units of product Q =y
Then
Objective function
P =700x + 500y
Subject to
4x +3y ≤ 240
2x +y ≤ 100
x,y ≥ 0

4x +3y =240 2x+y= 100 Zero Test


X-INTERCEPT X-INTERCEPT 4x+3y<240
put y=0 put y=0
x=60 x=50 0<240 (True)
(x,y)=(60,0) (x,y)=(50,0)
2x+y<100
Y INTERCEPT Y INTERCEPT
put x=0 put x =0 0<100 (True)
y=80 y=100
(x,y)=(0,80) (x,y)=(0,100)

The corner points and the solution points will give profit as

P(0,80) =700 x0 + 500 x80 =40,000

P(50,0) =700 x50 + 500 x0 =35,000

P(30,40) =700 x30 + 500 x40 =41,000

The solution points are (30,40)

19 | P a g e
Question.5
A factory is planning to buy some machines to product boxes and has a choice of B-1 or B-9
machines. Rs. 9.6 million has been budgeted for the purchase of machines. B-1 machines
costing Rs. 0.3 million each, require 25 hours of maintenance and produce 1,500 units in a
week. B-9 machines costing Rs. 0.6 million each, require 10 hours of maintenance and
produce 2,000 units a week.

Each machine, B-1 or B-9, needs 50 square meters of floor area. Floor area of 1000 square
meters and maintenance time of 400 hours are available each week. Since all production
can be sold, the factory management wishes to maximize output.

Required:

a) List down the objective function and constraints.


b) Graph the constraints shading the feasible region.

Solution:
Suppose number of B-I type of Machines =x
Suppose number of B-9 type of Machines =y
Then
Amount Floor Area Maintenance Production
B-I 0.3 50 25 1500
B-9 0.6 50 10 2000
Total 9.6 1000 400

Objective function
P = 1500x + 2000y
Subject to
0.3x + 0.6y ≤ 9.6
50x +50y ≤ 1000
25x 10y ≤ 400
x,y≥ 0

0.3x +0.6y =9.6 50x +50y =1000 25x+10y= 400 Zero Test
X-INTERCEPT X-INTERCEPT X-INTERCEPT 0.3x +0.6y <9.6
put y=0 put y=0 put y=0
x=32 x=20 x=16 0<9.6(True)
(x,y)=(32,0) (x,y)=(20,0) (x,y)=(16,0)
50x +50y <1000
Y- INTERCEPT Y INTERCEPT Y INTERCEPT
put x =0 put x=0 put x =0 0<1000 (True)
y=16 y=20 y=40
(x,y)=(0,16) (x,y)=(0,20) (x,y)=(0,40) 25x+10y< 400
0<400 (True)

20 | P a g e
The solution points are (0,16),(8,12) , (13.6,6) and (16,0)
Putting these values in objective function

P(0,16) = 0x1500 + 2000x16 =32000

P(8,12) = 8x1500 + 2000x12 =36000

P(13.6,6) = 13.6x1500 + 2000x6 =32400

P(16,0) = 16x1500 + 2000x0 =24000

Production is maximum at (8,12)

Question.6
A team consist of batsmen and bowlers. If A is the number of batsmen and B is the number
of bowlers, what will be inequality constraint that the number of batsmen must be no more
than 50% of the total players?

Solution:
Given

Number of batsmen =A

Number of Bowlers =B

Total number of team =A+B

By the given condition

A ≤50%(A+B)

A ≤1/2(A+B)

A ≤1/2A+1/2B

A -1/2A ≤1/2B

1/2A ≤1/2B

A ≤B

21 | P a g e
Question.7
A firm manufacture two products, X and Y, for the production, each X requires 2.5 hours
in department A, 3 hours in department B and 1 hour in department C. each Y requires 1
hour in department A, 3 hours in department B and 2 hours in department C. the firm can
use no more than 20 hours in department A, 30 hours in department B and 16 hours in
department C each week. Its profit margin is Rs. 3 per X, and Rs. 4 per Y.

(i) Express the data as equations or inequalities.


(ii) Graph the inequalities constraints and indicate the feasible region on the
graph.

Solution:

Product A B C Price
X 2.5 3 1 3
Y 1 3 2 4
20 30 16

Objective function
P=3X + 4Y
Subject to
2.5x +y ≤ 20
3x +3y ≤30
x +2y ≤ 16
x,y ≥ 0

2.5x +y =20 3x +3y =30 x+2y= 16 Zero Test


X-INTERCEPT X-INTERCEPT X-INTERCEPT 2.5x +y <20
put y=0 put y=0 put y=0
x=8 x=10 x=16 0<20(True)
(x,y)=(8,0) (x,y)=(10,0) (x,y)=(16,0)
3x +3y <30
Y- INTERCEPT Y INTERCEPT Y INTERCEPT
put x =0 put x=0 put x =0 0<30 (True)
y=20 y=10 y=8
(x,y)=(0,20) (x,y)=(0,10) (x,y)=(0,8) x+2y< 16
0<16 (True)

There are two solution points which are (4,6) and (6.5,2.5) and other corner point are (0,8) ,(0,0)
and (8,0)

P(4,6) = 4x3 +6x4=36

22 | P a g e
P(6.5,2.5) = 6.5x3 +2.5x4=31.5
Hence best solution is (4,6)

Question.8
(A) A factory produces transformers and switch gears. The production is carried out in
two department B can work up to 7.5 hours per day. A transformer requires 4 hours in
department A and 3 hours in department B, whereas a switch gear require 2 hours in
department A & 5 hours in department B. the profit margin on a transformer is Rs. 5,000
and on a switch-gear is Rs. 6,000. How many transformers and switch gears may be
produced to maximize profit in a month giving 20 working days? (Use graphical method).

(B) how many machine hours will remain idle during the month in each department at
the maximum level of profit?

Solution:
(a)
Available hours in department A = 6x20 =120 hours

Available hours in department B = 7.5 x 20 =150 hours

Let

Number of transformers produced =x

Number of switch gear produced =y

Department A Department B
Transformers 4 3
Switch gear 2 5
Total 120 150

Objective function

P =5000x + 6000y

Subject to

23 | P a g e
4x +2y ≤ 120

3x +5y ≤ 150

x,y ≥ 0

4x +2y =120 3x+5y= 150 Zero Test


X-INTERCEPT X-INTERCEPT 4x +2y <120
put y=0 put y=0
x=30 x=50 0<120(True)
(x,y)=(30,0) (x,y)=(50,0)
3x +5y <150
Y INTERCEPT Y INTERCEPT
put x=0 put x =0 0<150 (True)
y=60 y=30
(x,y)=(0,60) (x,y)=(0,30)

P(30,0) =5000(30) + 6000(0) =150,000

P(0,30) =5000(0) + 6000(30) =180,000

P(21,17) =5000(21) + 6000(17) =20,7000

So optimal solution is production of 21 transformers and 17 switch gears

(b)

4x21 +2 x17 =118 (2hours capacity unutilized in department A)

3x21 + 5x17 = 148 (2hours capacity unutilized in department B)

24 | P a g e
Question.9
A manufacturer produces two product X1 and X2. Resources available for the production
of these two items are restricted to the following maximum weekly amounts:

200 support staff hours, 320 machine hours and 280 labor hours X1 requires for its
production 1 support staff hour, 1 machine hour and 2 labor hours.

X2 require 1 support staff hours, 2 machine hours and 0.8 labor hour. X1 yields Rs. 300
profit per unit and X2 yields Rs. 200 profit per unit. The manufacturer wants to determine
the profit maximizing weekly output of each product while operating within the set of
resource limitations.

Express the above data in the form of equations and in equalities.

Solution:
Support staff Machine hours Labour hours Sale values (Rs)
hours
X1 1 1 2 300
X2 1 2 0.8 200
Total 200 320 280

Objective function

P =300 X1 + 200X2

Subject to

X1 + X2 ≤200

X1 + 2X2 ≤320

2X1 +0.8 X2 ≤280

X1, X2 ≥ 0

Question.10

A firm makes two types of products: Type A and Type B. The profit on product A is Rs.20
each and that on product B is Rs.30 each. Both types are processed on three machines M1,
M2 and M3. The time required in hours by each product and total time available in hours
per week on each machine are as follow:

Machine Product A Product B Available Time


M1 3 3 36
M2 5 2 50
M3 2 6 60

25 | P a g e
The constraints can be formulated taking
x1 = number of units A and
x2 = number of unit of B as:

Solution:

(a) x1 + x2 < 12 (b) 3x1 + 3x2 > 36


5x1 + 2X2 < 50 5x1 + 2X2 < 50
2x1 + 6X2 < 60 2x1 + 6X2 > 60
x1 > 0, x2 > 0
(c) 3x1 + 3X2 < 36 (d) None of these
5x1 + 2X2 < 50
2x1 + 6x2 < 60
x1 > 0, x2 > 0

Question.11
A manufacturer produces two products X1 and X2. Resources available for the production
of these two items are restricted to 200 support staff hours, 320 machine hours and 280
labour hours. X1 requires for its production 1 support staff hour, 1 machine hour and 2
labour hours. X2 requires 1 support staff hour, 2 machine hours and 0.8 labour hour. X1
yields Rs.300 profit per unit and X2 yields Rs.200 profit per unit. The manufacturer wants
to determine the profit maximizing weekly output of each product while operating within
the set of resource limitations. Situation of the above data in the form of equations and
inequalities is:

Solution:

(a) Z = 300x + 200y (b) Z = 300x + 200y


x + y < 200 x + y < 200
x + 2y < 320 3x + 2y < 320
2x + 0.8y < 280 2x + 0.8y < 280
x > 0, y > 0 x > 0, y > 0

(c) Z = 300x + 200y (d) None of these


x + y < 200
x + 2y < 320
2x + 0.8y < 280

Question.12
A factory is planning to buy some machine to produce boxes and has a choice of B-1 or B-9
machines. Rs.9.6 million has been budgeted for the purchase of machines. B-1 machines
costing Rs.0.3 million each require 25 hours of maintenance and produce 1,500 units a week.
B-9 machines costing Rs.0.6 million each require 10 hour of maintenance and produce 2,000
units a week.

26 | P a g e
Each machine needs 50 square meters of floor area. Floor area of 1,000 square meters and
maintenance time of 400 hours are available each week. Since all production can be sold, the
factory management wishes to maximize output.
Situation of above data in the form of objective function and constraints is:

Solution:
(a) Z = 1500x + 2000y (b) Z = 1500x + 2000y
0.3x + 0.6y < 9.6 0.3x + 0.6y > 9.6
25x + 10y < 400 25x + 10y < 400
50x + 50y < 1000 50x + 50y > 1000
x > 0, y > 0 x > 0, y > 0

(c) Z = 1500x + 2000y (d) Z = 1500x + 2000y


0.3x + 0.6y < 9.6 0.3x + 0.6y < 9.6
25x + 10y < 400 25x + 10y > 400
50x + 50y < 1000 50x + 50y < 1000
x > 0, y > 0 x > 0, y > 0

Question.13
A firm manufactures two products. The products must be processed through one
department. Product A requires 6 hours per unit, and product B requires 3 hours per unit.
Total production time available for the coming week is 60 hours. There is a restriction in
planning the production schedule, as total hours used in producing the two products cannot
exceed 60 hours. This situation can be expressed as:

Solution: 6x +3y < 60

Question.14
A manufacturer produces two products P and Q which must pass through the same processes
in departments A and B having weekly production capacities of 240 hours and 100 hours
respectively. Product P needs 4 hours in department A and 2 hours in department B.
Product Q requires 3 hours and 1 hour respectively, in department A and B. Profit yields
for product P is Rs.700 and for Q is Rs.500.
The manufacturer wants to maximize the profit with the given set of inequalities.

27 | P a g e
The objective function and all the constraints are:

Solution:

(a) Z = 700x + 500y (b) Z = 700x + 500y


2x + 3y < 240 4x + 3y < 240
2x + y < 100 2x + y < 100
x, y > 0 x, y > 0
(c) Z = 700x + 500y (d) None of these
4x + 3y < 240
2x + y < 100

Question.15
A dietician wishes to mix together two kinds of food so that the vitamin content of the mixture
is at least 9 units of vitamin A, 7 units of vitamin B, 10 units of vitamin C and 12 units of
vitamin D. The vitamin content per Kg. of each food is shown below:

A B C D
Food 1: 2 1 1 2
Food II: 1 1 2 3

Assuming x units of food I is to be mixed with y units of food II the situation can be
expressed as:

Solution:

(a) 2x + y < 9 (b) 2x + y > 30


x+y<7 x+y<7
x + 2y < 10 x + 2y < 10
2x + 3y < 12 x + 3y > 12
x > 0, y > 0

(c) 2x + y > 9 (d) None of these


x+y>7
x + y < 10
x + 3y > 12

Question.16

An assembly of plant assembles refrigerators and T.V sets in addition to its regular
production so as to utilize the existing spare capacity profitably. The assembly is done in
two departments 1 and 2 department No. 1 has spare capacity of 210 hours and department
No.2 of 150 hours. Each unit of refrigerator takes 8 hours in department 1 and 3 hours in
departments 2 while each unit T.V sets takes 2 hours each in department 1 and department

28 | P a g e
2. The profit margins on each refrigerator and T.V sets are Rs.3000 and Rs.1000
respectively. How many units of refrigerator and T.V sets should be produced to earn the
maximum profit?

Solution:
Let
Number of units of refrigerator = x
Number of units of T.V sets = y
If profit function is P, then
P = 3000x + 1000y
Inequalities are
8x + 2y ≤ 210
3x + 2y ≤ 150

Required points are


B (26.25, 0), P (12, 57) and C (0, 75)
Now substituting the values in profit equation
P = 3000x + 1000y
We have
P (26.25, 0) = 3000 (26.25) + 1000 (0) = RS.78,750
P (12, 57) = 3000 (12) + 1000 (57) = RS. 93,000
P (0, 75) = 3000 (0) + 1000 (75) = Rs.75,000
Form the above analysis
x = 12 and y = 57
So production of 12 units of refrigerator and 57 units of T.V sets, would result in maximum
amount of profit of Rs.93, 000

Question: 17
An animal feed is prepared by mixing two products A and B. products A contain 5 units of
vitamin C while product B contains 8 units of vitamin C. each feed should not be more than
30kg in weight and should contain at least 180 units of vitamin C. the cost of products A
and B are Rs.3 per kilogram and Rs.6 per kilogram respectively. How much should
products A and B be mixed such that the cost of the feed be minimum?

Solution:
Let
The product A = x
The product B = y
Then cost equation is
C = 3x + 6y
Also inequalities are

29 | P a g e
x + y ≤ 30
5x + 8y ≥ 180
So the required points are
A (0, 30), P (20,10) and C (0, 22.5)

Now substitute these values in the cost function


C = 3x +6y is
C (0,30) = 3(0) + 6 (30) = Rs.180
C (20.10) = 3 (20) + 6 (10) = Rs.120
C (0, 22.5) = 3 (0) + 6 (22.5) = Rs.135
It clearly indicates that x = 20 and y = 10 is the minimum cost.

Question:18
A manufacturer has 170 hours of spare capacity per week in plant I and 115 hours in plant
2. He wants to produce two products A and B. product a takes 2 hours in plant I and 3
hours in plant 2 per unit, while product B requires 3 hours in plant I and 1 hours in palnt 2
per unit. If the marginal incomes per unit of a and B are Rs.20 and Rs.25 rtespectively, how
many units of each should be produced to maximize the profit?
Solution:
Let
Product A = x
Product B = y
Then
P = 20x + 25 y
Required inequalities are
2x + 3y ≤ 170 __ (i)
3x + y ≤ 115 ___ (ii)
From (i)
(0, 56.66) and (85 , 0)
From (ii)
(0, 115) and (38.33, 0)
Point of intersection of (i) and (ii)
X = 25 , y = 40
Put in
P = 20x + 25y
Hence profit will be maximum at
X = 25 , y = 40

30 | P a g e
Question.19
A manufacturer has 200 lb.’s of material 𝑴𝟏 and 290 lbs of material 𝑴𝟐 . He is producing
two products p and q. product p requires 4 lbs of 𝑴𝟏 and 3lbs of 𝑴𝟐 and product Q
required 2lb in 𝑴𝟏 and 5 lb in 𝑴𝟐 If product P and Q earn a marginal income of Rs.4 and
Rs.5 per unit respectively, how many units of each should be produced to maximize the
profit?

Solution:
Let product p = x
Product Q = y
Then
P = 4x +5y
Subject to
4x + 2y ≤ 200 ___(i)
3x + 5y ≤ 290 ___ (ii)
From (i)
(0, 100) , (50, 0 )
From (ii)
(0, 58) , (96.66 , 0)
Hence profit will be maximum at
X = 30, y = 40

Question.20
Masood toys company is manufacturing different variety of toys and is planning to
introduce two new types of toys cars and airplanes in the spare capacity of two machine.
Machine- I has spare capacity of 15 hours and machine -2 of 20 hours per week. Each cars
require 3 hours on machine –I and 2 hours on machine-2, while each aero plane requires 1
hours on machine-I and 4 hours on machine-2. The margin of profit on each unit of car
and aero plane are Rs.7 and Rs.9 respectively. What is the combination of cars and aero
planes to be produced and sold per week so as to earn maximum profit?

Solution:
Let
Car produced = x
Aero plane produced = y
Then
P = 7x + 9y
Subject to
3x + y ≤ 15 ___ (i)
2x + 4y ≤ 20 ____ (ii)

31 | P a g e
From (i)
(0, 15) and (5,0)
From (ii)
(0,5) and (10, 0 )
Profit will be maximum at x = 4 , y = 3

Question.21
The minimum weekly requirement for a person are 9 units of proteins and 16 units of
carbohydrates. Food F, contains 3 units of proteins and 5 units of carbohydrates. Per kg
and sells for Rs.9 per kg. Food 𝑭𝟐 contains 1 unit of proteins and 2 units of carbohydrates
per kg and sells for Rs.5 per kg. How many kilograms of each food should be the person
buy per week to satisfy his minimum requirement at minimum cost?
Solution:
Let
Units of 𝐹1 = x
Units of 𝐹2 = y
Then
C = 9x + 5y
Subject to
3x + y ≥ 9 __(i)
5x + 2y ≥ 16 ___ (ii)
From (i)
(0, 9) and (3, 0)
From (ii)
(0, 8) and (3.2, 0)
Hence minimum requirement is
X=2,y=3

Question: 22
A manufacturer produces two products A and B both the products go through two
processes. Product A requires 2 hours in process- I and 2 hours in process-2 while product
B requires one hour in process-I and 2 hours in process-2. Process-I has 104 hours and
process-2 has 76 hours available capacity each month. Profit on A is Rs.6 and B is RS.11
per unit. Assuming that all units that can be produced will be sold, how many of each type
off product should be manufactured so as to earn the maximum profit. What will be this
profit?

Solution:
Let the product A = x
The product B = y

32 | P a g e
Then
P = 6x + 11y
Subject
2x + y ≤ 104 __ (i)
2x + 2y ≤ 76 __ (ii)
From (i)
(0, 104) and (52, 0)
From (ii)
(0, 38) and (38, 0)
Profit will be maximum at x = 44 and y = 16
P = 6 (44) + 11 (16)
P = 164 + 176
P = 440

Question: 23
A small furniture manufacturing unit produces chair and tables. A chair requires 3 hours
on machine a and 2 hours on machine B, while a table requires 2 hours on machine A and 4
hours on machine B. the profit on a chair is Rs.42 per unit and the profit on each table is
Rs.36 per unit. If the machines operator for only 8 hours a day, how many of each product
should be produces maximize the profit? What will be the amount of the optimum profit?

Solution:
Let No. of chairs = x
No. of table = y
Maximize
P = 42x +36y
Subject to
3x + 2y ≤8 __ (i)
2x + 4y ≤ 8 __ (ii)
From (i)
(0, 4) and (4, 0)

Profit will be maximum at (2, 1) is


P = 42(2) + 36 (1)
P = 86 + 36
P = 120

Question: 24
A company is considering to produce two products x and y. product x requires 15 minutes
on the drill and two hours on the lather. Product y requires 30 minutes on the drill and

33 | P a g e
three hours on the lather. The profit on x and y per unit are Rs.70 and Rs.102 per unit
respectively. If the machines are limited to 8 hours per day, how many of each product
should be produced to maximize profit. What is the maximum daily profit?
Solution:
Let
Product one = x
Product two = y
Maximize
P = 70x + 102y
Subject to
15 30
x. 60 + y. 60 ≤ 8
𝑥 𝑦
+ 2 ≤ 8 __ (i)
4
Also
2x + 3y ≤8 __ (ii)
From (i)
(0, 16) and (32, 0)
From (ii)
(0, 2.6667) and (4, 0)

Question: 25
The minimum weekly requirements for a person are 9 units of proteins, 11 units of
carbohydrates and 8 units of fats. Food a contains 3 units of proteins, 5 units of carbohydrates
and 1 unit of fats per lb’s and is sold for Rs.9 per lbs. food B contains 1 unit of protein, 2 units
of carbohydrates and 3 units of fats per lbs, and is sold for Rs.5 per lbs. how many pounds of
each food should a person buy per week to satisfy his minimum requirements at minimum
cost?
Question: 26
A calculator company produces a scientific calculator and a graphing calculator. Long-
term projections indicate an expected demand of at least 100 scientific and 80 graphing
calculators each day. Because of limitations on production capacity, no more
than 200 scientific and 170graphing calculators can be made daily. To satisfy a shipping
contract, a total of at least 200 calculators much be shipped each day. If each scientific
calculator sold results in a $2 loss, but each graphing calculator produces a $5 profit, how
many of each type should be made daily to maximize net profits?
Solution:
Let
number of scientific calculators produced =x
number of graphing calculators produced =y
objective function

34 | P a g e
P = –2x + 5y

constraints,

x >0

and y > 0.

x > 100

and y > 80.

x < 200

and y < 170.

x + y > 200

The feasibility region graphs as:

When you test the corner points at (100, 170), (200, 170), (200, 80), (120, 80), and (100, 100),
you should obtain the maximum value of P = 650 at (x, y) = (100, 170). That is, the solution is
"100scientific calculators and 170 graphing calculators".

Question: 27
You need to buy some filing cabinets. You know that Cabinet X costs $10 per unit, requires
six square feet of floor space, and holds eight cubic feet of files. Cabinet Y costs $20 per
unit, requires eight square feet of floor space, and holds twelve cubic feet of files. You have
been given $140 for this purchase, though you don't have to spend that much. The office

35 | P a g e
has room for no more than 72 square feet of cabinets. How many of which model should
you buy, in order to maximize storage volume?

Solution:
Let
x: number of model X cabinets purchased
y: number of model Y cabinets purchased

Naturally, x > 0 and y > 0. I have to consider costs and floor space (the "footprint" of each
unit), while maximizing the storage volume, so costs and floor space will be my constraints,
while volume will be my optimization equation.

cost: 10x + 20y < 140


space: 6x + 8y < 72
volume: V = 8x + 12y

This system (along with the first two constraints) graphs as:

When you test the corner points at (8, 3), (0, 7), and (12, 0), you should obtain a maximal
volume of 100 cubic feet by buying eight of model X and three of model Y.

Question: 28

A company manufactures two types of stereo-system M and N each unit of system M


required 3 worker hours for process A and 2 workers hours for process B while each unit
of system N requires 4 workers hours in process A and six worker hours for process B.
Available skilled labor allows a maximum of 480 workers hours per week for process a
process A and 540 workers hour per week for process B. how many units of stereo system
36 | P a g e
may be produced to maximize the profit, if profit of each unit of M and N is Rs. 2,000 and
Rs. 5,000 respectively.

Solution:
. 𝑃 = 2,000𝑥 + 5,000𝑦

Constraints are

3𝑥 + 4𝑦 ≤ 480

2𝑥 + 6𝑦 ≤ 540

Solve

𝑥 = 72, 𝑦 = 66

Maximize profit is

𝑃 = 2,000(72) + 5,000(66)

= 14,400 + 330,060

P = 474,000

Question: 29

An animal feed is prepared by mixing two products A and B. product A contains 5 units of
vitamin c, while product B contains 8 unit of vitamin C. each feed should not be more than
30kg in weight and should contain at least 180 units of vitamin C. the product A and B are
Rs.3 per kg and Rs. 6 per kg respectively. How much should product A and B mixed such
that the cost of feed is minimum.

Solution:
let product A in kg= 𝑥

Product 𝐵 𝑖𝑛 𝑘𝑔 = 𝑦

Objective functions

𝐶 = 3𝑥 + 6𝑦

Constraints

Total weight 𝑥 + 𝑦 ≤ 30

Vitamin C 5𝑥 + 8𝑦 ≥ 180

Put values in cost function


37 | P a g e
𝐶(0, 30) = 3(0) + 6 (30) = 𝑅𝑠 180

𝐶(20, 10) = 3(20) + 6(10) = 𝑅𝑠. 120

𝐶(0, 22.5) = 3(0) + 6(22.5) = 𝑅𝑠. 135

Hence 𝑥 = 20, 𝑦 = 10

Is the minimum cost


Question: 30

A company makes two kind of leather belts. Belt A is high quality belt and belt B is lower
quality. The respective profit are Rs 4 and Rs 3 per belt. Each belt of type A requires twice
as much as a belt of type B and if all the belts were of type B. the company could make Rs.
1,000 per day. The supply of the leather is sufficient for only Rs. 800 belts per day (both) A
and B combine belt A is required a fancy buckle and only 400 per day available. There are
only 700 buckles available for belt B. what should be the daily production of belt A, belt B
and maximum profit respectively.

Solution:
let 𝑥1 = 𝑁𝑢𝑚𝑏𝑒𝑟 𝑜𝑓 𝑏𝑒𝑙𝑡 𝐴 𝑏𝑒 𝑝𝑟𝑜𝑑𝑢𝑐𝑒𝑑

𝑥2 = 𝑁𝑢𝑚𝑏𝑒𝑟 𝑜𝑓 𝑏𝑒𝑙𝑡 𝐵 𝑏𝑒 𝑝𝑟𝑜𝑑𝑢𝑐𝑒𝑑

Objective 𝑧 = 4𝑥1 + 3𝑥2

Constraints 2𝑥1 + 𝑥2 ≤ 1,000

𝑥1 + 𝑥2 ≤ 800

𝑥1 ≤ 400

𝑥1 ≤ 700

𝑥1 , 𝑥2 ≥ 0

Question: 31

The maximum weekly requirement for a person is 9 units of protein, 11 units of


carbohydrate and 8 units of fats. Find A contain 3 units of protein, 5 units of carbohydrate
and 1 unit of fat and is sold for Rs 9 per kg. Find B contain 1 unit of protein, 2 units of
carbohydrates and 3 units of fat per kg. and is sold for Rs. 3 per kg. How many kilograms
of each food should a person buy per week to satisfy his minimum requirement at
minimum cost?

38 | P a g e
Solution:
(19/8,15/8)

Question: 32

A manufacturer has 110 hours of spare capacity per week in plant 1 and 115 hours in plant
2. He wants to produce two product “A” and “B”. product “A” takes 2 hours in plant 1 and
3 hours in plant 2 per unit. While product “B” requires 3 hours in plant 1 and 1 hour in
plant 2 per unit. If the marginal income per unit of “A” and “B” are Rs. 20 and Rs. 25
respectively. How many units of each should be produce to maximize product?

Solution:
(25, 40)

Question: 33

The relationship between number of beer consumed (𝒙) and blood alcohol (𝒚)was studied
in 16 male college students. By using least square regression the following regression
equation obtain from this study.

𝒚 = 𝟎. 𝟎𝟏𝟐𝟕 + 𝟎. 𝟎𝟏𝟖𝟎𝒙

Solution:
The above equation implies that each beer consumed increase blood alcohol by exactly 0.0180.

39 | P a g e
EXTRA PRACTICE QUESTION

40 | P a g e
Question:1

Inequalities x ≥ 0 and y ≥ 0 suggest that feasible region will be in _____ quadrant.

(a) 1st (b) 2nd


(c)3rd (c) 4th
Question:2

Which of the following values of x and y does not satisfy the inequality 2x+3y ≤ 6

(a) (1,1) (b) (0,1)


(c) (3,1) (d) (1,0)
Question:3

A machine can work for 16 hours per day at most. It takes 4 hours to produce one unit of product A and
8 hours to produce one unit of product B. This information can be expressed as:

(a) 4a + 8b≤16 (b)4a + 8b≥16


(c)4a + 8b = 16 (d) Not possible to express information
Question:4

Which of the following constraint will have a feasible region with coordinate (2,3) in it?

(a)10x+5y≤40 (b)10x + 5y≤30


(c)10x+5y≤10 (d)10x + 5y≤20
Question:5

Pharma limited produces two medicines. Each of which requires two types of raw material with
following detail:

Product A(kg/unit) Product B(kg/unit) Total available (kg)


Raw material X 2 3 120
Raw material Y 4 6 360
Form appropriate constraints as per above information

(a)2a+3b≤120 and 4a+6b≤360 (b)2x+4y≤120 and 3x+6y≤360


(c)4a+6b≤120 and 2a+3b≤360 (d)2x+3y≤120 and 4a+6b≤360
Question:6

The maximum number of constraints while solving a business problem using linear programming is

(a)1 (b)2
(c)3 (d)There is no specific limit
Question:7

Feasible region is a ______

41 | P a g e
(a)Region which satisfies all given (b)Region which is in first quadrant
constraints
(c)Region above the last-mentioned (d)Always below all the constraints in a given
constraint scenario
Question:8

Corner point theorem states _____

(a)Optimal solution is at one of the corners (b)Optimal solution is within feasible region
of feasible region
(c)Profit is maximum at all corners of a (d)Revenue is maximum at all corners of a
feasible region feasible region
Question:9

Linear programming model can be used for ___

(a)Profit maximization (b)Cost minimization


(c)Both a and b (d)None
Question:10

For the following set of inequalities and objective function. Identify the optimal solution using corner
point theorem

Constraints
1. x≥0
2. y≥0
3. x+ y≤8
Objective function
Profit = 10x+2y
(a) (8,0) (b) (0,8)
(c) (4,4) (d) (0,0)
Question:11

For the following set of inequalities and objective function. Identify the optimal solution using corner
point theorem

Constraints

1. x≥0
2. y≥0
3. 2x+y≤8
4. 2x+2y≤10
Objective function
Profit = 10x + 2y
(a) (8,0) (b) (5,0)
(c) (4,4) (d) (0,0)

42 | P a g e
Question:12

For the following set of inequalities and objective function. Identify the optimal solution using corner
point theorem

Constraints

1. X≥1
2. Y≥2
3. X+ y≤8
Objective function
cost = 10x + 2y
(a) (0,0) (b) (1,2)
(c) (2,1) (d) (0,0)
Question:13

Identify the redundant constraints from the following set of constraints:

1. x+ y≤6
2. x≥0
3. y≥0
4. 4x + 2y≤8
(a)4x + 2y≤8 (b) x≥0
(c) y≥0 (d) x+ y≤6
Question:14

A company wants to make product A which will contain both chemical x and chemical y. The product
must contain at least 1 units of x and 2 units of y. The company has to be careful that total of x and y
units in the product must not exceed 8 units.

If the cost of each unit of x and y is 10 and 2 respectively, formulate constraints and objective function
respectively.

(b) constraints
(a) Constraints 1. x≥1
1. x≥1 2. y≥2
2. y≥2 3. x + y≤8
3. x + y≤8 Objective function
Objective function Cost = 2x + 10y
Cost = 10x + 2y
(c) Constraints (d) Constraints
1. x≥0 1. X≥1
2. y≥0 2. Y≥2
3. x + y≤8 3. X + y≤2
Objective function Objective function
Cost = 10x+2y Cost = 10x + 2y

43 | P a g e
Question:15

A company produces two types of cola drinks. Each of which requires two types of raw material with
following detail:

Product A(kg/unit) Product B(kg/unit) Total available (kg)


Raw material X 3 4 480
Raw material Y 2 6 540
How many units of each type of cola drink may be produced to maximize the profit, if the profit on each
unit of A and B is Rs 1,800 and Rs. 6,300 respectively?

(a) (72,66) (b) (0,90)


(c) (160,0) (d) (66,72)
Question:16

For the following constraints:

1. x + y≥40
2. x + y≤10
(a)There is no feasible region (b) One feasible region
(c) Two feasible regions (d) Three feasible regions
Question:17

Minimum number of constraints to draw a feasible region is/are:

(a)1 (b)2
(c)3 (d)4
Question:18

A bounded feasible region will:

(a) Have both maximum and minimum value (b) Have maximum value only
(c)Have minimum value only (d)None of these
Question:19

y≤x will have a feasible region: ___

(a)Above y axis only (b) Below y axis only


(c)Above X axis only (d) Below or on the line y = x

44 | P a g e
1/1/2022 CHAPTER-5
FINANCIAL
MATHEMATICS
SIMPLE INTEREST

1-INTEREST
Fee for the use of money is called interest

2-TIME VALUE OF MONEY


The value of money received today is different from the value of money received after some
time in the future. So, value of money is time dependent

SIMPLE INTEREST
Simple interest is paid or earned on the principal amount lent or borrowed

Interest = I = Prt
And

P = amount borrowed = principal


r = rate of interest
t = time

Also

Future Amount = S = P (1 + rt)

Present Amount = P = S/ (1 + rt)

Note:

i- Unit(time) must be same in r and t


ii- When use time add fee
iii- When do not use time reduce fee?
iv- Amount borrow is called principal amount
v- Amount after time is called future amount

1|Page
Question.1-2003 -K-6 A person borrowed Rs. 20,000 from a bank at a simple
interest rate of 12 percent per annum. In how many years will he owe
interest of Rs.3600?

A. 2.3 B. 1.6 C. 1.4 D. 1.5

Solution
Given
P = 20,000
r = 12%
t =?
I = 3600
Using
I = Prt
3600 = 20,000 x 0.12 x t
3600 = 2400t
3600
=t
2400
1.5 = t

Question.2 The sum required to earn a monthly interest of Rs.1200 at 18% per
annum S.I is:

A. 78000 B. 79000 C. 80000 D. 81000

Solution
Given
P =?
I = 1200
r = 18% = 0.18
t = 1/12
Using
I = Prt
1200 = P x 0.18 x 1/12
1200 = P(0.015)
1200/0.015 = P
80,000 = P

Question.3-k-9 In what time will Rs.1800 yield simple interest of Rs.390 at the rate
of 5% per annum is.

A. 54 B. 52 C. 53 D. 50

Solution
Given
t =?
P = 1800
I = 390
r = 5% = 0.05
Using

2|Page
I = Prt
390 = 1800 x 0.05 x t
390 = 90t
390/90 = t
t = 4 years 4 months

Question.4-k-5 A bank charges mark-up (interest) @ Rs. 0.39 per day per
Rs. 1000/- Find rate of mark-up as percent per annum is:

A. 14.56% B. 13.67% C. 14.24% D. 15.56%

Solution
Given
I = 0.39 x 365= Total mark up for whole year
= 142.35
P = 1000
t=1
r% = ?
Using
P rt
I=
100 r
142.35 = 1000 x 100 x 1
142.35
=r
10
r = 14.24%

Question.5- k-13 A total of Rs.14000 is invested for a year, part at 5% and the rest at
6%. If Rs.740 is the total interest, amount invested at 5% is.

A. 15000 B. 16000 C. 10000 D. 17000

Solution
Given
I1 +I2 = 740
Let

Investment one investment two


P=x P=14000-x
r=5% r=5%
t=1 t=1
using
I1 = P x r x t I2 = Prt
5 6
I1 = x x 100 x 1 I2 = (14000 - x) x 100 x 1
I1 = 0.05x I2 = 0.06 (14000 - x)

According to statement
Total interest = I1 + I2
740 = 0.05x + 0.06 (14000 – x)
740 = 0.05x + 840 – 0.06x

3|Page
740 – 840 = -0.01x
-100 = -0.01x
−100
x = −0.01
x = Rs.10, 000
𝟏
Question.6- k-14 If the simple interest on a certain sum for 15 months at 7 𝟐% per
𝟏
annum exceeds the simple interest on the same sum for 8 months at 12 𝟐% per annum
by Rs. 32.50, then the sum (in Rs) is

A. 3167 B. 3425 C. 3120 D. 2345

Solution
Given
Investment I Investment II
P=x P=x
r = 7.5% = 0.075 r = 12.5% = 0.125
15
t = 12 years t = 8/12 years
I1 =? I2 =?
Using Using
I1 = Prt I2 =Prt
I1 = (x) (0.075) (15/12) I2 =(x)(0.125) (8/12)
I1 = 0.09375x I2 = 0.083333x

According to statement
I1 = I2 + 32.50
0.09375x = 0.083333x + 32.50
0.010417x = 32.50
x = 3120

Question.7- k-16 A sum of Rs.7700 is to be divided among three brothers Zain, Zaid and
Zohaib in such a way that simple interest on each part at 5% per annum
after1,2 and 3 years respectively remains equal. The share of Zain is more
than that of Zohaib by:

A. 4100 B. 4200 C. 4300 D. 4400

Solution
Given
Total amount to be divided = 7700
Let
Zain share = x
Zaid share = y
Zohaib share = z
x+y+z =7700 -------(i)
r = 5% = 0.05

Zain Zaid Zohaib


P=x P=y P=z
I1 =? I2 =? I3 =?

4|Page
t = 1 year t = 2 year t = 3 year
r = 5% = 0.05 r = 5% = 0.05 r = 5% = 0.05
Using Using Using
I1 = Prt I2 = Prt I3 = Prt
I1= x x 0.05 x 1 I2 = y x 0.05 x 2 I3 = z x 0.05 x 3
I1 = 0.05x I2 = 0.10y I3 = 0.15z

According to statement
I2 = I2 = I3
0.05x = 0.10y = 0.15z
5 10 15
x = 100 y = 100 z
100
1 1 3
x = 10 y = 20 z
20
1 3
x=y= z
2 2
x 3
=y = 2 z
2
x = 2y -------- (i)
2y = 3z ------- (ii)
x = 3z ---- (iii)
Also
x + y + z = 7700 ---- (iv)
from (i)
y = x/2
from (iii)
𝑥
=z
3
Put in (i)
x x
x + 2 + 3 = 7700
6x + 3x + 2x = 7700 x 6
11x = 7700 x 6

Zain share = x = 4200


4200
Zohaib share Z = 3 = 1400
x 4200
Zaid share = Y= 2 = = 2100
2

Question.8- k-17 A person borrowed Rs. 500 @ 3% per annum S.I and 600 @ 4.5%
per annum on the arrangement that the whole sum will be returned only when the total
interest becomes Rs. 126. The number of years, after which the borrowed sum is to be
returned is:

A. 2 B. 3 C. 4 D. 5

Solution
Given
Investment I Investment II
P1 = 500 P2 = 600
r1 = 3% = 0.03 r2 = 4.5% = 0.045
t1 = t t2 = t year

5|Page
I1 =? I2 =?

Using Using
I1 = Prt I2 = Prt
I1 = 500x0.03xt I2 = 600 x 0.045 x t
I1 = 15t ----- (1) I2 = 27t ------- (2)

According to statement
I = I1 + I2
126 = 15t + 27t
126 = 42t
126
t
42
t=3

Question.9- k-18 A lend Rs. 2500 to B and a certain sum to C at the same time at 7%
p.a. Simple interest. If after 4 years. A altogether receives Rs. 1120 as interest from B
and C, then the sum lent to c is.

A. 1200 B. 1300 C. 1400 D. 1500

Solution
Total interest = 1120
Given
Investment I Investment II

P1 = 2500 P2 = x
r = 7% = 0.07 r = 7% = 0.07
t1 = 4 years t2 = 4 year
Using Using
I1 = Prt I2 = Prt
I1= 2500 x 0.07 x 4 I2= x x 0.07 x 4
I1 = 700 I2 = 0.28x

According to statement

I = I1 + I2
1120 = 700 + 0.28x
1120 - 700 = 0.28x
420
=x
0.28
x = 1500

Question.10- k-19 An investor receives a total of Rs. 5700 per annum in interest from 3
stock yielding 4%, 5% and 8% per annum respectively. The amount at 4% is Rs. 20,000
more than the amount invested at 5% and the interest from the 8% investment is 8
times the interest of 5%. Amount invested in each stock is.

A. 10.20,30 B. 20,30,40 C. 30,10,50 D. 20,45,60

6|Page
Solution

Total amount of interest = 5700

Investment- I at 4% Investment- II at 5% Investment -III at 8%


Let
P1 = x + 20,000 P2 = x I3 = 8 x I2
r1 = 4% = 0.04 r2 = 5% = 0.05 = 8 x 0.05x
t1 = I year t = I year I3 = 0.4x
I1 =? I2 = ?

Using Using
I1 = Prt I2 = Prt
I1 = 0.04(x + 20,000) I2= x x 0.05 x 1
I2 = 0.05x

According to statement
I = I1 + I2 + I3
5700 = 0.04(x + 20,000) + 0.05x + 0.4x
5700 = 0.04 + 800 + 0.05x + 0.4x
5700 = 0.49x + 800
5700 - 800 = 0.49x
4900
=x
0.49
P2 =x = 10,000 at 5% investment
Also
P1 =Amount investment at 4% = x + 20,000
= 10,000 + 20,000
= 30,000
Now we find P3
I3 = P3 r3 t3
0.4x = P3 x 0.08 x 1
0.4 x 10,000 = 0.08P3
P3 = 50,000 at 8% investment

Question.11-2003 An individual has purchased Rs. 275,000 worth of defence saving


certificate. The certificate expires in 25 years and s simple interest rate is
computed quarterly at a rate of 3 percent per quarterly. Interest checks
are mailed to certificate holders every 3 months. Determine the interest
the individual can expect to each every three months. What amount can
be expect to receive at the end of 25 years?

A. 500000 B. 700000 C. 1100000 D. 1300000

Note: find first quarter interest and then multiple by 4 and then by 25
Solution
Given
P = Rs. 275,000
r = 3% per quarter
t = 1 quarter

7|Page
Using
I = Prt
3
= 275000 x 100 x 1
= 8250 amount of interest after three months
Interest after 25
Years = (8250)(4)(25)
= 825,000
Total amount at the end of 25 years = 275,000 + 825000
= Rs. 11,00000

Question.12 The simple interest on a certain sum for 8 months at 4% per annum is Rs.129
less than the simple interest on the same sum for the 15 months at 5% per
annum. What is the sum?

A. B. C. D.

Solution
Given
According to statement
I1 = I2 ‒ 129
Prt = Prt-129
8 15
P × 0.04 × =P × × 0.05 − 129
12 12
0.0266P = 0.0625 x P - 129
0.0625P- 0.0266P = 129
0.0359P = 129
P= 3593

Question.13 At what rate of simple interest per annum does a person get an interest of
Rs. 4706.1 on the principal amount of Rs.11205 after 7 years?

A. B. C. D.

Solution
Given
r =?
I = 4706.1
P = 11205
t=7
Using
I = Prt
4706.1 = 11205 × r × 7
r = 6%

Question.14 Pinky invested an amount of Rs.24500 at the rate of 9% per annum after how
many years will she get a simple interest of Rs.37485.

A. B. C. D.

8|Page
Solution
Given
P = 24500
I = 37485
r = 9%
t =?
Using
I = Prt
37485 = 24500 × 0.09 × t
37485 = 2205t
t = 17 years

Question.15 If total amount of simple interest on a sum of money at the rate of 8% per
annum in 4 years is Rs. 571.20, then what is principal.

A. B. C. D.

Solution
Given
I = 571.20
r = 8%
t=4
P =?
Using
I = Prt
571 = P × 0.08 × 4
571
P=
0.32
P = 1785

Question.16 The rate of simple interest on a sum of money is 6% per annum for the last 3
years. 8% per annum for the next 5 years and 10% per annum for the period
beyond 8 years. If the simple interest accrued by the sum for total period of 10
years is Rs.1560. What is the sum in first investment

A. B. C. D.

Solution
Given
Investment – I Investment - II Investment - III
p=x p=x P=x
r = 6% r = 8% r = 10%
t = 3 years t=5 t=2
Using
I1 = Prt I2 = Prt I3 = Prt
I1 = x × 0.06 × 3 I2 = x × 0.08 × 5 I3 = x × 0.1 × 2
I1 = 0.18x I3 = 0.4x I3 = 0.2 x

According to statement
I1 + I2 + I3 = 1560

9|Page
0.18x + 0.4x + 0.2x = 1560
0.78x = 1560
x = 2000

Question.17 Mr. Amir made an investment in two schemes A and B. Investment in scheme A
is Rs.2000 more than in scheme B. Investment in Scheme A carries 6% rate of
interest whereas investment in scheme B carries 8% rate of interest. What is the
sum invested in scheme A, if the difference in interest in 3 years is Rs. 120?

A. B. C. D.
Solution
Given
Investment – A Investment – B
p = x + 2000 p=x
r = 6% r = 8%
t=3 t=3
Using Using
I1 = Prt I2 = Prt
= (x + 2000) × 0.06 × 3 I2 = x × 0.08 × 3
I1 = 0.18x + 360 I2 = 0.24x

According to statement
I1 – I2 = 120
0.18x + 360 – 0.24x = 120
‒0.06x = ‒240
x = 4000
Investment is scheme A = x + 2000
= 4000 + 2000
= 6000
Question.18 Mr. Mustafa invested Rs.10000 in scheme A at 8% interest and Rs.15000 in
scheme B carrying interest at x%. If the interest for the first year is Rs.2150 for
both the investment, the investment in scheme B carries what rate of interest.

A. B. C. D.

Solution
Given
Scheme – A Scheme - B
P = 10000 P = 15000
r = 8% r = x%
t=1 t=1
I1 =? I2 =?
According to statement
I1 + I2 = 2150
Prt + Prt = 2150
10,000 × 0.08 × 1 + 15000 × x% × 1 = 2150
800 + 15000 x% = 2150
15000 x% = 1350
x% = 0.09
x = 9%

10 | P a g e
Question.19 The interest on a certain deposit at 4.5% per annum is Rs.202.5 in one year.
How much will additional interest in one year be on the same deposit at 5% per
annum is.

A. B. C. D.

Solution
Given Also
P=x P = 4500
r = 4.5% r = 0.05
t=1 t=1
I1 =? I2 =?
Using Using
I1 = Prt I2 = Prt
202.5 = x × 0.045 × 1 = 4500 × 0.05 × 1
P=x = 4500 I2 = 225

So the additional interest will be =I2-I1 =225-202.5=22.5

Question.20 A what rate percent per annum will the simple interest on a sum be 1/4 of the
amount in 16 years.

A. B. C. D.

Solution
Given
1
I= x = 0.25x
4
P=x
r =?
t =16
Using
I = Prt
0.25x = x × r × 16
0.25
r=
16
r = 1.5625%

Question.21 At what rate percent per annum will the simple interest on a sum of money be
2/5 of the amount in 10 years?

A. B. C. D.

Solution
Given
P=x
2
I= x
5
t = 10 years
Using

11 | P a g e
I = Prt
2
x = x × r% × 10
5
0.4 = 10r
0.4
r=
10
r = 4%

Question.22 In how many years RS.150 will produce the same interest at 8% as 800 produce
in 3 years at 9/2%.

A. B. C. D.

Solution

Option- I Option- II
Given Given
P = 150 P = 800
r = 8% r = 9/2%
t =? t =3

According to statement
I1 = I2
Prt = Prt
150 × 0.08 × t = 800 × 0.045 × 3
12t = 108
t = 9

Question.23 A lent Rs.8000 to B for 2 years and Rs.4000 to C for 4 years on simple interest at
the same rate of interest and received Rs.3840 in all from both of them as
interest. The rate of interest per annum is.

A. B. C. D.

Solution

Investment - I Investment - II
Given A----B Given A------C
P = 8000 P = 4000
t=2 t=4
r =? r =?
I1 =? I2 =?

According to statement
I1 + I2 = 3840
Prt + Prt = 3840
8000 × 2 × r + 4000 × 4 × r = 3840
16000r + 16000r = 3840
32000r = 3840
r = 0.12

12 | P a g e
r = 12%

Question.24 On the day you enter college you borrow Rs.18000 from your local bank. The
terms of the loan include an interest rate of 5.75 percent. The term stipulate that the principal is
due in full one year after you graduate. Interest is to be paid annually at the end of each year.
Assume that you complete college in four years. How much total interest will you pay on this
loan?

A. B. C. D.

Solution
Given
P = 18000
t=5
r = 5.75
I =?
Using
I = Prt
I = 18000 × 0.0575 × 5
I = 5175

13 | P a g e
FUTURE VALUE:
Question.25- K-3 Future value of Rs. 1,355/- invested@8% P.a for 5 year is:

A. 1900 B. 1950 C. 1897 D. 1800

Solution
Given
S =?
P = 1355
r = 8%
t=5
Using
S = P (1 + rt)
S = 1355 (1 + 0.08 x 5)
S = 1355 (1 + 0.4)
S = 1355 (1.4)
S = 1897

Question.26 Mr. Ahmed secures (borrow) a loan of Rs. 1500 from a bank for four
months at 9%. How much amount he will have to repay after the expiry
of the term?

A. 1460 B. 1545 C. 1345 D. 1654

Solution
Given
P = 1500
r = 9%
t = 4/12 years
S=?
Using
S = P (1 + rt)
S = 1500(1 + 0.09 x 4/12)
S = 1500(1.03)
S = 1545

Question.27-K-12 A sum of money would double itself in 10 years. The number of


years it would be four times is

A. 35 B. 53 C. 36 D. 30

Solution
Given
S = 2p
P=P
t = 10
r =?
Using
S = P (1 + rt)

14 | P a g e
r
2P = P (1 + x 10)
100
2 = 1 + 0.1r
1 = 0.1r
r = 10%
Also
4P = P (1 + 0.1t)
4 = 1 + 0.1t
4 – 1 = 0.1t
3 = 0.1t
3
=t
0.1

t = 30 Years

Question.28 –K-7 How long will it take for a sum of money to double itself at 10%
simple interest?

A. 12 B. 13 C. 14 D. 10

Solution
Given
t =?
S = 2P
r = 10% = 0.1
P=P
Using
S = P (1 + rt)
2P = P (1 + 0.1 x t)
2 = 1 + 0.1t
2-1 = 0.1t
1 = 0.1t
t = 10 Years

Question.29 Mr. Akram owes Rs. 2000 due in 3 months and Rs. 4000 in 7 months. He
and his creditor agreed to settle the debts by 2 equal payments, one in 5
months and the other in 11 months. What will be the amount of each
payment if money is worth 6% and is maturity date is 5 months from
now?

A. 3045.54 B. 3034.30 C. 3425.76 D. 4328.76

Solution
Due Due
2000 4000
X X

0 3 5 7 11
Payment I Payment II

15 | P a g e
Debt Debt Payment Payment
Given P=? Total Transfer value of 11 months at 5
P = 2000 S=6% payme months, using
r = 6% r=2/12=1/6 nt S = P(1 + rt)
t = 2/12 = 1/6 years S =? using at 5 months 6
x = P(1 + 0.06 x 12)
Using S=P( 1+ rt) is
x = P(1.03)
S = P(1 + rt) 4000=P(1+0.06*1/6) P=x 𝑥
4000=P(1.01) =P
1.03
S= 2000(1 + 0.06 x 1/6) 4000/1.01 =P 0.9708x = p
P=3960.396
S=2020

Total debt at 5 months


= 2020 + 3960.396
= 5980 ------ (1)
Also for equal payment convert all the payments at single comparison point i.e. transfer all
values at comparison factor and compare.

Also
Total payment at 5 months is
= x + 0.9708x
= x(1 + 0.9708)
= x(1.9708) -------------------(2)

Comparing 1 and 2
Total debt =Total payment
5980 = x(1.9708)
x = 3034.30

Question.30 Mr. Akbar invested two sums of money on simple interest. The first was
Rs. 2200 invested at 8% per annum and the second Rs. 1960 at 10% per
annum. The two sums were allowed to grow till both amounted to the
same figure. In how many years did this happen and what was the final
amount.

A. 11 B. 12 C. 13 D. 14

Solution
Given
Investment I Investment II
P = 2200 P = 1960
r = 8% r = 10%
t =? t =?
S1=? S2=?
Using
S1 = P(1 + rt) S2 = P(1 + rt)
= 2200(1 + 0.08t) = 1960(1 + 0.10t)

According to statement

16 | P a g e
S1 = S2
2200(1 + 0.08t) = 1960(1 + 0.10t)
2200 + 176t = 1960 + 196t
2200 - 1960 = 196t - 176t
240 = 20t
t = 12 years

Question.31 Mr. Sameer opened two accounts to grow up by simple interest. The first
account with a principle of Rs. 2500 grew at a 8% annually and the
second account with a principal of Rs. 4400 at 10% annually till such time
that the second amount become double of the first amount. In how many
years this happened and what were the two amounts.

A. 13 B. 14 C. 15 D. 16

Solution
Given
Investment I Investment II
P = 2500 P = 4400
r = 8% r = 10%
t =? t =?
S1=? S2=?
Using
S = P(1 + rt) S = P(1 + rt)
= 2500(1 + 0.08t) = 4400(1 + 0.6t)
According to statement
2A = B
2[2500(1+0.08t)] = 4400[1 + 0.01t]
2[2500 + 200t)] = 4400 + 440t
5000 + 400t = 4400 + 440t
600 = 40t
t = 15 years

Question.32 A man wants to sell his scooter. There are two offers one at Rs.12000 cash and
the other at a credit of Rs.12880 to be paid after 8 months, money being at 18%
annum. Which is the better offer?

A. B. C. D.

Solution
Given
Offer – I Offer – II
Cash = 12000 S = 12880
t = 8 months
r = 18%
P =?

17 | P a g e
Using
S = P(1 + rt)
8
12880 = P(1 + 0.18 x 12)
12880 = P(1 + 0.12)
12880 = P(1.12)
P = 11500

Offer-I is better

Question.33 A certain sum of money lent out at S.I amount to Rs.690 in three years and
Rs.750 in 5 years. The sum lent is.

A. B. C. D.

Solution
Given
Investment I Investment II
P=x P=x
S= 690 S = 750
t=3 t=5
r =? r =?
Using Using
S = P (1 + rt) S = P (1 + rt)
690 = x (1 + 3r) - (i) 750 = x (1 + 5r) (ii)

Dividing (i) and (ii)


750 x(1 + 5r)
=
690 x (1 + 3r)
1 + 5r
1.0869 =
1 + 3r
(1.0869) (1 + 3r) = 1 + 5r
1.0869 + 3.2607r = 1 + 5r
1.0869 - 1= 5r – 3.2607r
0.0869 = 1.7393r
r = 0.0499
Also
690 = x [1 + 3 (0.0499)]
690 = x (1.497)
x = 600

Question.34 The rate at which a sum becomes four times itself in 15 years at S.I will be.

A. B. C. D.

Solution

Given
P=P
S = 4P

18 | P a g e
r =?
t = 15 years
Using
S = P(1 + rt)
4p = P(1 + 15r)
4 = 1 + 15r
3 = 15r
3
=r
15
r = 20%

Question.35 Bilal invested a certain sum at 8% p.a. simple interest for n years. At the end of
n years, he got back four times his original investment. What is the value of n?

A. B. C. D.

Solution
Given
P=x
S = 4x
r = 8%
t=n
Using
S = P(1 + rt)
4x = x(1 + 0.08n)
4 = 1 + 0.08n
3 = 0.08n
3
=n
0.08
n = 37.5

Question.36 A sum of money is invested for a certain number of years at 8% per annum
simple interest grows to Rs.180. The same sum of money invested for the same
number of years at 4% per annum simple interest grows to Rs.120 only. The
sum of money has been invested for.

A. B. C. D.

Solution
Given
Investment I Investment II
P=x P=x
S = 180 S = 120
r = 8% r = 4%
t =? t =?
Using Using
S = P (1 + rt) S = P(1 + rt)

19 | P a g e
180 = x (1 + 0.08t) (i) 120 = x (1 + 0.04t) (ii)

Dividing (i) and (ii)


180 1 + 0.08t
=
120 1 + 0.04t
3 1 + 0.08t
=
2 1 + 0.04t
3(1 + 0.04t) = 2 (1 + 0.08t)
3 + 0.12t = 2 + 0.16t
1 = 0.04t
t = 25 years

Question.37 Rs.1200 becomes Rs1569 in 3 years at a certain rate of simple interest. If the rate
of interest is increased by 5.75%. What amount will Rs.800 become in 4 years.

A. B. C. D.

Solution
Given
P = 1200
S = 1569
r =?
t=3
Using
S = P(1 + rt)
1569 = 1200 (1 + 3r)
1.3075 = 1 + 3r
0.3075 = 3r
r = 0.1025
r = 10.25%
Also
r = 10.25% + 5.75% = 16%
P = 800
t=4
S =?
Using
S = P (1 + rt)
S = 800 (1 + 4 × 0.16)
S = 1312

Question.38 We have total amount of Rs.2379, now, divide this amount in three parts so that
their sum becomes equal after 2, 3 and 4 years respectively. If rate of interest is
5% per annum then first part will be?

A. B. C. D.

Solution
Given
Total amount = 2379

20 | P a g e
∴ x + y + z = 2379 (i)
According to statement as sums are equal
∴ S1 = S2 = S3
P(1 + rt) = P(1 + rt) = P(1 + rt)
x(1 + 0.05 × 2) = y(1 + 0.05 × 3) = z(1 + 0.05 × 4)
1.1x = 1.15y = 1.2z
11x 115y 12z
= =
10 100 10
110x = 115y = 120z
Comparing
110x = 115y
110
y= x
115
…y = 0.9565x…
Also
110x = 120z
110x
= z
120
…Z = 0.9166x…
Putting value of ‘y’ and ‘z’ in (i)
x + 0.9565x + 0.9166x = 2379
2.8731x = 2379
x = 828

21 | P a g e
PRESENT VALUE:

Question.39-2008 How much should an individual deposit now to yield Rs. 600,000 at
the end of five years at 10% simple interest.

A. 340,000 B. 410,000 C. 400,000 D. 425,000

Solution
Given
S = 600,000
t = 5 years
P =?
r = 10%
Using
S = P (1 + rt)
600,000= P (1 + 0.1 x 5)
600,000 = P (1.5)
600,000
P = 1.5
P = 400,000

Question.40-2005 Bashir owes (Return) Rs. 50,000 to Arshad due to a court decision.
The money must be paid in 10 months with no interest. Suppose Bashir
wishes to pay the money now. What amount should Arshad be willing to
accept? Assume simple interest of 8% per annum.

A. 54675 B. 34267 C. 46904 D. 34259

Solution
Given
S = 50,000
r = 8%
t = 10/12 years
P =?
Using
S = P (1 + rt)
10
50,000 = P (1 + 0.08 x 12)
50,000 = P (1.066)
P = 46904

Question.41-k-4 The present value of Rs. 1400 at 8 percent simple interest for 5
years is:

A. 1050 B. 1200 C. 1500 D. 1000

Solution
Given
P =?
S = 1400

22 | P a g e
r = 8%
t=5
Using
S = P (1 + rt)
1400 = P (1 + 0.08 x 5)
1400
=P
1.4
1000 = P

Question.42-k-11 A sum of money would amount to Rs. 6200 in 2 years and Rs. 7400
in 3 years. The principal and rate of simple interest are.

A. 4100 B. 3800 C. 4200 D. 4300

Solution
Given
S = 6,200
t=2
r =?
P =?
Using
S = P (1 + rt)

6200 = P(1 + 2r) -------- (i)


Also
S = 7400
t=3
r =?
P =?
Using
S = P (1 + rt)

7400 = P(1 + 3r) --------- (ii)

Divide (i) and (ii)

7400 P(1+3r)
= P(1+2r)
6200
74(1 + 2r) = 62(1 + 3r)
37(1 + 2r) = 31(1 + 3r)
37 + 74r = 31 + 93r
37 – 31 = 93r – 74r
6 = 19r
r = 6/19
r = 31.57% = (0.3157)

Put in equation (i)

6200 = P(1 + 2 x 0.3157)


6200 = P(1 + 0.6314)
6200 = P(1.6314)

23 | P a g e
P = 3800

Question.43-k-10-2012 An amount of Rs. 20,000 is due in three months. What is the


present value if it includes simple interest @ 8%?

A. 19650 B. 19607 C. 18976 D. 17687

Solution
Given
S = Rs. 20,000
P =?
r = 8% = 0.08
t = 3/12 years
Using
S = P (1 + rt)
3
20,000 = P (1 + 0.08 x 12)
20,000 = P (1.02)
20,000
=P
1.02
19607 = P

Question.44 How much Mr. Dawood borrow from the bank so that he may have to
repay Rs. 4,425 after 3 years if the bank charges a simple interest rate of
6%.

A. 3760 B. 4320 C. 3750 D. 3758

Solution
Given
S = 4425
t = 3 years
r = 6%
P=?
Using
S = P(1 + rt)
4425 = P(1 + 0.06 x 3)
4425
1.18
=P
P = 3750

Question.45 Mrs. Bashir plans to purchase a T.V set. She has been offered the option
of paying Rs. 2,000 down payment and Rs. 3000 after 4 months, or of
paying Rs. 3000 down payment and Rs. 2000 after 5 months. If the rate of
interest is 9%. Which option would be better for her?

A. Ist B. 2nd C. 3rd D. 4th

Solution

Offer I Offer II

24 | P a g e
Down payment = 2000 Down payment = 3000
S = 3000 S = 2000
r = 9% t = 5/12 year
t = 4/12 = 1/3 years r = 9%
P =? P =?

Using Using

S = P(1 + rt) S = P(1 + rt)


S 5
P = 1+rt 2000 = P(1 + 0.09 x 12)
3000 2000
= 1+0.03 =P
1.03
P = 2912 P = 1941

Cash price Cash price


= Down payment + p = Down payment + p
= 2000 + 2912 = 3000 + 1941
= 4912 = 4941

Hence 1st offer would be better

Question.46 A debt of Rs. 2000 is due in 6 months. If the rate of interest is 6%. What is
the value of the debt if it is paid after (i) 4 months (ii) nine months from
now?

A. 1962 B. 1960 C. 1961 D. 1963

Solution
(i)
Given
S = 2,000
P =?
r = 6%
t = 2/12 = 1/6 years
Using
S = P(1 + rt)
S
P = 1+rt
2000
= 1+0.06 x 1/6
2000
=
1.01
P = 1980

(ii)

Given

P = 2,000
r = 6%
t = 2/12 =1/6 years (next three month because 2000 due in 6 months)

25 | P a g e
S =?

Using

S = P (1 + rt)
S = 2000 (1 + 0.06 x 3/12)
S= 2000(1.015)
S = 1980

Note:
(i) total debt = total payments
(ii) The time you have used subtract from x. So that you can calculate the remaining
time.

Multipe payments
Question.47 Mr. Maher owes Rs. 1000 due in 2 months and Rs. 2400 due in 6 months
and Rs. 600 due in 8 months. In how many months he should make a
single payment of Rs. 4000 to discharge the three debts if interest rate is
6%?

A. 4.8 B. 5.9 C. 5.3 D. 5.8

Solution
1000 2400 600 payment

0 2 6 8 x

x−2 x−6 x−8


4000 = 1000[(1 + 0.06( 12 )] + 2400 [(1 + 0.06( 12 )] + 600[(1+( 12 )0.06]
x−2 x−6 x−8
4000 = (1000 + 60( 12 )] + (2400 + 144( 12 )) + (600 + 36 ( 12 ))
4000 = 4000 + [5(x - 2) + 12(x - 6) +3(x - 8)]
0 = 5x – 10 + 12x – 72 + 3x – 24
20x = 106
106
x = 20 = 5.3 months

Question.48-k-15 A certain sum is invested for T years. It amount to Rs. 400 at 10%
simple interest per annum. But when invested at 4% simple interest per annum’. It
amount to Rs. 200 then time (t) is.

A. 46 B. 50 C. 56 D. 54

Solution

Given
Investment I Investment II

26 | P a g e
S1 = 400 S2 = 200
r = 10% = 0.1 r = 4% = 0.04
t1 = t t2 = t
P=? P=?

Using Using
S1 = p (1 + rt1) S2 = p(1 + rt2)
400 = p(1 + 0.1t) (i) 200 = p(1 + 0.04t2) (ii)

Dividing (i) by (ii)


400 p(1+0.1t)
= p(1+0.04t)
200
1+0.1t
2=
1+0.04t
2(1 + 0.04t) = 1 + 0.1t
2 + 0.08t = 1 + 0.1t
2 – 1 = 0.1t - 0.08t
1 = 0.02t
t = 50 years

27 | P a g e
DISCOUNTING
Question.49-2007 The holder of promissory note of Rs. 50,000 bearing interest at the
rate of 9% wishes to have it discounted three months before the maturity
date. The note was originally payable after one year of the date of issue
compute the amount which the holder would receive if the discount rate is
10%.

A. 45357.76 B. 34526.87 C. 53137.50 D. 54362.98

Solution

Given
P = Rs.50,000
t = 1 year
r = 9%
S=?
Using
S = P(1 + rt)
S = 50,000(1 + 0.09 x 1)
S= 50,000 (1.09)
S= 54500
Discounted amount
S= 54500
t = 3/12 years
r = 10%
P =?
Using
P = S (1 - rt)
3
P = 54500 (1 - 0.10 x 12)
P = 54500 (0.975)
P = Rs. 53,137

Question.50 A 3 months, interest bearing note date March 2, 1984 was discounted on
April 3 at 6%. The principal was Rs. 1485. What was the future value of
the note?

A. 3500 B. 2345 C. 1678 D. 1500

Solution
Given
P = 1485
r = 6%
3 months note, one-month discount remaining 2 months.
t = 2/12 = 1/6
Using
P = S (1 – rt)
P
S = 1−rt

28 | P a g e
1485
S= 1−0.06x1/6
1485
S= 0.99
S = 1500

Question.51 Mr. Sadiqi had a note for Rs. 2500 with an interest rate of 6%. The note
was dated March 15, 1984 and his maturity date was 90 days after date on
March 30, 1984 he took the note to his bank which discounted it at a
discount rate of 7%. How much did he receive from the bank?

A. 2300.788 B. 2500.495 C. 2300.987 D. 4509.988

Solution
Given
P = 2500
r = 6%
t = 3/12 = 1/4
Using
S = P(1 + rt)
= 2500(1 + 0.06 x ¼)
= 2500(1.015)
S = 2537.5
Also
P =?
1
t = 22 = 5/2 = 5/24 years
r = 7%
S = 2537.5
Using
P = S (1-rt)
P= 2537.5(1 – 0.07 x 5/24)
P= 2537.5(0.98542)
P = 2500.495

Question.52 A men borrowed Rs. 10,000 and agreed to repay the amount plus 8%
interest in 6 months. Two months after the money was borrowed, the creditor agreed to
settle the debt discounted it at simple interest rate of 9%. How much did the creditor
receive when he discounted the debt?

A. 10097.087 B. 12347.098 C. 54367.987 D. 67548.987

Solution
Given
P = 10,000
r = 8%
t = 6/12 = 1/2 years
Using
S = P(1 + rt)
S = 10,000(1 + 0.08 x1/2)
S= 10,000(1.04)

29 | P a g e
S = 10,400

After two months


S = 10400
t = 4/12 = 1/3 year
P =?
r = 9%
Using
S
P = 1+rt
10400
P = 1 + 0.09 x 1/3
10400
P= 1.03
P = 10097.087

Question.53 A men borrowed Rs. 2000 on May 1, and agreed to repay the amount plus
8% interest in 6 months two months after the money was borrowed the
creditor agreed to settle the debt discounting it the rate of 9%. How much
the creditor received when he discounted the debt?

A. 2769.98 B. 1427.98 C. 2019.42 D. 6575.89

Solution
Given
P = 2000
r = 8%
t = 6/12 = 1/2 years
Using
S = P(1 + rt)
S = 2000(1 + 0.08 x1/2)
S= 2000(1.04)
S = 2080

After two months


S = 2080
r = 9%
t = 4/12 = 1/3 year
Using
S
P = 1+rt
2080
P= 1 + 0.09 x 1/3
2080
P= 1.03
P = 2019.42

Question.54 A bill of Rs. 3500 due in 9 months was discounted by a bank at 8%


discount rate. What would the bank pay for the bill now?

A. 3239 B. 3489 C. 3290 D. 3456

30 | P a g e
Solution
Given
P =?
S = 3500
r = 8%
t = 9/12 years
Using
P = S(1 - rt)
P= 3500(1 - 0.08 x 9/12)
P = 3500(0.94)
P = 3290

Question.55 Mr. Mubeen receives Rs. 3948 by discounting a bill at the bank. If the bill
was due in 6 months and the discount rate was 12%. What was the face
value of the bill?

A. 4500 B. 4100 C. 4700 D. 4200

Solution
Given
P = 3948
t = 6/12 year
r = 12%
S =?
Using
P = S (1 - rt)
P
=S
1−rt
3948
S = 1−0.12 x 1/2
3948
S = 0.94
S = 4200

Question.56 A 90 days, Rs. 4000, 7% interest bearing note dated April 4, was
discounted on May 4 at a discount rate of 8%. What was the discounted
value of the note?

A. 4532.56 B. 4015.73 C. 5678.09 D. 6754.78

Solution
Given
P = 4000
r = 7%
t = 3/12 = 1/4 year
S =?
Using
S = P(1 + rt)
S = 4000(1 + 0.07 x ¼)

31 | P a g e
S = 4000(1.0.175)
Face value = S = 4070
Also
S = 4070
r = 8%
t = 2/12 = 1/6 year
Using
P = S(1 - rt)
= 4070(1 - 0.08 x 1/6)
= 4070(0.9866)
P = 4015.73

Question.57 How much Mr. Salim borrow from a bank that has a 8% simple interest
in order to receive Rs. 2400 now if the repayment is made after two years.

A. 2857.14 B. 2765.56 C. 2687.78 D. 2435.43

Solution
Given
P = 2400
t = 2 years
r = 8%
S=?
Using
P = S (1 - rt)
2400 = S (1 - 0.08 x 2)
2400
=S
0.84
S = 2857.14

Question.58 Mr. Mogembo borrows Rs.1600 for one year from a loan company. He is given
only Rs.1560 and is expected to repay the Rs.1600 at the end of one year. What is the simple
discount?

A. B. C. D.

Solution
Given
S = 1600
P = 1560
r =?
t=1
Using
P = S (1 – rt)
1560 = 1600 (1 ‒ r)
0.975 = 1 ‒ r
‒0.25 = ‒r
r = 0.025
r = 2.5%

32 | P a g e
COMPOUND INTEREST

C.I= P [(1+r)t -1]

Question.59 If P = Rs. 1000, r = 5% P.a, t = 4 and C.I is:

A. 216.78 B. 230.50 C. 215.50 D. 218.89

Solution
Given
P = 1000
r = 5% = 0.05
t=4
S =?
C.I =?

Using
C.I= P [(1+r)t -1]
C.I =1000[(1+0.05)4 -1
C.I =1000[(1.05)4 -1]
C.I =215.50
𝟏
Question.60 The C.I on Rs. 16,000 for 1 𝟐 year at 10% p.a payable half yearly is.

A. 18644 B. 18522 C. 18654 D. 18675

Solution
Given
P = 16,000
r = 10% = 0.10
1
t = 12 years = 3/2 years (Semi-annually)
Using
C.I= P [(1+r/2)2t -1]
C.I =16000[(1+0.05)3 -1
C.I =16000[(0.157625]
C.I =2522

Question.61 The compound interest on Rs. 1000 at 6% compounded semi-annually for


6 years is:

A. 435.87 B. 434.67 C. 425.76 D. 234.78

Solution
Given
P = 1000
r = 6%
t = 6 years

33 | P a g e
S =?
Using
S = P (1 + r)t
0.06
= 1000(1 + 2 )2(6)

= 1000 (1+ 0.3)12


= 1425.76
Now
C.I = S - P
= 1425.76 - 1000
C.I = 425.76

Question.62 What will be the difference in the compound interest on Rs. 50,000 at
12% for one year? When the interest is paid yearly and half yearly?

A. B. C. D.

Solution
Given
Investment I Investment II
P = 50,000 P = 50,000
r = 12% r = 12% (semi-annually)
t=1 t=1

Using Using
t r
C.I = [P(1 + r) -1] C.I = [P(1 + 2)2t -1]
= 50,000 [(1 + 0.12)1 -1] = 50,000 [(1 + 0.06)2 -1]
= 50,000 (0.12) C.I = 50,000(0.1236)
C.I = 6000 C.I = 6180

According to statement
= C.I1 – C.I2
= 6180 – 6000
C.I = 180

𝟏
Question.63 Mr. X borrowed Rs. 5120 at 𝟏𝟐 𝟐 % p.a. C.I. At the end of 3 years, the
money was repaid along with the interest accrued the amount of interest
paid by him is.

A. B. C. D.

Solution
Given
P = 5120

34 | P a g e
r = 12.5%
C.I =?
t = 3 years
Using
C.I = P [(1 + r)t -1]
C.I = 5120 [(1 + 0.125)3 – 1]
C.I = 5120 [1.4238281 – 1]
C.I = 5120 (0.4238281)
C.I = Rs.2170

Question.64 Ali has 40,000. He invested some amount in scheme A at C.I at 15% and the
remaining amount is scheme B at S.I at 10%. If he got the same interest from
both the investments at the end of one year. How much did he invest in scheme
B?

A. B. C. D.

Solution
Given
Scheme A Scheme B
P=x P = 40,000 – x
r = 15% r = 10%
t=1 t=1
C.I =? S.I = ?
Using Using
C.I = P[(1 + r) – 1]
t
S.I = Prt
C.I = x [(1 + 0.15)1 – 1] S.I = (40,000 – x) × 0.1 × 1
C.I = x (1.15 - 1) S.I = 4000 - 0.1x
C.I = x (0.15)

According to statement
C.I = S.I
0.15x = 4000 - 0.1x
0.25x = 4000
Investment in Scheme A = x = 16000
Investment in scheme B = 40,000 - 16000 = 24000

Question.65 Rs.6100 was partly invested in scheme A at 10% p.a. compound interest
(Compounded Annually) for 2 years and partly in scheme B at 10% p.a. simple
interest for 4 years. Both the schemes worth equal interests. How much was
invested in scheme.

A. B. C. D.

Solution
Given
Scheme –A Scheme –B
P=x P = 6100 – x

35 | P a g e
r = 10% r = 10%
t=2 t=4
Using
C.I = P [(1 + r)t - 1] S.I = Prt
C.I = x [(1 + 0.1)2 - 1] S.I = (6100 - x) 0.10x4
C.I = 0.21x = 2440 - 0.4x

According to statement
C.I = S.I
0.21x = 2440 - 0.4x
0.61x = 2440
x = 4000

Question.66 An amount of Rs.110000 is invested at compound interest payable annually. If


the rate of interest is 11% p.a. what will be the total interest after two years.

A. B. C. D.

Solution
Given
P = 110000
r = 11%
t=2
Using
C.I = P[(1 + r)t – 1]
= 110000 [(1 + 0.11)2 – 1]
= 110000 (0.2321)
= 25531

Question.67 There is 90% increase in an amount in 9 years at simple interest. What will be
the compound interest of Rs.15000 after 4 years at the same rate?

A. B. C. D.

Solution
Given
P=x
S = 0.90x + x = 1.9x
r =?
t=9
Using
S = P (1 + rt)
1.9x = x (1 + r × 9)
1.9 = 1 + 9r
0.9 = 9r
… r = 0.1…
Also
C.I = P[(1 + r)t – 1]

36 | P a g e
= 15000 [(1 + 0.1)4 – 1]
= 15000 [(1.1)4 – 1]
= 15000 (0.4641) = 6962

Question.68 The difference between the compound interest and simple interest earned at the
end of the second year on a sum of money at 10% per annum is Rs.20 the sum is.

A. B. C. D.

Solution
Given
According to statement
C.I – S.I = 20
P[(1 + r)t ‒1] – Prt = 20
P[ (1 + 0.1)2 – 1] – P × 0.1 × 2 = 20
P [(1.1)2 – 1 – 0.2] = 20
P [0.01] = 20
P = Rs.2000

Question.69 The difference between the compound interest on Rs.1600 for one year at 20%
per annum, when compounded half yearly and quarterly is.

A. B. C. D.

Solution
Given
According to statement
= C.I – C.I
r 2t r 4t
= p [ (1 + ) − 1] − p [(1 + ) − 1]
2 4
0.2 4
= 1600 [ (1 + 0.1)2 − 1] − 1600 [(1 + ) − 1]
4
= 1600 [(1.1)2 – 1] – 1600 [(1.05)4 – 1]
= 1600 (0.21) – 1600 (0.2155)
= 336 – 344.80
= 8.8

Question.70 Find the compound interest on Rs.24000 at 20% per annum for 9 months,
compounded quarterly.

A. B. C. D.

Solution
Given
P = 24000
r = 20%
t = 9/12 months
Using
C.I = P[(1 + r)t – 1]

37 | P a g e
r 4 × 9⁄12
= 24000 [(1 + ) − 1]
4
0.20 3
C. I = 24000 [(1 + ) − 1]
4
C.I = 24000 (0.1576)
C.I = 3783

Question.71 Simple interest on a certain sum of money for 4 years at 12%per annum is half
the compound interest on Rs.7200 for 2 years at 10% per annum the sum placed
on simple interest is.

A. B. C. D.

Solution
Given
P=?
r= 10%
t=2
According to given statement
S.I = C.I/2
Prt = P[(1 + r)t – 1]/2
(P × 0.12 × 4) = 7200 [(1 + 0.1)2 ‒ 1]/2
0.48p = 756
P = 1575

Question.72 If the simple interest on a sum of money for 2 years at 13% per annum is Rs.169,
what will be the compound interest on same value?

A. B. C. D.

Solution
Given
I = 169
r = 13%
t=2
Using
I = Prt
169 = P × 0.13 × 2
169 = 0.26p
P = 650
Also
C.I = P[(1 + r)t – 1]
= 650 [(1 + 0.13)2 – 1]
= 650 [(1.13)2 – 1]
= 650 (0.2769)
= 180

Question.73 What will be the difference between simple and compound interest @16% per
annum on the sum of Rs.2000 after 3 years?

A. B. C. D.

38 | P a g e
Solution
Given
According to statement
= C.I ‒ S.I
= P[(1 + r)t – 1] ‒ Prt
= 2000 [(1 + 0.16)3 – 1] – [2000 × 0.16 × 3]
= 1121 – 79 - 960
C.I = 161.80

Question.74 Rs.800 becomes Rs.956 in 3 years at a certain rate of simple interest. If the rate
of interest is increased by 4%. What amount will Rs.800 becomes in 3 years.

A. B. C. D.

Solution
Given
P = 800 1.195 = 1 + 3r
S = 956 0.195 = 3r
t=3 r = 0.065
r =? r = 6.5%
Using When r is increased by 4%
S = P (1 + rt) r = 0.065 + 0.04 = 0.105
956 = 800 (1 + r × 3) Also
S = P(1 + rt)
S = 800 (1 + 0.105 × 3)
S = 1052

39 | P a g e
FINDING FUTURE VALUE
Question. 75 This morning you borrow Rs.6000 at 8.45 percent annual compound interest.
You are to repay the loan principle plus all of the loan interest in one lump sum
three years from today. How much will you have to repay?

A. B. C. D.

Solution
Given
P = 6000
r = 8.45%
t=3
S=?
Using
S = P(1 + r)t
S = 6000 (1 + 0.0845)3
S = 6000 (1.0845)3
S = 7653.14

Question.76 Osama invested Rs. 8000 for 3 years at 5% C.I in a post office. If the
interest is compounded once in a year? What sum will he get after 3 years?

A. 9261 B. 6587 C. 6876 D. 8976

Solution
Given
P = 8000
r = 5% = 0.05
t = 3 years
S =?
Using
S = P(1 + r)t
= 8000(1 + 0.05)3
= 8000(1.05)3
S = 9261

Question.77 The population of a city was 8 million on January 1, 2010. The population
is growing at the exponential rate of 2 percent per year. What will the
population be on January 1, 2015?

A. B. C. D.

Solution
Given
P = 8,000,000
r = 2%
t = 5 years
S =?
Using

40 | P a g e
S = Pert
= 8,000,000 x e0.02x5
= 8,000,000 x (1.10517)
S = 8841367.34
Question.78 The capital of a business grows @12% per annum. Compounded
quarterly, if present capital is Rs. 300,000 the capital after 12 years would
be.

A. B. C. D.

Solution
Given
P = 300,000
r = 12%
t = 12 years
s =?
Using
r
S = P(1 + 4)4t
0.12 4x12
S = 300,000(1 + )
4
= 300,000 (1 + 0.03)48
= 300,000 (1.03)48
S = Rs. 1239676

Question.79 The difference between simple and compound interest on a certain


amount of money for 8 years at 14% per annum is Rs. 12500. Find the
amount.

A. B. C. D.

Solution

Given
P=x
t=8
r = 14%
Also
C.I – S.I = 12500
P[ (1 + r)t-1] – Prt = 12500
x[ (1 + 0.14)8 - 1] – [x x 0.14 x 8] = 12500
x[(1.14)8 - 1] - 1.12x = 12500
x[(2.8525) – 1] - 1.12x = 12500
x[1.852586 - 1] = 12500
0.732586x = 12500
x = 17062.84

41 | P a g e
Question.80-2006 A person deposited Rs. 100,000 in a bank for three years. The
bank paid interest at the rate of 8% per annum. Compounded half yearly during the
first year and at the rate of 12% per annum. Compounded quarterly during the last two
years. His balance after three years is:

A. B. C. D.

Solution
Given
Investment I Investment II
P = 100,000 P = 108,160
t = 1 years r = 12%
r = 8% t = 2 years
Using Using
r r
S = P(1 + 2)2t S = P(1 + 4)4t
= 100,000 (1 + 0.04)2 S= 108160 (1 + 0.03)8
= 100,000 (1.04)2 S= 108160 (1.03)8
S=108160 S= 137013.85

Question.81 A firms labour force is growing at the rate of 2 percent per annum the
firm now employees 500 people. How many employees is it expected to hire during the
next five years?

A. B. C. D.

Solution
Given
P = 500
r = 2%
t = 5 years
S =?
Using
S = P(1 + r)t
S= 500(1 + 0.02)5
S= 500(1.02)5
S= 552
Number of employees for next five years is
= 552 - 500
= 52 employees

Question.82 A sum of money at compound interest amounts to thrice itself in 3 years. In how
many years will it be 9 times itself?

A. B. C. D.

42 | P a g e
Solution
Given
P=P Also
S = 3P 9P = P (1 + 0.44)t
t=3 9 = (1.44)t
r =? Taking log on both side
Using log 9 = t log 1.44
S = P(1 + r)t 0.9542 = t(0.1583)
3P = P (1 + r)3 t= 6 years
3 = (1 + r)3
1.44 = 1 + r
0.44 = r

Question.83 An investor places Rs.8000 into an investment for 10 years. The compound rate
of interest is 8% for the first 4 years and 12% for the last 6 years. At the end of
10 years the investment (to the nearest) is worth.

A. B. C. D.

Solution
Given Also
P = 8000 P = 10883.91
S=? S =?
r = 8% t=6
t=4 R = 12%
Using Using
S = P(1 + r)t S = P (1 + r)t
S = 8000 (1 + 0.08)4 S = 10883.81 (1 + 0.12)6
S = 10883.91 S = 21482.71

Question.84 A sum of the 12000 deposited at compound interest become double after 5 years.
After 20 years it will be.

A. B. C. D.

Solution
Given Also
S = 24000 S = P (1 + r)t
p = 12000 S = 24000 (1 + 0.14)15
r =? = 24000 (1.14)15
t=5 = 171310
Using Total amount after 20 years is
S = P (1 + r)t = 24000 + 171310
24000 = 12000 (1 + r)5 = 195310
2 = (1 + r)5
r = 0.148698
r = 14%

Question.85 A man owes Rs.4000 due in 4 years and 3400 due in 6 years. His creditor has
agreed for him to pay the two debts with payment of Rs.3000 in 3 years and
remainder in 8 years. If money is worth 7% compounded quarterly, what will be
the amount of second payment?

43 | P a g e
A. B. C. D.

Solution
Given
3000 4000 34000
3 4 6 8

Given Using
P=? r 4t
S = P (1 + )
4
S = 4000 0.07 4
4000 = P (1 + )
4
t=1 4000 = P (1.0718)
r = 7% P = 3732

Also
S = 3400
P =?
r = 7%
t=3
Using
r 4t
S = P (1 + )
4
0.07 4×3
3400 = P (1 + )
4

3400 = (1.2314)P
P = 2761
Now
Remaining amount
= 3732 + 2761 – 3000
= 3493

Also
P = 3493
S =?
r = 0.07
t=5
Using
r 4t
S = P (1 + )
4
0.07 4×5
𝑆 = 3493 (1 + )
4
S = 3493 (1.4147)
S = 4941
Question.86 Rs.5887 is divided between Saqlain and Ali, such that Saqlian’s share’s at the
end of 9 years is equal to Ali’s share at the end of 11 years, compounded
annually at the rate of 5% share of Saqlain is?

44 | P a g e
A. B. C. D.

Solution
Given
According to statement
P=x P = 5887 ‒ x
Saqlain’s share = Ali shares
P(1 + r)t = P (1 + r)t
x (1 + 0.05)9 = (5887 ‒ x) (1 + 0.05)11
x(1.5513) = (5887 ‒ x) (1.7103)
1.5513x = 10068 – 1.7103x
3.2616x = 10068
x = 3087

Question.87 The government has imposed a fine on the shady trader. The fine calls for
annual payment of Rs.650,000, Rs.900, 000 and Rs.1.2 million respectively over
the next three years. The first payment is due one year from today. The
government plans to invest the funds until the final payment is collected and
then donate the entire amount, including the investment earnings, to a national
securities training centre. The government will earn 4.25 percent on the fund
held. How much will national securities training centre receive three years from
today?

A. B. C.0+ D.

Solution 6,50,000
0 1 2 3
Given
r = 4.25%
Using
S = P(1 + r)t + P(1 + r)t + 1,200,000
S = [650,000 (1 + 0.0425)2] + [900,000 (1 + 0.0425)1] + 1,200,000
S = 706424.06 + 938250 + 1,200,000
S = 2844674.06

45 | P a g e
FINDING PRESENT VALUE
Question.89 A person will receive Rs. 5000 six years from now present value at a
compounded discount rate of 8 percent is.

A. B. C. D.

Solution
Given
S = 5,000
t=6
P =?
r = 8%
Using
S = P (1 + r)t
5,000 = P (1 + 0.08)6
5,000 = P(1.08)6
5,000 = P (1.5868)
5000
=P
1.5868
3150.99 = P

Question.90 A non -interest bearing note of Rs. 500,000 is due in 4 years from now. If
the note is discounted now at 8% semi-annually, what will be the proceeds
and compound discount?

A. B. C. D.

Solution

Given
S = 500,000
r = 8%
t = 4 years
P =?
Using
S = P(1 +)2t
0.08
500,000 = P(1 + 2 )4x2
500,000 = P(1.04)8
500,000 = P(1.3685)
P = 365363.53
Compound discount =S-P=500000 – 365363.53= 134636.46

Question.91 How much an individual deposit should now to yield Rs. 600,000 at the
end of five years at 9% compounded half yearly.

A. B. C. D.

Solution

46 | P a g e
Given
P =?
S = 600,000
t=5
r = 9%

Using
r
S = P(1 + 2)2t
0.09
600,000 = P(1 + 2 )2x5
600,000 = P(1+ 0.045)10
600,000 = P(1.55296)
P = 386,358.95

Question.92 A sum of money invested at compound interest amount to Rs. 4624 in 2


years and to Rs. 4913 in 3 years the sum of money is.

A. B. C. D.

Note: to find the present value first find the rate of interest. So take first sum as P and
end sum as Sand find rate of interest
Solution
S=P = 4624 S = 4913

0 1 2 t=1 3
Given

P = 4624
S = 4913
r =? Now (Reverse the value of P which is
t =1 now s and find p.

Using Using (only reverse the 2nd year amount


to find P
S = P (1 + r)t S = P(1 + r)t
4913 = 4624(1 + r)1
1.0625 = (1 +r) 4624 = P (1+.0625)2
0.0625 = r
r = 6.25% P = 4096

Question.93 If annual interest rate falls from 12 to 8 percent per annum, how much
more be deposited in an account to have Rs. 600,000 in 5 years, if both
rates are compounded semi-annually.

A. B. C. D.

47 | P a g e
Solution
Option I Option II

Given
P =? P =?
S = 600,000 S = 600,000
r = 12% r = 8%
t = 5 years t = 5 years

Using Using
r 2t r
S = P(1 + ) S = P(1 + 2)2t
2
0.12 2x5 0.08 2x5
600,000 = P(1 + ) 600,000 = P(1 + )
2 2
600,000 = P(1 + 0.06)10 600,000 = P(1 + 0.04)10
600,000 = P(1.06)10 600,000 = P(1.04)10
600,000 = P(1.7908) 600,000 = P(1.4802)
P1 = 335036.78 P2 = 405350.62

Now to find how much amount is to be deposited


P= P2 - P1 = 405350.62 – 335036.78
P= 70301.62

Question.94 A trust fund for a child education is being set up by a single payment so
that at the end of the 10 years there will be Rs. 240,000 if the fund years at
the rate of 8% compounded semi-annually amount of money should be
paid into the fund initially is.

A. B. C. D.

Solution
Given
S = Rs. 240,000
t = 10 years
r = 8%
P=?
Using
r
S = P(1 + )2t
2
0.08 2x10
240,000 = P(1 + )
2

240,000 = P(1.04)20
240,000 = P(2.19112)
P = 109533.024

48 | P a g e
Question.95 The difference between C.I and S.I on a certain sum of money invested
for 3 years at 6% p.a is Rs. 110.16. The sum is:

A. B. C. D.

Solution
Given
According to statement
C.I - S.I = 110.16
P[(1 + r)t-1] - prt = 110.16
P[(1 + r)t -1 - rt] = 110.16
P[(1 + 0.66)3 -1 - 0.06 x 3] = 110.16
P[(1.06)3 -1 - 0.18] = 110.16
P(0.011016) = 110.16
P = 10,000

Question.96 The compound interest on a certain sum for 2 years at 10% per annum is
Rs. 525. The simple interest on the same sum for double the time at half
the rate percent is.

A. B. C. D.

Solution
Given

Investment I Investment II
C.I = 525 P = 2500
P=? r = 5%
r = 10% t = 4 years
t = 2 years S.I = ?

Using Using
C.I = P[(1 + r)t -1] I = Prt
525 = P[(1 + 0.12)2 -1] = 2500 x 0.05 x 4
525 = P[(1.01)2 -1] I = 500
P = 2500

Question.97 A sum of money lent at compound interest for 2 years at 20% per annum
would fetch Rs.482 more, if the interest was payable half yearly than if it
was payable annually. The sum is:

A. B. C. D.

Solution
Given

Investment I Investment II

49 | P a g e
Annually Semi-annually
Given
P=x P=x
r = 20% r = 20%
S1 =? S2 =?
t = 2 years t=2

Using Using
t r
S1 = P(1 + r) S2 = p(1 + 2)2t
S1 = x(1.2)2 S2= x(1.1)4
= 1.44x S2 = 1.4641

According to statement
S2 = S1 + 482
S2 - S1 = 482
1.4641x – 1.44x = 482
0.0241x = 482
482
x = 0.0241
x = 20,000

Question.98 Rashid wants to invest Rs.1,500,000 for one year, Bank A has offered
simple interest @10% per annum whereas bank B has offered interest
@8% per annum compounded half yearly.
Rashid believes that investment in bank A is risky and therefore he
prefers to invest in bank B. However, as he wants to earn interest of at
least Rs.140,000, he would also have to invest some of the amount in bank
A. Determine the minimum amount that he will have to invest in Bank A
to achieve the above objective.

A. B. C. D.

Solution

Bank A Bank B
Given
P=x P = 1,500,000 - x
r = 10% r = 8%
t=1 t = 1 (semi-annually)
S.I =? C.I =?
r
S.I = Prt C.I = P(1 + 2)2t -1)
10 0.08
S.I= x x 100 x 1 C.1= (1500000-x)[(1 + 2 )2 -1]
S.I = 0.1x C.I= (1,500,000 – x)[0.0816]
C.I= 122400 – 0.0816x

Total interest gained from both investment = 140,000


0.1x + 122400 – 0.0816x = 140,000

50 | P a g e
0.0184x = 140,000 – 122,400
0.0184x = 17600
x = 956521.73

Question.99 How much should on individual deposit now to yield Rs. 600,000 at the
end of five years at 9% compounded half yearly.

A. B. C. D.

Solution

Given
P =?
S = 600,000
t = 5 years
r = 9%
Using
r
S = P(1 + 2)2t
0.09
600,000 = P(1 + 2 )10
600000 = P (1.5529)
P = 386373.88
Question.100 The compounded interest on a certain sum of money for 2 years at 5% is Rs.328.
Then the sum is.

A. B. C. D.

Solution
Given
P=p
t=2
r = 5%
Using
C.I = P [(1 + r)t – 1]
328 = P[1 + 0.05)2 – 1]
328 = P[(1.05)2 – 1]
328 = P (0.1025)
P = 3200

Question.101 An amount is given at an interest of 8% p.a, what is the amount if the difference
of compound and simple interest for 2 year is Rs.352?

A. B. C. D.

Solution
Given
C.I – S.I = 351
P[(1 + r)t – 1] – Prt = 352
P [(1 + 0.08)2 – 1] - P × 0.08 × 2 = 352

51 | P a g e
P[(1.08)2 – 1) – 0.16] = 352
P[0.0064] = 352
P = 55000
Question.102 You are considering changing job. Your goal is to work for three years and then
return to school full time in pursuit of an advanced degree. A potential employer just offered
you on annual salary of Rs.36000, Rs.39000 and Rs.42000 a year for the next three years,
respectively. The offer also includes a starting bonus of Rs.10,000 payable immediately. What is
this offer worth to you today at a discount rate of 7.5 percent?

A. B. C.0+ D.

Solution 10,000 36,000 39000 42,000


0 1 2 3
Given
r = 7.5%
36000 39000 42000
Present value = 10,000 + 1
+ 2
+
(1 + r) (1 + r) (1 + r)3
36000 39000 42000
P = 10,000 + + 2
+
1 + 0.075 (1 + 0.075) (1 + 0.075)3
36000 39000 42000
P = 10,000 + + 2
+
1.075 (1.0.75) (1.075)3
P = 10,000 + 33488.37 + 33747.97 + 33808.34
P = 111044.68

Question.103 Your local travel agent is advertising an upscale winter vacation package for
travel two years from how to Antarctica. The package requires that you pay
Rs.15000 today, Rs.25000 on years from today and a final payment of Rs.65000
on the day you depart two years from today. What is the cost of this vacation in
today’s rupees if the discount rate is 9 percent?

A. B. C.0+ D.

Solution 15000 25000 65000

0 1 2
Given
r = 9%
Using
25000 65000
P = 15,000 + 1
+
(1 + r) (1 + r)2
25000 65000
P = 15,000 + +
1 + 0.09 (1 + 0.09)2
25000 65000
P = 15,000 + +
1.09 (1.09)2
P = 15000 + 22935.77 + 54709.19
P = 92644.96

52 | P a g e
FINDING RATE OF INTEREST
Question.104 At what rate of interest compounded semi-annually will Rs. 6000 amount
to Rs. 9630 in 8 years?

A. B. C. D.

Solution

Given
P = 6,000
S = 9630
t = 8 years
r =?

Using
r
S = P(1 + 2)2t
r
9630 = 6,000 (1 +2)16
9630 r
= (1 +2)16
6000
r
1.605 = (1 +2)16

(1.605)1/16 = 1 + r/2
(1.605)0.0625 = 1 + r/2
1.0300 = 1 + r/2
0.0300 = r/2
r = 0.06
r= .06 x 100%
r =6%

53 | P a g e
FINDING VALUE OF TIME
Question.105 The population of a town increases every year by 2%, of the population at
the beginning of that year. The number of years by which the total
increase of population be 40% is.

A. B. C. D.

Solution
Given
P=x
S = x + 40%x
S= x(1 + 0.4)
S = (1.4) x
r = 2% = 0.02
Using
S = P(1 + r)t
1.4x = x(1 + 0.02)t
taking long on both side
log 1.4 = log(1.02)t
log 1.4 = t log(1.02)
log 1.4
=t
log 1.02
0.1461280
=t
0.0086
17(app) = t

Question.106 A shop keeper sold goods worker Rs. 3.0 million during 2008. If he is able
to increase his sale by 15% annually in which year he would achieve
annual sale of Rs. 25 million?

A. B. C. D.

Solution
Given
P = 3,000,000
r = 15%
t =?
S = 25,000,000
Using
S = P(1 + r)t
25,000,000 = 3,000,000 (1 + 0.15)t
25
= (1.15)t
3
Taking log on both side
log 8.333 = log(1.15)t
log 8.333 = t log(1.15)t
0.9206 = t(0.06069)

54 | P a g e
0.9206
t = 0.06069
t = 15.17 years
Also the year in which sales become 25 million is 2008 + 15 = 2023

Question.107 The population of a country increases at the rate of 3% per annum. How
many years will it take to double itself?

A. B. C. D.

Solution
Given
P =P
S = 2P
r = 3% = 0.03
t =?
Using
S = P(1 + r)t
2P = P(1 + 0.03)t
2 = (1.03)t
taking long on both side
log2 = log(1.03)t
log2 = t log(1.03)
0.3010 = t(0.0128)
0.3010
=t
0.0128
23.51 = t

Question.108 Mr. Rashid invested Rs. 60,000 in a company but found that his
investment was losing 6% of its value per annum. After two years, he
decided to pull out what was left of the investment and place at 4%
interest compounded twice a year. He would recover his original
investment in the.

A. B. C. D.

Solution

First determine the losses for two years


Given
P = 60,000
r = 6%
t = 2 years
S =?
Using

55 | P a g e
S = P(1 - r)t
S = 60,000(1 - 0.06)2
S = 60,000(0.94)2
S = 53016
Now we will find time to recover the loss.

Given
P = 53016
S = 60,000
r = 4%
t =?
Using
r
S = P(1 + 2)2t
0.04 2t
60,000 = 53016(1 + )
2
1.131733 = (1 + 0.02)2t
1.131733 = (1.02)2t
Taking log on both side
log(1.131733) = 2t log(1.02)
0.053743 = 2t(0.008600)
2t = 6.2491
t = 3.12

Question.109 The population of country is growing exponentially at a constant rate of 2


percent per year. How much time this population will take to double
itself?

A. B. C. D.

Solution

Given
P=P
S = 2P
r = 2%
t =?
Using
S = Pert
2P = Pe0.02t
2 = e0.02t
Taking log on both side
ln2 = 0.02t (ln e)

In2 = 0.02t(1)

56 | P a g e
0.6931 = 0.02t(1)
34.65 = t

Question.110 Companies A and B earned profit of Rs. 600,000 and 1,320,000


respectively during 2009. It is estimated that future annual profit would
increase@8% and 5% respectively. In which year, the profit of both
companies would be equal.

A. B. C. D.

Solution

Investment A Investment B
P = 600,000 P = 1,320,000
r = 8% r = 5%
t =? t =?
S1 =? S2 =?

Using Using
t
S = P(1 + r) S = P(1 + r)t
= 600,000(1 + 0.08)t = 1,320,000(1 + 0.08)t
= 600,000(1.08)t = 1,320,000(1.05)t

Now at time, where both investment are equal is


S1 = S2
600,000(1.08) = 1,320,000(1.05)t
t

1,320,000 (1.08)t
= (1.05)t
600,000
2.2 = (1.02857)t

Taking log on both side


log(2.2) = t log(1.02857)
0.342422 = t(0.01223)
t = 27.98
t = 28 (years) app

Question.111 If Rs. 110,000 is to grow to Rs. 250,000 in ten years period, at what annual
interest rate must it be invested, given that interest is compounded semi-
annually.

A. B. C. D.

Solution

Given

57 | P a g e
P = Rs. 110,000
S = Rs. 250,000
t = 10 years
r =?
Using

S = P(1 + r/2) 2t
250,000 = 110,000(1 + r/2)20
25
= (1 + r/2)20
11

Taking power (1/20) on both side


(2.272727)1/20 = 1 + r/2
1.041903 – 1 = r/2
r/2 = 0.041903
r = 0.083806
r=0.083806 x 100%
r= 8.3806%
Question.112 A car was moving at a speed of 135 km per hour. When brakes were
applied, the speed of the car reduced to 43.2 km per hour in five seconds.
Find the rate of decline in speed of the car per second. If the percentage
decrease after each second was the same.

A. B. C. D.

Solution
Given
Initial speed = P = 135 km/h
Final speed = S = 43.5 km/h
r =?
t = 5 seconds

Since
S<p
Using
S = P(1 - r)t
43.2 = 135(1 - r)5
0.32 = (1 - r)5
0.796214 = 1 - r
r = 1 - 0.796214
r = 0.2037
r = 0.2037 x 100%
r = 20. 37%

58 | P a g e
Question.113 Shiraz borrowed Rs.120,000 for Eight months at 15% simple interest Also
Compute the annual rate of interest compounded monthly, which would
result in payment of the same amount of interest.

A. B. C. D.

Solution

Given

SIMPLE INTEREST

P = Rs. 120,000
r = 15%
t = 8/12
S =?
Using
S = P(1 + rt)
8
= 120,000(1 + 0.15 x 12)
= 120,000 (1.1)
S = 132,000

COMPOUND INTEREST

P = 120,000
S = 132,000
r =?
t = 8/12 year
Using
S = P(1 + r/12) 12t
r
132,000 = 120,000(1 + 12)12t
r
1.1 = (1 + 12)12x8/112
r
1.1 = (1 + 12)8
r
(1.1)1/8 = (1 + 12)
r
1.01198 = 1 + 12
r
0.01198 = 12

r = 14.38%
Question.114 Khan deposited an amount into a bank which will be doubled in eight
years.
i) Find the rate of interest considering that the amount is
compounded annually
ii) How many years will it take for an amount to Triple at the above
calculated rate of interest?

59 | P a g e
A. B. C. D.

Solution

Given
P=p
S = 2p
t=8
r =?
Using
S = P(1 + r)t
2P = P(1 + r)8
2 = (1 + r)8
(2)1/8 = 1 + r
r = 0.09050
r = 9.05%

(ii) Given
P=p
S = 3p
r = 9.05%
t=?

Using
S = P(1 + r)t
3p = P(1 + 0.0905)
3 = (1.0905)t
Taking log on both side
log 3 = t log (1.0905)
0.47712 = t(0.037625)
t = 12.68 years

Question.115 Determine the interest rate needed to have money double itself in 12 years
Under annual compounded.

A. B. C. D.

Solution
Given
P=P
S = 2P
r =?
t = 12 years
Using
S = P(1 + r)t
2p = P(1 + r)12

60 | P a g e
2 = (1 + r)12
(2)1/12 = 1 + r
1.05946 = 1 + r
0.05946 = r
r = 5.94%

Question.116 Suppose that Mr. Rashid owes Mr. Ahmad two sums of money Rs.
100,000 due in 2 years and Rs. 60,000 due in 5 years. If he wishes to pay
off the total debt now by a single payment, how much he should pay?
Assume an interest rate of 8% per annum compounded quarterly.

A. B. C. D.

Solution

Loan- I Loan -II


S1 = 100,000 S = 60,000
r = 8% (compound quarterly) r = 8%
t=2 P2 =?
P1 =?

Using Using
4t r
S1 = P1(1 + r/4) S2 = P2(1 + 4)20
0.08 0.08
100,000 = P1(1 + 4 )4t 60,000 = P2(1 + 4 )4t
100,000 = P1(1 + 0.02)8 60,000 = P2(1 - 485947)
P1 = 85349.06 P2 = 40378.29

Paying off both the loan now is

P= P1 + P2
= 85349.06 + 40378.29
= 125727.3171

Question.117 At what rate of simple interest will Rs.4000 amount to Rs.6480 in a span of 7
years?

A. B. C. D.

Solution
Given
P = 4800
S = 6480
t=7
r =?
Using
S = P (1 + rt)
6480 = 4800 (1 + r × 7)

61 | P a g e
6480
= 1 + 7r
4800
1.35 = 1 + 7r
0.35 = 7r
r = 5%

Question.118 If sum of money compounded annually becomes 1.44 times of itself in 2 years,
then the rate of interest per annum is.

A. B. C. D.

Solution
Given
P=p
S = 1.44p
t=2
r =?
Using
S= P (1 + r)t
1.44P = P(1 + r)2
1.44 = (1 + r)2
1.2 = 1 + r
r = 0.2
r = 20%

Question.119 A sum of money at compound interest amounts to thrice itself in 3 years. In how
many years it is 9 times itself.

A. B. C. D.

Solution
Given
P=p
S = 3p
t=3
r=?
Using
S= P (1 + r)t
3P = P (1 + r)3
3 = (1 + r)3
1.44 = 1 + r
r = 0.44

Question.120

A person opened an account on April, 2001 with a deposit of Rs 800. The account paid
6% interest compounded quarterly. On October 1 ,2001, he closed the account and

62 | P a g e
added enough additional money to invest in a 6-month time deposit for Rs. 1000,
earning a 6% compounded monthly
(i) How much additional amount did the person invest on October 1?
(ii) What was the maturity value of his time deposit on April 1, 2002?
(iii) How much total interest was earned?
Solution
(i) Given
P=800, r=0.06 , t=6/12

S=P(1+r/4)4t

S=800(1+0.06/4)4*6/12
S = 824.18
Additional amount invests on October, 1 is =1000-824.18=175.82
(ii) P =1000 ,r=6% t=6/12

S=P(1+r/12)12t
S=1000(1+0.06/4)12*6/12

S = 1030.40

(iii) Total interest earned =24.18 +30.40 =54.58

63 | P a g e
EFFECTIVE RATE OF INTEREST
𝐫
e = [(1 + )2 - 1] x 100%
𝟐

Question.120 The Effective rate of interest corresponding to a nominal rate 3% p.a.


Payable half yearly is.

A. B. C. D.

Solution

Given
r = 3%
t = 2 periods
Using
r
e = [(1 + 2)2 - 1] x 100%
0.03 2
e = [(1 + ) - 1] x 100%
2
e= [(1 + 0.015)2 - 1] x 100%
e = 0.030225 x 100%
e = 3.0225%

Question.121 The effective rate of interest corresponding a nominal rate 7% p.a.


convertible quarterly is.

A. B. C. D.

Solution

Given
e =?
r = 7% = 0.07
t = 4 periods
Using
r
e = [(1 + 4)4 - 1] x 100%
= [(1 + 0.0175)4 - 1] x 100% = 7.18%

Question.122 Compute effective rate of interest where a nominal rate is 8%


compounded Quarterly.

A. B. C. D.

Solution

Given
r = 8%
t=1

64 | P a g e
Using
r
e = [(1 + 4)4 - 1] x 100%
0.08
= [(1 + 4 )4 - 1] x 100%
e = 8.24%

Question.123 An investor can earn 9.1% interest compounded semi-annually or 9%


interest compounded monthly. Determine which option he should prefer?

A. B. C. D.

Solution

Option-I Option-II
r = 9.1% r = 9%

Using Using
r r
e = [(1 + 2)2t - 1] x 100% e = [(1 + 12)12x1 - 1] x 100%
0.091 2 0.09
e = [(1 + ) - 1] x 100% e = [(1 + 12 )12 - 1] x 100%
2
e = 9.30% e = 9.38%

Option II is selected. Because effective rate of option II is greater than option I so option II is
preferred.

Question.124 Rashid wants to obtain a bank loan. Bank A offers a nominal rate of 14%
compounded monthly. Bank B a nominal rate of compounded quarterly
14.5% and bank C offers an effective rate of 14.75%. Which option he
should prefer, if all other terms are same?

A. B. C. D.

Solution

Investment A Investment B Investment C

Given Given Effective rate

r = 14% r = 14.5% = 14.75%


t = 12 periods = 1 year t = 1 year
e =? e =?

Using Using

r r
e = [(1 + 2)12 - 1 e = [(1 + 4)4 - 1

= (1 + 0.01166)12 - 1 = (1 + 0.3625)4 - 1

65 | P a g e
= 0.14937 = 0.1530
= 14.93% = 15.3%
Effective rate of Bank C is smaller than Bank A and Bank B. So Bank C is selected.
Question.125 The nominal interest rate on an investment is 12 percent per year
determine the effective annual rate if interest is compounded quarterly.

A. B. C. D.

Solution

Given
r = 12%
Effective rate of interest compounded
Quarterly
r
e = [(1 + 4)4 - 1] x 100%
0.12 4
e = [(1 + ) - 1] x 100%
4

e = [(1 + 0.03)4 - 1] x 100%


= [(1.03)4 - 1] x 100%
= [(1.1255 - 1] x 100%
= (0.1255) x 100%
= 12.55%
Question.126 The nominal interest rate on an investment is 16 percent per annum.
Determine the effective annual interest rate if interest is compounded
quarterly.

A. B. C. D.

Solution
Given
r = 16%
Effective rate of interest compounded quarterly is
r
e = [(1 + 4)4 - 1] x 100%
0.16 4
e = [(1 + ) - 1] x 100%
4

e = [(1 + 0.04)4 - 1] x 100%


= [(1.04)4 - 1] x 100%
= [(1.1698 - 1] x 100%
= 16.98%

66 | P a g e
Question.127 Find out effective rate of interest equivalent to the nominal rate of 10
percent compounded semi-annually.

A. B. C. D.

Solution
Given
r = 10%
Effective rate of interest semi-annually is
r
e = [(1 + 2)2 - 1] x 100%
0.10 2
e = [(1 + ) - 1] x 100%
2

e = [(1 + 0.05)2 - 1] x 100%


= [(1 + 0.05)2 - 1] x 100%
= [(1.05)2 -1] x 100%
= 10.25%

Question.128 Find out effective rate of interest equivalent to nominal rate of 8 percent
compounded monthly.

A. B. C. D.

Solution

Given
r = 8%
Effective rate of interest compounded monthly is

r
e = [(1 + 12)12 - 1] x 100%
0.08
e = [(1 + 12 )12 - 1] x 100%

= (0.08299) x 100%
= 8.29%

Question.129 Bank A offers 12.25% interest compounded semi-annually, on its saving


accounts, while Bank B offer 12% interest compounded monthly, which
bank offers the higher effective rate?

A. B. C. D.

Solution

67 | P a g e
Given Given

r = 12.25% r = 12%

Effective rate compounded semi-annually Effective rate compounded monthly


r r
e = [(1 + 2)2 – 1] x 100% e = [(1 + 12)12 - 1] x 100%
0.1225 2 0.12 12
e = [(1 + ) - 1] x 100% e = [(1 + ) - 1] x 100%
2 12

= [0.12625) x 100% e = (0.1268) x 100


= 12.62% = 12.68%
Accept offer B. because it has higher effective rate which gives more return.

Question.130 Rayan invested an amount of Rs. 400,000 in an investment scheme and


got Rs. 545,881 at the end of three years. Find
i) The effective annual rate of interest if interest was compounded monthly.
ii) The nominal rate if interest was compounded quarterly.

A. B. C. D.

Solution
First find the rate of interest

Given
P = 400,000
S = 545,881
t = 3 years
r=?
Using
r
S = P[(1 + 12)12
r
545881 = 400,000 [(1 + 12)12x3
r
1.3647 = [(1 +12)36
r
(1.3647)1/36 = 1 +
12
r
(1.3641)0.027777 = 1 + 12
r
1.608662 = 1 + 12
r
8.66227 = 12

0.103947 = r
r = 10.39%

68 | P a g e
(i) Effective rate: compounded monthly
r
e = [(1 + 12)12 - 1] x 100%
0.103a 12
e = [(1 + ) - 1] x 100%
12

e = [1.10899 – 1] x 100%
e = 10.89%
(ii) Nominal rate compounded quarterly

Given
S = 545881
P = 400,000
r=?
t = 3 years
Using
r
S = P(1 + 4)4t
r
545881 = 400,000 (1 + 4)4t
r
1.3647025 = (1 + 4)12
r
(1.3647025)1/12 = 1 + 4
r
(1.3647025)0.083333 = 1 + 4
r
1.02624 = 1 +
4
r
0.02624 = 4

0.10499 = r
r = 10.49%

Question.131 A bank has provided two options to an investor

i) 11.1% profit compounded semi-annually


ii) 11.0% profit compounded monthly
Which option would you recommended?

A. B. C. D.

Solution

Given
r = 11.1%

Effective rate of interest compounded semi-annually.


r
e = [(1 + 12)2 - 1] x 100%

69 | P a g e
0.111 2
e = [(1 + ) - 1] x 100%
2

e = [1.11408 - 1] x 100%
e = 14.40%

ii)

Given
r = 11%
Effective rate of interest compounded monthly
r
e = [(1 + 12)12 - 1] x 100%
0.111 12
e = [(1 + ) - 1] x 100%
12

e = [1.11682 - 1] x 100%
e = (0.11682) x 100%
e = 11.68%
Question.132 Compute effective rate of interest where nominal rate is 8% compounded
quarterly

A. B. C. D.

Solution

Given
r = 8%
Effective rate of interest compounded quarterly.
r
e = [(1 + 4)4 - 1] x 100%
0.08 4
e = [(1 + ) - 1] x 100%
4

e = (0.6824) x 100%
e = 8.24%

Question.133 An investor can earn 9.1% interest compounded semi-annually or 9%


interest compounded monthly. Determine which option he should prefer.

A. B. C. D.

Solution

Option I Option II
Given Given
r = 9.1% r = 9%

70 | P a g e
Effective rate of interest semi-annually Effective rate of interest compounded
monthly
r r
e = [(1 + 2)2 - 1] x 100% e = [(1 + 12)12 - 1] x 100%
0.091 2 0.09 12
e = [(1 + ) - 1] x 100% e = [(1 + ) - 1] x 100%
2 12

e = [0.093070) x 100% e = (0.093806) x 100%


e = 9.30% e = 9.38%

Question.134 Find effective rate of interest equivalent to the nominal rate of 8% compounded
quarterly.

A. B. C. D.

Solution
Given
Effective rate of interest
r m
e = [(1 + ) − 1] × 100%
m
r 4
e = [(1 + ) − 1] × 100%
4
0.08 4
e = [(1 + ) − 1] × 100%
4
= (0.0824) × 100%
e = 8.24%

EQUIVALENT RATE OF INTEREST


Question.135 An interest rate 12% compounded semi-annually is equivalent to %
compounded Quarterly.

A. B. C. D.

Solution
Given
Equivalent rate of interest
r 2 x 4
(1 + ) = (1 + )
2 4
0.12 2 x 4
(1 + ) = (1 + )
2 4
x 4
(1. 06)2 = (1 + )
4
x
1.029563 = 1 +
4
x
= 0.029563
4
x = 0.11825
x = 11.82

71 | P a g e
Question.136 At what nominal rate compounded quarterly will a principal accumulate to the
same amount as at 8% compounded semi-annually.

A. B. C. D.

Solution

Given
Equivalent rate of interest
r 2 x 4
(1 + ) = (1 + )
2 4
2
0.08 x 4
(1 + ) = (1 + )
2 4
x
1,0198 = 1 +
4
x
= 0.0198
4
x = 0.0792
x = 7.92%

Question.137 A financier claims to be lending money at simple interest. But he includes the
interest every three months for calculating the principal. If he is charging an
interest of 16%, the effective rate of interest becomes.

A. B. C. D.

Solution
Given
r= 16%
Using
r 4
e = [(1 + ) − 1] × 100%
4
0.16 4
e = [(1 + ) − 1] × 100%
4
e = 16.98%

Question.138 Your credit card company quotes you a rate of 18.9 percent. Interest is billed
monthly. What is the actual rate of interest you are paying?

A. B. C.0+ D.

Solution
Given
r = 18.9
Using
r 12
e = [(1 + ) − 1] × 100%
12
0.189 12
e = [(1 + ) − 1] × 100%
12
e = [1.2062 – 1] × 100%

72 | P a g e
e = (0.20626) × 1005
e = 20.63%

Question.139 You are considering two loans. The terms of the two loans are equivalent with
the exception of the interest rates. Loan A offers a rate of 8.10 percent compounded daily. Loan
B offers a rate of 8.25 percent compound semi-annually.

A. B. C. D.

Solution
Given
Loan A Loan b
Given Given
r = 8.10 r = 8.25
Using Using
r 365 r 2
e = [(1 + ) − 1] × 100% e = [(1 + ) − 1] × 100%
365 2
0.081 365 0.0.825 2
e = [(1 + ) − 1] × e = [(1 + 2
) − 1] × 100%
365
100%
e = 0.08436 × 100% E (1.08420 – 1) × 100%
e = 8.44% E = 8.42%
Loan B is better offer because the effective annual rate is 8.42 percent.

Question.140 You are paying an effective annual rate of 19.56 percent on your credit card.
The interest is compounded monthly. What is the annual percentage rate on this
account?

A. B. C. D.

Solution
Given
e = 19.56%
Using
r 12
e = (1 + ) −1
12
r 12
19.56% = (1 + ) −1
12
r 12
0.1956 = (1 + ) −1
12
r 12
1.1956 = (1 + ) −1
12
r
1.01499 = 1 +
12
r
0.01498 =
12
0.1799 = r
r = 17.99%
Question: 141 you are given following rates for different years.
Years Rate
2013 12%
2014 20%

73 | P a g e
2015 18%
Find effective rate
Solution:
𝑛
Effective rate = √(1 + 𝑟1 )(1 + 𝑟2 )(1 + 𝑟3 ) - 1
e = 3√(1 + 0.12)(1 + 0.20)(1 + 0.18) - 1
e = 0.166166
e = 16.62%

74 | P a g e
EXTRA PRACTICE QUESTION
Question: 1 The difference between the interests received from two different banks on
Rs. 500 for 2 years is Rs 2.50. The difference in their rates is

Solution:
Given
P = 500 , t = 2, I1 −𝐼2 = 2.50
𝑟1
Then 𝐼1 = 500 * 100 *2
𝐼1 = 10 𝑟1
Also
𝑟2
𝐼2 = 500 * 100 *2
𝐼2 = 10𝑟2
Given
𝐼1 − 𝐼2 = 250
So
2.50 = 10𝑟1-10𝑟2
𝑟1 - 𝑟2 = 0.25%

Question: 2 Goods were bought for Rs. 600 and sold the same day for Rs. 688.50 at a
credit of a month and thus gain 2%. The rate of interest per annum is

Solution:
Purchase=600 × 0.02 = 12

P = 600 + 12 = 612

I = 688.50 – 612 = 76.5

Using I = prt
9
76.5 = 612 × 𝑟 × 12

76.5
𝑟= 459

1
𝑟=6

2
𝑟 = 16 %
3
Question: 3 A building society offers a low-start mortgage of Rs. 40,000 with 10 annual
repayments starting one year from the loan being taken the interest rate applying
throughout will be 11% per annum. But the repayments will be only Rs. 5000 per
annum for the first 5 years. What equal annual repayments will be required in each of
the last 5 years of the mortgage

Solution:

75 | P a g e
40,000 t=5 67402 t=5 R=?

I----------------------------I-----------------------------I------------------------------------I

R = 5,000 s = 31,339

S = 40,000(1 + 0.11)5

S = 67402

Also
(1+𝑟)𝑡 −1
S = 𝑅( )
0.11

(1+0.11)5 −1
S = 𝑅( )
0.11

S = 31,139

Now

𝑃 = 67,402 − 31,139 = 36,263

1−(1+0.11)−5
Using 36,263 = 𝑅 = ( )
0.11

𝑅 = 9,811

Question: 4 If the equal monthly installment of a housing loan of Rs. 750,000 bearing
interest at 6% compounded monthly is Rs 9,500. The term of the loan is

Solution:
𝑟
1−(1+ )−12𝑡
12
𝑃= 𝑅( 𝑟⁄ )
12

1−(1+0.06)−12𝑡
750,000 = 9500( 0.06⁄ )
12

1−(1.06)−12𝑡
78.9473 = 0.005

0.3947365= 1 − (1.06)−12𝑡

−0.6052635 = −(1.06)−12𝑡

log(0.6052635) = −12𝑡 𝑙𝑜𝑔 (1.06)

−0.218055 = 12𝑡(0.02530)

0.218055 = 0.3036t

76 | P a g e
t = 0.71823

t = 0.71823 × 12

t = 8.618

t = 8 years and 6 months.

Question: 5 Mr. Raheel intend to create an endowment fund to provide for a yearly
pension of Rs. 5,000 every year. If the fund is invested in high yielding securities at
12.5% compounding interest. The amount of endowment will be

Solution:
𝑅 5,000
𝑃= =
𝑟 0.125

P = 40,000

Question: 6 What equal annual payments will discharge a debt of Rs 7,620 due in 3
𝟐
years at 𝟏𝟔 𝟑 % per annum

Solution:
1−(1+0.167)−3
𝑃 = 𝑅( )
0.167

1−(1+0.167)−3
7,620 = 𝑅( )
0.167

7,620 = 𝑟(2.22037)

R=3,431.86

Question: 7 As a result of law suit settlement, you will Receive Rs 5,000 per year for
the next ten years. If you deposit these payments immediately into an account paying an
interest rate of 6.5 percent. How much is the annuity worth to you today.

Solution:
1−(1+0.065)−10
𝑃 = 500( )
0.065

𝑃 = 35,944

Question: 8 A motor car advance of Rs. 250,000 bearing interest at 9% per annum
compounded monthly is to be paid back in 100 equal monthly installment. The equal
monthly installment will be

Solution:
1−(1+0.09⁄12)−100
250,000= 𝑅( 0.09⁄ )
12

𝑅 = 3,562

77 | P a g e
Question: 9 The amount of an annuity for 10 years is Rs. 25,000. What would be the
size of the quarterly installment if the interest rate is 9% compounded quarterly?

Solution:
𝑟
(1+ )4𝑡 −1
𝑆= 𝑅( 𝑟4 )
⁄4

0.09
(1+ )40 −1
4
25,000 = 𝑅( 0.09⁄ )
4

25,000 = R(48.8863)

R = 511.39

Question: 10 You plan to accumulate Rs. 450,000 over a period of 12 years by making
equal annual deposit in an account that pays an annual interest rate of 9% (assume all
payments will occur at the beginning of each year). What amount must you deposit each
year to reach your goal.

Solution:
(1+𝑟)12 −1
𝑆 = 𝑅( )
𝑟

(1.09)12 −1
450,000 = 𝑅( )(1 + 𝑟)
0.09

450,000= 𝑅(20.1407)(1.09)

450,000 = 𝑅(21.9533)

R = 20498.05

Question: 11 A loan of Rs 100,000 with interest @2% compounded quarterly is to pay


off in four quarterly installment. What will be the principal amount and interest pay
back in second payment.

Solution:
1−(1+0.03)−4
𝑃 = 𝑅( )
0.03

1−(1.03)−4
100,000 = 𝑅( )
0.03

R = 26,903

Now

Loan I R Principal repay


100,000 100,000×0.03=3,000 26903 23903
100,000-23,903 76,097×0.03=2283 26903 24620

78 | P a g e
Question: 12 A man borrow Rs. 12,500 from a bank at 20% compound interest. At the
end of every year he pays Rs. 2,000 as part repayment. How much does he still owe to
the bank after three such installments.

Solution:
𝑆 = 12,500(1 + 0.2)3

𝑆 = 21,600

Also
(1+𝑟)𝑡 −1
𝑆 = 𝑅( )
𝑟

(1+0.2)3 −1
𝑆 = 2,000( )
0.2

S = 7,280

Question: 13 A company is considering a project which would cost Rs. 13,000 now and
would yield Rs. 2,100 per annum in perpetuity, starting one year from now. The cost of
capital is 14% what is the net present value of the project

Solution:
2,100
𝑁𝑃𝑉 = −(13,000) + 0.14

𝑁𝑃𝑉 = -13,00 + 15000 = 2000

Question: 14 A Father of a girl planned to save some amount monthly to meet the
marriage expenditure of his girl who is now 15 years old. Father foresight marriage of
his girl at the age of 21 years with an amount of 200,000. If money grow at the rate of
12% p.a compounded monthly then find how much amount he has to solve each month.

Solution:
(1+𝑟⁄12)12𝑡 −1
𝑆 = 𝑅( 𝑟⁄ )
12

(1+0.12⁄12)12×6 −1
200,000 = 𝑅( 0.12⁄ )
12

200,000 = 𝑅(104.70993)

R = 1,910

Question: 15 A man purchased a cow for Rs. 3,000 and sol it the same day for Rs. 6,000
allowing the buyer a credit of 2 years if the rate of interest be 20% compounded semi-
annually. The man has gain of

Solution:
𝑠 = 𝑝(1 + 𝑟)𝑡

79 | P a g e
6,000
𝑝 = (1+0.20⁄ 4
2)

𝑝 = 4,098

Profit = 4,098 - 3,000 = 1,098


1,098
Gain% = 3,000 × 100 = 36%

Question: 16 An interest bearing note of Rs 5,000 dated January 2010 at 6%


compounded quarterly for 10 years was discounted on January 1, 2014. What were the
proceeds (Received) and compound discount if the note was discounted at 8%
compounded semi – annually.

Solution:
𝑆 = 𝑝(1 + 𝑟⁄4)4𝑡

𝑆 = 5,000(1 + 0.06⁄4)40

𝑆 = 9,070

Also

𝑆 = 𝑝(1 + 𝑟⁄2)2𝑡

9,070 = 𝑝(1 + 0.08⁄2)2𝑡

9,070= 𝑝(1.04)12

Also

Compound discount = 9,070 - 5,665

= 3,405

Question: 17 If money is worth 10% simple interest rate. What is the value of debt of
Rs. 3,000due in 9 months with interest at 14% after 12 months from now?

Solution:
𝑠 = 𝑝(1 + 𝑟𝑡)
9
𝑠 = 3,000(1 + 0.14 × 12)

S = 3,315

Also

𝑆 = 𝑝(1 + 𝑟𝑡)

80 | P a g e
3
𝑠 = 3,315 (1 + 0.1 × 12) = 3,398

Question: 18 If x,y,z are three sum of money such that y is the simple interest of x & z
is the simple interest on y for the same time period and at the same rate of interest, then
we have

Solution:
𝐼 = 𝑝𝑟𝑡

𝑦 = 𝑥𝑟𝑡 … … … … … . (𝑖)

Also

𝐼 = 𝑝𝑟𝑡

𝑧 = 𝑦𝑟𝑡 … … … … … . (𝑖𝑖)

From (i) and (ii)


𝑦 𝑥𝑟𝑡
= 𝑦𝑟𝑡
𝑧

𝑦 𝑥
=𝑦
𝑧

𝑦 2 = 𝑥𝑧

Question: 19 A true discount on a bill due 9 months hence at 16% per annum is Rs.
189. The amount of the bill is

Solution:
Let the present value 5 p.

Then

𝐼 = 𝑝𝑟𝑡
9
189 = 𝑝 × 0.16 × 12

𝑥 = 1575

Now

𝑆 = 𝑝 + 𝐼 = 1575 + 189 = 1,764

Question: 20 A company is currently evaluating a project which require investment of


Rs. 5,000 now and Rs. 2,000 at the end of year 1. The cash inflow from the project will
be Rs 7,000 at the end of year 3. If discount rate is 16%. What is npv of project.

Solution:

81 | P a g e
(5,000) (2,000) (7,000)
𝑁𝑝𝑣 = (1+0.16)0 + (1+0.16)1 + (1+0.16)3

𝑛𝑝𝑣 = (2240)

Question: 21 There is 80%, increase in an amount in 8 years at simple interest. What


will be the compound interest of Rs. 14,000 after 3 years at the same rate

Solution:
Given

𝑝=𝑥

𝑠 = 𝑥 + 80%𝑥 = 1.8𝑥

𝑟 =?

𝑡=8

Using

𝑠 = 𝑝(1 + 𝑟𝑡)

1.8𝑥 = 𝑥(1 + 𝑟 × 8)

1.8 = 1 + 8𝑟

𝑟 = 0.1 = 10%

Also

𝐶. 𝐼 = 𝑝((1 + 𝑟)𝑡 − 1)

= 14,000(1 + 0.1)3 − 1)

= 4,634

Question: 22 A sum of money at compound interest amount to thrice in 3 years. In how


many years will it be 9 times itself.

Solution:
𝑝=𝑥

𝑠 = 3𝑥

𝑡= 3

𝑟 =?

Using

𝑠 = 𝑝(1 + 𝑟)𝑡

82 | P a g e
3𝑥 = 𝑥(1 + 𝑟)3

3 = (1 + 𝑟)3

Taking power 1⁄2 𝑏𝑜𝑡ℎ 𝑠𝑖𝑑𝑒𝑠

1⁄ 1⁄
(3) 3 = (1 + 𝑟)3× 3

1.44= 1 + 𝑟

𝑟 = 0.44

𝑟 = 44%

Also

𝑝=𝑥

𝑠 = 9𝑥

𝑟 = 33%

𝑡 =?

Using

𝑠 = 𝑝(1 + 𝑟)𝑡

9𝑥 = 𝑥(1 + 0.44)𝑡

9 = (1.44)𝑡

𝑙𝑜𝑔9 = 𝑙𝑜𝑔(1.44)𝑡

0.9542 = 𝑡(0.1583)

𝑡 = 6 𝑦𝑒𝑎𝑟𝑠

Question: 23 Rahil invested Rs. 700 on 1 January each year starting in 2,000
compounded, interest of 10% is credited on 31 December each year to the nearest Rs.
The value of his investment on 31 December 2019 will be.

Solution:
Given

𝑅 = 700

𝑡 = 10

𝑟 = 10%

𝑠 =?

83 | P a g e
Using
(1+𝑟)𝑡 −1
𝑆 = 𝑅( )(1 + 𝑟)
𝑟

(1+0.1)10 −1
𝑆 = 700( )(1 + 0.1)
0.1

𝑆 = 12,272

Question: 24 Sale of product A in year 1 will be Rs. 6,750 units. Thereafter sale volume
will increase by 20% per annum. What will be the sale volume in year 12, to interest
unit?

Solution:
𝑆 = 𝑝(1 + 𝑟)𝑡

𝑆 = 6,750(1 + 0.2)11

𝑆 = 50,153

Question: 25 It is estimated that because of productivity improvements, cost will fall by


𝟏
𝟐 𝟐 % per annum on a compound basis. If annual costs are now 160,000 then by end of
five years. They will have fallen to the nearest

Solution:
𝑠 = 𝑝(1 − 𝑟)5

𝑠 = 160,000(1 − 0.025)5

𝑠 = 140,975

Question: 26 Freed need Rs. 5800 at the end of the five years from now. If freed can
earn 9% p.a how much should he invest now?

Solution:
𝑠 = 𝑝(1 + 𝑟)𝑡

5,800 = 𝑝(1 + 0.09)5


5,800
=𝑝
1.53

P = 3,790
Question: 27 Sum of all future annuity payments evaluating at a given discount rate is

Solution:
Present value of annuity

Question: 28 Find present value of alpha company share which is expected to yield Rs.
3.84 every 6 months. If money is worth compounded semi-annually.

84 | P a g e
Solution:
3.84
𝑃 = 0.06⁄ 𝑃 = 128
2

Question: 29 A machine cost Rs. 15,000 is to be depreciated at 16.67% per annum on


written down value. The remaining value of machine at the end of 6th year will be

Solution:
𝑠 = 𝑝(1 − 𝑟)𝑡

𝑠 = 15,000(1 − 0.1677)6

S = 5,022

Question: 30 What is the difference between compound interest on R. 5,000 for 1.5
years at 4% per annum compounded yearly and half yearly

Solution:
Difference = C.I -C.I
𝑟
Difference= 𝑝((1 + 2)2𝑡 − 1) − 𝑝((1 + 𝑟)𝑡 − 1)

0.04 2×1.5
Difference= 5,000((1 + ) − 1) − 5,000((1 + 0.04)1.5 − 1)
2

Difference= 5,000((1.02)3 − 1) − 5,000((1.04)1.5 − 1)

𝐷𝑖𝑓𝑓𝑒𝑟𝑒𝑛𝑐𝑒 = (306.04) − (302.98)

𝐷𝑖𝑓𝑓𝑒𝑟𝑒𝑛𝑐𝑒 = 3.66

Question: 31 An auto mobile financier claims to be lending money at sample interest ,


but he include the interest every six months for calculating the principal. If he is
charging an interest of 10%. The effective rate of interest becomes.

Solution:
Simple interest has no effective rate

Question: 32
A sum of money at sample interest amount to Rs 815 in 3 years and to Rs 854 in 4 years.
The sum is

Solution:
𝑝=𝑥 𝑝=𝑥

𝑠 = 815 𝑠 = 854

𝑡=3 𝑡=4

𝑟 =? 𝑟 =?

85 | P a g e
𝑆 𝑝(1+𝑟𝑡)
= 𝑝(1+𝑟𝑡)
𝑆

854 𝑥(1+4𝑟)
= 𝑥(1+3𝑟)
815

1+4𝑟
1.04 = 1+3𝑟

1.04 + 3.10𝑟 = 1 + 4𝑟

0.04 = 0.88r

𝑟 = 0.04

𝑟 = 4%

Now 𝑠 = 𝑝(1 + 𝑟𝑡)

854 = 𝑝(1 + 0.04 × 4)

854 = 𝑝(1.16)

𝑝 = 736.20

Question: 33 To find out the total compound interest accrued on a sum of money after
5 years, which of the following information given the statements A and B is/are
sufficient

A) Sum was Rs. 20,000


B) Total amount of simple interest on the sum after 5 years was Rs. 4,000
Sol.

i) only A is sufficient
ii) only B is sufficient
iii) Both A and B are needed
iv) Either A or B is sufficient
Question: 34
Companies A and B earned profits of Rs 600000 and Rs 1320000 respectively during
2009, it is estimated that future annual profit would increase @8% and 5%
respectively. In which year the profit of both companies would be equal
Solution:
S=P(1+r)t ---------(1)

S=600000(1.08)t
Also
S=P(1+r)t --------(2)

S=1320000

86 | P a g e
Compare (1) and (2)

1320,000 (1.08)𝑡
600000
= (1.05)𝑡
2.2= (1.02857)t
log 2.2 =t log(1.02857)
t=28 years
Question: 35

A cheque of Rs. 1025 is discounted from bank at a simple interest rate of 10% for 3 months before
maturity date. Find proceeds

Solution:

S = 1025, r = 10 %, t = 3/12

Using

P = s(1-rt)

P = 1025(1-0.1 x 3/12)

P = 1025 (1- 0.025)

P = 1000

Question: 36
Ghazi borrow Rs. 300 at 5% and Rs 450 at 6% simple interest at the same time and on the condition
that the whole loan will repay when the total interest amount to 105. The loan will have to be repaid
after how many years.

Solution:

I 1 + I 2 = 105

360 x 0.05 x t + 450 x 0.06 x t = 105

15t + 27t = 105

T = 2.5 years

Question: 37
Sam has taken out 30,000 mortgages over 25 years. Interest rate is to be charged at 12%. Calculate
monthly repayment

Solution:

P = R [ 1- (1 + r/12) -12 t]

r/12

30,000 = R [ 1- (1 + 0.01)-300 ]

0.01

87 | P a g e
R = 315.95

Question: 38
John Johnstone borrow Rs. 50,000 now at an interest rate of 7% p.a. The loan has to be repaid through
ten equal installments after each of next ten years. What is the annual repayment?

Solution:

S = p (1 + r) t

S = 50,000 (1.07) 10

S = 98357.56

Also

S = R [ (1 + r) t – 1]

98357.56 = R (13.81)

R = 7122.19

Question: 39
An investor puts 10,000 into an investment for 10 years. The annual rate of interest earned is 15% for
the first four years 12% for the next four years and 9% for the final two years. How much will the
investment be worth at the end of ten years

Solution:

For first four years

S = 10,000 (1 + 0.15)4

S = 17490.06

For next four years

S = 17490.06 (1 + 0.12)4

S = 27520.95

For last two years

S = 27520.95 (1.09)2

S = 32697.64

Question: 40
Naila Purchased a washing machine on installment each installment of Rs. 500 p.m. The first
instalment is paid at the time of purchase. If the interest charged is 24% p.a. Compounded monthly.
Find the cash price of the machine if deals have to be completed in one year.

Solution

88 | P a g e
P = R [ 1 – (1 + r/12) -12t ] (1 + r/12)

r/12

p = 500 [1- (1.02) -12 ] (1.02)

0.02

P = 5393.42

Question: 41
Mr Badar is making an investment of Rs 21500 every year in government bonds which would give
compound interest of 10% p.a. The redeem value of his investment after a years will be

Solution:

S = 21500 [ (1 + 0.1) 9 -1]

0.1

S = 291958.75

Question: 42
Sales for the first five months of the year average 8200 per month. For the last four months of the
year’s sales averaged 2500 per months. If sales for the year totaled 102,000. The average for the sixth,
seventh and eight months must be

Solution:

= 10200 – (8200 x 5) – (8500 x 4)

= 10200 – 41000 – 34000

= 27000

= 9000

Question: 43
A sum was put at simple interest at a certain rate for 2 years, had it been put at 3% higher rate, it
would save fetched 72 more. The sum is

Solution:

Prt - prt =72


𝑟
X (r + 3)% x 2 - 𝑥 1𝑤 × 2 = 72

𝑥(𝑟 + 3) ∗ 2 2𝑥𝑟
− = 72
100 100
𝑥(𝑟 + 3) 𝑥𝑟
− = 72
50 50

89 | P a g e
3x = 72 x 50

3x = 3600

X = 1200

Question: 44
The management of a school having 1000 students estimated that the number of students will grow at
the rate of 6% p.a. Presently a monthly fee of Rs 500 is charged from each student. The total
expenditure of school is Rs 5.5 million per year. It is estimated that expenditure will increase by 8%
per year. How much fee the school will have to charge in the 10th year if it wishes to each a profit of
Rs. 9.6 million

Solution:

Step 1:

Number of students expected at the end of 9th year to the beginning of 10th year

S = p (1 + r) t

S = 1000 (1 + 0.06) 9

S = 1689

Step 2:

Expected expenditure in 10th year is

S = P (1 + r) t

S = 5500,000 (1 + 0.08) 10

S = 11874087

Step 3:

Total expenditure and profit required at the end is

= 9600000 + 11874067

= 21474087

Step 4:

Per month fee in 10th year is


21474087
= 16989×12

= 1060

Step 5:

Rate of increase in fee

90 | P a g e
S = p (1 + r) t

1060 = 5.0 (1 + r) 9

r= 8.7%

91 | P a g e
ICAP BOOK QUESTIONS
1 Mr. X borrowed Rs.5.120 at 121/2 % pa. C1. At the end of 3 years the money was repaid along with
the solleret accrued. The amount of interest paid by him is:

a) Rs. 2,100 b) Rs. 2,170


c)Rs. 2000 d) None of these
2. Dawood has to repay a loan along with interest, three years from now. The amount payable after
three years is Rs. 428,000. The amount of loan presently if interest rate is 8% compounded Semi-
annually is:

a) Rs. 345,161 Rs. 339,760


c) Rs. 338,254 d) Rs. 336,421
3. The banker's interest to the nearest paisa's Principal: Rs.2500: Rate: 9%; Time 180 days: Interest
is:

a) Rs. 104.32 b) Rs. 109.75


c) Rs. 112.50 d) Rs. 110.96
4. The maturity value of a loan of Rs. 2,800 after three years. The loan carries a simple interest rate
of 7.5% per year is:

a) Rs. 3,429 b) Rs. 3,430


c) Rs. 3,431 d) Rs. 3,440
5. To increase present value, the discount rate should be adjusted:

a) Upward b) Downward
c) Not relevant d) will depend on time period
6. A borrower has agreed to pay Rs 10,000 in 9 months at 10% simple interest. How much did this
borrower receive?

a) Rs. 9,090 b) Rs. 9,250


c) Rs. 9,500 d) 9,302
7. Bashir owes Rs.50,000 to Arshad due to a court decision. The money must be paid in 10 months
with no interest. Suppose Bashir wishes to pay the money now. What amount should Arshad be
willing to accept? Assume simple interest of 8% per annum.

a) Rs. 45,875 b) Rs. 47,875


c) Rs. 46,875 d) Rs. 46,575
8. Mr. Junaid received Rs. 48,750 in cash as the proceeds of a 90 days loan from a hank which
charges 10% simple interest. The amount he will have to pay on the maturity date is (assume
366days in a year):

a) Rs. 49,969 b) Rs. 50,000


c) Rs. 47,548 d) Rs. 53,625
9. Present Value of a future cash flow is the amount that:

a) Investor invests in future to get the amount b) Investor invests today to get the amount in
today future
c) Is not equivalent to the value of future cash d) Is not equivalent to the value of present cash
flow in present day flow same time in the future

92 | P a g e
10. How would the future value of an amount invested today change in 8 years if the amount is
invested at the rate of 9% compound interest?

a) Double b) Half
c) Quadruple d) will remain same
11. Laura had invested Rs. 250,000 in a land ten years ago. Now, she wants to evaluate the worth of
the land she bought. What will be the value of land today if it is compounded at 6% per annum?

a) Rs. 265,000 b) Rs. 235,850


c) Rs. 447,712 d) Rs. 416,667
12. A three-year investment yields a return of 25% over the period. Compute the effective annual
rate of return.

a) 8.33% b) 7.72%
c) 6% d) 5%
13. The annual cost of a company is Rs 500,000 at the moment. It is expected that due to re-
engineering the cost will reduce by 20% per annum for next two years. Compute the annual cost for
fourth year from now.

a) Rs. 320,000 b) Rs. 163,840


c) Rs. 200,000 d) Rs. 260,000
14. Value of a particular asset decreases by 10% every year of its cost. If the cost is Rs 700,000 today,
compute its value two years from today.

a) Rs. 630,000 b) Rs. 560,000


c) Rs. 567,000 d) Rs. 500,000
15. A sum of Rs. 100,000 is invested at 10% per annum compounded quarterly. Compute the
amount to which it will grow after 3 years.

a) 134,489 b) 143,984
c) 413,489 d) 489,134
16. The present value of Rs. 5,000 at 16% per annum for 4 years is:

a) Rs. 18,000 b) Rs. 3,761.46


c) Rs. 4,600 d) Rs. 2,761.46
17. The discount factor for 3 years at 10% is:

a) 0.751 b) 0.683
c) 0.826 d) 0.909
18. Taha invested Rs 200,000 for 5 years at simple interest at 10%. At the end of this year he
withdraw entire amount and invested at 10% per annum compound interest for 2 years. compute
the amount that have with him at the end of 7th year from now.

a) Rs. 463,000 b) Rs. 363,000


c) Rs. 563,000 c) Rs. 663,000
19. Marfani lends Rs 500,000 to Taha at 12% per annum compounded annually for 3 years and Rs.
300,000 to Jawad at 10% per annum compounded quarterly for a period of 3 years. Compute the
effective annual rate of interest for amount lent to Jawad

a) 10% b) 12.38%

93 | P a g e
c) 10.38% d) 11.38%
20. Luqman Invests Rs 450,000 at 10% simple interest for 4 years and Rs 900,000 at 10% compound
interest for the same time period. Compute the difference between the receipts of two investments
at the end of 4th year.

a) Rs. 417,690 b) Rs. 687,690


c) Rs. 450,000 d) Rs. 867,690
21. A sum of money invested at compound interest amounts to Rs 5,280 in 2 years and to Rs 5,808
in 3 years. The sum of money is:

a) Rs. 4,363.64 b) Rs. 4,080


c) Rs. 4,750 d) Rs. 4,500
22. Find the future value of Rs. 7,000 invested at 10% per annum compounded quarterly for three
years and further invested at 12% per annum compounded semi-annually for two years.

a) Rs. 11,858.24 Rs. 11,885.24


c) Rs. 11,588.24 d) Rs. 11,558.24
23. Which is more in present value terms if interest rate is 16% per annum compounded annually?

i) Rs 5,000 one year from now.

ii) Rs 10,000 in three years from now.

iii) Rs, 15,000 in two years from now.

iv) d) Rs 20,000 in four years from now.

a) i) b) ii)
c) iii) d) iv)
24. Which is most in future value terms if interest rate is 16% per annum?

i) Rs 5,000 invested for one year.

ii) Rs 10,000 invested for three years.

iii) Rs 15,000 invested for two years.

iv) Rs 20,000 invested for four years.

a) i) b) ii)
c) iii) d) iv)
25. Mr. Khalid invested Rs. 200,000 at 12% simple interest and Rs. 189,744 at 14% simple interest.
The two amounts were left to grow till such time that the total of interest and principal of both
investments become equal.

a) 4 b) 3
c) 2 d) 5
26. Mr. Asim has following investment option:

i) Invest in Bank A at 12% per annum ii) Invest in Bank B at 10% per annum
compounded quarterly compounded semi-annually

94 | P a g e
iii) Invest in Bank C at 8% per annum iv) Invest in Bank D at 15% per annum
compounded monthly
Which of the above options must he opt

a) Bank A b) Bank B
c) Bank C d) Bank D
27. Compute the effective annual rate of following options:

i) A: Investment of Rs. 200,000 resulting in a single 25% return on investment after 4 years.

ii) B: Investment of Rs. 300,000 resulting in a 15% per annum compounded semi-annual return.

a) A: 5.74% B: 15.56% b) A: 5.54% B: 15.56%


c) A: 5.64% B: 14.56% d) A: 5.64% B: 15%
28. Bakar has to pay Zeeshan Rs. 900,000 three years from today. If the market interest rate is 10%
per annum compounded annually how much will Zeeshan be willing to accept today to settle the
debt

a) Rs. 676,183 b) Rs. 576,183


c) Rs. 767,183 d) Rs. 876,183
29. Sheeraz bought a bike on cash basis for Rs. 170,000. The seller is willing to offer it on following
terms as well:

i) Pay Rs. 200,000 at the end of 1st year.

ii) Pay Rs. 300,000 at the end of 4th year.

iii) Pay Rs. 430,000 at the end of 5th year

If the market interest rate is 17% per annum compounded annually. Shall Sheeraz accept any of the
offers given by the dealer, if yes which offer is most beneficial?

a) No b) Yes, Option # i
c) Yes, Option # ii d) Yes, Option # iii
30. Asim bought a machine for Rs 500,000. The value of the machine decreases by 15% per annum
of its net book value. The life of the machine is three years. At the end of its life Asim will sell it and
the expected receipts from disposal will be 12% above its net book value. Asim will also need a
replacement machine which will cost Rs 600,000 in future value terms. How much more will he need
at the end of three years to buy the machine, if the proceeds from initial machine will also be used
to pay for the new machine?

a) Rs. 331,525 b) Rs. 600,000


c) Rs. 213,525 d) Rs. 256,090
31. Rs. X are invested for 7 years at Y% per annum compound interest. Compute future value in
terms of x and y.

a) X(1 + y/ 100)7 b) X(1+100/Y)7


c) Y(1+x/100)7 d) Not possible to determine

95 | P a g e
ICAP BOOK SOLUTIONS:
1. C.I = P [(1 +r) t – 1]

= 5120 [(1+12.5%) 3 – 1]

C.I = 2170
𝒔
2. p =
𝒓 𝟐𝒕
(𝟏+ )
𝟐

P = 338,254

3. I = p r t
180
I = 2500 x 0.09 x 365

I = 110.96

4. S = P (1 + rt)

S = 2800 (1 + 7.5% x 3)

S = 3430

5. Downward

6. S = p (1 + rt)

10,000 = p (1 + 0.1 x 9/12)

10,000 = p (1.075)

P = 9302
𝒔
7. P = 𝟏+𝒓𝒕
50,000
P=
1+0.08× 10/12

P = 46875

8. S = p (1 + rt)

S = 48750 (1 + 0.1 x 90/360)

S = 49969

9. S = (1 + r) t

= x (1 + 0.09) 8

96 | P a g e
S = 2x

10. S = p (1 + r) t

S = 250,000 (1 + 0.06) 10

S = 447712

11. Since Nominal rate is not given

S = p (1 + r)t

125 = 100 (1+r)3

1.25 = (1+r)3

r = 7.72%

12. Year expense

0 500000

1 500000 x 80% = 400,000

2 400000 x 0.80 = 320,000

3 320,000

4 320,000

13. S = P (1 - r) t

S = 700000 (1 – 0.1) 2

S = 567000

14. S = p (1 + r/4) 4t

= 100,000 (1+ 0.1/4) 4x3

S = 134489

𝑺
15. P =
(𝟏+𝒓)𝒕

50000
P = (1+0.16)4

P = 3761.46

𝟏
16. D.F =
(𝟏+𝒓)𝒕

97 | P a g e
1
D.F =
(1+0.1)3

D.F = 0.751

17. S = p (1 + rt)

S = 200,000 (1 + 0.1 x 5)

S = 300,000

Also

S = p (1 + r) t

S = 300,000 (1 + 0.1)2

S = 363,000

18. Given:

r = 10%

e = (1 + r/4) 4 – 1

e = [ (1+10%/4) 4 – 1] x 100%

e = 10.38%

19.

P = 450,000 P = 900,000

r = 10% r = 10%

t=4 t=4

using using

s = p (1 + rt) s = p (1 + r)t

S = 450,000(1+ 0.1 x 4) s = 900,000 (1 + 0.1)4

S = 630,000 s = 1317690

Difference = 687,690

98 | P a g e
20.

P = 5280

S = 5808

t=1

r=?

Using

S = P (1 + r)t

5808 = 5280 (1 + r)t

1.1 = 1 + r

0.1 = r

r = 10%

Now:

S = 5280

P=?

r = 10%

t=2

using:

S = P(1 + r) t

5280 = p (1 + 0.1) 2

P = 4363.64

21.

Given r = 12%
P = 7000 using
t=3 s = p(1 + r/2) 2t
r = 10% s = 9414.22(1 + o.12/2) 2x2
using s = 11885.24
s = p (1 + r/4) 4t
= 7000 (1 + 0.1/4)4x3
S = 9414.22

99 | P a g e
Also:

P = 9414.22

S=?

t=2

22. Given

200,000 (1 + 12%t) = 189744 (1 + 14% t)

1.054 (1 + 0.12t) = 1 + 0.14t

1.054 + 0.126t = 1 + 0.14t

0.054 = 0.014t

t=4

23. For A

Given use formula because Nominal Rate 5 not given

P = 200,000, s = 200,000 x 25% = 250,000, t = 4

Using

S = p (1 + r)t

250,000 = 200,000 (1 + r)4

r = 5.74%

For B:

e = [ ( 1 + r/2)2 – 1] x 100%

e = [ (1 + 0.15/2)2 – 1] x 100%

e = 15.56%

24. s = 900,000, t = 3, r = 10%

Using S = p (1 + r) t

900,000 = p (1 + 0.1) 3

P = 676183.32

100 | P a g e
25. Given

P = 500,000, r = 15%, t = 3

Using

S = p (1 - r) t

S = 500,000 (1 – 0.15) 3

S = 307062.5

12% increase in net book value

S = 343910

Also remaining amount for replacement

= 600,000 – 343910

= 256,090

26. P = x, r = y% , t = 7, s = ?

Using

S = p (1 + r) t

S = x (1 + y%) 7

S = x (1 + y/100) 7

101 | P a g e
CHAPTER :5-6-MATHEMETICS OF FINANCE
1) Tahir took Rs. 200,000 from Bank at 13.5% rate per year. What total amount he will have to
pay after 5 years to Bank?

a) 335,000 b) 356,700

c) 337,500 d) None of these

2) Tina invested Rs 4000 per year (starting from 2nd year) @ 12% compounded annually for 8
years. Find the present value of all those savings
a) 17741.5706 b) 197,867
c) 299,857 d) None of these

3) Tom and Jerry both invested same amount for 8 years. If Tom’s rate of return is 9% and
Jerry’s rate is 10% compounded annually then find how much more Jerry will have after 8
years than Tom

a) 5.7% b) 7.6%

c) 15.1% d) 6.7%

4) A woman invested Rs 5000 every year for 10 years at 10% per annum, find total amount available
to her after 10 years.
e) 79,687.12 f) 97,867
g) 99,857 h) None of these

5) Suppose an amount of Rs. X is invested and he receives Four times of x ,interest rate is 10%
compounded annually. How long it requires to be invested?
a) 13 years b) 14 years 6 month
c) 11 years 6 months d) 12 year 3 month

6) Rowen borrows Rs. 500,000 from his friend Hasid for the period of seven year and Hasid
receives 896,793Rs. If interest rate was 10% for the last four year find the interest rate of first
3 years?
a) 7% b) 12%
c) 10% d) 15

7) The company offers a bond of Rs. 200,000 for five years. the Company will pay Rs 300,000
at maturity date, interest rate is 8%, Whether this is feasible for the company along with
reasons?
a) Yes, because PV is greater than b) No because PV is greater than
200,000 200,000

c) On Break even d) None of these

8) A Mother wants Rs 300,000 after 3 years from now & Rs 500,000 after five years from now
Separately. if interest rate is 12% compounded quarterly, what would be the amount of
installment to be deposited by her at the start of 1st, 8th and 13th Quarter?
a) 38588.82, 38588.82, 18065.88 b) 30588.80, 30588.80, 18065.80
c) 38588.80, 20522.94, 18065.80 d) None of these

102 | P a g e
9) If present value of a unpaid bill of Rs. 650,000 reaches to 1,060,000 in 6 years, and interest
rate in last four year charged at the rate of 10% compounded annually. So, what interest
should charge in first 2 years compounded annually?
a) 8.6% b) 7%
c) 5.54% d) 6.4%
10) Mena has invested Rs. 700,000 in an investment scheme. In return, she would receive Rs.
74,587 semi-annually in arrears, for the six years. She would not receive any amount
afterwards. Find the nominal and effective rate of return of the scheme.
a) 8% and 8.16% b) 8% and 9.16%
c) 8% and 10.16% d) None of these
11) A person makes investment of Rs 8,000 now and in next two years at 10% compounded annually
find his total investment at end of three years
a) Rs 29,000 b) 28,000
c) 27,000 d) 26,000
12) If Rs. 200,000 is to grow to Rs. 649,464 in ten years’ period, at what annual interest rate
must it be invested, what is the effective rate of interest is compounded semi-annually?
a) 15.15% b) 12.5%
c) 10.5% d) 25.5%

13) A company is considering whether to invest in a new item of equipment costing Rs. 45,000
to make a new product. The product would have a four-year life, and the estimated cash
profits over the four-year period are as follows.
Year Rs.

1 17,000
2 25,000
3 16,000
4 4,000
The project would also need an investment in working capital of Rs. 8,000, from the beginning of
Year The company uses a discount rate of 11% to evaluate its investments Using the NPV you have
calculated at 11%, and the NPV at a discount rate of 15%, estimate the internal rate of return (IRR) of the
project.

A) 16.5% B) 18%
C) 16.73% D) 20%

14) Amount= Rs 2500, Interest=8% compounded monthly, find perpetuity.?


a) 377,500 b) 375,000
c) 75,0000 d) Indefinite

15) Which of the following is true about perpetuity?


a) It has unlimited time series b) It has no future value

c) Used to find purchase price of a share d) All of these

16) A bank offers to pay 500,000 after 8 years if interest rate is 9% compounded annually then
value of money to be paid to bank today will be?

a) 290,697.67 b) 250,933.14
c) 996,281.32 d) None of these

17) Suppose an amount of Rs. X is invested in a bank @10% compounded annually rate. How
long would it be invested if 3 times amount is required?

103 | P a g e
a) 12.8 years b) 11.52 years
c) 15 years d) 19 years

18) Two companies made profits from investments in different projects:


Year 1 2 3 4
Company A 98,000 95,000 150,000 -
Company B 85,000 90,000 70,000 80,000
Find the rate of at which NPV of both companies will be same
a) 10% b) 20%
c) 30% d) None of these

19) A person invested some amount today @0.7% per month compounded for 10 years find his
investment if he receives Rs. 10 million
a) 9.324 million b) 4.13 million
c) 4.329 million d) None of these

20) A person invested some amount today @ 1.8% per quarter for 10 years find his investment if
he receives Rs 10 million.
a) 8.356 million b) 4.13 million
c) 4.899 million d) None of these

21) 𝑃 = 500,000 𝑟 = 1.5% simple interest per month time = 5 years find total amount
a) 950,000 b) 770,000
c) 351,000 d) None of these

22) If 500,000 is invested in a scheme @ 1.5 per month for 5 years then
a) He will get 150,000 per month b) He will get 75,000 per month
c) He will get gain of Rs 450,000 after d) All of these
5 years

23) A person deposited Rs. 600,000 in a bank @ 9% simple interest for 3 years and 3 months.
Calculate the amount of money he received at the end of period.
a) 793,939.425 b) 675,925
c) 775,500 d) None of these

24) Ali and company bought a bond of Rs. 200,000 five-year age. The company received Rs.
300,000 at maturity. interest rate is 85%, whether the company is in profit?
a) yes b) no
c) data insufficient d) none of these

25) if P.V of a bill is RS. 500,000 reaches to 860,000 in 7 years, and interest rete charged in last 4
years is 10% compounded annually that what interest rate was charged in first 3 years,
compounded annually

a) 5.52% b) 5.12%
c) 6.25% d) 10%

26) A note has face value of Rs. 90,000 @ 8% compounded quarterly it will mature at 9.5 years
calculate amount of interest.
a) 56,114.9 b) 48,925.3
c) 12,345.2 d) 101,007

104 | P a g e
27) Two companies made different investments in different projects:
Year 1 2 3 4
Company A 80,000 90,000 100,000 -
Company B 85,000 50,000 75,000 95,000

a) 17.88% b) 87.17%
c) 37.88% d) None of these

28) To increase present value of project t the rate should be adjusted


a) Upward b) Downward
c) Depend on duration d) Depends on rate

29) Asif borrowed Rs 100,000 and promised to pay Rs 1,000 each month to settle the obligation. If
interest rate is 1.8% compounded monthly find the time required to settle the obligation.
a) 3 years b) 4 years
c) 5 years d) 9 years

30) You are given the sum of annuity due of Rs. 200,000 find the sum of ordinary annuity if r =
12% compounded quarterly
a) 194,174.76 b) 200,000
c) 206,000 d) None of these

31) You are given the following data


𝑅 = 𝑥, 𝑖 = 10% 𝑛 = 5 present value of annuity Due is required
a) 4.169865x b) 41.16985x
c) 14.9865x d) None of these

32) An investment of 1.5 million is made in a business for 4 years and gain is 0.5 million find rate
of gain
a) 7.456% b) 4.754%
c) 6.546% d) None of these

33) Haiyan invested Rs. 400,000 in an investment scheme and got Rs. 545,881 at the end of three
years. Find effective rate, if interest was compounded monthly
a) 10.41% b) 10.92%
c) 10% d) None of these

34) Ali borrowed Rs. 1,660,000 on first year he returned Rs. 80,000 and then he increases his
instalment by 20% of the previous instalment every year.
In how many years he will be able to return the loan?

a) 9 years b) 8 years
c) 10 years d) None of these

35) A loan is borrowed of Rs. 800,000 for 4 years and simple interest payable on loan is 650,000
find interest rate
a) 18.2% b) 18.3%
c) 18.03% d) 20.3%

36) Ali borrowed Rs. 1,500,000 on first year he returned Rs 80,000 and then increases his
installment by 1.5 times of the previous installment every year. In how many years he will be
able to return the loan?

105 | P a g e
a) 9 years b) 8 years
c) 10 years d) 6 years

37) Mr. Adel saved Rs x in January, then each subsequent month he saved Rs 100 more than the
previous month. If his total savings at the end of December stood at Rs 16,200 how much did
he save in January?
a) 700 b) 800
c) 900 d) 1,000

38) Given normal rate 10% compounded monthly effective rate=?


a) 10.47% b) 14.07%
c) 47.1% d) None of these

39) Question 𝑆0 = 1,500,000 R = 80,000 r = 10% n = ?.


a) 11 b) 20
c) 10 d) None of these

40) Cash outflow = 2.5 million


Cash inflow = 0.2 million (1st year) 0.3 million (2nd) 2.9 million (3rd)
Find IRR
a) 11.7% b) 21.7%
c) 33.3% d) None of these

41) Cash outflow = 2,500,000


Cash inflow = 200,000 (3 rd year) 290,000(6th year) 2,900,000(9th year)
Find IRR
a) 3.71857% b) 13.71857%
c) 31.74857% d) None of these

42) If discount rate would increase, then present value of annuity will
a) Increased upward b) Decrease downward
c) Remain same d) None of these

43) A company invested 3 million. Interest rate was 10%, 12% and 14% per year for first, second
and third year respectively. Find NPV if cost of capital is 10%
a) 0.15 million b) 0.1656 million
c) 0.10 million d) 0

44) Azam & Moazzam invested Rs 10 million each compounded quarterly for 5 years. After 5 year
Azam has 20% more than Moazzam. Azam rate is 10%. Find rate of Moazzam.
a) 5.25% b) 5.45%
c) 6.28% d) None of these

45) Awais made an investment of Rs 1.2 million @ 12% compounded quarterly & Salman made
invested of Rs. 1.8 million @ 10% compounded annually. In how many years Awais’s
investment would exceed Salman’s investment.
a) 17348 years b) 17.68 years
c) 18 years d) None of these

46) A person has current savings of Rs. 500,000 and he also deposit 25,000 quarterly @ 12%
compounded quarterly for 10 years. Find total amount.
a) 3516050.389 b)

106 | P a g e
c) d)

47) Following 3 options are available for an investment for 4 years.


i) 9.7% simple interest
ii) 9.3% compounded quarterly
iii) 9.4% camp semi annually
Which option is best?
a) (i) b) (ii)
c) (iii) d) All are same

48) Initial investment is 170,000 @8% for 5 years find total amount
a) 149,785.77 b) 559,785.77
c) 249,785 d) None of these

49) 𝑃𝑉 = 𝑥 find if rate = 10% per year and time is 4 years.


a) 1.4x b) 1.14x
c) 0.4x d) 0.14x

50) Initial investment = 500,000 R=25,000 m=12 n=10 year


Rate = 12% compounded monthly find total in 10 years
a) 7,401,160.68 b) 8,401,260
c) 3,7401,601.68 d) None of these

51) If R=9% compounded monthly R=3,000 perpetuity=?


a) 1,140,000 b) 111,000
c) 400,000 d) None of these

52) Ali deposited 200,000 for seven years in an investment scheme and got 300,000 at the end of
maturity. If he received 8% interest compounded bi-monthly during the last three years at what
rate of interest he requires to balance the amount during the first four year compounded
bimonthly.
a) 4.2% b)
c) d)

53) Present value of a certain amount @7% for 7 years is “X”. what will be the present value of
same amount for 5 years.
a) 1.14x b) 2.24x
c) 14x d) 0.14x

54) R=10% n=4 R=x 𝑝𝑜 =?


a) 𝟑. 𝟏𝟔𝒙 b) 1.16𝑥
c) 13.6𝑥 d) None of these

55) What is the effect in NPV if discount rate is increased?


a) NPV will decrease b) NPV will increase
c) Both a & b d) None of these

56) What are the quality of perpetuity?


a) Used to find purchase price of share b) Used to find value of maintenance
fund
c) Used to find initial deposit required d) All of these
for pension scheme

107 | P a g e
57) From perpetuity we can’t find
a) Present value b) Future value
c) Both d) None of these

58) Mr. Ali borrows Rs. 1,600 for one year from a loan company. He is given only Rs. 1,560 and
is expected to repay the Rs. 1,600 at the end of one year. What is the simple discount rate?
Give the simple discount rate as a percent.
a) 3.5% b) 2.5%
c) 3.9% d) 3.2%

59) Calculate the net present value of a project which requires an initial investment of Rs. 243,000
and it is expected to generate a cash inflow of Rs. 50,000 each month for 12 months. Assume
that the salvage value of the project is zero. The target rate of return is 12% per annum.
a) Rs. 319,754 b) Rs. 419,754
c) Rs. 319,123 d) Rs. 219,744

60) Find the IRR of an investment having initial cash outflow of Rs. 213,000. The cash inflows
during the first, second, third and fourth years are expected to be Rs. 65,200 Rs. 96,000 Rs.
73,100 and Rs. 55,400 respectively.
a) 11.25% b) 12.15%
c) 13.12% d) 14.67%

61) A piece of equipment cost a certain factory Rs. 600,000. If it depreciates in value, 15% the first
year, 13.5% the next year, 12% the third year, and so on, what will be its value at the end of 10
years, all percentages applying to the original cost? (Hint: Arithmetic Progression)
a) 200,000 b) 405,000
c) 105,000 d) 650,000

62) Mr. Ahmed plans on retiring on his 60 th birthday. He wants to put the same amount of funds
aside each year for the next twenty years – starting the next year – so that he will be able to
withdraw Rs. 50,000 per year for the next twenty years once he retires, with the first
withdrawal on his 61st birthday. Ahmed is 20 years old today. How much must he set aside
each year for his retirement if he can earn 10% on his funds?
a) 2,135.26 b) 1,500.23
c) 1,104.75 d) 1,401.14

63) Ali purchased a new car and made a down payment of Rs. 50,000. He is further required to pay
Rs. 30,000 at the end of each quarter for five years. The cash purchase price of the car, if the
quarterly payment include 12% interest compounded quarterly is
a) Rs 498,324.25 b) Rs. 496324.25
c) Rs. 497,324.25 d) None of these

64) What is the present value of X Limited’s share which is expected to earn Rs 5.60 every month,
if money is worth 4% p.m.?
a) Rs. 138.61 b) Rs. 69.31
c) Rs. 140 d) None of these
65) A loan was repaid in 7 annual installments of Rs. 168 each. If the rate of interest be 10% per
annum, compounded annually, the sum borrowed was:
a) 850.1 b) 817.9
c) 1593.8 d) 936.3

66) An amount of Rs. 3,000 is due in 5 years from now. If the interest rate is 6% compounded
semi-annually, what is the present value?

108 | P a g e
a) 2,232.28 b) 2,553.35
c) 2,487.26 d) 2,264.26

67) A company wishes to replace a machine in five years’ time at an estimated cost Rs. 30,000.
The company can earn interest of 15% per annum on money invested in a fund. How much
must be invested at the end of each of five years in order to have sufficient funds to replace the
machine
a) 4,806 b) 4,449
c) 4,080 d) 5520

68) Mr. Akbar intends to create an endowment fund to provide for a yearly pension of Rs. 4,000
every year. If the fund is invested in high yielding securities at 7.5% compound interest, the
amount of endowment will be
a) 43,333 b) 35,333
c) 53,333 d) 33,333

69) Jameel invested a certain sum of money in a simple interest bond whose value grew to Rs. 300
at the end of 3 years and to Rs. 400 at the end of another 5 years. What was the rate of interest
in which he invested his sum?
a) 12% b) 8.33%
c) 6.67% d) 6.25%

70) A man borrows Rs. 12,500 from a bank at 20% compound interest. At the end of every year.
He pays Rs. 2,000 as part repayment. How much does he still owe to the bank after 3 such
installments?
a) Rs. 15,600 b) Rs. 23,864
c) Rs. 12,000 d) None of these

71) Mrs. Ahmed bought a sewing machine by paying Rs. 50 each month for 10 months, beginning
from now. If money is worth 12% compounded monthly, what was the selling price of the
machine on cash payment basis?
a) 411.22 b) 256.22
c) 715.33 d) 478.30

72) A research foundation was established by a fund of Rs. 500,000 invested at rate that would
provide 20,000 payments at the end of each year forever. What interest rate was being earned
on fund?
a) 5% b) 3%
c) 4% d) 6%

73) To find out the total compound interest accrued on a sum of money after 5 years, which of the
following information’s given in the statements P and Q will be sufficient?
Statement P: The sum was 20,000
Statement Q: The total amount of simple interest on the sum after 5 years, was 4,000.
a) Only P is sufficient b) Only Q is sufficient
c) Either P or Q is sufficient d) Both P and Q are needed
74) The nominal interest rate compounded semi-annually deposits of Rs. 500 will accumulate to
Rs. 6,000 in 5 years. The rate of interest is: (Hint: sum of annuity)
a) 7% b) 8%
c) 7.98% d) 7.42%

75) When interest is being calculated on principal and accumulated interest, the situation is called
a) Compound interest b) Simple interest

109 | P a g e
c) Both a and b d) None of these

76) Which of the following is incorrect about perpetuity?


a) It is used to find stock price b) Used in pension schemes
c) In perpetuity we find present value d) In perpetuity we find future value

77) A person invests Rs. 8,000 per year at the start of year @ 10% compounded annually for 5
years, find his total investment at the end of 5 years.
a) 53724.88 b)
c) d)

78) Rate of interest compounded annually which gives the same amount of interest as obtained by
nominal rate compounded over number of conversion periods is:
a) Equivalent Rate b) Effective Rate
c) Nominal Rate d) Exponential Rate

79) An annuity of Rs. 500 payables at the end of each quarter amounts to Rs 24,983.75 in 7 years.
What is the nominal rate of interest if interest is compounded quarterly?
a) 0.04 b) 0.05
c) 0.16 d) 0.25

80) Calculate the net present value of a project which requires an initial investment of Rs. 243,000
and it is expected to general a cash inflow of Rs. 50,000 each month for 12 months. Assume
that the salvage value of the project is zero. The target rate of return is 12% per annum.
a) Rs. 319,754 b) Rs. 419,754
c) Rs. 319,123 d) Rs. 219,744

81) If a sum of money is invested for the same period of time, which of the following will yield
maximum amount of interest?
a) 8.4% compounded semi-annually b) 8.3% compounded quarterly
c) 8.2% compounded monthly d) 8.1% compounded daily

82) A building society offers a low start mortgage of Rs. 40,000 with 10 annual repayments
starting one year from the loan being taken out. The interest rate applying throughout will be
11% per annum, but the repayments will only be Rs. 5,000 per annum for the first five years.
What equal annual payments will be required in each of the last 5 years of mortgage?
a) 5815 b) 7710
c) 9812 d) 11810

83) Shahid (Pvt.) Ltd has deducted the net present value of potential investment project at two
discount rates. The relevant data are as follows:
Discount rate Net present value
20% 60
30% (120)
(i) What is the approximate internal rate of return of the project =?
a) 20% b) 23.3%
c) 26.6% d) 30%

84) Mr. Raheel intends to create an endowment fund to provide for a yearly pension of Rs. 5,000
every. If the fund is invested in high yielding securities at 12.5% compound interest, the
amount of endowment fund will be:
a) 39,500 b) 40,000
c) 40,500 d) Can’t be determined

110 | P a g e
85) Basit took a certain amount as a loan from a bank at the rate of 8% per annum simple interest
and give the same amount to Ahmad as a loan at the rate of 12% per annum simple interest. If
at the end of 12 years, he made a profit of Rs. 320 in the deal, what was the original amount?
a) 2,000 b) 3,000
c) 4,000 d) None of these

86) The difference between the interests received from different banks on Rs. 500 for 2 years is Rs.
2.50. the difference between their rate is:
a) 1% b) 0.5%
c) 0.25% d) 2.5%

87) A owes B, Rs. 1573 payable 1 & ½ years hence, Also B owes A Rs. 1,444.50 payable 6
months hence. If they want to settle the account forth with, keeping 14% as the rate of interest,
then who should pay and how much?
a) 28.50 b) 37.50
c) 50 d) 75

88) The compound interest on a sum for 2 years is Rs. 832 and simple interest for the same sum for
the same period is Rs. 800. The difference between the compound interest and the simple
interest for 3 years will be:
a) Rs. 48 b) Rs. 66.56
c) Rs. 98.56 d) None of these

89) A sum of Rs. 12,000 deposited at compound interest becomes double after 5 years. After 20
years it will become:
a) Rs. 120,000 b) Rs. 192,000
c) Rs. 124,000 d) Rs. 96,000

90) Mr. X took a loan at 10% p.a S.I. after 4 years, he returned the principal along with interest. If
he returns in all Rs. 3500, what is the principal amount?
a) 3,250 b) 2,500
c) 3,150 d) 2,100

91) What is the PV (today) of perpetuity if next year’s cash flow is expected to be Rs.2,398 and the
discount rate is constant at 12%?
a) 19,983.33 b) 30,983.33
c) 20,983.33 d) 21,983.33

92) There is 60% increase in an amount in 6 years at simple interest. What will be the compounded
interest of Rs. 12,000 after 3 years at the same rate?

a) 3,972 b) 2,160
c) 3,120 d) 6,240

93) Farhan borrowed Rs. 100,000 for one year at 12% annual interest compounded monthly. The
loan is to be paid in equal instalments. The amount of each instalment, principal repayment in
first instalment and total interest paid during the year are:
a) 8884.86, 6618.32, 7,884.86 b) 7,884.86, 8884.89, 6618.32
c) 7895.51, 6618.32, 4213.23 d) 8884.86, 7884.86, 6618.32

94) Have I got a deal for you! If you lend me Rs 60,000 today, I promise to pay you back in
twenty-five annual installments of Rs 5,000, starting five years from today (that is, my first

111 | P a g e
payment to you is five years from today). You can earn 6% on your investments. Will you lend
me the money?
a) No, you will not lend b) Yes, you will lend
c) Date is incomplete d) Both options are same

95) A loan company is willing to lend you Rs. 10,000 today if you promise to repay the loan in six
monthly payments of Rs. 2,000 each, beginning today. What is the effective annual interest
rate on trust worthy’s loan terms?
a) b)
c) d)

96) The nominal interest rate compounded semi-annually deposits of Rs. 500 will accumulate to
Rs. 6,000 in 5 years. The rate of interest is:
a) 7% b) 8%
c) 7.98% d) 7.42%

97) The indebtedness at any time in amortization is called


a) Principal repayment b) Opening balance
c) Closing balance d) Outstanding balance

98) A debt of Rs. 12,000 is to be amortized by equal payments at the end of every six months for 3
years. If the interest charged is 6% compounded semi-annually, find the outstanding loan after
the 4th payment?
a) 4500.59 b) 4238.67
c) 4327.59 d) 4651.26

99) Rs. 2500 invested on 1st January 1985 had grown to be worth Rs 61,482 on 31st December
1999. The equivalent annual compound growth rate (to 2 decimal places) is
a) 23.80% b) 25.70%
c) 57.29% d) 63.95%

100) Find the compound amount and compound interest when Rs. 450,000 are invested for
3 years and 2 months at 6% compounded semi-annually?
a) Rs 552,250 and Rs 102,585 b) Rs 542,696 and Rs 92,696
c) Rs 500,000 and Rs 50,000 d) None of these

101) An investor places Rs 8,000 into an investment for ten years. The compound rate of
interest earned is 8% for first four years and 12% for last 6 years. At the end of the 10 years the
investment (to the nearest Rs) is worth.
a) Rs. 61,320 b) Rs. 21,483
c) Rs 21,517 d) 6,399

102) Walter invested Rs 5,000 in a bank deposit account which pays interest of 9% per
annum, added to the account at the end of each year. He made one withdrawal of Rs 1,500 at
the end of 3 years. What was the balance in the account at the end of 5 years, to the nearest Rs?
a) Rs 5,285 b) Rs 5,911
c) Rs 6,193 d) None of these
103) An equipment currently costs Rs 4,000, the rate of inflation for next three years is
expected to be 8% per annum, then 10% for the following 2 years. The price of the equipment
is expected to increase in line with inflation. The price, to the nearest Rs, after 5 years will be
a) Rs 5,760 b) Rs 5,800
c) Rs 6,097 d) Rs 6,155

112 | P a g e
104) A credit card company charges its customers compound interest @ rate of 2.25% per
month. The equivalent annual percentage rate, to 1 decimal place, is
a) 27% b) 27.7%
c) 30.6% d) 34.5%

105) A bank offers depositors a nominal interest rate of 10% per annum, with interest added
their accounts quarterly. The effective annual percentage rate, to 1 decimal place, is….
a) 8.2% b) 8.3%
c) 10.4% d) 11.10%

106) Raheel invests Rs 700 on 1 January each year, starting in 2,000. Compound interest of
10% is credited on 31 December each year. To the nearest Rs, the value of his investment on
31 December 2009 will be
a) Rs 10,456 b) 11,156
c) Rs 12,272 d) Rs 12,972

107) Sara took a loan of Rs. 1,200 with simple interest for as many years as the rate of
interest. If she paid Rs. 432 as interest at the end of the loan period, what was the rate of
interest?
a) 3.6 b) 6
c) 18 d) Cannot be determined

108) How much a father invests at the rate of 4.3% now so as to receive Rs. 50,000 at the
time of marriage of his newly born daughter who is expected to be solemnized after 20 years?
a) 19,000 b) 18,622
c) 15,000 d) 21,554

109) A non-interest bearing note of Rs. 3,000 is due in 5 years from now. If the note is
discounted now at 6% compounded semi-annually, what will be the compound discount?
a) 525.25 b) 646.46
c) 700.00 d) 767.72

110) The least number of complete years in which a sum of money put out at 20%
compound interest will be more than doubled is:
a) 3 b) 4
c) 5 d) 6

111) The indebtedness at any time in amortization is called


a) Principal Repayment b) Closing balance
c) Opening balance d) Outstanding balance

112) If the discount rate is 9% then find the present value of “X” which is paid in equal
annual installments for next five years?
a) 4.641X b) 0.3158X
c) 3.89X d) 0.2155X
113) Ali plans to invest Rs. 8000 every year for 3 years starting from today. Interest rate is
10% per annum compounded annually. After the end of year 3 he will receive.
114) Project A would provide annual inflows of Rs. 525,000 ,Rs. 853,000 and Rs. 2844,000
at the end of year1 to 4 respectively, whereas project B would yield annual inflows of Rs.
947,000, Rs.1155000 and Rs.2068000 from year 1 to 3 respectively. The discount rate at
which both projects would have same net present value is

113 | P a g e
115) A company intends to invest Rs. 4 million into a project which would yield 12,14 and
16 percent during three years respectively. The company would also recover the original
investment after 3 years. Company’s cost of capital is 10% . Npv of the project would be

116) A project costing Rs.2 million is expected to yield Rs. 300000 , Rs. 400000, Rs.
1900000 at the end of each of the next 3 years respectively. The nearest IRR approximation of
the project is

117) The rate of interest is 8% per annum compounded monthly, the value of perpetuity of
Rs 2500 per month would be

118) If the rate of interest is 10% per annum compounded monthly, the value of perpetuity
3000 per month would be achieved by investing

119) If the discount rate is 10%, the present value of Rs X received at the end of each year
for the next five years is equal is

120) Ashfaq is planning to invest in a scheme whereby he would be required to invest Rs.
130,000 annually(at the start of the year) for 5 years. If the interest rate is 13% compounded
annually, what amount would he receive at the end of the 5 th year?

121) Saeed invested Rs. 400,000 for ten years after which he received a lump sum amount
of Rs. 900,000, if he earned 8.50% interest compounded quarterly during last five years which
of the following rate compounded quarterly did he earn during first five years

122) Ali invested Rs. 500,000 for 5 years after which he received a lump sum amount of
Rs.762,150, if he earned 10% interest compounded annually during last 2 years, what rate of
interest compounded annually did he earn during the first three years?

123) Raza wants to save money over a period of ten years in order to meet the expenses to
be incurred on higher education of his son. He has recently invested a sum of Rs. 200,000 and
plans to further invest Rs. 20,000 at end of each quarter, which of the following amount will
be available to him at the end of 10 th year if he earns a profit of 6% per annum compounded
quarterly?

114 | P a g e
124) Ali borrowed Rs 700,000 from Sara for a period of 2 years and 3 months at r% simple
interest. She paid a total of 950,000 at the end of loan period. The value of r is

125) Haroon has borrowed a certain amount at an interest of 12% compounded semi-
annually. In how many years the amount owed would double?

126) Hashir has borrowed X amount at an interest of 13% compounded semi-annually if he


does not return the amount, in which of the following years the amount owed would be 2x?

127) Saad borrowed Rs 600, 000 from Fahad for a period of 3 years and 9 months at r%
simple interest . He paid Rs 400,000 in excess of the borrowed amount at the end of loan
period. The value of r is

128) Baber borrowed Rs. 900,000 from a bank at simple interest of 9.68% per annum.At the
end of the loan period he repaid a total of Rs 1510,000. Period of the loan was

129) Saleem borrowed Rs. 500,000 from a bank at simple interest of 2% per month for a
period of 3 years. The principal is payable in equal monthly instalments, along with interest.
Which of the following statements is correct?

115 | P a g e
CH # 5,6 ( Finance )
Solution 1:
Given
P = 200,000 , r = 13.5% , t = 5 , s = ?
using
S= p (1+rt)
S= 200,000 (1+0.135*5)
S = 3,35,000

Solution 2:

Given
R= 4000, r= 12%, t= 8, P=?
using
1−(1+𝑟)−𝑡
P= R [ ] (1+r)
𝑟
1−(1+0.12)−8 ]
P= 4000 [ (1+0.12)
0.12

S=P= 22255.026
Also using
𝑆
P=
(11+𝑟)𝑡

22255.026
P=
(1+0.12)2

P= 17,741,5706

Method: 2

1−(1+𝑟)−𝑡
P= R [ ] (1 + 𝑟)−1
𝑟

1−(1+0.12)−8
P= 400 [ ] (1 + 0.12)−1
0.12

P = 17,741,5706

Solution : 3

Difference = P [(1 + 𝑟)𝑡 -1] - P [(1 + 𝑟)𝑡 -1]

Difference = P [(1 + 𝑟1 )𝑡1 – (1+𝑟2 )𝑡 ]

= x [(1 + 0.1)8 - (1 + 0.09)8 ]

= x [2.1435 - 1.99256]

116 | P a g e
Difference = x (0.1509)

Increase = x (0.1509) * 100%


= 15.19 x%
=15.1 % increase

Solution : 4

Given
R=5000, r= 10%, t=10, S=?

(1+𝑟)𝑡 −1
S= R [ ]
𝑟

(1+0.1)10 −1
S= 5000 [ ]
0.10

(1.1)10 −1
S= 5000 [ ]
0.10

S= 79687.12

Solution : 5

Given
P=x, s=4x, r=10%, t=?

using

S= P(1 + 𝑟)𝑡

4x= x (1 + 0.10)𝑡

4= (1 + 0.10)𝑡

4= (1.1)𝑡

Log4= t log (1.1)

t= 14.545 = 14 year + 0.545*12 = 6.54 14 year and 6 months

Solution : 6

Given
P= 500,000 , s= 896793 , r= 10%

Using

S= P (1 + 𝑟)3 (1 + 𝑟)4

896793 = 500 000 (1 + 𝑟)3 (1 − 10)4

117 | P a g e
896793 = 732 050 (1 + 𝑟)3

1.22 = (1 + 𝑟)3

Log (1.22) = 3 log (1+r)

0.08635 = 3 log (1+r)

0.0287 = log (1+r)

1.0683 = 1+r

r = 0.0683

r = 7%

Solution: 7

P= 200,000, t= 5, S= 300,000 , r= 8%

Using
S = P(1 + 𝑟)𝑡
300 000 = P (1 + 0.08)5
P = 204 174.95

No, because P.V is greater than 200,000

Solution : 8

At start point of 300,000


𝑟
(1+ )4𝑡 −1 𝑟
4
S=R[ 𝑟⁄ ] (1+ )
4 4

1+0.03)12 −1
300,000 = R [ ] (1+0.03)
0.03

R = 20522.93

At start point of 500,000

(1+𝑟⁄4)4𝑡 −1 𝑟
S=R[ 𝑟⁄ ] (1 + 4 )
4

(1.03)20 −1
500,000 = R [ ] (1.03)
0.03

R = 180 65.87

So first and 8th installment will be


= 20522 + 180 65.88
= 385 88.8

118 | P a g e
But 13th installment will be only related to annuity of 500,000 i.e = 180 65.8

Solution: 9

Given
P = 650,000 , S = 10,60,000 , r = 10%
Using
S = P (1 + 𝑟)𝑡 (1 + 𝑟)𝑡

1060,000 = 650,000 (1 + 𝑟)2 (1 + 0.10)4

1060,000 = 951,665 (1+𝑟)2

101146 = (1 + 𝑟)2

r = 5.5 %

Solution : 10

Solution : 11

Given
R = 8000 , t = 3 , r = 10% , S = ?
Using
(1+𝑟)𝑡 −1
S=R[ ]
𝑟
(1+0.1)3 −1
S = 8000 [ ]
0.1
S = 29,128

Solution : 12

Given
P = 200,000 , S = 649,464 , t = 10
Using
𝑟
S = P (1 + 2)2𝑡
𝑟
649, 464 = 200,000 (1 + 2)20

r = 12.3 % (Nominal Rate)


Now effective rate is
𝑟
e = (1 + 𝑚)𝑚 −1
0.1213 2
e = (1 + ) −1
2
e = 12.5 %

119 | P a g e
Solution : 13

NPV at 11 % is
17000 25000 16000 4000 45000
NPV of 11 % = + + + -
(1+0.11)1 (1+0.11)2 (1+0.11)3 (1+0.11)4 (1+0.11)0

= 49939.86 – 45000

= 4939.86

NPV at 15 % is
17000 25000 16000 4000 45000
NPV of 15 % = + + + -
(1+0.15)1 (1+0.15)2 (1+0.15)3 (1+0.15)4 (1+0.15)0

Now ( IRR ) / interpolation is


𝑁𝑃𝑉 (𝐻𝑖𝑔ℎ)
IRR = lower rate + (High rate – low rate)
𝐻𝑖𝑔ℎ 𝑁𝑃𝑉−𝐿𝑜𝑤 𝑁𝑃𝑉

4939.86
IRR = 0.11 + 4939.86−1493.47 (0.15-0.11)

IRR = 16.73 %

Note :
Working Capital outflows is not related to investment in machine. It is indirect expense
of Company like electricity expense or salaries expense.

Solution : 14

Given
P = 2500 , r = 8%
Using
𝑅
P=𝑟

2500
P = 0.08/12

P = 375,000

Solution : 16

Given

P = ? , S = 500,000 , t = 8 , r = 9%
Using
S = P (1 + 𝑟)𝑡
500 000 = P (1 + 0.08)8
P = 250 933.14

120 | P a g e
Solution : 17

P = X , r = 10% , t = ? , S = 3x

Using

3x = x (1 + 𝑟)𝑡

3x = x (1 + 0.1)𝑡

3 = (1 + 0.1)𝑡

3 = (1.1)𝑡

Log 3 = t log (1.1)

t = 11.52

Solution : 18

Present value of both investments


98000 95000 150 000 85000 90000 75000 95000
+ + = + + +
(1+𝑟)1 (1+𝑟)2 (1+𝑟)3 (1+𝑟)1 (1+𝑟)2 (1+𝑟)3 (1+𝑟)4

Putting value of r in both side and equate the answers

Solution : 19
Given
P = x , r = 0.7% , t = 10
S = 10,000,000

Using
𝑟
S = P (1+12)12𝑡

10,000,000 = x (1 + 0.00058)120

P = x = 9324

Solution : 20

Given
P = x , r = 1.8% per quarter (Already divided by 4)
t = 10 , S = 10,000,000
Using
𝑟
S = P (1+4)4𝑡

10,000,000 = x (1 + 0.018)40
x = P = 4898,790

121 | P a g e
Solution : 21

Given
P = 500,000 , r = 1.5% , t = 5
Using

S = P (1 + 𝑟 𝑡 )

S = 500,000 (1+1.5% * 5 * 12)

S = 950,000

Solution : 22

Using
I = prt

I = 500,000 * 0.15 * 5 * 12

I = 450,000

Note: As rate is monthly, so time will also be in months (5 years = 60 months)

Solution : 23
3
P = 600,000, r = 9% , t = 3.3 = 3 + 12 = 3.25
Using

S = P (1+rt)

S = 600,000 (1+0.09*3.25)

S = 775 500

Solution : 24

Given
S = 300,000 , r = 8.5% , t = 5
Using
S = P (1 + 𝑟)𝑡
300,000 = P (1 + 0.085)5

P = 199513.62
Less present value , the company is not in profit

Solution : 25

S = P (1 + 𝑟)𝑡 (1 + 𝑟)𝑡

860,000 = 500,000 (1 + 0.1)4 (1 + 𝑟)3

122 | P a g e
1.174 = (1 + 𝑟)3
1.055 = 1+r
0.055 = r
5.52 % = r

Solution: 26
𝑟
C.I = P [ (1 + 4)4𝑡 -1 ]

0.08 4∗9.5
C.I = 90,000 [ (1 + ) -1 ]
4

C.I = 101 007

Solution : 27

𝑁𝑃𝑉𝐴 = 𝑁𝑃𝑉𝐵
80000 90000 100000 85000 50000 75000 95000
+ + = + + +
(1+𝑟)1 (1+𝑟)2 (1+𝑟)3 (1+𝑟)1 (1+𝑟)2 (1+𝑟)3 (1+𝑟)4

r = 17.88%

Solution : 29

Given
P = 100,000 , R = 1000 , r = 1.8% , t = ?

Using
𝑟
1−(1+ )−12𝑡
12
P = R [ 1- 𝑟⁄ ]
12
0.018
(1+ )−12𝑡
12
100,000 = 1000 [ 1- 0.018⁄ ] = 9 years
12

Solution : 30

𝑆𝑑 = 200,000 , 𝑆𝑜 = ? , r = 12%

Using
𝑟
𝑆𝑑 = 𝑆𝑜 (1 + 4)4

0.12
200,000 = 𝑆𝑜 (1+ )
4

𝑆𝑜 = 194 174.76

123 | P a g e
Solution: 31

Given
R = x , r = 10% , t = 5 , P = ?

Using
1−(1+𝑟)−𝑡
P=P[ ] (1+r)
𝑟

1−(1+0.1)−5
P=x[ ] (1+0.1)
0.1

P = 4.169865x

Solution : 32

Given
P = 1.5 , t = 4 , I = 0.5 , r = ?
Using
I = P [ (1 + 𝑟)𝑡 − 1 ]

0.5 = 1.5 [ (1 + 𝑟)4 − 1 ]

r = 7.456%

Solution : 33

Given
P = 400,000 , S = 545,881 , r = ? , t = 3

Using
𝑟
S = P(1 + 12)12𝑡

𝑟
545 881 = 400,000 (1 + 12)12(3)

𝑟
1.3647 = (1 + 12)36

r = 0.1040
r = 10.40% ( Nominal Interest Rate)
Effective Rate
𝑟
e = (1 + 𝑚)𝑚 − 1

0.1040 12
e = (1 + ) −1
12

e = 10.92%

124 | P a g e
Solution : 34

Using
𝑎 (𝑟 𝑛 −1)
S=
𝑟−1

(1.20)𝑛 −1
16,60,000 = 80,000 ( )
1.20−1

5.15 = (1.20)𝑛

n = 9 years

Solution : 35

P = 800,000 , t = 4 , I = 650,000 , r = ?
Using

I = prt
650,000 = 800,000*r*4
r = 0.20.3
r = 20.3%

Solution : 36

Given
S =1500,000 , a = 80,000 , r = 1.5 , n = ?
Using
𝑎(𝑟 𝑛 −1)
S=
𝑟−1

800000 (1.5𝑛 −1)


1500 000 =
1.5−1

r = 5.77

r = 6years

Solution : 37

Given

a = x , d = 100 , n = 12 , S = 16 200

Using
𝑛
Sn = 2 [ 2a + (n-1)d ]

12
16 200 = [ 2x + (1200-1) 100]
2

2 700 = 2x + 1100

125 | P a g e
1 600 = 2x

x = 800

Solution : 38

Given
r = 10% , e = ?
Using
𝑟
e = (1+𝑚)𝑚 -1

0.10
= (1+ 12 )12 − 1

= 10.47%

Solution : 39

Using
(1+𝑟)𝑡 −1
S=R[ ]
𝑟

(1+0.10)𝑛 −1
1500,000 = 80,000 [ ]
0.10

18.75 = (1.1)𝑛−1

.875 = (1.1)𝑛

n = 11

Solution : 40

IRR
P.V of inflows = P.V of outflows
0.2 0.3 2.9 2.5
+ + =
1+𝑟 (1+𝑟)2 (1+𝑟)3 (1+𝑟)0

Solution : 41

IRR
P.v pf cash inflows = P.V of cash outflows
200,000 290,000 2900,000
+ + = 2500,000
(1+𝑟)3 (1+𝑟)6 (1+𝑟)9

IRR = r = 3.71857

126 | P a g e
Solution : 43

Future value of investment

S = P (1 + 𝑟1 ) (1 + 𝑟2 ) (1 + 𝑟3 )

S = 3 (1+0.10) (1+0.12) (1+0.14)

S = 4.21344

Also
𝑆 4.21344
P= = = 3.1656
(1+𝑟)𝑡 (1+0.10)3

Now
NPV
= P.V of benefits – P.V of cast
= 3.1656 - 3 = 0.1656

Solution: 44

Equivalent rate of interest

Azam=Moazzam +20% of Moazzam

Azam =1.2Moazzam
0.10 4∗5 𝑟
10 (1+ ) = 1.2 [ 10 (1+ 4)4∗5 ]
4
𝑟
16.3861644 = 12 (1+4)20

𝑟
1.3655137 = (1+ 4)20

𝑟
1.015698481 = 1+ 4

𝑟
0.015698481 = 4

r = 6.28%

Solution : 5

Amount of Awais investment = Amount of Salman investment


0.12 4𝑡
1.2 (1+ ) = 1.8 (1 + 0.10)4𝑡
4

n = 17.68 (Using reverse method)

So in 17.68 years both investments will be same and anytime above 17.68 year. Investment of
Awais will exceeds that of Salman

127 | P a g e
Solution : 46

0.12 4∗10
(1+ ) −1 0.12 40
4
Total amount = 25000 [ 0.12⁄ ] + 500,000 (1 + )
4
4

Total amount = 1885031.493 + 1631018.896

S = 3516050.389

Solution : 47

(i) S = P (1+rt) = 100 (1+0.097*4) = 138.8


𝑟 4𝑡 0.0963 16
(ii) S = P (1 + 4) = 100 (1 + ) = 144.446
4
𝑟 0.094 8
(iii) S = P (1 + 2) 2𝑡 = 100 (1 + ) = 144.40
2

Option (ii) is better as it gives highest result.

Solution : 48

S = P (1 + 𝑟)𝑡

S = 170,000 (1 + 0.08)5

S = 249,785.77

Solution : 49

S = P (1 + 𝑟)𝑡

S = x (1 + 0.1)4

S = 1.4x

Solution : 50

0.12 120
(1+ ) −1 0.12 120
12
Total amount = 25000 [ 0.12 ] + 500,000 (1 + 12
)
12

Total amount = 7401160.68

Solution : 51
𝑅 3000
P=𝑟 = 0.09 = 400,000
⁄12
12

Solution : 52

𝑟 𝑚𝑡 𝑟 𝑚𝑡
S = P (1 + 𝑚) (1 + )
𝑚

128 | P a g e
0.08 18 𝑟 24
300,000 = 200,000 (1 + ) (1 + 6 )
6

𝑟 24
1.5 = (1.2692) (1 + )
6

𝑟 24
1.1818 = (1 + 6 )

𝑟
1.0069 = 1 + 6

r = 4.2%

Solution : 53

S = P (1+rt)

S = x (1+0.07*2)

S = 1.14x

Solution : 54

Using

1−(1+𝑟)−𝑡 1−(1+0.1)−4
P=R[ ]=x[ ] = 3.16x
𝑟 0.10

Solution :

Using

1− (1+𝑟)−𝑡
P=R[ ] (1+r)
𝑟

1− (1+0.1)−5
=x[ ] (1+0.10)
0.10

P = 4.17x

Solution : 58

P = 1600 , S = 1560 , r = ? , t = 1

Using

P = S (1+rt)

1600 = 1560 (1+r)

1.0256 = 1+r

r = 0.02564

129 | P a g e
r = 2.5%

Solution : 59

𝑟 −12𝑡
1−(1+ )
12
NPV = (243 000) + R [ 𝑟⁄ ]
12

0.12 −12
1−(1+ )
12
= (243 000) + 50,000 [ 0.12 ]
12

= (243 000) + 562753.87

NPV = 319 754

Solution : 60

P.V of cash inflows = P.V of cash outflows


65200 96000 73100 55400
+ + + = 213 000
(1+𝑟)1 (1+𝑟)2 (1+𝑟)3 (1+𝑟)4

r = 14.67%

Solution : 61

Let cot of equipment = 100

Percentage of depreciation

15 , 13.5 , 12 , _ _ _ _ which are in A.P

a = 15 , d = -1.5

Percentage of depreciation in both year is

= a + (10-1) d

= 15+9 (-1.5) = 1.5

Also total value depreciate in 10 years

15+13.5+12p_ _ _ _ 1.5 = 82.5

Value of equipment at the end of ten years

= 100-82.5 = 17.5
600,000
Total cost is * 17.5 = Rs 105,000
100

130 | P a g e
Solution : 63

D.P = 50,000

𝑟 −4𝑡
1− (1+ )
4
P=R[ 𝑟⁄ ]
4

0.12 −4∗5
1− (1+ )
4
P = 30,000 [ 0.12 ]
4

P = 446324.24

Cash price = D.P + P

= 446324.24 + 50,000

= 496324.25

Solution : 64
𝑅 5.60
P = 𝑟 = 0.04 = 140

Solution : 65

1− (1+𝑟)−𝑡
P=R[ ]
𝑟

1− (1+0.1)−7
P = 168 [ ]
0.1

P = 817.9

Solution : 66
𝑆 3000
P= 𝑟 2𝑡
= 0.06 2∗5
= 2232.28
(1+ ) (1+ )
2 2

Solution : 67

(1+𝑟)𝑡 −1
S=[ ]
𝑟

(1+.15)5 −1
30,000 = R [ ]
0.15

R = 4449

131 | P a g e
Solution : 68
𝑅
P=𝑟
4000
P = 0.075 = 53333

Solution : 69

S = P (1+rt) S = P (1+rt)

300 = P (1+r*3) 400 = P (1+r*5)

300 = P (1 + 3r) -(i) 400 = P (1 +5r) -(ii)

Dividing (i) and (ii)


400 𝑃(1+8𝑟)
= 𝑃(1+3𝑟)
300

400 1+8𝑟
=
300 1+3𝑟

400+12000 = 300+2400r

100 = 1200r

r = 0.0833

r = 8.33%

Solution : 70

P = 12500 , R = 2000

(1+𝑟)𝑡 −1
S = P (1 + 𝑟)𝑡 , S=[ ]
𝑟

(1+0.2)3 −1
S = 21600 , S = 2000 [ ] = 7280
0.2

Remaining amount = 14320

Solution : 71
𝑟 −10
1− (1+ ) 𝑟
12
P=R[ 𝑟⁄ ] (1+12)
12

1− (1+0.01)−10
P = 50 [ ]
0.01

P = 478.30

132 | P a g e
Solution : 72
𝑅
P=𝑟

20,000
500,000 = 𝑟

r = 0.04

r = 4%

Solution : 74
𝑟
(1+ ) 2𝑡 −1
2
S=R[ 𝑟⁄ ]
2
(1+𝑟⁄2)10 −1
6000 = 500 [ 𝑟⁄ ]
2
𝑟 10
(1+ ) −1
2
12 = 𝑟⁄ using table
2

Solution : 77

(1+𝑟)𝑡
S=R[ ] (1+r)
𝑟

(1+0.1)5 −1
S = 8000 [ ] (1+0.1)
0.1

S = 53724.88

Solution : 79

R = 500 , S = 24983.75 , t = 7 , r = ?

Using
𝑟
(1+ )4𝑡 −1
4
S=R[ 𝑟⁄ ]
4

𝑟 28
(1+ ) −1
4
2483.75 = 500 [ 𝑟⁄ ]
4

𝑟 28
(1+ ) −1
4
49.96 = 𝑟⁄ (use of table)
4

Solution : 80

NPV = Cash in flow – Cash out flow

133 | P a g e
−12𝑡
1− (1+𝑟⁄12)
NPV = R [ 𝑟⁄ ] – 243, 000
12

1− (1+0.01)−12
NPV = 50,000 [ ] – 243, 000
0.01

NPV = 562753.87 – 243 000

NPV = 319754

Solution : 82

S = 40,000 (1 + 0.11)5

S = 67402

Also

(1+𝑟)5 −1
S=R[ ]
𝑟

(1+0.11)5 −1
S = 5000 [ ]
0.11

Also

P = 67402 – 31139

P = 36263

Using

1− (1+0.11)−5
36263 = R [ ]
0.11

R = 9811

Solution : 83
𝑁𝑃𝑉𝐴
IRR = A% + 𝑁𝑃𝑉 (B-A) %
𝐴 − 𝑁𝑃𝑉𝐵

60
IRR = 20% 60+120 (30-20) %

1
IRR = 20% + 3 (10)%

IRR = 20% + 3.3%

IRR = 23.3%

134 | P a g e
Solution : 84
𝑅
P=𝑟

5000
P = 12.5%

P = 40, 000

Solution : 85

𝐼1 + 𝐼2 = 320

prt + prt = 320

-P*0.08*12 + P*0.12*12 = 320

- 0.96P + 1.44P = 320

0.48P = 320

P = 666

Solution : 86

𝐼1 - 𝐼2 = 2.50

𝑃1 𝑟1 𝑡1 -𝑃2 𝑟2 𝑡2 = 2.50

500*𝑟1*2 – 500*𝑟2 *2 = 2.50

1000 (𝑟1 − 𝑟2 ) = 2.50

𝑟1 − 𝑟2 = 0.0025

𝑟1 − 𝑟2 = 0.25%

Solution : 87

P = 1573 P = 1444.50

t = 18⁄12 t = 6⁄12

r = 4% r = 14%

Using

S = P (1+rt) S = P (1+rt)
1573 1444.50
18 =P 6 =P
1+0.14∗ 1+0.14∗
12 12

135 | P a g e
P = 1300 P = 1350

Difference = 1350 – 1300

Difference = 50

Solution : 88

C.I-S.I = P [ (1 + 𝑟)𝑡 – 1 ] –P(1+rt)

Solution : 89

P = 12000 2 = (1 + 𝑟)5

S = 2400 1.14 = 1 + r

r=? r = 14.86%

t=5 Also, P = 24000

r = 0.1486

t = 15

Using Using

S = P (1 + 𝑟)𝑡 S = P (1 + 𝑟)𝑡

24 000 = 1200 (1 + 𝑟)5 S = 192 000

Solution : 90

Given

P = ? , r = 10% , S = 3500 , t = 4

Using

S = P (1 + 𝑟)𝑡

3500 = P (1 + 0.14)

P = 2500

Solution : 92

P=x

S = x + 60%x = 1.6x

r=?

t=6

136 | P a g e
Using

S = P (1+rt)

1.6x = x (1+6r)

1.6 = 1 + 6r

0.6 = 6r

r = 0.1 = 10%

Also

C.I = P [(1 + 𝑟)𝑡 -1]

C.I = 12000 [(1 + 0.1)3 -1]

C.I = 3972

Solution : 93

P = 100,000 , r = 12% , R=?, t=1

Using

𝑟 −12𝑡
1− (1+ )
12
P=R[ 𝑟⁄ ]
12

0.12 −12
1− (1+ )
12
100,000 = R [ 0.12 ]
12

1− (1+0.11)−12
100,000 = R [ ]
0.11

100,000 = R (11.2551)

R = 8884.86 = installment

Also

Interest paid = 100,000 * 0.01 = 1000

Repaid amount = 8884.86 – 1000 = 7884.86

Also

Interest = nR – P = 12 (8884.86) – 100,000

Interest = 661832

137 | P a g e
Solution : 94

Deffered Annuity :

R = 5000 , t = 25 , r = 6%
Using
1− (1+𝑟)−𝑡
P=R[ ]
𝑟
1− (1+0.06)−25
P = 5000 [ ]
0.06
P = 63916.78

Also
𝑆
P = (1+𝑟)𝑡

63916.78
P=
(1+0.06)4

P = 50,528.08

No, you will not lend.

Solution :95

Given

P = 10,000 , R = 2000 , r=?, n=6

Using

𝑟 −2𝑡
1−(1+ ) 𝑟
2
P=R[ 𝑟⁄ ] ( 1+ 2 )
2

𝑟 −2∗6
1− (1+ ) 𝑟
2
60,000 = 2000 [ 𝑟⁄ ] (1+2 )
2

𝑟 −12
1− (1+ ) 𝑟
2
30 = [ 𝑟⁄ ] (1+2 )
2

Use of table

Solution : 96

R = 500 , S = 6000 , t = 5 , r=?

Using

138 | P a g e
𝑟 2𝑡
(1+ ) −1
2
S=R[ 𝑟⁄ ]
2

𝑟 2∗5
(1+ ) −1
2
6000 = 500 [ 𝑟⁄ ]
2

𝑟 10
(1+ ) −1
2
12 = 𝑟⁄
2

Use of table

Solution : 98

Given

P = 12000 , r = 0.06 , t = 18⁄12

𝑟 −2𝑡
1− (1+ )
2
P=R[ 𝑟⁄ ]
2
18
0.06 −2∗ 12
1− (1+ )
2
12 000 = R [ 0.06⁄ ]
2

12 000 = R [ 2.8286 ]

R = 4242.38

Solution : 99

P = 2500 , S = 61482 , t = 15 , r = ?

Using

S = P (1 + 𝑟)𝑡

61482 = 2500(1 + 𝑟)15

24.59 = (1 + 𝑟)15

1.2380 = 1 + r

0.2380 = r

r = 23.80%

139 | P a g e
Solution : 100

P = 450, 000

S=?

r = 6%
2
t = 3 + 12 = 3.17

Using

𝑟 2𝑡
Compound Amount = S = P (1 + 2)

0.06 2∗3.17
S = 450, 000 (1 + )
2

S = 542750

Also

Compound Interest = S – P

I = 542750 – 450 , 000

I =92750

Solution : 101

Given

P = 8000 , r = 8% , t = 4 , r = 12 % , t=6

Using

S = P (1 + 𝑟)𝑡

S = 8000 (1 + 0.08)4

S = 10883.91

Also

S = P (1 + 𝑟)𝑡

S = 10883.91 (1 + 0.12)6

S = 21482.90

140 | P a g e
Solution :102

Given

P = 5000 , r = 9% , t = 3

Using

S = P (1 + 𝑟)𝑡

S = 5000 (1 + 0.09)3

S = 6475.14

Remaining = 6475.14 – 1500

S = 4975.145

Also

S = P (1 + 𝑟 )𝑡

S = 4975.145 (1 + 0.09)2

S = 5911

Solution : 106

P = 700 , r = 10% , t = ?

Using

(1+𝑟)𝑡 −1
S=R[ ] (1 +r)
𝑟

(1+0.1)10 −1
S = 700 [ ] (1 + 0.1)
0.1

S = 12272

Solution : 107

I = prt

432 = 1200 * r% * r

0.36 = 𝑟 2 %

36 = 𝑟 2

r=6

141 | P a g e
Solution : 108

S = 50,000 , r = 4.3% , P = ? , t = 20

Using

S = P (1 + 𝑟)𝑡

50,000 = P (1 + 0.043)20

50,000 = P (2.3210)

P = 21542

Solution : 109

S = 3000, t = 5 , r = 6% , P=?

Using

𝑟 2𝑡
S = P [(1 + 2)

0.06 5∗2
3000 = P (1 + )
2

P = 2232.30

Compoun Discount

=S–P

= 3000- 2232.30

= 767.69

Solution : 110

Given

P = x , r = 20% , S = 2x

S = P (1 + 𝑟)𝑡

2x = x (1 + 0.2)𝑡

2 = (1 − 2)𝑡

𝑙𝑜𝑔2
t= ⁄𝑙𝑜𝑔1.2

t = 4 (approx.)

142 | P a g e
Solution : 112

r = 9% , t = 5 , R=x, P=?

Using

1− (1+𝑟)−𝑡
P=R[ ]
𝑟

1− (1+0.09)−5
P=x[ ]
0.09

P = x (3.89)

P = 3.89x

Solution : 113

Given

R = 8000 , t = 3 , r = 10% , S=?

Using

(1+𝑟)𝑡 −1
S=R[ ] (1 + r)
𝑟

(1+0.1)3 −1
S = 8000 [ ]
0.1

S = 29,128

Solution : 114

𝑁𝑃𝑉𝐴 = 𝑁𝑃𝑉𝐵
525 000 648 000 853 000 28 44 000 947 000 11 55 000 20 68 000
+ + + = + +
(1+𝑟)1 (1+𝑟)2 (1+𝑟)3 (1+𝑟)4 (1+𝑟)1 (1+𝑟)2 (1+𝑟)3

Using Calculator

r = 18.27 Or putting value of x on both sides

Solution :115

Year 1 Year 2 Year 3

Cash flows Percent 480,000 560, 000 640,000

40, 000 00 = 46,40,000

P.V 436363.63 462809.9 3486100.67

Total 4385247.20

143 | P a g e
Remaining = 4385274.20 – 40,00,000

= 385274.20

Solution : 116
(20,00,000) 300 000 400,000 19 00 000
NPV = + + +
(1+𝑟)0 (1+𝑟)1 (1+𝑟)2 (1+𝑟)3

IRR = 10.66% = r

IRR is the point where NPV becomes zero.

Solution : 117

Given

r = 8%, R = 2500
𝑅 2500
P = 𝑟 = 0.08⁄ = 375,000
⁄2 2

Solution : 118

Given

r = 10% , R = 3000

Using
𝑅 3000
P=𝑟 = 0.1⁄ = 360, 000
⁄12 12

Solution : 119
1− (1+𝑟)−𝑡
P=R[ ]
𝑟

1− (1+0.1)−5
P=x[ ]
0.1

P = 3.79x

Solution : 120

(1+𝑟)𝑡 −1
S=R[ ] (1+r)
𝑟

(1+0.13)5 −1
S = 130,000 [ ]
0.13

S = 951951.75

144 | P a g e
Solution : 121

Using

𝑟 4𝑡 𝑟 4𝑡
S = P (1 + 4) (1 + 4)

𝑟 20 0.085 20
900,000 = 400,000 (1 + 4) (1 + )
4

r = 0.078843

r = 7.89%

Solution : 122

S = P (1 + 𝑟)3 (1 + 𝑟)2

762 150 = 500 000 (1 + 𝑟)3 (1 + 0.1)2

r = 0.080014

r = 8%

Solution : 123

Using
𝑟 4𝑡
(1+ ) −1
𝑡
S = P (1 + 𝑟) + R [ 𝑟4 ]
⁄4

0.06 40
(1+ ) −1
10 4
S = 200,000 [1 + 0.06] + 20,000 [ 0.06 ⁄4
]

0.814018
S = 358169.53 + 20,000 ( )
0.015

S = 358169.53 + 1085357.33

S = 1443527.408

Solution : 124

Given
3
P = 700,000 , S = 950,000 , t = 2 + 12 = 2.25 , r = ?

Using

S = P(1 + 𝑟)𝑡

950,000 = 700,000 (1 + 𝑟)𝑡

145 | P a g e
1.357 = (1 + 𝑟)2.25

1.1453 = 1 + r

0.1453 = r

r% = 14.53%

Solution : 125

P = x , S = 2x , r = 12%

S = P(1 + 𝑟)𝑡

0.12 2𝑡
2x = x (1 + )
2

2 = (1.06)2𝑡

Log 2 = 2t log (1.06)

t = 6 years

Solution : 126

P = x , S = 2x , r = 13% , t = ?

Using

𝑟 2𝑡
S = P (1 + 2)

0.13 2𝑡
2x = x (1 + )
2

2 = (1.065)2𝑡

Log2 = 2t log (1.065)

t = 6 years

Solution : 127
9
P = 600,000 , I = 400,000 , t = 3 + 12 = 3.75

Using

I = prt

400,000 = 600,000 (r) (3.75)

r = 17.78%

146 | P a g e
Solution : 128

P = 900,000 , r = 9.68% , S = 15,10,000

Using

S = P(1 + 𝑟)𝑡

15,10,000 = 900,000 (1 + 0.0968)𝑡

1.68 = (1.0968)𝑡

log (1.68) = t log (1.0968)

t = 6 years

Solution : 129

Given

P = 500,000 , r = 2% , t = 3

Using

𝑟 −4𝑡
1−(1+ )
P=R [ 𝑟4 ]
⁄4

0.02 −4∗3
1−(1+ )
4
500,000 = R [ 0.02⁄ ]
4

1−(1+0.005)−12
500,000 = R [ ]
0.05

500,000 = R (11.6189)

R = 43033

Solution: 130

Given r = 4.3%, S = 50, 000 , t = 20

Using

S = P (1 + 𝑟)𝑡

50, 000 = P (1 + 0.043)20

P = 21541.89

147 | P a g e
Solution: 131

A company intends to invest Rs 3 million into a project which would yield 10,12, and 14%
during the first three years respectively. The company would also recover the original
investment after three years. If the company’s cost of capital is 8%. The NPV of this project
would be

Cash Flow Year 1 Year 2 Year 3

Amount yield of Project 300,000 360,000 420,000

30,00,000

P.V at 8% 277777 308641 2714,906

Total = 3301324.26

Remaining = 3301324.26 – 30,00,000

= 301324.26

Solution : 132

Given

R = 50, 000 , n = 20 , r = 10%

Using

1− (1+𝑟)−𝑡
P=R[ ]
𝑟

1− (1+0.1)−20
P = 50,000 [ ]
0.1

S = P = 425678.19

Also
𝑆
P = (1+𝑟 )𝑡

425678.19
P = (1+0.10)20

P = 63274.35

Again

(1+𝑟)𝑡 −1
63274 = R [ ]
𝑟

(1+0.1)20 −1
63274 = R [ ]
0.1

148 | P a g e
63274 = R (57.27)

R = 1104.83

Solution:133
𝑅
P=𝑟

2398
P= = 19983.33
0.12

149 | P a g e
1/1/2022 CHAPTER 8
STATISTICAL MEASURES OF DATA

✓ INTRODUCTION:

From descriptive to predictive analysis, statistical measure helps in better


understanding of available data. For Example, in summarizing a dataset, measure of
central tendency (mean, median and mode) and measure of dispersion (range,
standard deviation and variance) are used for primary statistical analysis

COMPILED BY
ABDUL AHAD BUTT
MEASURES OF CENTRAL TENDENCY

1|Page
Measures of central tendency, or more simply measures of center, indicate where the center or most typical
value of a data set lies.

• A data set can be summarized in a single value


• Such value, usually somewhere in the center and representing the entire data set, is a value at which the
data have a tendency to concentrate
• The tendency of the observations to cluster in the central part of the data set is called central tendency
and the summary value as measure of central tendency
• Since a measure of central tendency indicates the location or general position of the distribution or the
data set in the range of observations, it is also known as a measure of location or position
• The measures of central tendency or location are generally known as averages.

Three common measures of central tendency are the arithmetic mean (mean), median, and mode.

Mean:

The mean is the sum of the observations divided by the number of observations.

We use x to denote sample mean and to denote population mean.

Example The following are salaries for four randomly selected employees at Microsoft.

$30,000 $35,000 $32,500 $36,000

30,000 + 35,000 + 32,500 + 36,000


x= = $33,375
4
Formula: Types of Averages

1. Arithmetic mean
i) Direct
∑X
i) ̅=
X ( ungroup date)
n

∑fX
ii) ̅
X= ( group date)
∑f

ii) Deviation/short cut method


∑D
i) ̅
X =A+ ( ungroup date)
n

∑fD
ii) ̅ = A+
X ( group date)
∑f

iii) Step deviation/coding method


∑U
i) ̅
X =A+ x h ( ungroup date)
n

2|Page
∑fU
ii) ̅ = A+
X xh ( group date)
∑f

2. Geometric Mean
i) G.M = 𝑛√x1 x x3 x … … . . xn
OR
∑ log x
= Antilog ( )
n
∑f log x
ii) G.M = Antilog ( )
∑f

Properties of G.M
• The G.M must be used in finding average period-to-period percent change
• For this you have to compute the G.M of the growth factors
• A growth factor is the ratio of a number, say, price or sales, for one period to the corresponding number
for a previous period
• Always use G.M for % increase or decrease
• Multiplicative in nature

Merits of G.M
• Rigorously defined by a mathematical formula
• Based on all values (Gives equal weightage to all observations)

Demerits of G.M
• Not easy to calculate
• Not easy to understand
• Vanishes if any observation is zero
• In case of negative values, it cannot be computed at all

3. Harmonic Mean:
n
i) H.M = 1 ( ungroup date)
∑( )
x

∑f
ii) H.M = f ( group date)
∑( )
x
Merits of HM

• rigorously defined by a mathematical formula


• based on all observations in the data

Demerits of H.M

• cannot be calculated if any one of the observations is zero

4-Weighted Mean:

3|Page
A weighted mean is the mean of a data set whose entries have varying weights. A weighted mean is given by,
where w is the weight of each entry x.
∑WX
̅
Xw = ∑W
5. Combined mean
̅ 2 +n3X
n1 X + n2 X ̅3
̅
XC = n1 + n2 + n3

6- Median:
• Median is a value that divides the data into two equal parts.
• Central value of data
• Median is the value that has 50% of the values below it and 50% of the values above it
• If the number of observations is odd, then the median is the observation exactly in the middle of the data
set.
• If the number of observations is even, then the median is the mean of the two middle observations in the
ordered list.
𝒏+𝟏
Position of median = 𝟐 (ungroup date)

Example Find the median for the sample of four salaries from Microsoft.

First put the salaries in order:

$30,000, $32,500, $35,000, $36,000

Median = 32,500 + 35,000 $33,750


=
2
Formula: Procedure ungroup data
• Arrange the data in ascending order
• Count the values
• If the total number of values is ODD the middle value is Median
• If the total number of values is even then the midpoint of middle two values will be Median

Merits of Median
• Easily calculated & understood
• Not affected by extreme values
• In a highly skewed distribution, median is an appropriate average to use

Demerits of median
• Not rigorously defined
• Not capable of further statistical treatment
• Arrangement of data into array can be tedious and time consuming

7. Median
𝒏
See in C.F
𝟐

h n
ii) ̃
X = l+ + (2 − c) G r o u p / F r e q u e n c y d i s t r i b u t i o n d a t a
f
4|Page
8. Quartiles: are the quantities that divide the data into four equal parts and every part contains ( ¼)
value(25% of data). They are three in numbers
n+1
i) Qk = k ( ) t h value where k =1, 2,3
4
h kn
ii) Qk = l + ( 4 − c)
f

9. Decile: are the quantities that divide the data into ten equal parts
n+1
i) Qk = k ( 10 ) t h value
h kn
ii) Qk = l + (10 − c) where k =1, 2,3…..9
f

10. Percentiles: are the quantities that divide the data into 100 equal parts
n+1
i) pk = k ( 100 ) t h value
h kn
ii) pk = l + (100 − c) where k =1, 2,3…….99
f

Note: Q2= D5 =P50

11.Mode:

The mode is the value that occurs most frequently in a data set.

• There can be more than one mode.


• When no value is repeated, we say there is no mode.
• French word: meaning fashion: “most frequent”
• One mode=unimodal data
• Two mode = bimodal data
• Mode =3median -2mean

Merits of mode:
• Is an actual value
• Simply defined
• Easily calculated
• Not affected by extreme value
• Can be determined for both quantitative and qualitative data

Demerits of mode
• Not rigorously defined
• Often indeterminate & indefinite
• Not based on all the observations made
• Not capable for further statistical treatment
• Mode may not exist

5|Page
Example: Find the mode for the following test scores

84 89 82 91 86 84

The mode is 84.

Mode
fm −fi
̂
X = l+ x h
(fm −fi )+( fm −f2)

12. Symmetric distribution


Mean = Median = Mode

13. Moderately skewed distribution/Empirical relationship


Mode = 3 Median – 2 mean
14. Relation between arithmetic mean, geometric mean, and harmonic mean is
A.M > G.M > H.M

15. for a constant observation the relation


A.M > G.M > H.M becomes as A.M = G.M = H.M
Note:

Which measure of central tendency do we use for a given data set?

• For qualitative data, the mode is the only one of the three that makes sense.
• For quantitative data, mean or median is often preferred over the mode as a measure of center because
the value that occurs most frequently may not necessarily be located near the center of the data set.
• If there are outliers in the data, median is preferred over the mean as the mean can be misleading when
outliers are present.
• If there are no outliers, the mean is often the preferred choice because it has some nice properties, which
will be important in making inferences later in this course.

6|Page
QUESTION.1 Calculate Mean (Direct mean, Short Cut method, Coding method), Geometric Mean and
Harmonic Mean for the following frequency distribution

Class interval 3-5 6-8 9-11 12-14 15-17 18-20 21-23 24-26 27-29 30-32

Frequency 6 8 9 14 16 17 11 8 7 4

Solution:

Class interval f Mid-point fx D=X-A fD U=D/h fu logx flogx f/x

3-5 6 4 24 -15 -90 -5 -30 0.6020 3.6123 1.5

6-8 8 7 56 -12 -96 -4 -32 0.8450 6.76 1.14

9-11 9 10 90 -9 -81 -3 -27 1 9 0.9

12-14 14 13 182 -6 -84 -2 -28 1.1139 15.5946 1.07

15-17 16 16 256 -3 -48 -1 -16 1.2041 19.2656 1

18-20 17 19 323 0 0 0 0 1.2787 21.728 0.89

21-23 11 22 242 3 33 1 11 1.3424 14.7664 0.5

24-26 8 25 200 6 48 2 16 1.3979 11.1832 0.32

27-29 7 28 196 9 63 3 21 1.4471 10.1297 0.25

30-32 4 31 124 12 48 4 16 1.4913 5.9652 0.12

∑ 100 1693 -207 -5 -69 117.975 7.703

∑𝑓𝑥 1693 ∑𝑓𝑑 −207 ∑𝑓𝑢 −69


1- 𝑋̅ = ∑𝑓 = 100 = 16.93 2. 𝑋̅ = A+ ∑𝑓 = 19+ 100 = 16.93 3. 𝑋̅ =A+ ∑𝑓 x h = 19+ 100 *3=16.93

∑𝑓 𝑙𝑜𝑔𝑥 117.975
4. G.M = Antilog ( )= Antilog ( ) = Antilog(1.17975)=15.12
∑𝑓 100

∑𝑓 100
5. H.M = 𝑓 = 7.703 = 12.98
∑( )
𝑥

𝑓𝑚−𝑓1 17−16
6. Mode 𝑋̂ = 1+(𝑓𝑚−𝑓1)+(𝑓𝑚−𝑓 ) x h= 18 + (17−16)+(17−11) * 3 = 18.42
2

7|Page
QUESTION.2 Calculate Median , Quartile , Percentile, Decile , Bowley’s coefficient of skewness, quartile
deviation and coefficient of Quartile deviation

Class interval 3-5 6-8 9-11 12-14 15-17 18-20 21-23 24-26 27-29 30-32

Frequency 6 8 9 14 16 17 11 8 7 4

Solution

Class interval f Class boundaries Cumulative Frequency

3-5 6 2.5-5.5 6

6-8 8 5.5-8.5 14

9-11 9 8.5-11.5 23

12-14 14 11.5-14.5 37

15-17 16 14.5-17.5 53

18-20 17 17.5-20.5 70

21-23 11 20.5-23.5 81

24-26 8 23.5-26.5 89

27-29 7 26.5-29.5 96

30-32 4 29.5-32.5 100

∑ 100
𝒉 𝒏
1) Median = 𝑸𝟐 = =1+ 𝒇 (𝟐 − 𝒄)

Solution:
3
= 14.5+16 (50-37) = 16.93
𝒉 𝒏
2) - 𝑸𝟏= = L + 𝒇 (𝟒 − 𝒄)
Solution:
𝑛 100
= = 25
4 4
3
= 11.5 + 14 (25-23) = 11.92

𝒉 𝟑𝒏
𝟑− 𝑸𝟑= = L + 𝒇 ( 𝟒 − 𝒄)

8|Page
Solution:
3
= 20.5+11 (75-70) = 21.86

𝒉 𝟕𝒏
4- Decile = 𝑫𝟕= =L+𝒇 (𝟏𝟎 − 𝒄)

Solution:
7𝑛
= 70
10

3
17.5 + 17 (70-53) = 20.5

𝒉 𝟔𝟎𝒏
5- 𝑷𝒆𝒓𝒄𝒆𝒏𝒕𝒊𝒍𝒆= 𝑷𝟔𝟎 = L + 𝒇 (𝟏𝟎𝟎 − 𝒄)

Solution:
3
17.5+17 (60-53)= 18.73

𝑸𝟑 +𝑸𝟏 −𝟐𝑸𝟐
6- Bowley’s coefficient of Skewness = 𝑸𝟑 −𝑸𝟏

Solution:
21.86+11.92−2(16.93)
= 21.86−11.92

= -0.0080
𝑸𝟑 −𝑸𝟏
7- Quartile deviation = 𝟐

Solution:
21.86−11.92
= 2

= 4.97
𝑸 −𝑸
8- Coefficient of quartile deviation = 𝑸𝟑 +𝑸𝟏
𝟑 𝟏

Solution:
21.86−11.92 9.94
= 21.86+11.92 = 33.78

= 0.29

9|Page
QUESTION.3 In this classy you get score 84,75,72,90,100 and 85, your grade (weight) is determined from
the following sources: 20% from exam 1, 20% from exam 2, 25% from the final exam, 15% from lab, 15%
from homework, and 5% from the project. If you end up with the following scores, what is your final weighted
average in this class?

Solution:

Source Score, x Weight, w x•w

Exam 1 84 0.2 16.8

Exam 2 75 0.2 15

Final Exam 72 0.25 18

Lab 90 0.15 13.5

Homework 100 0.15 15

Project 85 0.05 4.25

 w =1  ( x • w) =82.55

x=
 ( x • w) 82.55 = 82.55
w = 1
QUESTION.4

The average salary of a group of unskilled workers is RS.10000 and that of a group of skilled workers is
Rs.15, 000. If the combined salary is Rs.12000, then what is the percentage of skilled workers?

Solution:

𝑛1 + 𝑛2 = 100 𝑛2 = 100 - 𝑛1
15000𝑛1 +(100−𝑛1 )10000
12000 =
100

1200000 = 15000𝑛1 + 1000000-10000𝑛1


200000 = 5000 𝑛1
n = 40%

10 | P a g e
MEASURE OF DISPERSION

11 | P a g e
MEASURE OF DISPERSION
• It is quite possible that two or more sets of data may have the same average (mean, median, or mode)
but their individual observations may differ considerably from the average. Thus, a value of central
tendency does not adequately describe the data. We therefore need some additional information
concerning with how the data are dispersed about the average.
• This is done by measuring the dispersion by which we mean the extent to which the observations in a
sample or in a population vary about their mean
• A quantity that measures this characteristic is called measure of dispersion, scatter or variability
• There are two types of measures of dispersion
• Absolute
• Relative
• An absolute measure of dispersion is one that measures the dispersion in terms of the same units as the
units of the data e.g., if the units of the data are rupees, meters, kilograms etc. the units of measures of
dispersion will also be rupees, meters, kilograms etc.
• A relative measure of dispersion is one that is expressed in the form of a ratio, coefficient or
percentage and is independent of the units of measurement
• It is useful for comparison of data of different nature

It is defined to be the amount of deviation of observation from an appropriate measure of central tendency

Absolute Measure of Dispersion:

1. Range
= Xm – Xo

2. Quartile range = Q3 – Q1

3. Semi-inter quartile range/quartile deviation


Q3 – Q1
= 2

4-Mean deviation (ungroup data)


The mean deviation (M.D) of a set of data is defined as the arithmetic mean of the deviations
measured either from the mean or from the median, all deviations being counted as positive
̅|
∑|X− X
(i) M. Dx̅ = from mean
n

̃|
∑|X− X
M. Dx̅ = from mean
n

(ii) Mean deviation (Frequency distribution)


̅|
∑f|X− X
M. Dx̅ = ∑f

12 | P a g e
̃|
∑f|X− X
M. Dx̅ = from mean
∑f

5-Variance:
• The variance of a set of observation is defined as the mean of the squares of deviations of
all the observations from the mean
• When it is calculated from the entire population, the variance is called the population
variance denoted by population variance
• If, instead, the data from the sample is used to calculate eh variance, it is referred to as
the sample variance and denoted by sample standard deviation
̅ )2
∑(X− X
(i) S2 = (ungroup Date)
n
̅ )2
∑f(X− X
(ii) S2 = (frequency distribution)
∑f
OR
Variance
∑X2 ∑x 2
S2 = -(n)
n

∑fX2 ∑fx 2
S2 = - ( ∑f )
∑f

Relative measure of dispersion (unit less quantity)/because ratio is unit less


Xm – Xo
1-Coefficient of range = Xm+ Xo

Q3 – Q1
2-Coefficient of quartile deviation = Q3+ Q1

3. Coefficient of variation
S
C.V = (X̅ x 100)%

4.Cofficient of Mean Deviation

M.Dx̅
= ̅
X
SKEWNESS

5. Pearson’s Coefficient of skewness


3(Mean−Median)
S.K = S.D

6. Bowley’s Coefficient of skewness


Q3 +Q1 −2Q2
S.K = Q3 −Q1

13 | P a g e
Properties of variance
i) Var (c) = 0, variance of constant is zero
ii) Var (x + a) = var x
iii) Var (x + y) = var x + var y
iv) Var (ax) = a2Varx
Properties of standard deviation

i) S.D (a) = 0
ii) S.D (x + a) = S.D(x)
iii) S.D (ax) = |a|S. D (x)
iv) S.D (x + y) = √var x + var (y)

Example
If var(X) = 16, Find var (3x) and var (3x – 10)
Sol
Var(3x) = 9 var x
= 9(16)
= 144
Also Var(3x - 10) = 9 var x+ var (10)
= 9(16) + 0
= 144

Empirical Rule: For a distribution that is approximately bell-shaped,

Approximately:

• 68.26% of the observations fall within one standard deviation of the mean. (µ ±ơ)
• 95.44% of the observations fall within two standard deviations of the mean. (µ ±2ơ)
• 99.74% of the observations fall within three standard deviations of the mean. (µ ±3ơ)

14 | P a g e
QUESTION.1 Calculate Mean, Mean deviation, Variance and coefficient of variance

Class interval 3-5 6-8 9-11 12-14 15-17 18-20 21-23 24-26 27-29 30-32

Frequency 6 8 9 14 16 17 11 8 7 4

Solution

Class interval f Midpoint 𝑥 − 𝑥̅ 𝑓|𝑥 − 𝑥̅ | fx fx2


(X)

3-5 6 4 -12.93 77.58 24 96

6-8 8 7 -9.93 79.44 56 392

9-11 9 10 -6.93 62.37 90 900

12-14 14 13 -3.93 55.02 182 2366

15-17 16 16 -0.93 14.88 256 4096

18-20 17 19 2.07 35.19 323 6137

21-23 11 22 5.07 55.77 242 5324

24-26 8 25 8.07 64.56 200 5000

27-29 7 28 11.07 77.49 196 5488

30-32 4 31 14.07 56.28 124 3844

∑ 100 578.58 1693 33643

∑𝑓𝑥 1693
̅=
1- 𝒙 = = 16.93
∑𝑓 100

∑𝑓|𝑥−𝑥̅ | 578.58
2- M.𝑫𝒙̅ = = = 5.78
∑𝑓 100

∑𝑓(𝑥−𝑋̅ )2 ∑𝑓𝑥 2 ∑𝑓𝑥 2 33643 1693 2


3- 𝑺𝟐 = = - ( ∑𝑓 ) = - ( 100 ) =336.43-286.62 S= √49.81 = 7.05
∑𝑓 ∑𝑓 100

𝑠 7.05
4- C.V = (𝑥̅ ∗ 100) % = 16.93 * 100%= 41.64%

15 | P a g e
QUESTION.2 The following table gives the marks of students: Calculate:
a) Quartile deviation b) Co-efficient of skewness

Marks f
30 – 39 8
40 – 49 87
50 – 59 190
60 – 69 86
70 – 79 20

A. B. C. D.

Solution
Marks f C.B C.f
30 – 39 8 29.5 – 39.5 8
40 – 49 87 39.5 – 49.5 95
50 – 59 190 49.5 - 59.5 285
60 – 69 86 59.5 – 69.5 371
70 – 79 20 69.5 – 79.5 391
∑ ∑f = n = 391 ------ ------

Quartile deviation
Q3 −Q1
Q.D = 2
h n
Q1 = I + f (4 − C) Q1 group
10 n
= 49.5 + 190 (97.75 – 95) th value
4
10 391
= 49.5 + 190 (2.75) = 4
th value
= 49.5 + 0.145 = 49.645 = 97.75th value

h 3n
Q3 = I + f ( 4 − C) Q3 group
10 3n
= 59.5 + 86 (293.25 – 285) th value
4
10 3(391)
= 59.5 + 86 (8.25) = th value
4
= 59.5 + 0.959 = 60.459 = 3(97.75)th value
Q3 −Q1
Q.D = =293.25
2
60.459−49.645 10.814
= = = 5.407
2 2

h n
Median = l + f (4 − C) Median group

16 | P a g e
10 n
= 49.5 + 190 (195.5 - 95) th value
4
391
= 49.5 + 5.289 = 54.789 = th value
2
= 49.5 + 5.289 = 54.789 = 195.5 th value
(b) Co-efficient of skewness is given as:
Q3 + Q1 −2Q2
Sk = Q3 −Q1
60.459+49.645−2(54.789) 0.526
= = 10.814 = 0.049
60.459−49.645

QUESTION.3 Calculate Mean Deviation from Mean, Mean co-efficient of dispersion and variance from
the data given below:
Weights (kg) No. of students Weights (kg) No. of students
50-53 23 65-68 66
53-56 24 68-71 49
56-59 39 71-74 38
59-62 46 74-77 21
62-65 54 77-80 12

Also calculate Range, Quartile Deviation, and co-efficient of Quartile Deviation.

Solution:
Weights (kg) f X fx |X - 𝐗̅| f|X-𝐗 ̅| fX2 C.f.
50 – 53 23 51.5 1184.5 13.08 300.84 61001.75 23
53-56 24 54.5 1308.0 10.08 241.92 71286 47
56 – 59 39 57.5 2242.5 7.08 276.12 128943.75 86
59-62 46 60.5 2783.0 4.08 187.68 168371.50 132
62-65 54 63.5 3429.0 1.08 58.32 217741.5 186
65-68 66 66.5 4389.0 1.92 126.72 291868.50 252
68 – 71 49 69.5 3405.5 4.92 241.08 236682.25 301
71-74 38 72.5 2755.0 7.92 300.96 199737.50 339
74-77 21 75.5 1585.5 10.92 229.32 119705.25 360
77-80 12 78.5 942.0 13.92 167.04 73947 372
∑ 372 ----- 24024 ----- 2130 1569285 -----

∑fX 24024
̅
X = = = 64.58
∑f 372
∑f|X−X̅| 2130
M.D (x̅) = = = 5.73
∑f 372
M.D (x̅) 5.73
Mean co-efficient of dispersion = ̅
= 64.58 = 0.09
X
∑fX2 ∑fX 2 1569285 24024 2
Variance = - ( ∑f ) = - ( )
∑f 372 372
= 4218.51 – 4170.65 = 47.85
Range = Xm – Xo
= 78.5 – 51.5 = 27
Q3 −Q1
Quartile Deviation = Q.D = 2
17 | P a g e
h n
Q1 = I + f (4 − C) Q1 group
3 n
= 59 + 46 (93 - 86) th value
4
372
= 59.5 + 0.46 = 59.46 = 4 th value
= 93rd value
h 3n
Q3 = I + f ( 4 − C) Q3 group
3 3n
= 68 + 49 (279 - 252) th value
4
3(372)
= 68 + 1.65 = 69.65 = 4 th value
= 279th value
Q3 − Q1 69.65 − 59.46
Now Q.D = = = 5.1
2 2
Q3 − Q1
Co-efficient of Q.D =Q
3 + Q1
69.65 − 59.46 10.19
= = = 0.08
69.65+ 59.46 129.11

QUESTION.4 Find Karl Pearson’s co-efficient of skewness from the following data:

Ages 22 27 32 37 42 47 52
Cumulative Frequency 1 5 13 24 39 48 50

Solution:

X C.f C.B f fx fx2


22 1 19.5-24.5 1 22 484
27 5 24.5-29.5 4 108 2916
32 13 29.5-34.5 8 256 8192
37 24 34.5-39.5 11 407 15059
42 39 39.5-44.5 15 630 26460
47 48 44.5-49.5 9 423 19881
52 50 49.5-54.5 2 104 5408
∑ ----- ∑ 50 1950 78400

∑fX 1950
Mean = ̅
X = ∑f
= 50
= 39
∑fX2
S.D = √ ̅) 2
− (X
∑f
78400
= √ − (39)2 = √1568 − 1521 = √47 = 6.85
50

h n
Median = I + f (2 − C)
Median group:
5 50 n 50
= 39.5 + 15 ( 2 − 24) th value = th value = 25 th value
2 2
372
= 39.5 + 0.33(1) = 39.83 = th value
4
= 93rd value

18 | P a g e
Karl Pearson’s co-efficient of skewness:

3 (Mean− Median)
s .k =
S.D

3(39−39.83) −2.49
= = = -0.36
6.85 6.85

Hence, the distribution in negatively skewed.

QUESTION.5
The mean and standard deviation of a sample of 100 observations were found to be 104 and 4.7 respectively.
Later, error was detected in three records as enumerated below:
S. No. Correct figure Amount taken
(as per original record) (For computation)
1 151 115
2 78 87
3 98 89
Find the correct mean and standard deviation

Solution
∑𝑥
𝑀𝑒𝑎𝑛 = = 𝑥̅ = 104
𝑛
∑ 𝑥 = 𝑛𝑥̅ = 100(104) = 10400
With correct figure

∑ 𝑥 10400 + 151 + 78 + 98 − 115 − 87 − 89 10,436


𝑀𝑒𝑎𝑛 = = = = 104.36
𝑛 100 100
2
∑ 𝑥2 ∑𝑥
𝑆𝐷 = √ − ( ) = 4.7
𝑛 𝑛

∑ 𝑥2 10,400 2
−( ) = 22.09
𝑛 100

∑ 𝑥2
= 22.09 + 10,816 = (22.09 + 10,816)100 = 1,083,809
𝑛
With correct figures
∑ 𝑥 2 = 1,083,809 + (151)2 + (78)2 + (98)2 − (115)2 − (87)2 − (89)2 = 1,093,583
2
∑ 𝑥2 ∑𝑥 1,093,583 10,436 2
𝑆𝐷 = √ −( ) =√ −( )
𝑛 𝑛 100 100
= √10935.83 − 10891.0096 = √44.8204 = 6.7

19 | P a g e
QUESTION.6
A manufacturer of television tubes produces two types A and B of tubes. The tubes have respective mean life
̅A = 1496 hours and X
times as X ̅B = 1895 hours and standard deviations S̅A = 280 hours and S̅B = 310 hours.

(i) Absolute dispersion (ii) relative dispersion

Solution
(i) Absolute dispersion (All dispersion without coefficient)
Since SB > SA
Hence, Tube B has greater absolute dispersion

(ii) Relative dispersion (All coefficient is called relative dispersion)


Tube – A Tube – B
̅
XA = 1496 ̅
XB = 1895
SA = 280 SB = 310
S A S B
C.V(A) = (̅̅̅̅ x 100)% C.V(B) = (̅̅̅̅ x 100)%
X A X B
280 310
= (1496 x 100)% = (1895 x 100)%
= 18.72% = 16.36%
Tube A has greater relative dispersion.
Question 7.
For the flowing distribution calculate the variance, standard deviation and coefficient of variation
X f
20 – 29 4
30 – 39 12
40 – 49 19
50 – 59 25
60 – 69 20
70 – 79 14
80 – 89 6

Solution
X Midpoint (Y) D fD 𝒇𝑫𝟐
20 – 29 4 24.5 -30 -120 3600
30 – 39 12 34.5 -20 -240 4800
40 – 49 19 44.5 -10 -190 1900
50 – 59 25 55.5 0 0 0
60 – 69 20 65.5 10 200 2000
70 – 79 14 74.5 20 280 5600
80 – 89 6 84.5 30 180 5400

100 110 23300


∑ 𝑓𝐷 110
𝑀𝑒𝑎𝑛 = 𝐴 + = 54.5 + = 54.5 + 1.1 = 55.6
∑𝑓 100
20 | P a g e
2
∑ 𝑓𝐷 2 ∑ 𝑓𝐷 23300 110 2
𝑣𝑎𝑟𝑖𝑎𝑐𝑛𝑒 = −( ) = −( ) = 233.00 − 1.21 = 231.79
∑𝑓 ∑𝑓 100 100
𝑆𝐷 = √𝑣𝑎𝑟𝑖𝑎𝑛𝑐𝑒 = √231.79 = 15.22
𝑆. 𝐷 15.22
𝐶. 𝑉 = × 100 = × 100 = 27.37
𝑀𝑒𝑎𝑛 55.6

Empirical relation between Mean, Median and Mode

The following relation hold between mean, median and mode if the distribution is moderately skewed

Mode=3median -2mean

(i) If frequency distribution skewed to right then the position of Mean, Median and mode

Mean>Median>Mode (Positive skewed)


and its shape will be

(ii) If the frequency distribution skewed to the left then the position of mean, and mode will be as

Mean<Median<Mode (Negative skewed)


nd its shape will be

(iii) But if distribution is symmetrical then Mean, Median will coincide i-e equal
Mean=Median=Mode

21 | P a g e
and its shape will be

Relation between A.M, G.M and H.M

It is observed that if all the values are same then

A.M=G.M= H.M

Symmetry

A frequency distribution is called symmetrical if its frequency curve has same shape on both sides of central
highest line which divides the curve into two equal parts.

OR

A frequency distribution is called symmetrical if deviation below the mean exactly equal the deviation above
the mean.

For symmetrical distribution

Mean = Median = Mode

𝑄3 _ 𝑄2 = 𝑄2 _ 𝑄1

Skewness (Lack of symmetry)

If curve of any frequency distribution has large tail to either side, then it is called skewed distribution.

Positive Skewness

If any frequency distribution has larger tail to right side, then is called positively skewed or skewed to right. In
positively skewed distribution

Mean > Median > Mode And 𝑄3 _ 𝑄2 > 𝑄2 _ 𝑄1

22 | P a g e
Negative Skewness

If curve of any frequency distribution has longer tail to the left side, then it is called negatively skewed or
skewed to left.

In negatively skewed distribution

Mean < Median < Mode

And 𝑄3 _ 𝑄2 < 𝑄2 _ 𝑄1

Coefficient of Skewness

It measures the degree of skewness in frequency distribution

(a) Karl person Coefficient of skewness


𝑀𝑒𝑎𝑛−𝑀𝑜𝑑𝑒
(i) SK = 𝑆.𝐷
3(𝑀𝑒𝑎𝑛−𝑀𝑒𝑑𝑖𝑎𝑛)
(ii) Sk = 𝑆.𝐷
(b) Bowley’s Coefficient of Skewness
𝑄3 + 𝑄1 −2𝑄2
SK = 𝑄3 − 𝑄1

Question:1

What can you say about skewness in each of the following?

(i) Mean = 10 Mode = 12


(ii) Median = 15 Mode = 12
(iii) Mean = 13 Median = 15
(iv) 𝑄1 = 10 𝑄2 = 20 and 𝑄3 = 40
(v) 𝑄1 = 10 𝑄2 = 20 and 𝑄3 = 30
(vi) 𝑄1 = 10 𝑄2 = 15 and 𝑄3 = 17

Solution

Here we give the answer without using formulas, as it is required to examine the skewness only, not degree.

(i) As Mean < Mode


∴ Distribution is negatively skewed
(ii) As Median > Mode
∴ Distribution is positively skewed
(iii) As Mean < Median
∴ Distribution is negatively skewed
(iv) 𝑄3 - 𝑄2 = 40 – 20 = 20
𝐴𝑛𝑑 𝑄2 - 𝑄1 = 20 – 10 = 10
As 𝑄3 _ 𝑄2 > 𝑄2 _ 𝑄1

23 | P a g e
∴ Distribution is positively skewed
(v) 𝑄3 - 𝑄2 = 30 – 20 = 10
And 𝑄2 - 𝑄1 = 20 – 10 = 10
As 𝑄3 _ 𝑄2 = 𝑄2 _ 𝑄1
∴ Distribution is symmetrical

𝑄3 - 𝑄2 =17 – 15 = 2

And 𝑄2 - 𝑄1 = 15 – 10 = 5
As 𝑄3 _ 𝑄2 < 𝑄2 _ 𝑄1
∴ Distribution is negatively skewed

Question:2

Find the quartile deviation and coefficient of quartile deviation from the following marks obtained by students
on a test in mathematics.

29,37,53,74,82,39,42,20,28,81,68,58,67,54,93,70,30,61,55,36

Solution

After arranging the observations in ascending order, we get:


𝑛+1 Th 20+1 th
𝑸𝟏 =( ) item =( ) item
4 4

= (5.25) th item = 5th item + 0.25 (6th item – 5th item)

= 36 + 0.25 (37 – 36) = 36+0.25 = 36.25


3(𝑛+1)th 3 (20+1)th
𝑸𝟑 = item = item
4 4

= (15.75)th item = 15th item + 0.75 (16th item – 15th item )

= 68 + 0.75 (70 -68) = 68 +1.5 = 69.5


𝑄3− 𝑄1 69.5+36.25 33.25
Quartile Deviation (Q.D) = = = = 16.625
2 2 2

𝑄 −𝑄 69.5−36.25 33.25
Coefficient of Quartile Deviation (Q.D) = 𝑄3+ 𝑄1 = 69.5+36.25 = 105.75 = 0.314
3 1

24 | P a g e
PRACTICE QUESTION

25 | P a g e
QUESTION.1 Calculate mean, median and mode for the grouped data of the following table:

Classes 0-4 4-8 8-12 12-16 16-20 20-24


Frequency 5 10 12 14 6 4

Solution:

Classes F x fx c.f
0-4 5 2 10 5
4-8 10 6 60 15
8-12 12 10 120 27
12-16 14 14 196 41
16-20 6 18 108 47
20-24 4 22 88 51
∑ 51 --- 582 ----

∑fX 582
Mean = ̅
X = ∑f = = 11.41
51

h n
Median = I + f [2 − C] Median class:
4 n
= 8 + 12 [25.5 − 15] = [2]th value
51
= 8 + 3.5 = 11.5 = th value
2
= 25.5 th value
fm −f1
Mode = I + x h
(fm −fl)+ fm −f2

Since maximum frequency is 14; therefore 12 - 16 is modal class.

(14 −12)
Mode =12 + (14 −12) + (14 − 6) x 4= 12 + 0.8= 12.8

QUESTION.2 For a certain distribution with the mean and median are 45 and 36 respectively, find the mode
using the empirical relation.

Solution:
Mean = 45 Median = 36 Mode=?
As we know that
Mode = 3 Median -2 Mean
= 3 (36) -2(45) = 108 - 90 = 18
QUESTION.3

26 | P a g e
Bilal gets a rise of 10% in salary at the end of his first year of service and further rise of 20% and 25% at the
end of second and third year respectively. The rise in each case being calculated on his salary at the beginning
of the year. To what annual percentage increase in this equivalent.

Solution:
Let salary of Bilal at the beginning of 1st year is 100.
Increment at the end of (X)
st
1 year (10%) 110
nd
2 year (20%) 120
rd
3 year (25%) 125

G.M = n√X1 x X2 x … … . x Xn
3
= √110 x 120 x 125
3
= √1650000 = 118.17
The annual percentage increases
= 118.17-100, = 18.17%

QUESTION.4
The frequency distribution given below has been derived from the use of working origin. If
D = X-18, find arithmetic mean and geometric mean.

D -12 -8 -4 0 4 8 12 16
F 2 5 8 18 22 13 8 4

Solution:
D F fD X = D+18 log X f log X
-12 2 -24 6 0.7781 1.5562
-8 5 -40 10 1 5
-4 8 -32 14 1.1461 9.1688
0 18 0 18 1.2552 22.5936
4 22 88 22 1.3424 29.5328
8 13 104 26 1.4149 18.3937
12 8 96 30 1.4771 11.8168
16 4 64 34 1.5314 6.1256
2 80 256 ----- ----- 104.1875

Arithmetic Mean
∑fD
̅
X=A+ ∑f
256
= 18+ = 18 + 3.2 = 21.2
80

27 | P a g e
Geometric Mean:
∑f log X
G.M = antilog ( )
∑f
104.1875
G.M = ( ) = antilog (1.3023) = 20.06
80

QUESTION.5
Reciprocal of X are given below.
0.0267, 0.0235, 0.0211, 0.0191, 0.0174, 0.0160, 0.0148
Calculate H.M.

Solution:

1 0.0267 0.0235 0.0211 0.0191 0.0174 0.0160 0.0148 0.1368


x

Harmonic Mean
n 7
H.M = 1 H.M = 0.1368 = 50.505
∑( )
X

QUESTION.6
3 Cities A, B, C are equidistant from each other. Fatima travels from A to B at the speed of 30 miles per
hour by car. From B to C, at speed of 50 miles per hour. Determine her average speed for the entire trip.

Solution:

Speed (X) 30 50 ∑
1 0.033 0.02 0.0533
x

Harmonic Mean
n 2
H.M = ∑(1/X) = 0.0533 = 37.5 km/h

QUESTION.7
Find out. The average rate of motion in the case of a person who rides the first mile at the rate of 10 miles
per hour the next mile at the rate of 8 miles per hour and third at the rate of 6 miles per hour.
Solution:
Speed (X) 10 8 6 ∑

1 0.1 0.125 0.1667 0.3917


x
Harmonic Mean:
n 3
H.M = ∑(1/X) = 0.3917 = 7.6596

28 | P a g e
QUESTION.8 Increase in population in the first decade has increased 20% in the next 25% and in
the third 4%. Find out the percentage increase in population.

Solution: Let the population at start of the 1st decade be 100.


Increase in Population 1st decade (10%) 2nd decade (25%)
Y 120 125
Geometric Mean:

G.M = n√Y1 x Y2 x … … . x Yn
3 3
G.M = √120 x 125 x 104 = √1560000 = 115.98
Increase percentage in population
= 115.98 - 100= 15.98%

QUESTION.9
Compute Mean, Median, Mode, 6th Decile, and 74th Percentile for the data given in the table.
Classes 0.7312- 0.7314- 0.7316 - 0.7318 - 0.7320- 0.7322 - 0.7324 -
0.7313 0.7315 0.7317 0.7319 0.7321 0.7323 0.7325
Frequency 10 15 20 25 30 8 2
Solution:

Classes f X fx C.F C.B


0.7312 - 0.7313 10 0.73125 7.3125 10 0.73115 -0.73135
0.7314 - 0.7315 15 0.73145 10.9718 25 0.73135 - 0.73155
0.7316 - 0.7317 20 0.73165 14.633 45 0.73155 - 0.73175
0.7318 - 0.7319 25 0.73185 18.2963 70 0.73175 - 0.73195
0.7320 - 0.7321 30 0.73205 21.9615 100 0.73195 - 0.73215
0.7322 - 0.7323 8 0.73225 5.858 108 0.73215 - 0.73235
0.7324 - 0.7325 2 0.73245 1.4649 110 0.73235 - 0.73255
∑ 110 ---- 80.4979 ---- ----

Mean:
∑fX
̅
X = ∑f

80.4979
= = 0.7318
110
Median: Median group:
h n n
̃
X = l + ( − C) th value of the array
f 2 2
0.0002 110
= 0.73175 + (55 - 45) = th value of the array
25 2
= 0.7318 = 55 th value of the array

29 | P a g e
Mode
fm − f1
̂ =l +
X x h
(fm −fl)+(fm −f2 )

30 − 25
= 0.73195 + (30 −25) + (30 − 8) x 0.0002 = 0.7319
D6 = ? D6 group:
h 6n 6n
D6 = l + f (10 − C) th value of the array
10
0.0002 6(110)
D6 = 0.73175+ (66 − 45) = th value of the array
25 10
= 0.73192 = 66 th value of the array

P74 = ? P74 group:

h 74n 74n
P74 = l + f (100 − C) th value of the array
100

0.0002 74(110)
P74 = 0.73195+ (81.4 − 70) = th value of the array
30 100
= 0.7320 = 81.4 th value of the array

QUESTION.10

If for any distribution the Mean is 45 and Median is 30. Find approximately using formula connecting
the three.

Solution: We know that


Mode = 3 Median - 2 Mean
= 3(30) - 2(45) = 90 - 90 = 0

QUESTION.11
For a certain distribution, if ∑(X-15) = 5, ∑(X - 18) = 0 , ∑(X - 211) = -21 What is the values of A.M and
why?
Solution:
A.M = 18. Because sum of deviations of observation from their mean is zero.

QUESTION.12
Arithmetic Mean of 15 values is 20 and by adding 3 more values the mean remains 20. Find the new three
values if ratio is a : b : c :: 3 : 2 : 1

Solution:
For 15 Values Adding 3 more values
n = 15 n = 18
̅
X = 20 ̅
X = 20
∑X ∑X
̅
X= ̅
X=
n n

30 | P a g e
∑X ∑X
20 = 20 =
15 18
∑X = 20(15) = 300 ∑X = 18(15) = 360

Total of 3 values = 360 - 300 = 60

The ratio is
a : b : c =3 : 2:1
a : b :c Sum of ratio=3+2+1=6
3 2 1

3
a = 60 x 6 = 30

2
b = 60 x 6 = 20

1
c = 60 x 6 = 10
a: b: c
30 : 20 : 10

QUESTION.13- Autumn 2003


The rate of inflation in four successive years in a country was 7 percent, 11 percent, 15 percent and 19 percent.
Using the geometric mean, find the overage rate of inflation per year.

A. B. C. D.
Solution
Rate 7% 11% 15% 19%
Increase 107 111 115 119

Now
G.M = (107 x 111 x 115 x 119)1/4
G.M = 112.91
And Average rate of inflation per year = 112.9 - 100
= 12.91%

QUESTION.14- Spring 2003


The mean temperature in Karachi in the month of January is 16° C with a standard deviation of 0.5 C. On
January 15, the temperature is 4° C standard deviation above the mean. What is the temperature on January 15?

A. B. C. D.

Solution
Temperature on January 15 = x̅ + 4σ(Standard deviation)
= 16 + 4(0.5)
31 | P a g e
= 16 + 2
= 18oc
QUESTION.15- Autumn 2002
In which situation, will Arithmetic Means, Harmonic Mean and Geometric Mean give the same result?

A. B. C. D.

Solution
In symmetrical distribution the value of mean, median and mode is same.

QUESTION.16- Spring 2002


In a perfectly symmetrical frequency curve, the mean of the data is 88. Find median and mode.

A. B. C. D.

Solution:
Given
Mean = 88
In perfectly symmetrical frequency curve
Mean = Median = Mode = 88

QUESTION.17- spring 2002


In a moderately skewed frequency distribution mean 60 and median = 55, Find mode.

A. B. C. D.
Solution
Mode = 3 median - 2 Mean
Mode = 3(55) - 2(60)
Mode = 165 - 120
Mode = 45
2004
Question.18 Calculate 1st quartile, 5th decile and 70th percentile for the following frequency distribution.

A. B. C. D.

Class 3-5 6-8 9-11 12-14 15-17 18-20 21-23 24-26 27-29 30-32
interval

Frequency 6 8 9 14 16 17 11 8 7 4

Solution:
Class f Class Cumulative
interval (C.B) Frequency
32 | P a g e
boundaries
3–5 6 2.5 - 5.5 6
6–8 8 5.5 - 8.5 14
9 - 11 9 8.5 - 11.5 23
12 - 14 14 11.5 - 14.5 37
15 - 17 16 14.5 - 17.5 53
18 - 20 17 17.5 - 20.5 70
21 - 23 11 20.5 - 23.5 81
24 - 26 8 23.5 - 26.5 89
27 - 29 7 26.5 - 29.5 96
30 - 32 4 29.5 - 32.5 100
100

Quartile
n ∑f 100
4
= 4
= 4
= 25th value
Lies in the class (11.5 - 14.5)
n
l = 11.5, f = 14, 4 = 25, c = 23, r = 3
now
r n
Q1 = l + f (4 – c)
13
Q1 = 11.5 + 14 (25 - 23)
13
Q1 = 11.5 + 14 (2)
3
Q1 = 11.5 + 7
Q1 = 11.5 + 0.43
Q1 = l1.93

Decile
5n 5x100 500
= = = 50th value
10 10 10
50th value lies in the class (14.5 – 17.5)
5n
l = 14.5, h = 3, f = 16, 10 = 50, c = 37
n 5n
D5 = l + 10 (10 – c)
3
D5 = l4.5 + 16 (50 - 37)
3
D5 = l4.5 + 16 (13)
D5 = l4.5 + 2.43
D5 = l6.93

Percentile
70n 70 x 100
= = 750th value
100 100

33 | P a g e
70th value lies in the class (17.5 – 20.5)
l = 17.5, h = 3, f = 17, c = 53

Using
h 70n
P70 = l + f (100 - c)
3
P70 = l7.5 + 17 (70 - 53)
3
P70 = l7.5 + 17 (17)
P70 = 20.5

Question.19 The mean annual salary of all employees in a company is Rs. 150,000. The mean annual
salary of male and female employees is Rs. 162,000 and Rs. 102,000 respectively. Find the percentage of
male and female employees in the company.

A. B. C. D.

Solution
Given
̅
Xc = 150,000
̅
X1 = 162,000
̅ 2 = 102,000
X
and let
n1 + n2 = 100
n2 = 100 – n1
now using
̅ +n X
n X ̅
̅
X2 = 1 1 2 2
n1 + n2
n1 (162,000)+ (1 00− n1 )(102,000)
150,000 = 100
15,000,000 = 162,000 n1 + 10,200,000 - 102,000 n1
4,800,000 = 60,000n1
n1 = 80%
n2 = 100 - 80% = 20%

Question.20
If the A.M and G.M of two numbers are 6.5 and 6 respectively. Then the two numbers are
Solution:
Let the required numbers are a & b
Then
𝑎+𝑏
A.M = = 6.5
2
A +b = 13 __ (i)
Also
1
G.M = (𝑎𝑏) ⁄2 = 6
(ab) = 36 ___ (2)
From (i)
34 | P a g e
A +b = 13
B = 13 –a ____ (iii) in (ii)
a(13-a) = 36
13a - 𝑎2 = 36
𝑎2 – 13a + 36 = 0
Using calculator
a=9,4
Put in (iii)
b=4,9
Hence required numbers are 9 , 4

Question.21
If the A.M of 10 numbers is 15. But the number 15 and 17 are wrongly added in place of 51 and 71
respectively. Find the corrected mean.
Solution:
∑𝑥
𝑋̅ = 𝑛
∑𝑥
15 = 10
∑x = 150
Correct total = 150- 15 -17 + 51 +71
= 240
∑𝑥
Corrected Mean = 𝑛
240
= 10
Corrected Mean = 24

Question.22
What is the average for a student who received grades 85,76 and 82 on 3 midterm tests and a 79 on the
final examination, if the final examination count 3 times as much as each of the three tests?
Solution:
̅ = 𝟖𝟓+𝟕𝟔+𝟖𝟐+𝟕𝟗+𝟕𝟗+𝟕𝟗
𝑿 𝟔
𝟒𝟖𝟎
̅=
𝑿 𝟔
̅ = 80
𝑿

Question.23
The average daily sale and related standard deviation of Ali, Atif, Ahmed and Azam in thousands of Rs.
Are 41& 4.3, 36&3.2, 26&2.9 and 24&2.5 respectively, then the most inconsistent among them is
Solution:
𝑆 4.3
Ali = 𝑋̅ *100 = ∗ 100 = 10.48
41
𝑆 3.2
Atif = 𝑋̅ *100 = 36
∗ 100 = 8.88
𝑆 2.9
Ahmed = 𝑋̅ *100 = ∗ 100 = 11.15
26

35 | P a g e
𝑆 2.5
Azam = 𝑋̅ *100 = ∗ 100 = 10.41
24

Question.24
If 5 is the harmonic mean between 2 and b. Find b
Solution:
2
5=1 1
+
2 𝑏
4𝑏
5 = 𝑏+2

5b+10 = 4b
b = -10
Question: 25
If the relationship between two variables u and v are given by 2u + v +7 = 0 and if the AM of u is 10, then
the AM of v is.
Solution:
2u + v +7 = 0 and 𝑢̅ = 10
Then
2𝑢̅ + 𝑣̅ + 7 = 0
2(10) + 𝑣̅ + 7 =0
𝑣̅ +27 = 0
V = -27
Question.26
The average donation paid by 10 students for poor students’ welfare fund was RS.325. if 9 of the students
paid the amount 425, 320,210,290,335,380,290,350 and 375. What amount was paid by the 10th students
also find median and mode of the data.
Solution:
∑x = 325* 10= 3250
Sum of 9 students = ∑x = 2975
Donations by 10th student = 3250 – 2975 = 275
Median = 210, 275, 290, 290, 320,335, 350, 375,380, 425
320+335
Median = = 327.5
2

Mode = 290

36 | P a g e
Question. 27
The age (in years) of 50 persons range from 20 to 44 years. They are grouped in 5 classes of size 5 years
each. The class frequencies from lowest to highest class are: 6, 15,19,6 and 4:
Calculate sum of f𝒙𝟐 for the above data.
Solution:
Classes f X fx f𝑥 2 C.B C.F
20-24 6 22 132 2904 19.5-24.5 6
25-29 15 27 405 10935 24.5-29.5 21
30-34 19 32 608 19456 29.5-34.5 40
35-39 6 37 222 8214 34.5-39.5 46
40-44 4 41 164 6724 39.5-44.5 50
∑f=50 1531 48565
i) ∑f𝑥 2 = 48565
∑𝑓𝑥 1531
ii) 𝑥̅ = = = 30.7
∑𝑓 50
2
∑𝑓𝑥 2 ∑𝑓𝑥 2
iii) 𝑣𝑎𝑟𝑖𝑎𝑛𝑐𝑒 = -( )
∑𝑓 ∑𝑓

48565 1531 2
= -( )
50 50

Variance = 971.3 – 937.58


Variance = 33.72
iv) 20% are of above what age?
ℎ 80𝑛
P80 = L+𝑓 (100 − 𝑐)
80𝑛 80∗50
= = 40th value
100 100
5
∴ P80 = 34.5 + 6 (40-40)
P80 = 34.5
Question.28
The median and modal profits for the following data.
1-4 10
5-9 25
10-14 45
15-19 55

37 | P a g e
20-24 62
25-29 65

Question: 29
For the following incomplete distribution of marks of 100 pupils, median mark is known to be 32.

Solution:
Marks X f C.f
0-10 5 10 10
10-20 15 X 10+x
20-30 25 25 35+x
30-40 35 30 65+x
40-50 45 Y 65+x+y
50-60 55 10 75+x+y

Since
∑f = 100
75+x+y = 100

x+y = 25 ___(i)

Also
ℎ 𝑛 𝑛 100
Median = L+𝑓 ( 2 − 𝑐) where 2 = = 50
2
10
32=30+30 (50-35-x)
2*3 = 15-x
6 = 15-x
X=9
Put in (i)
a + y = 25
y = 16
Also
Marks 0-10 10-20 20-30 30-40 40-50 50-60
No. of 10 9 25 30 16 10
students
X 5 15 25 35 45 55
Fx 50 135 625 1050 720 550
Total

∑𝑓𝑥 3130
𝑥̅ = = = 31.3
∑𝑓 100

Question; 30
Sales of product P in May and June were 58 units and 48 units respectively. The arithmetic mean of
monthly sales for the period January to April was 44 units per month. The arithmetic mean of monthly
sales for the period January to June is….
Solution:
38 | P a g e
𝑀𝑎𝑦+𝐽𝑢𝑛𝑒 58+48
= = 53
2 2
Now
Jan Feb March April May June
44 44 44 44 53 53

44+44+44+44+53+53
𝑥̅ = 6
𝑥̅ = 47

Question: 31
The mean annual salary of all employees in a company is Rs.150, 000. The mean annual salary of male
and female employees is Rs.162, 000 and Rs.102, 000 respectively. Find the ratio of male and female
employees in the company.

Solution:
Let
Male = x
Female = y
Then
162,000𝑥+102,000 𝑦
= 150,000
𝑥+𝑦
162,000x + 102,000y = 150,000x+ 150,000y
12,000x = 48000y
𝑥 48000
=
𝑦 12000
𝑥 4
=1
𝑦
∴ x = y = 4:1
∴ Male = 80%
Female = 20%

Question: 32
The variance of set of data is 9. If every item is doubled then what will be the variance of the resulting
data.

Solution:
Var (2x) = 4.var x
= 4 (9) = 36

Question: 33
∑ (x-20) = 0, ∑(𝒙 − 𝟐𝟎)𝟐 = 400, find C.V

Solution:
𝑆 20
*100 = 20 *100
𝑋̅

Question: 34
̅ , and 2S = 7𝒙
IF 2C.V = 7 𝒙 ̅ , find 𝒙
̅

Solution:
2C.V = 7𝑥̅ ___ (i)

39 | P a g e
2S = 7𝑥̅ ___ (ii)
Compare
C.V = S
C.V = S
𝑆
*100 = S
𝑥
100
=1
𝑥̅
𝑥̅ = 100

Question: 35
For a data having two observation S.D =4. Then range

Solution:
𝑅𝑎𝑛𝑔𝑒
S.D = 2
𝑟𝑎𝑛𝑔𝑒
4=
2
8 = range

Question: 36
Mean of 10 values is 20, whereas 2 incorrect values 13 and 17 were used instead of 18 and 15. What is the
correct mean?
Solution;
∑𝑥
n = 10, 𝑥̅ = 20, 𝑥̅ = 𝑛
∑x = 200- 13 -17 +18 +15
∑x = 203
Now
Corrected Mean
203
𝑥̅ = 10 = 20.3

Question: 37
The average wage of 100 workers of two shifts is RS.284. the average wage of 70 workers of the first shift
is Rs.290. what is the ratio of average wage of the workers of second shift to that of average wage of total
workers?

Solution:
𝑥̅ 𝑐 = 284, 𝑛1 + 𝑛2 = 100, 𝑛1 = 70, 𝑛2 = 30 𝑥̅ 1 = 290, 𝑥̅ 2 =?

Using
70(290)+30(𝑥̅ 2 )
284 = 100
28400= 20300 + 30𝑥̅ 2
𝑥̅ 2 = 270
Now
Shift 2 = Shift total
270 = 284
135 = 142

40 | P a g e
Question: 38
The average runs of a cricketer in four innings are 32. How many runs he should score in the fifth
innings so that his average score becomes 50 runs?

Solution:
∑x = 32 * 4
128 = ∑x
Also
∑𝑥+𝐴
𝑥̅ = 𝑛+1
𝟏𝟐𝟖+𝑨
50 = 𝟓
250 = 128 + A
122= A

Question: 39
The number of articles produced per day (x) and the total cost (C in Rs.) of these articles are related by
C= 50-0.05x
Mean and variance of the daily production for 5 working days are computed as 100 and 13.5 respectively.
Determine the mean and variance of the daily cost of production.

Solution:

Mean (C) = 50- 0.05 Mean(x)

Mean(C) = 50- 0.05 (100)

C = 50- 5 = 45
Also
Variance(C) = (−0.05)2 (13.5)
= 0.03375

Question: 40
Over a period, a firm made purchases of 400,500,550 and 600 on items, the unit costs of which were 10,
12.50, 11.00 and 12.00. The average price paid per item was

Solution:
X fx f
10 400 40
12.50 500 40
11 550 50
12 600 50
Total 2050 180

∑𝑥
𝑥̅ =𝑛
2050
𝑥̅ = 180
𝑥̅ = 11.39

41 | P a g e
Question: 41
Sales for the first five months of the year average 8200 per month. For the last four months of the year.
Sales averaged 8500 per month. If sales for the year totaled 102,000.The average for the sixth, seventh
and eighth months be
Solution:

̅=
𝒙
∑𝒙
̅=
𝒙
∑𝒙 =Total -∑x - ∑x ̅=
𝒙
∑𝒙
𝒏 𝒏 𝒏
∑𝑥 ∑𝑥 27000
8200= 5 8500= 5 =102000-41000-34000 𝑥̅ = 3
∑x=41000 ∑x=340000 =270000 𝑥̅ = 9000

ASSIGNMENT
Question: 1
Over a period of 4 years, an employee’s salary has increased in the ratios, 1.072, 1.086, 1.069 and 1.098. The
average of these ratios and hence the average percent increases are:
1.08125 and 8.125% 1.086 and 8.6%
1.08119 and 8.6% None of these
Question: 2
The average daily sale and the related standard deviation of Ali, Atif, Ahemd and Azeem in thousands of
Rupees are 41 & 4.3, 36 &2.9 and 24 &2.5 respectively, then the most inconsistent among them is:
Ali Atif
Ahmed Azeem
Question: 3
If the AM and GM for two numbers are 6.50 and 6 respectively then the two numbers are:
6 and 7 9 and 4
10 and 3 8 and 5
Question: 4
If the AM and HM for two numbers are 5 and 3.2 respectively then the GM will be:
16 4.10
4.05 4.00
Question: 5
If there are two groups containing 30 and 20 observations and having 50 and 60 as arithmetic means, then the
combined arithmetic mean is:
55 56
54 52
Question: 6
The average salary of a group of unskilled workers is Rs.10000 and that of a group of akilled workers is
Rs.15,000. If the combined salary is Rs.12000, then what is the percentage of skilled workers?

42 | P a g e
40% 50%
60% None of these
Question: 7
An aero plane flies A to B at the rate of 500 km/hour comes back from B to A at the rate of 700 km/hour. The
average speed of the aero plane is:
600km per hour 583.33 km per hour
100 √35 km per hour 620 km per hour
Question: 8
If a variable assumes the values 1, 2, 3 ….5 with frequencies as 1, 2, 3 ……5, then what is the AM?
11/3 5
4 4.50
Question: 9
Two variables x and y are given by y = 2x – 3,. If the median of x is 20, what is the median of y?
20 40
37 35
Question: 10
If the relationship between two variables u and v are given by 2u + v 7 = 0 and if the AM of u is 10, then the
AM of v is:
17 -17
-27 27
Question: 11
IF GM of x is 10 and GM of y is 15, then the GM of xy is:
150 Log 10 * log 15
Log 150 None of these
Question: 12
The average donation paid by 10 students for poor students’ welfare fund was Rs.325. if 9 of the students paid
the amount 425, 320, 210, 290, 400, 380, 290, 350 and 275. What amount was paid by the 10th student also find
median and mode of the data:
322.5, 275 334, 5 ,255
321.6, 276 310, 315, 290
Question: 13
The age (in year) of 50 persons range from 20 to 44 years. They are grouped in 5 classes of size 5 years each.
The class frequencies from lowest to highest class are: 6, 15, 19, 6 and 4:
Calculate sum of f𝑥 2 for the above data.
36541 24282.5
48565 None of these
Question: 14
Calculate mean of the above data:
43 | P a g e
25.5 30.7
26.45 28.7
Question: 15
Calculate variance of the above data.
32.21 30.41
26.71 28.81
Question: 16
20% of the persons are above what age?
22.5 41.5
34.5 14.5
Question: 17
The number of articles produced per day (x) and the total cost (c in Rs.) of these articles are related by
C = 50 -0.05x
Mean and variance of the daily production for 5 working days are computed as 100 and 13.5 respectively.
Determine the mean and variance of the daily cost of production.
45, 0.03375 23, 0.04522
22, 0.016875 22, 0.032157
Question: 18
The data relating to the daily wage of 20 workers are shown below:
Rs.50, RS.55, Rs.60, Rs.58, Rs.59, Rs.72, Rs.65, Rs.68, Rs.53, Rs.50, Rs.67, Rs.58, Rs.63, Rs.69, Rs.74, Rs.63,
Rs.61, Rs.57, Rs.62, Rs.64
The employer pays bonus amounting to Rs.100, Rs.200, Rs.300, Rs.400, and Rs.500 to the wage earners in the
wage groups Rs.50 and not more than Rs.55, and not more than Rs.60 and so on and lastly Rs.70 and not more
than Rs.75, during the festive month of October.
What is the average bonus paid per wage earner?
RS.200 Rs.250
Rs.285 Rs.300
Question: 19
For the following incomplete distribution of marks of 100 pupils, median mark is known to be 32.
Marks 0-10 10-20 20-30 30-40 40-50 50-60
No. of 10 ? 25 30 ? 10
students
What is the mean mark?
32 31
31.30 31.50

44 | P a g e
Question: 20
From the following data of weight in ponds of 36 workers in an industry form a frequency distribution table
using 7 class intervals of equal size? Start first class interval from the lowest observation in the data.
130,134,136,142,160,167,124,150,144,142,149,133,133,154,157,146,150,132,120,147,160,146,161,164,141,14
7,136,148,129,145,144,144,147,143,140,135.
Find appropriate class interval of the above data.
4 5
6 7
Question: 21

Sales of product P in May and June were 58 units and 48 units respectively. The arithmetic mean of monthly
sales for the period January to April was 44 units per month. The arithmetic mean of monthly sales for the
period January to June is.

39 units 40 units
47 units 50 units
Question: 22

The arithmetic means of nine positive numbers 7, 2, q, 10, 3q, 8, 3q, 10 and 5 is equal to 7. The value of q is.

3 4
6 7
Question: 23

A sales person travels between four customers who are situated equal distances apart. The journey between
customers 1 and 2 is made at the speed of 30 kph, between customers 2 and 3 at 60 kph, and customers 3 and 4
at 50 kph. What was the mean speed travelled over the total of all journeys?

35kph 42.9kph
46.7 kph 57.1 kph
Question: 24

A share had a market price of Rs.2.50 on 1 January, by 1 July; the share price had increased to Rs.3.60. the
geometric mean of the two share prices is.

Rs.2.47 Rs.3.00
Rs.3.05 Rs.4.50
Question: 25

The geometric mean of two numbers is 10. When a third numbers N is included, the overall geometric mean
remains unaltered at 10. What is the value of N?

0 1
10 900
Question: 26

Question Numbers 12.32_ 12.33 refers to the following data:


45 | P a g e
The mean, median and variance of a frequency distribution are given as:

X = 27.2, x = 24.6, 𝑆 2 = 256.0

Find the coefficient of variation of provided data.

60.25 58.82
75.32 50.25
Question: 27

The blood pressure of Mr. A, is measured daily over several weeks averaged 182 with a standard deviation of
12.6, while that of Mr. averaged 124 with a standard deviation of 9.4. Which patient’s blood pressure is
relatively more variable?

Mr. A Mr.
Both have same Blood pressure Data is incomplete
Question: 28

For a frequency distribution if 𝑋̅ + Mode = 60 and 𝑋̅ – Mode = 10. Find mode for the distribution.

25 5
35 None of these
Question: 29

The variance of a set of data is 9. If every item is doubled then what will be the variance of the resulting data?

9 36
18 45
Question: 30

∑ (x-20) = 0, ∑(𝑥 − 20)2 = 400 find C.V.

0 100
20 2000
Question: 31

If the S.D of x is 3.5 what is the variance of 10-2x?

49 3
-14.5 51
Question: 32

For a data having two observations, S.D = 4 then range is.

8 2
1 None of these
Question: 33

If the arithmetic means of 7, 5, 13, x and 9 be 10, then the value of x is.

10 12
46 | P a g e
14 16
Question: 34

For a data having: mean = 87 Median=92 S.D = 8 Find skewness:

-0.625 -1.875
21.125 75.5
Question: 35

Mean of 10 values is 20, whereas 2 incorrect values 13 and 17 were used instead of 18 and 15. What is the
correct mean?

19.7 20
20.3 20.7
Question: 36

The mean of a data is 10 and coefficient of variation is 20%, while the coefficient of skewness is -3. What is the
median of the data?

12 13
14 Cannot be determined
Question: 37

The average wage of 100 workers of two shifts is Rs.284. the average wage of 70 workers of the first shift is
Rs.290. what is the ratio of average wage of the workers of second shift to that of average wage of total
workers?

145:142 142:145
142:135 135:142
Question: 38

Mean of 5 observations is 8. Out of them, three observations are 2, 6, 11. In remaining two observations, one is
twice the other. Find the least of the remaining two observations.

g 14
6 12
Question: 39

The average runs of a cricketer in four innings is 32. How many runs he should score in the fifth innings so that
his average score becomes 50 runs?

132 122
128 126

47 | P a g e
Points to Remember
1- Statistical measure which defines the central of set of data are called “Measure of central tendency
2- Statistical measures which provide a measure of variability among the observations are called “Measure
of Dispersion”
3- The ratio of the frequency of a given class to the cross-ponding class interval size is called frequency
density i-e f/h
4- The ratio of the frequency of the given class to the total frequency is called relative frequency i-e f/∑f
5- Any process that generates a set of data is called a statistical experiment
6- If each observation is decreased by 2, the mean of new set of observation will decrease by 2
7- If each observation is increased by 5, the mean of the new set of observation is increased by 5 times.
8- If each observation is multiplied by 5, the mean of new set of observation will increase by 5 times
9- If each observation is divided by 10, the mean of new set of observation will decrease by 10 times
10- If some constant is added to each observation of a given set of data, then variance will remain
unchanged
11- If some constant is subtracted from each observation of given set of data variance will remain
unchanged
12- If each observation of the given set of data is multiplied by a constant, then variance will increase by (2
times)
13- If each observation of the given set of data is divided by a constant, then the variance will decrease by 2
times
14- The real disadvantages of the mean are that it may be affected by extreme value
15- The median is not influence by extreme values and give a true average.
16- It is the only average that can be used for qualitative as well as quantitative data
17- A distribution that lacks symmetry with respect of vertical axis is said to be skewed
18- For a bell-shaped distribution, the interval
(i) (Mean ±S, contain 68.27% of observation)
(ii) (Mean ±2S, contain 94.44% of observation)
(iii) (Mean ±3S, contain 99.74% of observation)
19- Mean deviation (M.D) is not affected by extreme observation
20- If any observation is added, subtracted, multiplied and divided by any number then coefficient of
variation remains unchanged
21- Coefficient of variation is a relative measure of dispersion
22- Absolute dispersion is measured in units of data and square units of data
23- When a distribution is uni modal and symmetric the highest point on the curve is
Mean=Median =Mode
24- If the mean is less than mode, the distribution is called negatively skewed
25- Empirical relationship between mean, median and mode is
Mode= 3 Median -2 Mean
26- A.M cannot be determined by graph
27- Median is a positional average
28- Quartile deviation is a measure of dispersion which is consider as the best general purpose measure of
dispersion
29- Coefficient of variation is independent of units of data
30- Coefficient of variation measure the variation of two or more than two series
31- The standard deviation is independent of change of origin
32- The square root of standard deviation of a distribution is called variance
33- The variance of a constant is equal to zero
34- For two numbers standard deviation is always half of the range i-e
S.D = Range /2

48 | P a g e
CHAPTER 12: STATISTICAL MEASURES OF DATA
1-Any numerical value describing a characteristic of a sample is called:

A) Sample mean B) Sample variance


C) Statistic D) None of these
2- The value of a statistic is used to estimate the:

A) Size of the population B) Probability


C) Population parameter D) None of these
3- If each observation is decreased by 2, the mean of new set of observations will:

A) Increase by 2 B) Decrease by 2
C) Remain unchanged D) None of these
4- If each observation is increased by 5, the mean of new set of observations will:

A) Increase by 5 B) Decease by 5
C) Remain unchanged D) None of these
5- If each observation is multiplied by 5, the mean of new set of observations will:

A) Increase by 5 times B) Divided by 5 times


C) Remain unaltered D) None of these
6- If each observation is divided by 10, the mean of new set of observations will:

A) Decrease by 10 times B) increase by 10 times


C) Remain unchanged D) None of these
7- The middle most value of arranged set of observations is known as:

A) Mode B) Median
C) Mean D) None of these
8- “As a result of recent nuclear bomb explosion, we can expect that prices of all commodities will go up.” The
above statement is:

A) Descriptive statistics B) Inferential statistics


C) Not a statistical statement D) None of these
9 -“At least 10% of firing incidents reported by the police were fictitious.” The above statement is:

A) Descriptive statistics B) Inferential statistics


C) Not a statistical statement D) None of these
10- “Of all patients who received this particular type of drug at a local clinic, 60% later developed significant
side effects.” The above statement is:

A) Descriptive statistics B) Inferential statistics


C) Not a statistical statement D) None of these
11- “As a result of a recent conflict, most people are in favor of producing nuclear weapons.” The above
statement is:

A) Descriptive statistics B) Inferential statistics


C) Not a statistical statement D) None of these

49 | P a g e
12- “The survey of consumers indicate that more than 50% consumers use brand A detergent.” The above
statement is:

A) Descriptive statistics B) Inferential statistics


B) Not a statistical statement C) None of these
13- The unit of measurement of coefficient of variation of speed of balls bowled by a bowler is:

A) Feet per second B) Yards per second


C) ) Neither (a) nor (b) D) Both (a) as well as (b)
14- The average preferred, to such data as rates of change in ratios, economic index numbers, population sizes
over consecutive time periods, is:

A) Arithmetic mean B) Geometric mean


C) Harmonic mean D) None of these
15- The value which occurs most or the value with the greatest frequency is called:

A) Mode B) 𝑄3
C) 𝑄1 D) None of these
16- The real ____________ of the mean is that it may be affected by extreme values:

A) Advantage B) Disadvantage
C) Choice D) None of these
17- The median is _______ by extreme values and gives a truer average:

A) Not influenced B) Influenced


C) Affected D) None of these
18- It is the only average that can be used for qualitative as well as quantitative data:

A) Mean B) Mode
C) Median D) None of these
19- The sum of deviations from a mean is always:

A) Zero B) Less than mean


C) Greater than mean D) None of these
20- If Σ(𝑥 − 7) = 0, then 𝑥̅ = _________ .

A) 8 B) 7
C) 5 D) None of these
21- The sum of square of deviations from mean is always:

A) More than zero B) Zero


C) Less than zero D) Less than Mean
22- If for two observations, deviations from mean are -3 and +3 respectively, then the variance is:

A) 0 B) Does not exist


C) 9 D) None of these

50 | P a g e
23- The positive square root of variance is called:

A) Mean deviation B) Standard deviation


C) Mean deviation from mean D) None of these

24-The co-efficient of variation of 3 numbers i.e. x, x+4 and x+11 is y. If x is increased, the value of y would:

A) Decrease B) Increase with the same amount


C) Increase in the same ratio D) Remain the same

25-The distribution is used when:

A) Size of sample is less than 40 B) Size of sample is less than 30


C) Mean is less than 40 D) Mean is less than 30
26-A z-score measures how many standard deviations an observation is:

A) Above the mean B) Below the mean


C) Above or below the mean D) None of these
27-If z=-2 then it is correct to say that:

A) Observation is less than mean B) Observation is less than standard deviation


C) Observation is more than standard deviation D) None of these
28-It is possible to compare two observations measured in completely different units by z-score because:

A) z-score has its own units B) z-score is a unit less quantity


29-The quantity which expresses the standard deviation as a percentage of mean is called:

A) z-score B) Co-efficient of variation


C) Co-efficient of dispersion D) None of these
30-A distribution that lacks symmetry with respect to a vertical axis is said to be:

A) Normal B) Probability distribution


C) Skewed D) None of these
31-In a given distribution if mean is less than median, the distribution is said to be:

A) Symmetrical B) Positively skewed


C) Negatively skewed D) None of these
32-In a given distribution if mean is greater than median, the distribution is said to be:

A) Symmetrical B) Positively skewed


C) Negatively skewed D) None of these
33-For the observations 7, 7, 7, mean and variance are respectively:

A) 0 and 7 B) 7 and 0
C) 7 and 49 D) None of these
34- If some constant is added to each observation of a given set of data then the variance will:

A) Increase by that constant B) Decrease by that constant

51 | P a g e
C) Remain unchanged D) None of these
35- If some constant is subtracted from each observation of the given set of data, variance will:

A) Decrease by the that constant B) Increase by that constant


C) Remain unchanged D) None of these
36- If each observation of the given set of data is multiplied by a constant x, then the variance will:

A) Increase by x times B) Decrease by x times


C) Increase by x2 times D) None of these
37- If each observation of the given set of data is divided by a constant x, then the variance will:

A) Decrease by x times B) Increase by x times


C) Decrease by x2 times D) None of these

38- If the variance of a given set of observations is 81, the standard deviation is:

A) ±9 B) +9
C) -9 D) None of these
39- Which of the following is likely to produce a bell shaped curve:

A) Ages of people of Pakistan B) Incomes of people of Pakistan


C) Heights of students in a class D) None of these
40- For a bell-shaped distribution the interval 𝑥̅ ± 𝑠 approximately contains:

A) 75% of observations B) 68% of observations


C) 95% of observations D) None of these
41- For a bell shaped distribution the interval 𝑥̅ ± 2𝑠 approximately contains:

A) 89% of observations B) 95% of observations


C) 99.7% of observations D) None of these
42- For a bell shaped distribution the interval 𝑥̅ ± 3𝑠 approximately contains:

A) 89% of observations B) 95% of observations


C) 99.7% of observations D) None of these
43- The 50th percentile, fifth decile, second quartile and median of a distribution are:

A) Unequal B) Equal
C) Approximately equal D) None of these
44- The measure of variation which measures the length of interval that contains the middle 50% of data is
known as:

A) Quartile-deviation B) Semi-inter quartile range


C) Inter-quartile range D) None of these
45- Any process that generates a set of data is called:

A) An experiment B) A random process


C) A statistical experiment D) None of these

52 | P a g e
46- Measures of central tendency for a given set of observation measures:
A) The scatternets of the observations B) The central location of the observations
C) Both A & B D) None of these
47- While computing the AM from a grouped frequency distribution, we assume that:

A) The classes of equal length B) the classes have equal frequency


C) All the values of a class are equal to the D) All of these
mid-value of that class
48- Which of the statement is not true?

A) Mean is rigidly defined B) Mean is not affected due to sampling


fluctuations
C) Mean has some mathematical properties D) All of these

49- Which of the following statement is true?

A) Usually mean is the best of central B) Usually, median is the best measure of
tendency central tendency
C) Usually, mode is the best measure of central D) Normally, GM is the best measure of
tendency central tendency
50- For open-end classification, which of the following is the best measure of central tendency?

A) AM B) GM
C) Median D) Mode
51- The presence of extreme observations does not affect?

A) AM B) Mode
C) Median D) Any of these
52- In case of an even number of observations which of the following is median?

A) Any of the two middle-most value B) The simple average of these two middle
values
C) The weighted average of these two middle D) Any of these
values
53- The most commonly used measure of central tendency is:

A) AM B) Mode
C) Median D) All of these
54- Which of the following is not uniquely defined?

A) Mean B) Mode
C) Median D) All of these
55- Which of the following measure of the central tendency is difficult to compute?

A) Mean B) Mode
C) Median D) GM

53 | P a g e
56- Which measure(s) of central tendency is(are) considered for finding the average rates?

A) AM B) GM
C) HM D) Both A & B
57- For a moderately skewed distribution, which of the following relationship holds?

A) Mean- Mode = 3(Mean- Median) B) Median- Mode = 3 (Mean - Median)


C) Mean – Median = 3 (Mean- Mode) D) Mean – Median = 3 (Median- Mode)
58- Weighted averages are considered when:
A) The data are not classified B) The data are put in the form of grouped
frequency distribution
C) All the observations are no of equal D) Both A & B
importance
59- Which of the following results hold for a set of distinct positive observations?
A) AM ≥ GM ≥ HM B) HM ≥GM ≥ AM
C) AM > GM > HM D) GM > AM > HM
60- When a firm register both profits and losses, which of the following measure of central tendency cannot be
considered?

A) AM B) GM
C) Median D) Mode
61- Quartiles can be determined graphically using?

A) Histogram B) Frequency polygon


C) Ogive D) Pie Chart

62- Which of the following measure(s) possesses (possess) mathematical properties?

A) AM B) GM
C) HM D) All of these
63- Which of the following measure(s) satisfies (satisfy) a linear relationship between two variables?

A) Mean B) Median
C) Mode D) All of these
64- Which of the following measures of central tendency is based on only fifty percent of the central values?

A) Mean B) Median
C) Mode D) Both A & B
65- Which of the following is positional average?

A) A.M B) G.M
C) H.M D) Median
66- Which of the following cannot be determined by graph?

A) A.M B) Median
C) Mode D) Quartiles

54 | P a g e
67- The probability of a random value chosen being larger than its median value for a population of 15 values
is:

A) 0.5 B) 0.15
C) 0.16 D) 0.34
68- Which of the following is a empirical relationship between mean, median, mode?

A) Mean – Median = 1/3 (Mean – Mode) B) Mode = 3 Median – 2 Mean


C) Median = Mode + 2/3 (Mean - Mode) D) All of these
69- Which of the following measures satisfy linear relationship between two variables?

A) A.M B) G.M
C) H.M D) All of these
70- If an organization wants to give more importance to marks obtained in personal interview than the marks
obtained in written test then the best method is:

A) Simple average B) Weighted average


C) Combined average D) None of these
71- 55% of the observation are more than a value ‘A’ in a given data. What is ‘A’ :

A) P45 B) P55
C) 0.55 D) None of these
72- Which of these averages cannot be less than zero?

A) A.M B) Geometric mean


C) Harmonic mean D) Median
73- When calculating the average growth rate of population for a country, the correct average to use it:

A) A.M B) Median
C) H.M D) G.M
74- When a median is uni-modeD and symmetric the highest point on the curve is called:

A) Mean B) Median
C) Mode D) All of these
75- If mean is less than mode, the distribution is:

A) Positively skewed B) Negatively skewed


C) Symmetrical D) None of these
76- The arithmetic mean is affected by the change of:

A) Origin B) Scale
C) Both origin and scale D) All of these
77- The best central value of an average data is:

A) A.M B) G.M
C) Median D) Mode

55 | P a g e
78- The geometric mean of two numbers “a” and “b” is:

A) ab B) a + b/2
C) √𝒂𝒃 D) √𝑎 + 𝑏
79- The suitable average for qualitative data is:

A) Mean B) Median
C) Geometric mean D) Weighted mean
80- The ratio among the number of values and sum of reciprocal values is called:

A) G.M B) A.M
C) Mode D) H.M
81- Which measure of dispersion is not affected by extreme observations?

A) Range B) Q.D
C) M.D D) S.D
82- Root- mean squared deviation from mean is:

A) Range B) Quartile deviation


C) Mean deviation D) Standard deviation
83- For the two numbers S.D is always:
A) Half the range B) Double the range
C) Square the range D) Equal to the range
84- If all the observation are multiplied by 3:

A) C.V is also three times the previous C.V B) S.D remains same
C) C.V remains unchanged D) None of these
85- Which of the following is relative measure of dispersion?

A) Standard Deviation B) Mean deviation


C) Co- efficient of variation D) variance
86- The square of standard deviation of distribution is:

A) Co-efficient of variation B) Variance


C) Quartile deviation D) None of these
87- The variation of a constant is equal to:

A) Constant B) Zero
C) Unity D) None of these
88- The standard deviation is independent of:

A) Change of origin B) Change of scale


C) Change of origin and scale D) All of these
88- Which of the following measures of dispersion is independent of units of data?

A) Standard deviation B) Quartile deviation


C) Variance D) Co- efficient of variation

56 | P a g e
89- Which measures of dispersion is considered as the nest general purpose measure of dispersion?

A) range B) Quartile deviation


C) Quartile deviation D) Mean deviation
90- In a symmetric distribution, the co-efficient of skewed will be:

A) 1 B) 0
C) 1 D) More than one
91- To compare the variation of two or more than two series we use:
A) Mean absolute deviation B) Variance
C) Co-efficient of variance D) Range
92- Which of the following statement is true?
A) The standard deviation of a constant is constant B) Variance is the square root of standard deviation
C) Standard deviation is the mean of squared D) Standard deviation is the positive square root of
deviations from mean mean squared deviations from mean
93-The distribution is positively skewed if:
A) Mean < Mode B) Mean > Mode
C) Mean > Median D) Both B & C
94- Absolute dispersion is measured in:
A) Units of data B) Square units of data
C) Both A & B D) Without units of data
95- Standard deviation is greater than:
A) Mean deviation B) Quartile deviation
C) Both A & B D) Range
96- Sum of absolute deviation from mean is:
A) Zero B) Positive
C) Negative D) All of these
97- Which of the following statement is not true?
A) Some data sets do not have means B) calculation of mean is affected by extreme data
values
C) A weighted mean should be used when it is D) All these statements are correct
necessary to take the importance of each value into
account
98- Which of the following is the first step in calculating the median of a data set?
A) Average the middle two values of the data set B) Array the data
C) Determine the relative weights of the data values D) None of these
in, terms of importance
99- When referring to a curve that tails off to the left end, you would call it:
A) Symmetrical B) Skewed right
C) Positively skewed D) None of these
100- Which of the following is an example of a relative measure of dispersion?
A) Standard deviation B) Variance
57 | P a g e
C) Co-efficient of variation D) All of these
101- Which of the following is true?
A) The variance can be calculated for grouped or B) The standard deviation can be calculated for
ungrouped data grouped or ungrouped data
C) The standard deviation can be calculated for D) A & B, but not C
grouped data but the variance can be calculated only
for grouped data
102- If one were to divide the standard deviation of a population by the mean of the same population and
multiply this value by 100, one would have calculated the:
A) Population standard score B) Population variance
C) Population standard variation D) population co-efficient of variation
103- The square of the variance of a distribution is the:
A) Standard deviation B) Absolute deviation
C) Co-efficient of variation D) None of these
103-Which of the following statement is correct relationship among Arithmetic, Geometric and Harmonic
means:
A) A.M > G.M > H.M B) A.M < G.M < H.M
C) 𝐴. 𝑀 ≥ 𝐺𝑀 ≥ 𝐻. 𝑀 D) A.M≤ 𝐺. 𝑀 ≤ 𝐻. 𝑀
104- An attribute is a:
A) Quantitative characteristic B) Measurable characteristic
C) Qualitative characteristic D) All of these
105- A variable which can be assume any value between two given values is called:
A) Discrete variable B) Random variable
C) Continuous variable D) Both A & B
106- If each item of the same data or observation is decreased by 50%, then the variance will:
A) Remain same B) decrease by 25%
C) decrease by 50% D) decrease by 75%
107- co-efficient of variation is a relative measure of:
A) Mean B) Deviation
C) Range D) Dispersion
108- Measures of central tendency for a given set of observations:
A) Summarize the data into a single value B) Tends to lie the centre of the data
C) Measures the scatterness of the data D) Both A & B
109- What is the major assumption that one makes, when computing a mean from a grouped data?
A) All values are discrete B) No value occurs more than once
C) every value in a class is equal to mid-value D) each class contains same number of values
110- Which of the following is affected by extreme observations in a grouped data?
A) Mean B) Median
C) Mode D) Quartile

58 | P a g e
111- Which of the following cannot always be determined from a grouped data?
A) Mean B) Median
C) Mode D) Quartile
112- Which of the following cannot be determined by graphical method?
A) Mean B) Median
C) Mode D) Quartile
113- Which of the following is not an advantage of median?
A) It is particularly useful in open end classes B) It is not influenced by extreme values
C) It is the most appropriate average in dealing with D) It is not determined by each and every
qualitative data observation
114- Which of the following is true for A.M?
A) The sum of devotions of the observation from A.M B) The sum of deviations of the observations from
(ignoring signs) is minimum A.M (taking signs into account) is minimum
C) The sum of deviations of the observations from D) The sum of deviations of the observations from
A.M (ignoring signs) is zero A.M (taking sings into account) is zero
115- The sum of the squares of deviations of deviations has the smallest value, when the deviations are taken
from their?
A) Mode B) A.M
C) Median D) H.M
116- Which of the following statement is true?
A) The sum of deviations taken median is zero B) The sum of squares of deviation taken from median
is minimum
C) The sum of absolute deviation taken from D) None of these
median is minimum
117- The mean, median and mode of a data are of a single point. The data will be:
A) Symmetrical B) positively skewed
C) Negatively skewed D) Open- ended
118- Which of the following measure is considered to have sampling stability the most?
A) G.M B) A.M
C) Median D) Mode
119- If every item in the data set is replaced by its A.M then the sum of these substitutions will be:
A) Greater than the sum of the individual observations B) Less than the sum of the individual observations
C) Equal to the sum of the individual observations D) Always positive
120- When a company registers both profit and loss, which of the following measure of central tendency cannot
be registered?
A) A.M B) G.M`
C) Median D) Mode
121- Which is the most popular and widely used measure of central tendency?
A) Median B) G.M
C) A.M D) Mode

59 | P a g e
122- A good measure of Central tendency should be:
A) Capable of further algebraic treatment B) Based on all observations
C) Rigidly defined D) All of these
123- Deciles divides the given data set into-equal parts:
A) 10 B) 4
C) 100 D) 2
124- Which of the best measure of finding average rate of increase or decrease?
A) G.M B) Mean
C) Mode D) Median
125- Which of the following results hold for a set of distinct positive observations?
A) 𝐴. 𝑀 ≥ 𝐺. 𝑀 ≥ 𝐻. 𝑀 B) 𝐻. 𝑀 ≥ 𝐺. 𝑀 ≥ 𝐴. 𝑀
C) 𝑨. 𝑴 > 𝑮. 𝑴 > 𝑯. 𝑴 D) G.M>A.M>H.M
126- Median can be determined graphically using:
A) Histogram B) Ogive curves
C) Pie chart D) Frequency polygon
127- The reciprocal of A.M of reciprocal of observations in an ungrouped data is:
A) H.M B) A.M
C) G.M D) Weighted mean
128- What % of observations lies between P20 and P78?
A) 20 B) 78
C) 58 D) None of these
129- The appropriate measure of dispersion for open end classification is:
A) Range B) Quartile deviation
C) Mean deviation D) Standard deviation
130- When the variability of two series is to be compared, which of the following should be used?
A) Absolute measure of average B) Absolute measure of dispersion
C) Relative measure of dispersion D) Relative measure of average
131- Which measure of dispersion is used to find pooled measure of dispersion after combining several
groups?
A) Range B) Q.D
C) M. D D) S.D
132- A shift scale has no impact on:
A) Range B) Q.D
C) S.D D) C.V
133- A salesman whose figures are most consistent, will have the:
A) Largest average sales per year B) Maximum variance in his sales figures
C) Maximum co-efficient of variation D) Minimum co-efficient of variation
134- Which of the following is true, if there is no dispersion in a data set?
A) All the mathematical averages are equal but the B) All the mathematical and positional averages
positional averages are not equal are equal
60 | P a g e
C) All the positional averages are equal but the D) All mathematical averages are zero
mathematical averages are not equal
135- If all the observations in a data is decreased by the same quantity, then which of the following is true?
A) Range decreases by same quantity B) Standard deviation remains unchanged
C) Co-efficient of variations increases D) Both B & C
136-. The unit of measurement of coefficient of variation of speed of balls bowled by a bowler is:

(a)Feet per second (b)Yards per second


(c)Neither(a) nor (b) (d)Both (a) as well as (b)
137- A family travels 500 Kms each day for 3 days. The family averages 80 Kms per hour the first day,93
Kms per hour the second day,87 Kms per the third day. The average speed for the entire trip is :

(a)87Kms/Hr (b)85 Kms/Hr


(c)86.7Kms/Hr (d)None of these

138- The average preferred, to such data as rates of change in ratios, economic index numbers, population sizes
over consecutive time period are:

(a)Arithmetic mean (b)Geometric mean


(c)Harmonic mean (d)None of these

139- Over a period of 4 years, an employee’s salary has increased in the ratios,1.072,1.086,1.069 and 1.098.
The average of these ratios and hence the average percent increase are:

(a)1.08125 and 8.125% (b)1.086 and 8.6%


(c)1.08119 and 8.6% (d)None of these

140- Median of 82,86,93,92,79 is:

(a)86 (b)93
93+92 (d)None of these
(c ) 2

141- Median of 2.5,3.6,3.1,4.3,2.9,2.3 is:

(a)Dose no exist (b)2.9


(c)3 (d)None of these

142- The value which occurs most or the values with the greatest frequency is called:

(a)Mode (b)Q
(c)Q (d)None of these

143- Mode of 2.5,3.6,3.1,4.3,2.3 is :

61 | P a g e
(a)4.3 3.1+4.3
(b) 2
(c)Does not exist (d)None of these
144- The real ______ of the mean is that it may be affected by extreme values.

(a)Advantage (b)Disadvantage
(c)Choice (d)None of these

145- The median is _____ by extreme values and gives a truer average.

(a)Not influenced (b)Influenced


(c)Affected (d)None of these

146- It is the only average that can be used for qualitative as well as quantitative data:

(a)Mean (b)Mode
(c)Median (d)None of these

147- A car averages 20 kilometres per litre on the highway. How many litres are required to complete 300
Kilometre trip?

(a)60 (b)25
(c)15 (d)None of these

148- A person invests Rs. 5,000/= at 10.5% interest, Rs. 6,300/ = at 10.8% and Rs. 4,500 at 11%. What is the
average percentage return to the saving?

(a)10.762% (b)10.8%
(c)10.766% (d)10.666%

149- What is the average for a student who received grades of 85, 76 and 82 on 3 mid term test and a 79 on the
final examination, if the final examination counts 3 times as much as each of the three tests?

(a)81 (b)80
(c)80.5 (d)None of these

150- Three sections of a statics class containing 28,32 and 35 students averaged 83, 80 and 76 respectively.
What is the average of all 3 sections?

(a)79.41 (b)79.67
(c)80 (d)None of these

62 | P a g e
151- The sum of deviations from a mean is always:

(a)Zero (b)Less then mean


(c)Greater than mean (d)None of these

152- if ∑ (x – 7) = 0, then x = ______ .

(a)8 (b)7
(c)5 (d)None of these

153- The sum of square of deviations from mean is always:

(a)More than zero (b)Zero


(c)Less than zero (d)Less than mean

154- The average daily sale and the related standard deviation of Ali, Atif, Ahmed, and Azeem in thousands of
Rupees are 41 & 4.3, 36 & 3.3, 26 & 2.9 and 24 & 2.5 respectively, then the most inconsistent among them is :

(a)Ali (b)Atif
(c)Ahmed (d)Azeem

155- if for two observations, deviations from mean are -3 and +3 respectively, then the variance is :

(a)0 (b)Does not exist


(c)9 (d)None of these

156- The positive square root of variance is called:

(a)Mean deviation (b)Standard deviation


(c)Mean deviation from mean (d)None of these

157- The co-efficient of variation of 3 numbers i-e x, x + 4 and x + 11 is y. if x is increased, the value of y
would:

(a)Decrease (b)Increase with the same amount


(c)Increase in the same ratio (d)Remain the same

158- For the observations 7,7,7, mean and variance are respectively:

(a)0 and 7 (b)7 and 0


(c)7 and 49 (d)None of these

63 | P a g e
159- If the variance of 2,5,7,10 and 14 is 17.o4 then the variance of 5,8,10,13, and 17 is:

(a)17.04 (b)20.04
(c)51.12 (d)None of these

160- If the variance of 2,5,7,10 and 14 is 17.o4 then the variance of 6,15,21,30, and 42 is:

(a)17.04 (b)51.12
(c)153.36 (d)None of these

161- If some constant is added to each observation of a given set of data then the variance will:

(a)Increase by that constant (b)Decrease by that constant


(c)Remain unchanged (d)None of these

162- If some constant is subtracted from each observation of the given set of data, variance will:

(a)Decrease by the that constant (b)Increase by that constant


(c)Remain unchanged (d)None of these

163- If each observation of the given set of data is multiplied by a constant x, then the variance will:

(a)Increase by x times (b)Decrease by x times


(c)Increase by x2 times (d)None of these

164- If each observation of the given set of data is divided by a constant x, then the variance will:

(a)Decrease by x times (b)Increase by x times


(c)Decrease by x2 times (d)None of these

165- For the values -2,-5,-7,-10 and -14 the variance is:

(a)-17.04 (b)+17.04
(c)-8.54 (d)None of these

166- If the variance of a given set of observation is 81, the standard deviation is:

(a)±9 (b)+9
(c)-9 (d)None of these

167- The 50th percentile, fifth decile, second quartile and median of a distribution is:

(a)Unequal (b)Equal
(c)Approximately equal (d)None of these
64 | P a g e
168- The measure of variation which measures the length of interval that contains the middle 50% of data is
known as:

(a)Quartile-deviation (b)Semi-inter quartile range


© inter quartile range (d)None of these

169- Statistical measures which define the center of a set of data are called:

(a)Median (b)Measure of data


(c)measures of central (d)None of these
tendency

170- statistical measures which provide a measure of variability among the observation are called:

(a)Quartile (b)Measures of dispersion


(c)Co-efficient of variation (d)None of these

171- Arithmetic mean for a set of data with six observations was calculated as 30, however during data input
an observation valuing 31 was considered 13 by mistake. Compute correct value of mean.

(a)30 (b)33
(c)36 (d)39

172- Which of the following is/ are correct?

(1) The formula of arithmetic mean is: sum of (2) The algebraic sum of the deviation of the
the observation / total number of observations. observation from their mean is always zero

(3) Harmonic mean is not used in averaging 4) None of these


speeds.

173- Select one or more of the possible correct answers:

(1) Mode is that value which divides the sets of (2) Sum of squares of deviations of observation
observation into two equal parts. from their mean is minimum

174- Select one or more of the possible correct answers:

(1) Coefficient of variation is independent of (2) Coefficient of variation is an absolute


units and expressed in percentages measure of dispersion.

(3) Coefficient of variation is a relative (4) coefficient of variation cannot be


measure of dispersion. calculated when standard deviation and mean are

65 | P a g e
available

175- Select one or of the possible correct answer:

(1) Variance and standard deviation cannot (2) Variance and standard deviation cannot be
be negative. zero.

(3) Variance and standard deviation can be zero. (4) Variance and standard deviation do not change if
each value in data is multiplied by a constant

176- Select one or more of the possible correct answers:

(1) Range is the difference of highest and (2) Standard deviation is the positive
lowest observation in the data. square root of variance.

(3) Arithmetic mean of 7 and 9 is 8.5 (4) None of these

177- Select one or more of the possible correct answers:

(1) The weighted arithmetic mean is (2) The geometric mean is useful to work
computed by taking into account the relative out the average growth rate over several years.
importance of each item.

(3) Coefficient of variation is an absolute (4) None of these


measure of dispersion.

178- For the following data compute mean

Number of goals Number of matches (frequency)

0 5

1 3

2 2

3 1

(a)2.1 (b)0.81
(c)1.1 (d)0.91

179- For the following data compute standard deviation

Number of goals Number of matches (frequency)

66 | P a g e
0 5

1 3

2 2

3 1

(a)0.9959 (b)0.9121
(c)0.8812 (d)0.9359

180- A person wants to calculate average growth rate in his property value over past few years. which method
of average is suitable?

(a)Arithmetic mean (b) Geometric mean


(c)Harmonic mean (d)None of these

181- Compute coefficient of variation for following data:

Mean:80

Standard deviation: 20

(a)400% (b)20%
(c)80% (d)25%

182- Arithmetic mean for a set of data with four observation is 50. Which of the following cannot be a possible
value of observation?

(a)190 (b)200
(c)50 (d)250

183- Arithmetic mean for a set of data with four observation is 50. What can be the maximum possible value a
single observation?

(a)200 (b)50
(c)250 (d)400

184- Arithmetic mean of three observation is 12. The observation is such that they have a common difference
of 2 between them. Compute median value.

(a)12 (b)10
(c)14 (d)36

67 | P a g e
185- Arithmetic mean of three observation is 15. The observations are such that they have a common
difference of 3 between them. If Unity is subtracted from mid value of these three observations find the mean
after changes

(a)15 (b)14.67
(c)16 (d)18

186- If the standard deviation of a and b, two observation is 2.5 then the standard deviation of a+3 and b+3 is:

(a)2.5 (b) 5.5


(c)0 (d)Cannot be determined with limited information

187- If the standard deviation of a and b, two observation is 2.5 then the standard deviation of 3a and 3b is:

(a)5.5 (b)7.5
(c)0 (d)Cannot be determined with limited information

188- A combined mean is simply a _____, where the weights are the size of each group.

(a) Weighted (b)Geometric


(c)Harmonic (d)None of the options are the valid
189- A class of 20 students had mean marks 25. If there are 18 boys in the class with mean marks of 20. what
can be the highest possible marks of a single girl student?

(a)140 (b)150
(c)20 (d)70

190- A class of 20 students had mean marks of 25. If there are 18 boys in the class with mean marks of 20.
Compute total marks scored by the two girls.

(a)140 (b)70
(c)150 (d)180

68 | P a g e
EXTRA PRACTICE QUESTION
Question:1 define raw data

The data collected in large volumes Result of sampling enquires or census

The data collected in surveys All of these

Question-2 Average

a) Summarize the concentration of a set of data b) Measures the concentration of a set of

c) Measures the scatternets of a set of data d) None of these

Question-3 If classification is done according to differences in time, classification is called:

a) Qualitative b) Spatial

c) Quantitative d) Temporal

Question-4 The model letter of the word "STATISTICS" is:

a) S b) T

c) Both a and b d) None of these

Question-5 Following data s given find mean and SD

36,26,19,5,4

a) b)

c) Mean-18, SD=12.28 d)

Question-6 Following data is given find coefficient of variation. 36,26,19,5,4

a) 146.58% b)

C) d) 68.22%

Question- 7 Find ∑ 𝑓 and median?

Stem Leaf

2 3,4,6

3 7,8,9,9

4 7,6,5,5,5,2,2,6,6,9

5 7,7,8,9,9

69 | P a g e
6. 1,2,3,6

a. 26,46 b. 27,39 c. 39,37.5 d.46,27

Question-8 Find median of values 30,15,20

a) 30 b) 15

c) 20 d) None of these

Question-9 Relation between two qualitative

variables is

a) Rate b) Ratio

c) Proportion d) All of these

Note: Rate compares two quantities of different units. Eg km/hr, miles/sec,

Ratio compares two quantities of same units eg men to women ratio

Question-10 Consider the following stem and leaf display and choose the wrong statement

Stem Leaf

1) 0,2.

2)1.3.5

3)0,5,

4)1,2,3,5

5) 0,2

6)0,8

Its mean is 58, It has median 71, Its range is 68

a) Statement 1 only b) Statement 2

c)Statement 1,2,3 d)Statement 2 and 3

Question-11 Find lower and upper quartile from the following data

19,19,25,25,25,20,21.21.28

a) Q1=19.5 & 03=25 b) Q1 = 19 and Q3 = 25.5

c) Q1=19.5 & Q3=25.5 d) Q1 = 19 and Q3 =25

Question-12

70 | P a g e
21.22.21.22.27,22,28.27.27.28.28,28 find Mode

a) 28 b. 27

c. 21 d. 21

Question-13

15.16.23,23.23.25.2 (28,28,28,29,29,29,29 Find Mode

a) 29 b) 27

c)28 d)23

Question- 14 A cricketer score in a cricket series as 51.15,0,3,85,15,51,38. Find mode

a) 51 b) 15

c) 51&15 d) No mode in data

Question-15 A production process consists of consumption of the following material in Kg's:

49, 50, 38, 60, 75 of materials A, B, C, D, E respectively. How many angle difference between C&D in pie chart?

a)25.12% b)29.12%

c) 38.15% None of these

Question- 16 The quantity which expresses the standard deviation as a percentage of mean is Called:

a) Z- b) Co-efficient of variation

score

c) Co-efficient of dispersion d) None of these

Question-17 In positive skewed distribution, the Median is ______ than Mean

a. Less b. More

c. Equal d. None of these

Question 18 A data has Mode=1850 and mean = 1450. You are required to

find whether data is

a) Positively skewed b) Negatively skewed

c) Normal d) None of these

71 | P a g e
Question- 19 The chart which is used to compare relation between whole and its components is

a) Component bar chart b) pie chart

c)Both a and b d) None of these

Question-20

If data one has 50 and data

two has mean y-75 and a

constant 3 is added to both of

these data then their means w

a) Increase by 3 times b) Increase by 3 units

c) Decrease by 3 d) Decrease by 3 units

times

Question-21 What is the definition of raw data?

a) The data collected in surveys b) Data often collected in large volume

c)Result of sampling enquiries or census d) All of these

Question- 22 What is the definition of sampling?

a) a process or method of drawing a b) a process of arranging population


representative group of individuals or

cases from a particular population

c) a process of finding probabilities d) all of these

Question- 23 The graph obtained by joining the mid points of the tops of adjacent rectangles in histogram is called?

a) Frequency polygon b) Ogive

c) Pie Chart d) Histogram

Question-24 Which of the following is correct. about median?

a) There can be more than one median in a Data b) Median will be affected by increase or
decrease in extreme values

c) It divides the data in two equal parts in terms d) All of these


of values

72 | P a g e
Question-25 Following data is given. Find lower and upper quartile 19,19,25,25,25,20,21,21,28

a) Q1 = 19 and Q3 = 25 b) Q1=19.5 and 03=253

c) Q1 = 19.5 and Q3 = 25 d) Q1 = 19.5 and Q3= 25.5

Question 26 Ogive is?

a) Cumulative frequency curve b) Frequency distribution curve

c) Both A and B d) None of these

Question 27 The sums of deviations taken from 10, 20, 30 and 40 are-56, -20, 0 and 60 respectively. If distribution is
treated as normal or symmetric, what is mode of the distribution?

a) 20.5 b) 30.0

c) 15.5 d) . more than 30.0

Question 28 Which the of the following is used to represent grouped frequency distribution

a) Pie-chart b) Simple bar chart

c) Histogram d) All of these

Question-29 What is true about mean?

a) Mean is usually a data value b) Mean is unaffected by extreme values

c) Mean is best average to describe data d) All of these

Question-30 Find harmonic mean from the following data

X f

0 - - - -9 2

10 - - - 19 3

20 - - - 29 4

30 - - - 39 3

40 - - - 49 2

a) b)

c) d)

73 | P a g e
Question-31 Mean of 35 values is 68, mean of 15 values is 54, find the mean of remaining 20 values

a) b)

c) d)

Question: 32 Which of the following is correct for Bar graph?

a) It can be used for the continuous distribution b) It can be vertical

c) It can be horizontal d) Both vertical and horizontal

Question-33 Mean of 38 values is 62, mean of 10 values is 57, find the mean of remaining 28 values

a) b)

c) d)

Question-34 10 readings of temperature are given below

a) Variance can't be calculated as all the values b) Variance can be positive or negative
are negative

c) Variance must be at least 0 d) Variance of this data is negative

Question-36 Following data is given

2,10,15,25,18,3.5,16,45,50,25,45,40,.2,40,3,42,45,13,17,18,25,15,22,23,25,39,8,12,25.16.18,40,32,3.5,33,36,29,26,22.

Find mean and median

a) 22 & 22.425 b) 22.425 & 22

c) 21 & 23 d) None of these

Question-37 Salaries of 6 employees are given below you are required to find median

40,000 50,000,55,000,44,000, 25,000, 30,000

a) 44,000 b) 40,000

c)42,000 d) 50,000

Question-38 A student calculated mean and standard deviation of a data of 10 values as 104 and 4 respectively.

Later on it was discovered that he has used some incorrect values as given below in table.

Incorrect values Correct values

6 4

74 | P a g e
8 9

You are required to find correct mean and standard deviation.

a) Mean 103.9 & SD=6.041 b) Mean 113.9 & SD = 6.041

c) Mean 103. & SD=4.04 d) Mean 113. & SD=4.041

Question-39 What will be the shape of distribution?

a) Symmetric b) Positively showed

c) Negatively showed d) None of these

Question-40 What is true about Histogram?

a) Set of rectangles b) Heights are proportionate to frequency

c) Both A and B d) None of these

Question-41 Which of the following statement is not true about continuous variable?

a) Ages of students in a class b) Ages of people in Pakistan

c) Height of all the students in world d) None of these

Question 42 Which of the following is used to represent grouped frequency distribution

a) Pie char b) Histogram

c) Simple Bar char d) All of these

Question-43 What is true about mean?

a) Mean is usually a data value b) Mean is unaffected by extreme values

c) e) Mean is best average to describe data d) All of these

Question-44 Find harmonic mean of 78, 79, 80

a) 70.00 b) 70.09

c) 78.99 d) None of these

Question-45 Which of the following is used to represent grouped frequency distribution?

a) Histogram b) Frequency polygon

c) Ogive d) all of these

Question-46 The frequency distribution of a group of persons according to age is given

75 | P a g e
AGE <1 1-4 5-9 10-19 20-29 30-39 40-59 60-79

PERSONS 5 10 11 12 22 18 8 7

Find mean,

Median

a) b)

c) d)

Question-47 variance can never be

a) 0 b) Negative

c)Positive d) Both a and b

Question-48 Following data is given for two sales department of a multinational company

Department Sales force Revenue SD

A 200 10 million 25000

B 150 36 million 80000

In your opinion which department is more consistent?

a) b)

d) c)

Question-49 Following data is given for two sales department of a multinational company

Department Sales force Revenue SD

A 200 25 million 250000

B 250 36 million 80000

In your opinion which department is more consistent?

a) b)

c) d)

Question-50 Following data is given

2,4,6,8,10,11,12,13,15,17,18,20,22,23,25,26,28,30,32,34,36,38,39,40,42,43,45,47,48,49

You are required to find that how many values fall in the

range of X
76 | P a g e
a) 18 b) 19

c) 17 d) 20

Question- 51 The lower and upper quartile of a normal distribution are 20 and 40 find median

a) 30 b) 32

c) 25 d) Data is incomplete

Question-52 The lower and upper quartile of a normal distribution are 20 and 40 find mode

a) 30 b) 32

c) 25 d) Data is incomplete

Question- 53 The lower quartile and median of a normal distribution are 20 and 40 find upper quartile

a) 60 b) 50

c) 30 d) Data is incomplete

Question-54 The mean and upper quartile of a normal distribution are 20 and 30 find lower quartile

a) 10 b) 50

c) 30 d) Data is incomplete

Question-55 For a data set, median is 49.27 and QI and Q2 are 37.21, 61.33 respectively. Determine if they are:

a) Positively Skewed b) Negatively Skewed

c) Symmetric d) None of these

Question-56 If frequency is negatively skewed show the relation between mean median and mode.

a) Mean>Median>Mode b) Mean<Median<Mode

c) Mean Median-Mode d) All of these are true.

Question-57 Performance of two randomly selected players is given bellow:

Player A (runs/inning) 85 12 75 43 9 49

Player B (wickets/ 4 3 1 1 0 2
inning)

Performance of which player is more consistent?

a) Player A b) Player B

c) Both are same d)Insufficient data

77 | P a g e
Question-58 If frequency distribution is skewed to left tail (negatively skewed) then show the relation mean, median and
mode

Mean=median=mode b) Mean<median<mode

c) Mean>median>mode d) median <Mean<mode

Question-59 If frequency distribution is skewed to right tail (positively skewed) then show the relation between mean,
median and mode

a) Mean=median=mode b) Mean<median<mode

c) Mean>median>mode d) median <Mean<mode

Question-60 If frequency distribution is symmetrical then show the relation between mean, median and mode

a) Mean=median=mode b) Mean<median<mode

c) Mean>median>mode d) median <Mean<mode

Question- 61 Data given

Classes Frequency

46-50 15

51-55 25

56-60 105

61-65 170

66-70 180

71-75 150

76-80 80

81-85 60

86-90 48

Find Standard deviation and variance

a)9.17 and69.00 b) 9.17 and 84.02

c)3.03 and 9.17 d) 84.02 and 9.17

78 | P a g e
Question- 62 Find H.M.

CLASS f

0-9 2

10-19 3

20-29 4

30-39 3

40-49 2

a) b)

c) d)

Question-63 Data given

Find mean, median and mode

X F fx C.f

4 8 32 8

5 9 45 17

6 10 60 27

7 7 49 34

8 6 48 40

40 234

a) b)

c) d)

Question-64 Bar chart

a) Is used for discrete data b) Show data horizontally

c) Show data vertically d) Both b and c

79 | P a g e
Question-65 Following data is given 5, 10, 13, 17, 20, 25, 30, 33, 35, 40, If all of the values are increased by 20% then
what will be the revised mean and S.D

a) b)

c) d)

Question-66 If values are increased by 20% then which of the following will change

a) Mean only b) S.D only

c) Botha and b d) None of these

Question-67 Following data is given

No. of employees Total salary St. Deviation

Company A 200 25,000,000 25,000

Company B 150 36,000,000 8,000

In your opinion which company is more stable?

a) Company A b) Company B

c) Both companies are same d) Cannot be determined

Question-68 Following data is given 5,10,13,17,20,25,30,33,35,40 If all the values are increased by 20% then find the
mean and SD of revised values.

a) d)

b) c)

Question-69 If all the values are increased by 20% then which of the following will change?

a) Mean only b) SD only

c) Both a and b d) None of these

Question-70 What are the properties of variance

a) Variance of constant is 0 b) variance is independent of change of origin

c) Variance is affected by squares times of change d) Variance cannot be negative even if all the
of scale values in a data are negative

80 | P a g e
Question-71 What the following graph is showing?

a) Data is symmetrical b) Data is positively skewed

c) Data is balanced d) Data is constant

Question-72 Find Harmonic mean of 7,8,9

a) 7.9 b) 8

c) 7.5 d) None of these

Question- 73 Which of the following graph represent grouped frequency distribution?

a) Pie chart b) Histogram

c) Simple bar chart d) All of these

Question-74 The scale on y-axis of ogive is

a) Frequency b) Histogram

c) Cumulative frequency d) None of these

Question-75 Following are the marks obtained by 7 students in a test 29, 82, 53, 37, 74, 42, 68 What Semi-Inter Quartile
Range?

a) 18.5 b) 55.5

c) 37 d) None of these

Question-76 For the data having: mean=87, median=92, variance=64 Find Co-efficient of skewness.

a) 0.625 b) 21.125

c) -1.875 d) 75.5

Question-77 What % of observations lie above P20?

a) 20 b) 80

c) 30 d) 40

81 | P a g e
Question-78 In histogram what does y-axis represent?

a) Class limits b) Class height

c) Frequency d) Range of data

Question-79 Which of the following cannot be determined through ogive?

a) Median b) Percentile

c) Quartiles d) Mode

Question- 80 Find mean and SD from the following data.

Classes 1 - - - 10 11 - - - 20 21 - - - 30 31 - - - 40 41 - - - 50 51 - - - 60

Frequency 3 5 7 10 6 4

a) Mean 31.57 and SD-14.33 b)

c) d)

Question-81 Mean of 10 observations is 7.5. if 25 is deducted from each observation and then it is multiplied by 2. Mean
of new observation is:

a) 10 b) 20

c) 15 (d) 5

Question-82 The median of a given frequency distribution is found graphically with the help of:

a) Histogram b) Pie Chart

c) Frequency curve d) Ogive

Question- 83 Find geometric mean for following distribution:

MARKS 0-3 30-50 50-80 80-100

No. of Students 20 30 40 10

a) 43.29 b) 45.36

c) 47.26 d) 41.26

Question-84 Calculate harmonic mean for the data given below:

Marks 30-39 40-49 50-59 60-69 70-79 80-89 90-99

82 | P a g e
f 2 3 11 20 32 25 7

a) 60.3 b) 69.6

c) 65.1 d)75.6

Question-85

Terrier and SFP are two stocks traded on the New York Stock Exchange. For the past seven weeks

you recorded the Friday closing price (dollars per share):

Terrier: 32 35 34 36 31 39 41

SFP: 51 55 56 52 55 52 57

Which stock is more

stable?

a) Terrier b) SFP

Both are not d) Both are stable

c) stable

Question- 86 If the laboratory technician A completes 40 analyses with a standard deviation of San technician B
completes 160 analyses per day with standard deviation of 15, find which employee show less variability.

a) Technician b) Technician B

c) Both are same d) Data is insufficient

Question-87 When any value in the data is negative

a) Geometric Mean becomes zero b) Harmonic Mean becomes zero

c) Hypergeometric Mean becomes zero d) None of the above

Question-88 Which mean is least affected by extreme values?

a) Arithmetic Mean b) Harmonic Mean

c) Geometric Mean d) All of the above

83 | P a g e
Question-89 The following stem and leaf display shows the number of units produced in a day

7 0.1.2.3.4.4

% 1,2,3,4,5.5,6

9 0,3,4,6,6,7,8,9

10 1,2,3,5,5,5,5,7,8,9

86.7.8.9

What is the middle value?

a) 66 b) 77

c) 85 d) 79

Question-90 Types of dispersion which measures the variation present among the values with same square of units of
variable?

a) Absolute Dispersion b) Relative Dispersion

c) Binomial Dispersion d) None of the above

Question-91 Kaleem determines the coefficient of determination as 86 %. This means that the variation

explained by the variable was:

a)86% b) 14%

c) 84% d) 15%

Question-92 Which of the following is not affected by Extremely Large or Extremely small values of data?

a) A.M b) H.M

c) Median d) None of these

Question-93 By Joining the midpoints of the rectangles of histogram, we get graph of:

a) Relative frequency curve b) Frequency curve (polygon)

c) Ogive d) All of these

84 | P a g e
Question-94 The stem and leaf display constructed from weight(kg) of 15 persons is given below:

STEM LEAF

4 5,

2, 3

5 4,

7, 4,8,9,5

6 6,2,3,1

7 5,7

Find Median:

a) 58.5 b) 58

c) 59 d) 5.5

Question 95 The stem and leaf display constructed from weight(kg) of 15 persons is given below:

STEM LEAF

1 3,5,9

3 1,3,5,5,8,9

5 1,6,7,9,9

8 1,2,4,4,5,9

Find Mean and Median:

a) 52.5 and 53.5 b) 58 and 50

c) 51 and 59 d) 51 and 55

Question 96 Following are the marks obtained by 7 students in a test. 29, 82, 53, 37, 74, 42, 68. What is

Semi-Interquartile Range?

85 | P a g e
a) 18.5 b) 37

© 55.5 d) None of these

Question-97 Standard deviation expressed as a percentage in terms of mean is:

a) Co-efficient of skewness b) Co-efficient of variation

c) Co-efficient of correlation d) None of these

Question-98 If the standard deviation of the values 2, 3, 6, 7, 12 is 3.52 then standard deviation of 6, 7, 10, 11,16 will be:

a) 7.52 b) 0.48

c) 3.52 d) None of these

Question-99 Which of the following is affected by change of origin as well as scale?

a) A.M b) G.M

c) H.M d) All of these

Question-100 The amount of milk produced by a Cow is:

a) Quantitative variable: b) Discrete variable

c) Continuous variable d) Qualitative variable

Question-101 The arrangement of data into classes according to the size and magnitude is

a) Frequency b) Frequency distribution

c) Relative frequencies d) All of these

Question-102 An ogive is

a) Graph of frequency curve b) Graph showing linear relationship

c) Graph of the cumulative frequency distribution d) Graph of DE cumulative frequency distribution

Question-103 Find mode

a) 2 b) 5

c) 25 d) 46

Question-104 Which of the following statement is true?

a) Variance is equal to S.D b) Variance is greater than

c) Variance is less than S.D. d) All are true in different situations

86 | P a g e
Question-105 G.M can be located graphically by:

a) Frequency curve b) Cumulative frequency curve

c) Histogram d) None of these

Question-106 Following are the marks obtained by 7 students in a test. 29, 82, 53, 37, 74, 42, 68. What is Semi-Inter
Quartile Range?

a) 18.5 b) 55.5

c) 37 d) None of these

Question-107 For the data having: mean = 87, median=92, variance = 64 Find Co-efficient of skew

a) 0.625 b) 21.125

c) 1.875 d) 75.5

Question-108 What % of observations lie above P20?

a) 20 b) 80

c) 30 d) 40

Question-109 In histogram what does y-axis represent?

a) Class limits b) Class height

c) Frequency d) Range of data

Questing-110 Which of the following cannot be determined through ogive?

a) Median b) Percentile

c) Quartiles d) Mode

Question-111 Find mean and SD from the following data.

Classes 1 - - - 10 11 - - - 20 21 - - - 30 31 - - - 40 41 - - - 50 51 - - - 60

Frequency 3 5 7 10 6 4

a) Mean=31.57 and a) SD=14.331 b)

c) d)

87 | P a g e
Question-112 The relative frequency distribution and cumulative relative frequency of the middle number group
respective is:

Class 1.5 – 1.9 2.0 – 2.4 2.5 – 2.9 3.0 – 3.4 3.5 – 3.9 4.0 – 4.4 4.5 – 4.9
interval

F 2 1 4 15 10 5 3

a) 0.375,55 % b) 55% 0.375

c) 0.55 37.5% d) 0.375,0.55

Question-113 Mean of 10 observations is 7.5. if 2.5 is deducted from each observation and then it is multiplied

by 2 Mean of new observation is:

a) 10 b) 20

c) 15 d) 5

Question- 114 The median of a given frequency distribution is found graphically with the help of:

a) Histogram b) Pie Chart

c) Frequency curve d) Ogive

Question-115 Find geometric mean for following distribution:

Marks 0 - 30 30-50 50-80 80-100

No. of students 20 30 40 10

a) 43.29 b) 45.36

c) 47.26 d) 41.26

Question-116 Calculate harmonic mean for the data given below:

Marks 30-39 40-49 50-59 60-69 70-79 80-89 90-99

f 2 3 11 20 32 25 7

a) 60.3 b) 69.6

c) 65.1 d) 75.6

88 | P a g e
Question-117

Terrier and SFP are two stocks traded on the New York Stock Exchange. For the past seven weeks

you recorded the Friday closing price (dollars per

share):

Terrier: 32 35 34 36 31 39 41

SIP 51 55 56 52 55 52 57

Which stock is more stable?

a) Terrier b) SFP

c) Both are not stable d) Both are stable

Question-118 If the laboratory technician A completes on average 40 analyses with a standard deviation of 5 and
technician B completes on average 160 analyses per day with standard deviation of 15, find which employee shows less
variability.

a) Technician A b) Technician B

c) Both are same d) Data is insufficient

Question-119 When any value in the data is negative

a) Geometric Mean becomes zero b) Harmonic Mean becomes zero

c) Hypergeometric Mean becomes zero d) None of these

Question-120 Which mean is least affected by extreme values?

a) Arithmetic Mean b) Harmonic Mean

c) Geometric Mean d) All of the above

Question-121 Types of dispersion which measures the variation present among the values with same unit or

square of units of variable?

a) Absolute Dispersion b) Relative Dispersion

c) Binomial Dispersion d) None of the above

89 | P a g e
Question-122 Kaleem determines the coefficient of determination as 86%. This means that the variation

explained by the variable was:

a) 86% b) 14%

c) 84% d) 15%

Question-123 Which of the following is not affected by Extremely Large or Extremely small values of a data?

a) A.M b) H.M

(c) Median d) None of these

Question-124 By Joining the midpoints of the rectangles of histogram, we get graphs of:

a) Relative frequency curve b) Frequency curve

c) Ogive d) All of these

Question-125 Which of the following Statement(s) is/are correct?

a) Ogives are used to find Median b) Cumulative Frequency is used to construct the
Ogives

c) Ogives are obtained from joining the midpoint d) d) Both A and B


of histogram

Question- 126 Compute Harmonic mean of Following Marks obtained in Math

Amjad sidra Nazia

Subject Math 69 65 59

a) 64.06 b) 64.33 c) 63.20 d) 71.0

Question- 127 The stem and leaf display constructed from weight(kg) of 15 persons is given below:

STEM LEAF

4 5,2,3

5 4,7,4,8,9,5

6 6,2,3,1

7 5,7

Find Median:

90 | P a g e
a) 58.5 b) 58

c) 59 d) 5.5

Question 128 Following are the marks obtained by 7 students in a test, 29, Semi Interquartile Range?

a) 18.5 b) 37

c) 55.5 d) None of these

Question-129 Standard deviation expressed as a percentage in terms of mean is:

a) Co-efficient of skewness b) Co-efficient of variation

c) Co-efficient of correlation d) None of these

Question-130 If the standard deviation of the values 2, 3, 6, 7, 12 is 3.52 then standard deviation of 6, 7, 10,11. 1o will
be:

a) 7.52 b) 0.48

c) 3.52 d) None of these

Question-131 Which of the following is affected by change of origin as well as scale?

a) A.M. b) G.M

c) H.M d) All of these

Question-132 The amount of milk produced by a Cow is;

a) Quantitative variable b) Discrete variable:

c) Continuous variable d) Qualitative variable

Question-133 The arrangement of data into classes according to the size and magnitude is

a) Frequency b) Frequency distribution

c) Relative frequencies d) All of these

Question-134 An ogive is

a) Graph of frequency curve. b) Graph showing linear relationship

c) Graph of cumulative frequency distribution d) Graph of DE cumulative frequency

Question-135 Which of the following statement is true?

a) Variance is equal to S.D b) Variance is greater than S.D

c) Variance is less than S.D. d) All are true in different situations

91 | P a g e
Question-136 G.M can be located graphically. by:

a) Frequency curve b) Cumulative frequency curve

c) Histogram d) None of these

Question-137 The mean of 200 items is 48 and the standard deviations is 3. The sum of squares of all items is

a) 462,600 b) 9600

c) 600,462 d) 426,600

Question-138 The following stem and leaf display shows the number of units produced in a day:

Stem Leaf

0,1,3,3,5,5,9

0,2,3,6,7,7,8

5,9

0,0,1,5,6

103,6

What is the range?

a) 22 b) 0

c) 68 d) 65

Question-139 The weights (in kgs) of 5 girls are 47, 48, 49, 50, 51. Find coefficient of variation

a) 41 b) 49

c) 2.89 d) 8.29

Question-140 When calculating the average rate of debt expansion for a company, the correct mean to use is:

a) Arithmetic Mean b) Weighted Mean

c) Geometric Mean d) All of the above

92 | P a g e
Question-141 Co-efficient of variation is affected by the change of:

a) Origin b) Scale

c) Origin and scale d) All of these

Question-142 For the data having:

Mean=87, Median = 92 and variance = 64

Find Co-efficient of skewness

a) 0.625 b) -1.875

c) 21.125 d) 75.5

Question- 143 If a clock strikes once at I O’clock, thrice at 3 O’clock and so on and again once at one O’clock and so on,
then how many times will the bell be struck in the course of 2 days?

a) 78 b) 156

c) 288 d) 312

Question- 144 The arithmetic mean of profits earned by two subsidiaries A and B of a company is Rs. 10million whereas
the geometric mean is Rs. 8 million. Find the profit earned by subsidiary A.it is known. That subsidiary A earns more
profit than subsidiary B

a) Rs 16 million b) Rs 4 million

c) Rs 20 million d) Rs 15 million

Question- 145 The S.D of X is 5.

_________

What will be the Variance of Y if Y

a) 37.5 b) 100

c) 56.25 d) 7.5

Question-146 The weights of 5 students are 47, 48, 49, 50, and 51. Find coefficient of variation

a) 1.41% b) 49%

c) 2.89% d) 8.29%

93 | P a g e
Question-147 A frequency distribution has Q1=10, Q2=20 and Q3=40. The distribution is:

a) Symmetric b) Positively skewed

c) Negatively skewed d) None of these

The combined mean of 3 groups is 12 and combined mean of first two groups is 3. If the first, second and third groups
have 2, 3, and 5 items respectively, then mean of third group is:

a) 10 b) 21

c) 12 d) 13

Question-148 The quartile deviation of 8 numbers is 20. If each number is increased by 5, the quartile deviation will

a) Remain Same b) Increase by 5

c) Decrease by d) None of these

94 | P a g e
1/1/2022 CHAPTER 9
INDICES

COMPILED BY
ABDUL AHAD BUTT

0|Page
INDEX NUMBER
• An index number is the measure of change in a variable (or group of variables) over
time.
• It is typically used in economics to measure trends in a wide variety of areas including:
stock market prices, cost of living, industrial or agricultural production, and imports.
• Index numbers are one of the most used statistical tools in economics.

Use of index number


• Index numbers are use in the field of commerce, meteorology, labour, industry, etc.
• Index number measure fluctuations during intervals of time, group difference of
geographical position of degree, etc.
• They are used to compare the total variations in the prices of different commodities in
which the unit of measurements differs with time and price etc.
• They measure the purchasing power of money
• They are helpful in forecasting future economic trends
• Index numbers of import prices and export prices are used to measure the changes in
trade of country

Limitations of index number


• They are simply rough indications of the relative changes
• The choice of representative commodities may lead to fallacious conclusions as they are
based on sample
• There may be errors in the choice of base periods or weights, etc.
• Comparisons of changes in variables over long periods are not reliable
• They may be useful for one purpose but not for another
• They are specialized types of averages and hence are subject to all those limitations, which
an average suffers from

Rebasing an index
• Rebasing/splicing involves redefining the base year of an index in a particular year and
then restating the index values in previous years
• It is sometimes necessary to change the base of a time series (to rebase) of fixed base
relatives, perhaps because the base time point is too far in the past

Main steps in the construction of index numbers


• Selection of products to be included, their number and price quotations
• Selection of base period and calculation of price relatives
• Selection of average to be used
• Selection of appropriate weight

1|Page
UNWEIGHTED INDEX NUMBER

Formula: Unweighted index number


• Unweighted index number is an index number which contain only one variable (price or
quantity) is taken for comparison
• Unweighted indices are generally classified into simple aggregative and simple average of
relatives

pn
1. Price relative = x 100
po

pn
2. Link relative = x 100
pn−1

qn
3. Quantity relative = qo
x 100

4. Simple aggregative index (Composite index number)


∑pn
Pon = x 100
∑po

QUESTION NO -1 Given the prices of a commodity per maund for the period 1945 to
1959 as:-

Years Prices Years Prices Years Prices


1945 12.70 1950 24.85 1955 15.65
1946 18.97 1951 20.90 1956 16.15
1947 19.70 1952 19.80 1957 20.20
1948 13.50 1953 23.65 1958 25.25
1949 15.65 1954 24.55 1959 32.40
Construct index numbers correct to 2 decimal place:
(i) 1945 as base.
(ii) Average price of all the year as base price.
Solution:
𝐩 𝐩𝐧 𝐩 𝐩𝐧
Year Prices (i) I = 𝐩𝐧 x 100 = 𝟏𝟐.𝟕 x 100 (i) I = 𝐩𝐧 x 100 = 𝟐𝟎.𝟐𝟔 x 100
𝐨 𝐨

1945 12.7 100 62.69


1946 18.97 149.37 93.63
1947 19.70 155.12 97.24
1948 13.50 106.30 66.63
1949 15.65 123.23 77.25
1950 24.85 195.67 122.66
1951 20.90 164.56 103.16
1952 19.80 155.91 97.73
1953 23.65 186.22 116.73
1954 24.55 193.31 121.17
1955 15.65 123.23 77.25
1956 16.15 127.17 79.71
1957 20.20 159.06 99.70
1958 25.25 198.82 124.63
2|Page
1959 32.40 255.12 159.92
∑ 303.92 --------- ---------
Required:
(i) 1945 as base; p0 = 12.7
303.92
(ii) po = Average price of all the year = = 20.26
15

QUESTION NO -2
From the data given below, compute, the index numbers of prices, taking 1962 as base.
Commodities (Prices in Rs.)
Year Firewood Soft cake Kerosene oil Matches
1962 3.25 2.50 0.20 0.06
1963 3.44 2.80 0.22 0.06
1964 3.50 2.00 0.25 0.06
1965 3.75 2.50 0.25 0.06

Solution:
𝐏 Index Number
Year Price Relatives = 𝐏𝐧 x 100
𝐨 (Mean)
Firewood Soft cake Kerosene oil Matches
1962 100 100 100 100 100
1963 105.85 112 110 100 106.96
1964 107.69 80 125 100 103.17
1965 115.38 100 125 100 110.10

CHAIN INDEX NO:

𝒗𝒂𝒍𝒖𝒆 𝒊𝒏 𝒕𝒉𝒆 𝒄𝒖𝒓𝒓𝒆𝒏𝒕 𝒑𝒆𝒓𝒊𝒐𝒅


Link relative= 𝒙 𝟏𝟎𝟎
𝒗𝒂𝒍𝒖𝒆 𝒊𝒏 𝒕𝒉𝒆 𝒑𝒓𝒆𝒗𝒊𝒐𝒖𝒔 𝒑𝒆𝒓𝒊𝒐𝒅

𝒓𝒆𝒍𝒂𝒕𝒊𝒗𝒆 𝒊𝒏𝒅𝒆𝒙 𝒐𝒇 𝒄𝒖𝒓𝒓𝒆𝒏𝒕 𝒚𝒆𝒂𝒓 𝒙 𝒄𝒉𝒂𝒊𝒏 𝒊𝒏𝒅𝒆𝒙 𝒐𝒇 𝒑𝒓𝒆𝒗𝒊𝒐𝒖𝒔 𝒚𝒆𝒂𝒓


Chain index =
𝟏𝟎𝟎

QUESTION NO -3 Find chain index numbers (using geometric mean to average the
relatives) for the following data of prices, taking 1970 as the base year.

Years
Commodities
1970 1971 1972 1973 1974
A 40 43 45 42 50
B 160 162 165 161 168
C 20 29 52 23 27
D 240 245 247 250 255

3|Page
Solution:

Arrange the data

Years COMMODITIES
A B C D
1970 40 160 20 240
1971 43 162 29 245
1972 45 165 52 247
1973 42 161 23 250
1974 50 168 27 255

𝐩𝐧
Year Link Relatives = 𝐩 x 100 G.M.= Chain Indices
𝐧−𝟏
4
√X1 . X2 . X3 . X4
A B C D
1970 100 100 100 100 100 100
100 x 112.66
1971 107.50 101.25 145 102.08. 112.66 = 112.66
100
112.66 x 117.82
1972 104.65 101.85 179.31 100.81 117.82 = 132.74
100
132.74 x 79.91
1973 93.33 97.58 44.23 101.21 79.91 = 106.07
100
106.07 x 110.43
1974 119.05 104.34 117.39 102 110.43 = 117.13
100

QUESTION NO -4
Construct chain indices for the following years. Taking 1940 as base.
Years
Item
1940 1941 1942 1943 1944
Wheat 2.80 3.40 3.60 4.0 4.20
Rice 2.95 3.60 2.90 2.75 2.75
Maize 3.10 3.50 3.40 2.50 3.70

Solution:
ARRANGE THE DATA

Item
Wheat Rice Maize
1940 2.80 2.95 3.10
1941 3.40 3.60 3.50
1942 3.60 2.90 3.40
1943 4.0 2.75 2.50
1944 4.20 2.75 3.70

4|Page
𝐏𝐧
Link Relatives = 𝐏 x 100 ∑𝐗 Chain
Years 𝐧−𝟏 Mean = 𝐧 Indices
Wheat Rice Maize
1940 100 100 100 100 100
1941 121.43 122.03 112.90 118.79 118.79
1942 105.88 80.56 97.14 94.53 112.29
1943 111.11 94.83 73.53 93.15 104.59
1944 105 100 148 117.66 123.06

WEIGHTED INDEX NO:

Formula:

1. Weighted index number


i) Laspeyre’s index number / Base year weighted index number
∑pn qo
= x 100
∑po q0

ii) Paasche’s price index number /current year weighted index number
∑pn qn
= x 100
∑po qn

iii) Fisher index number


= √Laspeyre’s index x Paasche’s price x100

2. Value index number


∑pn qn
= x 100
∑po qo
3-Laspeyre Quantity Index
∑p0 qn
= x 100
∑po q0
4-Laspeyre Quantity Index
∑pn qn
= x 100
∑𝑛q0
5-
∑pn qo
(i) Pon = x 100 (Aggregate expenditure method)
∑po qo
∑WI
(ii) Pon = (Family/house hold budget method)
∑W
pn
Where I = x 100
po

5|Page
QUESTION NO -5
Calculate Laspeyre’s, Paasche’s and Fisher’s ideal index for the following data.

Average Price (Rs) Quantity (Units)


Item
1992 1993 1992 1993
Wheat flour 4.38 4.57 20 kg 16kg
Rice 14.15 15.58 10kg 12kg
Moong pulse 18.67 17.28 1 kg 1 kg
Gram pulse 10.41 16.36 1 kg 1 kg
Solution:

1992 1993
Items
po qo Pn qn Pnqo poqo Pnqn Poqn
Wheat flour 4.38 20 4.57 16 91.4 87.6 73.12 70.08
Rice 14.15 10 15.58 12 155.8 141.5 186.96 169.8
Moong Pulse 18.67 1 17.28 1 17.28 18.67 17.28 18.67
Gram Pulse 10.41 1 16.36 1 16.36 10.41 16.36 10.41
∑ — — — — 280.84 258.18 293.72 268.961

∑p q 280.84
Laspeyer’s Price Index No. = ∑pnqo x 100 = 258.18 x 100 =108.78
o o

∑p q 293.72
Paasche’s Price Index No. = ∑pnqn x 100 = 268.86 x 100 = 109.20
o n

∑p q ∑pn qn
Fisher’s Price Index No. = √∑pnqo x x 100
o o ∑po qn

280.84 293.72
= √258.18 x 268.96 x 100

= √1.1879 x 100 = 108.99 = 109


QUESTION NO -6 An inquiry into the budgets of the middle class families in a city in
England I gave the following in-formation. What change in cost of living the figures of 1929
show as compared in 1928?
Expenses on Food 35 Rent 15 Clothing 20 Fuel 10 Misc 20
Price (1928) 150 30 75 25 40
Price (1929) 145 30 65 23 45
Solution:
Items Po Pn W I WI
Food 150 145 35 96.67 3383.45
Rent 30 30 15 100 1500
Clothing 75 65 20 86.67 1733.4
Fuel 25 23 10 92 920
Misc 40 45 20 112.5 2250
∑ — — 100 — 9786.85

6|Page
Cost of Living Index Numbers:
∑IW 9786.85
Po = = = 97.8685
∑W 100

QUESTION NO -7

Compute the weighted index numbers for 1964 from the following data with 1960 as base.

Price Quantity
Commodities
1960 1964 1960 1964
Milk .3.95 4.25 97.75 104.36
Cheese 34.80 38.90 78 83
Butter 61.56 59.70 118 116

Solution:

Commodities P0 Pn q0 qn P0 q0 Pn q0 Pn qn P0 qn
Milk 3.95 4.25 97.75 104.36 386.11 415.44 443.53 412.22
Cheese 34.80 38.90 78 83 2714.40 3034.2 3228.7 2888.4
Butter 61.56 59.70 118 116 7264.08 7044.60 6925.2 7140.96
10364.59 10494.24 10597.43 10441.58

Weighted Index Numbers:


Laspeyre’s price index:
∑p q 10494.24
P01 = ∑pnq 0 x 100 = 10364.59 x 100 = 101.25
0 0

Paasche’s price index:


∑p q 10597.43
P01 = ∑pnq n x 100 = 10441.58 x 100 = 101.49
0 n

Fisher’s price index:


P01 = √Laspeyre′s index x Paasche′s index
= √101.25 x 101.49 = 101.37

QUESTION NO -8
Computer Consumer’s Price Index for 1996 on the basis of 1995 from the following data by
using
(i) Aggregate expenditure Method
(ii) Family Budget Method

Commodity Quantity Consumed Price 1995 Price 1996


Rice 260 kg 30 32
Wheat 360 kg 10 8

7|Page
Gram 100 kg 35 38
Sugar 100 kg 20 17
Ghee 60 kg 60 65
Electricity 500 units 1.50 1.75
Solution
𝒑
Commodity 𝒒𝒐 𝒑𝒐 𝒑𝒏 1=𝒑𝒏 *100 W=𝒑𝒐 𝒒𝒐 𝒑𝒏 𝒒𝒐 WI
𝒐

Rice 260 30 32 106.67 7,800 8320 832,026


Wheat 360 10 8 80.00 3,600 2880 288,000
Gram 100 35 38 108.57 3,500 3800 379,995
Sugar 100 20 17 85.00 2000 1700 170,000
Electricity 500 1.50 1.75 116.67 750 875 87,503
Ghee 60 60 65 108.33 3600 3900 389,988
21,250 21,475 2,147,512

(i) Aggregate Expenditure Method

∑𝒑 𝒒
𝒑𝒐𝒏 = ∑ 𝒑𝒏𝒒𝒏*100
𝒐 𝒐

𝟐𝟏𝟒𝟕𝟓
𝒑𝒐𝒏 = 𝟐𝟏𝟐𝟓𝟎 *100 = 101.06

(ii) Family Budget Method

∑ 𝑾𝑰 𝟐𝟏𝟒𝟕𝟓𝟏𝟐
𝒑𝒐𝒏 = ∑𝑾
= = 101.06
𝟐𝟏𝟐𝟓𝟎

Test yourself:
If the current year’s weighted index is 5% higher than the base year and Fisher ideal index
number is 250. Find out the lespeyre’s price and paaches price index
Solution:
Laspeyres x paaches =(fisher)2

x * (x +5%x) =(250)2

1.05 x2 =62500
Laspeyres =x =244
paaches = x +5%x =256

8|Page
Formula:
Consumer price index number
1-Deflation of Index Number

It is the device of removing the effect of inflation from the Index Number,

2-Purchasing power
when the prices increase, C.P.I increases. In other words, purchasing power of money decreases,
because purchasing power of money is inversely proportional to the C.P.I i.e.

𝟏
Purchasing power of money = 𝑪.𝑷.𝑰 𝒙 𝟏𝟎𝟎
Generally, the Income increases over a period of time, but real income may actually decrease
because of increase in cost of living (C.P.I).

𝑰𝒏𝒄𝒐𝒎𝒆 𝒐𝒇 𝑪𝒖𝒓𝒓𝒆𝒏𝒕 𝒚𝒆𝒂𝒓


3-Real Income (Wage) = x 100
𝑪.𝑷.𝑰 𝒐𝒇 𝑪𝒖𝒓𝒓𝒆𝒏𝒕 𝒀𝒆𝒂𝒓

4.Buying power =Real wages/wages

CONSUMER PRICE INDEX NUMBER (CPI) / RETAIL PRICE INDEX NUMBER


Is design to measure the changes in the composite price of a specified basket of goods and
services during the given period as compared with the base period
• The so-called basket would comprise various commodities consumed and services
received in the base period or the given period, grouped under the main heading
▪ Food and beverages
▪ Clothing and footwear
▪ Fuel and lighting
▪ Housing
▪ Services
There are two methods to compute CPI
• Aggregate expenditure method
• Family budget method (weighted average of price relatives
Limitation of CPI
• A limitation CPI is that the consumer goods it considers do not provide an index that
measures all production or consumption in the economy. Therefore, as a basic economic
barometer, the CPI is inherently flawed
• Because of CPI is purposely constructed with focus on the buying habits of urban
consumers, it has often been criticized as not providing an accurate measure of either
prices of goods or consumer buying habits for more rural areas

9|Page
• Although the CPI is widely used as the core indicator of inflation, its accuracy in this area
has drawn increasing criticism. For example, during a period when energy costs rose by
more than 50% and the prices of some the most commonly purchased grocery items
increased by nearly 30%, the CPI continued to show a very modest inflation rate when
other indicators measuring the buying power of consumers showed a dramatic increase in
the cost of living

QUESTION NO -9
The annual Income of industrial worker from 1990 to 1993 and consumer price Index Number to
the base 1990 were as follows: -

Year 1990 1991 1992 1993


Incomes (Rs.) 20,000 21000 24,000 26,000
C.P.I. 100 98 105 120
Find Real Income
Solution
Year Income C.P.I Real Income
1990 20,000 100 20,000
*100 = 20,000
100
1991 21,000 98 21,000
*100 = 21,429
98
1992 24,000 105 22,857
1993 26,000 120 21,667

QUESTION NO -10
In 1980 the wage of an employee was Rs.1200 and C.P.I was 180. It rose to 220 in 1982.
Calculate the additional dearness allowance to be paid to employee if he has to be compensated.
Solution

𝑊𝑎𝑔𝑒 𝑖𝑛 1980 𝑊𝑎𝑔𝑒 𝑖𝑛 1982


=
𝐶.𝑃.𝐼 𝑖𝑛 1980 𝐶.𝑃.𝐼 𝑖𝑛 1982

𝑊𝑎𝑔𝑒 𝑖𝑛 1980
Wages in 1982 = * C.P.I in 1982
𝐶.𝑃.𝐼 𝑖𝑛 1980

1200∗220
= = 1467
180
So additional dearness allowance = 1467 – 1200 = Rs.267

QUESTION NO -11
The following data represents the average monthly take home salary of the employees of an
organization.
Year 2005 2006 2007 2008
Pay (Rs) 12,350 13,350 14,800 16,500
Price index 110.1 122.3 137.6 160.2
Compute the real wages for each of the above years.
Solution:
Years Pay Price index Real wages = pay/index
2005 12,350 110.1 (12,350/110.1) x 100 = 11217.08
10 | P a g e
2006 13,500 122.3 (13,350/122.3) x 100 = 10915.78
2007 14,800 137.6 (14,800/137.6) x 100 = 10755.81
2008 16,500 160.2 (16,500/160.2) x 100 = 10299.63

QUESTION NO -12 (Taking data from Question 11)


Compute the amount of pay needed in 2008 to provided buying power equal to that enjoyed in
2006.

Solution:
Pay of 2008 Pay of 2006
=
Index of 2008 Index of 2006

13500
Pay of 2008 = x 160.2 = 17683.57
122.3

QUESTION NO -13 (Taking data from Question 11)


. If the price index for the year 2011 is 191.2. Calculate the amount of wages whose buying
power would be the same as that of year 2007.

Solution:
Pay of 2007 Pay of 2011
=
Index of 2007 Index of 2011

14800 Pay of 2011


=
137.6 191.2

Pay(wages) of 2011 = 20,565.11

QUESTION NO -14
The per capital income in a company in a country has increased from Rs. 45,000 in the
year, 2000 to Rs. 96,000 in the year 2007. Taking base as 100 in the year 2000. Consumer
price index number in 2007 stood at 160. Compute the real per capital income and the
purchasing power of money, in the year.

Solution:
Year Income C.P.I P.P Real Income
2000 45000 100 1 45000
x 100 = 1
100

2007 96000 160 1 96000 x 0.625


x 100 =
160
= 60,000
0.625

QUESTION NO -15
Nawaz company increased the wages of its workings by 20% during the year 2013,
whereas, the consumer price index changed from 240 to 270. Compute the
increase/decrease in real wages?

11 | P a g e
Solution:
Index of wages C.P.I Real wages
100 240 41.67
120 270 44.44

QUESTION NO -16
Use the following consumer price indices to find the purchasing power of a rupees for each
of year relative to the base year and deflate the per capital income.
Year Per capital income C.P.I

2010 19,500 100


2011 21,200 113
2012 22,300 120
2013 24,100 128

Solution:
Year Per capital C.P.I Purchase Deflated
income 𝟏𝟎𝟎 𝟏𝟎𝟎
power = 𝐂.𝐏.𝐈 income = 𝐂.𝐏.𝐈 x
P. capital
2000 19,500 100 1 19,500
2001 21,200 113 0.88 18,656
2002 22,300 120 0.83 18,509
2003 24,100 128 0.78 18,798

QUESTION NO -17
Use the following consumer price index to find the purchasing power of rupees for each of year
relative to the base year and deflate the per capital income.

Year Per capital income Consumer price


index
2007 6200 100
2008 6700 113
2009 7250 120
2010 7850 128
2011 8600 140
2012 9000 150

Solution:
Year income Consumer price P.P Deflated income
index
2007 6200 100 1 1x6200 = 6200
100
x 100 = 1

2008 6700 113 1 0.8850x6700=5929.2


x 100 = .8850
113

2009 7250 120 1 0.8333x7250=6041.67


x 100 = .8333
120

12 | P a g e
2010 7850 128 1 0.7813x7850=6132.81
x 100 = 0.7813
128

2011 8600 140 1 0.7143x8600=6142.86


x 100 = 0.7143
140

2012 9000 150 1 0.6667x9000=600


x 100 = 0.6667
150

QUESTION NO -18
For the following consumer price index. Calculate the purchasing power of rupees for each of the
year.
Solution:

Year Consumer price P.P


index
2006 100 1
= 100 x 100 = 1.00

2007 107 1
= 107 x 100 = 0.96

2008 121 1
= 121 x 100 = 0.83

2009 115 1
= 115 x 100 = 0.87

2010 109 1
x 100 = 0.92
109

2011 117 1
x 100 = 0.85
117

QUESTION NO -19
Calculate rate of inflation/deflation of the following data
Year Cost of basket CPI
2009 5.55 100
2010 6.67 120.18
2011 6.26 112.79
2021 6.45 116.21

Solution:
𝑪𝑷𝑰−𝑪𝑷𝑰
Inflation rate = 𝑪𝑷𝑰
Year Change Inflation/Deflation
2009 to 2010 𝟏𝟐𝟎. 𝟏𝟖 − 𝟏𝟎𝟎 20.18%
𝟏𝟎𝟎
2010 to 2011 𝟏𝟏𝟐. 𝟕𝟗 − 𝟏𝟐𝟎. 𝟏𝟖 -6015%
𝟏𝟐𝟎. 𝟏𝟖
2011 to 2012 𝟏𝟏𝟔. 𝟐𝟏 − 𝟏𝟏𝟐. 𝟕𝟗 3.03%
𝟏𝟏𝟐. 𝟕𝟗

Note: Real income/Deflated income/Real wage

13 | P a g e
EXTRA PRACTICE QUESTIONS
Question:1
The per capita income in a country has increased from 450 in the year 2000 to 960 in the
year 2007. Taking base as 100 in the year 2000. The consumer price index in 2007 stood at
160. Compute the real per capita income and the purchasing power of money in the year
2007.
Solution:
1
Purchasing power in 2007 = 𝐶.𝑃.𝐼 𝑥100
1
= 160 𝑥100

= 0.625
960
Real income in 2007 = 160 𝑥100

= 600
Question:2
Sales of product over the last four years have been
Year Amount
2000 100
2001 107.193
2002 117.384
2003 119.355
Annual price inflation of the production over the same period has been
From 2000 to 2001 5.1%
From 2000 to 2001 4.5%
From 2000 to 2001 5.6%
Required
Calculate the index number series of annual sales volume for the production for the year
2000 to year 2003 (using year 2000 as base)
Solution:
Year Inflation Sales Potential sales Index number
2000 ---- 100 100 100
2001 5.1% 107.193 101.991 101.99
2002 4.5% 117.384 112.329 112.33
2003 5.6% 119.355 113.026 113.03
Note:
𝒔𝒂𝒍𝒆𝒔
Potential sales = 𝟏+𝒊𝒏𝒇𝒍𝒂𝒕𝒊𝒐𝒏 𝒓𝒂𝒕𝒆

14 | P a g e
Question:3
Index number assuming 1995 as base year are as under
1995 100
1996 110
1997 125
1998 145
1999 150
Calculate the index for each year on chain base method
Solution:
Years Index Link Relative
1995 100 100
1996 110 110
1997 125 137.5
1998 145 199.375
1999 150 299.06
Question:4
A company sells 3 products A, B and C. From period “O” to “1”. Price of A, B and C rose
respectively from Rs 80, Rs 25 and Rs 50 to Rs 88, Rs 27 and Rs 53 in period “O”.
products A, B and C contributed respectively 25,45 and 30 percent to total rupees sales
volume.
(i) Which formula should be used to compute price index for period “1” with
period “O” as base
(ii) Compute the index
Solution:
Quantity Period “O” Period “1” P0q0 pnqo
(q0)
A 25 80 88 2000 2200
B 45 25 27 1125 1215
C 30 50 53 1500 159
Total 4625 5005
(i) The formula for computing index number when quantity of base period and prices for
base period as well as for other periods are given is Laspeyres only

∑𝐏𝐧𝐪𝐨
(ii) Pon = ∑𝐏𝟎𝐪𝟎 𝑥 100

𝟓𝟎𝟎𝟓
= 𝑥 100
𝟒𝟔𝟐𝟓
= 108.22

15 | P a g e
Points to remember:

1- Index numbers are used as the barometers of inflation and to measure changes in a
variable over time
2- The value of index number is called time series
3- The quantity which is based on the relative change over time in the sum of the values of
two or more time series variable is called simple aggregate or composite
4- The relative importance of each time series variable is determined by assigning an
appropriate factor called weighting factor
5- Generally, the appropriate weighting factors in the calculation of index numbers are
quantities
6- Generally, the time series variables used to calculate different types of index numbers
are price values
7- The quantity which is based on the relative change overtime in the weighted s um of two
or more time series variable is called weighted aggregate index number
8- The period with which others periods are to be compared is called base period
9- The index that uses quantities of base period as weights, so that only prices are allowed
to change in calculating weighted price aggregate index is known as Laspeyer’s index
number or base year quantities as fixed
10- If the quantities of current year are used to calculate weighted price index number so
calculated is called paasche’s index number
11- The Geometric mean of Laspeyer’s and paasche’s index number is called Fisher ideal
index number
12- The equation 1995=100 indicates that the index number are calculated using base year
1995
13- Consumer price index number
(i) Cost of living index number /Aggregate Expenditure method
(ii) Family Budget method (The retail price index)
14. Purchasing power (reciprocal of C.P.I)
=1/C.P.I x100
15. Real(Deflated) per capita income
=purchasing power x current per capita income
16. If the relative changes in the current year prices are expressed on the basis of
Previous year prices the simple index so calculated is known as chain base index
Or link relative
17. A scaling factor is used to change the base year
18. If all the values considered in calculating an index are of equal importance the
index is called unweighted
19. The weights used in quantity index are prices
20. An index number that studies variation into two or more variable is called a
composite index
21. The A.M of group indices gives the general index
22. The most appropriate average for index number is geometric mean
23. seasonal variation have minimal impact on chain index
24. P on is the price index for the time (n on o)
25. A measure of relative change or a percentage of relative or a special type of an
average is called index number
26. The value index number measures the change in the total monetary value of a
basket of commodities over a period of time

16 | P a g e
CHAPTER 9: INDICES

1-The quantity which is based on the relative change over time in the sum of the values of two
or more time series variables is called:
A) Simple index number B) Composite index number
C) Simple aggregate (or composite) D) None of these
2-The relative importance of each time series variable is determined by assigning
an appropriate factor called:
A) Weighting factor B) Composite factor
C) Price factor D) None of these
3- Generally the time series variables used to calculate different types of Index
numbers are:
A) Price values B) Quantities
C) (a) and (b) both D) None of these
4- Generally the appropriate weighting factors, in the calculation of index numbers
are:
A) Price values B) Quantities
C) (a) and (b) both D) None of these
5- The quantity which is based on the relative change over time in the weighted sum
of two or more time series variable is called:
A) Simple aggregate index (b) Simple index number
(c) Weighted aggregate index number (d) None of these
6- The period with which other periods are to be compared is called:
A) Current period B) Base period
C) Chain-base period D) None of these
7- The index that uses quantities of base period as weights, so that only prices are
allowed to change, in calculating weighted price aggregate is known as:
A) Laspeyer’s Index Number B) Paasche’s Index Number
C) Fisher’s Index Number D) None of these
8- If the quantities of current year are used to calculate weighted price aggregate,
the index number so calculated is called:
A) Laspeyer’s Index Number B) Paasche’s Index Number
C) Fisher’s Index Number D) None of these
9- The geometric mean of Laspeyer’s and Paasche’s Index Number is called:
A) Mean Index Number B) Marshall Edge Worth’s
C) Fisher’s Ideal Index Number D) None of these
10- The equation 1995 = 100 indicates that the index numbers are calculated using:
A) Base year 1995 B) Current year 1995
C) Previous year 1995 D) None of these

11- The index that measures changes in prices that affect the cost of living of a large
fraction of population is called:
A) Whole sale price Index Number B) Simple price relatives
C) Consumer Price Index Number D) None of these

12- Index numbers are used as the barometers of:

A)Prices B) Quantities

17 | P a g e
C)Inflation D)None of these
13- If the reciprocal of consumer price Index is expressed as the percentage the
resulting value is called:

A)Rate of inflation B)Rate of deflation


C)Purchasing power of money D)None of these
14- If the purchasing power of money is multiplied by the current per capita income
the resulting value is known as:

A)Rate of inflation B)Error in per capital income


C)Real (or deflated) per capita income D)None of these
15- If the relative changes in the current year prices are expressed on the basis of
previous year prices the simple Index so calculated is known as:

A)Simple Price Relative B)Fixed base Index


C)Chain-base Index or Link Relative D)None of these
16- An index number is used:
A) To measure changes in price` B) To measure change in demand
C) To measure change in quality D) To measure change in a variable over
time
17- A simple aggregate quantity index is used to:
A) Measure the overall change in quantity B) Measure the overall change in price of a
of a range of products range of products
C) Measure the range in quantity of a product D) Measure the change in price of a product
18- A simple aggregate price index:
A) Ignores relative quantitates B) compares relative quantities to relative
prices
C) Considers relative quantities D) Compares absolute prices o absolute
quantities
19- This index measures the change from month to month in the cost of a respective ‘basket’ of
goods and services of the type bought by a typical household:
A) Retail price index B) Financial times index
C) Laspyers price index D) Paasche price index
20- The laspeyrs and Paasche index are examples of:
A) Weighted price index only B) Aggregate index numbers
C) Weighted index numbers D) Weighted quantity index only
21- The Laspeyres price index:
A) Regards the current year quantities as B) Regards the base year process as fixed
fixed
C) Regards the base year quantitates as D) None of these
fixed
22- A scaling factor is used to:
A) Change an aggregate index to a weighted B) Change the base year
index
C) Convert the Paasche index to a Laspeyers D) Change a simple index to a weighted index
index
23- If an index number calculation over 8 years with a base value of 100 gave an index for 1993
of 110. What would be the percentage relative for 1993?

18 | P a g e
A) 110 B) 90.0
C) 13.75 D) 880
24- If all the values considered in calculating an index are of equal importance, the index is:
A) weighted B) Simple
C) Unweighted D) None of these
25- When the base year values are used as weights, the weighted average of relatives prices
index is the same as:
A) Laspeyers index B) Paasches index
C) The unweighted average of relatives price D) None of these
index
26- Commodities subject to considerable price variations can best be measured by a:
A) Price index B) Quantity index
C) Value index D) None of these
𝑃𝑛
27- If an weighted average of relative index 𝑃 × 100 is calculated for each product in the
0
composite. What is then done to finish the calculation?
A) The values are multiplied together B) The largest value is found
C) The values are averaged D) None of these
28- The weights used in quantity index are:
A) Percentage of total quantity B) Prices
C) Average of quantities D) None of these
29- The weights used in price index are:
A) Percentage of total quantity B) Prices
C) Quantities D) None of these
30- The base period can be described as a ‘normal’ period if:
A) It is neither the peak not the trough of a B) It is the most recent period for which we
fluctuation have data
C) It is the methods are useful for price D) None of these
indices
31- A primary difference between average of relativesand aggregate methods is that:
A) Aggregate methods sum all useful for B) Average of relatives methods sum all
price indices prices before finding the ratio
C) Aggregate methods are useful for price D) None of these
indices
32- Which one of the following methods uses quantities consumed in the current period when
computing a weighted index?
A) Laspeyres’ method B) paasche’ method
C) Marshal Edgeworth’s method D) Fishers’s method
33- Which of the following methods uses quantities consumed in the base period when
computing a weighted index?
A) Laspeyers’ method B) Paasches’ method
C) Fishers’ method D) None of these
34- It is possible to change the base year without changing the quantities used for weights when
using:
A) Laspeyres’ method B) Paasche’s method
C) The weighted aggregate method D) None of these
35- To measure how much the cost of some variable changes over time, you would use:
A) A value index B) An inflation index
C) A quantity index D) None of these
19 | P a g e
36- When computing weighted average of relatives index, we would be best able to compare
indices from various periods if:
A) Base values used as 𝑝𝑛 𝑞𝑛 B) Current values used as 𝑝𝑛 𝑞𝑛
C) Fixed values used as 𝑝𝑛 𝑞𝑛 D) Either base or fixed values were used as
𝒑𝒏 𝒒𝒏
37- Theoretically best average used in construction of composite index is:
A) The arithmetic mean B) The geometric mean
C)v The median D) The harmonic mean
38- The prices used in the construction of consumer price index numbers are:
A) The retail prices B) The wholesale price
C) The fixed prices D) None of these
39- The consumer price index number also called:
A) the cost of living index number B) The retail price index
C) the wholesale price index D) both A and B
40- Which method of construction of consumer price index number is the Laspeyres’ index
number?
A) Aggregate expenditures method B) family budget method
C) Both A & B D) None of these
41- To measure changes in total monetary worth, one should calculate:
A) A price index B) A quantity index
C) A value index D) None of these
42- In times of inflation …… provides a better measure of actual output than a corresponding
value index:
A) A price index B) A quantity index
C) A composite index D) None of these
43- An index number that studies variations into two or more variables is called:
A) A price index B) A quantity index
C) A composite index` D) None of these
44- Index number is:
A) A measure of relative change B) A percentage relative
C) A special type of an average D) All of these
45- Index number is equal to:
A) Product of price relatives B) Average of price relative
C) Sum of price relatives D) None of these
46- The value index numbers measure the:
A) Change in price of a basket of B) Change in the total monetary value of a
commodities from one period to another basket of commodities over period of time
C) Change in retail price of various D) Change in the general price level in the
commodities country
47- Which of the following price indices will tend to underestimate the rise in prices?
A) Laspeyre’s method B) Paasche’s method
C) Fisher’s method D) Value index
48- The weighted average of price relatives using base values as weights is same as the:
A) Unweight aggregates price index B) Unweighted aggregates quantity index
C) Laspeyre’s price index D) Paasche’s price index
49- Seasonal variations have minimal impact on:
A) Unweighted index B) chain index
C) price index D) Quantityindex
20 | P a g e
50- In consumer price index number we use:
A) Discounted price B) Wholesale price
C) retail prices D) None of these
51- P0n is the price index for the time:
A) n on 0 B) 0 on n
C) n on n D) 0 on 0
52- Purchasing power of money is:
A) Reciprocal of price index B) Equal to price index
C) Square of price index number D) None of these
53- If all values considered in calculating an index are of equal importance, the index is:
A) Weighted B) Simple
C) Unweighted D) None of these
54- When the lease year values are used as weights, the weighted average of relatives price index
is the same as:
A) The Paasche’s index B) The Laspeyre’s index
C) The unweighted average of relatives price D) None of these
index
55- The weights used in price index are:
A) percentage of total price B) Quantities
C) Average of prices D) None of these
56- Which of the following method uses quantities consumed in the lease year when computing
weighted index:
A) Laspeyre’s method B) Paasche’s method
C) Fisher’s method D) None of these
57- Which of the following indices has an upward bias:
A) Laspeyre’s index B) paasche’s index
C) Fisher’s index D) None of these
58- The prices used in the construction of consumer price index numbers are:
A) The retail prices B) the wholesale prices
C) Fixed prices D) None of these
59- In Chain lease method, the base period is:
A) Fixed B) Changed
C) Constant D) None of these
60- The prices current period divided by prices of the lease period and then multiplied by 100 is
called:
A) Link relative B) Price relative
C) Chain relative D) None of these
61- A value index is given by the formula:
∑ 𝑃𝑛 𝑞𝑛 ∑𝑷 𝒒
A) ∑ 𝑃0 𝑞0
× 100 B) ∑ 𝑷𝒏𝒒𝒏 × 𝟏𝟎𝟎
𝟎 𝒏
∑ 𝑃𝑛 𝑞𝑛 D) None of these
C) ∑ 𝑃0 𝑞0
× 100
62- Which one of the following is an ‘Ideal’ index:

A) Laspeyre’s index B) paasche’s method


C) Fisher’s method D) Marshal Edgeworth’s index
63- The most appropriate average for index number is:
A) G.M B) H.M
C) A.M D) None of these
21 | P a g e
64- An increase in price index from 120 to 128 means that the price have increased by ______.
(a)8% (b)8.667%
(c)6% (d)6.667%
65-The increase in index from 100 to 113 and a price increase from 0.9 to 1.35 in terms of
percentage is.
(a)13% for both (b)50% for both
(c)13% and 50% respectively (d)50% and 13% respectively
66- ______ tends to understate inflation.
(a)Laspeyer’s Index Number (b)Paasche’s Index Number
(c)Fisher’s Index Number (d)All of these.
67- If Laspeyer’s Index Number is of 113 and paasche’s Index Number of 118, then the Fisher’s
Price Index would be
(a)115.4 (b)112.34
(c)154.7 (d)123.4
68-Construct price index using Laspeyre formula for the following data;
Product Price in 2020 Price in 2021 Quantity in 2020 Quantity in 2021
A 5 7 10 8
B 12 13 12 10
C 15 20 14 9

(a)125.25 (b)124.07
(c)124.66 (d)73.02
74- Provided there is no change in prices between base year and current year but the quantity
consumed for each type of product has increased by 5 units each. What will be the Laspeyre
price index?
(a)105 (b)100
(c)95 (d)Cannot de determined
75- Provided there is a 20% rise in prices from base year to current year of each of the
product under consideration. Which of the following will increase by 20% as well?
(a)Laspeyre price index (b)Paasche price index
(c)Fisher price index (d)All of these
76- If Paasche price index is 20% higher than Laspeyre price index. What will be the value
of Fisher’s price index?
(a)Cannot be determined (b)120
(c)109.55 (d)20% higher than Laspeyre price index
77- A machine cost Rs 5,000 in 2015. The price indices are as follows:
2015: 45
2021 : 63
How much will the machine cost in 2021?

22 | P a g e
(a)8,000 (b)7,000
(c)6,000 (d)10,000
78- Four years ago the price index of a particular product was 60. If the same index is now
108.Compute percentage increase in the price of a product.
(a)80% (b)48%
(c)58% (d)60%
79- Five years ago the price of a particular product was Rs. 5,000 and the relevant index was
90. Compute the price index now if the price of the product today is Rs. 7,500.
(a)135 (b)140
(c)40 (d)125
80- The price index linked to a particular product has increased from 80 to 96 in one year’
time and is expected to increase by same percentage every year. Which of the following
statement is true?
(a)Price of the product has increase by 16% (b)Price of the product has increased by
20%
(c)Price of the product will increase by 16% (d)Index will be 112 in next year
every year
81- Select one or more correct option from below:
(a)Index numbers are used to measure (b)A chain index series is an index series
changes over time where each new value in the index is
calculated based on the previous year
(c)consumer price index measures the cost (d)A chain index series uses same base year
of a consumer basket purchased by a
“typical” urban family at a point in time as
compared to the base year
82-. In Laspeyres price index number, quantities belonging to __BASE __ year are taken as
weight. Therefore, sometimes this method is called __BASE___ year quantity weight method.
83- The percentage change in the consumer price index can be used to determine rate of
_INFALTION/Deflation the CPI increase there is _INFLATION ___ and if the CPI
_DECREASE there is deflation
84- Fisher price index is a price index, computed for a given period by taking the _SQUARE
root of the product of the paasche index value and the Laspeyre index value

85- In the calculation of the ___LESPEYRE__ price index (p o Q0 ) , The denominator does
not change from year to year . The only information that has to be collected each year is the
prices of items in the index
86- The Paasche price index measure _PRICE ____ changes with reference to current
quantities of good in the basket.
87- __LESPEYRE___ price index tends to overstate inflation whereas the ___PAACHE___
price index tends to understate it. This is because consumer react to price increases by changing
what the buy.

23 | P a g e
EXTRA PRACTICE QUESTION

Question: 1

In laspeyre price index, what value is to changed every year

A) Price b) Quantity
c) both d) done
Question :2

Laspeyre price index shows prices of

a) Base year b) Current year


c) Both d) None
Question :3

Why we find it difficult to calculate Paasche index

a) Base year b) current year


c) Both d) None
Question :4

Which of the following has to be calculated every year for Paasche price index in addition to current
year prices, which make it difficult to calculate compared to Laspeyre index

a) Numerator b) Denominator
c) Both d) None
Question :5

Which of the following has to be calculated every year for Paasche price index in addition to

Current year prices, which make it difficult to calculate compared to Laspeyre index

a) Prices b) Quantities
c) Both d) None
Question :6

The only new information that has to be collected each year in the calculation of Laspeyre index is

a) Price of items b) Quantities of items


c) Both d) None
Question 7 Why Laspeyre price index is used more than the Paasche price index?

a) Because quantities of base year has to be b) Because more information has to be


collected to construct Paasche index collected to construct Laspeyre indes
c) Because less information has to be collected to d) Because more information has to be collected
construct Paasche index to construct Paasche index
Question :8 Laspeyre price index tends to overstate inflation, this is because

a) Both prices and quantities of current year are b) Both prices and quantities of base year
used for infilation are used as numerator for inflation.
c) Consumers reacts to price increases by d) All of these
changing what they buy

24 | P a g e
Question :9 Which of the following does not change in the calculation of Laspeyre index every year?

a) Numerator b) Denominator
c) Both d) None
Question: 10

Laspeyre price index fails to account for the following

fact that

a) People will buy less of those items have risen b) People will buy more of those items which
in prices more than others have risen in prices more than
c) It is based on quantities bought in d) All of these
current
year instead of base year.
Question 11

________ index is based on most recent quantities purchased

a) Laspeyre b) Paasche
c) Fisher d) Marshal
Question :12 The formula for buying power is:

a) Real wages b) wages


---------------- -------------
wages Real wages
c) 100 d) Both a and c
-------------
Index
Question :13 What is true about Paasche’s index?

a) It overstates inflation b) Quantity has to be calculated every year.


c) Paasche’s index understates inflation because d) Both B and C
consumer reac to price increase
Question :14 In Laspeyre price index what value is to be changed every year.

a) Price b) Quantity
c) Both d) None of these
Question :15 Laspeyre index ---------- inflation?

a) understate b) overstate
c) Index number d) None
Question :16 The barometer of commerce is:

a) Standard Deviation b) Co efficient Of variation


c) Index Number d) None of the above
Question :17 The following data represents the average monthly take-home salary of the employees of
an organization:

Year 2005 2006 2007 2008


Pay (Rs) 12,350 13,500 14,800 16,500
Price Index 110.1 122.3 137.6 160.2
Compute the real wages for each of the above years.

25 | P a g e
a) 11217, 11038, 10766, 10300 b) 11217, 11038, 10756, 10330
c) 11217, 11138, 10756, 10300 d) 11217, 11038, 10756, 10300
Also Compute the amount of pay needed in 2008 to provide buying power equal to that enjoyed in 2006

a) 10,300 b) 17,864
c) 17,684 d) None of these
Question :18 If the current year’s weighted index is 5% higher than the base year and Fisher’s Ideal

Index Number is 250, find out the Laspeyre’s Price Index Number and Paasche’s Price

Indes Number.

a) 243 and 256 b) 244 and 256


c) 243 and 255 d) None of these
Question: 19

Year 2005 2006 2007 2008 2009 2010


Wages 11,000 12,000 13,500 14,800 16,500 19,000
(Rs)
Real Wages 11,000 10,800 11,300 11,100 10,550 10,900
(Rs)
Compute the price index for the years 2006 to 2010, round to one decimal place, taking 2005 as the
base year

a) 100, 111, 119, 133, 157, 175, 191 b) 100, 111, 119, 133, 157, 174, 192
c) c) 100, 111, 120, 133, 157, 174, 191 d) 100, 111, 119, 133, 157, 174, 191
Question :20

The relation between purchasing power of money and consumer price index is:

a) Direct b) Inverse
c) can be a or b depending upon situation d) All of these
Question :21

If the price index for the year 2011 is 191.2, calculate the amount of wages whose

buying power would be the same as that of year 2007?

a) Rs 17.352 b) Rs 18,192
c) Rs 21,600 d) None of these
Question: 22

Calculate Fisher Price Index for the year 2011

a) 116.7 b) 116.5
c) 116.6 d) None of these
Question :23

If Fisher Price Index for 2012 is 110.7, calculate Paasche's Price Index for year 2012

using 2010

as base:

26 | P a g e
a) 112.27 b) 109.15
c)116 d) None of these
Question :24

Laspeyre's index number?

2002 price 2000 price 2000 quantity Pn Qo Po Qo


12 112 50 6150 5600
13
24 220 40 9760 8800
4
30 290 35 10535 10150
1
26445 24550

Question: 25

If the Fisher index is 10% more than the Laspeyre's index and base year index is 109.5,

find Paasche's index.

a. 115.6 b. 96.5
c. 100.51 d. 132.5
Question :26 Find chain index

Year Price

20 x 1 49

20x2 53

20x3 58

20x4 68

Question :27 Find Paasche Price index for given data taking Jan 1 20XX as base year to compare the
prices and quantities of Dec 1 20XX for per kg of product having prices respectively?

January 1 20XX December 1 20XX


Price for product Quantity per kg Price for product Quantity per Kg
10.49 13.1 17.99 14.2
4.99 17.1 7.49 18.2
7.39 48.9 9.39 59.2

27 | P a g e
Also find what is change occur in prices in % for Jan 1 20XX to De 1 20XX is?

a)139.925 and change in price 52.47% b)


c) d)
Question :28 Find chain index

Price index (Question) Chain index (Answer)


100 10
108.7 x 100 =
108.7
110 x 100 =
101.2
115 x 100 = 104.55
120 x 100 =
104.35
Question :29 Find index Number

2002 2003

Price Qty Price Qty

10 20 15 22

12 22 17 25

Find quantity index using 2002 as base

Laspeyre method

Paasche method

Question :30 The following data represent the average monthly take-home salary of the employees of
an organization:

Year 2005 2006 2007 2008


Pay (Rs) 12,350 13,500 14,800 16,500
Price Index 110.1 122.3 137.6 160.2
Compute the real wages for each of the above years.

a)11217,11038,10766,10300 b)11217,11038,10756,10330
c)11217,11038,10756,10300 d)11217,11138,10756,10300
Question :31 The following data represent the average monthly take-home salary of the employees of
an organization.

Year 2005 2006 2007 2008


Pay (Rs) 12,350 13,500 14,800 16,500
Price Index 110.1 122.3 137.6 160.2
Compute the real wages for the year 2005.

a)10,755.81 b)11,217.08
c)11,038.43 d)None of these

28 | P a g e
Question :32 The following data represent the average monthly take-home salary of the employees of
an organization:

Year 2005 2006 2007 2008


Pay (Rs) 12,350 13,500 14,800 16,500
Price Index 110.1 122.3 137.6 160.2
Compute the amount of pay needed in 2008 to provide buying power equal to that enjoyed in 2006.

a) Rs. 17,683.57 b) 16,387.42


c) Rs. 19,264.27 d) None of these
Question :33 Find chain index from the following data

2012 2013
106 100
100 102
100 108

a) b)
c) d)
Question :34 The income of the person in 2004 is Rs. 250,000 and cost of living index for that class of
persons is 237.5. What is the real income?

a)593,750 b)105,263.16
c)1,052.63 d)None of these
Question :35 Pn Qo = 2322 Pn Q n =2569 Po Q o =1970 Po Q n =2170

The price index by using Paasche formula is?

a)117.8 b)118.09
c)118.39 d)All of these
Question :36 Calculate the Laspeyres and the Paasche Price index of travel cost for January 2000. If
your weekly travel cost were train fares to and from work, and bus fares to and from the town center.
The cost and number per week are as follows:

Price Jan 99 Qty Jan 99 Price Jan 00 Qty Jan 00


Train fare 1.85 10 2 6
Bus fare 0.75 4 1 4

a)111.6 and 113.5 b)123.6 and 142.55


c)102.3 and 106.9 d) 101.1 and 103.2
Question : 37 Below is the price of product X per Kg in the respective years. If you see chain index
number for comparing prices, what is the chain indices of year 1979?

Year 1974 1975 1976 1977 1978 1979


Price 18 21 25 23 28 30

a)155.68 b)166.68
c)175.68 d)107.14

29 | P a g e
1/1/2022 CHAPTER 10
REGRESSION AND CORRELATION

COMPILED BY
ABDUL AHAD BUTT

[Type text]
Regression
Regression
• Simple regression is used to examine the relationship between two variables, one
dependent(y) and another independent variable (x)
• If a link can be established, an understanding of that link can support forecasting and
planning
• The aim is to identify that a change in one variable cause a change in the other variable
Example
• Height of children and age
• Number of units produced and cost
• Advertising spends and sales

• The dependent variable is also called the regressand, the predictand, the response or the
explained variable whereas the independent variable is also called the regressor, the
predictor and the explanatory variable
• After performing an analysis, the regression statistics can be used to predict the
dependent variable when the independent variable is known
Correlation

• The word correlation is used in everyday life to denote some form of association. We
might say that we have noticed a correlation between foggy days and attacks of
wheeziness (breathlessness).

• However, in statistical terms we use correlation to denote association between two


quantitative variables.

• We also assume that the association is linear, that one variable increase or decreases a
fixed amount for a unit increase or decrease in the other.

• The other technique that is often used in these circumstances is regression, which
involves estimating the best straight line to summarize the association.

Correlation coefficient
The correlation coefficient, denoted by r, is a measure of the strength of the straight-line or linear
relationship between two variables. The correlation coefficient takes on values ranging between
+1 and -1. The following points are the accepted guidelines for interpreting the correlation
coefficient:

• 0 indicates no linear relationship.


• +1 indicates a perfect positive linear relationship: as one variable increases in its values,
the other variable also increases in its values via an exact linear rule.

1|Page
• -1 indicates a perfect negative linear relationship: as one variable increases in its values,
the other variable decreases in its values via an exact linear rule.
• Values between 0 and 0.3 (0 and -0.3) indicate a weak positive (negative) linear
relationship via a shaky linear rule.
• Values between 0.3 and 0.7 (-0.3 and -0.7) indicate a moderate positive (negative) linear
relationship via a fuzzy-firm linear rule.
• Values between 0.7 and 1.0 (-0.7 and -1.0) indicate a strong positive (negative) linear
relationship via a firm linear rule.
• The value of r squared is typically taken as “the percent of variation in one variable
explained by the other variable,” or “the percent of variation shared between the two
variables.”
• Linearity Assumption. The correlation coefficient requires that the underlying
relationship between the two variables under consideration is linear. If the relationship is
known to be linear, or the observed pattern between the two variables appears to be
linear, then the correlation coefficient provides a reliable measure of the strength of the
linear relationship. If the relationship is known to be nonlinear, or the observed pattern
appears to be nonlinear, then the correlation coefficient is not useful, or at least
questionable.

2|Page
Points to remember:

1- The linear relation between a dependent and an independent variable is called regression
2- A measure of the strength or degree of relationship or the interdependence is called
correlation
3- The dependent variable is also called response or the explained variable
4- The independent variable is also called predictor or unexplained variable
5- Slope of regression line is called regression coefficient
6- In regression analysis, if the value of a is positive then value of b may take any value

Scatter diagram
7- If the values of two different variables (say x and y) are plotted on a rectangular axis, such
a plot is referred to as a scatter diagram
8- From the inspection of scatter diagram if it is seen that the points follow closely a straight
line it indicates that the two variables are to some extent linearly related
9- In a scatter diagram, if he points follow closely a straight line of positive slope, the two
variables are said to have high positive correlation
10- In a scatter diagram if the points follow clearly a straight line of negative slope, the two
variables are said to have high negative correlation
11- In scatter diagram, if the points follow a strictly random pattern, the two variables are said
to have no linear relationship
12- If two variables tend to increase (or decrease) together, the correlation is said to be direct
or positive
13- If one variable tends to increase as the other variable decrease, the correlation is said to be
inverse or negative
14- Correlation coefficient remain unchanged by added, subtracted, multiplied and divided by
any number
15- The coefficient of correlation is independent origin and scale
16- If all the plotted points in a scatter diagram lie on a single line, then the correlation is
perfect positive
17- If the plotted points in a scatter diagram lie from upper left to lower right, then the
correlation is negative
18- The correlation between shoe-size and intelligence is zero
19- Spurious(Fake/Bogus/false) correlation means no causal relation

3|Page
LINEAR REGRESSION ANALYSIS

4|Page
Formula Sheet:1
Linear regression analysis is used to calculate values for a and b in the linear cost equation
Y = a + bx

1. Regression line Y on X
Y = a + bx

2. Regression line x on y
X = a + by

The linear regression formulae for calculating a and b are shown below

3. Regression coefficient y on x (Value of b)


Syx ̅ )(Y−Y
∑(X−X ̅) n∑xy − ∑x∑y 𝐶𝑜−𝑣𝑎𝑟𝑖𝑎𝑛𝑐𝑒 (𝑥,𝑦) Sy
(i) byx = = ̅)
= = = 𝑟 (y on x)
S2 x ∑(X−X 2 n∑X2 −(∑X)2 𝑣𝑎𝑟𝑖𝑎𝑛𝑐𝑒 𝑜𝑓 𝑥 Sx

Sxy ̅ )(Y−Y
∑(X−X ̅) n∑xy − ∑x∑y Sx
(ii) bxy = = = =𝑟 (x on y)
S2y ∑(Y−Y̅) 2 n∑y2 −(∑y)2 Sy
(iv)

4. Y – intercept y on x (Value of a)
̅ - byx X
ayx = Y ̅ (y on x)
̅ - bxy Y
axy = X ̅ (x on y)

5. Covariance of x and y.
̅ )(Y− Y
∑(X− X ̅) ∑xy ∑xy ∑x ∑y
Sxy = = - x̅y̅ = −
n n n n n

6. Variance of x.
̅ )2
∑(X− X ∑x2 −nx̅2 ∑ 𝑥2 ∑ 𝑥 2
S2x= = = −[ ]
n n 𝑛 𝑛

7. Variance of y.
̅)2
∑(y− y ∑y2 −ny
̅2 ∑ 𝑦2 ∑ 𝑦 2
S2y= = = −[ ]
n n 𝑛 𝑛

5|Page
Question.1 The following data show in convenient units, the yield Y of a chemical
reaction run at various different temperatures X.

n = 7, ∑ X = 980, ∑ Y = 27.40, ∑ X Y = 3958, ∑ X2 = 140000, ∑ y2 = 1 1 5 . 5 4

Assuming that a linear regression model Y = a +bx is appropriate ,Estimate the


regression line of yield on temperature.

Solution: Here:
n= 7
∑ X = 980
∑ Y = 27.40
∑ XY = 3958
∑ X 2 = 140000
∑ Y 2 = 115.54
Y = a + bX
n∑XY− ∑X∑Y
b= n∑X2 −(∑X)2

7(3958)−(980)(27.40)
= 7(140,000)−(980)2
854
= 19600
= 0.04
∑Y− b∑X
a= 𝑛
27.40−0.04(980)
=
7
= -1.685
Hence fitted line
𝑌̂ = -1.685 + 0.04X

Question.2 Compute the regression coefficients of the following case;


̅)2 = 170, ∑ (Y − Y
n = 10, ∑ (X − X ̅)2 = 140, ∑ (X − X
̅) (Y − Y
̅) = 92

Solution: Here:
n= 10
̅) 2 = 1 7 0
∑ (X − X
∑ (Y − ̅
Y )2 = 1 4 0
∑ (X − ̅
X) (Y − ̅ Y) =92
b =?
byx =?
Regression coefficient Y on X Regression coefficient X on Y
̅ ) (Y−Y
∑(X−X ̅) ̅ ) (Y−Y
∑(X−X ̅)
𝑏𝑌𝑋 = ̅ )2
𝑏𝑋𝑌 = ̅)2
∑(X−X ∑(y−y
92 92
= 170 = 140

6|Page
= 0.54 = 0.66

Question.3 For a set of 50 pairs of observations the standard deviations of X and Y are 4.5
and 3.5 respectively. If the sum of products of deviations of X and Y values from their
respectively means be 420, find the Karl Pearson’s coefficient of correlation.

Solution: Here:
n = 50
S x = 4.5
SY = 3.5
∑(X − X) (Y − ̅
̅ Y) = 4 2 0
r=?
̅ ) (Y−Y
∑(X−X ̅)
SXY = 𝑛
420
= = 8.40
50

Coefficient of correlation:
S
r = S XY
S
x Y
8.40
= 4.5(3.5)
= 0.53

7|Page
CORRELATION

8|Page
Formula Sheet:2

1. Coefficient of correlation
n∑xy − ∑x∑y Sxy ∑(X− X̅ )(y − y
̅)
r= =S = ̅ )2 ∑(y− y
√n∑x2 − (∑x)2 √n∑y2 − (∑Y)2 x .Sy √ ∑(X− X ̅)2

2. Coefficient of correlation
r = ± √byx x bxy
Note:
(i) byx and bxy = + ve then r =+ ve
(ii) byx and bxy = -ve then r = -ve
(iii) Coffient of correlation is pure number, it does not dependent upon the unit of
measurement

Question.1 The two regression coefficients have following value, bYX = 0.86, bxy= 0.95 find
r.

Solution: Here:
byx = 0.86
bxy = 0.95
r=?

Coefficient of correlation:
r = + √bYX x bXY
= + √0.86 x 0.95
= +0.90

Question.2 Find the coefficient of correlation if the two regression coefficients have the
following values -0.1 and -0.4

Solution: Here:
bYx = -0.1
bXY = -0.4
r=?

Coefficient of correlation:
r = + √bYX x bXY
= - √(−0.1)𝑥 (−0.4) = -0.2

Question.3 The following are the results are given r = 0.60, S2 x = 9, bYx = 0.80, find SY.
9|Page
Solution: Here:
r = 0.60
S2x = 9
Sx = √9
=3
bYx = 0.80
SY = ?
S
bYX = r SY
X
SY
0.80 = 0.60 3
3(0.80)
= SY
0.60
4 = SY

Question.4 For a given set of data, we have


r = 0.48, S2 x = 16, SXY = 36, find SY.

Solution: Here:
r = 0.48
S2x = 16
SY =?

Coefficient of correlation:
SXY
r=
Sx Sy
36
0.48 = 4 Sy

SY = 9/0.48
= 18.75

10 | P a g e
Spearman’s Rank Correlation
Simple correlation coefficient is used when original values of variables are used.
Rank correlation is an alternative method of measuring correlation base on the ranks of values.

Therefore, the correlation between the ranks of two variables is called rank correlation. Ranks of
‘x’ values and ranks of ‘y’ values are performed quite independently and values are ranked in
either way i.e. ascending or descending order.
6 ∑𝑑 2
r = 1 - 𝑛 ( 𝑛 2 −1 ) where d = difference rank

Question.1 Rank of the equation is given


Judge x 1 2 3 4 5 6 7 8
Judge y 6 5 1 4 3 2 8 7

Find spearman’s rank correlation


Solution:
X Y d = x-y 𝑑2
1 6 -5 25
2 5 -3 9
3 1 2 4
4 4 0 0
5 3 2 4
6 2 4 16
7 8 -1 1
8 7 1 1
∑ 𝑑 2 = 60
6 ∑ 𝑑2
r = 1- 𝑛 ( 𝑛2 −1 )

6 (60) 360
r = 1- 8 [64−1 ] = 1- 504

r = 0.2857
Question.2 (When the Ranks are not given)
Ten students got the following percentage of marks in mathematics and physics.
Mathematics 8 36 98 25 75 82 92 62 65 35
x
Physics y 84 51 91 60 68 62 86 58 35 49
Find rank correlation

11 | P a g e
Solution
We first assign ranks to x and y by giving rank 1 to the highest value in both series.
X Y Ranks d = x-y d2
x Y
8 84 10 3 7 49
36 51 7 8 -1 1
98 91 1 1 0 0
25 60 9 6 3 9
75 68 4 4 0 0
82 62 3 5 -2 4
92 86 2 2 0 0
62 58 6 7 -1 1
65 35 5 10 -5 25
35 49 8 9 -1 1
∑ 𝐷2 = 90

6 ∑ 𝑑2
r = 1 - 𝑛 ( 𝑛2−1 )

6 (90)
r = 1 - 10 (100−1)

r = 0.545
Note: if values are repeated use average of rank number)
Question.3 Find the coefficient of rank correlation for the following data
A 35 40 42 43 40 53 54 49 41 55
B 102 101 97 98 38 101 97 92 95 95
Solution:
A B Ranks d = A-B d2
A B
35 102 10 1 9 81
40 101 8.5 2.5 6 36
42 97 6 5.5 0.5 0.25
43 98 5 4 1 1
40 38 8.5 10 -1.5 2.25
53 101 3 2.5 0.5 0.25
54 97 2 5.5 -3.5 12.25
49 92 4 9 -5 25
41 95 7 7.5 -0.5 0.25
55 95 1 7.5 -6.5 42.25
∑ 𝑑2 =
200.50

12 | P a g e
∑𝑑2
r =1 - 𝑛(𝑛2−1)

6(200.50)
r = 1 - 10(102 −1)

1203
r=1- 990

r = 1 – 1.215 = -0.215

Probable error and Interpretation


The probable error is calculated to avoid any wrong conclusion. It is given by the formula
1− 𝑟 2
P.E = ±0.6745 ( )
√𝑛

(i) If r < P.E there is no correlation


(ii) If r > P.E Correlation exists
(iii) If r > 6 * P.E highly significant correlation

13 | P a g e
EXTRA PRACTICE QUESTION

14 | P a g e
Question.1 Suppose that four randomly chosen plots where treated with various level of
fertilizer resulting in the following yields of corn.
Fertilizer(kg/Acre) X 100 200 400 500
Production (Bushels/Acre) Y 70 70 80 100

(i) Estimate the linear regression 𝝁𝒀|𝑿 = α + βX of production Y on fertilizer X


(ii) Estimate the yield when no fertilizer is applied.
(iii) Estimate the yield when the average amount of fertilizer is applied.
(iv) Estimate how much yield is increased for every kilogram of fertilizer

Solution: (i)
X Y X XY
100 70 10000 7000
200 70 40000 14000
400 80 160000 32000
500 100 250000 50000
1200 320 460000 103000

Regression line:
Y = a + bX
n∑XY− ∑X∑Y
byx = n∑X2 −(∑X)2
4(103000) − (1200)(320)
= 4(460000)−(1200)2
28000
= 400000

= 0.07
∑Y− b∑X
a= n
320−0.07(1200)
= 4
= 59
Hence fitted line:
𝑌̂= 59 + 0.07X
(ii) For X = 0
𝑌̂= 59 + 0.07(0)
= 59

∑X 1200
(iii) ̅ =
For X = X = = 300
n 4

𝑌̂= 59 + 0.07(300)
= 80
(iv) 0.07 bushels per kg fertilizer

15 | P a g e
Question.2
The following are the measurements of height and weight of 8 men.
Height(inches) X 78 89 97 69 59 79 68 61
Weight (Pound) Y 125 137 156 112 107 136 123 104

(i) Calculate the correlation coefficient between the height and weight of eight men by
using deviations from their means.

Solution:
X Y (X-X ̅) (Y -Y ̅) (X-X ̅)2 (Y- Y̅)2 ̅) ( Y - Y
(X-X ̅)
78 125 3 0 9 0 0
89 137 14 12 196 144 168
97 156 22 31 484 961 682
69 112 -6 -13 36 169 78
59 107 -16 -18 256 324 288
79 136 4 11 16 121 44
68 123 -7 -2 49 4 14
61 104 -14 -21 196 441 294
600 1000 0 0 1242 2164 1568

∑X ∑Y
̅
X= n ̅
Y= n
600 1000
= 8 = 8
= 75 = 125
Coefficient of correlation:
̅ ) (Y−Y
∑(X−X ̅)
𝑟XY = ̅ ) ∑(Y−Y
2 ̅ )2
√∑(X−X

1568
=
√(1242)(2164)

1568
= 1639.4

= 0.956
Question.3 From the data given below calculate the coefficient of correlation between the
ages of husbands and ages of wives at the time of their marriage.
Couple i 1 2 3 4 5 6 7 8 9 10
Husband’s age X 28 27 28 23 29 30 36 35 33 31
Wife’s age Y 27 20 22 18 21 29 29 28 29 27
Find the regression coefficients. Verify that r is the geometric mean of the two
regression coefficient.

16 | P a g e
Solution:
X Y X2 Y2 XY
28 27 784 729 756
27 20 729 400 540
28 22 784 484 616
23 18 529 324 414
29 21 841 441 609
30 29 900 841 870
36 29 1296 841 1044
35 28 1225 784 980
33 29 1089 841 957
31 27 961 729 837
300 250 9138 6414 7623

Regression coefficient Y on X Regression coefficient X on


Y
Y = aYX + bYXX X = axy + bXYY
n∑XY− ∑X∑Y n∑XY− ∑X∑Y
𝑏𝑌𝑋 = n∑𝑋 2− (∑X)2 𝑏𝑋𝑌 = n∑𝑌 2 − (∑Y)2
10(7623)−(300)(250) 10(7623)−(300)(250)
= =
10(9138)− (300)2 10(6414)− (250)2
1230 1230
= 1380 = 0.89 = 1640 = 0.75

Coefficient of correlation:
n∑XY− ∑X∑Y 10(7623)−(300)(250)
r= =
√n∑X2 −(∑X )2 √n∑y2 −(∑y )2 √(10(9138 )− (300)2 )(10(6414) − (250)2 )

1230
=
√(1380)(1640)
1230
= 1504.39 = 0.82

Verification:
r = + √bYX x bXy

0.82 = √(0.89)x (0.75)


0.82 = 0.82
L.H.S = R.H.S

17 | P a g e
Question.4 The following data were obtained for a sample of 10 men from a height and
weight distribution.

̅
X = 70, ̅
Y = 155, ∑ (X − ̅X)2 = 120, ∑ Y 2 = 240550, ∑ (X − ̅
X) (Y − ̅
Y) = 150
Calculate covariance, correlation coefficient the two regression lines.

Solution: Here:
n= 10
̅ = 70
X
̅
Y = 155
∑ (X − ̅
X)2 = 1 2 0
∑Y2=240550
∑ (X − ̅
X) (Y − ̅ Y) = 150

̅ ) (Y−Y
∑(X−X ̅)
𝑆𝑋𝑌 = n
150
= = 15
10

̅ )2
∑(X−X ∑Y ̅2
2 −nY
SX = √ SY = √
𝑛 n

120 240550− 10(155)2


= √ 10 = √ n

= 3.45 = 5.48

Coefficient of correlation:
S
r= S XY
S
X Y
5
= 3.46(5.48) = 0.79

Regression Equations Y on X Regression Equations X on Y


SY S
(Y − ̅
Y) = r (X − ̅
X) (X − ̅
X) = r SX (Y − ̅
Y)
SX Y

5.48 3.46
Y – 155 = 0.79 3.46 (X - 70) X – 70 = 0.79 5.48 (Y - 155)

Y – 155 = 1.25(X - 70) X – 70 = 0.50(Y - 155)


Y – 155 = 1.25X – 87.50 X – 70 = 0.50Y – 77.50
Y = 155 + 125X – 87.50 X = 70 + 0.50Y – 77.50
𝑌̂ = 67.50 + 1.25X 𝑋̂ = -7.50 + 0.50Y

18 | P a g e
Question.5 For 8 pairs of observations ∑ 𝒙 = 𝟒𝟖 , ∑ 𝒚 = 𝟔𝟒, ∑(𝒙 − 𝟔)(𝒚 − 𝟖) = 𝟐𝟔,
∑(𝒚 − 𝟖)𝟐 = 𝟐𝟎 , ∑(𝒙 − 𝟔)𝟐 = 𝟒𝟎 what is the value of coefficient of non-determination.
𝑆𝑥𝑦
Solution: 𝑟 = 𝑆𝑥−𝑆𝑦
26
8
𝑟 = 20 40
√ .√
8 8

𝑟 = 0.92
𝑟 2 = 0.845 (𝑒𝑥𝑝𝑙𝑎𝑖𝑛𝑒𝑑)
1 − 𝑟 2 = 0.15 (𝑢𝑛𝑒𝑥𝑝𝑙𝑎𝑖𝑛𝑒𝑑)
Question.6 The two regression lines are 𝟑𝒙 + 𝟐𝒚 = 𝟏𝟗 and 𝟐𝒙 + 𝟑𝒚 = 𝟏𝟔, then
arithmetic mean of x is

Solution:
3𝑥 + 2𝑦 = 19 … … … … . (𝑖)
2𝑥 + 3𝑦 = 16 … … … … . (𝑖𝑖)
______________
6𝑥 + 4𝑦 = 38
-6𝑥 ± 9𝑦 = 48
−5Ȳ = −10
Ȳ=𝟐
Put in (i)
3𝑥 + 2(2) = 19
3𝑥 = 15
𝒙=𝟓

Question.7 𝒚 = −𝟏. 𝟗𝟔𝒙 + 𝟏𝟓(𝒚 𝒐𝒏 𝒙)


𝒙 = −𝟎. 𝟒𝟓𝒚 + 𝟕. 𝟏𝟔(𝒙 𝒐𝒏 𝒚)
Find the coefficient of determination in each case
Solution:

19 | P a g e
𝑏𝑦𝑥 = −1.96
𝑏𝑥𝑦 = −0.45

𝑟 = −√(1.96)(0.45) = -0.93

𝑟 = −√0.882
𝒓𝟐 = 𝟎. 𝟖𝟖𝟐
Question.8 𝒚 = 𝟒𝟓 − 𝟑𝒙 is the regression line of y on x. What number of units is expected
to increase in y if x is decreased by two units?
Solution:
𝑦 = 45 − 3𝑥
𝑦 = 45 − 3(−2)
𝑦 = 45 + 6

Question.9 In an effort to reduce crime, the S.P of far town has requested the inspector
general to increase police strength in his town of the city gathered information from other
towns of the city and submitted the following details support his request.
Towns Bee Cee Dee Gee Jay Kay Pee Tee
Police 150 170 250 270 170 120 110 220
stations
No. of 170 110 50 40 90 210 188 60
crimes
per
month
Required:
i. Determine the regression equation and interpret your result (Assuming that
ratio of police strength to total number of people is same an all towns)
ii. Determine the coefficient of correlation and determine and interpret your
results.
iii. Using the above regression equation, determine whether police of joy town is
more efficient than police of pee town.

20 | P a g e
Solution:
(i)
X Y XY 𝑿𝟐 𝒀𝟐
150 170 25500 22500 28900
170 110 18700 28900 12100
250 50 12500 62500 2500
270 40 10800 72900 1600
170 90 15300 28900 8100
120 210 25200 14400 44100
110 188 20680 12100 35344
220 60 13200 48400 3600
1460 918 141880 190880 13624

Regression line of Y on X is
Y = a+bx …………………………..1
Where
∑ 𝑥𝑦 ∑ 𝑥 ∑ 𝑦
𝑠𝑦𝑥 ( )( )
𝑛 𝑛 𝑛
𝑏𝑦𝑥 = = ∑ 𝑥2 ∑𝑥 2
𝑠𝑥 −(( ))
𝑛 𝑛

141880 918 1460


−( )( )
8 8 8
𝑆𝑦𝑥 = 290600 1460 2
−(( ))
8 8

−3206.875
= 3018.75

𝑏𝑦𝑥 = −1.0623

Also
̅-bX
ayx = Y ̅

a = 114.75+1.06(182.25)
a = 308.62
Put in (1)
Y = 308.62-1.0623X
Interpretations:
By increasing police strength no of crimes will decrease.

21 | P a g e
(ii)
∑ 𝑥𝑦 ∑ 𝑥 ∑ 𝑦
𝑠𝑥𝑦 ( )( )
𝑛 𝑛 𝑛
𝑏𝑦𝑥 = 𝑠2 𝑦 = ∑ 𝑦2 ∑𝑦 2
−(( ))
𝑛 𝑛

−3206.875
=
3018.75
= -0.83
Now

𝒓 = √𝒃𝒙𝒚 × 𝒃𝒚𝒙

𝒓 = √(−𝟎. 𝟖𝟑)(−𝟏. 𝟎𝟔)

= −𝟎. 𝟗𝟑𝟕𝟗
Also coefficient of determination = 𝑟 2
𝑟 2 = (−0.9379)2
𝑟2 = 87.97%
Interpretation: 87.97% decrease in no of crimes Is due to increase in police strength. The
rest 12.03% is due to some other factors
(iii) for Pee town
Y=308.62 – 1.06(110) = 192.02
Also
for jay town
Y=308.62 – 1.06(170) = 180.2

So jay town police is more efficient than pee town.


Question.10 Find the coefficient of correlation between 𝒙 and 𝒚 if regression line of x on y
is 𝟓𝒙 = 𝟒𝒚 + 𝟐 = 𝟎 And regression line of y on x is 𝒙 − 𝟓𝒚 + 𝟑 = 𝟎
Solution: 𝟓𝒙 = 𝟒𝒚 − 𝟐
4 2
𝑋 = 5𝑦 − 5
4
𝑏𝑦𝑥 = 5 = 0.8

22 | P a g e
Also
5𝑦 = 𝑥 + 3
1 3
𝑦 = 5𝑥 + 5
1
𝑏𝑦𝑥 = 5 = 0.2

Now

𝑟 = √𝑏𝑥𝑦 × 𝑏𝑦𝑥

𝑟 = √0.8 × 0.2
𝑟 = 0.4
Question.11
If x denotes advertising expenditure in one period and y denotes sales revenue in the next
period. The regression equation linking x and y has been found to y=40+10x , interpret the
value
Solution:
if nothing is spent on advertising, sales will be 40 on average
Question.12
20 pairs of values of (x,y) with x ranging from 15 to 45 were used to obtain. The regression
equation y 480 -5x , The correlation coefficient is -0.95 , it has been estimated that when x -
10, y=430, which of the following reduces the reliability of the estimate.
Solution
Sample size

23 | P a g e
CHAPTER 13: REGRESSION AND CORRELATION
1- If the values of two different variables (say x and y) are plotted on a rectangular axis, such a
plot is referred to as a:

A) Frequency diagram B) Value diagram


C) Scatter diagram D) None of these
2- From the inspection of scatter diagram if it is seen that the points follow closely a straight line,
it indicates that the two variables are to some extent:

A) Unrelated B) Related
C) Linearly related D) None of these
3- In a scatter diagram, if the points follow closely a straight line of positive slope, the two
variables are said to have:

A) No correlation B) High positive correction


C) Negative correlation D) None of these
4- In a scatter diagram, if the points follow clearly a straight line of negative slope, the two
variables are said to have:

A) No correlation B) High positive correlation


C) High negative correlation D) None of these
5- In a scatter diagram, if the points follow a strictly random pattern, the two variables are said to
have:

A) No linear relationship B) Low positive relationship


C) Low negative relationship D) None of these
6- A measure of the strength or degree of relationship or the interdependence is called:

A) Correlation B) Regression
C) Least square estimate D) None of these
7- The phenomenon that investigates the dependence of one variable on one or more independent
variables is called:

A) Correlation B) Regression
C) Least square estimate D) None of these
8- The linear relation between a dependent and an independent variable is called:

A) Regression line B) Regression co-efficient


C) Co-efficient of correlation D) None of these
9- Slope of the regression line is called:

A) Regression parameter B) Sample parameter


C) Regression co-efficient D) None of these
10 In regression analysis, if the value of a is positive the value of b:
A) Must be positive B) May take any value

24 | P a g e
C) Must be negative D) Less than -1or more than 1
11- The procedure which selects that particular line for which the sum of the squares of the
vertical distances from the observed points to the line is as small as possible, is called:
A) Sum of squares method B) Sum of squares of errors method
C) Least square method D) None of these
12- The numerical values of regression co-efficient must be:

A) Both positive B) Both negative


C) Both positive or both negative D) None of these
13- In regression, the dependent variable is assumed to be a random variable whereas the
independent variable is assumed to have:

A) Random values B) Fixed values


C) Both (a) or (b) D) None of these
14- The dependent variable is also called response or:

A) The explained variable B) Unexplained variable


C) The explanatory variable D) None of these
15- Which of the statements about Spearman’s Co-efficient of Correlation is NOT correct:

A) It can co-relate two or more set of rankings B) It applies only when no ties exist
C) Both (a) and (b) D) None of the above
16- The explained variable or response is also called:

A) The independent variable B) The dependent variable


C) Non-random variable D) None of these
17- The predictor or unexplained variable is also called:
A) The independent variable B) The dependent variable
C) Random variable D) None of these
18- In regression analysis, b = 2.8, indicates that the value of dependent variable:

A) Increases by 2.8 units at per unit B) Decreases by 2.8 units at per unit increase
increase in independent variable in independent variable
C) Increases by 2.8 units at per unit decrease D) None of these
in independent variable
19- If Y is the observed value and 𝑌 ̂ is the estimated value (estimated by using the regression
line) then Σ(𝑌 −𝑌 ̂ ):

A) Should be zero B) Is likely to be close to zero


C) In majority of the cases would be equal to D) None of these
zero
20-If two variables tends to vary simultaneously in some direction, they are said to be:

A) Dependent B) Independent
C) Correlated D) None of these

25 | P a g e
21- If two variable tends to increase (or decrease) together, the correlation is said to be:

A) Zero B) Direct or positive


C) 1 D) None of these
22- If one variable tends to increase as the other variable decreases, the correlation is said to be:

A) Zero B) Inverse or negative


C) -1 D) None of these
23- While calculating “r” if x and y are interchanged i.e. instead of calculating 𝑟𝑥𝑦 if 𝑟𝑦𝑥 is
calculated then:

A) 𝑟𝑥𝑦 = 𝑟𝑦𝑥 B) 𝑟𝑥𝑦 > 𝑟𝑦𝑥


C) 𝑟𝑥𝑦 < 𝑟𝑦𝑥 D) None of these
24- Limits of the co-efficient of Correlation are:

A) -1 to 0 B) 0 to 1
C) 1- to +1 D) None of these
25- If r = 0.9 and if 5 is subtracted from each observation of x, then r will:

A) Decrease by 5 units B) Decreases by less than 5 units


C) Remain unchanged D) None of these
26- If r = 0.9 and if 5 is added to each observation of x, then r will:

A) Increase by 5 units B) Increase by more than 5 units


C) Remain unchanged D) None of these
27- If r = 0.9 and if 3 is subtracted from each observation of Y, then r will:

A) Decrease by 3 units B) Decrease by less than 3 units


C) Remain unchanged D) None of these
28- If r = 0.9 and if 3 is added to each observation of y, then r will:

A) Increase by 3 units B) Increase by more than 3 units


C) Remain unchanged D) None of these
29- If r = 0.9 and if 3 is subtracted from each observation of x and 5 is added to each observation
of y, then r will:

A) Decrease by 2 units B) Increase by 2 units


C) Remain unchanged D) None of these
30- If r = 0.9 and each observation of x is multiplied by 100, then r will:

A) Increase by 100 times B) Less than 100 times


C) Remain unchanged D) None of these
31- If r = 0.9 and each observation of Y is divided by 10, then r will:

A) Decrease by 10 times B) Decrease by less than 10 times

26 | P a g e
C) Remain unchanged D) None of these
32- If r = 0.9 and each observation of x and y is divided by 10, then r will:

A) Decrease by 10 times B) Decrease by 100 times


C) Remain unchanged D) None of these
33- The co-efficient of correlation is independent of:

A) Only origin B) Only scale


C) Origin and scale D) None of these

34- The geometric mean of two regressions co-efficient is equal to:

A) Co-efficient of determination B) Co-efficient of correlation


C) Co-efficient of rank correlation D) None of these
35- If 𝑏𝑥𝑦 = -0.78 and 𝑏𝑦𝑥 = -0.45, then r is equal to:

A) +0.351 B) -0.351
C) Cannot be determined D) None of these
36- If 𝑏𝑥𝑦 = -0.78 and 𝑏𝑦𝑥 = 0.45, then r is equal to:

A) +0.351 B) -0.351
C) Cannot be determined D) None of these
37- If 𝑏𝑥𝑦 = +1.93 and 𝑏𝑦𝑥 = 0.6, then r is equal to:

A) 1.158 B) 1.0761
C) Data is fictitious D) None of these
38- If 𝑏𝑥𝑦 = 1.93 and 𝑏𝑦𝑥 = 0.51, then r is equal to:

A) 0.9843 B) 0.992
C) Data is fictitious D) None of these
39- If 𝑏𝑥𝑦 = -1.93 and 𝑏𝑦𝑥 = 0.51, then r is equal to:

A) -0.9843 B) -0.992
C) Data is fictitious D) None of these
40- If Σ 𝑥𝑖= 68, Σ 𝑦 = 112, Σ 𝑥𝑦 = 1292, Σ 𝑥2 = 786, Σ 𝑦2 = 2128 then r is equal to:
6𝑖
A) 0.947 B) 0.8968
C) Cannot be determined D) None of these
41-If 𝑆𝑥𝑦 = 2166.67, 𝑆𝑥 = 115.47, 𝑆𝑦 = 19.1485 then r is equal to:

A) 0.9799 B) 0.9899
C) -0.9799 D) None of these
42- The co-efficient of correlation can never be:

A) Negative B) Positive

27 | P a g e
C) Zero D) Can assume any value
43- If 𝑆𝑥𝑦 = 0.6, 𝑆𝑥 = 4.1 and 𝑆𝑦 = 0.23 then r is equal to:

A) 0.7976 B) 0.636
C) 0.4048 D) None of these
44- The square of r is known as:

A) Co-efficient of correlation B) Co-efficient of regression


C) Co-efficient of determination D) None of these
45- The lower and upper limits of 𝑟2 are:

A) -1 to + 1 B) 0 to 1
C) -∞ to +∞ D) None of these
46- The quantity which describes that the proportion (or percentage) of variation in the
dependent variable explained (or reduced) by the independent variable is called:

A) Co-efficient of determination B) Co-efficient of regression


C) Co-efficient of correlation D) None of these
47- If r = 0.8, then the variation in the dependent variable y due to independent variable x is
about:
A) 80% B) 64%
C) 64% to 80% D) None of these
48- If r = 0.8 and 𝑏𝑦𝑥= 1.04 then 𝑏𝑥𝑦 is equal to:
A) 0.769 B) 0.615
C) Cannot be determined D) None of these
49- If r2 = 0.796 and 𝑏𝑥𝑦 = -1.04 then 𝑏𝑦𝑥 is equal to:
A) 0.765 B) -0.765
C) Cannot be determined D) None of these
50- Correlation analysis is aim at:
A) Predicting one variable for a given value B) Establishing relation between two variable
of the other variable
C) Measuring the extent of relation between D) Both B & C
two variables
51- Regression analysis is concerned with:
A) Establishing mathematical relationship B) Measuring the extent of association
between two variables between two variables
C) Predicting the value of the dependent D) Both A & C
variable for a given value of the independent
variable
52- What is spurious correlation?
A) It is bad relation between two variables B) it is very low correlation between two
variables
C) It is the correlation between two D) It is negative correlation
variables having no casual relation

28 | P a g e
53- Scatter diagram is considered for measuring:
A) Linear relationship between two variables B) Curvilinear relationship between two
variables
C) Neither A or B D) Both A & B
54- if the plotted points in a scatter diagram lie from upper left or lower right, then the
correlation is:
A) Positive B) Zero
C) Negative D) None of these
55- If the plotted points in a scatter diagram are evenly distributed, then the correlation is:
A) Zero B) negative
C) Positive D) None of these
56- If all the plotted points In a scatter diagram lie on a single line, then the correlation is:
A) Perfect positive B) Perfect negative
C) Both A & B D) Either A or B
57- The correlation between shoe-size and intelligent is:
A) Zero B) Positive
C) Negative D) None of these
58- The correlation between the speed of an automobile and the distance travelled by it after
applying the brakes is:
A) Negative B) Zero
C) Positive D) None of these
59- Scattered diagram helps us to:
A) find the nature correlation between two B) Compute the extent of correlation
variables between two variables
C) Obtain the mathematical relationship D) Both A & C
between two variables
60- Pearson’s correlation is used to for finding:
A) Correlation for any type of relation B) Correlation for linear relation only
C) Correlation for curvilinear relation only D) Both A & C
61- Product moment correlation coefficient is considered for:
A) Finding the nature of correlation B) Finding the amount of correlation
C) Both A & B D) Either A & B
62- If the value of correlation coefficient is positive, then the points in a scatter diagram tend to
cluster:
A) From lower left corner to upper right B) From lower left corner to lower right
corner corner
C) From lower right corner to upper left D) From lower right corner to upper right
corner corner
63- Product moment correlation coefficient may be defined as the ratio of:
A) The product of standard deviations of the B)The covariance between the variables to the
two variables to the covariance between them product of the variance of them
C) The covariance between the variables to D) Either B or C
the product of their standard deviations
64- The covariance between two variables is:
A) Strictly positive B) Strictly negative

29 | P a g e
C) Always 0 D) Either positive or negative or zero
65- Which of the following is NOT a possible value of the correlation coefficient?
A) Negative 0.9 B) Zero
C) positive 0.15 D) Positive 1.5
66- The coefficient of correlation between two variables:
A) can have any unit B) Is expressed as the product of units of the
two variables
C) Is a unit free measure D) None of these
67- What are the limits of the correlation coefficient?
A) No limit B) -1 and 1
C) 0 and 1, including the limits D) -1 and 1, including the limits
68- For finding correlation between two attributes, we consider:
A) Person’s correlation coefficient B) scatter diagram
C) Spearman’s rank correlation coefficient D) Coefficient of concurrent deviations
69- For finding the degree of agreement about beauty, between two judges in a Beauty Contest,
we use:
A) Scatter diagram B) Coefficient of rank correlation
C) Coefficient of correlation D) Coefficient of concurrent deviation
70- If there is a perfect disagreement between the marks in Geography and Statistics, then what
would be the value of rank correlation coefficient?
A) Any value B) Only 1
C) Only -1 D) B or C
71- When we are not concerned with the magnitude of the two variables under discussion, we
consider:
A) Rank correlation coefficient B) Product moment correlation coefficient
C) Coefficient of concurrent deviation D) A or B but not c
72- What is the quickest method to find correlation between two variables?
A) Scatter diagram B) Method of concurrent deviation
C) Method of rank correlation D) Method of product moment correlation
73- What are the limits of the coefficient of concurrent deviations?
A) no limit B) Between -1 and 0, including the limiting
values
C) Between 0 and 1, including the limiting D) Between -1 and 1, the limiting values
values inclusive
74- If there are two variables x and y, then the number of regression could be:
A) 1 B) 2
C) Any number D) 3
75- Since Blood Pressure of a person depends on age, we need consider:
A) The regression equation of Blood B) The regression of age on Blood Pressure
Pressure on age
C) Both A & B D) Either A or B
76- The method applied for deriving the regression equations is known as:
A) Least squares B) Concurrent deviation
C) Product moment D) Normal equation

30 | P a g e
77- The difference between the observed value and the estimated value in regression analysis is
known as:
A) Error B) Residue
C) Deviation D) A or B
78- The errors in case of regression equations are:
A) Positive B) Negative
C) Zero D) All of these
79- The regression line of y on is derived by:
A) The minimization of vertical distances in B) The minimization of horizontal distances
the scatter diagram in the scatter diagram
C) Both A and B D) A or B
80- The two lines of regression become identical when:
A) r = 1 B) r = -1
C) r = 0 D) A or B
81- What are the limits of the two regression coefficients?
A) No limit B) Must be positive
C) One positive and the other negative D) Product of the regression coefficient
must be numerically less than unity
82- The regression coefficients remain unchanged due to a:
A) Shift of origin B) Shift of scale
C) Both A and B D) A or B
83- The correlation between the ages of husbands and wives is:
A) Positive B) Negative
C) Might be any D) None of these
84- The correlation between the amount of rubber on tires and the number of miles they have
been driven:
A) Positive B) Negative
C) Might be any D) None of these
85- The correlation between Shoes size and IQ:
A) Positive B) Negative
C) Might be any D) None of these
86- Interpret your result:
A) There is a positive but weak relationship B) There is a positive but strong relationship
between rank performance in Accounting between rank performance in Accounting and
and Mathematics Mathematics
C) There is a positive but linear relationship D) Both A and C
between rank performance in Accounting and
Mathematics
87- Interpret the results of co-efficient of correlation:
A) Strong direct relation between strength of B) Strong inverse relation between strength
police and crime rate of police and crime rate
C) Average direct relation between strength D) Average inverse relation between strength
of police and crime rate of police and crime rate
89- If the coefficient of determination is positive then ‘r’:
A) Must be positive B) Must be negative

31 | P a g e
C) Might be any D) None of these
90- Bivariate are the data collected for:
A) One variable B) Two variables at different point of time
C) Two variables at some point of time D) None of these
91- Correlation analysis is used to:
A) Predict one variable for a given value of B) Establish relationship between two
other variable variables
C) Measure the extent of relation between D) Both B or C
two variables
92- If the plotted points in a scatter diagram lie from lower left to upper right, then correlation is:
A) Negative B) Positive
C) Perfect negative D) Perfect positive
93- Sign of product moment correlation co-efficient depends on:
A) Variance of X B) variance of Y
C) Co-variance D) Product of two standard deviation
94- Karl-Pearson’s Correlation is the ration of:
A) Two variances B) The product of standard deviation of two
variables to the covariance
C) The covariance to the product of D) The covariance to the product of variance
standard deviations of two variables of two variables
95- Which of the following method take magnitude of observation into account:
A) Scatter diagram B) Correlation
C) Rank correlation D) All of these
96- The difference between actual and estimated value in regression analysis is known as:
A) y- intercept B) Error
C) Slope D) None of these
97- Which of the following represents the proportion of variation in dependent variable that is
explained by the independent variable?
A) Co-efficient of determination B) Co-efficient of correlation
C) Regression co-efficient D) None of these
98- As the angle between two regression lines increases the correlation co-efficient:
A) Remains same B) increases
C) Decreases D) None of these
99- The independent variable is also called:
A) Regressor B) Predictor
C) Regression D) All of these
100- In regression problem, the independent and dependent variables are:
A) Both fixed B) Both random
C) Independent variable fixed & dependent D) Dependent variable fixed & independent
variable random variable random
101- if a constant amount of change in the predicted variable is associated with a unit change in
the predicting variable the relation is said to be:
A) Linear B) Non linear
C) Inverse D) None of these

32 | P a g e
102- The two regression co-efficient always have:
A) Opposite signs B) Same signs
C) Not definite D) No signs
103- The two lines of regression are perpendicular to each other when:
A) r = 0 B) r = 1/3
C) r = -1/2 D) r = ±1
104- The co-efficient of correlation between variables height (in inches) and weight (in pounds)
will be expressed in:
A) Pounds B) Inches
C) Both A & B D) As a pure number
105- If both regression co-efficient are negative then correlation co-efficient will be:
A) Negative B) Positive
C) Zero D) 1
106- In case of correlation, both variables x and y are assumed to be:
A) Fixed B) Unknown
C) Random D) None
107- The regression line of y on x is derived by:
A) Minimizing the horizontal distances is B) Maximizing the horizontal distances in
scatter diagram scatter diagram
C) Minimizing the vertical distances in D) Maximizing the vertical distances in
scatter diagram scatter diagram
108- Which of the following is true when the slope of a regression line is positive?
A) Correlation co-efficient between the B) The regression line is parallel to the
independent and independent variable is 1 horizontal line
C) There is positive correlation between D) None of these
the dependent and independent variables
109- The regression co-efficient in independent of change of:
A) Origin B) Scale
C) Both D) None of these
110- Regression equation is also called:
A) Predicting equation B) Estimating equation
C) Line of average relationship D) All A,B and C
111- The regression line of x on y derived by method of least square:
A) Minimizes the horizontal distances in B) Maximizes the vertical distances in scatter
scatter diagram diagram
C) Minimizes the vertical distances in scatter D) Maximizes the horizontal distances in
diagram scatter diagram
112- The correlation coefficient between two variables:
A) is a unit free measure B) Is expressed as product of units of two
variables
C) Is expressed in units of first variable D) Is expressed in units of second variable
113- Correlation co-efficient is independent of change of origin:
A) Is always false B) Is always true
C) can be false D) Can be true

33 | P a g e
114- In rank correlation the association should be linear:
A) False B) True
C) A & B D) None of these
115. If 𝑋̅ = 11.33; 𝑌̅ =33.56 and b y x = 2.832 then a equal is to:

(a)0.96 (b)1.47
(c)11.85 (d)4.00
116. If a random sample of 9 observation yielded the values ∑ x = 102, ∑ y =302, ∑ xy =3583
2
and ∑ x =1308 then the value of b is:

(a)1.47 (b)2.831
(c)Cannot be determined (d)None of these
117. For the following pair of values compute correlation coefficient (1,3), (4,8), (8,17), (10,18)

(a)0.9887 (b)0.9775
(c)1.7743 (d)0.5510
118. Find the following pair of values compute coefficient of determination (1,3), (4,8), (8,17),
(10,18)

(a)0.9887 (b)0.9775
(c)1.7743 (d)0.5510
119. For the following pair of values compute equation of least squares line of Y on X (1,3),
(4,8), (8,17), (10,18)

(a)y= 1.297 +1.774x (b)x= 0.5860 – 0.55y


(c)y= 1.774 + 1.297x (d)x= 0.55 - 0.5860y
120. For the following pair of values compute equation of least squares line of X on Y (1,3),
(4,8), (8,17), (10,18)

(a)y=1.297 + 1.774x (b)x = - 0.5860+0.55y


(c)y=1.774 + 1.297x (d)x = 0.55 – 0.5860y
121. For the following pair of values compute regression coefficient of Y on X (1,3), (4,8),
(8,17), (10,18)

(a)0.9887 (b)0.9775
(c)1.7743 (d)0.5510
122. For the following pair of values compute regression coefficient of X on Y (1,3), (4,8),
(8,17), (10,18)

(a)0.9887 (b)0.9775
(c)1.7743 (d)0.5510
123. Select one or more of the possible correct statements:

(a)Correlation may be direct or positive (b)Correlation must be direct or positive


(c)Correlation may be inverse or negative (d)Correlation coefficient ranges from 0 to 1

34 | P a g e
124. Select one or more of the possible correct statements:

(a)A positive relationship between two (b)Correlation coefficient is not expressed in


variables means that high values of one the units of measurement from which it is
variable are paired with high values of the obtained
other.
(c)Regression coefficient ranges between -2 (d)Regression coefficient cannot be 1
and + 1
125. Select one or more of the possible correct statement :

(a)Coefficient of determination is obtained by (b)Coefficient of determination is obtained


squaring the regression coefficient by squaring the coefficient of correlation
(c)Coefficient of determination can be (d)Coefficient of determination cannot be
negative negative
126. Given N=4 and ∑x =23.Calculate 𝑋. ̅

(a)5.75 (b)11
(c)0.0782 (d)0.25
127. Given b=1.89, 𝑋̅=5.75 and 𝑌̅=11. Calculate a.

(a)5.75 (b)11
(c)0.0782 (d)0.25
128. Given r=0.9745, regression coefficient y on x =1.8994. Calculate regression coefficient x
on y

(a)0.5 (b)11
(c)0.0782 (d)0.25
129. Using y = 1.297 +1.774x. Compute Y when x=3

(a)6.619 (b)7.719
(c)8.819 (d)cannot be determined
130. Using x= -0.55 +2y. Compute x when y=4

(a)7.45 (b)8.45
(c)9.45 (d)6.45
131. Identify regression coefficient from the following equation: Y=3+4x

(a)3 (b)4
(c)3/4 (d)4/3
132. The regression coefficient of a perfectly positively correlated data will be:

(a)-1 (b)0
(c)+1 (d) It can take any value
133. The coefficient of correlation of a perfectly positively correlated data will be:

(a)-1 (b)0

35 | P a g e
(c)+1 (d) It can take any value
134. The coefficient of determination of a perfectly positively correlated data will be:

(a)-1 (b)0
(c)+1 (d) It can take any value
135. From its past experience a company has observed that it is able to achieve sales revenue
of around Rs, 500,000 in years when where is no marketing expenditure and also that the sales
revenue increases by around Rs, 150,000 for every Rs.75,000 spending on marketing
expenditure from the data provided

(a)Sales = 500,000 + 75,000 × marketing (b)Marketing expenditure = 500,000 + 2 ×


expenditure sales
(c)sales = 2 × marketing expenditure (d) Sales = 500,000 + 2 × marketing
expenditure
135. Identify the limitation (s) of a scatter diagram:

(a)Useful aid to give a visual impression of (b)Might lead to incorrect conclusions if


the relationship between variables. there are only few data points available or
the data collected is a typical for some
reason
(c)Might indicate a relationship where there (d) It can take any value
is none

36 | P a g e
EXTRA PRACTICE QUESTIONS:
Question :1
Find regression equations and tell which is best from and b category repression equation which
one is best from c and d according to regression equation
Town Aay Bee Cee Dee
Police 190 180 250 200
number of 145 160 130 125
crimes
a. Aay and Bee b. Bee and Dee c. Aay and Dee d. Cee and Dee
Question:2
𝛴 x-1,239, 𝛴y =79, 𝛴 xy-17,233, 𝛴 x 2 =568,925, 𝛴 y 2 =293 n=100 find line 'y on x' and x on
y" and their point of intersection?
Question: 3
R = 0.6 bxy=1.2 find byx
a)0.3 b)0.2
c)3.33 d)None of these
Question 4
Co-efficient of rank of correlation?
x-{80,35.25.12,7} Y= { 12,14,17,22,3}
a) -1.00 b) -0.80 c)-0.12 d) None of these
Question:5
Following data is given, Find rank correlation
X 1 3 3 5 4 4 Total
Y 1 2 3 4 5 6
d2 0 1 0 1 1 4 ∑ d 2= 7
r= 0.7714
Question:6
Following data is given, Find Rank correlation
X 1 2 3 4 5 6 Total
Y 1 5 6 4 5 3
d2 0 9 9 0 0 9 ∑ d 2=
27

37 | P a g e
a) 0.2271 b) 5.0
c) 14.0 d) 1.5

Question:7 Following data is given. Find Rank correlation


X 1 2 3 4 5 6 Total
Y 1 5 4 6 5 3
d2 0 9 1 4 0 9 ∑ d 2=
27
r= 0.32857
Question:8
Co-efficient of rank of correlation of the following data is?
X=68, 20, 75, 86, 45
Y=94, 28, 62, 63, 65
a)0.055 c)1.00
b)0.508 d)0.2
Question: 10
r=? ΣΧΥ = 28.480 ∑x=360 Σ Χ2 =20,400
∑ y 2 =43,175 n=8 𝑥⃗ = 45 𝑦̅ 71.125
a) 0.8599 b) 0.8929
c) 0.6725 d) 0.9878
Question :11
If y on x is y=16-1.5x, then every increase in x, y would be
a) Increase by 16 b) Increased by 16-1.5
c)Decreased by 1.5 d) None of these
Question :12
r=0.8 means that
a)80% variation in y due to x b)20% variation in y due to x
c) 64% variation in y due to x d) 36% variation in y due to x
Question :13
Following regression lines are given
Regression line x on y (x + 2y = 3)
Regression line y on x (x+4y =10)
Find both slopes

38 | P a g e
a) b xy = -2 and b xy = -1/4 b) bxy= -2 and byx = -4
c) bx y=2 and byx =4 d) None of these
Question: 14
Following regression lines are given
Regression line x on y (x+2y-3)
Regression line y on x (x+4y-10)
Find r 2
a)0.5 b)-0.5
c)0.25 d)1
Question: 15
Following regression lines are given
Regression line x on y (x+2y-3)
Regression line y on x (x+4y=10)
Find explained variation
a) 50% in x due to y b) -50% in y due to x
c) 50% in y due to x d) All of these
Question: 16
Following regression lines are given
Regression line x on y (x+2y=3)
Regression line y on x (x+4y=10)
- Find 𝑥̅ and 𝑦̅
a) -4 and 3.5 b) 1 and 2
c) 2 and 4 d) Impossible to find in given data
Question: 17
Regression line y-2+3x is given. You are required to find the change in y due to 1-unit increase
in x
a) Y will increase 3 units b) Y will increase 5 units
c) Y will increase 2 units d) Y will decrease 3 units
Question: 18
(Variation in y due to x) r2=55%, find r if both b xy and b yx are negative

39 | P a g e
Question: 19
Find spareman's rank correlation between two innings from the data given below:
Batsman A B C D E F G
Innings1 29 32 45 19 65 0 7
Innings2 30 36 62 25 20 6 3

a) -0.61 b) 0.61
c) 0.55 d) -0.55
Question :20
𝛴(x-𝑥̅ ) (y-𝑦̅) =13, S2 x =2 s 2 y =5.2 r=0.8, n=?
Question: 21
Find the coefficient of correlation between x and y if:
Regression line of x on y is: 5x-4y+2=0
Regression line of y on x is: x-5y +3=0
a) 0.4 b) 0.6
c) 0.8 d) 1.0
Question:22

Σ(x-𝑥̅ ) (y-𝑦̅) =956 S x -21.5 S y 10.61, r=0.524 find n=?


Question:23
If r 2 is positive, then r will be?
a) Positive b) Negative
c) May be positive or negative d) None of these
Question:24
Following
equations are
given
x+2y+5=0
x+3y-10=0
Find coefficient of correlation
a) 0.8165 b) -0.8165
c) 1-2247 d)-1.2247

40 | P a g e
Question:25 From following information given, find coefficient of
correlation.
X 3 5 8 11 9
Y 1 0 4 0 1

a. r= 0.038 b. r= 0.082
c. r=0.018 d. r=none of these
Question:26
Which of the following is incorrect about Co efficient of determination?
a) Range from 0 to + 1 b) square of r
c) Range from 0 to -1 d) explains the contribution of independent in
dependent
Question:27
If two variables are independent, then correlation will be:
a) 0 b) 1
c) -1 d) t
Question: 28
If bxy=1.44 r=0.6 find byx=?
a) 0.11 b) 0.15
c) 0.25 d) 0.30
Question: 29
A computer operator calculated the correlation coefficient from 7 pairs (x, y) and obtain
following sums:
Σ x = 476, Σ y = 483, Σ xy = 32,864, Σ x² = 32,396, Σ y² = 33,359
It was later discovered at the time of checking that he has copied down two pairs as (70, 72) and
(71, 69) while the correct pairs were (60, 70) and (70, 65). Obtain correct value of correlation
coefficient
a) r=-0.24 b) r=0.32
c) r=-0.52 d) None of these
Question:30
n = 10, Σ (x-𝑥̅ ) 2 = 180 variance of u = 3x + 10 would be
a) Same b) 162
c) 64 d) 172

41 | P a g e
Question :31 If Variances of X and Y are 16 and 25. And X has 49% Influences in the
variation of Y then find out the slope of X
a)0.87 b) 0.56
c) 1.56 d) 0.82
Question :32 Below is the list of averages of batsman (rounded to whole number) in ODI and
test matches
Batsman A B C D E F G H I J
ODI 33 42 31 46 36 35 24 39 40 45
AVERAGES
TEST 44 50 38 42 31 44 31 35 41 49
AVERAGES
(1) Find the coefficient of correlation for averages in ODI and test matches
a) 0.601 b) 0.605
c) 0.425 d) 0.1125
(2) Interpret your result.
a) Strong correlation b) Average
c) Weak d) Above average
Question 33: If r= 0.5624 and bxy = 2.523 then find byx
a) 0.1253 b) 3.1253
c) 5.0345 d) None of these
Question :34 There is 25% deviation in y due to x. coefficient of correlation is?
a) +0.5 b) -0.54
c) Both a and b d) None of these
Question :35 If r2 = 0.25 then what is the meaning?
a) It means correlation is 25% b) It means correlation is 25%
c) It means correlation is 50% d) It means correlation is 75%
Question :36 Correlation b/w car weight and reliability r= -0.30
Correlation b/w car weight and maintenance cost r =+0.7
Which of the following statement are true?
a) Heavier cars tend to be less reliable b) Heavier cars tend to cost more to maintain
c) Car weight is related more strongly to d) Both (a) and (b)
reliability than to maintenance cost
Question :37
Correlation b/w Car weight and reliability r=0.30
Correlation b/w Car weight and maintenance cost r = 0.7
Which of the following statement are true?

42 | P a g e
a) Heavier cars tend to be less reliable b) Heavier cars tend to cost more to
maintain
c) Car weight is related more strongly to d) Both (a) and (b)
reliability than to maintenance cost
Question :38
∑ x=172 ∑y=613 ∑x 2 =4119 ∑xy=12,865 n=10
byx=?
Question :39
∑ =475 Y=326 ∑ X=4119 ∑Y=12,865 n=10 ∑XY=?
Question :40
In a scatter diagram, if the points follow closely a straight line of positive slope, The two
variables are said to have:
a) No correlation b) Perfect correlation
c) High positive correlation d) High negative correlation
Question :41
Which of the following correct about scatter diagram?
a) Perfect correlation (if all the points lie on b) Perfect-ve correlation (if all the points lie
the line of regression) on the line regression sloping downward)
c) +ve correlation (if scatter diagram shows d) -ve correlation (if scatter diagram shows
increasing trend on right side) decreasing trend on right side)
Question :42 How many of the following statement (s) is (are) correct?
(a) Correlation can be determined b/w more than 2 variables
(b) More than 2variable can be drawn on scatter diagram
(c) If plotted dots are close to the regression line than it means strong correlation
(1) only 1 (2) (1 and 2) (3) Only 3 (4) All of these
Question :43 Properties of scatter diagram
a) Graphical representation of relation b/w independent and dependent variables
b) It is only used for 2 variables
c) Independent variable is usually x
d) Dependent variable is usually y
Question :44 If r range from +0.9 to +1.00 then which of the following is correct
a) There is strong correlation b) There is perfect correlation
c) There is high correlation d) None of these

43 | P a g e
Question :45 If all points lie on scatter line of least square, then error will be
a)1 b) e
c) standard error d) 0
Question :46 ∑ (X-𝑥̅ )2 =50, ∑(y-𝑦̅)2 = 106, ∑(Y-𝑦̅) (𝑋 − 𝑥̅ ) = -67
Question :47 Find Coefficient of correlation
a) 0.9203 b) -0.9203
c)-0.2903 d) 0.2903
Question :48 𝑥̅ = 6, 𝑦̅ = 10, 𝛴𝑋𝑌 = 293, ∑𝑥 = 266, 𝑛 = 6, 𝛴𝑦 2 706
Find coefficient of correlation
a) 0.9203 b) -0.9203
c) -0.2903 d)0.2903
Question :49 Y=1.96x + 15 (y on x), X=0.45y +7.16 (X on Y). Find Co-efficient of
determination.
a) 0.94 b) 0.9983
c) 0.88 d) 0.5569
Question :50 If the Co-efficient of determination is equal to 1, then correlation Co-efficient is:
a) Must be equal to 1 b) Any value between -1 and +1
c) Either -1 or +1 d) Must be -1
Question :51 In regression, the sum of the residuals is always:
a) 0 b) >0
c) < 0 d) All of these
Question :52 In a simple linear regression equation, the number of independent variable is/are:
a)1 b)2
c)3 d) All of these
Question :53 The manager of an educational computer facility would like to develop a model to
predict
sample of 10 terminals was selected. The data follows
Terminal A B C D E F G H I J
No. of 3 4 3 5 5 7 8 10 10 12
service
class (X)
Age 1 1 2 2 3 3 4 4 5 5
(years)(y)

a) 0.7 and 0.092 b) 0.7 and 0.91


c) 0.9 and 0.7 d) 0.7 and 1.7

44 | P a g e
Question :54 In a study of relationship between family income and the amount of firm’s
product consumed by the by the family, a sample of 15 families yielded the following results
𝛴 2
2 2 = 𝑥
∑X=80. n =15, 𝑥̅ = 79, 𝑦̅ = 4 ∑𝑥𝑦 = 400, 𝛴𝑥 = 600, ∑𝑦 2

a)0.784 b) 0.6156
c) -0.784 d) 𝑂̅.6156
Question :55 If two variables x and y have perfect positive correlation and bxy=2/3 then byx
is?
a) -3/2 b)3/2
c)-2/3 d)2/3
Question :56 For the following table, the correlation between x and y and the correlation co-
efficient between
x and z
are:
X 2 4 6 8 10
Y 10 15 20 25 30
z 40 36 32 28 24
Select one:
a) rxy= 1 rxz=-1 b) rxy and rzx are perfectively correlated
c) rxy is perfect positive and rxz is d) All of these
perfectively negative
Question :57 The formula of regression coefficient of line X on Y
̅̅̅̅̅̅̅̅̅̅̅
∑ (x−𝑥)(𝑦−𝑦)̅̅̅ 𝑠
a) 𝑥𝑦
(𝑦−𝑦 )2 b) 𝑆2𝑦
𝑛×𝑦−∑(𝑥)(∑𝑦) d) All of these
c) 𝑛𝑦 2 −(𝛴𝑦)2
Question :58 In the regression line of X on Y, which of the following is/are co
a) ∑X=∑x b)𝑥̅ = 𝑥
c) ∑X-∑X=0 d)All of the above
Question :59 Given:
∑(𝑥 − 𝑥̅ )2 = 170, ∑ (y − y̅)2 = 140 and ∑ (X − x̅) (y-𝑦̅)=92
The coefficient of correlation and coefficient of determination are respectively:
a)0.3564, 0.8 b)0.597, 0.3564
c)0.9, 0.3 d)0.8, 0.64

45 | P a g e
Question :60 The value of r 2 for a particular situation is 0.36. What is the coefficient of
correlation?
a)0.6 b)-0.6
c)0.06 d)±𝟎. 𝟔
Question :61 What is the coefficient of correlation?
a)0.9770 b)0.9545
c)0.0455 d)0.0230
Question :62 If y=20 – 3x and x=4 – 0.25y then rxy =?
a)0.75 b)-0.75
c)-0.87 d)0.87
Question :63 If two variable x and y have perfect positive correlation and bxy =2/3 then byx is?
a)-3/2 b)3/2
c)-2/3 d)2/3
Question :64 The correlation between daily travelling and number of marriages is:
a) Positive b) Negative
c) No correlation d) Zero correlation
Question :65 A biologist assume that there is a linear relationship between amount of fertilizer
supplied to tomato plants and the subsequent yield of tomatoes obtained.
Eight tomato plants of the same variety were selected at random and treated weekly with a
solution in which x grams of fertilizer was dissolved in a fixed quantity of water. The yield, y
kilograms of tomatoes was recorded.
PLANT A B C D E F G H
x 1 1.5 2 2.5 3 3.5 4 4.5
y 3.9 4.4 5.8 6.6 7 7.1 7.3 7.7
Estimate the yield of a plant treated weekly with 3.2 grams of fertilizer
a)6.7 b)7.6
c)3.9 d)8.2
Question :66 The data in the following Table gives the Market share of product Television
Advertising Expenditure:
X= Advertising Expenditure 15 17 13 14 16
Y=Market Share 23 25 21 24 26
Estimate Market Share when advertising Expenditure is 20:
a)28 b)28.8
c)26.5 d)25.6

46 | P a g e
Question :67 If rxy=0.9 then ruv=will be =?
If u= -2x, v=7-3y
a) 0.9 b)-0.9
c)-1.8 d)1.8
Question :68 If b yx =1.5 then b xy will be
a) Less than -1 b) Less than 1
c) Both a and b d) None of these
Question :69 Y=45 – 3x is the regression line of y on x. What number of units is expected to
increase in ‘y’ if ‘x’ is decreased by Two units?
a)6 b)7
c)8 d)9
Question :70 If the coefficient of determination is equal to 1, then correlation coefficient is:
a) Must be equal to one b) Either -1 or +1
c) Any value between -1 and +1 d) Must be -1
Question :71 A regression analysis between sales (in Rs. 1,000) and advertising (in Rs. 1,000)
resulted in the following least squares line Y =80 +5x, this implies that:
a) As advertising increases by Rs.1,000 b) As advertising increases by Rs. 1,000, sales
Sales increases by Rs. 5,000 increases by Rs. 80,000
c) Advertising increases by Rs. 5, sale d) None of these
increases by Rs.80
Question :73 The unknown value of dependent variable can be estimated on the basis of given
value of independent variable by using:
a) Scatter Diagram b) Least square regression line
c) Both a and b d) None of these
Question :74 The two regression lines obtained from a set of data 5X +3y =15 and
4X+2y=10. Find mean values of X and Y
a)0,5 b)3,2
c)4,3 d)2,5
Question :75 The regression line Y=5+2X. Which of the following is true?
a) Rate of change in X per unit of Y=5 b) Rate of change in X per unit of Y=2
c) Rate of change in Y per unit of X=2 d) Rate of change in Y per unit of X=5
Question :76 If the two variables have perfect positive correlation, then
a) byx = bxy b) byx >bxy
c) byx< bxy d) b yx = 1/b xy

Question :77 The square root of the coefficient of determination gives us the exact:
a) Coefficient of correlation b) Absolute deviation
c) Variance d) None of these

47 | P a g e
Question :78 Given the sum of the squares of the difference of the first and second ranks for
10 residents of C.A, AFC-3 comes to be 218. The rank correlation is:
a) 0.23 b) -0.32
c) -0.23 d) 0.32
2
Question :79 For r =0.6 the explained variation independent variable due to independent
variable is:
a) 0.6 b) 0.4
c) 0.36 d) 0.16
Question :80 If the regression line is a perfect estimator of the dependent variables then which
of the following is false.
a) Co-efficient of determination is one b) Co-efficient of correlation is zero
c) All the data points fall on regression line d) None of these

48 | P a g e
1/1/2022 CHAPTER 11
COUNTING METHODS AND
PROBABILITY
1. VARIABLE (A variable is a quantity whose value changes)

Any observed characteristics can be termed as a variable if it varies from one object or
individual to other. For example, heights, weights of individual, prices of goods and if the
variable contains only single value it is referred to as a constant,

2. DISCRETE VARIABLE (A discrete variable is a variable whose value is obtained by


counting)
It is quantitative variable can assume only finite values between the given ranges of
values that is variable characterized by gaps or breaks in the values is called a discrete
variable. In other words, a discrete variable or discontinuous, represents count data such
as number of persons in a family, the number of rooms in a houses, income of an
individual, number of ones passed by a person. A data based on discrete variables is
called discrete data.
.
Examples: Number of student’s present
Number of red marbles in a jar
Number of heads when flipping three coins
Students’ grade level

3. CONTINUOUS VARIABLE (A continuous variable is a variable whose value is


obtained by measuring)
If a Quantitative Variable can assume value within a given range, without gaps then this
variable is known as continuous variable. In other words, a continuous variable represents
measurement data such as age of person, height of individual, weight of commodity,
temperature at place, speed of train., The data based on continuous variable is called
continues data.

Examples: Height of students in class


Weight of students in class
Time it takes to get to school
Distance traveled between classes

4. RANDOM VARIABLE
A random variable is a variable whose value is a numerical outcome of a random
phenomenon(occurrence)

5. RANDOM EXPERIMENT
A random experiment is a process leading to at least two possible outcomes with us
certainty as to which will occur

Example:
(i) a coin is tossed or a die is rolled

1|Page
(ii) A consumer asked which of the two products he prefer
(iii) Annual average inflation rate
(iv) counting of number of detective light bulbs produces per hour.

6. MUTUALLY EXCLUSIVE EVENTS (Nothing common)


Two events A and B are called mutually exclusive event if they have no common
outcomes and their intersection is said to be the empty set. It means A ∩ B cannot occur
P(AUB) = P(A) + P(B)
OR
Events are mutually exclusive if the occurrence of one event excludes the occurrence of the
other(s). Mutually exclusive events cannot happen at the same time. For example: when tossing a
coin, the result can either be heads or tails but cannot be both.

7. NON-MUTUALLY EXCLUSIVE EVENTS(Common events)


Two events are said to be non-mutually exclusive events when they can occur at the same
time.
Example:
(i) a card is drawn from a well shuffled deck of playing cards, it can be both a queen
and heart.
(ii) Inflation and recession are non-mutually exclusive.

P(AUB) = P(A) + P(B) –P (A ∩ B)

8. INDEPENDENT EVENTS
Events are independent if the occurrence of one event does not influence (and is not influenced
by) the occurrence of the other(s). For example: when tossing two coins, the result of one flip
does not affect the result of the other.

This of course means mutually exclusive events are not independent, and independent events
cannot be mutually exclusive

P (A ∩ B) = P(A) x P(B)

9. EXHAUSTIVE EVENTS
If the union of two events A and B is the sample space. These are said to be exhaustive
event. Let E1, E22, E3 …. Ek be k event in sample space S, of E1 UE2 UE3 … UEk = S
then k events are said to be collectively exhaustive
P (A UB) = 1
10. EQUALLY LIKELY
Events are said to be equally likely when they have same chance of occurrence.
Example:
(i) Tossing a coin has equal chance of occurrence
(ii) Rolling a dice has equal chance of occurrence

2|Page
Explanation:
Mutually Exclusive Events?
Two events are defined to be mutually exclusive if they cannot happen at the same time. In
other words, if one event happens, the other event cannot happen. Mutually exclusive events are
sometimes referred to as disjointed events.
Examples: of events that are mutually exclusive and then a few that are not:
A: Today is Saturday.
B: Today is Tuesday.
A: A coin lands on heads.
B: The same coin lands on tails.
A: You have passed your statistics class.
B: You have failed your statistics class.
What about this set, though?
A: Today is Saturday.
B: It is raining outside.
A: A coin lands on heads.
B: I rolled a die and it landed on four.
A: You passed your statistics class.
B: You passed your accounting class.

3|Page
COMBINATION (Selection/Order does not matter)
On many occasions we are not interested in arranging but only in selecting r objects from given n
objects. A combination is a selection of some or all of a number of different objects where order
of selection is immaterial (order not matter)
Example:
From a team of two from Amy, Bob and Roy
Sol: Amy and Bob (Bob and Amy)
Amy and Roy (Roy and Amy)
Bob and Roy (Roy and Bob)
Formula:

Question.1 Evaluate the following:


(i) 8!
(ii) 5C
3
(iii) 52 C3
(iv) 11C
4

A. B. C. D.

Solution
Given
(i) 8! = 40320
5
(ii) C3 = 10
52
(iii) C3 = 22100
11
(iv) C4 = 330

Question.2 A three-person committee is to be formed form a list of four persons. How


many sample points are associated with the experiment?

A. B. C. D.

Solution
n
Cr = 4C3 = 4

Question.3 How many sample points are in the sample space when a person draws a
hand of 5 cards from a well-shuffled ordinary deck of 52 cards?
4|Page
A. B. C. D.

Solution
Given
n
Cr = 52C5 = 2598,960
Question.4 There are 8 gentlemen and 4 ladies. Find the number of ways in which the
committee of 7 members can be formed form these, if each committee is to include at least 3
ladies.

A. B. C. D.

Solution
Given
Required number of ways = 8C4 x 4C3 + 8C3 x 4C4
= 280 + 56
= 336

K-14
Question.5 If 5Cr = 10, then r =?

A. 2 B. 4 C. 6 D. 7

Solution
Given
5
Cr = 10
5
Cr = 5C2
r =2

Question.6 In a group of 6 boys and 4 girls, four children are to be selected. In how many
different ways can they be selected such that at least one boy should be there?

Solution:
We may have (1 boy and 3 girls) or (2 boys and 2 girls) or (3 boys and 1 girl) or (4 boys).
Required
number = (6C1 x 4C3) + (6C2 x 4C2) + (6C3 x 4C1) + (6C4)
of ways

= (24 + 90 + 80 + 15)

= 209.

5|Page
Question.7 Out of 16 men, in how many ways a group of 7 men may be selected so that
particular 4 men will not come.
Solution: When four men are not selected.
12
C7=792
Question.8 Out of 16 men, in how many ways a group of 7 men may be selected so that
particular 4 men will always together.
Solution: When four men are already selected.
=12C3 x
4
C4 = 220
Question.9 A person has got 15 acquaintances (friends) of whom 10 are relatives. In how
many ways may be invite 6 guests so that 4 of them would be relatives?
Solution: =10C4 x
5
C2 = 2100
Question.10 Out of five ladies and 3 gentlemen, a committee of 6 is to be selected, In how
many ways can 4 ladies be selected.
Solution: Number of ways = 5C4 x 3C2= 15
Question.11 In an examination paper, 10 Questions are set, In how many different ways
can you choose 6 questions to answer. If however no. 1 is made compulsory. In how many
ways can you select to answer 6 questions in all?
10
Solution: (i) Number of ways for 6 question = C6 = 210
Solution: (ii) Number of ways when question is compulsory = 9C5 =126
Question.12 Out of 5 ladies and 3 gentlemen, a committee of 6 is to be selected. In how
many ways can majority of ladies be selected?
5 3 5 3
Solution: = C4 x C2 + C5 x C1 = 18
Question.13 A cricket team of 11 player is to be selected from two group consisting of 6
and 8 players respectively. In how many ways can the selection be made on the supposition
that the group of six shall contribute no fewer than 4 players?
Solution: number of selection = 6C4 x 8C7 +
6
C5 x
8
C6 + 6C6 x
8
C5 = 344
Question.14 There are 5 questions in group A, 5 in group B and 3 in c. in how many ways
can you select 6 questions taking 3 from group A, 2 from group B. and 1 from group C.
Solution: =5C3 x 5C2 x 3C1 = 300
Question.15 How many different types of launches are possible consisting of soup , a
sandwich, desert and soft drink, if one can select from different varieties of 4 soups, 3
sandwiches, 5 deserts and 4 cold drinks.

6|Page
Solution: Number if selection = 4C1 x 3C1 x 5C1 x 4C1
= 4*3*5*4
= 240

7|Page
Type:1
Tossing a Coin

8|Page
Question.1 Find the probability of each of the following:
(i) A head appears in tossing a fair coin

A. B. C. D.

Solution
Given
(i) Sample space when a coin is tossed S = {H, T}
n(S) = 2
Let A be the event that a head appears
A = {H}
n(A) = 1
n(A) 1
P(A) = =
n(S) 2

K-12
Question.2 4 coins are tossed simultaneously the number of outcomes containing at least
one head are

A. B. C. D.

Solution
Given
Sample space = { HHHH, HHHT, HHTH, HTHH, HHTT, HTHT, HTTH, HTTT,
THHH, THHT, THTH, TTHH, THTT, TTHT, TTTH, TTTT}
Number of outcomes containing at least one head = 15

K-13
Question.3 If 4 coins are tossed simultaneously the number of outcomes considering at
least two tails are.

A. B. C. D.

Solution
Given
Sample space = {HHHH,
TTTT}
Number of outcomes containing at least two tails = 11
Note:
H HHH, HHT, HTH, THH, HTT, THT, TTH, TTT
T HHH, HHT, HTH, THH, HTT, THT, TTH, TTT

9|Page
Question.4 Three coins are tossed, what is the probability of getting at least one head?

A. B. C. D.

Solution
Given
When 3 coins are tossed, the sample space is
S = {HHH, HHT, HTH, THH, HTT, THT, TTH, TTT}
n(s) = 8
let A be the event that getting at least one head
A = {HHH, HHT, HTH, THH, HTT, THT, TTH}
n(A) = 7
n(A) 7
P(A) = =
n(s) 8

K-20
Question.5 If a coin is tossed four times, the probability of four heads is equal to.

A. B. C. D.

Solution
Given
Sample space = (2)4 = (2)4 = 16
Let A be the event of four heads.
Now
n(A) = 1
n(A)
𝑃(A) =
n(5)
1
𝑃(A) =
16

K-44
Question.6 If one coin is tossed 7 times the number of possible outcomes would be.

A. B. C. D.

Solution
Given
Number of outcomes = (2)7
= 27
= 128

10 | P a g e
Question.7 Find the probability of getting exactly 4 heads when 6 coins are tossed.

A. B. C. D.

Solution
Given
Here
Sample points = 26 = 64
Event = 6C4 =15

Hence
6
C4
P (exactly 4 heads)= 64

15
=
64

11 | P a g e
Type:2
Rolling a Fair Dice

12 | P a g e
Question.1 Find the probability of each of the following:

(i) A 5 appears in rolling a six-faced cubical dice.


(ii) An even number appears when a perfect cubical die is rolled.

A. B. C. D.

Solution
Given
Sample space = S = {1, 2, 3, 4, 5, 6}

(i) B be the event that 5 appeals on the die.


B = {5}
n(b) = 1
n(B) 1
P(B) = =
n(5) 6
(ii) Sample space for a perfect cubical die is
Sample space = S = {1, 2, 3, 4, 5, 6}
n(S) = 6
let C be the event that an even number appears
c = {2, 4, 6}
n(c) = 3
n(c) 3 1
P(c) = = =
n(5) 6 2

K-15
Question.2 If the experiment, tossing a pair of dice, is repeated over and over again in a
very long series of trials. What proportion of outcomes do you think would
result in a sum less than 7?

A. B. C. D.

Solution
Given
Sample Space = {(1, 1), (2, 1), (3, 1), (4, 1), (5, 1), (6, 1)
(1, 2), (2, 2), (3, 2), (4, 2), (5, 2), (6, 2)
(1, 3), (2, 3), (3, 3), (4, 3), (5, 3), (6, 3)
(1, 4), (2, 4), (3, 4), (4, 4), (5, 4), (6, 4)
(1, 5), (2, 5), (3, 5), (4, 5), (5, 5), (6, 5)
(1, 6), (2, 6), (3, 6), (4, 6), (5, 6), (6, 6)

n(S) = 36
let A be the event that sum is less than 7 is

13 | P a g e
A = {(1, 1), (1, 2), (1, 3), (1, 4), (1, 5), (2, 1), (2, 2), (2, 3), (2, 4), (3, 1), (3, 2), (3, 3), (4,
1), (4, 2), (5, 1)}
n(A) = 15
n(A) 15 5
P(A) = = =
n(15) 36 12

K-16
Question.3 If the experiment, tossing a pair of dice, is repeated over and over again in
the long series of trials, what proportion of outcomes do you think would
result in a sum equal to 7?

A. B. C. D.

Solution
Given
S = {(1, 1), (1, 2), (1, 3), (1, 4), (1, 5), 1, 6)
(2, 1), (2, 2), (2, 3), (2, 4), (2, 5), (2, 6)
(3, 1), (3, 2), (3, 3), (3, 4), (3, 5), (3, 6)
(4, 1), (4, 2), (4, 3), (4, 4), (4, 5), (4, 6)
(5, 1), (5, 2), (5, 3), (5, 4), (5, 5), (5, 6)
(6, 1), (6, 2), (6, 3), (6, 4), (6, 5), (6, 6)
n(S) = 36
let A be the event that sum is equal to 7 is
A = {(1, 6), (2, 5), (3, 4), (5, 2), (6, 1), (4,3)}
n(A) = 6
n(A) 6 1
∴ P(A) = = =
n(S) 36 6

K-17
Question.4 If the experiment tossing a pair of dice, is repeated over and over again in a
very long series of trial, what proportion of outcomes do you think would
result in a sum more than 7

A. B. C. D.

Solution
Given
Sample Space = {(1, 1), (1, 2), (1, 3), (1, 4), (1, 5), 1, 6)
(2, 1), (2, 2), (2, 3), (2, 4), (2, 5), (2, 6)
(3, 1), (3, 2), (3, 3), (3, 4), (3, 5), (3, 6)
(4, 1), (4, 2), (4, 3), (4, 4), (4, 5), (4, 6)
(5, 1), (5, 2), (5, 3), (5, 4), (5, 5), (5, 6)
(6, 1), (6, 2), (6, 3), (6, 4), (6, 5), (6, 6)
n(S) = 36

14 | P a g e
let A be the event that sum is more than 7 is
A = {(2, 6), (3, 5), (3, 6), (4, 4), (4, 6), (5, 3), (5, 4), (4,5), (5, 5), (5, 6), (6, 2), (6, 3),
(6, 4), (6,5), (6, 6) }
n(A) = 15
n(A) 15 5
∴ P(A) = = P(A) = =
n(S) 36 12
K-46
Question.5 In the game of craps, two dice are rolled. If the sum on the dice is 7 or 11 he
wins, the probability of his winnings is.

A. B. C. D.

Solution
Given
Sample space = [(1, 1) ,
= (1, 2) ,
= (1, 3) ,
n(s) = 36
Let A be event that sum is 7 or 11
A = {(1, 6), (2, 5), (3, 4), (4, 3), (5, 2), (6, 1), (5, 6), (6, 5)}
n(A) = 8
Then
n(A)
P(A) =
n(S)
8
=
36
2
=
9

K-47
Question.6 In the game of craps, two dice are rolled. and if the sum is 2, 3 or 12, he loses.
The probability that the loss is.

A. B. C. D.

Solution
Given
Sample space = [(1, 1) ,
= (1, 2) ,
= (1, 3) ,
= (1, 4) ,
n(s) = 36
Let A be event that sum of dots is 2, 3 or 12
A = {(1, 1), (1, 2), (2, 1), (6, 6)}

15 | P a g e
n(A) = 4

n(A)
P(A) =
n(S)
4
=
36
1
=
9

Question.7 A die is rolled. What is the probability of.


(i) Number 1 is shown up
(ii) Number 6 is shown up
(iii) An even number
(iv) A number greater than 4.

A. B. C. D.

Solution
Given
(i) Total number of outcomes
Sample space = [1, 2, 3, 4, 5, 6]
n(S) = 6
let A be the event that number 1 is shown up is
n(A) = 1
Hence
n(A) 1
P(A) = =
n(S) 6
(ii) Let B be the event that number 6 is shown up
Hence
n(B)
P(B) =
n(S)
= 1/6
(iii) Let C be the event that an even number appears.
n(C) = {2, 4, 6} = 3
Hence
n(C)
P(C) =
n(S)
3
=
6
= 1/2
(iv) Let D be the event that a number greater than 4 is
n(D)
P(D) =
n(S)

16 | P a g e
2
=
6
1
=
3

Question.8 What is the probability of getting neither 7 nor 11 when a pair of dice is
tossed?

A. B. C. D.

Solution
Given
Sample space = 36 = n(S)
The event “a total of 7” and “a total of 11” are mutually exclusive events.
 P(neither a total of 7 nor a total of 11)= 28/36
= 7/9
Question.9 Two balanced dice are rolled. What is the probability that the sum of dots is
a multiple of 5?

A. B. C. D.

Solution
Given
Sample space = n(S) = 36
Let A be the event that sum of dot is
Multiple of 5 means either sum is 5 or 10
A = [(1, 4), (2, 3), (3, 2), (4, 1), (4, 6), (5, 5), (6, 4)]
n(A) 7
P(A) = ==
n(S) 36

Question.10 A fair dice is roll thrice. What is the probability that each time a six will
appear?

A. B. C. D.

Solution
Given
Sample space = 63=216
P (a six) = 1/6
1
P (3 times a six) = 1/6 × 1/6 × 1/6 =
216

Question.11 Show that in a single throw with two die, the chance of throwing more than 7
is equal to that of throwing less than 7?

17 | P a g e
A. B. C. D.

Solution
Given
When two dice a5re rolled sample, space is
(i) S = {(1, 1) l, (1, 2), (1, 3), (1, 4), (1, 5), 1, 6)
(2, 1), (2, 2), (2, 3), (2, 4), (2, 5), (2, 6)
(3, 1), (3, 2), (3, 3), (3, 4), (3, 5), (3, 6)
(4, 1), (4, 2), (4, 3), (4, 4), (4, 5), (4, 6)
(5, 1), (5, 2), (5, 3), (5, 4), (5, 5), (5, 6)
(6, 1), (6, 2), (6, 3), (6, 4), (6, 5), (6, 6)
n(S) = 36
let A be the event that sum is less than 7 is
A = {(1, 1), (1, 2), (1, 3), (1, 4), (1, 5), (2, 1), (2, 2), (2, 3), (2, 4), (3, 1), (3, 2), (3, 3), (4,
1),
(4, 2), (5, 1)}
n(A) = 15
n(A) 15
P(A) = =
n(5) 36
(ii) Let B be the event that sum is more than 7
B = {(2, 6), (3, 5), (3, 6) (4, 4), (4, 5), (4, 6), (5, 3), (5, 4), (5, 5), (5, 6), (6, 2), (6, 3), (6,
4), (6, 5), (6, 6)}
n(B) = 15
n(B) 15
P(B) = =
n(s) 36

18 | P a g e
Type:3
Selecting a Card

19 | P a g e
Note: Name of cards
Diamond, Heart, Club, Spade

Question.1 What is the probability of selecting a card of diamonds from a pack of


playing cards consisting of the usual 52 cards?

A. B. C. D.

Solution
Given
Sample space for 52 cards
N(S) = 52
Let A be the event that card is a diamond card.
N(A) = 13
n(D) 13 1
P(D) = = =
n(5) 52 4
K-41
Question.2 From a deck of 52 cards two cards are drawn in succession. The probability
that both cards are spade is.

A. B. C. D.

Solution

Required Probability = 13/52 x13/52


= 169/2704
= 1/16

Question.3 If a card is drawn from an ordinary deck of 52 cards, find the probability
that (i) card is a red card (ii) the card is a diamond (iii) the card is 10.

A. B. C. D.

Solution
Given
Total number of possible outcomes = 52 = n(S)
Let A be the event that the card drawn is red card
n(A)
Hence
(i)
n(A)
P(A) =
n(S)
P(A) = 26/52
P(A) = 1/2

20 | P a g e
(ii) Let B be the event that the card drawn is a diamond
n(B) = 13
Hence P(B) = 13/52
P(B) = ¼
(iii) Let C be the event that the card drawn is a 10
Hence
n(C)
P(C) =
n(S)
= 4/52
= 1/13

Question.4 Two cards are drawn without replacement from a pack of 52 cards what is
the probability that:
(i) Both are king
(ii) Both are Queen
(iii) First is king and second is queen

A. B. C. D.

Solution
Given
(i) Required probability = 4/52 x 3/51 =1/221

(ii) Required probability = 4/52 x 3/51 =1/221

(iii) Required probability = 4/52 x 4/51 = 16/2652 =4/663

Question.5 A card is selected at random from 80 cards numbered from 1 to 80. Find the
probability that the number on the card is a perfect square.

A. B. C. D.

Solution
Given
Given
S = [1, 2, 3, 4, 5, ……… 80]
n(S) = 80
let A be event that the number is a perfect square.
A = [1, 4, 9, 16, 25,36,49, 64]
Hence
n(A)
P(A) =
n(S)

21 | P a g e
8
=
80
1
=
10
1
=
10

Question.6 Four cards from a regular pack of 52 cards are placed random from the deck.
Find the probability that all four are aces.

(i) With replacement


(ii) Without replacement

Solution
(i) Required Probability = 4/52 x 4/52 x 4/52 x 4/52 = 256/7311616 =1/28561
(ii) Required Probability = 4/52 x 3/51 x 2/50 x 1/49 = 24/6497400 = 1/270725

Question.7 From a deck of 52 cards, three cards are drawn one by one without
replacement. Find the probability that each time it is a card of spade.

Solution

P(A) =13/52 x 12/52 x 11/50 = 11/850

22 | P a g e
Type :4
Multiple objects

23 | P a g e
Question.1 A bag contains 12 balls of which 3 are marked, if 5 balls are drawn out
together what is the probability that 3 of the marked balls are among these
5?

A. B. C. D.

Solution
Given
Marked balls Unmarked balls Total balls Selected balls
3 9 12 5

12
Sample Space = ( )
5

3 9
Event = ( ) ( )
3 2
3 9
( )( )
∴ P(3 balls are marked) = 3 2
12
( )
5
1 × 36 1
= =
792 22

Question.2 An integer between 3 and 12 inclusive is chosen at random. What is the


probability that?
(i) it is even number?
(ii) it is even and divisible by 3?

A. B. C. D.

Solution
Given
Sample space = [3, 4, 5, 6, 7, 8, 9, 10, 11, 12]
n(S) = 10
(i) Let A be event that the integer chosen is an even number then.
A = [4, 6, 8, 10, 12]
n(A) = 5
n(A)
P(A) =
n(S)
5
=
10
1
=
2
(ii) Let B the event that the integer chosen in an even number and divisible by 3, then

24 | P a g e
B = [6, 12]
n(B) = 2
Hence
n(B)
P(B) =
n(S)
2
=
10
1
=
5

K-33
Question.3 A box contain 10 items, 3 of which are defective. If 4 are selected at random
without replacement, the probability that at least two are defective is.

A. B. C. D.

Solution
Given
P (at least 2 are defective P(2) + P(3)
3
C2 × 7C2 3
C3 × 7C1
= 10 + 10
C4 C4

3 × 21 1×7
= +
210 210

63 7
= +
210 210

70
=
210
= 1/3
Question.4 Six white balls and four black balls, which are indistinguishable apart, from
color, are placed in a bag. If six balls are taken from the bag, find the
probability of their being three white and three black.

A. B. C. D.

Solution
Given
Here
Total number of possible equally likely outcomes in S is
10
n(S) = ( ) = 210
6
Let A represent the event that three white and three black balls are taken

25 | P a g e
6 4
n(A) = ( ) ( ) = 80
3 3
Hence
n(A)
P(A) =
n(S)
80
P(A) =
210
8
P(A) =
21

Question.5 An employer wishes to hire three people from a group of 15 applicants, 8


men and 7 women, all of whom are equally qualified to fill the position. If he
selects the three at random. What is the probability that (i) all three will be
men (ii) at least one will be a woman?

A. B. C. D.

Solution
Given
(i) Here
Number of ways in which 3 people can be selected from 15 applicants.
15
n(S) = ( ) = 455
3
Hence
n(A)
P(A) =
n(S)
56
=
455
8
=
65
(ii) Let be denote the event that at least one women is selected.
7 8 7 8 7 8
n(B) = ( ) ( ) + ( ) ( ) + ( ) ( )
1 2 2 1 3 0
= 196 + 168 + 35
= 399
Hence
n(B)
P(B) =
n(S)
399
=
455
57
=
65
= 0.877

26 | P a g e
Question.6 Four items are taken at random from a box of 12 items and inspected. The
box is rejected if more than 1 item is found to be faulty. If there are 3 faulty
item in the box. Find the probability that the box is accepted.

A. B. C. D.

Solution
Given
Here
12
n(s) = sample space = ( ) = 495
4
let A denote the event the number of faulty items chosen is 0 or 1, then
3 9 3 9
n(A) = ( ) ( ) + ( ) ( )
0 4 1 3
= 126 + 252
= 378
Hence
378
P(A) =
495
=42/55

Question.7 A bag contains 4 red and 5 black balls. A ball is drawn from it. What is the
probability that?
(i) The ball drawn is red
(ii) The ball drawn is black

A. B. C. D.

Solution
Given
(i) Total number of possible outcome = 4 + 5 = 9C1
n(A) = number of outcomes favorable to red ball = 4C1
Hence
P (the ball drawn is red) = 4/9
(ii) Number of outcomes favorable to black = 4C1
 P (the ball drawn is black) = 5/9

Question.8 The following table gives a distribution of wages of 1000 workers.

Wages (in 120 – 140 140 - 160 160 - 180 180 - 200 200 - 220 220 - 240 240 – 260
Rs.)
No of works 9 118 478 200 142 35 18
An individual is selected at random from the above group. What is the
probability that his wages are?
(i) Under Rs.160

27 | P a g e
(ii) Above Rs.200
(iii) Between Rs.160 and 200?

A. B. C. D.

Solution
Given
(i) Total number of workers - 1000
Number of workers having wages under Rs. 160 = 9 + 118 = 127
127
Required probability = 1000 = 0.127
(ii) Number of workers having above Rs.200 = 142 + 35 + 18 = 195
195
Required probability = 1000 = 0.195
(iii) Number of workers having wages between Rs. 160 and Rs.200 is = 478 + 200 = 678
678
Required probability = 1000 = 0.678
Question.9 An urn consists of 3 white balls, 4 red balls and 5 black balls. Two balls are
drawn. What is the probability that?
(i) Both are red
(ii) Both are white
(iii) One is red, one is white

A. B. C. D.

Solution
Given
Total balls, 3 + 4 + 5 = 12
(i) Required probability = 4C2 / 12C2 – 1/11
(ii) Required probability = 3C2 / 12C2 = 1/22
4
C1 3C1 2
(iii) Required Probability = 12 =
C2 11

Question.10 A class consist of 10 boys and 8 girls. A committee of three students is


constituted what is the probability that the committee has.
(i) All boys
(ii) All girls
(iii) 1 boy and 2 girls
(iv) At least one girl.

A. B. C. D.

Solution
Given
(i) Total students = 10 + 8 = 18

28 | P a g e
10
C3 5
Required Probability = 18 =
C3 34

8
(ii) Required Prob C3 7
18 =
= C3 102
10
C1 ×8C2
(iii) Required Prob= 18
C3
(iv)
8C1 ×10C2 8C2 ×10C1 8C3 ×8C0
= + +
18C3 18C3 18C3
=696/816
=29/34

Question.11 The board of directors of a company consists of 12 members, 3 of whom are


women. A committee of 3 members is randomly selected from the board.
(i) What is the probability that all the members of the committee are men.
(ii) What is the probability that least one member of the committee is a
woman.

A. B. C. D.
Solution
Given
Total = 3 + 9 = 12
No. of members selected = 3
9
C3×3C0 84
(i) P(3 men) = 12
C3 = 220

(ii) P(at least one woman) = P(1 women and 2 men) + P(2 women 1 man) + P(3
women)
3
C1 × 9C2 3
C2 × 9C1 3
C3 ×
9
= + + C0
12 12 12
C3 C3 C3

108 27 1
= + +
220 220 220

136
=
220
=34/55

29 | P a g e
Question.12 The board of directors of a company consists of 8 men and 4 women. A four-
member committee is to be chosen at random from the board.
(i) What is the probability that all 4 members of the committee will be men
(ii) What is the probability that all 4 members of the committee will be
women
(iii) Why the sum of the probabilities for A and B does not equal 1? Explain

A. B. C. D.

Solution
Given
Total = men + women
12 = 8 + 4
n(S) = 12
(i) A = all four members will be
n(A) = n(A) = 8C4 × 4C0 = 70
n(A)
P(A) =
n(S)
70
= = 0.1414
495
(ii) B = all four members with the women
n(B) = 4C4 × 8C0 = 1
Hence
n(B) 1
P(B) = = = 0.002
n(S) 495
(iii) Because they are not mutually exclusive and completed exhaustive.
Question.13 Suppose a population consists of 15 items, 10 of which are acceptable. A
sample of 4 items is selected without replacement. What is the probability
that exactly 3 are acceptable?

A. B. C. D.

Solution
Given
Acceptable Not Acceptable Total
10 5 15
Number of items selected = 4
P(3 acceptable and 1 not acceptable)
10
C3 ×5C1 120 × 5 600
= 15 = =
C4 1365 1365

Question.14 Out of 12 eggs in a refrigerator, 2 are rotten from these 12 eggs, 4 eggs are
selected at random to make a cake. What are the probabilities that
(i) Exactly one is rotten

30 | P a g e
(ii) At least one is rotten.

A. B. C. D.

Solution Given
Good Rotten Total
10 2 12
Number of eggs chosen = 4
(i) P(1 rotten and 3 goods, eggs)
2
C1 ×10C3 240
= 15 =
C4 495
(ii) P(at least one rotten) = P(1 rotten and 3 goods) + P(2 rotten and 2 good)
2 2
C1 × C2 × 10C2
10
= C3 =
15 12
C4 C4
= 240 + 45
495 495
= 285
495

Question.15 Among 18 members of a cricket club, there are 2 keepers and 5 bowlers. In
how many ways can be team of 11 members be chosen so as to include only
one wicket keeper and at least three bowlers?

A. B. C. D.

Solution
Given
Keepers Bowlers Other Total
2 5 11 18
No of ways one keeper and at least 3 bowlers
= (one keeper, 3 bowlers and 7 others) + one keeper 4 bowlers and 6 others + one keeper
5 bowlers and 5 other.
= 2C1 × 5C3 × 11C7 + 2C1 × 5C4 × 11C6 + 2C1 × 5C5 × 11C5
= 2 × 10 × 330 + 2 × 5 × 462 + 2 × 1 × 462
= 6600 + 4620 + 942
= 12144

Question.16 The name of 4 men and 6 women are written on a slips of paper and placed
in a box. Four names are drawn without replacement. What is the
probability that 2 are men and 2 are women?

A. B. C. D.

31 | P a g e
Solution
Given
Total = 4 + 6 = 10
4
C2 × 6C2 6 × 15
P (2 men and 2 women) 10 =
C4 210
Question.17 A firm buys 3 shipment of part each month. The purchasing agent select at
random from among four in-state supplier and six out of state supplier.
What is the probability that the orders are placed with?
(i) The in-state supplier
(ii) The out-of-state supplier only
(iii) At least one in-state supplier?

A. B. C. D.

Solution
Given
(i) Here
n(S) = 10C3 = 120
let A be event that the orders place in-state in(A) = 4C3 = 4
Hence
n(A) 4 1
P(A) = = =
n(S) 120 30
(ii) Let B be the event that orders are placed out-of-state
6
n(B) = ( ) = 20
3

Hence
n(B) 20 1
P(A) = = =
n(S) 120 6
(iii) Let C be the event that
At least one in-state mean that one, two or three in-state supplier.
4 6 4 6 4 6
n(C) = ( ) ( ) + ( ) ( ) + ( ) ( )
1 2 2 1 3 0
= 60 + 35 + 4
= 100
Hence
n(C) 100 5
P(C) = = =
n(S) 120 6

Question.18 From a group of 6 men and 8 women, 5 people are chosen at random. Find
the probability that there are more men chosen then women.

A. B. C. D.

32 | P a g e
Solution
Given
14
Here n(S) = ( ) = 2002
5
Let A be the event that more men are chosen than women.
6 8 6 8 6 8
n(A) = ( ) ( ) + ( ) ( ) + ( ) ( )
3 2 4 1 5 0
= (20 × 28) + (15 × 8) + (16 × 1) = 686
Hence
686
P(A) = = 0.34
2002

Question.19 Three applicants are to be selected at random out of 4 boys and 6 girls. What
is the probability of selecting (i) all girls (ii) all boys (iii) at least one boy?

A. B. C. D.

Solution
Given
Sample space = n(S) = 10C3 = 120
(i) Let A be the event that 3 selected are girl,
n(A) = 6C3 = 20
Hence
n(A)
P(A) =
n(S)
20
=
120
= 1/6
(ii) Let B be the event that 3 selected are boys
4
P(B) = ( ) = 4
3
4
=
120
1
=
30
(iii) Let C the event that at least on boy
4 6 4 6 4 6
n(C) = ( ) ( ) + ( ) ( ) + ( ) ( )
1 2 2 1 3 0
= 60 + 36 + 4
n(C) = 100
Hence
100 5
P(C) = =
120 6

33 | P a g e
Type:5
(Use of Formula)

34 | P a g e
MULTIPLICATION RULE FOR INDEPENDENT EVENTS
P(A and B) = P(A).P(B)
Independent: the occurrence of one event has no effect on the probability of the occurrence of
another event
Example:
A survey by American auto mobile association revealed that 60% of its members are made
airline reservation last year two member are selected at random. What is the probability both
made airline reservation last year?
Sol: P(R1 and R2)= P(R1) *P (R2)=0.6*0.6=0.36

ADDITION RULE
MUTUALLY EXCLUSIVE
P(A or B)=P(A) +P(B)
NON-MUTUALLY EXCLUSIVE
P(A or B)=P(A) +P(B) - P(A and B)
OR
P(A  B) = P(A) + P(B) - P(A  B)
Example: if you randomly choose one of the integer from 1-10 what is the probability of
choosing number is either an odd number or an even number
Sol: P(odd or even)= P(odd) + P(even)
=5/10 + 5/10=1/2
Question.1 For two independent events A and B ,P(A) = 0.25, and P(B) = 0.40. Find
P(A ∩ B)?

A. B. C. D.

Solution
Given
P(A) = 0.25
P(B) = 0.40
Since A and B are independent events
P(A B) = P(A) . P(B)
= (0.25) (0.40)
= 0.10

35 | P a g e
Question.2 In a group of 20 adults, 4 out of 7 women and 2 out of 13 men wear glasses.
What is the probability that a person chosen at random from the group is a
women or someone who wear glasses?

A. B. C. D.

Solution
Given
Here
7
n(S) = 20 P(W) = 20
6
n(W) = 7 P(G) = 20
4
n(W  G) = 4 P(W  G) = 20
Hence
P(W  G) = P(W) + P(G) – P(W  G)
7 6 4
= + −
20 20 20
9
=
20
= 0.45

Question.3 A and B can solve 60% and 80% of the problem in a book respectively. What
is the probability that either A or B can solve a problem chosen at random?

A. B. C. D.

Solution
Given
P(A) = 60% = 0.60
P(B) = 0.80
∴ P(A  B) = P(A) + P(B) - P(A  B)
= P(A) + P(B) - P(A) P(B)
= (0.60) + 0.80 - (0.60) (0 .80)
= 0.60 + 0.80 - 0.48
P(A  B) = 0.92
K-19
Question.4 If the probability of an event A is 0.2 and the probability an event B is 0.3
and the probability of either A or B is 0.5, the A and B are.

A. B. C. D.

Solution
Given

36 | P a g e
P(A) = 0.2
P(B) = 0.3
Then
P(A × B) = P(A) + P(B)
P (A  B) = P(A) + P(B)
0.5 = 0.2 + 0.3
A and B are mutually exclusive events

K-32
Question.5 If P(A) = –0.3 and P(B) = +1.3 then P(A  B) is, when A and B are mutually
exclusive events.

A. B. C. D.

Solution
Given
P(A) = - 0.3
P(B) = +1.3
Then
P(A  B) = P(A) + P(B)
= –0.3 + 1.3
=1
K-48
Question.6 Given P(A) = 0.24, P(B) = 0.39 and P(A  B) = 0.49. If A and B are non-
mutually exclusive events, then P(A  B) is equal to.

A. B. C. D.

Solution
Given
If A and B two not mutually exclusive events then
P(A  B) = P(A) + P(B) – P(A  B)
0.49 = 0.24 + 0.39 – P(A  B)
0.49 = 0.63 – P(A  B)
P(A  B) =0.63 – 0.49
P(A  B) = 0.14

K-49
Question.7 If P(A) = 0.5, P(B) = 0.6 and P(A  B) = 0.4, then P(A  B) is.

A. B. C. D.

Solution

37 | P a g e
Given
P(A  B) = P(A) + P(B) – P(A  B)
P(A  B) = 0.5 + 0.6 – 0.4
P(A  B) = 1.1 – 0.4
P(A  B) = 0.7

K-50
Question.8 If P(A) = 0.20, P(B) = 0.08, and P(A  B) = 0.03 then P(A  B) is.

A. B. C. D.

Solution
Given
P(A  B) = P(A) + P(B) – P(A  B)
= P(A) + P(B) – P(C)
= 0.20 + 0.08 – 0.03
=0.28 – 0.03
P(A  B) = 0.25

K-34
Question.9 Three horses A, B and C are in a race. A is twice as likely to wins as B and B
is three times as likely to wins C. The probability that
(i) A wins is
(ii) P(B or C)
(iii) P(A or B)

A. B. C. D.

Solution
Given
A : B : C
2 : 1
3 : 1
6 : 3 :1
Sum of ratio = 10
6
(i) P(A) = 10 = 3/5
(ii) P(B or C) = P(B) + P(C)
3 1
= +
10 10
4
=
10
= 2/5

38 | P a g e
(iii) P(A or B) = P(A) + P(B)
6 3
= +
10 10
9
=
10

Question.10 One card is drawn from a pack of 52, what is the probability that it is a king
or queen?

A. B. C. D.

Solution
Given
The event “card drawn is king” and “card drawn is a queen” are mutually exclusive
 P(card drawn is a king or q1ueen)
P(Card drawn is king) + P(Card drawn s queen)
4 4
= +
52 52
2
=
13

Question.11 One card is drawn from a pack of 52 cards, each of these 52 cards are
equally likely to be drawn. What is the probability that the card drawn is either red or
king?

A. B. C. D.

Solution

Given
P(card drawn is red or king) = P(card drawn is red) + P(card drawn is king) – P(card drawn is red king)
26 4 2
= + −
52 52 52
28
=
52

Question.12 A bag contains tickets bearing number 1 through 25. A ticket is drawn from
it. Find the probability that the number on the ticket being a multiple of 3 or
7.

A. B. C. D.

Solution

39 | P a g e
Given
25
n(A)= 3 =8
25
n(B)= 8 =8
25
n(P(A  B))=21 =1
8 3 1
= + −
25 25 25
10
=
25
= 2⁄5

Question.13 The probability that a student passes Mathematics is 2/3 and the probability
that he passes English is 4/9. If the probability of passing at least one course
is 4/5. What is the probability of this passing both the courses?

A. B. C. D.

Solution
Given
Let “M” be event passing mathematics and “E” be the event passing English P(M) = 2/3,
P€ = 4/9
P(M or E) = 4/3
Hence P(M or E) = P(M) + P(E) – P(M and E)
4/5 = 2/3 + 4/9 – P(M and E)
P(M and E) = 2/3 + 4/9 – 4/5
= 14/45

Probability with replacement


Event that are take in the probability are completely independent of each other

Question.14 Two cards are drawn from a pack of 52 cards with replacement (i.e., the
second card is drawn after replacing the first card in the pack. Find the
probability
(i) Both are king
(ii) First is king and second is queen
(iii) One is king and other is queen

A. B. C. D.

Solution
Given
(i) P(both cards are king)
= P(first is king and second card is king)

40 | P a g e
= 4/52 × 4/52
= 1/169
(ii) P(first is king and second is queen)
= 4/52 × 4/52
= 1/169
(iii) P(one is king and other is queen)
= P(first is king and second is queen) or (first is queen and the second is king)
= 4/52 × 4/52 + 4/52 × 4/52
= 2/169

Question.15 Two balls are drawn with replacement from a bag containing 3 red and 4
black balls. Find the probability that:
(i) Both are red
(ii) The first is red and second is black.
(iii) One is red and other is black.

A. B. C. D.
Solution
Given
(i) P(both are red) 3/7 x 3/7 = 9/49
(ii) P(first is red and second is black) = 3/7 × 4/7 = 12/49
(iii) P(one is red and other is black)
= P(first red and second black) or (first black and second red)
= 3/7 × 4/7 + 4/7 × 3/7
= 12/49 + 12/49
= 24/49

Question.16 Two cards are drawn without replacement from a pack of 52. What is the
probability that the first is king and second is queen.

A. B. C. D.

Solution
Given
P(first is king) and the( second is queen)
= 4/52 × 4/51
= 4/663
Question.17 An integer is chosen at random from the first 200 positive integers. What is
the probability that the integers chosen is divisible by 6 or 8.

A. B. C. D.
Solution
Given
Sample space = [1, 2, 3 ……… 199, 200]

41 | P a g e
n(S) = 200
let A be the event that the integer is divisible by 6
200
n(A) = [ ] = 33
6
let B the event that integer is divisible by 8.
200
n(B) = [ ] = 25
8
200
n(A ∩ B) = [ ]=8
24
Hence P(A  B) = P(A) + P(B) – P(A  B)
33 25 8
= + −
200 200 200
50
=
200
= 1⁄4
Question.18 A class contain 10 men and 20 women out of which half of men and half
women have brown eyes. Find the probability that a person chosen at random is a man or
has brown eyes.

A. B. C. D.
Solution
Given
Let M represent man and B be the event that the person has brown eyes.
men Women total
Brown eyes 5 10 15
Not brown eyes 5 10 15
Total 10 20 30
P(M  V) = P(M) + P(B) – P(M  B)
10 15 5
= + −
30 30 30
20 2
= =
30 3

Question.19 A sample survey conducted in a city shows that the probability are 0.87, 0.36
and 0.29, that a family randomly chosen will a color TV set, a black-and-
white TV set or both respectively. What is the probability that such a family
will own at least one of the two kind of set?

A. B. C. D.

Solution
Given
let A = Color + V
B = black and white TV

42 | P a g e
P(A) = 0.87
P(B) = 0.36
P(A and B) = 0.29
P(A  B) = P(A) + P(B) – P(A  B)
= 0.87 + 0.36 – 0.29
= 0.94

Question.20 A firm installed two machines U and V on January 1, 2007. The probability
that the machines will break down during the first of operations is 0.2 are 0.1 for machines
U and V respectively. Compute the probability that one of the two machines will break
down during the year.

A. B. C. D.
Solution
Given
P(Break down for U) = (0.2) = P(U)
P(Not break down for U) = P(U̅ ) = 0.8
P(Break down for V) = P(V) = 0.1
P(Not break down for V) = P(V̅) = 0.9
P(One of the two machine)
= P(U) P(V̅) + P(U̅ ) P(V)
= (0.2) (0.9) + (0.8) (0.1)
= 0.18 + 0.8
= 0.26

Question.21 At a university, 40% of the students take statistics without mathematics and
25% take mathematics without statistics. 12% of the students take both. If a students is
random selected from a statistics class. What is the probability that he or she has also taken
mathematics?

A. B. C. D.

Solution
Given
Math Stats with Without Total
With 0.12 0.25 0.37
Without 0.40 0.23 0.63
Total 0.52 0.48 1
0.12
P(Math with Stat) = = 0.2308
0.52
Note: 0.23=0.48-0.25, 0.48=1-0.52, 0.63=1.0.37
Question.22 Three missiles are fired at a target. If the probability of hitting the target are
0.4, 0.5 and 0.6, respectively and if missiles are fired independently, what is
the probability?
(i) That all missiles hits the target.
43 | P a g e
A. B. C. D.

Solution
Given
Let A1, A2, A3 represent the events that the first, the second and the third missile hit the
target respectively then.
P(A1) = 0.40, P(A2) = 0.50, P(A3) = 0.60 and
P(A̅1) 0.50, P(A
̅2) = 0.50 P(A3) – 0.40
(i) P(all the missiles hit the target)
P(A1  A2  A2)
= P(A1) × P(A2) × P(A3)
= (0.4) (0.5) )0.6)
= 0.12

Question.23 A and B play 12 games of chess, of which 6 are won by A, 4 are by B and 2
and in a tie they are agree to play a tournament consisting of 3 games. Find
the probability that A wins all three games.

A. B. C. D.

Solution
Given
6 1
P(A wins any one game) = 12 = 2
4 1
P(B win any one game) = =
12 3
2 1
P(any one game ends in a tie) = =
12 6
Now
P(A wins all three games) = P(A1,  A2  A3)
= P(A1), P(A2), P(A3)
1 1 1
= . .
2 2 2
1
=
8

Question.24 A bag contain 50 bolts and 150 nuts. Half of the bolts and half of the nuts are
rusted. If one item is chosen at random. What is the probability that it is
rusted or it’s a bolt?

A. B. C. D.

Solution
Given
Sample Space = 200 = n(S)

44 | P a g e
Let A be event that item chosen is a bolt and B the event that the item chosen is rusted.
n(A) = 50
n(B) = 100
(A  B) = 25
50
P(A) =
200
100
P(B) =
200
25
𝑃(A  B) =
200
Hence
P(A  B) = P(A) + P(B) – P(A  B)
50 100 25
= + −
200 200 200
125 5
= =
200 8

Question.25 One integer is chosen at random from the number 1, 2, 3, …. 50. What is the
probability that the chosen number is divisible by 6 or by 8?

A. B. C. D.

Solution
Given
Here
n(S) = 50
50
n(A) = [ ]=8
6
50
n(B) = [ ] = 6
8
50
n(A  B) = [ ] = 2
24
Hence
P(A  B) = P(A) + P(B) – P(A  B)
8 6 2
= + −
50 50 50
12
=
50
6
=
25

45 | P a g e
Type:6
CONDITIONAL PROBABILITY
DEPENDENT EVENT

46 | P a g e
Question.1 A pair of dice is thrown and sum of the number on the two dice comes to be
7. What is the probability that the number 3 has comes on one of the dice?

A. B. C. D.

Solution
Given
Let n(S) = 36
A = The number 3 has come(Condition) ={(4,3),(3,4)}=2
B = The sum of number on the two dice is 7
n(B) = (6, 1), (5, 2), (4, 3), (2, 5), (3, 4), (1, 6), = 6
6
P(B) =
36
n(A  B) = [4, 3), (3, 4)]
P(A  B) = 2/36
𝐏(𝐀  𝐁)
Required probability = P(A/B) = 𝐏(𝐁)
2⁄ 2 1
= 36 = =
6⁄ 6 3
36

Question.2 A coin is tossed two times. The toss resulted is one head and one tail. What is
the probability that the first throw resulted in head.

A. B. C. D.

Solution
Given
Sample space = [HH, HT, TH , TT]
n(S) = 4
let A =The first throw result in head(Condition)={HT,HH}
B = one head and one tail
B = [HT, TH]
n(B) = 2
P(B) = 2/4
also
A  B = (HT)
P(A  B) = 1/4
P(A  B)
Required probability = P(A/B) = P(B)
1⁄
= 4
2⁄
4
= 1/2

47 | P a g e
Question.3 A die is rolled. If the outcome is an odd number. What is the probability that
it is a prime number?

A. B. C. D.

Solution
Given
Total number of outcomes = 6
Let
A = The number is prime (Condition)={2,3,5}
B = The outcomes is an odd number = {1, 3, 5}

3
P(B) = 6
2
P(A  B) = P[3, 5] = 6
Required condition probability = P(A/B)
P(A  B)
=
P(B)
2/6
=
3⁄
6
= 2⁄3

Question.4 A pair of dice is thrown and sum of number on the two dice is observed to be
6. What is the probability that there is 2 on one of the dice?

A. B. C. D.

Solution
Given
Let
A =2 on one of the dice (Condition) ={(2,4),(4,2)}
B = Sum of number is 6
Total outcome = 36
B = {[(5, 1), (4, 2), (3, 3), (2, 4), 1, 5)]
5
P(B) =
36
P(A  B) = P[(4, 2), (2, 4)]
= 2/36
Required conditional probability = P(A/B)
P(A  B)
=
P(B)

48 | P a g e
2⁄
= 36
5⁄
36
2
= ⁄5

Question.5 A pair of dice is rolled. If the sum on the two dice is 9, find the probability
that one of the dice showed 3?

A. B. C. D.

Solution
Given
Total outcomes = 6 × 6 = 36
Let
A : one of the dice showed 3(Condtion) ={(6,3),(3,6)}
B : sum of the two dice is 9
4
 P(B) = P{(6, 3), (5, 4), (4, 5), (3, 6)} = 36
2
P (A  B) = P[(6,3), (3, 6)] = 36
Required conditional probability = P(A/B)
P(A  B)
=
P(B)
2⁄
= 36
4⁄
36
2
=
4
1
=
2

Question.6 A number is chosen from the first 50 natural numbers and its appears to be
an odd number. Find the conditional probability that the number is a prime number.

A. B. C. D.

Solution
Given
Total number of outcomes = 50
Let
A: The number is prime(Condition)={2,3,5,7,11,13,17,19,23,29,31,37,41,43,47}
B: The number appears to be odd.
={1,3,5,7,9,11,13,15,17,19,21,23,25,27,29,31,33,35,37,39,41,43,45,47,49}
P(B) = 25/50

49 | P a g e
14
P (A  B) = P[ 3, 5, 7, 11, 13,17, 19, 23, 29, 31, 37, 41, 43, 47] = 50
P (A  B)
∴ P (A⁄B) =
P (B)
14⁄
= 50
25⁄
50
14
=
25
P(A/B) = 3/5

Question.7 If P(A) = 3/8, P(B) = 5/8, and P(A  B) = 3/4 Find P(A/B) and P(B/A). Is the
event independent?

A. B. C. D.

Solution
Given
P(A  B) = P(A) + P(B) – P(A  B)
3 3 5
= + − P(A  B)
4 8 9
8 3
P (A  B) = =
8 2
1
P (A  B) =
2
P(A  B)
P (A⁄B) =
P(B)
1⁄
= 4
5⁄
8
2
= ⁄5
And
P(A  B)
P(B⁄A) =
P(A)
1⁄
= 4
3⁄
8
= 2⁄3
Since P(A/B)  P(A), the event is not independent.

Question.8 If A and B are two independent events such that P(A) = 0.4, P(B) = 0.6, find:
(i) P(A  B)
(ii) P(A/B)

50 | P a g e
A. B. C. D.

Solution
Given
Sine A and B are independent events
P(A  B) = P(A) × P(B)
= 0.4 × 0.6
= 0.24
(i) P(A  B) = P(A) + P(B) – P(A  B)
= 0.4 + 0.6 – 0.24
= 1 – 0.24.
= 0.76
A P(A  B) 0.24
(ii) P (B) = = = 0.4
P(B) 0.6

51 | P a g e
Type:7
PERMUTATIONS

52 | P a g e
Outcomes:
The result of an operation is called outcomes

FUNDAMENTAL PRINCIPLES OF COUNTING


i) Multiplication principle.

Suppose one operation has M possible outcomes and that second operation has N outcomes. The
number of possible outcomes when performing the first operation followed by second operation
is M*N
Example:
Performing one operation and another operation means we multiply the number of possible
outcomes
First operation and second operation
M x N

ii) Addition principle


suppose one operation has M possible outcomes and second operation has N
outcomes the number of possible outcomes of the first operation or the second
operation is given by M+N.

Example:
Performing one operation or another operation means we add the number of possible outcomes.
First operation or 2nd operation
M + N
Example:1
A, B, C, D, E and F are six students. In how many ways can they be seated in a row if
i) There are no restrictions on the seating.
ii) A and B must sit beside each other.
iii) A and B must not sit beside each other.
iv) D, E and F must sit beside each other .
v) A and F must sit at the end of each other.

Solution:
i) No restriction

Number of arrangements = 6*5*4*3*2*1 = 720

53 | P a g e
ii) A and B sit beside each other , consider A and B as one person

A, B, C, D, E, F
The five persons can be arranged = 5! = 120
But A and B can be arranged in = 2! = 2
Thus number of arrangement = 2! * 5! = 240
iii) A and B must not sit beside each other.

(Number of arrangement A and B not together) = (Total number of arrangement – Number of


arrangement A and B together)
= 720 – 240 = 480
iv) D, C and F must sit beside each other.

Consider D, E and F as one person.


A, B, C, D, E, F
The four persons can be arranged=4!
The four person can be arranged in = 3! Ways = 6
Thus number of arrangement = 24 * 6 = 144
v) A,E and F must sit at the end of each row.

B, C, D and E can be arranged in = 4! = 24


A and F can be arranged in = 3! = 6
Total number of arrangement = 24 * 6 = 144
Example: 2
How many different four digits numbers greater than 6000 can be formed using the digits 1, 2,
4, 5, 6, 7 if
i) No digit can be repeated
ii) Repetitions are allowed

Solution:
i) Repetitions are not allowed the number must be greater than 6000 thus first place
can be filled by 2 ways (6 or 7).
2 * 5 * 4 * 3 = 120
ii) Repetition are allowed

54 | P a g e
2 * 6 * 6 * 6 = 432
Example:3

How many different five digits number can be formed from the digits 1, 2, 3, 4, and 5 if
i) There are no restrictions on digits and repetitions are allowed.
ii) The number is odd and no repetition are allowed
iii) The number is even and repetitions are allowed
iv) The number is greater than 50000 and no repetitions are allowed.

Solution:
i) Repetition are allowed

5 × 5 × 5 × 5 × 5 = 3125
ii) Last place can be filled in 3 ways (1, 3, 5).
4× 3 × 2 × 1 × 3 = 72
iii) Last place can be filled in only 2 ways (2 or 4)

5× 5 × 5 × 5 ×2 = 1250
iv) First place can be filled in only 1 way (with 5)

1×4×3×2×1 = 24

55 | P a g e
Permutation (arrangement)
A permutation is an arrangement of objects in a definite order

(i) PERMUTATION OF N DIFFERENT OBJECTS


The number of permutation of n objects taken all at a time, denoted by the symbol
“P” is given by
n
Pn = n!
also number of permutation of n objects taken r at a time is

Example:, say your locker “combo” is 5432. If you enter 4325 into your locker it won’t open
because it is a different ordering (Permutation). The permutations of 2, 3, 4, 5 are:

• 5432, 5423, 5324, 5342, 5234, 5243, 4532, 4523, 4325, 4352, 4253, 4235, 3542, 3524, 3425,
3452, 3254, 3245, 2543, 2534, 2435, 2453, 2354, 2345

(ii) PERMUTATIONS WITH REPETITION OF OBJECT IS ALLOWED


Number of permutation of r object from n object = nr

Example: There are 10 numbers to choose from (0,1,2,3,4,5,6,7,8,9) and we choose 3 of them:

10 10 × ... (3 times) = 103 = 1,000 permutations

(iii) PERMUTATIONS WHEN OBJECTS ARE NOT DISTINCT


The number of permutation of n objects of which P are of first kind and p1 are of second kind and
………Pk
𝒏!
=
𝑷𝟏 ! 𝒙𝑷𝟐 ! 𝒙…..𝑷𝒌!
Points to remember:
Permutation (order matter)
(i) A permutation is an arrangement of items in a particular order, where order matter
and no repetition
(ii) Arranging people, digits, number, alphabets, letter, colour
(iii) Key words (arrange, arrangement)

56 | P a g e
Question.1 Evaluate the following:
(i) 8!
(ii) 7P
6
(iii) 15P
7

A. B. C. D.

Solution
Given
(i) 8! = 40320
7
(ii) P6 = 5040
15
(iii)) P7 = 32432400

Question.2 In how many ways six students can be arranged in a row.

A. B. C. D.

Solution
Given
Number of ways = 6P6 = 720

Question.3 A club consists of four members. How many sample points are in the sample
space when three officers, president, secretary treasurer are to be chosen?

A. B. C. D.

Solution
.
In other words, the number of permutations is 4P3 = 24
K-5
Question.4 If three coins are tossed then the number of outcomes are.

A. B. C. D.

Solution
Given
Number of side of coins = n = 2
Number of coins = r = 3
Number opf ways = nr
= 23
=8

57 | P a g e
K-6
Question.5 If three dice were rolled. Then the number of outcomes are?

A. B. C. D.

Solution
Given
Number of side of dice = n= 6
Number of dices = r = 3
Number of ways = nr
= 63
= 216
K-7
Question.6 If an experiment consists of rolling a dice and then flipping a coin, the
number of outcomes are?

A. B. C. D.
Solution
Given
n1 = Number of side of dice = 6
n2 = Number of sides of a coins = 2
Number of outcome = n1 × n2
=6×2
= 12
K-10
Question.7 If 10P2 =720, then 10C3 = ?

A. B. C. D.

Solution
Given
10
C3 = 120

K-15.4
Question.8 If nP2 = 20, then the value of n =?

A. B. C. D.

Solution
Given
n
P2 = 20
n!
= 20
(n − 2)!

58 | P a g e
n(n − 1)(n − 2)!
= 20
(n − 2)!
n (n – 1) = 20
n (n – 1) = 5 × 4
Compare n = 5

WORDS ARRANGEMENT:

Question.9 Write all the permutation of letter of the word EXAMINATION.

A. B. C. D.

Solution
Given
The given word consists of 11 letters including A twice, N twice and I twice
11!
∴ required number of permutations = = 4989600
2! 2! 2!

Question.10 Write all the permutation of the word EXAMINATION in which vowels are
together.

A. B. C. D.

Solution
Given
There are 6 vowels treat them as one letter only N occurs twice. Therefore, number of
6!
words formed = = 360
2!
Also
6!
Vowels can be arranged themselves in ways 2! 2!
= 180
Total words having vowels together = 360 × 180 = 64800

Question.11 In how many different ways can the letters of the word 'LEADING' be
arranged in such a way that the vowels always come together?

Solution:
The word 'LEADING' has 7 different letters.
When the vowels EAI are always together, they can be supposed to form one letter.
Then, we have to arrange the letters LNDG (EAI).
Now, 5 (4 + 1 = 5) letters can be arranged in 5! = 120 ways.

59 | P a g e
The vowels (EAI) can be arranged among themselves in 3! = 6 ways.
Required number of ways = (120 x 6) = 720.

Question.12 In how many different ways can the letters of the word 'CORPORATION' be
arranged so that the vowels always come together?

Solution:
In the word 'CORPORATION', we treat the vowels OOAIO as one letter.
Thus, we have CRPRTN (OOAIO).
This has 7 (6 + 1) letters of which R occurs 2 times and the rest are different.
7!
Number of ways arranging these letters = = 2520.
2!
Now, 5 vowels in which O occurs 3 times and the rest are different, can be arranged
5!
in = 20 ways.
3!
Required number of ways = (2520 x 20) = 50400.

Question.13 In how many ways can the letters of the word 'LEADER' be arranged?

Solution:
The word 'LEADER' contains 6 letters, namely 1L, 2E, 1A, 1D and 1R.
6!
Required number of ways = = 360.
(1!)(2!)(1!)(1!)(1!)

Question.14 In how many different ways can the letters of the word 'DETAIL' be arranged
in such a way that the vowels occupy only the odd positions?

Solution:
There are 6 letters in the given word, out of which there are 3 vowels and 3 consonants.
Let us mark these positions as under:
(1) (2) (3) (4) (5) (6)
Now, 3 vowels can be placed at any of the three places out 4, marked 1, 3, 5.
Number of ways of arranging the vowels = 3P3 = 3! = 6.
Also, the 3 consonants can be arranged at the remaining 3 positions.

60 | P a g e
Number of ways of these arrangements = 3P3 = 3! = 6.
Total number of ways = (6 x 6) = 36.
Question.15 How many 4-letter words with or without meaning, can be formed out of the
letters of the word, 'LOGARITHMS', if repetition of letters is not allowed?
Solution:
'LOGARITHMS' contains 10 different letters.
Required number of words = Number of arrangements of 10 letters, taking 4 at a time.

= 10P4

= (10 x 9 x 8 x 7)
= 5040.

NUMBER ARRANGEMENT

Question.16 How many 3-digit numbers can be formed from the digits 2, 3, 5, 6, 7 and 9,
which are divisible by 5 and none of the digits is repeated?

Solution:
Since each desired number is divisible by 5, so we must have 5 at the unit place. So, there is 1
way of doing it.
The tens place can now be filled by any of the remaining 5 digits (2, 3, 6, 7, 9). So, there are 5
ways of filling the tens place.
The hundreds place can now be filled by any of the remaining 4 digits. So, there are 4 ways of
filling it.
Required number of numbers = (1 x 5 x 4) = 20.

Note: repetition means (111 - 232 -544) not allowed

Question.17 How many three digits number can be formed from 5 numbers (1,2,3,4,5)
(i) Repetition not allowed
(ii) Repetition allowed

Solution: (i) Repetition not allowed

5ways x 4ways x 3ways =60 ways

(ii)Repetition allowed
5ways x 5 ways x 5ways =125

Question.18 How many three digits odd number can be formed from five number

61 | P a g e
(1,2,3,4,5)

Solution:
4 x3 x 1 (Fix) =12

4 x3 x 3 (Fix) =12

4 x3 x 1 (Fix) =12
Total 36
Question.19
How many arrangements can be made of the letter P, Q, R, S, T, taking two letters at a
time? If no letter can be rejected.
Solution:
5 120
P2 = 6 = 20

Question.20
There are 20 situations on a railway line. How many different kinds of single first class
tickets must be printed so as to enable a passenger to go from one station to another.
Solution:
20
P2 = 380
Question.21
Four travelers arrive in a town where there are six hotels. In how many ways can they take
their quarters each at different hotel?
Solution:
6
P4 = 360
Question.22
In how many ways can 8 mangoes of different sizes be distributed amongst 8 boys of
different ages so that the largest one is always given to the youngest boy?
Solution:
no of ways =
1×7×6×5×4×3×2×1
= 5040
Question.23 Find the number of different numbers of 4 digits that can be formed with the
digits 1, 2, 3, 4, 5, 6, and 7. The digits in the unit place being different and the digit in the
unit place being always 7.
Solution:

62 | P a g e
no. of ways = 1×6×5×4 = 120
Question.24
how many ways different odd numbers of 4 digits can be formed with the digits 1, 2, 3, 4, 5,
6, 7 the digits in any number being all different?
Solution:
4 5 6 7 (one way) = 120
4 5 6 5 (one way) = 120
4 5 6 3 (one way) = 120
4 5 6 1 (one way) = 120
Total number of ways = 480
Question.25
in how many ways can the colors of rainbow be arranged, so that the red and blue colors
are always together?
Solution:
colors of Rainbow = 7
Red and blue together = 6! = 720
Red and blue arrangement = 2! = 2
Number of ways = 720×2 = 1440
Question.26
how many numbers lying between 1000 and 2000 can be formed from the digits 1, 2, 4, 7,
8, 9 each digit not occurring more than once in number.
Solution:
number lies between 1000 and 2000 are four-digit numbers of which
1 5 4 3
× × ×
𝑤𝑎𝑦 𝑤𝑎𝑦 𝑤𝑎𝑦 𝑤𝑎𝑦
Number of ways = 60 ways
Question.27
in how many ways can 7 papers be arranged so that the best and worst paper never comes
together.
Solution:
Number of arrangements = 7! = 5040
Best and worst comes together = 6! × 2! = 1440
Best and worst never comes together = 5040 – 1440 = 3600

63 | P a g e
Question.28
in how many ways can the letter of the word (MONDAY) be arranged. how many of them
begin with M. how many of them do not begin with M, but end with Y.
Solution:
i) Monday can be arranged = 6! = 720
ii) Monday can be arranged starting with M = 5! = 120
iii) Monday can be arranged not starting with M but ending with Y.
4! × 4 = 24 × 4 = 96
Question.29
in how many ways can five boys form a Ring.
Solution:
number of ways = 4! = 24
Question.30 in how many ways 5 different beads be strung on a necklace.
Solution:
(𝑛−1)! 4!
Number of beads = 2 = = 12 𝑤𝑎𝑦𝑠
2!

Question.31
in how many ways can the color of the rainbow be arranged so that the red and blue colors
always repeated
Solution:
=6! * 2! =1440
Question.32
in how many ways can the seven color of the rainbow be arranged so that the blue and
green colors never comes together?
Solution:
Total number of ways=7! =5040
Number of ways in blue and green together =6! * 2! =1440
Number of ways in which blue and green never comes together = 5040 -1440 =3600
Question.33
How many 5 digits number , divisible by 3 ca you make using the digits 0,1,2,3,4 and with
repeating any digit
Solution:
Divisible Rule
Sum of all digits should multiple of 3

64 | P a g e
0,1,2,3,4,5
Take five number
Sum of number =1+2+3+4+5=15
Divisible by 3,so
5*4*3*2*1 =120

Also
Take five number
0,1,2,4,5
Sum of numbers =0+1+2+4+5=12
Divisible by 3
Without zero(4)*4(included zero)*3*2*1 =96

Total number =120+96=216

Question.34
A 5-digits pin code can begin with any number except 0 and 1. If repetition of the same
digit is allowed then probability of a pin code beginning with a 7 and ending with an 8 is
Solution:
Number of possible codes
8 x10 x10x10x10 =80,000
Number starting with 7 an ending with 8 is
1 x10x10x10x1 =1000
Probability = 1000/80000 =1/80

65 | P a g e
EXTRA PRACTICE QUESTIONS

66 | P a g e
Question.1 Three cards are selected from a pack of 52 cards. What is the probability that two are
of same suit (type)?

Solution:
13𝑐2∗ 39𝑐
1
= 4. 52𝑐3

234
= 425

Question: 2 Two numbers are chosen from 1-20. What is the probability that they are consecutive
numbers?

Solution:
19𝑐1
= 20
𝑐2

19
= 190
1
= 10

Question: 3 During a T20 cricket match Naeem scored 47 runs in Six overs with the help
of five fours and three singles. If a T.V channel were to show two of his scoring shots
during its news up date and the shots were to be selected on random basis. Find the prob of
rather of the following.
i) On both the shots Naeem had scored different runs.
ii) On one of the shots he had hit a six.

Solution:
Fours Sixes Singles Total Selected
5 4 3 12 2
5 4 5 3 4 3
𝑐∗ 𝑐 + 𝑐∗ 𝑐 + 𝑐 ∗ 𝑐
1 1 1 1 1 1
i) P(both shots different runs =
12𝑐2

47
= 66
4 8
𝑐∗ 𝑐
1 1
ii) P (one shot six and other not six) = 12𝑐2

32
= 66

Question: 4 if one ticket is drawn at random from 20 tickets numbered 1 to 20. The prob
that it is multiple of 5 or 7 is.

67 | P a g e
Solution:
20 4 1
n(5) = n (A) = = 4 , p (A) = 20 = 5
5
20 2 1
n (7) = n(B) = = 2 p (B) = 20 = 10
7

Now
P (5 or7) = p (5) + p (7)
1 1
= 5 + 10
2+1
= 10
3
= 10

Question: 5 A bag contain 6 red and 8 green balls. If two balls are drawn at random then
what is the prob that one is red and other is green.
Solution:
6 8
( )( ) 6∗8 48
1 1
P (A) = 14 = = 91
( ) 91
2

Question: 6 from 8 men and 12 women a committee is to be selected. If 6 people are


chosen at random. What is the prob that committee consist of 4 men and 2 women.
Solution:
8 12
( )( )
4 2
P (A) = 20
( )
6
70∗66
P (A) = 38760

Question: 7 Out of all integers 1 to 100. An integer is chosen at random. What is the prob
that the selected numbers is not divisible by 14.
Solution:
n (s) = 100
A = (14, 28, 42, 56, 70, 84, 98)
n (A) = 7
OR
100
n (A) = =7
14

68 | P a g e
7
p (A) = 100
7
p (𝐴̅) = 1 - 100
93
= 100

Question: 8 A fair dice is rolled thrice. What is the prob that each time six will appear?
Solution:
1
P (A) = 216

Question: 9 If two dices are rolled. Find the prob that either the sum of two will be seven
or at least one of the dice will show the number 5.
Solution:
n (S) = 36
Event = A = [ (1,6) , (2,5), (3,4), (4,3), (5,2), (6,1), n (A) = 6
B = [ (1,5), (2,5), (3,5), (4,5), (6,5), (5,1), (5,2), (5,3), (5,4), (5,5),(5,6)]
n(B) = 11
𝐴 ∩ 𝐵 = {(2,5),(5,2)}

p (AU B) = p (A) + p (B) - 𝑃(𝐴 ∩ 𝐵)


6 11 2
= 36 + 36 -36
15
P (AUB) = 36

Question: 10 if A , B and c are three mutually exclusive and exhaustive events such that
𝟏 𝟏
p(c) = 𝟐 p(A) = p (B) then p (B)
𝟑

Solution:
1 1
p(c) = 2 p(A) = p (B)
3
1 1
6 * 3 p (c) = 6 * 2 p (A) = 6 * p(B)

2 p (c) = 3 p(A) = 6 PB
Since total prob is equal to one
6 p(B) = 1
1
P (B) = 6

69 | P a g e
Question: 11 A ball is drawn at random from a box containing 10 red, 30 white , 20 blues
and 15 orange balls. The prob of ball drawn is red or orange is.
Solution:
R W B O Total
10 30 20 15 75
10 15
P (red) + p (orange) = 75 + 75
25 1
= 75 = 3

Question: 12 Three seats on a sofa set can be occupied by 4 persons


Solution(order )
= 4𝑃3 = 12

Question: 13 Selected
In how many ways a team of 4 players be chosen from a total of 10 persons.

Solution:
10𝐶4 = 210

Question: 14 Selected
In how many ways a committee of 4 consisting of 2 men and 2 women can be chosen from 5
men and 3 women.

Solution:
= 5𝐶2 . 3𝐶2 = 10 × 3 = 30

Question: 15 If one ticket is drawn at random from 20 tickets numbered 1 to 20. The probability
that it is multiple of 5 or 7 is

Solution:
Sample Space ={1,2,3…. 20}
= 𝑃(5, 10, 15, 20) + 𝑃(7, 14)

= 4⁄10 + 2⁄20 = 6⁄20

Question: 16 If 10 electronic bulbs, three are defective but it is not known which are defective. In
how many ways can three bulbs be selected.

Solution:
103 = 120

70 | P a g e
Question: 18 A box contains six blue pens and four red pens. In how many ways can three pen be
drawn in order to get at least one blue pen.

Solution:
(6𝐶1 × 4𝐶2 ) + (6𝐶2 × 4𝐶1 ) + (6𝐶3 × 4𝐶0 )

= 36+60+20 = 116

Question: 20 Two cards are drawn from a well shuffled pack of 52 cards. What is the probability
that they are both aces if the first card is replaced (with replacement).

Solution:
4 4 1
= 52 × 52 = 169

Question: 21 A box contains 4 red and 3 green balls. Another box contains 2 red and 1 green. A ball
is selected at random from each bag. Find the probability that both are different color.

Solution:

. Box – 1 Box - 2

Red Green Red Green

4 3 2 1
1 1
= =
7 3

1 1 10
= + =
7 3 21
Question: 22 The marks obtained by 10 students are 25, 30, 21, 55, 47, 10, 15, 17, 45, 35. What is
the probability that a students at random has marks more than the average marks.

Solution:
Average = 30

𝑛(𝐴) = (47, 55, 45, 35)

𝑃(𝐴) = 4⁄10

𝑃(𝐴) = 0.4

Question: 23 The probability that man will be alive in 40 years is 𝟑⁄𝟓 and the probability that his
wife will survive 40 years is 𝟐⁄𝟑. find the probability that both will be alive.

Solution:

71 | P a g e
3 2 6
𝑃(𝐴 ∩ 𝐵) = 𝑃(𝐴) − 𝑃(𝐵) = 5 × 3 = 15

Question: 24. A chance of getting scholarship is 0.6 and B’s chance of getting the same is 0.3. the
probability that at least one of them will get the scholarship is

Solution:
= 𝑃(𝐴 ∩ 𝐵) + 𝑃(Ā ∩ 𝐵) + 𝑃(𝐴 ∩ 𝐵)

= (0.6)(0.7) + (0.3)(0.4) + (0.6)(0.3)


= 0.42 + 0.12 + 0.18
= 0.72

Question: 25. The prob that A speak truth is 0.6 and the probability That B speaks truth is 0.7. what
is the probability that both contradict each other narrating the same incident?

Solution:
= 𝑃(𝐴 ∩ Ḃ) + 𝑃(Ā ∩ 𝐵)
= 𝑃(𝐴). 𝑃(𝐵 − ) + 𝑃(Ā). 𝑃(𝐵).
= (0.6)(0.3) + (0.4)(0.7)
= 0.18 + 0.28 = 0.46

Question: 26. A box contain 8 red and 12 white balls of which 2 red and 5 white are marked. If a red
ball is selected from the box what is the probability that it is marked.

Solution: Double Multiple object


R W total

M 2 5 7

Unmarked 6 7 13

8 12 20
2
𝑃(𝑅𝑒𝑑) = 8 = 1⁄4

Question: 27 If the A and B are two independent events (Not mutually exclusive) and P(A) = 0.4and
𝑷(𝑨 ∪ 𝑩) = 𝟎. 𝟕 then 𝑷(𝑩) =?

Solution:
𝑃(𝐴 ∩ 𝐵) = 𝑃(𝐴). 𝑃(𝐵) = 0.4𝑃(𝐵)
𝑃(𝐴 ∪ 𝐵) = 𝑃(𝐴) + 𝑃(𝐵) − 𝑃(𝐴 ∩ 𝐵)
0.7 = 0.4 + 𝑃(𝐵) − 0.4𝑃(𝐵)
0.3 = 𝑃(𝐵)(1 − 0.4)

72 | P a g e
0.3
0.6
= 𝑃(𝐵)

0.5= 𝑃(𝐵)

Question: 29 An urn contains 9 red and 7 white balls. Three balls are drawn at random without
replacement what is the prob that the first two balls drawn are red and 3rd ball drawn is white.

Solution:
9 8 7
= 16 . 15 . 14
504
= = 3⁄20
3360

Question: 30 A bag contain 3 white, 4 black, 5 red balls. Three balls are drawn at random from the
bag. What is the prob that they are of different color.

Solution:
3𝐶1 ×4𝐶1 ×5𝐶1 3
𝑃(𝐴) = 12𝐶3
= 11

Question: 31 A coin and a die are thrown simultaneously the prob of obtaining head on a coin and
number greater than 4 on the dice is

Solution:
𝑛(𝑠) = 𝑆𝑎𝑚𝑝𝑙𝑒 𝑠𝑝𝑎𝑐𝑒 = 2 × 6 = 12
𝑛(𝐸) 𝐸𝑣𝑒𝑛𝑡 = (𝐻, 5), (𝐻 ,6 ) = 2
2 1
𝑃(𝐴) = =
12 6

Question: 32. A wallet contain 6 notes of Rs 100, 4 notes o Rs 50, 2 notes of Rs 10. If 3 notes are
selected randomly from the wallet. Find the probability that no note of Rs 100 is selected

Solution:

Rs. 10 Rs. 50 Rs. 100 Total


2 4 6 12
6𝐶0 ×6𝐶3 20 1
Also 𝑃(𝐴) = 12𝐶3
= 220 = 11

Question: 33 What is the probability that a leap year is selected at random will have 53 Monday?
Solution:
52 complete week with two days over in 7 days.

One leap year = 366 days

No. of week in a year = 52

73 | P a g e
52 weeks = 52× 7 = 364 days

Difference between leap year and 52 weeks

= 366 – 364 = 2
2
Now 𝑃(𝐴) = 7

Note:

i. Independent -----→ Not mutually exclusive


ii. Dependent ---→ mutually exclusive
iii. With replacement ----→ independent
iv. 𝑃(𝐴 ∪ 𝐵) = 𝑃(𝐴) + 𝑃(𝐵) − − − − − − − − − − − −→
𝑑𝑒𝑝𝑒𝑛𝑑𝑒𝑛𝑡 𝑎𝑛𝑑 𝑚𝑢𝑡𝑢𝑎𝑙𝑙𝑦 𝑒𝑥𝑐𝑙𝑢𝑠𝑖𝑣𝑒

Question: 34 A bag contains 5 white and 3 black balls. Two ball are drawn one after the other
without replacement find the probability that both balls are white.

Solution:
Total balls = 8
5 4
𝑃(𝐴) = 8 . 7
5
𝑃(𝐴) = 14

Question: 35 Three cards are selected one by one with replacement from a pack of 52 cards. What
is the prob that all are aces?

Solution:
4 4 4
𝑃(𝐴) = × ×
52 52 52
64
𝑃(𝐴) = 140608
1
𝑃(𝐴) = 2197

Question: 36 A problem in statistics is given to three students whose chances of solving the
𝟏 𝟏 𝟏
problems are 𝟑 , ,
𝟐 𝟓
𝒓𝒆𝒔𝒑𝒆𝒄𝒕𝒊𝒗𝒆𝒍𝒚. What is the probability that the problem will be solved?

Solution:
. Note: (At least one will be solved)

𝑃(𝐴) = 1 − 𝑃(𝑁𝑜𝑛𝑒)

𝑃(𝐴) = 1 − 𝑃(Ā ∩ Ḃ ∩ Ḉ)
2 1 4
𝑃(𝐴) = 1 − (3 × 2 × 5)

74 | P a g e
8
𝑃(𝐴) = 1 − 30
22
𝑃(𝐴) =
30
11
𝑃(𝐴) = 15

Question: 37 If events A and B are non -mutually exclusive events and𝑷(𝑨) = 𝟎. 𝟐𝟎, 𝑷(𝑩) = 𝟎. 𝟔𝟎
Then 𝑷(𝑨 𝒐𝒓 𝑩) is

Solution:
𝑃(𝐴 ∪ 𝐵) = 𝑃(𝐴) + 𝑃(𝐵) − 𝑃(𝐴 ∩ 𝐵)
𝑃(𝐴 ∪ 𝐵) = 0.2 + 0.6 − (0.2 × 0.6)
𝑃(𝐴 ∪ 𝐵) = 0.2 + 0.6 − 0.12
𝑃(𝐴 ∪ 𝐵) = 0.68

Question: 38 A box in hard ware store, contains 150 bolts and 250 nuts. One fifths of the bolts and
three of fifth of the nuts are defective. What percentage of items is either a defective item or a bolt.

Solution:
180 150 30
𝑃(𝑑𝑒𝑓𝑒𝑐𝑡𝑖𝑣𝑒 𝑜𝑟 𝑏𝑜𝑙𝑡𝑠) = 400 + 400 − 400
300
= 400
3
=4
3
= 4 × 100%

= 75%

Question: 39 Two horses A and B run a race. If the prob of A’s wins is twice that of B and there is no
tie, the prob of B’s win is

Solution:
We known

𝑃(𝐴) + 𝑃(𝐵) = 1
2𝑝(𝐵) + 𝑃(𝐵) = 1
3𝑃(𝐵) = 1

𝑃(𝐵) = 1⁄3

Question: 40 What is the probability that 3 children selected random would have born on different
days of the week.

75 | P a g e
Solution:
7 6 5
𝑃(𝐴) = 7 . 7 . 7
30
= 𝑃(𝐴) = 49

Question: 41 An NGO has been studying the literacy level among the male and female adults in a
certain village. 6 out of every 10 adults in the village are males. It is observed that 5 out of 10 adults
males are illiterate and 8 out of 10 adults females are illiterate if an adults is randomly selected and
found to be illiterate. Then what is the prob that the adult is a female.

Solution:
Solution:
males females total

literate 30 8 38

Illiterate 30 32 62

Total 60 40 100
32
𝑃(𝑖𝑙𝑙𝑖𝑡𝑒𝑟𝑎𝑡𝑒 𝑓𝑒𝑚𝑎𝑙𝑒) = = 0.52
62
Question: 42 The prob that there is at least one error in a documents typed by 3 persons A, B, and c
are 0.1, 0.05, 0.15 respectively. If A, B and C prepare 50, 40 and 60 such documents then their
expected number of documents without error is

Solution:
. 𝑥 Documents with error Documents without error Expected value

50 0.1 1 − 0.1 = 0.9 0.9 × 50 = 45

40 0.05 1 − 0.05 = 0.95 0.95 × 40 = 38

60 0.15 1 − 0.15 = 0.85 0.85 × 60 = 51

𝐸𝑥𝑝 (𝑥) = 134

Question: 43 A die is tossed twice. If it shows the same number on two dice. Ramesh gets 100
otherwise he losses Rs. 5 what is his expectation.

Solution:
Same number on two dice= ((1,1), (2,2), (3,3), (4,4), (5,5), (6, 6))

𝑃(𝐴) 1⁄6

𝑥 𝑃(𝑥) 𝑥 𝑝(𝑥)

76 | P a g e
100 1⁄ 100⁄
6 6
-15 (1-p)5⁄6 −25⁄
6
𝐸(𝑥) = 12.5

Question: 44 A company has two machines. Machine (A) produces red toys and machine (B)
produces blue toys. Defective units produced by machine A and B average 25% and 30% respectively
and their daily production is 20 and 30 units respectively. If the daily production is mixed and three
toys are selected at random without replacement. Find the probability that the toy is defective unit.
The second toy is not defective whereas the third toy is defective of blue color.

Solution:

Items Total Defective Not defective


Red toys 20 5 15
Blue toys 30 9 21
50 14 36

i) Defective toy, Now defective Toy, blue defective toy


14 36 9
= × ×
50 49 48

= 0.03857
= 0.03857 × 100%

= 3.85%.

Question: 45 In a group of 100 CA students 30 are male, 55 are studying for certificate level and 6
of male students are not studying for certificate. A student is chosen at random is female. What is that
prob that she is not studying certificate level.

Solution:
male female total

Studying 24 31 55

Not studying 6 39 45

30 70 100

𝑃(𝐴) = 39⁄70 = 0.55

77 | P a g e
Empirical (or Experimental)
Probability When
you
gather data from observations during an
experiment, you will be calculating an empirical
(or experimental) probability.

The empirical (or experimental) probability of an


event is an "estimate" that an event will occur
based upon how often the event occurred after
collecting data from an experiment in a large
number of trials. This type of probability is based
upon direct observations. Each observation in an
experiment is called a trial.

Example:
A survey was conducted to determine Answer: There were 12 + 15 + 24 +
students' favorite brands of sneakers. 26 + 13 = 90 "trials" in this experiment
Each student chose only one brand from (each student's response was a trial).
the list of brands A, B, C, D, or E.
What is the probability that a student's 26 out of the 90 students chose brand
favorite sneaker was brand D? D.
Sneaker A B C D E The probability is :

Number 12 15 24 26 13

78 | P a g e
Theoretical Probability
With theoretical probability, you do not actually
conduct an experiment. Instead, you use what you
know about the situation to determine the
probability of an event occurring. You may use
your reasoning skills or an existing formula to
arrive at your answer.

The theoretical probability of an event occurring


is an "expected" probability based upon
knowledge of the situation. It is the number
of favorable outcomes to the number of possible
outcomes.

Example:
Find the probability of rolling a Answer: No experiment is needed. There are
6 on a fair die. 6 possible outcomes when rolling a die: 1, 2,
3, 4, 5, and 6.
The only favorable outcome is rolling a 6.

The probability is :

Under the best circumstances, we would


expect to roll one 6 out of every 6 rolls.

79 | P a g e
Q1. The set of all possible outcomes of a statistical experiment is called:
Data Statistical data
Sample space None of these
Q2. Something which can take different values with different probabilities is called:
A variable A random variable
A discreet variable A continuous variable
Q3. If two events have no elements in common than they are known as:
Union of two events Intersection of two events
Mutually exclusive events None of these
Q4. If two events have at least one element In common than the events are said to be:
Union of two events Intersection of two events
Not mutually exclusive events None of these
Q5. Which of the following applies to discreet as well as continuous variables?
Binomial distribution Normal distribution
Chi Square distribution (b) and (c)
Q6. The arrangement of some or all of the objects without considering the order of
arrangement is called:
Selection Permutation
Combination None of these
Q7. If two or more events cannot occur simultaneously then they are called:
Independent Mutually exclusive
Not equally likely None of these
Q8. If the probability of an event is 0.01, which of the following statements is correct:
The event is unlikely to occur The event is expected to occur about 10% of
the time
The event cannot occur None of these
Q9. If a coin is tossed five times, the probability of:
Five heads is equal to the probability of five Five heads is equal to the probability of zero
tails tails
Five heads is equal to the probability of zero All of the above
heads
Q10. The probability of two mutually exclusive events is:
P (A ∪ B ) P (A ∩B)
P (A)+P(B) None of these
Q11. The probability of two non-mutually exclusive events is:
P (A ∪ B ) P (A ∩B)
P (A)+P(B) -P (A ∩B) None of these

80 | P a g e
Q12. The probability of two independent events is:
P (A ∩B) P (A) * P (B)
P (A) * P(B/A) None of these
Q13. The probability of two non-independent events if the event A occurs first is:
P (A ∩B) P (A) * P (B)
P (A) * (B/A) None of these
Q14. The sum of the probabilities of two compliment events is always:
Zero 1
Does not exist None of these
Q15. The probability of an impossible event is always:
Equal to zero Less than zero
More than zero None of these
Q16. The probability of a certain event is always:
Equal to zero Equal to one
More than one None of these
Q17. The probability of an event A, given that an event B has occurred, is denoted as:
P (A ∩B) P (A/B )
P (B/A ) None of these
Q18. A random variable is also called a:
Chance variable Stochastic variable
Constant Both “ a” and “b”
Q19. When the possible outcomes of an experiment are equally likely to occur, this will apply:
Relative probability Subjective probability
Conditional probability Classical probability
Q20. The probability based on the concept of relative frequency is called:
Empirical probability Statistical probability
Both a and b Neither a nor b
Q21. A random experiment contains:
At least one outcome At least two outcome
At most one outcome At most two outcomes
Q22. The outcomes of tossing a coin is a:
Mutually exclusive event Compound event
Certain event Simple event
Q23. The events having no experimental outcomes in common is called:
Equally likely events `exhaustive events
Mutually exclusive event Independent events

81 | P a g e
Q24. A set of outcomes formed after some additional information is called:
Sample space Reduced sample space
Null set Random experiment
Q25. The probability associated with the reduced sample space is called:
Conditional probability Statistical probability
Mathematical probability Subjective probability
Q26. If three candidates are selected to attend a course from the ten candidates, the number of
ways Type equation here.of selecting the candidates is an example of:
Combination Permutation
Reduced sample space Both (a) and (b)
Q27. When the occurrence of one event has no effect on the probability of the occurrence of
another event, the events are called:
Independent Dependent
Mutually exclusive Equally likely
Q28. Which statement is false?
The classical definition applies when there areThe empirical definition occurs when number
“n” equally likely outcomes to an experiment of times an event happen is divided by the
number of observations
A subjective probability is based on whatever The general rule of addition is used when
information is available the events are mutually exclusive
Q29. The probability can be broadly divided into branch namely:
Subjective and objective probability Statistical and mathematics probability
Deduction and non- deduction probability None of these
Q30. The compliment of an event ‘A’ is given by 𝐴̅ which of the following is true:

P (A) + P (𝐴̅) = 0 P (A)+ P (𝐴̅) = 1


̅
P (A & 𝐴 )= 0 Both (a) & (c)
Q31. The two events are mutually exclusive if:
Their probabilities are less than 1 Their probabilities sum equal to 1
They cannot occur at the same time They contain every possible outcomes of an
experiment
Q32. If P(A/B) = P (A) then A and B are:
Dependent Independent
Conditional Mutually exclusive
Q33. Expected value of a random variable:
Is always positive May be positive or negative
May be positive or negative or zero None of these

82 | P a g e
Q34. Definition of probability states that probability is a numerical measure of the likelihood
or chance of occurrence of a (n):
Dependent events Mutually exclusive events
Certain event Uncertain event
Q35. If two events A and B are independent, then:
They can be mutually exclusive They can’t be mutually exclusive
They cannot be exhaustive None of these
Q36. Two events A and B are such that their joint probability is equal to the product of their
marginal probabilities. Which of the following describes events A and B?
A and B are mutually exclusive events They can’t be mutually exclusive
They cannot be exhaustive A and B are independent events
Q37. Whi8ch of the following is true with regard to the classical approach to probability?
Assumes that the outcomes are not equally The probability of an event is determined after
likely performing the experiment large number of
times
The probability of an event is determined It assumes that all possible outcomes of the
before performing the experiment experiment are not known
Q38. The possible outcomes favorable for an event and the total number of outcomes are
known without performing the experiment in case of:
Classical probability Marginal probability
Relative probability Subjective probability
Q39. In which of the following cases does the probability estimate of the occurrence of a
particular event differ from one decision- maker to another?
Classical probability Marginal probability
Subjective probability None of these

83 | P a g e
1/1/2022 CHAPTER-12
Probability Distribution

ABDUL AHAD BUTT


[COMPANY NAME]

1|Page
BINOMIAL DISTRIBUTION

It is a discrete probability distribution and applied when the number of trials is small and trial are
independent.

1. BINOMIAL DISTRIBUTION
If a random experiment has two possible outcomes “Success” and “Failure” In tossing a coin,
success may be a head and failure may be a tail. In production area good items are treated as
success and defective items may be treated as failure the random experiment with two outcomes
are calculated through
n
P(X = x) = ( ) Px qn-x
x
This is called Binomial Probability Distribution.
2. PROPERTIES
(i) The outcome of the experiment may be divided into two categories i-e success and failure
(ii) The prob of each outcome remain the same throughout the trial.
(iii) All the trials are independent.
(iv) The field of interest is called success.
(iv) The experiment is repeated for definite small members of times
(v) It has two parameters “n” and “p”
(vi) Its mean =np and variance =npq
(vii) If p=q=0.5 binomial distribution is symmetric then mean=median =mode otherwise when
p<0.5, it is skewed
(viii) In binomial distribution mean always greater than variance

ICAP-91
Write a short note on the Binomial Distribution.

A. B. C. D.

Solution
Suppose P is probability of success and q the probability of failure in a single trial. Then probability of x
success out of ‘n’ trial is
n
P(X = x) = ( ) Px qn-x
x
where
x = 0, 1, 2, … is
and P + q = 1
it is discrete probability distribution and applied when the number of trials is small and trials are
independent. More over the trial may be classified in to two categories (success and failure). The
probability of success remains same throughout the experiment

Question.1 If 20% of the bolts produced by a machine are defective, determine the probability
that out of 4 bolts chosen at random.
(i) Zero bolt is defective
(ii) 2 Bolts are defective

A. B. C. D.

2|Page
Solution
Given
P = 20% = 0.20
n=4
q = 1 – p = 0.80
(i) Zero bolt is defective
n
P(x = 0) = ( ) Po qn-o
o
4
= ( ) (0.20)0 (0.80)4−0
0
= (1) (1) (0.4096)
= 0.4096
(ii) 2 Bolts are defective
4
P(x = 2) = ( ) (0.20)2 (0.80)4.2
2
= (6) (0.04) (0 .64)= 0.1536

Question.2 Find the probability i.e., tossing a fair coin four time there will appear.
(i) 4 heads
(ii) at least two heads

A. B. C. D.

Solution
Let x denote the number of heads then
n = 4, p = 1/2, q = 1/2

we know that
n
P(x = x) = ( ) Px qn-x
x

(i) 4 heads
4
P(x = 4) = ( ) (1/2)4 (1/2)4-4
4
1
P( x = 4) = 1 x × 1 = 1/16
16
(ii) At least two heads
P(x ≥ 2) = P(x = 2) + P(x = 3) + P(x = 4)
4 1 2 1 4−2 4 1
3
1 4−3 4 1
4
1 4−4
= ( ) ( ) ( ) + ( )( ) ( ) + ( )( ) ( )
2 2 2 3 2 2 4 2 2
1 1 1 1 1
= 6 × × ⁄4 + = 4 × × + 1 × × 1
4 8 2 16
6 4 1
= + +
16 16 16
11
=
16

K-16.1
Question.3 If n = 5, P = 0.2, then P(x = 2) is

A. B. C. D.

Solution
3|Page
Given
n
P(X = x) = ( ) Px qn-x
x
5
P(x = 2) = ( ) (0.2)2 (0.8)5-.2
2
P(x = 2) = (10 (0.04) (0.512)
P (x = 2) = 0.2048

K-16.2
Question.4 If n = 5, P = 0.2, then P(x > 3).

A. B. C. D.

Solution
Given
n = 5, P = 0.2, q = 0.8
n
P(x = n) = ( ) Px qn-x
x
P(x > 3) = P(x = 4) + P(x = 5)
= 5 𝐶4 (0.2)4 (0.8)5.4 + 5 𝐶5 (0.2)5 (0.8)5−5
(5) (0.0016) (0.8) + (1) (0.00032) (0.8)0
= 0.0064 + 0.00032
= 0.00672
Question.5 The national pass rate for an examination is 40%. A school enters 6 candidates.
Calculate the probability that (i) 2 candidates will pass (ii) 5 candidates will pass. Explain why the
probability of all passing is not equal to the probability of all failing.

A. B. C. D.

Solution
Given
Let P denote the probability of passing then
P = 0.40 , q = 1-p = 0.6 , n = 6
6
(i) P(2 candidates will pass) = ( ) (0.4)2 (0.6)6-2
2
P(x=2)= 15 (0.4)2 (0.6)4
P(x=2)= 0.31104
= 0.31
6
(ii) P(5 candidate will pass) = ( ) (0.4)5 (0.6)6.-5
5
P(x=5)= 6(0.4)5 (0.6)
= 0.036864
= 0.04
(iii) Since P = 0.4 , q = 0.6 , probability are not equal. Also probability of all passing would
be equal to the probability of all failing when
P = q = 0.5

Question: 6 Among from a department’s 16 trucks. 5 emit excessive amount of smoke. If eight of
the trucks are randomly selected inspection. What is the probability that this sample
will include at least 4 of the trucks, which emit excessive amount of smoke?
4|Page
Solution:
5 5 11
P= 16
, q = 1- 16 = 16 , n = 8 and p(x≥ 4)
𝑛
Using p(X = x) = ( ) 𝑝 𝑥 𝑞 𝑛−𝑥
𝑥
P(x≥ 4) = P(x=4) + P(x=5) + P(x=6) + P(x=7) + P(x=8)
8 5 11 8 5 11 8 5 11 8 5 11 8 5 11
P(x≥ 4) = ( ) (16)4 (16)4 +( ) (16)5 (16)3 +( ) (16)6 (16)2 + ( ) (16)7 (16)1 +( ) (16)8 (16)0
4 5 6 7 8

P(x≥ 4) =0.1491+0.0542+0.0123+0.0016 +0.00009

P(x≥ 𝟒) =0.21729
Question: 7 Inam is going to play a series of 3 tennis matches with misbah. Assuming that
chances of winning and losing are equal. Find the following probabilities for inam.

(i) Two wins and one loss.


(ii) At least two wins.
(iii) No loss
(iv) At most two losses.

Solution:
1 1
P = 2, q = 2 , n = 3
1 1 3
i) P(x = 2) = 3𝑐2 (2)2 (2) = 8
ii) P(x≥ 2) = p(x=2) + p(x = 3)
1 2 1 1 3 1 0
= 3𝑐2 (2) (2) + 3𝑐3 (2) (2)
3 1 4 1
=8 +8=8=2
1 3 1 0 1
iii) P(x = 3) = 3𝑐3 (2) (2) = 8
iv) P(At most two losses) =P(x<2) = 1 –p(x = 0)
1 0 1 3−0
= 1 - 3𝑐3 ( ) ( )
2 2
1
=1-
8
7
=8
Question: 8 A multiple choice examination consist of ten questions and each question is followed
by four choices. A student will pass the examination if he answer five questions correctly. Assuming
that a student knows two correct answers and chooses the remaining answers at random. What is
the probability that he will pass the test?

Solution:
1
P = 4 = 0.25, q = 0.75, n =8
Now
P(pass) = p(x ≥ 3) = 1 – p(x< 3)
= 1 - [𝑝(𝑥 = 0) + 𝑝(𝑥 = 1) + 𝑝(𝑥 = 2)]
8 8 8
𝑥 (0.75)8−0 1 (0.75)8−1
= 1 – [𝑐 (0.25) +𝑐 (0.25) + 𝑐 (0.25)2 (0.75)8−2 ]
0 1 2

5|Page
= 1 - [0.3115 + 0.2670 + 0.1001]
= 1 – 0.6786
= 0.3214

Question: 9 In a T20 cricket match between falcon club (FC) and Eagle club(EC). The
probability to win by (FC) is 0.4. In a series of five t20 matches. Find the probability
that FC would win
i) Exactly two matches
ii) At least two matches
iii) Less than four matches

Solution:
P = 0.4, q = 0.6, n = 5
5
i) P(x =2) = 𝑐 (0.4)2 (0.6)5−2 = 0.3456
2
ii) P(x≥ 2) = 1 – p(x<2)
= 1 - [𝑝(𝑥 = 0) + 𝑝(𝑥 = 1)]
5 5
= 1 - [𝑐 (0.4)0 (0.6)5−0 + 𝑐 (0.4)1 (0.6)5−1 ]
0 1
= 1 - [0.0778 + 0.2592]
= 1 – 0.3470
= 0.663
iii) P(x<4) = 1 – p(x≥ 4)
= 1 - [𝑝(𝑥 = 4) + 𝑝(𝑥 = 5)]
5 5
= 1 - [𝑐 (0.4)4 (0.6)5−4 + 𝑐 (0.4)5 (0.6)5−5 ]
4 5
P(x<4) = 1 - [0.0768 + 0.0102]
P(x < 4) = 0.913

Question: 10 A local news channels has conducted an opinion poll for constructing more dams in
the country. The poll result indicates that 70% of the participating viewer support
the idea. 15% are against the idea and 15% are undecided. If a sample of six
participating viewer is selected at random, determine the probability that
i) At least five viewers will support the idea.
ii) Less than two viewers will not support the idea.
Solution:
P = 0.7, q = 0.3, n=6
Now
p(x≥ 5) = p(x=5) + p(x =6)
6 6
= 𝑐 (0.7)5 (0.3)6−5 + 𝑐 (0.7)6 (0.3)0
5 6
= 0.3025 + 0.1176
= 0.4201
Given
P = 0.3, q = 0.7, n=6
6|Page
Now
P(x <2) = p(x =0) + p(x=1)
6 6
= 𝑐 (0.3)0 (0.7)6 + 𝑐 (0.3)1 (0.7)5
0 1
= 0.1176 + 0.3025
= 0.4201

Question: 11 A missile radar detection systems consist of two radar screens. The probability that
any of the radar screens will detects an incoming missile is 0.95. if a missile enters
the detection space of this radar. What is the probability that at least one of the
radar screens will detect it (assume that the radar detections are independent
events)?

Solution:
P = 0.95, q = 0.05, n = 2
Now
P(x≥ 1) = p (x =1) + p(x =2)
2 2
P(x≥ 1) = 𝑐 (0.95) (0.05) + 𝑐 (0.95)2 (0.05)0
1 1

1 2
P(x≥ 1) = 0.095 + 0.9025
P(x≥ 1) = 0.9975

Question: 12 From the past records it has been determined that 60% of the college’s students
applications are approved. If ten applications are chosen at random. Compute the following
i) Probability that eight or more applications are approved
ii) Number of applications that are expected to be approved.
iii) Standard deviation of the number approved out of ten applications.

Solution:
P = 60% = 0.6, q = 1 – p = 0.4, n = 10
i) P(x ≥ 8) = p (x = 8) + p(x =9) + p(x =10)
10 10 10
= 𝑐 (0.6)8 (0.4)2 + 𝑐 (0.6)9 (0.4)1 + 𝑐 (0.6)10 (0.4)0
8 9 10
= 0.12093 + 0.04031 + 0.00605
= 0.16729
ii) Mean (Number of application expected to be approved)
𝜇 = np = 10(0.6) = 6
iii) Standard deviation
= √𝑛𝑝𝑞 = √(10)(0.6)(0.4) = 1.549

Question: 13 The whole seller of a toy has 1000 pieces in a stock out of which 200 pieces are
slightly defective mixed randomly with each other. A retailer buy one dozen toys.
What is the probability that exactly 10 toys are defect free?
Solution:

7|Page
Total pieces = 1000
Defective pieces = 200
200
P(defective) = 1000 = 0.2
q = 0.8
n = 12
Now
12
P(x =10) = 𝑐 (0.2)10 (0.8)2
10
= 0.28347

Question: 14 Seventy five percent of youths (12-15) year of age have a blood pressure less than
136. What is the probability that a sample of 12 youths of that age group will
include
i) Exactly 4 who have a blood pressure greater than 136?
ii) At least 4 who have a blood pressure greater than 136?
Solution:
Blood pressure greater than 136 = p = 0.25
Blood pressure less than 136 = q = 0.75 and n=12
12
i) P(x = 4) = 𝑐 (0.25)4 (0.75) = 0.1936
4

ii) P(x ≥ 4) = 1 – p(x < 4)


= 1- [𝑝(𝑥 = 0) + 𝑝(𝑥 = 1) + 𝑝(𝑥 = 2) + 𝑝(𝑥 = 3)]

12 12 12
=1 - [ 𝑐 (0.25) (0.75) + 𝑐 (0.25) (0.75) + 𝑐 (0.25)2 (0.75)10 +
0 12 1 11

0 1 2
12
𝑐 (0.25)3 (0.75)9 ]
3

= 1 - [0.0317 + 0.1267 + 0.2323 + 0.2581]


= 1- 0.6488
= 0.3512

Question: 15 Suppose a poll of 10 employees is taken in a large company. The purpose is to


determine ‘x’. The number who favors unionization. Suppose that 40% of all
company’s employees favor unionization.
i) Find Mean and standard deviation of x
ii) Find the probability that x <4
iii) Find the probability that x > 6

Solution:
Given
P = 40% , = 0.4, q = 0.6 , n = 10
i) Mean = n p = 10 (0.4) = 4

8|Page
Standard deviation = √𝑛𝑝𝑞 = √10(0.4)(0.6) = 1.55.
ii) P (x < 4) = p (x = 0) + p(x =1) + p(x =2) + p(x=3)
10 10 10 10
= 𝑐 (0.4)0 (0.6)10 + 𝑐 (0.4)1 (0.6)9 + 𝑐 (0.4)2 (0.6)8 + 𝑐 (0.4)3 (0.6)7
0 1 2 3
= 0.006046 + 0.040310 + 0.120932 + 0.214990
= 0.3822806

iii) P (x > 6) = p(x =7) + p (x =8) + p (x = 9) + p (x =10)


10 10 10 10
P (x >6) = 𝑐 (0.4)7 (0.6)3 + 𝑐 (0.4)8 (0.6)2 + 𝑐 (0.4)9 (0.6)1 + 𝑐 (0.4)10 (0.6)0
7 8 9 10
P (x >6) = 0.04246 + 0.010616 + 0.001572 + 0.000104
P (x > 6) = 0.05476

Question: 16 Current medical studies show that 30 percent of the population will suffer from
common cold each winter. A group of 12 people is randomly selected.
i) What is the probability that exactly five in the group will have the common cold this
winter.
ii) What is the probability that at least 5 in the group will have the common cold this
winter
iii) Complete the mean and variance of the number in the group that will have the common
cold this winter.

Solution:
P=30%=0.3 ,q=0.7 ,n =12
Question: 17 the probability that a computer recovers from a rare virus attacks is 0.4. If 15
computers are known to have contracted with this virus. What is the probability
that
i) At least 12 computers survive.
ii) From 3 to 8 computers survives.

Solution:
P=0.4,n=15,q=0.6,
P(x≥2) and (3≤x≥8)
Question: 18 in a graduate school of business an acceptable rate of 30% was reported. If 10
applicants are selected at random. Find the probability that
i) More than 8 were accepted
ii) Fewer than 8 were accepted
iii) Between 6 and 8 (including 6 and 8) were accepted

Solution:
P=0.3,q=0.7,n=10
P(x≥8) , P(x<8), P(6≤x≤8)
Question: 19 A firm of chartered Accountant has two vacancies for trainee students and it trying
to recruit foundation passed students. In the past 40 percent of the students who were offered the
training contracts have not reported to join. If 2 students are offered training contract. What is the
probability that at least one will join.
9|Page
Solution:
P=0.4, q=0.6, n=2, P(x≥1)

Question: 20 Management believes that 20% of the people watching television advertisement
about their new product will purchase the product. Five persons who watched their
advertisement are picked at random. What is the probability that 3 persons will
purchase the product?
Solution:
P=0.2,q=0.8,n=5, P(x=3)
Question.21 A die is rolled five times and a 5 or 6 is considered success. Find the probability of
(i) no success (ii) at least 2 success (iii) at least one but not more than 3 success.

A. B. C. D.

Solution
Given
Here P = 2/6 = 1/3 , q = 2/3
(i) P(x = 0)
(ii) P(x ≥ 2) = 1 – P(x < 2)
(iii) P(1 ≤ x ≤ 3) = P(x = 1) + P(x = 2) + P(x = 3)

Question.22 Using Binomial distribution, find the probability of


(i) 3 success in 8 trails when P = 0.4
(ii) 2 or fewer successes in 9 trials when P = 0.4

A. B. C. D.

Solution
Given
Here
(i) P = 0.4, q = 0.6, n = 8, x = 3
P(x = 3) = ?
(ii) P(x ≤ 2) = ? n = 9, P = 0.4, q = 0.6

Question.23 Find the probability of getting (i) Exactly 4 heads and (ii) not more than 4 head
when 6 coins are tossed.

A. B. C. D.

Solution
Given
Here
(i) p = 1/2, q = 1/2, n = 6, P (x = 4)
(ii) n = 6, p = 1/2, q = 1/2, P(x ≤ 4)

Question.24 If the probability of getting caught copying someone else’s exam is 0.2. Find the
probability of not getting caught in 3 attempts.

10 | P a g e
A. B. C. D.
Solution
Given
Here
p = 0.2, q = 0.8, n = 3, P(x = 0)

Question.25 If 60% of the voters in a large district prefer candidate A, what is the probability
that in a sample of 12 votes exactly 7 will prefer A?

A. B. C. D.
Solution
Given
Here
n = 12, P(x = 7), P = 0.60, q = 0.40

Question.26 The probability that a patient recovers from a delicate heart operation is 0.9. What
is the probability that exactly five of the next 7 patients having this operation
survive?

A. B. C. D.

Solution
Given
P = 0.9, q = 0.1, n = 7, P(x = 5) = ?

Question.27 An insurance salesman sells policies to 5 men, all of identical age and in good health.
According to the actuarial tables. The probability that a man of this particular age
will alive 30 years hence is 2/3. Find the probability that in 30 years (i) all men (ii) at
least 3 men (iii) only two men (iv) at most one man will be alive.

A. B. C. D.

Solution
Given
(i) Here
P = 2/3 , q = 1/3, n = 5 , P(x = 5)
(ii) P (x ≥ 3) = P(x = 3) + P (x = 4) + P(x = 5)
(iii) P(x = 2), n = 5, P = 2/3, q = 1/3,
(iv) P(x ≤ 1) = P(x = 0) + P(x = 1)

11 | P a g e
MEAN AND VARIANCE OF BINOMIAL DISTRIBUTION

91
Question.1 In a binomial distribution P = 0.2 and n = 10, Find its mean.

A. B. C. D.

Solution
Given
P = 0.2, n = 10
Mean = np = 10 × 0.2= 2
K-3
Question.2 If mean of Binomial Distribution is 1.0 and q = 0.98 then n =?

A. B. C. D.

Solution
Given
µ = 1.0, q = 0.98, n=?

Now
µ = np
µ = n(1 - q)
1.0 = n(1 - 0.98)
1.0 = n(0.02)
1.0
=n
0.02
n = 50
K-4
Question.3 If mean of Binomial Distribution is 1 and variance is 0.98 then P is

A. B. C. D.

Solution
Given
µ=np = 1 (i)
Variance = npq (ii)
putting value of nP in (ii),
0.98 = (1) q
q = 0.98
1 - p = 0.98
1 - 0.98 = p
0.02= p….
K-5
Question.4 If mean of Binomial Distribution is 1 and variance is 0.98, the q is

A. B. C. D.
12 | P a g e
Solution
Given
µ=np = 1
also
variance = npq
0.98 = npq
0.98 = 1q
q = 0.98

K-6
Question.5 If n = 10 and P = 0.9, the mean and variance of Binomial; Distribution is:

A. B. C. D.

Solution

Given
n = 10, P = 0.9, q = 0.1
Then
µ = np = (10) (0.9)= 9
and
Variance = npq
= (10) (0.9) (0.1)= 0.9

Question.6 Is it possible to have a binomial distribution with mean = 5 and s.d = 3?

A. B. C. D.
Solution
Given
np = 5 and √nPq = 3
npq = 9
Dividing
npq 9
=
np 5
q = 1.8
Which is greater than unity.
Hence it is not possible to have a binomial distribution with mean = 5 and s.d = 3

Question.7 The probability of a defective bolt is 0.2. Find the mean and S.D for the distribution
of defective bolts in a total of 1000.

A. B. C. D.

Solution
Given
p = 0.2, q = 0.8 , n = 1000
13 | P a g e
Mean = np = 1000 × .2 = 200

S.D = √npq = √1000 × 0.2 × 0.8 = √160 = 12.6

Question.8 Of the bolts produced by a factory 2% are defective. In a shipment of 3600 bolts from the
factory. Find expected number of defective bolts and standard deviation.
Solution
P = 0.02
q = 0.98
n = 3600

Now
µ = np = 3600*0.02
µ = 72
And
S.D = √𝑛𝑝𝑞 = √3600 ∗ 0.02 ∗ 0.98
S.D = 8.4

14 | P a g e
EXPECTED VALUE
K-25
Question.1 In a game, a man is paid Rs.50 if he gets all heads or all tails when 4 coins are tossed
and he pays out Rs.30 if 1, 2 or 3 heads appear. His expected gain is.

A. B. C. D.

Solution
Given
Trial X Calculation Frequency P(x) xP(x)
No head or 4 tails 50 4! 1 1/16 50/16
0! ∗ 4!
1 head or 3 tails -30 4! 4 4/16 -120/16
1! ∗ 3!
2 head or 2 tails -30 4! 6 6/16 -180/16
2! ∗ 2!
3 heard or 1 tail -30 4! 4 4/16 -120/16
3! ∗ 1!
4 heads or no tail 50 4! 1 1/16 50/16
4! ∗ 0!
16 1 ∑xP(x) = -20

Now
E(x) = ∑xP(x) = ˗20

K-27
Question.2 Let x assumes the values 0, 1, 2 and 3 with the respective probabilities 0.51, 0.38,
0.10 and 0.01. The mean of distribution is

A. B. C. D.

Solution
Given

X P(x) xP(x)
0 0.51 0
1 0.38 0.38
2 0.10 0.20
3 0.01 0.03
1 ∑xP(x) = 0.61

Now
Expected value = E(x) = mean = ∑xP(x)
= 0.61

Question.3 Let x assumes the value 0, 1, 2 and 3 with the respective probability 0.51, 0.38, 0.10
and 0.01. The variance of distribution is

15 | P a g e
A. B. C. D.

Solution
Given
X P(x) xP(x) x2p(x)
0 0.51 0 0
1 0.38 0.38 0.38
2 0.10 0.20 0.4
3 0.01 0.03 0.09

∑P(x) = 1 ∑xP(x) = 0.61 ∑x2P(x) = 0.87

Now
Variance = [E(x2)] – [E(x)]2

Variance = 0.87 – (0.61)2


= 0.87 – 0.3721
= 0.4979

Question.4 By investing into a particular stock, a person can make a profit in 1 year of Rs.5000
with probabilities 0.4 or take a loss of Rs.1500 with probability 0.8 the person’s
expected gain is?

A. B. C. D.

Solution
Given
Prob of success = P(A) = 0.4
Prob of failure = P(B) = 0.8
 total prob = P(A) + P(B)
= 0.4 + 0.8
= 1.2
Which is impossible because total prob is always is less than equal to one.

16 | P a g e
POISSON DISTRIBUTION

DISCRETE Distribution
When number of trials increases very large and probability of success is very small, the appropriate
distribution is named as poison distribution.
e−μ . 𝜇 𝑥
P(x = x) = where
x!
μ = Mean or average of random variable
(i) It is used when “n” is large and P is small. The question is how much large be in and how small
be P the experiment shows that if in is greater than 20 and P is 0.05 or less. The poison
approximation is good for binomial here µ = up.

PROPERTIES:

(i) Poison distribution has only one parameter i.e., µ.


(ii) Value of e =2.7182
(iii) Mean of poison distribution = E(x) = µ
(iv) Variance of Poisson distribution is equal to mean i.e., variance = µ

Question.1 Two hundred passengers have made reservation for an airplane flight. If the
probability that a passenger who has a reservation will not show up is 0.01. What is the probability
that exactly three will not show up?

A. B. C. D.

Solution
Given
P = 0.01 , n = 200
Since n is very large and P is very small use poison distribution.
 µ = np = (200 (0.01) =2
=2
Using
μx e−μ
P(x = 3) =
x!
(2)3 .e−2 1
= 3!
∴ e−2 (2.71828)2
= 0.1353
(8)(0.1353)
= 6
= 0.1804

K-16-11
Question.2 If µ = 1.2 then P(x = 0) is

A. B. C. D.

Solution
Given

17 | P a g e
We known
e−μ . μx
P(x = x) =
x!
e−1.2 . (1.2)0
P(x = 0) =
0!
(0.301194) (1)
=
1
= 0.301194
K-12
Question.3 If µ = 1.2 the P (X > 0) is

A. B. C. D.

Solution
Given
We know
e−μ . μx
P(x = x) =
x!
 P(x > 0) = 1 – P (x ≤
0)
e−1.2 . (1.2)0
= 1−
0!
(0.301194) (1)
P(x > 0) = 1 −
1
= 1- 0.301194
P (x > 0) = 0.698806

Question.4 In a certain manufacturing process. 5% of the tools produced turn out to be


defective. Find the probability that in a sample of 40 tools, at most 2 will be defective
(e-2 = 0.135)

A. B. C. D.

Solution
Given
Here
µ = np = 40 x 0.05 = 2
P(at most 2 defective) = P(x ≤ 2)
P(x = 0) + P(x = 1) + P(x = 2)
e−2 . 20 e−2 . 21 e−2 . 22
= + +
0! 1! 2!
= 0.135 + 0.135 × 2 + 0.135 × 2
= 0.675

Question.5 Ten percent of the tool production in a certain manufacturing process turn out to be
defective. Fine the probability that in a sample of 10 tools chosen at random, exactly
18 | P a g e
two will be defective by using (i) the binomial distribution (ii) the poison
approximation to the binomial distribution.

A. B. C. D.

Solution
Given
P=0.1, q=0.9,n=10,x=2
Binomial
(i) P(2 defective tool into) = 10c2 (0.1)2 (0.9)8
= 0.1937
Poisson
(ii) Let µ be the mean of the poison distribution.
µ = np = 10 (0.1) = 1
μ2 . e−μ
 P(X=2) =
2!
(1)2 . e−1 1
= 2!
= 2e
1
= 2(2.718)
= 0.1839
Question.6 A secretary makes 2 errors per page on the average. What is the probability that on
the next page? He makes (i) 4 or more errors?

A. B. C. D.

Solution
Given
Here µ = 2
(i) P(x ≥ 4) = 1 – P(x < 4)
= 1 – [P(x = 0) + P(x = 1) + P(x = 2) + P(x = 3)]
(ii) P(x = 0) = e-2 = 0.1357

Question.7 A car hire firm has 2 cars. Which it hires out day by day. The number of demands
for a car each day is distributed as a poison distribution with parameter 1.5.
Calculate the proportion of days on which neither car is used, and proportion of
days on which some demand is refused.

A. B. C. D.

Solution
Given
Here
µ = 1.5
Then
e−1.5 . (1.5)x
P(X = x) =
2!
(i) Proportion of days on which neither car is used
e−1.5 . (1.5)x
P(x = 0) = = 0.2231
0!
19 | P a g e
(ii) Proportion of days on which some demand is refused i.e., P(x > 2) = 1 – P(x ≤ 2)
= 1 – [P(x = 0) + P(x = 1) + P( x = 2)

= 0.1913

Question.8 A manufacturing of cotter pins known that 5 per cent of his product is defective. If
the sells cotter pins in boxes of 100 and guarantees that not more than 4 pins will be
defective. What is the approximate probability that a box will fail to meet the
guaranteed quality?

A. B. C. D.

Solution
Given
Here
P = 0.05, n = 100
µ =nP = 0.05 x 100 = 5
e−5 . 5𝑛
P(x = x) = =
x!
Also P(x > 4) = 1 – P (x ≤ 4)
= 1 – [P(x = 0) + P(x = 1) + P(x = 2) + P(x = 3) + P(x = 4)]

Question: 9 Small insurance company receives an average of 8 insurance claims daily during the
month of Ramadan every year using Poisson distribution. Find the probability that
on a certain day in Ramadan the company will receive
i) No insurance claims
ii) Less than four insurance claims
iii) At least two insurance claims
Solution:
𝜇=8
−𝜇 𝑥
.
𝑒 𝜇
𝑖) p(x=x) =
𝑥!
𝑒 −8 . 80
𝑝 (𝑥 = 0) = 0!= 0.00033546
𝑖𝑖) p (x <4) = p(x=0) + p(x =1) + p(x=2) + p (x =3)
𝑒 −8 . 80 𝑒 −8 . 8 𝑒 −8 . 82 𝑒 −8 . 83
= 0!
+ 1!
+ 2!
+ 3!
−8 512
= 𝑒 [1 + 8 + 32 + 6 ]
= 0.04238
𝑖𝑖𝑖) p (x ≥ 2) = 1 – p(x < 2)
= 1 − [𝑝 (𝑥 = 0) + 𝑝 (𝑥 = 1)]
𝑒 −8 . 80 𝑒 −8 . 81
= 1 - [ 0! + 1!
]
= 1 – 0.00319
= 0.99681

Question: 10 The average number of traffic accidents on a certain section of highway is two per
week. Assuming that the number of accidents follows a poison distribution. Find the
probability of at most 3 accidents on this section of highway during a two-week
period.

Solution:
𝜇 = n p = 2 (2) = 4
20 | P a g e
As total chance (probability) of accidents is 2.
Now
P (x ≤ 3) = p (0) + p (1) + p (2) + p (3)

𝑒 −4 . 40 𝑒 −4 . 41 𝑒 −4 . 42 𝑒 −4 . 43
= 0!
+ 1!
+ 2!
+ 3!
= 0.0183 + 0.0733 + 0.1465 + 0.1954

P (x≤ 3) = 0.4335

Question: 11 Out of every 4000 shirts made at a garment factory. 22 are defective using
poison distribution. Find the probability that a lot of 300 shirts contain.

i) More than 3 defective shirts


ii) Less than 2 defective shirts

Solution:
22
P = 4000
n = 300
22
So 𝜇 = n p = 4000 * 300 = 1.65
i) P (x ≥ 3) = 1 – p (x ≤ 3)
= 1 - [𝑝 (𝑥 = 0) + 𝑝 (𝑥 = 1) + 𝑝(𝑥 = 2) + 𝑝 (𝑥 = 3)]
𝑒 −1.65 (1.65)0 𝑒 −1.65 (1.65)1 𝑒 −1.65 (1.65)2 𝑒 −1.65 .(1.65)3
=1-[ + + + ]
0! 1! 2! 3!
=1- [𝑒 −1.65
( 1 + 1.65 + 1.36125 + 0.7486875]
= 1 – 0.914
= 0.086
ii) P (x < 2) = p (x = 0) + p (x =1)
𝑒 −1.65 .(1.65)0 𝑒 −1.65 .(1.65)1
= +
0! 1!
= 0.51

Question: 12 The PABX systems of Saleem industries receives an average of two calls in
every three minutes. Assuming an approximate poison distribution. What is
the probability that five or more calls will be received during a period of nine
minutes?
Solution:
𝜇=2*3=6
Now
P (x≥ 5) = 1 – p(x < 5)
= 1 – [𝑝 (𝑥 = 0) + 𝑝 (𝑥 = 1 ) + 𝑝 (𝑥 = 2) + 𝑝 (𝑥 = 3 ) + 𝑝 (𝑥 = 4)]
𝑒 −6 .60 𝑒 −6 .61 𝑒 −6 .62 𝑒 −6 .63 𝑒 −6 .64
=1-[ + + + + ]
0! 1! 2! 3! 4!
[𝑒 −6
=1- ( 1 + 6 + 18 + 36 + 54]
= 1 – 0.2875
= 0.7125

21 | P a g e
Question: 13 A loan officer in a bank estimate that the probability of default by an
applicant is 0.025. Last month the bank approved 40 loan applications using
poison distribution. Find

i) The probability that 3 loans will be defaulted


ii) The probability that at least 3 loans will be defaulted
Solution:
𝜇 = n p = 40 (0.025) = 1, n = 40
𝑒 −1 .(1)3
i) P (x = 3) = = 0.0613
3!
ii) P (x ≥ 3) = 1 – p (x< 3)
= 1 - [𝑝 (𝑥 = 0) + 𝑝 (𝑥 = 1 ) + 𝑝 (𝑥 = 2)]
𝑒 −1 .(1)0 𝑒 −1 (1)1 𝑒 −1 .(1)2
=1-[ + + ]
0! 1! 2!
=1- [𝑒 −1
(1 + 1 + 0.5)]
= 1 – 0.9197
= 0.0803

Question: 14 The probability that a car will have a flat tyre while driving through a
certain tunnel is 0.00006. Use the poison approximation to the binomial distribution to find
the probability that at least 2 out of 10,000 cars passing through this tunnel. Will have flat
tyre?

Solution:
𝜇 = n p = 10,000 (0.00006) = 0.6
Now
P (x ≥ 2) = 1 – p (x < 2)
= 1 - [𝑝 (𝑥 = 0) + 𝑝 (𝑥 = 1)]
𝑒 −0.6 (0.6)0 𝑒 −0.6 . (0.6)1
=1-[ + ]
0! 1!

= 1 - [𝑒 −0.6 (1 + 0.6)]

= 1 - [𝑒 −0.6 (1.6)]

= 1 - [(2.71828)−0.6 (1.6)]

= 1 – 0.8781

= 0.1219

Question: 15 It is estimated that 0.5% of the caller to a department will receive a busy
signal using the poison distribution to find the probability that of 1200 caller
at least 5 will receive a busy signal.
Solution:
P = 0.5% = 0.005, n = 1200
𝜇 = n p = 1200 (0.005) = 6
22 | P a g e
Now
P (x≥ 5) = 1 – p (x < 5)
= 1 - [𝑝 (𝑥 = 0) + 𝑝 (𝑥 = 1) + 𝑝 (𝑥 = 2) + 𝑝 (𝑥 = 3) + 𝑝 (𝑥 = 4)]
𝑒 −6 .60 𝑒 −6 .61 𝑒 −6 . 62 𝑒 −6 − 63 𝑒 −6 .64
P (x≥ 5) = 1 - [ + + + + ]
0! 1! 2! 3! 4!

= 1 - [𝑒 −6 + 6𝑒 −6 + 18𝑒 −6 + 36𝑒 −6 + 54𝑒 −6 ]


= 1 - 𝑒 −6 [1 + 6 + 18 + 36 + 54]
= 1 - 𝑒 −6 (115)
= 1 – 0.2875
P (x ≥ 5) = 0.7125
NOTE: this question can also be solved by binomial distribution.

Question: 16 It is known that 20 percent of all persons given a certain medication get
drowsy within two minutes. Find the probability that among 14 persons
given the medication.
i) At most two will get drowsy within two minutes.
ii) At least five will get drowsy within two minutes.
iii) Two, three or four will get drowsy within two minutes.
Solution:
P = 20% = 0.2, n= 14
𝜇 = n p = 0.2 *14 = 2.8
i) P (x ≤ 2) = p (0) + p(1) + p (2)
𝑒 −2.8 (2.8)0 𝑒 −2.8 . (2.8)1 𝑒 −2.8 (2.8)2
= + +
0! 1! 2!
−2.8 [1
=𝑒 + 2.8 + 3.92]
−2.8
=𝑒 (7.72)
= 0.4694
ii) P(x ≥ 5) = 1 – p(x ≤ 4)
= 1 - [𝑝 (𝑥 = 0) + 𝑝(𝑥 = 1) + 𝑝(𝑥 = 2) + 𝑝 (𝑥 = 3) + 𝑝 (𝑥 = 4)]
𝑒 −2.8 (2.8)3 𝑒 −2.8 (2.8)4
= 1 - [0.4694 + + ]
3! 4!
−2.8
= 1 - [0.4694 + 𝑒 (3.66 + 2.56)]
= 1 - [0.4694 + 0.3782]
= 0.1524
iii) P (2 or 3 or 4) = p (2)+ p(3) + p(4)
𝑒 −2.8 (2.8)2 𝑒 −2.8 (2.8)3 𝑒 −2.8 .(2.8)4
= + +
2! 3! 4!
= 𝑒 −2.8 [3.92 + 3.66 + 2.56]
= 0.6165

Question: 17 On a busy airport an Aeroplan lands after an average of 15 minutes. Based


on poison distribution what is the probability that in a 15 minutes interval 3
or more areophane’s will land (𝒆−𝟏 = 𝟎. 𝟑𝟔𝟕𝟗)

23 | P a g e
Solution:
𝜇 = n p= -1 (given)
Now
P (x≥ 3) = 1 – p (x <3) = 1 - [𝑝 (𝑥 = 0) + 𝑝(1) + 𝑝 (2)]
𝑒 −1 .(−1)0 𝑒 −1 .(−1)1 𝑒 −1 (−1)2
=1-[ + + ]
0! 11 2!

= 1 - 𝑒 −1 [1 + 1 + 0.5]
= 1 - 𝑒 1 (2.5)
= 1 – 0.91975
= 0.08025

K-13
Question.18 If e-4.5 =0.0111 and X=0,then the mean of poison distribution is

A. B. C. D.

Solution
Given
We know e-4.5 =0.0111 and x=0
e−μ . μx
P(x = x) =
x!
e−μ . (𝜇)0
P(x = 0) =
0!
−μ
e . 1
P (x = 0) =
1
Computer with
0.0111 = e-4.5
µ = 4.5
K-14
Question.19 If e-6.0 = 0.002478 and P(x = 0) then value of mean is

A. B. C. D.

Solution
Given
We known
e−μ . μx
P(x = x) =
x!
e−μ . (𝜇)0
P(x = 0) =
0!
−μ
e . 1
P (x = 0) =
1
P(x = 0) = e-µ
Computer with
0.002478 = e-6.0

24 | P a g e
…µ = 6….

K-16
Question.20 e-6 = 0.002478, then variance of passion distribution is

A. B. C. D.

Solution
e-6 = 0.002478
µ=6
also in poison distribution mean = variance = 6

K-17
(𝟏.𝟓)𝟒 . 𝐞−𝐲
Question.21 If P(x = 4) = and y=1.5 then prove that P(x = 4) = 0.04706
𝟒!

A. B. C. D.

Solution
Given
(1.5)4 . e−y
= 0.04706
4!
L.H.S
put
y = 1.5
(1.5)4 . e−1.5
P(x=4)= 4!
(5.0625) (0.2231)
P(x=4)= 24
= 0.04706
= R.H.S
K-18
(3.0)2 . e−3
Question.22 if P(x = y) = = 0.07468 then y =
y!

A. B. C. D.

Solution
Given
(3.0)2 . e−3
P (x = y) = = 0.07468
y!
As we know
e−μ . μx
P (x = x) =
x!
Compare
y=2

25 | P a g e
Question.23 In Poisson distribution the probability of 0 success is 10%. Find the mean of the
distribution.

A. B. C. D.

Solution
Given
Prob of 0 success = e-µ = 10%
e-µ = 0.1
(2.7183)-µ = 0.1

Taking log on both side


-µ log (2.7183) = log 0.1
-µ (0.4343) = -1
1
μ=
0.4343
µ = 2.3025

Question.24 If mean of a poison distribution is 4. Find standard deviation.

A. B. C. D.

Solution
Given
µ=4
This S.D = √μ = √4 = 2

Question.25 The member of minor injuries a football coach can expect during the course of a
game is a random variable having the poison distribution with µ = 4.4. Find the
probability that during the course of a game there will be at the most three minor
injuries.

A. B. C. D.

Solution
Given
µ = 4.4 , x = 0,1, 2, 3 and P(x≤3)
Put in
P(x = 0) = (4.4)0 . (e-4.4) /0! = 0.012
P(x = 1) = (4.4)1 . e-4.4 /1! = 0.053
P(x = 2) = (4.4)2 . (e-4.4)/2! = 0.116
P(x = 3) = (4.4)3 (e-4.4) /3! = 0.170
Hence
0.012 + 0.053 + 0.116 + 0.170 = 0.351

26 | P a g e
HYPER GEOMETRIC PROBABILITY DISTRIBUTION
Discrete Probability
• In Binomial prob distribution all the successive trials are independent and prob of success remain
the same from one trial to another trial.
• In hyper geometric distribution the trials are dependent
• Probability of success changes on each trial then the distribution is called Hyper geometric
probability distribution.
• The experiment is repeated for a number of times
• Suppose we have a population of N units classified as K success es and N-K failures and
a sample of n units containing x successes and (n-x) failures is drawn without
replacement, then the probability of x successes is given by the formula

k N−k
( )( )
P(x = k) = x n − x
N
( )
n
Note: Random selection is called n. total is called N and concern is called k and prob value is called x
Question.1 A committee of size 3 is selected from 4 men and 2 women. Find the probability
distribution by the hyper geometric experiment for the number of men on the
committee.

A. B. C. D.

Solution
Given
Let X denotes the number of men on the committee
Men women selected persons Total
K=4 N–k=2 n=3 N=6
The hyper geometric distribution of x
k N−k
( )( )
P(x = x) = x n − x
N
( )
n
4 2
( )( )
P(x = x) = x 3 − x = x = 1, 2, 3
6
( )
3
4 2
( )( )
x 3−x
X P(x = k) = 6 =
( )
3
4 2
( )( ) 4 ×1 4
1 2
1 = 6 = 20 = 20
( )
3
4 2
( )( ) 6 ×2 12
2 1
2 6 = 20 = 20
( )
3
4 2
( )( ) 4 ×1 4
3 0
3 6 = 20 = 20
( )
3
∑ 1

Question.2 A home owner plants 6 blubs selected at random from a box contain 4 tulip bulbs
and 4 daffodil bulbs. What is the probability that he planted 2 daffodil bulbs and 4
tulip blubs:

27 | P a g e
A. B. C. D.

Solution
Given
Here
4 8−4
( )( )
= 2 6−2
8
( )
6
= 3⁄14

Question.3 Ten vegetable cans, all the same size, have lost their labels. It is known that 5
contain tomatoes and 5 contain corn. If five are selected at random. What is the
probability that all contain tomatoes?

A. B. C. D.

Solution
Given
Here
N = 10, n = 5, K = 5, P(x = 5)
Thus the hyper geometric distribution is
5 5
( )( )
= 5 0
10
( )
5
= 0.003968

Question.4 Determine the probability that the income tax authorities will catch 3 income tax
returns with illegitimate deduction, if it randomly select 6 returns from among 20
income tax returns of which 8 contain illegitimate deductions.

A. B. C. D.

Solution
Given
Here
N = 20 , n = 6, k = 8 , x = 3
Using hyper geometric distribution
P(x = 3)

8 12
( )( )
= 3 3
20
( )
6
12320
=
38760
= 0.3179

Question.5 To avoid detection at customer, a traveler has placed six narcotic tablets in a bottle
containing nine vitamin pills that are similar in appearance. If the customs official

28 | P a g e
selects 3 of the tablets at random for analysis, what is the probability that the
traveler till be arrested for illegal possession of narcotics.

A. B. C. D.

Solution

Given

Here
N = 6+9 = 15
n=3
k=6
Using
6 9
( )( )
= x 3 − x
15
( )
3
The traveler will be arrested for illegal possession of narcotics if customs officer detects at least 1
narcotics tablet i.e.,
X = 1, 2, 3,
Hence
P(x = 1) + P(x = 2) + P(x = 3)
6 9 6 9 6 9
( )( ) ( )( ) ( )( )
= 1 2 + 2 1 + 3 0
455 455 455
216 135 20
= + +
455 455 455
20
=
455
= 0.8

29 | P a g e
NORMAL DISTRIBUTION
Continuous random variables and their associated density functions arise whenever our
experimental data are defined over a continuous sample space like time, weights and heights etc.
There is several continuous distributions but we will discuss only the normal distribution. It is
the distribution. It is the distribution that provides a basis upon which much of the theory of
statistical inference has been developed. A continuous random variable X having bell shaped
curve is called normal random variable. The mathematical equation for the probability
distribution of normal variable is

1 1 𝑥−𝜇 2
f(x) = 𝜎√2𝜋 .e ( ) where - ∝< 𝑥 < +∝
2 𝜎

Where 𝜇 = mean 𝜎 = S.D

𝜋 = 3.1416 e = 2.7183

Properties of normal distribution: -

i) It has bell shaped symmetric curve i.e mean = median= mode = 𝜇


ii) Its range is from -∝ 𝑡𝑜+∝
iii) Area under the normal curve is 1
iv) In a normal distribution
4
a) Mean Deviation = 5 (𝑆. 𝐷)
2
b) Quartile Deviation = 3 (S.D)
c) The two Quartiles are equi-distant from mean i.e.
𝑄1 = 𝜇 - .6745𝜎 (lower quartile)
𝑄3 = 𝜇 + .6745𝜎(upper Quartile)
v) Whatever the values of 𝜇 and 𝜎 are
a) The interval 𝜇 ± 𝜎 will contain 68% area of curve.
b) The interval 𝜇 ± 2𝜎 will contain 95.5% area of curve.
c) The interval 𝜇 ± 3𝜎 will contain 99.7% area of curve.

Standardized Normal Variate:

It is a variable denoted by z with zero mean and 1 standard deviation i.e.


𝑥−𝜇
Z= 𝜎

30 | P a g e
TYPE:1 NORMAL DISTRIBUTION USING Z VALUE

Question.1 (i) Let the r.v. Z have the standard distribution Find P(0 ≤ Z ≤ 1.20)

A. B. C. D.

Solution
Given
P(0 ≤ Z ≤ 1.20)

Use table of Area under Normal curve and find the


value 1.2 in column and zero in row, we get P(0 ≤ Z ≤ 1.20) = 0.3849

(ii) P(‒1.65 ≤ Z ≤ 0)

A. B. C. D.

Solution
Given
P(‒1.65 ≤ Z ≤ 0)
Use table of area under Normal curve and find the value 1.6 in
column and 0.05 in raw we get.
P(‒1.65 ≤ Z ≤ 0) = 0.4505

(iii) P(0.6 ≤ Z ≤ 1.67)

A. B. C. D.
31 | P a g e
Solution
Given
P(0.6 ≤ Z ≤ 1.67)
= P(0 ≤ Z ≤ 1.67) – P(0 ≤ Z ≤ 0.6)

Use Area table


= 0.4525+ 0.2257 = 0.2268

(iv) P(‒1.30 ≤ Z ≤ 2.18).

A. B. C. D.

Solution
Given
P(‒1.30 ≤ Z ≤ 2.18)
= P(‒1.30 ≤ Z ≤ 0) + P(0 ≤ Z ≤ 2.18)

Use area table


= 0.4032 + 0.4854
= 0.8886

(v) P(‒1.96 ≤ Z ≤ -0.84)

A. B. C. D.

Solution
Given
P(‒1.96 ≤ Z ≤ ‒0,84)
= P(‒1.96 ≤ Z ≤ 0) ‒ P (‒0.84 ≤ Z ≤ 0)
Using area table
= 0.4750 ‒ 0.2995
= 0.1755

(vi) P(Z ≥1.96)

A. B. C. D.

Solution
Given
P(Z ≥ 1.96) = 0.5 –P(0 ≤ Z ≤ 1.96)
Using area table
= 0.5 – 0.4750
= 0.0250

(vii) P(Z ≤ ‒2.15)

A. B. C. D.

32 | P a g e
Solution
Given
P(Z ≤ ‒2.15) = 0.5 ‒P(‒2.15 ≤ Z≤ 0)
= 0.5 ‒ 0.4842 = 0.0158

Question.2 A random variable x is normally distributed with µ = 50 and σ2 = 25. Find the
probability

(a) that it will fall between


(i) 0 and 40
(ii) 55 and 100

(b) that it will be


(i) large than 54
(ii) smaller than 58

A. B. C. D.

Solution
(i) Between 0 and 40
Given
µ = 50 , σ2 = 25
σ=5
using Z – score formula
0−50
At x = 0 , we find Z = = −10
5
40 − 50
At x = 40, we find Z = = −2.0
5
Now
P(0 ≤ x ≤ 40) = P(‒10 ≤ Z ≤ -2)
= P(‒10 ≤ Z ≤ 0) – P(‒2 ≤ Z ≤ 0)
= 0.5 ‒0.4772
= 0.0228
(ii) Between 55 and 100
Given
55−50
At x = 55 , we get Z = = 1.0
5
100 − 50
At x = 100, we get Z = 5
= 10
Now
P(55 ≤ x ≤ 100) = P(1.0 ≤ Z ≤ 10.0)
= P(0 ≤ Z ≤ 10) – P(0 ≤ Z ≤ 10)
= 0.5 ‒0.34213
= 0.1587
(b)(i) Large than 54
Given
54−50
At n = 54 , Z = 5
= 0.80

Now
P(x ≥ 54) = P(Z ≥ 0.7)

33 | P a g e
= 0.5 ‒P(0 ≤ Z ≤ 0.8)
= 0.5 ‒ 0.2881
= 0.2119
(ii) smaller than 57
Given
57−50
At x = 57 , Z = 5
= 1.40
Now
P(x ≤ 57) = P(Z ≤ 1.40)
= 0.5 + P(0 ≤ Z ≤ 1.40)
= 0.5 + 0.4192
= 0.9192

Question.3 Note: unit must be same


The length of life for an automatic dishwasher is approximately normally
distributed with a mean of 3.5 years and standard deviation of 1.0 years. If this type
of dishwasher is guaranteed for 12 months, what fraction of the sales will require
replacement?

A. B. C. D.

Solution
Given
At x = 1 = 12 months ,
1−3.5
𝑍= 1.0
= −2.5
Now
P(x ≤ 1.0) = P(Z ≤ ‒2.5)
=0.5 – P (‒2.5 ≤ Z ≤ 0)
= 0.5 – 0.4938
= 0.0062=0.0062x100% =0.62%
Hence 0.62% of sales need replacement before 12 months.

Question.4 The mean height of soldiers is 68.22 inches with a variance of 10.8 (in) 2. Assuming
the distribution of height to be normal, how many soldiers in a regiment of 1000 would you
expect to be over 6 feet tall(72 inches)?

A. B. C. D.

Solution
Given
µ = 68.22, σ2 = 10.8
σ = 3.29
At
x = 72 (6ft)
72 − 68.22 3.78
Z= = = 1.15
√10.8 3.29

34 | P a g e
Now
P(x ≥ 72) = P(Z ≥ 1.15)
= 0.5 –P(0 ≤ Z ≤ 1.15)
= 0.5 – 0.3749
= 0.1251
If there are 1000 solider in the regiment, then number expected to be over 6 feet (or 72 inches) is
1000 x 0.1251 = 125

Question.5 Score on a certain nation-wide college entrance examination follow a normal


distribution with a mean of 500 and a standard deviation of 100. Find the
probability that a student will score (i) over 650 (ii) less than 250 (iii) between 325
and 675.

A. B. C. D.

Solution
(i) Over 650
Given
µ = 500, σ = 100
P(x > 650) = ?
At
650−500
x = 650 Z= = 1.5
100
Now
P(x ≥ 650) = P(Z ≥ 1.5)
= 0.5 –P(0 ≤ Z ≤ 1.5)
= 0.5 – 0.4337
= 0.0663
(ii) Less than 250
Given
250−500
At x = 250 , Z= = −2.5
100
Using area table
P(x ≥ 250) = P(Z ≥ -2.5)
= 0.5 –P(-2.5 ≤ Z ≤ 0)
= 0.5 – 0.4938
= 0.0062
(iii) between 325 and 675
Given
325−500
At x = 325 Z = = −1.75
100
675−500
At x = 675 Z = 100
= 1.75
Using area table
P(325≤x < 675) = P(‒1.75 ≤ Z ≥ 1.75)
= P(‒1.75≤ Z ≥ 0) + P(0 < Z ≥ 1.75)
= 0.4599 + 0.4599
= 0.9198

35 | P a g e
Question.6 Given that the height of college boys is normally distributed with mean 5'-2" and
standard deviation 4', and that the minimum height required for joining the N.C.C

is 5'-4". Find percentage of boys who would be rejected on account of their height.

A. B. C. D.

Solution
Given
µ = 5'-2" = 62 σ = 4' and x = 5'-4" = 64
64.62
at x = 64 , Z =
4
Z = 0.5
Now
P(x < 64) = P(Z < 0.5)
= 0.5 + P(0 < Z < 0.5)
= 0.5 + 0.1915
= 0.6915

Question.7 Grades on a national aptitude best have been found to be normal distributed with a
mean of 250 and a standard deviation of 25. What is the probability that a student
selected at random will score between 230 and 256.

A. B. C. D.

Solution
Given
µ = 250 , σ = 25
At
230−250
x = 230 , Z = 25 = −0.8
256−250
At x = 256 , Z = = 0.24
25
Now
P(230 ≤ x ≤ 256) = P(-0.8 ≤ Z ≤ 0) + P(0 ≤ Z ≤ 0.24)
= 0.2881 + 0.0948
= 0.3829

Question.8 APZ Limited produces a components having a diameter of 3.0cm. A customer has
ordered 100,000 units and has indicated that he would be willing to accept a
variation of up to 0.01cm. The diameter of the components has a normal
distribution with mean of 3.0cm and standard deviation of 0.005cm. Estimate the
number of components that the customer would reject.

A. B. C. D.

Solution
Given
µ = 3 , σ = 0.005
At
2.99−3
(3-0.01)x = 2.99 , Z = 0.005 = −2
At
3.01−3
(3+0.01) x = 3‒01 , Z = 0.005 = 2
Now

36 | P a g e
P(‒2 ≤ x ≤ 2) = P(‒2 ≤ Z ≤ 0) + P(0 ≤ Z ≤ 2)

= 0.4772 + 0.4772
= 0.9544

Now P(rejecting component) = 1 – 0.9544


= 0.0456
Total components rejected = 100,000 x 0.0456
= 4560

Question.9 Section of tunnel is lit by 2000 electric bulbs, which are kept burning day and night.
The manufacturer says that the lives of the bulbs are normally distributed about a
mean of 820 hours with standard deviation of 90 hours. How many electric bulbs
will be expected to fail before 1000 hours/

A. B. C. D.

Solution
Given
µ = 820 , σ = 90
At
x−μ 1000−820
x = 1000 , Z = = =2
σ 90
Now
P(x ≤ 1000) = P(Z ≤ 2)
= 0.5 + 0.4772
= 0.9772
Number of bulbs expected to fail before 1000 hours = 2000 × 0.9772
= 1954
Question.10 Mean height, in the training camp, of the players of Pakistan Basket Ball team is
74.18 inches with a variance of 10.8 inches. If the heights of the players are normally
distributed, how many players in a group of 20 would expect to be over 6.5 feet tall?
Note 6.5*12=78
A. B. C. D.

Solution Given
µ = 74.18 , √σ2 = √10.8 = 3.286
78 − 74.18
At x̅ = 78 , Z=
3.286
= 1.16
Now
P(x > 78) = P(Z > 1.16)
= 0.5 – P(0 ≤ Z ≤ 1.16)
= 0.5 ‒ 3770
= 0.1230
Hence
Number of players having more than 78"
= 20 x 0.-1230
= .46
= 2 (Approximately)

37 | P a g e
Question.11 In an experiment to determine the amount of time required to assemble a toy, the
assembly time was found to be a random variable having approximately a normal
distribution with µ = 27.8 minutes and 0 = 4.0 minutes. What is the probability that
this kind of toy can be assembled in less than 25.0 minutes?

A. B. C. D.

Solution
Given
µ = 74.8 , σ
x−μ 25 − 27.8
At x = 25 , Z= =
σ 4
= ‒0.7
Now
P(x < 25) = P(Z < ‒0.7)
= 0.5 – P(-07 ≤ Z ≤ 0)
= 0.5 ‒ 0.2580
= 0.2420

TYPE:2 FIND PROB OF MEAN

Question.12 A random sample of size 15 is taken from a normally distributed population with
mean 60 and the standard deviation 4. Find the probability that the mean of the
sample is less then 58.

A. B. C. D.

Solution
Given
µ = 60 , σ = 14 , n = 15
x̅ − μ
At x̅ = 58 , Z= σ
⁄ n

58 − 60
=
4⁄
√15
= 1.94
Now
P(x̅ < 58) = P(Z < 1.94)
= 0.50 ‒ 0.4738
= 0.0262

Question.13 A population consist of 500 children has a mean I.Q score of 100 and a standard
deviation of 20 points. If a random sample of 30 children is selected what is the probability that the
mean I.Q of the group exceeds 110?

A. B. C. D.

38 | P a g e
Solution
Given
µ = 100 , σ = 20 , n = 30
x̅ − μ
At x̅ = 110 , Z= σ
⁄ n

110 − 100
=
20⁄
√30
= 2.74

Now
P(x̅ > 110) = P(Z > 2.74)
= 0.50 – P(0 ≤ Z ≤ 2.74)
= 0.50 ‒ 0.49693
= 0.00307

TYPE:3 FINDING VALUE OF X (Reversal Method)

Question.14 The heights of applicants to the police force are normally distributed with mean
170cm and standard deviation 3.8cm. If 30% of applicants are rejected because they
are small. What is the minimum acceptable height for the police force?

A. B. C. D.

Solution
Given
µ = 170 , σ = 3.8
let x denote minimum acceptance right for the police force. Then area between µ and x is
0.50 – 0.30 = 0.20
Using area table inversely
(Z/P = 0.20) = 0.527
As x lies to the left of µ, so Z is negative at this point.
x = µ - σZ
x = 170 – (0.527) (3.8)
x = 168cm

Question.15 In a normal distribution, 31% of the items are under 45 and 8% are over 64. Find
the mean and standard deviation of the distribution.

A. B. C. D.

Solution
Given
Area shaded in the left hand side = 0.31
So un-shaded area = 0.5 – 0.31
= 0.19
Using area table of value 0.19 is 0.4958

39 | P a g e
μ − 45
∴ = 0.4958
σ
µ ‒ 45 = 0.4958 σ (1)
Also
Area shaded on right hand side is 0.08 and un-shaded area is 0.42
So area table, value of 0.42 is 1.4053
64 − μ
∴ = 1.4053
σ

64 ‒ µ = 1.40536 σ (2)
Solving (1) and (2) , we get
µ = 50, σ = 10

TYPE:4 USE OF FORMULA


K-38
Question.16 If µ = 50, σ = 10 and z = 1.2 then the cross ponding x value must be

A. B. C.0+ D.

Solution
Given
µ=50
σ = 10
z =1.2
Then
x = µ + σz
x = 50 + (10) (1.2)
x = 50 + 12
x = 62

Question.17 If µ = 50, z = ‒0.5 and x = 45 the variance must be equal to:

A. B. C.0+ D.

Solution
Given
µ = 50
x = ‒0.5
z = 45
Then
x=µ+σz
45 = 50 + σ (-0.5)
45 = 50 ‒0.50
45 – 50 = -0.50
215 = ‒0.50
5 = 0.5 σ
5
= σ
0.5
10 = σ
Variance = σ 2 = 100
40 | P a g e
Question.18 One on examination the average grade was 74 and the standard deviation was 7. If
10% of the class are given A’s and the distribution of grades is to follow a normal
distribution. Then the lowest possible A and highest possible B if z10 = 1.28 is

A. B. C. D.

Solution
Given
µ = 74
σ=7
z10 = 1.28
Then
=µ±σz
= 74 ±7 (1.28)
= 74 ±8.96
=82.96, 65.04

Question.19 Given a normal distribution with µ = 200 and σ2 = 100 then the two x values
containing the middle 75% of the area if z0.125 = 1.15.

A. B. C. D.

Solution
Given
µ = 200
σ2 = 100  σ 10
z0.125 = 1.15
Then
Lower Value:
=µ-σz
= 200 - (10) (1.15)
= 200 - 11.5
= 188.5
Upper Value:
=µ + σz
= 200 + (10) (1.15)
= 200 + 11.5
= 211.5

41 | P a g e
EXTRA PRACTICE QUESTIONS

42 | P a g e
Question.1 (T-1)

The mean number of years of experience of a certain population of sales persons is 10 years. The
standard deviation is 3 years. What is the probability that a random sample of 81 of these sales
persons yields a means greater than 10 years and 8 months?

Solution
8
µ =10, σ = 3 n=81, 𝑥̅ = 10 + = 10.67
12
𝑥̅ − 𝜇 10.67−10
At x = 10.67, z = 𝜎 = 3 = 2.01
√𝑛 √81
Now
P (x > 10.67) = p (z > 2.01)
= 0.5 – p (0 < z < 2.01)
= 0.5 – 0.4772
= 0.0228
Question.2 (T-2)
A population follows a normal distribution with a mean of 45 and standard deviation of 6
(i) What proportion of this population will have values between 55 and 60?
(ii) What proportion of this population will have values between 60 and 65?
(iii) What proportion of this population will have values between 40 and 50?
(iv) What proportion of this population will have values more than 60?
(v) Find proportion of the population for values which are more than 65 and for values
which are less than 25
Solution:
i) 𝜇 = 45, 𝜎 = 6
55−45
At x = 55, = 6
= 1.67
𝑥− 𝜇 60−45
At x = 60, z = 𝜎 = 6 = 2.5
Also
= p (45≤ 𝑥 ≤ 60) – p (45≤ 𝑥 ≤ 55)
= p (0≤ 𝑧 ≤ 2.5) – p (0≤ 𝑥 ≤ 1.67)
= 0.04938 – 0.4525
= 0.0413
ii) Proportion of values more than 60 = 100* 0.0062 = 62%
𝑥−𝜇 65−45
At x = 65, z = 𝜎
= 6
= 3.33
𝑥−𝜇 25−45
At x = 25, z = 𝜎
= 6
= -3.33
[0.5 − 𝑝 (25 ≤ x ≤ 45)] + [0.5 − 𝑝 (45 ≤ 𝑥 ≤ 65)]
= [0.5 − 𝑝 (−3.33 ≤ 𝑥 ≤ 0)] + [0.5 − 𝑝 (0 ≤ 𝑧 ≤ 3.33)]
= (0.5 – 0.4996) + (0.5 – 0.4996)
= 0.0004 + 0.0004
= 0.0008
Proportion of values more than 65 and less than 25 is
= 0.0008 * 100 = 0.08%

43 | P a g e
Question.3 (T-2)
Of the item produced by a certain company 13% are defective. A sample of 25 items is selected for
inspection
(i) What is the probability that less than four defective are found?
(ii) What is probability that three or more defective are found?

Solution
i) P = 13% = 0.13, q = 0.87, n = 25
Also
𝜇 = n p = 25 * 0.13 = 3.25
𝜎 = √𝑛 𝑝𝑞 = 1.68
𝑖) p (x < 4) = p (x < 3.5) by applying continuity correction
𝑥−𝜇 3.5−3.25
𝐴𝑡 𝑥 = 3.5, 𝑧 = 𝜎
= 1.68
= 0.15
𝑝 (𝑥 < 3.5) = 𝑝 (0 ≤ 𝑧 ≤ 0.15) + 0.5
= 0.596 + 0.5
= 0.5596
ii) P (x>3) = p (x > 2.5)
𝑥− 𝜇 2.5−3.25
At x = 2.5, z = = = −0.45
𝜎 1.68
Now
P (x > 2.5) = p (z > -0.45)
= 0.5 + p (-0.45 < z < 0)
= 0.5 + 0.1736
= 06736

Question.4 (T-2)
A production supervisor finds that employees on the average complete a task in ten minutes. The
times required to complete the task are approximately normally distributed with a standard
deviation of three minutes
(i) The proportion of employees requiring more than five minutes, completing the task
(ii) The proportion of employees requiring less than five minutes, completing the task
Solution:
i) 𝜇 = 10, 𝜎 = 3, x = 5
𝑥− 𝜇 5−10
𝐴𝑡 𝑥 = 5, 𝑧 = 𝜎
= 3
= −1.67
Now
P (x > 5)= p (z > -1.67)
= p (-1.67 < z < 0) + 0.5
= 0.4525 + 0.5
= 0.9525
Also
Proportion of employees for more than five minutes to complete the task = 0.9525 * 100
= 95.25%
𝑥− 𝜇 5−10
(iii) At x 5, z = = 𝜎
= 3
= −1.67
Now
P (x < 5) = p (z < -1.67)

44 | P a g e
= 0.5 – p(-1.67 < z < 0)
= 0.5 – 0.4525
= 0.0475

Proportion of employees for less than five minutes to complete a task = 0.0475 * 100
= 4.75%
Question.5
If a set of observation is normally distributed, then the percentage of observation differs from mean
by more than 1.3 𝝈 is
Solution:
P (|𝑥 − 𝜇| ≥ 1.3𝜎) = 1 – p (|𝑥 − 𝜇| < 1.3𝜎)
= 1 – p (-1.3𝜎 < x - 𝜇 < 1.3𝜎)
𝑥− 𝜇
= 1 – p (-1.3 < 𝜎 < 1.3)
= 1 – p (-1.3 < z < 1.3)
= 1 – p (z < 1.3) + p (z > -1.3)
= 1 – p (0.9032 + 0.0968)
= 0.1936
= 19.36%

Question.6
The IQ of 300 applicants to a certain college are approximately normally distributed with mean of
115 and a standard deviation of 12. If the college required an IQ of at least 95. Then the number of
students those will be rejected on this basis regardless of their other qualification are
Solution:
𝑥− 𝜇
Z= 𝜎
95−115
Z= = −1.66
12
= 0.5 – p (0 <z < -1.66)
= 0.5

Question.7
The mean and variance of normal distribution are 80 and 64. Find third quartile

Solution:

Q3 = µ + 0.6745α

Q3= 80 + 0.6745(8)

Q3 = 85.4

Question.8
If 25% of the observation are less than 24.67 and 25% of the observation are more than 56.33 in a
normal distribution, what is the coefficient of variation.

Solution:

24.67 = µ - 0.6745 𝜎

45 | P a g e
56.33 = µ + 0.675 𝜎

________________

µ = 40.5, 𝜎 = 23.46
𝜎
Co efficient of variation = µ × 100

23.46
C.V = 40.5
× 100

C.V = 57.92

Question.9
In a normal distribution, the quartile deviation is
𝑄3−𝑄1
Solution: Q-D = 2

µ+0.6745𝛼−µ+0.6745𝛼
Q−D=
2

Q-D = 0.6745𝛼

Question.10
The prob of hitting a target is 𝟒⁄𝟓. If a person takes three trials. What is the prob that the is hit at least
once?

Solution: Here
4 1
𝑃 = ,𝑞 = ,𝑛 = 3
5 5

𝑃(𝑥 ≥ 1) = 1 − 𝑝(𝑥 = 0) = 1 − 𝑐03 (4⁄5)0 (1⁄5)3

1
𝑃(𝑥 ≥ 1) = 1 − 125

125−1
𝑃(𝑥 ≥ 1) = 125

124
𝑃(𝑥 ≥ 1) = 125

Question.11
A business has a large number of machines. These machines require minor adjustment on a regular
basis. The need for the adjustments occurs at random at an average rate of 7 per hour. What is the
prob that 4 or fewer adjustments will be required in an hour?

Solution: µ = 7, 𝑃(𝑥 ≤ 4)

𝑃(𝑥 ≤ 4) = 𝑃(𝑥 = 0) + 𝑃(𝑥 = 1) + 𝑃(𝑥 = 2) + 𝑃(𝑥 = 3) + 𝑃(𝑥 = 4)


𝑒 −µ .µ𝑥 𝑒 −µ .µ𝑥 𝑒 −µ .µ𝑥 𝑒 −µ .µ𝑥 𝑒 −µ .µ𝑥
= 𝑥!
+ 𝑥!
+ 𝑥!
+ 𝑥!
+ 𝑥!

46 | P a g e
𝑒 −7 .70 𝑒 −7 .71 𝑒 −7 .72 𝑒 −7 .73 𝑒 −7 .74
𝑃(𝑥 ≤ 4) = 0!
+ 1!
+ 2!
+ 3!
+ 4!

𝑃(𝑥 ≤ 4) = 0.000911 + 0.00638 + 0.02234 + 0.05212 + 0.09122

𝑃(𝑥 ≤ 4) = 0.1729

Question.12
According to municipal records 80% of the families living in certain area, own a car. If 5 families are
randomly selected from this area. Calculate the prob that

i) No family out of 5 will own a car.


ii) 1 family out of 5 will own a car.
iii) 2 families out of 5 will own a car.
iv) 3 families out of 5 will own a car.
v) At least one family out of 5 will own a car.

Solution:

P = 80% = 0.8, q = 0.2, n = 5

i) 𝑃(𝑥 = 0) = (𝑛𝑥 )𝑝 𝑥 𝑞𝑛−𝑥 = 50𝑐 (0.8)6 (0.2)5.0 = 0.00032


ii) 𝑃(𝑥 = 1) = (51 )(0.8)1 (0.2)4 = 0.0064
iii) 𝑃(𝑥 = 2) = (52 )(0.8)2 (0.2)3 = 0.0512
iv) 𝑃(𝑥 = 3) = (53 )(0.8)3 (0.2)2 = 0.2048
v) 𝑃(𝑥 ≥ 1) = 1 − 𝑝(𝑥 < 1) = 1 − 𝑝(𝑥 = 0) = 1 − (50 )(0.8)0 (0.2)5 = 1 − 0.00032
Question.13
The administrator of a telephone company claims that only 2.5% of the telephone owners are
dissatisfied with the company a random sample of 30 telephone owners is obtained. Assuming that
the claim of the company is true obtain the prob that out of 30 owners the member of dissatisfied
owner would be less than 3

Solution:

𝑃 = 2.5% = 0.025, 𝑛 = 3, 𝑝(𝑥 < 3)

µ= 𝑛𝑝 = 0.025 × 30 = 0.75

𝑃(𝑥 < 3) = 𝑝(𝑥 = 0) + 𝑝(𝑥 = 1) + 𝑝(𝑥 = 2)

𝑒 −0.75 . (0.75)0 𝑒 −0.75 . (0.75)1 𝑒 −0.75 . (0.75)2


= 0!
+ 1!
+ 2!

= 0.4723 + 0.3542 + 0.1328

= 0.9594

Question.14
A car manufacturing company has determined that 30% of all oil tanks that were installed on it’s 1988
products are defective.

i) If 5 cars are recalled what 5 the prob that more than 2 will need new oil tanks.
47 | P a g e
Solution:

𝑃 = 30% 0 , 𝑞 = 0.7, 𝑛 = 5, 𝑃(𝑥 > 2)

𝑃(𝑥 > 2) = 𝑃(𝑥 = 3) + 𝑃(𝑥 = 4) + 𝑃(𝑥 = 5)

= (53 )(0.3)3 (0.7)2 + (54 )(0.3)4 (0.7)1 + (55 )(0.3)5 (0.7)0

= 0.1323 + 0.6283 + 0.00243

= 0.1631

Question.15
In past, two players A and B have played 20 games of which 10 are won by A, 6 by B and 4 ended in a
tie. Now in a tournament they are to play 4 games. What is the prob that 2 games will end in a tie?

Solution:
4
𝑃= , 𝑞 = 0.8
20

P = 0.2

𝑃(𝑥 = 2) = 42𝑐 (0.2)2 (0.8)2


96
=
625

Question.16 A sales man has estimated that the probability of making a sales when calling
on a customer is 0.5. if this sales man calls on 5 customers on a given day what is the
probability of making exactly 3 sales

Solution:

Given 𝑝 = 0.5, 𝑞 = 0.5, 𝑛 = 5

Now 𝑝(𝑥 = 3) = (5 3 )(0.5)3(0.5)2

𝑝(𝑥 = 3) = 0.3125

Question.17 A binomial random variable has a mean of 200 and a standard deviation of
10. Find the value of n and p.

Solution:

Given 𝜇 = 2 𝑜𝑟 , 𝜎 = 10

We know 𝜇 = 𝑛𝑝 = 200 ______________(𝑖)

𝜎 = √𝑛𝑝𝑞 = 10

48 | P a g e
𝜎 2 = 100 ____________(𝑖𝑖)

Dividing (i) and (ii)


𝑛𝑝𝑞 100
= 200
𝑛𝑝

𝑞 = 1⁄2

And 𝑃 = 1⁄2

Also put 𝑝 = 0.5 , 𝑖𝑛 ________(𝑖)

𝑛(0.5) = 200
200
𝑛= = 400
0.5

Question.18 A pharmaceutical company produces a cough syrup in bottles and finds that
only 6% of these bottles are defective.

i. If 6 of these bottles are selected what is the probability that one would be
defective.
ii. If sample of 6 bottles are selected in a similar way. 100 times how many times
the selection would consist of exactly one defective.
Solution:

Given P= 6% = 0.06 , q = 0.94, n= 6

Using

𝑝(𝑥 = 1) = (6 1 )(0.06)1 (0.94)5

𝑝(𝑥 = 1) = 0.2642

ii) Numbers of times one defective occurs out of 100 times.

= 100 × 0.2642

= 26.42%

Question.19 In a serving of M.B.A students 75% said that expert to be promoted within a
month after receiving their degree. If this proportion hold for the population. Find for a
sample of size 5. The probability that the number expecting a promotion within a month
after graduation is

49 | P a g e
i. At least 3
ii. No more than 3.
Solution:

Given

𝑝 = 75% , 𝑞 = 0.25 , 𝑛 = 5

i. 𝑝(𝑥 ≥ 3) = 𝑝(𝑥 = 3) + 𝑝(𝑥 = 4) + 𝑝(𝑥 = 5)

𝑝(𝑥 ≥ 3) = (5 3 )(0.75)3 (0.25)2 + (5 4 )(0.75)4 (0.25)1 + (5 5 )(0.75)5 (0.25)0

= 0.2636 + 0.3955 + 0.2373

= 0.8964

ii. 𝑝(𝑥 < 3) = 1 − 𝑝(𝑥 > 3)


= 1 − (𝑝(𝑥 = 4) + 𝑝(𝑥 = 5))
= 1 − (0.3955 + 0.2373) = 0.3672

Question.20 A certain factory manufactures item , out of the factory total number of
items that are produced by the factory 3% are found to be defective A sample of ten items
is selected for inspection

i) Use binomial distribution to find the probability that two or more defective
items are found.
ii) Use of Poisson distribution to find the probability that 2 or more defective
items are found
Solution:

Given 𝑃 = 0.03 , 𝑞 = 0.97 , 𝑛 = 10

𝑢𝑠𝑖𝑛𝑔 𝑃(𝑥 > 2) = 1 − 𝑝(𝑥 < 2)

= 1 − (𝑝(𝑥 = 0) + 𝑝(𝑥 = 1))

=1 − ((10 0 )(0.03)0 (0.97)10 + (10 1 )(0.03)1 (0.97)9 )

= 1 − (0.7374) + 0.228)

= 1 − 0.9654

𝑝(𝑥 > 2) = 0.346

ii) Poisson distribution


50 | P a g e
Given 𝑃 = 0.03 , 𝑛 = 10

𝜇 = 𝑛𝑝 = 10 × 0.03 = 0.3

Using 𝑃(𝑥 > 2) = 1 − 𝑝(𝑥 < 2)

= 1 − (𝑃(𝑥 = 0) + 𝑃(𝑥 = 1))

𝑒 −0.3 . (0.3)0 𝑒 −0.3 (0.3)1


=1−( + )
0! 1!

= 1 − 𝑒 −3 (1 + 0.3)

= 1 − 𝑒 −3 (1.3)

= 1 − (1.3)(0.74082)

= 1 − 0.963066

= 0.36934

Question.21 The administration of a telephone company claims that only 2.5% of the
telephone owners are dissatisfied with the company. In order to verify the claim of the
company a random sample 30 telephone owners are obtained. Assuming that the claim of
the company is true. Obtain the probability that out of 30 owners, the number of
dissatisfied owners should be less than 3.

Solution:

Given

𝑃 = 2.5% = 0.025, 𝑛 = 30, 𝜇 = 𝑛𝑝 = 0.75

Now

𝑃(𝑥 > 3) = 𝑃(𝑥 = 0) + 𝑃(𝑥 = 1) + 𝑃(𝑥 = 2)

0−0.75 (0.75)0 𝑒 −0.75 (0.75)1 𝑒 −0.75 (0.75)2


= + +
0! 1! 2!

𝑒 −0.75 (0.5625)
= 𝑒 −0.75 + 0.75𝑒 −0.75 + 2!

= 𝑒 −0.75 (1 + 0.75 + 0.28125)

= 𝑒 −0.75 (2.03125)

𝑃(𝑥 < 3) = 0.95

51 | P a g e
MCQ’S:

Question:1) The probability distribution showing the probability of 𝑥 occurrences of an event over a
specified time, distance or space is known as:

A: Binomial probability distribution B: Poisson probability distribution


C: Chi-square distribution D: None of these
Question:2) The number of days in a given in which a 130-point change occurs in the Karachi stock
exchange index.

A: binomial distribution B: Poisson distribution


C: Normal distribution D: None of these
Question:3) The statement “The number of fatal accidents that occur per month in a large manufacturing
plant” may describe a random variable that may possess:

A: Poisson distribution B: binomial distribution


C: normal distribution D: none of these
Question :4) The number of blind babies born per month in the city of Karachi. This statement may
describes a random variable that may posses

A: binomial distribution B: Poisson Distribution


C: normal distribution D: none of these
Question :5) A complete list of formula that gives the probabilities associated with each value of a
random variables 𝑥 is called:

A: Frequency distribution B: Probability distribution


C: Random variable D: none of these
Question :6) A variable that assumes the numerical values associated with events of an experiment is
called:

A: Parameter B: Statistic
C: Random variable D: None of these
Question :7) A function whose value is a real number determined by each element in the sample space is
called:

A: Parameter B: Statistic
C: Random variable D: None of these
Question :8) A table or formula listing all possible values that a random variable can take on, along with
the associated probabilities is called:

A: Probability distribution B: Frequency distribution


C: Expected value D: None of these
Question :9) If 𝑥 is a random variable belonging to a continuous probability distribution, then P(𝑥 = 𝑎)
is:

A: Equal to zero B: Less than 1


C: More than zero D: None of these
Question :10) Let 𝑥 be a random variable with probability distribution

P(X=𝑥), then ∑ 𝑥𝑝(𝑥) is known as:

A: Sum of probabilities B: Mean or expected value

52 | P a g e
C: Mean of binomial distribution D: None of these
Question :11) In term of mathematical expectation the formula E(𝑥 2)-(E(𝑥))2 represents:

A: Standard deviation of distribution B: Variance of distribution


C: Difference of squared deviation D: None of these
Question :12) If an experiment is repeated for n trials, each trial results in an outcome classified as
success or failure, with constant probability of success, and trials are independent, the experiment is
known as:

A: Binomial experiment B: Poisson experiment


C: Hyper geometric experiment D: None of these
Question :13) (i) A random sample of size n is selected from a population of N items.

(ii) K on N items may be classified as success and N-K classified as failures


The above properties are related to a

A: Binomial experiment B: Poisson experiment


C: Hyper geometric experiment D: None of these
Question :14) If a random sample of size n is drawn without replacement the events are said to be:

A: Independent B: Not independent


C: Not mutually exclusive D: None of these
Question :15) If a random sample of size is drawn without replacement the events are said to be:

A: Independent B: Not independent


C: Not mutually exclusive D: None of these
Question :16) Whenever we measure time intervals, weights, heights, volumes and so forth, our
underlying population is described by a:

A: Discrete distribution B: continuous distribution


C: Frequency D: None of these
Question :17) A distribution whose graph is a symmetric bell shaped curve extending indefinitely in both
directions, with equal measures of central tendency is known as:

A: Non-skewed distribution B: Normal distribution


C: Binomial distribution D: None of these
Question :18) The normal probability distribution is a continuous distribution with parameter(s):

A: Mean B: Mean and variance


C: Mean and mean deviation D: None of these
Question :19) A standard normal distribution is one that has mean and variance equal to:

A: Zero and one B: One and one


C: One and zero D: None of these
Question :20) which of the following is a characteristic of the probability distribution for any random
variable?

A: A probability is provided for every possible B: The sum of all probabilities is 1


value.
C: No given probability occurs more than once D: Both “A” & “B”
Question :21) A discrete probability distribution may be represented by

A: A table B: A graph
53 | P a g e
C: A mathematical equation D: All of these
Question :22) A continuous probability distribution may be represented by

A: A table B: B: A graph
C: A mathematical equation D: All of these
Question :23) If 𝑥 and 𝑦 are random variable then E(𝑥 − 𝑦) is equal to

A: E(𝑥)+E(𝑦) B: E(𝒙)-E(𝒚)
C: 𝑥 E(y) D: E(𝑥)-𝑦
Question :24) If 𝑥 is a random variable and “A” & “B” are constants, then Var (a+b𝑥) is equal to

A: Var(a)+Var(b𝑥) B: Var(a)+bVar(𝒙)
C: b2var(𝑥) D: Var(𝑥)
Question :25) For a given binomial distribution with n fixed, if p<0.5, then

A: The binomial distribution will be skewed to B: The binomial distribution will be skewed to left
right
C: The binomial distribution will be symmetric D: None of these
Question :26) The standard deviation of binomial distribution depends on

A: Probability of success B: Probability of failure


C: Number of trials D: Both “A” & “C”
Question :26) Which of the following can never be describe by a binomial distribution.

A: The number of defective items produced by an B: The number of water used by a single
assembly line household
C: The number of students is a class who can D: All of these can always be described by a
answer this question binomial distribution
Question :27) The mean of binomial distribution depends on

A: Probability of success B: Probability of failure


C: Number of trials D: Both “A” & “C”
Question :28) The parameter of binomial distribution are

A: 𝑥 & 𝑛 B: 𝑥 & 𝑝
C: 𝒏 & 𝒑 D: 𝑥. 𝑛& 𝑝
Question :29) Which of the following is true for binomial distribution

A: mean>variance B: mean<variance
C: mean = variance D: mean = standard deviation
Question :30) The mean median and mode of the binomial distribution will be equal when

A: P=0.5 B: P<0.5
C: P>0.5 D: None of these
Question :31) The parameter of hyper geometric are

A: x, N, k B: N, n, k
C: x, n, k D: x, N, n, k
Question :32) A binomial random variable is a(an)

A: Independent variable B: Continuous random variable


C: Discrete random variable D: None of these

54 | P a g e
Question :33) In which distribution the probability of success remains constants from trial to trial

A: Hyper geometric B: binomial distribution


C: Sampling distribution D: continuous distribution
Question :34) In which distribution the probability of success changes from trial to trial

A: Hyper geometric B: binomial distribution


C: Sampling distribution D: continuous distribution
Question :35) Both binomial and hyper geometric distributions are

A: Continuous distributions B: Discrete distributions


C: None of these D: Bivariate distributions
Question :36) The binomial distribution is positively skewed if:

A: P = q B: P < q
C: P > q D: None of these
Question :37) The standard deviation of the binomial distribution is:

A: Square of np B: Square root of np


C: Square of npq D: Square root np (1 - p)
Question :38) For a binomial distribution which of following is correct:

A: Mean = 7 variance = 16 B: Mean = 5 variance = 9


C: Mean = 4 variance = 12 D: Mean = 6 variance = 2
Question :39) Poisson distribution is:

A: Always symmetric B: Always positively skewed


C: Always negatively skewed D: Symmetric only if µ = 2
Question :40) Which of the following is the characteristics of the probability distribution of any random
variable:

A: Probability provided for every possible value B: The sum of probabilities is 1


C: No given probability occur than one D: Both (A) & (B)
Question :41) A binomial random variable is:

A: A continuous random variable B: A discrete random variable


C: Dependent variable D: None of these
Question :42) In which distribution the successive trials are with replacement:

A: Binomial distribution B: Hyper geometric distribution


C: Continuous distribution D: None of these
Question :43) In which distribution the trials are dependent:

A: Binomial distribution B: Hyper geometric distribution


C: Continuous distribution D: None of these
Question :44) If 𝑥 = 𝑁 (𝑎, 𝑏) then variance of normal distribution is:

A: b2 B: b
C: a2 D: a

55 | P a g e
Question :45) Normal distribution is:

A: Uni parametric B: Bi parametric


C: Tri parametric D: None of these
Question :46) In a standard normal distribution the values of µ and ơ are:

A: 0 & 1 B: 0.5 & 2


C: 1& 0 D: 1& 1.5
Question :47) In a normal distribution, the quartile deviation is:

A: 0.7979ơ B: 0.6745ơ
C: 0.3468ơ D: 0.75ơ
Question :48) In a normal distribution mean deviation is equal to:

A: 0.6745ơ B: 0.7979ơ
C: 0.3468ơ D: -0.7979ơ
Question :49) The limits of the normal distribution are:

A: 0 to ∞ B: -∞ to+∞
C: 0 to 1 D: -∞ to 0
Question :50) The normal distribution has parameter:

A: µ B: ơ2
C: µ, ơ2 D: π, e
Question :51) The total area under the normal curve is:

A: 0.5 B: 1
C: 0 D: -0.5
Question :52) The symmetry of the normal distribution about its mean indicates that:

A: The distribution is bell shaped B: The area under the curve on both the sides of
mean is equal
C: The two trials do not meet at X-axis D: All of these
Question :53) The two quartiles (upper and lower) are equidistant from its:

A: Variance B: Mean
C: Standard deviation D: None of these
Question :54) The parameters of the binomial distribution are:

A: µ, ơ2 B: p, q
C: 0, 1 D: n, p
Question :55) For a binomial distribution, mean and mode:

A: Are always equal B: Are never equal


C: Are always equal when q = 0.5 D: Do not always exist
Question :56) Poisson distribution is:

A: Always symmetric B: Always positively skewed


C: Always negatively skewed D: Symmetric only when m= 2
Question :57) In Poisson distribution:

A: Mean= variance B: Mean > variance


C: Mean < variance D: Standard deviation

56 | P a g e
Question :58) Poisson distribution:

A: Always unimodal B: Always binomial


C: Multi-modal D: May be unimodal or binomial distribution
Question :59) The probability of occurrence of an event is ‘p’ . If the experiment is repeated n times.
Then the probability of the event occurring every time is

A: p/n B: pn
C: np D: n(1 - p)
Question :60) In Poisson distribution np is

A: infinite B: finite
C: 2 D: None of these
Question :61) Poisson distribution is known as …………… distribution.

A: Uni-parameter B: Bi-parameter
C: Tri-parameter D: None of these
Question :62) Binomial distribution can be approximated to Poisson distribution when

A: P ̶ ̶ > 0 B: P ̶ ̶ > ∞
C: np is finite D: All of these
Question :63) In a binomial distribution the probability of getting zero or more number of successes is
equal to

A: 1 B: 0
C: The probability of getting zero success D: q minus the probability of getting success in all
the trials
Question :64) The binomial distribution be approximated to Poisson distribution, if the probability of
success (p) is …………………….. and the number of trials (n) is ………………….

A: Large, small B: Small, large


C: Infinite D: Finite, infinite

57 | P a g e
PROBABILITY:15-16
1)When a coin tossed for 8 times then ____________ distribution can be used

a) Poisson b) Binomial
c) Normal d) Chi-square
2) Given P(A) = 1/3, P(B) = ¼. Suppose that the two events “A” & “B” are independent events, then the
probability of at least one of them is?

a) 6/12 b) 1/2.
c) A and b d) None of the above
3)The number of ways in which four books of AFC can be arranged on a shelf is:

a) 8 ways b) 12 ways
c) 16 ways d) 24 ways
4)The life of electric tube light follows normal distribution with mean life of 3,000 hours and standard
deviation of 200 hours. What is the probability that a tube light has life more than 3,400 hours?

a) 0.4772 b) 0.0040
c) 0.4960 d) 0.0228
5)In a binomial distribution if n = 4 and p = 1/2, then median is:

a) 4 b) 2
c) 1 d) None of these
6)An event that cannot be split into further events is known as:

a) Complex b) Mixed
c) Simple d) composite
7)A coin is tossed 3 times, all possible outcomes are

a) 6 b) 8
c) 3 d) 9
8) A coin is tossed 3 times all possible outcomes are
a) HHH, HHT, HTH, THH, TTH, THT, b) HHH, HHT,HTH, THH, TTH, THT,
HHT, TTT HTH, TTT
c) HHH, d) HHH, HTT, HTH, THH, TTH, THT,
HHT,HTH,THH,TTH,THT,HTT,TTT HTT, TTT
9) If two dice are rolled, what is the probability that either the sum of the two will be seven or at
least one of the dice will show the number 5.
a) 18/36 b) 6/36
c) 15/36 d) 12/36
10) Two dices are rolled what is the probability that at least one 6 appears.
a) 11/36 b) 15/36
c) 12/36 d) 10/36
11) A coin is tossed 3 times find the probability that at least 1 head appears
a) 8/7 b) 6/7
c) 7/8 d) 7/6
12) If 75% peoples use cellular phones and 15% use PTCL and 10% use both then find the
probability that a person selected at random use both
a) 0.10 b) 0.75
c) 0.15 d) 1.00
58 | P a g e
13) If 50% peoples use cellular phones and 40% use PTCL and 20% use both then find the
probability that a person selected at random use neither
a) 10% b) 70%
c) 30% d) 90%
14) You are given 20 true false questions and required to find the probability of at-most 4 correct.
a) 0.0059 b) 0.2645
c) 0.2145 d) 0.1325
15) You are given 20 true false questions and required to find the probability of at least 4 correct.
a) 0.9987 b)
c) d)
16) A dice is rolled 5 times. What is the probability that exactly 2 “four” appear?
a) 0.1125 b) 0.1608
c) 0.1305 d) 0.2315
(Hint: use binomial distribution)

17) Find the Euler number (Napier number)


a) 1.2323 b) 2.718281
c) 0.526 d) 10
18) Find the value of pie (π)
a) 3.141592 b) 3.15432
c) 3.76545 d) 3.148767
19) Find 0!
a) 0 b) 1
c) 10 d) 100
20) There are 2 red balls, 2 green and 3 blue balls in a bag. It two balls are drawn at random. What is
the probability that none is blue?
a) 0.2857 b) 1
c) 2.2875 d) 3.3857
21) There are 14 flowers in a bouquet 3 are Roses, 5 Tulip and 6 Jasmine, if 5 flowers are selected
find the probability that 2 roses, 2 Tulips and Jasmine are selected.
a) 3/5 b) 90/1001
c) 2/5 d) None of the above
22) Following numbers are given 1, 2, 5, 6, 8, 9 you are required to make a 3 digit number which is
divisible by 5. How many arrangements are possible?
a) 20 ways b) 40 ways
c) 50 ways d) 10 ways
23) In how many ways word BINOMIAL can be written.
a) 40320 b) 20160
c) 1440 d) None of the above
24) In how many ways word BINOMIAL can be written if vowel words are together
a) 2880 b) 1440
c) 20160 d) None of these
25) 80% people use UPS; 50% use generator; 20% use both; find the probability that people don’t use
both.
a) 10% b) 80%
c) 30% d) None of the above
26) Which of the following is not true of the normal distribution?
a) The measures of central tendency b) The curve approaches the x-axis
(mean, mode, and median) are equal in gradually on either side of the mean
value
c) The curve is bell-shaped d) The curve is asymmetrical

59 | P a g e
27) The area under the normal curve between µ- 3.5α and µ+ 1.0α is
a) 0.3416 b) 0.5
c) 0.8411 d) 0.8477
28) Binomial distribution does not carry the characteristics
a) Trails are dependent b) Probability will remain same in success
or failure
c) Fixed number of trails d) Probability will remain same in all
trails
29) There are 12 runners in marathon and all runners have equal chance of winning. What is the
probability that a person may win a bet on the race if he has to correctly select the top 3 runners
and the order they finish in?
a) 1/1320 b) 1/1728
c) 3/1728 d) 1/12
30) In how many ways word “CORRECT” can be arranged:
a) 5040 b) 2520
c) 1260 d) None of these
31) In a bakery store there are 3 cakes of pineapple; 4 cakes of chocolate and 2 cakes of strawberry
and 1 is without cream. 2 cakes are purchased by a customer. Find the probability that both are
chocolate cakes.
a) 2/15 b) 2/10
c) 3/15 d) 4/10
32) A problem in statistics is given to three students A, B, C whose chances of solving are 1/2 , 3/4
,1/4 respectively. What is the probability that problem will be solved.
a) 3/32 b) 29/32
c) 3/7 d) ½
33) There are total 25 eggs in a bag out of which 3 are defective, if two are randomly chosen find the
probability that both are defective
a) 1/100 b) 10/100
c) 10% d) None of these
34) There are total 25 keyboards out of which 3 are defective, A sample of 5 is selected, what is the
probability that exactly two are defective?
a) 0.5632 b) 0.7386
c) 0.1256 d) 0.08695
35) There are 7 balls in a box containing 2 red, 2 green and 3 blue; two balls are drawn, what is the
probability that none is blue?
a) 2/7 b) 3/7
c) 1/7 d) 7/7
36) If dice is rolled five times. What is the probability that exactly 2 “Even” appears?
a) 0.3125 b) 0
c) 2.3125 d) 3.1253
37) You are given the following data
Light smoker Smoker
Men 50 200 250
Women 60 10 70
110 210 320
(i)Find the probability that a person selected is man or light smoker
a) 31/32 b) 27/32
c) 1/32 d) None of the above
ii-Find the probability that a person selected is woman or light smoker
a) 3/8 b) 5/8
c) 1/7 d) 2/7
60 | P a g e
iii-Find the probability that a person selected is smoker man
a) 5/8 b) 1/8
c) 3/8 d) 2/8
iv-If a man is selected find the probability that he is a smoker
a) 4/5 b) 2/5
c) 1/5 d) 3/5
38) A TV channel conducted a poll regarding construction of dams in Pakistan 80% people was in
support of construction. 10% were against undecided. A sample of 10 persons is taken.
What is the probability that at least 8 will support the construction?
a) 0.3413 b) 0.6778
c) 0.6345 d) 0.7512
39) What of the following is the property of bell-shaped distribution?
a) Asymptotic b) Unimodal
c) Symmetrical d) All of these
40) In a group of 12 international referees, there are three from Africa, four from Asia and five from
Europe. To officiate at a tournament, three referees are chosen at random from the group. Find
the probability that:
i) A referee is chosen from each continent.
ii) Two referees are chosen from Asia.
iii) All the three referees are chosen from the same continent.
a) 3/33 b) 3/77
c) 3/44 d) 4/55
41) Among 18 members of a cricket club, there are 2 wicket keepers and 5 bowlers. In how many
ways can a team of 11 members be chosen so as to include only one wicket keeper and at least
three bowlers?
a) 12144 b) 21244
c) 42122 d) 31233
42) Out of 12 eggs in a refrigerator, 2 are rotten. From these 12 eggs, 4 egg are selected at random to
make a cake. What is the probabilities that:
i) Exactly one is rotten.
ii) At least one is rotten.
a) 240/495 b) 495/240
c) 440/220 d) 120/240
(ii)

a) 495/285 b) 245/695
c) 285/495 d) 330/500
43) The board of directors of a company consists of 8 men and 4 women. A 4 member’s committee is
to be chosen at random from the board:
i) What is the probability that all 4 members of the committee will be women?
a) 1.4141 b) 0.1414
c) 3.1414 d) 4.1414

ii) What is the probability that all 4 members of the committee will be men?
a) 1.222 b) 2.111
c) 0.002 d) None of these
44) Among a transport department’s 16 trucks, 5 emit excessive amount of smoke. If eight of the
trucks are randomly selected inspection, what is the probability that this sample will include at
least 4 of the trucks, which emit excessive amount of smoke?
a) 4.141 b) 1.444

61 | P a g e
c) 0.141 d) 2.121

45) A coin tossed 3 times find the probability of 1 head


a) 3/8 b) 8/8
c) 8/3 d) 2/8
46) 3 dices are rolled find the probability that at least one 3 number dice appears
a) 4.4213 b) 2.4213
c) 5.4213 d) 0.4213
47) following data is given
Goal 0 1 2 3 4 5 >5
P(X=x) 0.05 0.2 0.15 0.15 0.3 0.05 0.1
Find the probability that there would be total 5 goals in two matches
a) 0.17 b) 0.83
c) 0.84 d) 0.12
48) How many five digit positive integers that are divisible by ‘3’ can be formed using the digits 0, 1,
2, 3, 4 and 5, without any of the digits getting repeating
a) 15 b) 96
c) 216 d) 120
e) 625 f) None of these
49) In a group of 6 boys and 4 girls, four children are to be selected. In how many different ways can
they be selected such that at least one boy should be there?
a) 159 b) 194
c) 205 d) 209
50) A normal distribution has mean 5.03 and standard deviation 0.03 then find the probability of values
between 5 to 5.06
a) 2.6826 b) 6.2826
c) 0.6826 d) None of these
51) How many 3-digit numbers can be formed from the digits 2, 3, 5, 6, 7 and 9, which are divisible
by 5 and none of the digits is repeated?
a) 5 b) 10
c) 15 d) 20
52) A box contains 2 white balls, 3 black balls and 4 red balls. In how many ways can 3 balls be
drawn from the box, if at least one black ball is to be included in the draw?
a) 32 b) 48
c) 64 d) 96
53) In how many different ways can the letters of the word 'DETAIL' be arranged in such a way that
the vowels occupy only the odd positions?
a) 32 b) 48
c) 36 d) 60

54) In how many ways the word CORRECT be arranged


A) 360 B) 480
C) 720 D) 1260
55) In how many different ways can the letters of the word 'LEADING’ be arranged in such
a way that the vowels always come together?
a) 360 b) 480
c) 720 d) 5040
56) Out of 7 consonants and 4 vowels, how many words of 3 consonants and 2 vowels can be
formed?
a) 210 b) 1050
c) 25200 d) 21400
57) How many 3-digit numbers can be formed from the digits 2, 3, 5, 6, 7 and 9, which are
62 | P a g e
divisible by 5 and none of the digits is repeated?
a) 5 b) 10
c) 15 d) 20
58) There are 50 currency notes of Rs 1000 and 50 currency notes of Rs 500 in a safe. If 4 notes
are selected randomly with replacement, then find the probability that total amount obtained
is Rs 3,000
a) 0.3750 b) 0.155
c) 0.75 d) 0.5
59) There are 65 currency notes of Rs 1000 and 35 currency notes of Rs 500 in a Safe. If 4 notes are
selected randomly find the probability that total amount obtained is Rs 3000
a) 0.3156 b) 0.1526
c) 0.3052 d) 0.2056
60) There are 65 currency notes of Rs 1000 and 35 currency notes of Rs 500 in a Safe. If 4 notes are
selected randomly find the probability that total amount obtained is not less than Rs 3000
a) 0.8782 b) 0.3156
c) 0.3052 d) 0.1215
61) There are 65 currency notes of Rs 1000 and 35 currency notes of Rs 500 in a Safe. If 4 notes are
selected randomly find the probability that total amount obtained is less than Rs 3000
a) 0.1218 b) 0.8782
c) 0.3156 d) 0.1587
62) There are 65 currency notes of Rs 1000 and 35 currency notes of Rs 500 in a Safe. If 4 notes are
selected randomly find the probability that total amount obtained is at least Rs 4000
a) 0.1727 b) 0.1218
c) 0.3152 d) 0.8782
63) Two dices are rolled find the probability that at least 1 “six” appear
a) 1/6 b) 2/6
c) 12/36 d) 11/36
64) Two dices are rolled find the probability that exact 1 “six” appear
a) 1/6 b) 2/6
c) 12/36 d) 10/36
65) Two dices are rolled find the probability that “six” appear on first dice only
a) 1/6 b) 2/6
c) 11/36 d) 10/36
66) Mean of a certain normal distribution is 5.0 and SD is 0.03 you are required to find the probability of
5.0 ± 0.03
a) 68.268% b) 34.13%
c) 95.15% d) 100%
67) Mean of a certain normal distribution is 5.0 and SD is 0.03 you are required to find the probability of
5.0 + 0.03
a) 68.268% b) 34.13%
c) 95.15% d) 100%
68) Mean of a certain normal distribution is 5.0 and SD is 0.03 you are required to find the probability of
5.0 - 0.03
a) 68.268% b) 34.13%
c) 95.15% d) 100%
69) Mean of a certain normal distribution is 5.0 and SD is 0.03 you are required to find the probability of
5.0 - 0.06
a) 68.268% b) 47.725%
c) 95.15% d) 100%
70) Mean of a certain normal distribution is 5.0 and SD is 0.03 you are required to find the probability of
5.0 ± 0.06
a) 68.268% b) 34.13%
63 | P a g e
c) 95.15% d) 100%

71) In a normal distribution mean=5.05 and SD =0.02 find the probability of a value less than 5.0
a) 0.4938 b) 0.0062
c) 1.4938 d) 1.0062
72) In a normal distribution mean = 5.05 and SD = 0.02 find the area between 5.00 and 5.06
a) 0.6853 b) 6.6853
c) 5.8553 d) None of these
73) Mean of a certain normal distribution is 5.0 and SD is 0.03 you are required to find the probability of
5.0 ± 0.09
a) 68.268% b) 47.725%
c) 95.45 d) 99.73%
74) A telephone operator receives on average 2 calls in 3 minutes. Find the probability of receiving
more than or equal to 4 calls in 9 minutes
a) 0.8488 b) 0.8425
c) 0.1512 d) None of these
75) During peak hours a center receives 4 calls per 30 minutes. What is the probability of
getting 3 or more calls in an hour?
a) 0.9862 b) 0.8425
c) 0.1512 d) None of these
76) Ahmad received 4 calls in five minutes on average. What is the probability that more than
3 calls will be received in 10 minutes?
a) 0.9862 b) 0.8425
c) 0.9576 d) None of these
77) 5 digits are given 2, 3, 4, 7, 8. Find the probability that 5-digit number will be made such
that 7 is first number and 8 is last number
a) 1/20 b) 4/20
c) 1/10 d) 1/100
78) In a test there are 20 questions. All questions are selected are MCQs with 4 choices. All
20 questions are selected find the probability that 4 are correct.
a) 0.1897 b) 4
c) 2.9158 d) 1.8889

79) There are 12 CNG kits out of which 4 are defective. if 4 Kits are selected at random then
find the probability that at least 3 are defective
a) 1/15 b) 5/15
c) 1/100 d) 10/15
80) While coming and from a department store a consume pass through one out of 12 cash
counters C1 to C2 (All haring same probability) then his bill is verified by one of 3
officers (with same probabilities) v1, v2, v3, then he embarks 1 of two elevators E1, E2
and is twice likely to embark on E2 as E1
Find the probability a consumer will pass through C6 or C12 verified by V1 and embark on
E2
a) 1/27 b) 1/54
c) 5/6 d) 2/3
81) A 5-digit pin code is required which should start with 7 at first digit and 8 as last digit.
You can choose any number from 0 to 9. Repetition of number is allowed.
Find the probability of such code
a) 5/1000 b) 1/100
c) 1/1000 d) 5/100
82) There are 6 digits 1, 3, 4, 5, 7, 9. A three-digit number is to be made from these numbers
which should be divisible by 2 (no repetition of numbers). Find all possible ways.
a) 100 b) 20
64 | P a g e
c) 1000 d) 200
83) You have 6 values 1,3,5,7,9 and 11. How many 3-digit codes can be made (Without
replacement).
a) 20 b) 12
c) 120 d) 360
84) An (NAC) institute select 30 doctors and give them a medicine to test and give
feedback, each doctor selects 50 patients and apply that medicine to them. You are
required to determine the sample size for NAC?
a) 30 b) 50
c) 1500 d) 2500
85) A dice is rolled 3 times, what is the probability that 3 will appeal at least 2 times.
a) 2/27 b) 2/57
c) 1/37 d) 1/57
86) A committee is to be made of 4 members from a total of 13 people of which 5 from
Punjab, 2 from Baluchistan, 4 from Sindh and 2 from KPK, what is the probability that
committee contain none from Punjab?
a) 6.8% b) 0.7%
c) 9.8% d) None of these
87) Two dices are rolled find the probability that at least one 6 appear
a) 11/36 b) 36/11
c) 12/46 d) 1/36
88) What is the probability of getting at least one six in a single throw of three unbiased
dice?
a) 94/216 b) 90/216
c) 91/216 d) 84/216
89) When two dice are thrown simultaneously, what is the probability that the sum of the
two numbers that turn up is less than 11?
a) 11/23 b) 14/23
c) 11/12 d) 12/14
90) An income tax officer has received 10 files numbered from 1 to 10. On the direction of
the director, the income tax officer selected one file for inspection. The probability that
the selected file number is a multiple of 5 or multiple of 3 is:

a) 50% b) 1/2
c) Both a and b d) 80%
91) What is the probability that a two-digit number selected at random will be a multiple of
'3' and not a multiple of '5'?
a) 4/15 b) 8/15
c) 2/15 d) 6/12
92) After studying Mr. X's family history, a doctor determines that the probability of any
child born to this couple having a gene for disease X is 1 out of 4. If Mr. X has three
children, what is the probability that exactly two of the children have the gene for
disease X?
a) 12% b) 14%
c) 13% d) 15%
93) On any given day, the probability that the entire family of Mr. Y eats dinner together is
2/5. Find the probability that, during any 7-day period, family front has each dinner
together at least six times
a) 0.1255 b) 0.0188
c) 0.1386 d) 0.2526

94) Suppose that you have a bag filled with 50 marbles, 15 of which are green. What is the
probability of choosing exactly 3 green marbles if a total of 10 marbles are selected?

65 | P a g e
a) 0.2045 b) 0.2615
c) 0.2979 d) 0.1568
95) Three missiles are fired at a target. If the probability of hitting the target are 4/5, 5/10 &
6/10 respectively & if all the missiles are fired independently. The probability that at
least two missiles will hit to the target is:
a) 40% b) 7/10
c) 0.9 d) 0.1
96) Among 18 members of a cricket club, there are 2 wicket keepers and 5 bowlers. In how
many ways can a team of 11 members be chosen so as to include only 1 wicket keeper
and at least 3 bowlers?
a) 2,000 b) 12,458
c) 12,222 d) 12,144
97) On a single toss of a pair of fair dice, what is the probability that a sum of 7 appears
and both dice show a number less than 4?
a) 2/36 b) 1/36
c) 0 d) 1
98) A specialist shop attracts five customers per day. There is an independent probability of
1/3 that any one customer will make a purchase. The probability that the shop will
make at most one sale in a day is:
a) 16/243 b) 85/243
c) 48/243 d) 112/243
99) The probability of a high jumper clearing 1.8m on any jump is 0.6. What is the
probability of his clearing 1.8m in precisely three out of seven jumps?
a) 0.006 b) 0.273
c) 0.194 d) 0.290
100) A gambler plays a game of chance 150 times. On each play, he has a
probability of 0.45 of winning. What is the approximate probability of his winning
between 62 and 70 times?
a) 0.41 b) 0.525
c) 0.475 d) 0.59
101) Among a company’s 16 trucks, 5 emit excessive amount of smoke. If eight of
the trucks are randomly selected for inspection, what is the probability that this sample
will include at least 4 of the trucks, which emit excessive amount of smoke?

a) 0.2458 b) 0.3625
c) 0.2170 d) 0.1410

102) A firm of chartered accountants has two vacancies for trainee students and is trying to
recruit CAF passed students. In the past, 40% of students who were offered training contract have
not reported to join. If 2 students are offered training contract, what is the probability that at least
one will join?
a) 0.12 b) 0.84
c) 0.15 d) 0.56
103) The probability that a car will have a flat Tyre while driving through Kalma Chowk
Underpass is 0.00006. What is the probability that at least 2 out of 10,000 cars passing through
underpass will have a flat Tyres?
a) 0.1219 b) 0.6625
c) 0.2156 d) 0.2315
104) In one day a department manufactures four products, each of which has an independent chance of
20% being faulty. The probability that at least three products are not faulty is?

66 | P a g e
a) 0.1024 b) 0.5120
c) 0.8192 d) 0.2456

105) In which distribution the probability of success remains constant from trial to trial?
a) Hyper geometric distribution b) Binomial distribution
c) Sampling distribution d) Frequency distribution
106) During a T20 cricket match Mr. X scored 47 runs in six overs with the help of five fours, four
sixes and three singles. If a TV channel were to show two of his scoring shots during its News Update
and the shots were to be selected on random basis, find the probability that on both the shots Mr. X had
scored different runs?
a) 47/66 b) 23/66
c) 15/23 d) 74/666
107) A binomial distribution may be approximated by a Poisson distribution if
a) n is small and p is large b) n is large and p is large
c) n is small and p is small d) none of these
108) An unbiased dice with faces marked 1, 2, 3, 4, 5 and 6 is rolled four times. Out of four face
values obtained, the probability that the minimum value is not less than 2 and maximum value is not
greater than 5 is:
a) 1/81 b) 80/81
c) 16/81 d) 65/81
109) Out of the numbers 1 to 120, a number is selected at random. What is the probability that it is
divisible by 8 or 10?
a) 10% b) 20%
c) 15% d) 25%
110) In a group of students 50% of the students were taking statistics subject, 30% economics and
10% both the subjects. What is the probability that a student was taking neither statistics nor economics?
a) 30% b) 90%
c) 70% d) None of these
111) When a coin tossed for 8 times then _______ distribution can be used

a) Poisson b) Hypergeometric
c) Normal d) Binomial
112) The mean sales of all the different branches of METRO store is Rs. 100,000 with a standard
deviation of Rs. 30,000. The probability of shops; the sales of which is in between Rs. 110,000 and
Rs. 120,000 is:

a) 0.2486 b) 0.1193

c) 0.1293 d) 0.3779

113) If 20% of the bulbs manufactured by a company are defective. What is the average number of
defective bulbs in a batch of 400 bulbs?

a) 320 b) 80

c) 64 d) 8

114) 10 applicants consisting of 4 MScs, 3 MBAs, 3 CAs, candidates applied for 4 vacancies. If the
vacancies are filled at random. What is the probability that only 2 CA candidates will be selected?

a) 3/210 b) 200/210

c) 3/10 d) 105/210

67 | P a g e
115) An unprepared students go for test. There are 10 questions having 4 options in test. Find the
probability that at least 1 is correct.

a) 0.999 b) 0.99

c) 1 d) 0.9437

116) There are total 57 eggs in a basket out of which 36 are defective. If 4 eggs are selected from the
box find the probability that only 1 is defective

a) 1/11 b) 2/15

c) 4/33 d) None of these

117) A company Manufacture 25 Bulbs out of which 3 are defective on average, if a random sample
of 5 Bulbs is selected from it. What is the probability that it contains exactly 2 Defective Bulbs?

a) 1/11 b) 2/15

c) 2/23 d) None of these

118) A mobile service provider offers the following options to its customers

Call metering 1 Min 2 Min 3 Min 4 Min


Call package 300 Rs for 300 400 Rs 400 Min 500 Rs for 500
Min Min
GPS package 300 Rs for 300 400 Rs for 450 450 Rs for 500
MBs MBs MBs
In how many ways can a customer select a service package?

a) 36 b) 144
c) 16 d) None of these
119)

Call metering 1 Min 2 Min 3 Min 4 Min

Call package 300 Rs for 300 400 Rs for 400 500 Rs for 500

Min Min Min

GPS package 300 Rs for 300 400 Rs for 450 450 Rs for 500

MBs MBs MBs

SMS package 500 per month 750 per month 1000 per month

If a customer wants to choose one of these packages, then find the number of ways available to him

a) 36 b) 108
c) 120 d) None of these
120) If the two quartiles of a normal distribution are 14.6 and 25.4 respectively, what is the standard
deviation of the distribution?

a) 6 b) 8

68 | P a g e
c) 9 d) 10

121) From an industrial area 70 companies were selected at random and 45 of them were planning for
expansion next year. Find 95% confidence limits for the proportion of companies planning for
expansion.

a) 0.35, 0.57 b) 0.35, 0.75

c) 8.49, -7.21 d) 0.45, 0.57

122) Three cards are drawn at random one by one with replacement from a pack of 52 cards. What is
the probability that all are aces?

a) 1/5746 b) 1/2197

c) ¾ d) One of these

123) A committee of four (4) people is to be appointed from three(3) officers of the production
department, four(4) offices of the purchase department, two(2) officers of the sales department & one(1)
chartered accountant. The probability that in the committee, there must be one from each category is:

a) 24/210 b) 4/35

c) 0.1143 d) All of the above

124) A shipment of 20 similar laptop computers to a retail outlet contains 3 that are defective. If a
school makes a random purchase of two of these computers. The probability of at least one defective,
keeping in view that the probability changes from one trial to another trial is:

a) 27/95 b) 54/190

c) 0.2842 d) All of the above

125) Suppose that you have a bag filled with 50 marbles, 15 of which are green. What is the
probability of choosing exactly 3 green marbles if a total of 10 marbles are selected?

a) 0.2045 b) 0.2979

c) 0.2615 d) 0.1568

127) The arithmetic mean of the upper and lower limits of the confidence interval for population
mean is equal to:

a) Sample mean b) Population mean

c) Population standard deviation d) Sample standard deviation

128)When two coins & one dice are rolled together, all possible outcomes are

a) 6 b) 2

c) 36 d) 24

69 | P a g e
129) On any given day, the probability that the entire family of Mr.Y eats dinner together is 2/5. Find the
probability that, during any 7-day period, family front has each dinner together at least six times.

a) 0.1255 b) 0.1386

c) 0.0188 d) 0.2624

130) The maximum temperature on first June in a certain locality has been recorded and observed as
normally distributed over years. About 15% of the time, it has exceeded 30oC and about 5% of the time

it has been less than 20oC. The mean & variance which are the two parameters of the normal
distribution respectively are:

a) 26.12 and 18.17 b) 26.12 & 13.84

c) 24.27 & 13.84 d) 18.17 & 27.16

131) For two coins, the probability of no head is:

a) 0 b) ¼

c) 0.25 d) B and c but not a

132) A gambler plays a game of chance 150 times. On each play, he has a probability of 0.45 of
winning. What is the approximate probability of his winning between 62 and 70 times?

a) b)

c) d)

133) 10 applicants consisting of 4 ACMAs, 3 MBAs, 3 CAs, candidates applied for 4 vacancies. If the
vacancies are filled at random. What is the probability that only 2 CA candidates will be selected?

a) 3/210 b) 6/210

c) 3/10 d) None of these

134) Automobiles are randomly distributed with an average spacing of 1000 feet along a highway.
The probability that at least two cars are present in 1000 feet interval selected at random is:

a) 0.4642 b) 0.2642

c) 0.3642 d) 0.1642

135) An orderly arrangement of things is called:

a) Combination b) Permutation
c) Probability d) Sample space
136) Given P(A)=1/3, P(B)=1/4. Suppose that the two events “A” & “B” are independent events, then
the probability of either “A: or “B” is equal to:

a) 6/12 b) 1/2

c) A and B d) None of these

70 | P a g e
137) A Secretary makes 2 errors per page on the average. What is the probability that on the next
page she makes no error?

a) 0 b) 0.2704

c) 0.1352 d) 0.0676

138) The probability of hitting a target is 4/5. If a person takes three trials. What is the probability that
the target is hit at least once?

a) 48/125 b) 1/125

c) 124/125 d) 120/125

139) The life of electric Tube Lights follows normal distribution with mean life of 3,000 hours and
standard deviation of 200 hours. What is the probability that a Tube Light has life more than 3,400
hours?

a) 0.4772 b) 0.4960

c) 0.0040 d) 0.0228

140) In which distribution the probability of success remains constant from trial to trial

a) Hyper geometric distribution b) Binomial distribution

c) Binomial distribution d) Frequency distribution

71 | P a g e
141) The probability of the consonant pattern in the word "STATISTICS" is:

a) 7/10 b) 0.3

c) 0 d) a and b but not c

142) The average life of a certain type of motor is 10 years, with a standard deviation of 2 years. If the
manufacturer is willing to replace only 3% of the motors that fail, how long a guarantee should he
offer? Assume that the lives of the motors follow a normal distribution.

a) 5.5 years b) 6.24 years


c) 4.52 years d) 7.62 years
143) Mean = 0 and standard deviation = 1 in ____________ distribution.

a) Normal b) Binomial
c) Standard normal d) Poisson
148) A person has 3 suites and 2 ties. In how many ways can he wear a suit and a tie?

a) 9 b) 8

c) 6 d) 3

144) A bag contains 6 red and 8 green balls. If two balls are drawn without replacement, then what is
the probability that one is red and other is green.
a) 15/91 b) 91
c) 48/91 D 28/91
145) A card is drawn from a pack of 52 cards. What is the probability that it is Queen or red?
a) 26/52 b) 30/52
c) 28/52 d) 2/52
146) The scores made by employees are normally distributed with a mean of 600 and standard
deviation of 100. If top 10% are considered for promotion. What score must an employee make in order
to get promotion?

a) 765 b) 728

c) 796 d) 472

147) Indian cricket team losses twice as often as it wins. What is the probability that in 5 next
matches, India wins 3 matches?

a) 80/243 b) 10/320

c) 11/16 d) 40/243

148) If a binomial distribution is negatively skewed:

a) P=1/2 b) P<1/2

c) P>1/2 d) P=q

149) In a normal distribution mean is 50 and 8% of observation are more than 64.1. What is the
variance of distribution?
a) 10 b) 20

c) 100 d) 400

150) Twenty sheets of aluminum alloy were examined for surface flaws. The frequency of the
number of sheets with a given number of flaws per sheet was as follows:

Number of flaws Frequency

0 4

1 3

2 5

3 2

4 4

5 1

6 1

What is the probability of finding a sheet chosen at random which contains 3 or more surface flaws?

a) 0.20356 b) 0.30125

c) 0.40396 d) 0.51263

151) In Normal distribution which of the following is correct?

a) Area under the Curve is 1 as 100% b) Flat the curve higher standard deviation

c) The Curve is obtained by Mean( ) and d) all are correct

Standard deviation (

152) A group of items chosen from a larger number without regard for their order is called
a) Permutation b) Combination

c) Addition law d) Multiplication law

153) There are 10 balls in a bag each ball carrying a different number from 1 to 10. Three balls are
selected at random from the bag and not replaced before the next draw is made. How many possible
combinations are there?

a) 720 b) 20

c) 120 d) 540

154) Five students from AFC class of 10 are to be chosen to attend a competition. One must be the
head boy. How many possible teams are there?
a) 126 b) 120

c) 142 d) 24

155) An item is made in three stages. At the first stage, it is formed on one of four machines, A, B, C,
or D with equal probability. At the second stage it is trimmed on one of three machines, E, F, or G,
with equal probability. Finally, it is polished on one of two polishers, H and I, and is twice as likely
to be polished on the former as this machine works twice as quickly as the other. What is the
probability that an item is either formed on A or trimmed on F?

a) 1/2 b) 1/3

c) 1/4 d) 1/5

156) A large batch of items comprises some manufactured by process X and some by process Y. There are
twice as many items from X as from Y in a batch. Items from process X contain 9% defectives and those from
Y contain 12% defectives. If an item is taken at random from the batch and found to be defective. What is the
probability that it came from Y?

a) 0.1 b) 0.4

c) 0.04 d) 0.14

157) There are seven balls in a box out of which 2 are red, 2 green and 3 blue; two balls are drawn,
what is the probability that none is blue?

a) 2/7 b) 1/9

c) 3/8 d) 1/3
𝑒 −5 (5)2
158) In a Poisson distribution; P(X=2) = , the variance is:
2!

a) 2 b) -5
c) 5 d) None of these
Solution :2

P (A) = 1⁄3 , P (B) = 1⁄4 , and A & B are independent events then

1 3 1 2 1 1 3+2+1 6 1
P (at least one of them) = *
3 4
+4∗3-3∗4= 12
= 12 = 2

Solution : 3

No. of ways = 4! = 4.3.2.1 = 24

Solution : 4

µ = 3000, α = 200 , X = 3400


𝑋−𝑀 3400−3000
Z= 𝑂
= 200
=2

P (Z > 3400) = 0.5 – P (O < Z < 2)

= 0.0228

Solution : 5
Given
n = 4 , P = 1⁄2 , q = 1⁄2

np = Median = np = 4 * 1⁄2 = 2

Solution : 7

2𝑛 = 23 = 8

Solution : 9

Notes
Event A = [(1,6) , (6,1) , (2,5) , (5,2) , (3,4) , (4,3)]
Event B = [(2,5) , (5,2) , (1,5) , (3,5) , (4,5) , (5,5) , (6,5) , (5,1) , (5,3) , (5,4) , (5,6)]
A ∩ B = [(2.5) , (5,2)]
P (7 or at least 5) = P(A) + P(B) – P(A∩B)
6 11 2
= 36 + 36 - 36

17−2
=
36
15
= 36

Solution : 10

Sample space=36

Event =11

Probability =11/36

Solution : 11

P = 1⁄2 , q = 1⁄2 , n = 3 , P(X > 1)

Now

P (x > 1) = 1 – P(x=0)

3
= 1 - 𝑐 (0.5)0 (0.5)3
𝑜
1
=1-
8

= 7⁄8

Solution : 12

percentage of peoples use cellular phones = 75%

percentage of peoples use PTCL = 15%

percentage of peoples use both = 10%

then

probability =
here , number of outcome = 10%

total outcome = 75%+ 15% + 10% = 100%

therefore ,

probability( person selected at random use both) = =


Solution : 13

P (use neither) = 1 – P(0.5+0.4-0.2)


= 0.3 * 100%

= 30%

Solution : 14

P = 1⁄2 , q = 1⁄2 , n = 20

20 20 20 20 20
P(x < 4) = 𝑐 (0.5) (0.5) + 𝑐 (0.5) (0.5) + 𝑐 (0.5) (0.5) + 𝑐 (0.5) +(0.5) + 𝑐 (0.5)4 (0.5)16
0 1 19 2 18 3 17

𝑜 1 2 3 4

= 0.0059

Solution : 15

P (X > 4) = 1 – P(x<4)
20 20 20 20
= 1 – [ 𝑐 (0.5)0 (0.5)20 + 𝑐 (0.5)1 (0.5)19 + 𝑐 (0.5)2 (0.5)18 + 𝑐 (0.5)3 (0.5)17 ]
𝑜 1 2 3
= 1 – 0.0012884
= 0.9987

Solution : 16

n = 5 , P = 1⁄6 , q = 5⁄6 , P(x=2)

5 2 3
P(x=2) = 𝑐 (1⁄6) (5⁄6)
2

= 0.1608

Solution : 20
4 3
𝑐∗𝑐
𝐸𝑣𝑒𝑛𝑡 2 0
𝑠𝑎𝑚𝑝𝑙𝑒 𝑠𝑝𝑎𝑐𝑒
= 7 = 0.2857
𝑐
2

Solution : 21

3 5 6
𝑐∗ 𝑐∗ 𝑐
2 2 1
P(X = x) = 14
𝑐
3
90
= = 0.08991
1001

Solution : 22

4 * 5 * 1 = 20 ways

Solution : 23
8!
No. of ways = = 20160
2!

Solution : 24

No. of ways = 5! = 120


4!
No. of ways of vowels = = 12
2!

Total No. of ways = 120 * 20 = 1440

Solution : 25

= 1 – (0.8 + 0.5 – 0.2)

= -0.1 (prob. can not be negative) which is impossible

Solution : 29

Total out come = 12𝑃3 = 1320

Probability = 1⁄1320

Solution : 30
7!
No. of ways = = 1260
21 ,2!

Solution : 31
4 6
𝑐∗ 𝑐
P (both chocolate cakes) = 2 0
10 = 2⁄15
𝑐
2

Solution : 32

P(A solve and B solve and C solve)

= 1 – P(𝐴̅ ∩ 𝐵̅ ∩ 𝐶̅ )
1 1 3
=1–( * * )
2 4 4

3
=1-
32

29
= 32

Solution : 33
3 22
𝑐∗ 𝑐
2 0 1
P (both defective) = 25 = 100
𝑐
2

Solution : 34
3 22
𝑐∗ 𝑐
2 3
P(exactly two are defective) = 25
𝑐
5

= 0.08695

Solution : 35
3 4
𝑐∗𝑐
P (None is above) = 6 2
7 = 2⁄7
𝑐
2

Solution : 36

n = 5 , P = 3⁄6 , q = 3⁄6 , P(X=2)

5 2 3
P(X=2) = 𝑐 (3⁄6) (3⁄6)
2

= 0.3125

Solution : 37

(i)
250 110 50 31
P (man or light smoker) = 320 + 320 - 320 = 32

(ii)
70 110 60 3
P (women or light smoker) = + - =
320 320 320 8
(iii)
200 5
P (smoker man) = =
320 8

(iv)

P (smoker) = 4⁄5

Solution: 38

Given

P = 0.80 , q = 0.20 , n = 10

P(x > 8) = P(x = 8) + P(x = 9) + P(x=10)

P(x > 8) = 0.6778

Solution: 35

P(A) + P(B) = P (A∪B)

P(A∪B) = 0.4 + 0.3 = 0.7

Solution : 40

12
Sample space = 𝑐 = 220
3

(i) Probability of having one referees from each continent


3𝑐1 ∗ 4
𝑐1 ∗ 5𝑐
1
=
12𝑐3
60 3
= 220 = 11
(ii) Probability of having referees from Asia

4𝑐2 ∗ 8
𝑐1
=
12𝑐3

48
= 220

12
=
55

(iii)

All three referees from same continent


3𝑐3 4𝑐3 5𝑐3
= + +
12𝑐3 12𝑐3 12𝑐3

1 4 10
= 220 + 220 + 220

15
= 220

3
= 44

Solution : 41

Keepers Bowlers Others Total

2 5 11 18

2 5 11 2 5 11 2 5 11
=𝑐*𝑐* 𝑐 + 𝑐*𝑐* 𝑐 + 𝑐*𝑐* 𝑐
1 3 7 1 4 6 1 5 5

= 6600 + 4620 + 924 = 12144

Solution : 42

Good Rotten Total Selection

10 2 12 4

2𝑐1 ∗ 10𝑐3
(i) P(1 rotten and 3 good eggs) =
12𝑐4
240
=
495

2𝑐1 ∗ 10𝑐3 2𝑐2 ∗ 10𝑐2


(ii) P(at least on rotten) = +
12𝑐4 12𝑐4

240 45
= 495 + 495

285
= 495

Solution : 43

Men Women Total Selection

8 4 12 4
8𝑐4 ∗ 4𝑐0 70
(i) P(A) = = = 0.1414
12𝑐4 495

4𝑐4 ∗ 8𝑐0 1
(ii) P(B) = = = 0.002
12𝑐4 495

Solution: 44

5𝑐4 ∗ 11𝑐4 + 5𝑐5 ∗ 11𝑐3


P(at least 4 defective) =
16𝑐5

1815
= 12870

= 0.141

Solution: 45

Sample Space = 23 = 8

P(A) = 3⁄8

Solution: 46

Given

n = 3 , P = 1⁄6 , q = 5⁄6

P(x > 1) = 1 – P(x < 1)

= 1 – P(X = 0)

3 0 3
= 1 - 𝑐 (1⁄6) (5⁄6)
0

= 1 – 0.5787

= 0.4213

Solution : 47

Solution : 48

Case – 1
(i) If we don’t use zero than
No. of ways = 5! = 120

Case – 2

(i) If we do not use 3 then


1 , 2 , 4 , 5 ---- 0
4 * 4! = 96

Hence total No. of ways

= 120 + 96 = 216

Solution : 49

6 4 6 4 6 4 6 4
P (at least one boy) = 𝑐 * 𝑐 + 𝑐 * 𝑐 + 𝑐 * 𝑐 + 𝑐 * 𝑐
1 3 2 2 3 1 4 0

= 209

Solution : 50

µ = 5.05 , O = 0.05 , X = 5.00 , X = 5.06


𝑋− µ 5.0−5.05
Z= = = -2.5 , Area = 0.4938
𝑂 0.02

𝑋− µ 5.06−5.05
Z= 𝑂
= 0.02
= 0.5 , Area = 0.1915

Required Area = 0.4938 + 0.1915

= 0.6853

Solution : 51

4 * 5 * 1 = 20

Solution : 52

Required number of ways = 3𝑐1 * 6𝑐2 + 3𝑐2 * 6𝑐1 + 3𝑐3

Required number of ways = 64

Solution : 53

= 3! . 3! = 6*6 = 36
Solution : 54
7!
2!.2!
= 1260

Solution : 55

= 5!.3!

= 120 * 6 = 720

Solution : 56

7𝑐3 + 4𝑐2 + 5!

= 210 * 120

= 25 200

Solution : 57

4 * 5 * 1 = 20

Solution : 58

As the selection is with replacement, so event becomes independent, hence we use binomial
distribution

P = 0.5 , q = 0.5 , n = 4

Let X denote the event that 1 notes of 500 and 2 notes of Rs 1000 is

P(X = 2) = 4𝑐 2 (0.5)2 (0.5)2

= 0.3750

Solution : 59
65𝑐 2 ∗ 35𝑐2
P (sum not less than 3000) = 100𝑐 4

= 0.3156

Solution : 60
65𝑐 1 ∗ 35𝑐 3 + 65𝑐 0 ∗ 35𝑐4
P (sum not less than 3000) = 1 - 100𝑐 4

1241
= 1 - 10185
= 0.8782

Solution : 61
65𝑐1 ∗ 35𝑐3 + 65𝑐0 ∗ 35𝑐4
P (sum not less than 3000) =
100𝑐4

1241
= 10185

= 0.1218

Solution : 62
65𝑐4 ∗ 35𝑐0
P (sum at least 4000) = 100𝑐4

= 0.1727

Solution : 63

n(S) = 36

Event = 11

P(A) = 11⁄36

Solution : 64

n(S) = 36

Event = 10

P(A) = 10⁄36

Solution : 65

n(S) = 36

Event = 6

P(A) = 6⁄36 = 1⁄6

Solution : 66

As we know

µ + 1α= 68.268%

5.0 + 0.03 = 68.268%


Solution : 67/68

µ + α = 68.268%

µ + α = 34.13%

µ - α = 34.13%

Solution : 69

µ + 2 α = 95.45%

µ + 2(0.03) = 95.45%

5.0 – 0.06 = 47.725%

Solution : 70

µ + 2 α = 95.45%

5.0 + 0.06 = 95.45%

Solution : 73

µ + 3 α = 99.73%

5.0 + 0.09 = 99.73%

Solution : 74

2 calls in three minutes

M = 6 calls in 9 minutes

P(X > 4) = 1 – P (X<4)

= 1 – [P(x=O) + P(X=1) + P(X=2) + P(x=3)]

= 1 – 0.1512

P(X>4) = 0.8488

Solution: 75

Given

µ = 8, P(X>3)

P(X>3) = 1 – P(X<3)
= 1 – [P(X=0) + P(X=1) + P(X=2)]

𝑒 −8 .80 𝑒 −8 .81 𝑒 −8 .82


=1–[ 0!
+ 1!
+ 2!
]

P(X>3) = 0.9862

Solution : 76

µ = 8 , P(X>3)

P(X>3) = 1 – P(X<3)

= 1 – [P(X=0) + P(X=1) + P(X=2)]

= 0.9576

Solution : 77

Event = 1 * 1 * 2 * 3 * 1 = 36

Sample space = 5! = 120


6 1
P(A) = 120 = 20

Solution : 78
3
P = 1⁄4 , q = 4 , n= 20

20
P(x=4) = ( ) (0.25)4 (0.75)16
4

P(x=4) = 0.1897

Solution : 79

Defective Non defective Total Selection

4 8 12 4

At least three are defective


4 𝑐3 ∗ 8 𝑐1 4 𝑐4 ∗ 8 𝑐0
= 12𝑐4
+ 12𝑐4

4∗8 1∗1
= 495 + 495

33
=
495
1
= 15

Solution : 80

P (𝐶6 𝑜𝑟 𝐶12 ) * (𝑉1) * (𝐸2 )

1 1 1 2
= (12 + 12) * (3) * (3)

1
= 27

Solution : 81

Event = No. of ways = 1 * 10 * 10 * 10 * 1 = 1000 ways

Sample space = All possible ways = 10 * 10 * 10 *10 * 10 = 100,000

1000 1
P(A) = 100,000 = 100

Solution : 82

4 * 5 * 1 =20

Solution : 83

No. of ways = 4 * 5 * 6 = 120 ways

Solution : 84

Sample size = No. of doctors * No. of patients

= 30 * 50

= 1500

Solution : 85

P = 1⁄6 , q = 5⁄6 , n = 3 , P(x>2)

P(x>2) = P(X = 2) + P(x = 3)

2 1 1 0
= 3𝑐2 (1⁄6) (5⁄6) + 3𝑐3 (1⁄6) (5⁄6)

5 5
= 72 + 216
2
= 27

Solution : 86

5𝑐0 ∗ 8𝑐4 14
= 143 = 0.0979 = 9.8%
13𝑐4

Solution : 87

Sample space = 36

Event = 11

P(A) = 11⁄36

Solution : 88

Sample space = 216

Event =91

P(A) = 91⁄216

Solution : 89

Sample space = 36

Event = 33

33
P(A) = 36

11
P(A) =
12

Solution : 90

2 3 5 1
P(Multiple of 5 or 3) = 10 + 10 = 10 = 2 = 50%

Solution : 91

Total number of two digit numbers = 90

Total number divisible by 3 = 30

Number which are also divisible by 5 (15,30,45,60,75,90) = 6


Total two -digit number which are divisible by 3 but not 5 = 30 – 6 = 24

24 4
P(A) = 90 = 15

Solution: 92

3 1 3
P(X=2) = (2) (4)2 (4)1

= 0.140625

9
= 64

= 14%

Solution : 93

n = 7 , P = 2⁄5 , q = 3⁄5

7 2 3 7 2 3
P(x > 6) = ( ) ( 5)6 (5)1 + ( ) (5)7 (5)0
6 7

= 0.0188

Solution : 95

Hit Not Hit

4⁄ 1⁄
5 5

5⁄ 5⁄
10 10

6⁄ 4⁄
10 10
4 5 4 5 6 1 4 6 5 4 5 6
At least two missile = 5 * 10 * 10 + 10
* 10 * 5 + 5
* 10 * 10 + 5
* 10 * 10

80 30 120 120
= 500 + 500 + 500 + 500

350
= 500

= 7⁄10
Solution : 97

P(A) = 0 [less than four cannot form sample]

Solution : 98

n = 5, P = 1⁄3 , q = 2⁄3

P(X < 1) = P(x = 0) + P(x = 1)

5 5
= ( ) (1⁄3)0 (2⁄3)5 + ( ) (1⁄3)1 (2⁄3)4
0 1
32 80
= 243 + 243

112
= 243

Solution : 100

N = 150 , P = 0.45 , q = 0.55 , X = 61.5 , X = 70.5

Solution : 101

Binominal
11
P = 5⁄16 , q = , n = 8 , P(X>4)
16

P(X > 4) = [P (x=4) + P (x=5) + P (x=6) + P (x=3) + P (x=8)]

8 8 8 8 8
5 4 11 4 5 5 11 3 5 6 11 2 5 7 11 5 5 8 11 0
P(X > 4) = [ 𝑐 ( ) ( ) + 𝑐 ( ) ( ) + 𝑐 ( ) ( ) + 𝑐 ( ) ( ) + 𝑐 ( ) ( ) ]
16 16 16 16 16 16 16 16 16 16
4 5 6 7 7

P(X > 4) = [0.149138 + 0.054232 + 0.012325 + 0.001601 + 0.000091]

P(X > 4) = 0.217387

Solution : 102

P = 40% = 2⁄5 , q = 3⁄5 , n = 2

P(X > 1) = 1 – P(X<1)

P(X > 1) = 1 – P(X=0)


2
P(X > 1) = 1 - 𝑐 (2⁄3)0 (3⁄5)2
0
4
P(X > 1) = 1 - 25

21
P(X > 1) = 25

P(X > 1) = 0.84

Solution : 103

P = 0.00006 , n = 10,000 , P(X>1)

M = np = 0.00006 * 10,000 = 0.6

P(X>2) = 1 – P(X>1)

= 1 – P(X = 0) + P(X = 1)

𝑒 −0.6 − (0.6)0 𝑒 −0.6 − (0.6)1


=1- 0!
+ 1!

= 1 – [0.5488 + 0.3292]

= 1 – 0.8780

P(X>2) = 0.1219

Solution : 104

q = 20%, P = 80%, n =4

q = 0.2 , P = 0.8, , P(x>3)

P(x>3) = P(x = 3) + P(x = 4)

4 4
= 𝑐 (0.8)3 (0.2)1 + 𝑐 (0.8)4 (0.2)0
3 3

= 0.8192

Solution : 108

Sample Space = 1296


Event = 256

256
P(A) =
1296

P(A) = 16⁄81

Solution : 109

Sample Space = [1, _ _ _ _ _ _ _ _ _120]

120
Event = 8
= 15

120
Event = 10
= 10

120
Event = 40
=3

15 12 3
P(8 or 10) = 120 + 120 + 120

= 1⁄5 = 0.2 = 20%

Solution : 110

P (neither statistic economics) = 1 – (0.5 + 0.3 – 0.1)

= 0.3

= 30%

Solution :112

µ = 100,000 , α = 30,000 , x = 1,10,000 , X = 1,20,000

𝑋−µ 1,10,000−100,000
Z= 𝛼
= 30,000
= 0.33

𝑋−µ 120,000−100,000
Z= 𝛼
= 30,000
= 0.66

P(0.33< Z <0.66) = P(0< Z <0.66) - P(0< Z <0.33)

= 0.1193

Solution : 113

P = 20% , n = 400
20
µ = np = 400 * 100 = 80

Solution : 114

3 7
𝑐 ∗ 𝑐
2 2 3
10 = 10
𝑐
4

Solution : 115

1 3
P = 4 , q = 4 , n = 10 , P(x>1)

P(x>1) = 1 – P(X<1)

= 1 – P(X = 0)

10 0 10
= 1 – 𝑐 (1⁄4) (3⁄4)
0

= 0.9437

Solution : 116

Defective N.Defective Total Select

36 21 57 4

36 21
𝑐 ∗ 𝑐
1 3
= 57
𝑐
4

4
= 33

Solution : 117

Defective N.Defective Total Select

3 22 25 5

3 22
𝑐 ∗ 𝑐
2 3
= 25
𝑐
5
2
= 23

Solution : 118

No. of ways = Call metering options * Call package options * GPs package options

= 4 * 3 * 3 = 36

Solution : 119

No. of ways = Call metering options * call package options * GPs package options

= 4 * 3 * 3 * 3 = 108

Solution : 120

𝑄1 = 14.6 , 𝑄3 = 25.4

𝑄1 = M – 0.6745r

𝑄3 = M + 0.6745r

14.6 = - 0.6745r

25.4 = + 0.6745r

- -_______

-10.8 = -1.349r

r=8

Solution :121
𝑥
P=
𝑛

45
P= = 0.64
70

q = 0.35

𝑍𝑥⁄2 = 0.95 = 1.96

𝑝𝑞
𝑃𝑜 = P + 𝑍𝑥⁄ √
2 𝑛
1.96 45⁄ ) (25⁄ )
45
= 70 + √( 70 70
70

45 1.96 1125
= 70 + √
70

= 0.64 + 7.85=8.49,-7.21

Solution : 122

1
P(A) = 4⁄52 . 4⁄52 . 4⁄52 =
2197

Solution : 123

10
Sample Space = 𝑐 = 210
4

3 4 2 1
Event = 𝑐 * 𝑐 * 𝑐 * 𝑐 = 24
1 1 1 1
24 6
P(A) = = = 0.1143
210 35

Solution : 124

Defective N.Defective Total Select

17 3 20 2

3 17 3 17
𝑐∗ 𝑐 + 𝑐∗ 𝑐
1 1 2 0 54 27
P(A) = 20 = 190 = 95 = 0.2842
𝑐
2

Solution : 125

Green Remaining Total Select

15 35 50 10

15 35
𝑐 ∗ 𝑐
3 7
P(A) = 50 = 0.2979
𝑐
10
Solution : 128

Outcomes = 22 * 6 = 24

Solution : 129

P = 2⁄5 , q = 3⁄5 , n = 7 , P(x > 6)

P(x > 6) = P (x = 6) + P (x = 7)

7 6 7−6 7 7 0
P(x > 6) = ( ) (2⁄5) (3⁄5) + ( ) (2⁄5) (3⁄5)
6 7
1472
= 78125 = 0.0188

Solution : 134

H = 1 , P(X>2)

P(X>2) = 1 – P(X<2)

= 1 – [P(X=0) + P(X=1)]

𝑒 −1 . (1)0 𝑒 −1 . (1)1
=1–[ + ]
0! 1!

1 1 2 𝑒−2
= [𝑒 + 𝑒] = 1 - 𝑒 = 2
= 0.2642

Solution : 136

Since A and B are independent then

P(A or B) = P(A) + P(B) – P(A) . P(B)

1 1 1 1
= 3 + 4 – (3) (4)

1 1 1
= 3 + 4 - 12

4+3−1 6 1
= 12
= 12 = 2

Solution : 137

µ = 2 , P(x=0)
𝑒 −2 . (2)0
P(x=0) = 0!

P(x=0) = 0.1352

Solution : 138

P = 4⁄5 , q = 1⁄5 , n = 3 , P(x>1)

P(x>1) = 1 – P(X<1)

= 1 – P(X=0)

3 0 4
= 1 – ( ) (4⁄5) (1⁄5)
0
1
= 1 – 625

625−1
= 625

624
= 625 = 0.99

Solution :139

µ = 3000 , O = 200 , X = 3400

𝑋−𝑀 3400−3000
Z= 𝑂
= 200
= 2

P(Z>2) = 0.5 – P(0<Z<2)

= 0.5 – Using R from calculator

= 0.0228

Solution : 141

Event = Constant = 7

Sample Space = total = 10

7
P(A) = 10

Solution : 142

µ = 10 , O = 2
x = µ – OZ

x = 10 – 2(1.90)

x = 6.2

Solution : 143

No. of wage = 3*2 = 6

Solution : 144

6 𝑐1 ∗ 8 𝑐1 48
P(A) = 14𝑐2
= 91

Solution : 145

4 26 2
P(A) = 52 + 52 - 52

28
= 52

Solution :146

M = 600 , O = 100

x = M + OZ

x = 600 + 100(1.28) = 728

Solution : 147

Wins one and losses two

P = 1⁄3 , q = 2⁄3 , n = 5 , P(x=3)

Using

5 3 2
P(x=3) = ( ) (1⁄3) (2⁄3)
3
1 4
= (10) ( ) ( )
27 9

40
= (243)
Solution : 149

µ = 50

x = µ + OZ

64.1 = 50 + O(1.41)

14.1 = O(1.41)

O = 10

𝑂2 = 100

Solution : 150

Total number of flaws given by

46
Average = 20 = 2.3

P(x > 3) = 1 – P(X<3) = 1 – [P(X=0) + P(X=1) + P(X=2)]

P(x>3) = 0.40396

Solution :153

No. of ways = 10𝑐 3 = 120

Solution : 154

9 1
𝑐 * 𝑐 = 126
4 1

Solution : 157

3𝑐 0 ∗ 4
P(A) = 7𝑐 2
𝑐2
= 2⁄7

Solution : 158

𝑒 −5 . (5)2
P(x=2) = 21

Variance = µ = 02 = 5

Solution : 159
P(women or light smoker male)

424 42
= 868 + 868

= 0.5369

Solution : 160

No. of ways = 4*5*1 = 20

Solution : 161

Number of possible codes

8*10*10*10*10 = 80,000

Number starting with 7 and ends with 8 is

1*10*10*10*1 = 100

1000 1
P(a) = 80,000 = 80

Solution : 162

No. of ways = 4*5*1 = 20

Solution : 163

A 5 digit pin code can not begin with zero and the remaining digit have no restriction. If repetition of
digit is allowed then what is the probability that the pin code begins with a 7 and ends with an 8
Sol.
Number of possible codes = 9*10*10*10*10 = 90,000

Number starting with 7 and end with 8

1*10*10*10*1 = 1000

1000 1
P(A) = 90,000 = 90

Solution : 164

1 1
P = 10 , q = 10 , n = 10 , P(x<1)

P(x<1) = 1 – P(x=0)
10 1 0 1 10
= 1 – ( ) (10) (10)
0
= 0.9999

Solution : 165

P(neither a ups nor a generator) = 1 – (0.7+0.3-0.2)


= 1 – 0.8
= 0.2 * 100%
= 20%

Solution :166

P(neither a cellular phone nor a pager) = 1 – (0.5+0.4-0.2)

= 1 – 0.7

= 0.30 * 100%

= 30%

Solution : 167

7! 5040
No. of ways = = = 1260
2! 21.21

Solution : 168

Math Accounting Total Select


6 12 18 2

6𝑐 2 ∗ 12𝑐 0 6𝑐 0 ∗ 12𝑐 2
P(A) = 18𝑐 2
+ 18𝑐 2

15 66
= =
153 153

81
P(A) = 153

Solution : 169

Total balls = 7
Blue = 2

2𝑐 2 ∗ 5𝑐 0 1∗1 1
P(A) = = = = 0.4762
7𝑐 2 21 21
Solution : 170

3𝑐 1 ∗ 4𝑐 1 12
P(A) = 7𝑐 2
= 21 = 0.5714

Solution :171

Sample space = 216

5 5 5 125 125
3 at ;east on on of three dice = 6 . 6 . 6 = 216 = 1 - 216

P(A) = 91⁄216

P(A) = 0.4212

Solution : 172

Sample space = 216

5 5 5 125
2 at least one of three dice = 1 – ( ) ( ) ( ) = 1 –
6 6 6 216

91
=
216

= 0.4212

Solution : 173

Sample space = 36

25
Getting at least one six = 36

25
= 1 – 36

11
= 36

Solution : 174

Batteries Defective Total Select


22 3 25 5

22𝑐 3 ∗ 3𝑐 2 2
P(A) = 25𝑐 5
= 23

= 0.0869 * 100%
= 8.7

Solution : 175

P(𝐶1 ) * (𝑉1 or 𝑉3 ) * 𝐸2

1 1 1 2
= (12) (3 + 3) (3)

1 2 2
=( )( )( )
12 3 3

1
= 27

Solution : 176

With vowels = 5! = 120

4!
Vowels = = 12
2!

No, of ways = 120* 12 = 1440

Solution : 177

With vowels = 5! = 120

Vowels = 3! = 6

No, of ways = 120* = 720

Solution : 178

µ = 6 , P(x>4)

P(x>4) = 1 – P(x<4)

= 1 – [P(x=0) + P(x=1) + P(x=2) + P(x=3)]

𝑒 −6 . 60 𝑒 −6 . (6)1 𝑒 −6 . (6)2 𝑒 −6 . (6)3


=1–[ 0!
+ 1!
+ 2!
+ 3!
]

= 1 – (0.002478 + 0.01487 + 0.04461 + 0.8923)

= 1 – 0.1511

= 0.8488
Solution : µ = 100 , O = 15 , Z = -1.20
then

x = µ + OZ

x = 100+ 15(1.20)

x = 82

Solution : 179

8! 5! ∗ 4!
No. of ways = –
2! 2!

= 20160 – 1440

= 18720

Solution : 180

Sample space = [HH,HT,TH,TT]

Event = 1

P(A) = 1⁄4 = 0.2

Question: 181

Vehicle passes through a junction on a busy road at an average rate of 300 per hour

(i) Find the prob. that none passes in a given minute


(ii) What is the expected number passing in two minutes?
(iii) Find the probability that this expected number actually passes through in a given two
minutes.

Solution:

300
Average = =5
60

𝑒 −5 . 50
(i) P(X=0) = 0!
= 0..0673
(ii) Expected number in two minutes = E(x) = 5*2 = 10
𝑒 −10 . 1010
(iii) M = 10, P(x=10) = 10!
= 0.12511
1/1/2022
SAMPLING AND
SAMPLING
DISTRIBUTION
CHAPTER 13
Sample:

Sample is a group of items or observations taken from the population for examination or
experiment A sample is a representative group from that population

Population:

A population include all people or items with characteristics that a researcher wishes to
understand. This mean that the term population means every single member of a particular
group.

Advantages of sampling:

• Cost is lower
• Data collection is faster
• Data set is smaller it is possible to ensure homogeneity and to improve the accuracy and
the quality of data

Example: if a drug manufacturer would like to research the adverse side effects of a drug on the
country’s population, it is almost impossible to conduct a research study that involves everyone.
In this case, the researcher decides a sample of people from each demographic and then
researches them, giving him/her indicative feedback on the drug’s behavior.

Sampling frame:
The sampling frame is a list of all the members of the population used as a basis for sampling.
The frame, defines the population
Sampling methods:
The objective of any of these techniques is to extract a sample which is representative of the
population. This sample is then tested and the result treated are representative of the full
population
• Simple random sampling: Each individual has the same probability of being chosen to be a
part of a sample.
Example: in an organization of 500 employees, if the HR team decides on conducting team
building activities, it is highly likely that they would prefer picking chits out of a bowl. In this
case, each of the 500 employees has an equal opportunity of being selected.

• Cluster sampling: Cluster sampling is a method where the researchers divide the entire
population into sections or clusters that represent a population. Clusters are identified and
included in a sample based on demographic parameters like age, sex, location, etc. This makes
it very simple for a survey creator to derive effective inference from the feedback.

1|Page
• Example: if the United States government wishes to evaluate the number of immigrants living
in the Mainland US, they can divide it into clusters based on states such as California, Texas,
Florida, Massachusetts, Colorado, Hawaii, etc. This way of conducting a survey will be more
effective as the results will be organized into states and provide insightful immigration data.

• Systematic sampling: Researchers use the systematic sampling method to choose the sample
members of a population at regular intervals. It requires the selection of a starting point for the
sample and sample size that can be repeated at regular intervals. This type of sampling method
has a predefined range, and hence this sampling technique is the least time-consuming.

Example: a researcher intends to collect a systematic sample of 500 people in a population of


5000. He/she numbers each element of the population from 1-5000 and will choose every 10th
individual to be a part of the sample (Total population/ Sample Size = 5000/500 = 10).

• Stratified random sampling: Stratified random sampling is a method in which the researcher
divides the population into smaller groups that don’t overlap but represent the entire
population. While sampling, these groups can be organized and then draw a sample from each
group separately.
Example: a researcher looking to analyze the characteristics of people belonging to different
annual income divisions will create strata (groups) according to the annual family income.

Example – less than $20,000, $21,000 – $30,000, $31,000 to $40,000, $41,000 to $50,000, etc.
By doing this, the researcher concludes the characteristics of people belonging to different
income groups. Marketers can analyze which income groups to target and which ones to
eliminate to create a roadmap that would bear fruitful results.

• Quota sampling: this is a sampling method where population can be divided into group(of
homogenous characteristics) and samples are taken from each group(quota). It is a rapid
method of collecting samples.

Example: an interviewer may be told to sample 200 females and 300 males between the age of
45 and 60
Convenience sampling
Convenience sampling is based on the availability of respondents or observations. It is also
called accidental or opportunity sampling. Ease of access and respondent willingness is the key
for convenience sampling. One problem with convenience sampling is lack of generalizability of
the results since the data may be biased or inclined towards a segment and may not be
representative of the whole population

Sampling theory
Sampling is used to investigate the characteristics of a population by studying small
groups(sample) taken from that population
Problems solved by sampling

2|Page
• Statistical inference
• Hypothesis testing

Sampling distribution
A sampling distribution is a distribution of a sample characteristics, selected from many samples
and is used to determine how close to the population parameter a sample statistic is likely to fall.
The sampling distribution is a probability distribution of a sample statistic.

Characteristics of sampling distribution


• The mean of sampling distribution is the same as the mean of population
• A sampling distribution is a normal distribution
• It has the same mathematical characteristics of all other normal distributions
• It is symmetrical about its mean (even if the distribution within the population being
sampled is not symmetrical)

Points to remember:
(i) A sample is subset of population
(ii) Any population constant is called parameter or any measure of population is called
parameter
(iii) Any calculation on the sampling data is called statistic
(iv) Standard deviation of sampling distribution of a statistic is called standard error
(v) Probability distribution of a statistics is called sampling distribution
(vi) The difference between a statistic and the parameter is called sampling error
(vii) A sampling distribution might be based on which of the following (sample mean,
sample correlations and sample proportion)
(viii) A good way to get a small standard error is to use a large sample
(ix) A population about which we want to get some information is called target population
(x) A study of population is called census
(xi) Standard error is always positive
(xii) Random sampling is also called probability sampling
(xiii) In sampling with replacement an element can be chosen more than once
(xiv) Standard deviation of sample means without replacement less than standard deviation
of sample mean with replacement
(xv) In sampling with replacement, the following is always true (N=n, N<n and N>n)
(xvi) A complete list of all the sampling unit is called sampling frame
(xvii) In systematic sampling the first element is randomly selected and thereafter all other
elements are selected based on predetermined plan
(xviii) Judgment sampling is not a type of random sampling
𝛔
(xix) In sampling with replacement 𝛔𝐱̅ = 𝐧 =standard Error

(xx) Random sampling is a technique of selecting sample in which all population elements
have equal chance of selection and every sample has an equal chance of selection

3|Page
Formula:

Sampling with/without replacement


(1) (i) µ x̅ = µ
σ N−n
(ii) σx̅ = (√ N−1)
√n
(2) (i) µx1-x2 = µ1 ‒ µ2
(ii) σ2 x1 −x2= σ2 x1 + σ2 x2

(3) Standard errors of mean


s
σx̅ =
√n
Standard deviation of a sampling distribution is called standard errors.

(4) Estimating Population Mean.


σ
(i) µ = x̅ ± z value n

s
(ii) µ = x̅ ± z value
√n

(5) Sampling distribution of proportion


pq
(i) Standard error of proportion σp = √ n

(6) Standard error of mean-large Sample


σ
Standard error =
√n
Where
∑(x − x̅)2
S= √
n

(7) Standard error of mean – small sample


S
σ=
√n
Where
∑(x − x̅)2
S= √
n−1

(8) For t distribution


µ = x̅ ± (t095,9) σ

(9) Required range = 1.96 × standard error

(10) Finite population correction (FPC)

4|Page
N−n
= √
N−1

Sampling with and without replacement

Question: 1
A population consists of four children with ages 2, 4, 6 and 8. Take all possible simple random
samples of size 2 with replacement. If X is the age of a child, Find
(i) The theoretical sampling distribution of 𝑥̅ , the mean age of two children in a sample.
(ii) The mean, variance and standard error of 𝑥̅ .
(iii) The mean, variance and standard deviation of the population.
Solution:
Population: 2, 4, 6, 8
Population size: N = 4
Sample size: n = 2
No. of Possible Samples: 𝑁 𝑛 = 42 = 16

Samples 𝑋̅ Samples 𝑋̅
2,2 2 6,2 4
2,4 3 6,4 5
2,6 4 6,6 6
2,8 5 6,8 7
4,2 3 8,2 5
4,4 4 8,4 6
4,6 5 8,6 7
4,8 6 8,8 8
Sampling Distribution:
𝑋̅ Tally f P(𝑋̅) 𝑋̅P (𝑋̅) 𝑋̅ 2 P(𝑋̅)
2 I 1 1/16 2/16 4/16
3 II 2 2/16 6/16 18/16
4 III 3 3/16 12/16 48/16
5 IIII 4 4/16 20/16 100/16
6 III 3 3/16 18/16 108/16
7 II 2 2/16 14/16 98/16
8 I 1 1/16 8/16 64/16
∑ ---- 16 16/16 = 1 80/16 440/16

Mean:
𝜇𝑥̅ = E (𝑋̅) = ∑ 𝑋̅P (𝑋̅)
= 80 / 16
=5
Variance:
𝜎𝑥̅ 2 = Var (𝑋̅) = ∑ 𝑋̅ 2 P(𝑋̅) - [∑ 𝑋̅ 𝑃(𝑋̅)]2

5|Page
440 80 2
= 16 - [16]
= 27.50 – 25
= 2.50
S.E:
𝜎𝑥̅ = √2.50
= 1.58

Mean variance and S.D of the Population:

X 𝑋2
2 4
4 16
6 36
8 64
20 120

Mean:
∑𝑋
𝜇= 𝑁
20
= 4
=5
Variance:
∑ 𝑋2 ∑𝑋 2
𝜎2 = -( )
𝑁 𝑁
120 20 2
= 4 -(4)
= 30 – 25
=5
S.D
𝜎 = √5
= 2.236

Verification:

𝜎2 𝜎
𝜇𝑥̅ = 𝜇 𝜎𝑥̅ 2 = 𝜎𝑥̅ =
𝑛 √𝑛
5=5 5 2.236
L.H.S = R.H.S 2.50 = 2 1.58 =
√2
2.50 = 2.50 1.58 = 1.58
L.H.S = R.H.S L.H.S = R.H.S

Question: 2

A population has five values 3,6,9,12,15

6|Page
(i) Calculate mean and variance of population
(ii) Select all possible samples of size 3 without replacement
(iii) Calculate mean of each sample
(iv) Make sampling distribution of 𝑋̅ and find its mean and variance
(v) Verify that

(a) 𝜇𝑥̅ = 𝜇
𝜎2 𝑁−𝑛
(b) 𝜎 2 𝑥̅ = 𝑛 𝑁−1

Solution:
(i)

X 𝑋2
3 9
6 36
9 81
12 144
15 225
45 495

∑𝑋 45
𝜇= = =9
𝑁 5
∑ 𝑋2 ∑𝑋 2
𝜎2 = -( )
𝑁 𝑁

495 45 2
𝜎2 = 5 - ( 5 )
𝜎 2 = 18

(ii)- NO of samples of size 3 without replacement = 5𝐶3 = 10

3,6,9 3,9,12 6,9,12


3,6,12 3,9,15 6,9,15
3,6,15 3,12,15 6,12,15
9,12,15

(iii)- Sample mean 𝑋̅

6 8 9
7 9 10
8 10 11
12
(iv)

𝑋̅ Tally f P(𝑋̅) 𝑋̅P (𝑋̅) 𝑋̅ 2 P(𝑋̅)


6 I 1 1/10 6/10 36/10

7|Page
I 1 1/10 7/10 49/10
8 II 2 2/10 16/10 128/10
9 II 2 2/10 18/10 162/10
10 II 2 2/10 20/10 200/10
11 I 1 1/10 11/10 121/10
12 I 1 1/10 12/10 144/10
10 90/10 84010

90
𝜇𝑥̅ =∑𝑋̅ P(𝑋̅) = 10 = 9

𝜎 2 𝑥̅ =∑ 𝑋̅ 2 P(𝑋̅) - [∑𝑋̅P (𝑋̅)]2

840 90 2
𝜎 2 𝑥̅ = 10 - (10)
𝜎 2 𝑥̅ = 3

(iv)-(a)

𝜇𝑥̅ = 9 𝜇=9

So
𝜇𝑥̅ = 𝜇 = 9

𝜎2 𝑁−𝑛
b) 𝜎𝑋2̅ = 𝑛 𝑁−1

18 5−3
3 = 3 . 5−1
3=3
L.H.S = R.H.S

Question: 3
A sample of five price /earnings ratios for companies in the service sector follows 37,11,14,17,12
Estimate the population mean p/e ratio for the services sector
Solution:
Given
37, 11,14,17,12
∑𝑥 91
𝜇= = = 18.2
𝑁 5
Question: 4
A sample of five price /earnings ratios for companies in the service sector follows 37,11,14,17,12
Estimate the population standard deviation for the p/e ratio for the services sector
Solution:
∑𝑥 2 ∑𝑥
𝜎=√ 𝑁
− ( 𝑁 )2

8|Page
2119
𝜎=√ − (118.2)2
5

= √423.8 − 331.24
= √92.24
= 9.62

Question: 5
A population has mean of 75 and a standard deviation of 10. Sample of size 20 are chosen and
the sample means recorded. What is the standard deviation of the sample means?
Solution:
𝜇= 75, 𝜎 = 10, n = 20
𝜎
𝜎𝑥̅ =
√𝑛
10
=
√20
= 2.24

Question: 6
Sample of size 81 are taken from a population of size 500 with mean 10 and standard deviation
3, Find the standard deviation of the sample means
Solution:
n = 81, N= 500, 𝜇 = 10, 𝜎 = 3
𝜎 3 3 1
𝜎𝑋̅ = 𝑛 = = 9 = 3 = 0.33
√ √81

Question: 7
From a given finite population sample drawn with replacement. If the sample size is decrease
from 70 to 50. The standard error would be

Solution:

1
S.E = =0.1195
√70
1
S.E = = 0.1414
√50
0.1414 −0.1195
Decrease in standard error = =0.1832=18.3%
0.1195
Note:
(i) If sample size increase then sampling error will decrease
(ii) If sample size decrease then sample error will increase

9|Page
Estimation
A sample can be taken to give an estimate of the value of the population. When single figure is
used as an estimate of a parameter of a population is called point estimate

Confidence levels
Refers to the percentage of all possible samples that can be expected to include the true
population parameter.

Example: suppose all possible sample were selected from the same population, and a confidence
interval were computed for each sample. A 95% confidence level implies that 95% of the
confidence intervals would include the true population parameter

Estimating population Mean


FORMULA:
𝒛∝ 𝒔
̅±
µ=𝒙 𝟐 √𝒏

Question: 1
A sample of 100 male ICAP students had a mean height of 170cm with a standard
deviation of 8cm. Estimate population mean at 95%. Confidence interval.

Solution:
We know
𝑧 𝑠
µ = 𝑥̅ ± 2∝
√𝑛
8
µ = 170 ± 1.96 ( )
√100
µ = 170 ± 1.96 (0.8)
µ = 168.43 to 171.57

Conclusion: We are 95% confident that the mean height of all male ICAP students is between
168.43 and 171.57cm. The value at each end of the range is called confidence
limits. The range itself is called confidence interval
Question: 2
A sample of 100 male students had a mean height of 170cm with a standard
deviation of 8cm.Estimate population mean at 99%. Confidence interval.

Solution:
We know
𝑧 𝑠
µ = 𝑥̅ ± 2∝
√𝑛
8
µ = 170 ± 2.58
√100
µ = 170 ± 2.58(0.8)
µ = 167.94 to 172.06

10 | P a g e
Conclusion: We are 99% confident that the mean height of all male ICAP students is between
167.94 and 172.06.

Estimating population proportion (population = P0

Question: 3
In a sample of 500 people 215 of them were males. Estimate the population
proportion at the rate of 95% confidence interval.
Solution:
We know
𝑧∝ 𝑃𝑞
P0 = 𝑝 ± √ ___ (1)
2 𝑛
Where
215
p= 500
= 0.43
∴ Put in 1
(0.43 )(0.57)
P0 = 0.43 ± 1. 96 √ 500
P0 = 0.43 ± 1. 96 (0.0221)
P0 =0.387 to 0.473
Conclusion: We are 95% confident that the population proportion of mean is between 0.387 and
0.473.

Conclusion: If the sample size is set at 2354 it will result in a confidence interval of the sample
proportion ± 2% at the 95% confidence level.

Question: 4
In a random sample of 400 adults and 600 teenagers who watched a certain television
programme. 100 adults and 300 teenagers indicated that they liked it. Construct (Estimate) 95%
and 99% confidence limits for the difference in proportions of all adults and all teenagers who
watched the programme and liked it.

Solution:

Adults: Teenagers:

𝑛1 = 400 𝑛2 = 600
𝑥1 = 100 𝑥2 = 300
𝑥 100 𝑥 300
Sample proportion=𝑝1 = 𝑛1 = 400 = 0.25 𝑝2 = 𝑛2 = 600 = 0.50
1 2

(i) 95% C.I =?


(ii) 99% C.I =?

11 | P a g e
Formula:
𝑝1 𝑞1 𝑝2 𝑞2
= (𝑝2 - 𝑝1) ± 𝑧𝛼 √ +
2 𝑛1 𝑛2
(i) 95%

0.25 (1−0.25 ) 0.50 (1−0.50)


= (0.50 – 0.25) ± (1.96) √ +
400 600

=0.25 ± (1.96) √0.0005 + 0.0004


=0.25 ± (1.96)(0.03)
=0.25 ± 0.06
0.25 – 0.06 = 0.19
0.25 + 0.06 = 0.31
Hence 95% confidence interval for (𝒑𝟐 -𝒑𝟏 ) obtained from the observed sample is (0.19,
0.31).
(ii) 99%
(0.50 – 0.25) ± (2.576)
0.25 (1−0.25) 0.50(1−0.50)
√ +
400 600
0.25 ± (2.576)
√0.0005 + 0.0004
0.25 ± (2.576)(0.03)
0.25 ± 0.08
0.25 – 0.08 = 0.17
0.25 + 0.08 = 0.33

Hence 99% confidence interval for (𝒑𝟐 − 𝒑𝟏 ) obtained from the observed sample is (0.17,
0.33)
Check your self :
Question: 1
on a random stock check of 40 items, the mean value per item was found to be Rs 380 and the
standard deviation Rs.100. calculate 95% confidence limits for the overall mean value per item
Question: 2
A dock uses a mathematical shovel to load grain. A random sample of 50 scoops of the shovel
has a mean weight of 510 kg and a standard deviation of 30 kg . construct a 95% confidence
interval for the mean of the population
Question: 3
A company is testing the breaking strengths of cables measured in Newtons per square
millimeter (N/mm2). A random smple of 50 cables had a mean breaking strength of 1600
N/mm2 with a standard deviation of 60 N/mm2. Construct a 99% confidence interval for the
mean breaking strength
12 | P a g e
Estimating value of n (Sample Size)
Question: 5
A sample of 100 ICAP students had a mean height of 170cm with a standard
deviation of 8cm. Calculate the sample size that will result in a confidence
interval of the mean ± 1cm for this data having 5% level of significance.
Solution:
We know
𝑧 𝑠
µ = 𝑥̅ ± 2∝ 𝑛

Also given
µ = mean ± 1cm
Compare
𝑧∝ 𝑠
=1
2 √𝑛
8
1.96 =1
√𝑛
(1.96) (8) = √𝑛
n = 246
Checking:
𝑧∝ 𝑠
µ = 𝑥̅ ± 2 √𝑛
8
µ = 170 ± 1.96 *
√246
µ = 170 ± 1.96 (0.51)
µ = 170 ± 1

Question: 6
In a sample of 500 people 215 of them were males. Calculate the sample size that
will result in a confidence interval of the sample Proportion ±2% at the 95%
confidence level.

Solution:
We know
𝑧∝ 𝑝𝑞
P0 = P ± √𝑛
2

215 (0.43 )(0.57)


P= ± 1. 96 √ ____ (1)
500 𝑛
Also given
Sample proportion ± 2% _____ (ii)

Compare (i) & (ii)


0.2451
1.96√ = 0.02
𝑛
1.96 √0.2451
=n
0.02
n = 2354
Estimating sample size (method # 2)

13 | P a g e
Question: 7

A television channel has claimed that on average a teenager spends 7 hours each month on
viewing its entertainment programs. An advertising agency wants to carry out a survey to assess
the reasonableness of the above claim. Determine the size of the sample which would be needed
in order to assert with 99% confidence that the error in the above claim does not exceed 30
minutes. Assume that 𝜎 = 3.2 hours.

Solution:
In sampling
𝑍𝛼⁄ 2
2 ∗𝜎
n= ( 𝑒 ) where 𝜇 = 7

𝜎 = 3.2, e = 30/60= 0.5 hours and 𝑍𝛼⁄2 = 𝑍0.005 = 2.58


2.58∗3.2 2
n=( ) = 272.646
0.5
Hence the sample size should be 273

Question: 8
In measuring reaction time, a psychologist estimated that the standard deviation is 0.05 second.
How large a sample of measurements must he take in order to be 95% confident that the error in
his estimate of mean reaction time will not exceed 0.01 second?

Solution:
The sample size that means reaction time will not exceed 0.01 seconds to 95% confident

𝑍𝛼 2
2
∗𝜎 (1.96)(0.05) 2
n=( ) =( ) = 96
𝑒 0.01

If sample size is 96 then it is 95% confidently said that reaction time will not exceed 0.01
seconds.

Question: 9
An advertising company wants to estimate with 97.5% confidence, the number of times a
website is hit during an hour. It has determined that 𝜎 = 26.
How large a sample should the company take, if it wishes that the margin of error should not
exceed 10?
solution:
𝑍𝛼⁄2 ∗ 𝜎 2
n=( )
𝑒
(2.24)(26) 2
n=( ) = 34
10

Question: 10

14 | P a g e
The owner of a shopping mall wants to estimate the mean length of time shoppers spend in the
mall. He wants the estimate to be within 5 minutes of the true mean time, with a 98% level of
confidence. The estimate of the standard deviation of the length of time spent in the mall is 20
minutes. How large a sample is needed?

Solution:
Given standard deviation 𝜎 = 20 minutes
𝛼0.02 = 2.326

̅=5
Let 𝝁 = 0, 𝑿

𝑍 𝛼⁄2 𝜎 20 (2.326)
√𝑛 = |𝑥̅ − μ| = 5
√𝑛 = 9.304 or n = 87 rounded

Question: 11
A survey is being planned to determine the mean amount of time, senior citizens (older than 60
years) watch T.V. A pilot survey indicated that the means time per week is 12 hours with a
standard deviation of 3 hours. It is desired to estimate the mean viewing time within a quarter
hour. 95% confidence level is to be used. How many senior citizens should be surveyed, and
𝑥̅ = 12.25

Solution:
Given
Mean time = 12 hours
Standard deviations = 3hours
The number of citizens to be surveyed for within a mean time of quarter hour

𝑍𝛼⁄2 𝜎 2
n=( ) where
𝑒
𝑍𝛼⁄2 at 95% = 1.96
̅ − 𝝁|
e = |𝒙
1.96 (3) 2 5.88 2
n = (12.25−12) = (0.25) = 553

Check yourself:
Question:
On a random check of 40 items, the mean value per item was found to be Rs 380 and
the standard deviation Rs 100. Calculate the sample size that will result in a
confidence interval of the mean ± Rs 20. For this data at the 95% confidence level.

15 | P a g e
Estimation of population mean -small sample using T-test
Question: 1
Estimate the population mean at the 95% confidence level from the
following sample
5.5, 6.1, 5.4, 5.8, 5.3, 4.8, 5.2, 4.6, 6.1, and 5.2
Solution:
X x - 𝑥̅ (𝑥 − 𝑥̅ )2
5.5 0.1 0.01
6.1 0.7 0.49
5.4 0.0 0.00
5.8 0.4 0.16
5.3 -0.1 0.01
4.8 -0.6 0.36
5.2 -0.2 0.04
4.6 -0.8 0.64
6.1 -0.7 0.49
5.2 -0.2 0.04
54 2.24

∑𝑥 54
𝑥̅ = 𝑛 = 10 = 5.4
Also
∑(𝑥− 𝑥̅ )2 2.24
S=√ = √10−1 = 0.5
𝑛−1
Now
𝑠
µ = 𝑥̅ ± t 95% (d.f)
√𝑛
0.5
µ = 5.4 ± (2.262 * )
√10
µ = 5.4 ± (2.262 * 0.158)
µ = 5.4 ± 0.36
= 5.04 to 5.76
Conclusion: we are 95% confident that the mean is between 5.04 and 5.76

Note: How to use t – table


i) Degree of freedom = n-1 = 10- 1 = 9
ii) Confidence level 95% = 1 -0.95 = 0.05 (level of significance)
0.05
iii) Level of significance = = 0.025
2
Note: How to use z – table
i) One tail , ∝ = 5%
= 1-0.05 = 0.95 – 0.50
= 0.45
ii) Two tail

16 | P a g e
0.95
95% = 2
= 0.475

Normal distribution in Sampling


Question: 1
Find out the probability that a random sample of size 36 selected with replacement from the
population 3,5,1,7,3,6,2,4,5,2 will yield a sample mean greater than 3.2 but less than 4.5

Solution:
∑𝑋 3+5+1+7+36+2+4+5+5
µ= = = 3.8
𝑛 10
∑𝑋 2 ∑𝑋 32 + 52 +12 +72 +32 +62 +22 +42 +52 +22 38
𝜎= √ − ( 𝑛 )2 = √ − (10)2
𝑛 10
= √17.8 − 14.44 = 1.833
Probability that sample mean falls between 3.2 and 4.5

̅− 𝝁
𝑿
Z= 𝝈
√𝒏

𝑋̅ − 𝜇 3.2−3.8
Z= 𝜎 = 1.833 = -1.96
√𝑛 √36

3.2 3.8 4.5


̅ 𝟐− 𝝁
𝑿
Z= 𝝈
√𝒏
4.5−3.8
= 1.833 = 2.29
√36
P(−1.96 ≤ 𝑧 ≤ 2.29)
= p (-1.96 ≤ Z ≤ 0) + p (0 ≤ Z ≤ 2.29) = 0.475+0.489 = 0.964

Question: 2
A sample of 50 DVD has a population mean is 720. The sample mean is 700, find the S.d if level
of significance is 1%

Solution:

̅−𝝁
𝒙
Z= 𝝈/
√𝒏

700−720
2.575 = 𝜎/√50

𝜎=54.92

Question: 3

17 | P a g e
A sample has been taken from normally distributed population and sample mean has been found
to be 62. The upper confidence limit of 95% for population mean is 84.60, population variance is
known to be 2401.what is sample size

Solution:

𝜎 = √2401 =49

𝑥̅ −𝜇
Z= 𝜎/
√𝑛

62−84.6
1.96= 49/√𝑛

n=18

Question: 4
In a rice mill the bags of rice has mean weight of 5.05 kg and standard deviation of 0.02 kg. if a
bag is selected at random then find the probability that its weight is below 5 kg.

Solution:

𝒙−𝝁
Z= 𝝈

5.0−5.05
Z= =-2.5
0.02

Area from normal table


Z =-2.5 is 0.4938
So required area is
0.5 -0.4938 =0.00662

Question:5
With a sample size of 900, standard error of 3, what should be sample so that we could be 95%
confident that population mean is within 4 units of sample mean.
Solution:
𝜎
𝜎𝑋̅ = Standard error =
√𝑛
𝜎
3=
√900

𝜎 = 90

18 | P a g e
Also
sample size
𝑍𝑥⁄2 . 𝜎 2
n=( )
𝑒
1.96×90 2
n=( )
4
n = 1944.81
n = 1945
Question:6
Given
𝛴𝑥 2 = 1850, Σx= 160, n = 16
Find standard error
Solution:
1 (𝛴𝑥)2
S = √𝑛−1 (𝛴𝑥 2 − )
𝑛

1 160 2
S = √14 (1850 − ( )
16

S = 4.0825
Also
Standard error
𝜎 𝑆 4.0825
𝜎𝑋̅ = = = = 1.02062
√𝑛 √𝑛 √16

Question:7
Given
Σ(𝑥 − 𝐱̄ )2 = 700, n = 50
Find
i) standard deviation
ii) standard error
Solution:
𝛴(𝑥− 𝐱̄ )2
i) S.D = √ 𝑛

700
= √ 50

= 3.7417
𝑆 3.7417
ii) S.E = = = 0.52915
√𝑛 √50

19 | P a g e
Question:8
Sample size increase from 40 to 75. Find change in standard error.
Solution:
let σ = 1
σ 1
S.E = = = 0.1581
√𝑛 √40
σ 1
S.e = = = 0.1154
√𝑛 √75
0.1581−0.1154
Change = 0.1581

Change = 26.97%
Question:9
n = 300, male = 109
Find 95% confidence interval af male student in college
Solution:
𝑝𝑞
𝑃𝑜 = P ± 𝑍𝑥⁄2 √ 𝑛
where
𝑥 109
P = 𝑛 = 300 = 0.3633

Now
1.96 0.3633 ×0.6367
𝑃𝑜 = 0.3633 ± √
300

𝑃𝑜 = 0.3633 ± 0.054
𝑃𝑜 = 30.9% and 41.73%
Question:10
A sample of 700 people in a city 313 are found men. You are required to construct a 95 %
confidence interval of all the men in a city (population proportion)
Question:11
Standard deviation of 10 values is 12. A.M is 246, then find sum of squared observations.
Solution:
𝛴𝑥 2 𝛴𝑥 2
S=√ − (𝑛)
𝑛

𝛴𝑥 2
12 = √ 10 − (246)2

20 | P a g e
Σ𝑥 2 = 606600
Question:12
A population is normally distributed with men 6000 and standard deviation of 400, if a value is
randomly selected. Find the probability that value is above 6800.
Question:13
A population has a mean of 75 and a standard deviation of 10. Sample size 20 are chosen and
sample means recorded what is the standard deviation of the sample means.
Solution:

µ = 75, σ = 10, n = 20
𝜎 10
S.E = 𝑛 = = 2.24
√ √20

Question:14
Sample of size 81 are taken from a population of size 500 with mean 10 and standard deviation
of 3. Find the standard deviation of sample means.
Solution:
Given
n = 81, N = 500 , x̄ = 10 , σ = 3
standard deviation of sample mean
𝜎 3
S.E = = = 0.33
√𝑛 √81

Question:15
The admission college as a college reported that this year’s freshman class had an average SAT
score of 1103 with a standard deviation of 95. What is the probability that a sample of 40
students from this year’s freshman class had and average score greater than 1120?

21 | P a g e
SAMPLING AND SAMPLING DISTRIBUTIONS
13.1 The probability distribution of a statistic is called a:
Probability distribution Sampling distribution
Frequency distribution None of these
13.2 The probability distribution of 𝑋̅is called:
Probability distribution of mean Sampling distribution of mean
Frequency distribution of mean None of these
13.3 It is customary to refer to the standard deviation of the sampling distribution as the:
Variance Standard deviation of mean
Standard error Mean deviation
13.4 If all possible samples of size n are drawn, without replacement, from a finite population
of size N with mean 𝜇 and standard deviation 𝜎, then the sampling distribution of the
sample mean X will be approximately normally distributed with a mean and standard
deviation given by:

𝑠 𝑁−𝑛 𝑠 𝑁−𝑛
𝜇𝑥 = x and 𝜎𝑥 = √ 𝜇𝑥 = x and 𝜎x = √
√𝑛 𝑁−1 √𝑛 𝑁−1
𝝈 𝑵−𝒏 None of these
𝝁𝒙 = x and 𝝈𝒙 = √
√𝒏 𝑵−𝟏
𝑁−𝑛
13.5 The quantity √ 𝑁−1 is known as:

Sampling fraction Population correction


Finite population correction None of these
13.6 If the population size is infinite or N is large as compared to n, then the finite population
correction factor will be approximately equal to:
Zero One
Infinite None of these
13.7 If n = 10 and N = 100 the value of finite population correction factor (FPC) is:
0.9909 1
0.9954 One of these
13.8 If n = 2 and N = 5 the value of FPC is:
0.75 0.86
0.866 None of these
13.9 If n = 40 and N = 10000 the value of FPC is:
0.99 0.996
0.998 None of these
13.10 IF N = 4, n = 2, 𝜇 = 5.25; 𝜎2 = 2.1875 then the values of 𝜇𝑥 and 𝜎𝑥 if sampling is done
without replacement:
5.25 and 1.2076 5.25 and 0.8539

22 | P a g e
5.25 and 0.729 None of these
13.11 If n = 2, 𝜇 = 5.25 and 𝜎2 = 2.1875 then the values of 𝜇𝑥 and 𝜎𝑥 if sampling is done with
replacement, are:
5.25 and 1.09375 5.25 and 1.0458
Cannot be determined None of these
13.12 From a given population with 𝜎 = 5.6 kilograms samples were drawn with replacement,
how may the standard error of the mean change, when the sample size is increased from
64 to 196:
Reduced from 0.7 to 0.4 Increased from 0.4 to 0.7
Reduced from .0875 to .0286 Increased from .0286 to .0875
13.13 From a given population with 𝜎 = 5.6 kilograms samples were drawn with replacement,
how may the standard error of the mean change, when the sample size is decreased from
784 to 49:
Decreased from 0.8 to 0.2 Increased from 0.2 to 0.8
Increased from 0.0071 to 0.114 Decreased from 0.114 to 0.0071
13.14 If the random variable x and y are independent and normally distributed with means 𝜇𝑥
and 𝜇𝑦 and variances 𝜎 2 x-y and 𝜎 2 y respectively then the distribution of the difference
x-y is normally distributed with mean 𝜇𝑥−𝑦 = 𝜇𝑥 - 𝜇𝑦 and variance:

𝜎 2 x-y = 𝜎 2 x - 𝜎 2 y 𝝈𝟐 x-y = 𝝈𝟐 x + 𝝈𝟐 y
𝜎 2 x-y = 𝜎 2 x - 2𝜎𝑥 𝜎𝑦 + 𝜎 2 y None of these
13.15 A sampling process that selects every kth element in the population for the sample, with
the starting point determined at random from the first k elements is known as:
Sample random sampling Stratified random sampling
Systematic random sampling None of these
13.16 A sampling process that selects simple random samples from mutually exclusive sub
populations, of population is called:
Simple random sampling Stratified random sampling
Systematic random sampling None of these
13.17 A sampling process which selects samples from the given population with equal chance
of selection to each unit is called:
Systematic random sampling Stratified random sampling
Simple random sampling None of these
8 3 9
13.18 Given 𝜇1 = 5, 𝜎 2 1 = 3 , 𝜇2 = 2 and 𝜎 2 2 = 4 for two independent normal variates 𝑥1 then
𝑥2 then 𝜇𝑥1 - 𝑥2 and 𝜎 2 𝑥1 - 𝑥2 are:
13 59 𝟕 𝟓𝟗
and 12 and 𝟏𝟐
2 𝟐
7 5 None of these
2
and 12

23 | P a g e
13.19 Sample is a sub-set of:
Population Data
Set Distribution
13.20 Any population constant is called a:
Statistic Parameter
Estimate Estimator
13.21 List of all the units of the population is called:
Random sampling Bias
Sampling frame Probability sampling
13.22 Any calculation on the sampling data is called:
Parameter Statistic
𝑥̅ Error
13.23 Any measure of the population is called:
Finite Parameter
Without replacement Random
13.24 If all the units of a population are surveyed, it is called:
Random sample Random sampling
Sampled population Complete enumeration
13.25 Probability distribution of a statistic is called:
Sampling Parameter
Data Sampling distribution
13.26 If you drew all possible samples from some population, calculated the mean for each of
the samples. And constructed a line graph(showing the shape of the distribution) based on
all of those means, what would you have?
A population distribution A sample distribution
A sampling distribution A parameter distribution
13.27 The difference between a statistic and the parameter is called:
Probability Sampling error
Random Non-random
13.28 The sum of the frequencies of the frequency distribution of a statistic is equal to:
Sampling size Population size
Possible samples Sum of x values
13.29 Standard deviation of sampling distribution of a statistic is called:
Serious error Dispersion
Standard error Difference

24 | P a g e
13.30 If we obtain a point estimate 𝑥̅ for a population mean𝜇, the difference between 𝑥̅ and 𝜇,
is:
Standard error Bias
Error of estimation Difficult to tell
13.31 What is the standard deviation of a sampling distribution called?
Sampling error Sample error
Standard error Simple error
13.32 A ______ is a subset of a _____.
Sample population Population , sample
Statistic, parameter Parameter, statistic
13.33 A _______ is a numerical characteristic of a sample and a ____ is a numerical
characteristic of a population.
Sample, population Population, sample
Statistic, parameter Parameter, statistic
13.34 A sampling distribution might be used on which of the following?
Sample means Sample correlation
Sample proportions All of these
13.35 A good way to get a small standard error is to use a______.
Repeated sampling Small sampling
Large sample Large population
13.36 This is the difference between a sample statistic and the corresponding population
parameter.
Standard error Sampling error
Difference error None of these
17.37 _______ is the standard deviation of a sampling distribution:
Standard error Sample standard deviation
Replication error Meta error
13.38 A distribution formed by all possible values of a statistic is called:
Binomial distribution Hyper geometric distribution
Normal distribution Sampling distribution
13.39 In probability sampling, probability of selecting an item from the population is known
and is:
Equal to zero Non zero
Equal to one All of these
13.40 A population about which we want to get some information is called:
Finite population Infinite population
Sampling population Target population

25 | P a g e
13.41 The population consists of the results of repeated trials are named as:
Finite population Infinite population
Real population Hypothetical population
13.42 Study of population is called:
Parameter Statistic
Error Census
13.43 For making voters list in Pakistan we need:
Sampling error Standard error
Census Simple random sampling
13.44 Sampling based upon equal probability is called:
Probability sampling Systematic sampling
Simple random sampling Stratified random sampling
13.45 In sampling with replacement, an element can be chosen:
Less than once More than once
Only once Difficult to tell
13.46 Standard deviation of sample mean without replacement _______ standard deviation of
sample mean with replacement:
Less than More than
2 times Equal to
13.47 In sampling without replacement, an element can be chosen:
Less than once More than once
Only once Any of the above
13.48 In sampling with replacement, the following is always true:
N=n N>n
N<n All of these
13.49 Which of the following statement is true?
Standard error is always one Standard error is always zero
Standard error is always negative Standard error is always positive
13.50 Random sampling is also called:
Probability sampling Non- probability sampling
Random sampling Representative sample
13.51 Non-random sampling is also called:
Biased sampling Non-probability sampling
Random sampling Representative sample
13.52 Sampling error can be reduced by:
Non-probability sampling Systematic sampling
Decreasing the sample size Increasing the sample size

26 | P a g e
13.53 The selection of cricket team for the world cup is called:
Random sampling Systematic sampling
Purposive sampling Cluster sampling
13.54 A Sampling distribution is
(a) an exclusive distribution (b) mean distribution
(c) a-symmetric distribution (d) normal distribution
13.55Any sampling procedure that produces inferences that consistently over estimate or content
under estimate some characteristics is said to be
(a) Simple random sampling (b) Systematic sampling
(c) Biased (d) None of these
13.56 In simple random sampling each unit of the population has _____ chance of selection:
(a) equal (b) Unequal
(c) maximum (d) None of these
13.57 _____ is a group of items or observations taken from the population for examination or
experiment.
(a) Sampling frame (b) Sampling distribution
(c)Sample (d)Units
13.58 Which of the following statements is true?
1. Sample is a group of items or observations taken from the population for
examination or
experiment.
2. Sampling frame is a list of all the members of the population used as a basis for
sampling.
3. Cost saving is one of the advantages of sampling
4. Time saving is not an advantage of sampling
13.59 _____ sampling relies on arranging the target population according to some ordering
scheme and then selecting elements at regular intervals through that ordered list.
(a) Systematic (b) Random
(c) Stratified (d) Cluster
13.60 A statistic is used as the basis of an estimate of a ____.
(a) Sample (b) Population
(c) Parameter (d)Mean
13.61 sampling distribution of the mean is a distribution made up of the means of many____.
(a) population (b) Samples
(c) Standard deviations (d) standard errors
13.62 Compute standard deviation of the population from following data.
1. Standard deviation of the population: 500
2. Sample size: 100
27 | P a g e
(a)50 (b)10
(c)500 (d)2
13.63 Compute standard deviation of the population from following data:
1. Standard error of the mean: 50
2. Sample size: 100
(a)50 (b)500
(c)100 (d)5
13.64 A population consists of five numbers. How many samples of size two can be drawn from
this population with replacement?
(a)25 (b)2
(c)10 (d)20
13.65 By how much will standard error change if sample size increases from 100 to 121
(a) Decrease by 9.09% (b) Increase by 21%
(c) Cannot be determined with available data (d)Increase by 9.09%
13.66 A machine produces certain component, sample of 40 such components was found to
have mean of 380 and standard deviation of 100. Calculate 99% confidence limits for the overall
production.
(a) 339.52 to 420.48 (b)380 to 420
(c) 334.6 to 415.7 (d)333.25 to 416.2
13.67 Confidence ____ are the values at each end of a confidence interval.
(a) Limits (b)Interval
(c)Level (d)region
13.68 _____is obtained if both confidence limits are added together and divided by two
(a) Standard error (b) Standard deviation
(c) Mean (d) Median
13.69 if the sample size increases, what will happen to standard error
(a) Increases (b)Decreases
(c) Remains unchanged (d)Cannot be determined with limited data
13.70 Compute confidence interval for the following data
1. Confidence level: 95%
2. Standard error: 0.8
3. Sample mean: 5
(a) 3.3 to 4.3 (b)3.43 to 6.57
(c) 3.5 to 7.5 (d)3.2 to 6.2
13.71 Compute confidence interval for the following data:
1. Confidence level: 95%
2. Standard error: 0.8
3. Sample mean: thrice of standard error
(a) 0.832 to 3.968 (b)0.5 to 3.5

28 | P a g e
(c) 1.832 to 2.832 (d)1.832 to 4.832
13.72 Compute confidence interval for the following data:
1. Confidence level: 95%
2. Standard deviation of population: 15
3. Sample size: 9
4. Sample mean: 15
(a) 5.2 to 24.8 (b)4.2 to 25.8
(c) 5.3 to 24.8 (d) 5.3 to 24.9
13.73 What happens to confidence interval if confidence level is increased?
(a) Increases (b)Decreases
(c) Remains unchanged (c) Cannot be determined
13.74 What happens to confidence interval if confidence level is decreased?
(a) Increases (b)Decreases
(c)Remains unchanged (d) Cannot be determined
13.75 What will happen to standard error of mean if sample size is decreased?
(a) Increases (b) Decreases
(c) Remains unchanged (d) Cannot be determined
13.76 What will happen to standard error of mean if sample size is increased?
(a) Increases (b) Decreases
(c) Remains unchanged (d) Cannot be determined

29 | P a g e
1/1/2022 HYPOTHESIS
TESTING
CHAPTER 14

Abdul Ahad Butt


What is Hypothesis Testing?
Significance testing is a term given to any test devised to check if a difference between a sample
characteristic and that of the population are only due to chance or not. If they are not solely due to
chance, they are said to be statistically significant. This means that they signify a difference between the
sample and population

Method of Hypothesis Tests:

Statisticians follow a formal process to determine whether to reject a null hypothesis, based on
sample data. This process, called hypothesis testing, consists of four steps.

 State the hypotheses. This involves stating the null and alternative hypotheses. The
hypotheses are stated in such a way that they are mutually exclusive. That is, if one is
true, the other must be false.
 Formulate an analysis plan. The analysis plan describes how to use sample data to
evaluate the null hypothesis. The evaluation often focuses around a single test statistic.
 Analyze sample data. Find the value of the test statistic (mean score, proportion, t
statistic, z-score, etc.) described in the analysis plan.
 Interpret results. Apply the decision rule described in the analysis plan. If the value of the
test statistic is unlikely, based on the null hypothesis, reject the null hypothesis

Steps to construct a hypothesis

1. Setting up two competing hypotheses - Each hypothesis test includes two hypotheses about
the population. One is the null hypothesis, notated as Ho, which is a statement of a particular
parameter value. This hypothesis is assumed to be true until there is evidence to suggest
otherwise. The second hypothesis is called the alternative, or research, hypothesis, notated as
Ha. The alternative hypothesis is a statement of a range of alternative values in which the
parameter may fall. One must also check that any assumptions (conditions) needed to run the
test have been satisfied e.g. normality of data, independence, and number of success and failure
outcomes.

2. Set some level of significance called alpha. This value is used as a probability cutoff for
making decisions about the null hypothesis. As we will learn later, this alpha value represents
the probability we are willing to place on our test for making an incorrect decision in regards to
rejecting the null hypothesis. The most common alpha value is 0.05 or 5%. Other popular
choices are 0.01 (1%) and 0.1 (10%).

1|Page
3. Calculate a test statistic. Gather sample data and calculate a test statistic where the sample
statistic is compared to the parameter value. The test statistic is calculated under the assumption
the null hypothesis is true, and incorporates a measure of standard error and assumptions
(conditions) related to the sampling distribution. Such assumptions could be normality of data,
independence, and number of success and failure outcomes.

4. Calculate probability value (p-value), or find rejection region - A p-value is found by


using the test statistic to calculate the probability of the sample data producing such a test
statistic or one more extreme. The rejection region is found by using alpha to find a critical
value; the rejection region is the area that is more extreme than the critical value.

5. Make a test decision about the null hypothesis - In this step we decide to either reject the
null hypothesis or decide to fail to reject the null hypothesis. Notice we do not make a decision
where we will accept the null hypothesis.

6. State an overall conclusion - Once we have found the p-value or rejection region, and made a
statistical decision about the null hypothesis (i.e. we will reject the null or fail to reject the
null). Following this decision, we want to summarize our results into an overall conclusion for
our test.

Test yourself:
Q.1 What is Hypothesis Testing?
Ans. It is a procedure which enables us to decide on the basis of information obtained from
sample data whether to accept or reject any specified statement or hypothesis or assumption
about the value of population parameter.
Q.2 What is Null Hypothesis?
Ans. Any hypothesis which is tested for possible rejection under the assumption that it is true is
called null hypothesis. A null hypothesis is generally denoted by the symbol Ho.
Q.3 What is Alternative Hypothesis?
Ans. Any hypothesis which is different from the null hypothesis is called alternative hypothesis
generally an alternative hypothesis is denoted by the symbol H1. It is accepted when null
hypothesis is rejected.
Q.4 What is Type-I Error?
Ans. We may reject a null hypothesis when it is actually true, it is called type-I error.
Q.5 What is type-II error?
Ans. We may accept a null hypothesis when it is actually false, it is called type-II error.
Q.6 What is level of significance?

2|Page
Ans. It is the probability used as a standard for rejecting a null hypothesis when a null hypothesis
is assumed to be true. This probability is equal to some small pre-assigned value α. The
most frequently used values of α are 5% and 10%. Note that α = 5% mean that we are 95%
confident in making the correct decision.

TESTING OF HYPOTHESIS
Testing of hypothesis about mean when σ2 is known.
Step-1
We formulate our hypothesis as
Ho = µ = µ0 (two Tailed test)
H1 = µ ≠ µ0
OR
H0 : µ ≤ µ0
H1 : µ > µ0 (One Tailed test)
OR
H0 : µ ≥ µ0 (One Tailed test)
H1 : µ < µ0
Step-2
Decide level of significance α = 5%
Step-3
Test – statistics
̅
X − μ0
Z= α
√n
Step-4 Calculation
Calculate the value of test statistics from sample data.
Step-5 Critical Regions:
Alternative Hypothesis Critical Region
H1 = µ ≠ µ0 │Z│ ≥ Zα/2
H1 = µ > µ0 Z > Zα
H1= µ < µ0 Z < ‒Zα
Step-6 Conclusion
(i) If value of test statistics falls in the critical region then reject Ho
(ii) If the value of test statistics falls in the acceptance region then accept Ho
Testing of hypothesis about mean when σ2 is unknown is > 30
Test statistics

3|Page
̅ − μo
X
Z=
S
√n

POINTS TO REMEMBER:

(i) A numerical value assigned to the unknown population parameter is called an estimate
(ii) The statistics(sample) is referred to as estimator of the unknown parameter
(iii) Hypothesis testing and estimation are both done in inferential statistics
(iv) A test of hypothesis is always about a population parameter
(v) The equality sign always appears in Ho
(vi) A two-tail test is a test with two critical regions
(vii) If the null hypothesis and alternative hypothesis are in the equal and not equal form it
is appropriate to use a two tailed test
(viii) A misfit person is selected for a job is called type - I error or rejection of the null
hypothesis when it is true is called type- I error
(ix) Acceptance of null hypothesis when it is false is called type-II error
(x) Any assumption about one or more population is called statistical hypothesis
(xi) The procedure of establishing a set of rules that lead to the acceptance or rejection of
some kind of statement about population parameters is called testing of hypothesis
(xii) The hypothesis for which we wish to gather supporting evidence is called alternative
hypothesis
(xiii) The area specified for the values which are significantly different from null hypothesis
value is called critical region
(xiv) If n is large i-e n>30, the variance of population is unknown, then the test statistics for
testing mean of population must be t-test
(xv) If the sample size is small (i-e n< 30) and variance of population mean, the test statistics
is to be selected is z-test
(xvi) While testing the hypothesis about population mean, if variance of population is known
only z –test is valid
(xvii) A single value used to estimate the unknown value of a population parameter is called
point estimate
(xviii) 𝜎 is a parameter and S is a statistic?

4|Page
Z –Test (greater than 30)

𝑋̅ − 𝜇0
Formula Z= ̂
𝑆
√𝑛
Question-1
The mean lifetime of electric light bulbs produced by a company has in the past been 1120 hours.
A random sample of 36 electric bulbs recently chosen from a supply of newly manufactured
bulbs showed a mean lifetime of 1087 hours with a standard deviation of 120 hours. Test the
hypothesis that the mean lifetime of light bulbs has not changed using a level of significance of
0.05.
Solution:
Step: 1 we formulate our hypotheses as:
𝐻0 : 𝜇 = 1120
𝐻1 : 𝜇 ≠ 1120
Step: 2 Level of significance 𝛼 = 0.05
Step: 3 Test statistic:
𝑋̅ − 𝜇0
Z= ̂
𝑆
√𝑛
Step: 4 Calculations:
Here
n = 36
𝑆̂ = 120
𝑋̅ = 1087
𝜇0 = 1120
1087−1120
Z= 120
√36
−33
= 20
= 1.65
Step: 5 Critical Region:
|Z| ≥ 1.96

Step: 6 Conclusion:
Since our calculated value falls in the acceptance region so we accept 𝐻0 .

5|Page
Question: 2
A company claims that the average amount of coffee it supplies in jars is 6.0 oz with a
standard deviation of 0.2 oz. A random of 100 jars is selected and average is found to b
5.9. Is the company cheating the customer?

Solution:
Step: 1 we formulate our hypotheses as:
𝐻0 : 𝜇 ≥ 6.0
𝐻1 : 𝜇 < 6.0
Step: 2 Level of significance 𝛼 = 0.05
Step: 3 Test statistics:
𝑋̅ − 𝜇0
Z= 𝜎
√𝑛

Step: 4 Calculations:
n = 100
𝜎 = 0.2
𝑋̅ = 5.9
𝜇0 = 6.0
5.9−6.0
Z= 0.2
√100
−0.1
=
0.02
= -5
Step: 5 Critical Regions:
Z < - 1.645
Step: 6 Conclusions:
Since our calculated value falls in the critical region so we reject 𝐻0.
Question: 3
Apex’s current packaging machinery for coffee is known to ground coffee into “1-pound cans”
with a standard deviation of 0.6 ounce. Apex is considering using a new packages machine
which is expected to pour coffee into “1-pound cans” more accurately with a standard deviation
of 0.3 ounce. Before deciding to invest in the new machine, apex wished to test its performance
against the old machine. A sample was taken on each machine to measure the mean weight of
contents of the “1-pound cans” with the following results.

Sample Size Mean


Using old machine 𝑛1 = 25 𝑋̅1 = 16.7 ounce
Using new machine 𝑛2 = 36 𝑋̅2 = 15.8 ounce

6|Page
Test at the 5% level of significance the hypotheses that there is no difference in the average
weight of the contents poured by the old machine versus the new machine.

Solution:

Step: 1 we formulate our hypotheses as:


𝐻0 : 𝜇1 - 𝜇2 = 0
𝐻1 : 𝜇1 - 𝜇2 ≠ 0
Step: 2 Level of significance 𝛼 = 0.05
Step: 3 Test statistics:
(𝑋̅1− 𝑋
̅ 2 )−(𝜇1 − 𝜇2 )
Z= 𝜎 2 𝜎 2
√ 1 + 2
𝑛1 𝑛2

Step: 4 Calculations:
Old machine new machine
𝑛1 = 25 𝑛2 = 35
𝑋̅ = 16.7 𝑋̅ = 15.8
𝜎1 = 0.6 𝜎2 = 0.3

(16.7−15.8)−0
Z= 2 2
√(0.6) + (0.3)
25 36

0.9
=
√0.0144+0.0025
0.9
= 0.13
= 6.92
Step: 5 Critical Regions:
|Z|≥ 1.96
Step: 6 Conclusions:
Since our calculated value falls in the critical region so we reject 𝐻0.

Question: 4
The test was given to a group of 100 scots and to a group of 144 guides. The mean score for the
scouts was 27.53 and the mean score for the guides was 26.81. Assuming a common population
standard deviation of 3.48 test, using a 5% level of significance whether the scout’s performance
in the test was better than that of the guides.

7|Page
Solution:
Step: 1 we formulate our hypotheses as:
𝐻0 : 𝜇1 - 𝜇2 ≤ 0
𝐻1 : 𝜇1 - 𝜇2 > 0
Step: 2 level of significance 𝛼 = 0.05.
Step: 3 Test statistics:
(𝑋̅1 − 𝑋̅2 )−(𝜇1 − 𝜇2)
Z= 1 1
𝜎√ +
𝑛1 𝑛2

Step: 4 Calculations:
Old machine new machine
𝑛1 = 100 𝑛2 = 144
𝑋̅1 = 27.53 𝑋̅2 = 26.81

𝜎 = 3.48

(27.53−26.81)−0
Z= 1 1
3.48 √ +
100 144
0.72
= 3.48
√0.01+0.0069
0.72
= 0.4524
= 1.59
Step: 5 Critical Regions:
Z > 1.645
Step: 6 Conclusions:
Since our calculated value falls in the acceptance region so we accept 𝐻0.

8|Page
T-test (less than 30) Always use level of significance in t-test

𝑋̅ − 𝜇𝑜
Formula t = ̂
𝑆
√𝑛

Question: 5
Workers at a production facility are required to assemble a certain part in 2.3 minutes in order to
meet production criteria. The assembly rate per part is assumed to be normally distributed. Six
workers are selected at random and timed in assembling a part. The assembly times (in minutes)
for the six workers as follows.
2.0, 2.4, 1.7, 1.9, 2.8, 1.8
The manager wants to determine if the mean for all workers differs from 2.3. perform a test of
hypotheses at the 5% level of significance.

Solution:
Step: 1 We formulate our hypotheses as:
𝐻0 : 𝜇 = 2.3
𝐻1 : 𝜇 ≠ 2.3
𝐒𝐭𝐞𝐩: 2 level of significance 𝛼 = 0.05.
𝐒𝐭𝐞𝐩: 𝟑 Test statistic:
𝑋̅ − 𝜇𝑜
t= ̂
𝑆
√𝑛

Step: 4 Calculations:
X 𝑋2 ∑𝑋
𝑋̅ = 𝑛
2.0 4.00 12.6
= = 2.1
2.4 5.76 6

∑𝑋 −𝑛𝑋̅ 2 2
1.7 2.89 𝑆̂ = √ 𝑛−1
1.9 3.61
27.34−6(2.1)2
2.8 7.84 =√ 6−1
1.8 3.24
27.34−26.46
12.6 27.34 =√ 5

= 0.42
Here
n=6
𝑆̂ = 0.42
𝑋̅ = 2.1

9|Page
2.1−2.3
t= 0.42
√6
0.2
= 0.17
= 1.17
Step: 5 Critical Region:
Degree of freedom v = n - 1= 6 – 1 =5
|t| ≥ 𝑡0.025,(5)
|t| ≥ 2.571

Step: 6 Conclusion:
Since our calculated value falls in the acceptance region so we accept 𝐻𝑜.
Question: 6
Injection of a certain type of hormone into hens is said to increase the mean weight of eggs by
0.3 ounces. A sample of 30 eggs has an arithmetic mean 0.4 ounces above the pre injection mean
and a value of 𝑆̂ equal to 0.20. IS this enough reason to accept the statement that the mean
increase is more than 0.3 ounces?
Solution:
Step: 1 we formulate our hypotheses as:
𝐻0 : 𝜇 ≤ 0.3
𝐻1 : 𝜇 > 0.3
Step: 2 level of significance 𝛼 = 0.05
Step: 3 Test statistics:
𝑋̅ − 𝜇𝑜
t= ̂
𝑆
√𝑛

Step: 4 Calculations:
n= 30
𝑆̂ = 0.20
̅ = 0.4
𝑿
𝜇0 = 0.3

0.4−0.3
t= 0.20
√30
0.1
= 0.04
= 2.5

Step: 5 Critical Regions:

10 | P a g e
d.f = n – 1= 30 – 1 =29
t > 𝑡0.05,(29)
t > 1.699
Step: 6 Conclusions:
Since our calculated value falls in the critical region so we reject 𝐻0.
Check your self
Question:1
The average distance spent by each person commuting to the office each month in a large city is
asserted to be 460 km. A sample of 100 commuters showed a mean of 450 km and had a
standard deviation of 25 km. Does the sample support the assertion about the population at the
5% significance level?
Question:2
The management of a company maintained that the average time taken to make a component was
50 minutes. A sample of 100 components showed that the mean time taken to make each was 52
minutes with a standard of 15 minutes. Does the sample support the assertion about the
population at the 95% confidence level?
Question:3
A transport manager asserts that the mean journey time is 14 hours a day. A random sample of
64 journeys showed a mean of 13 hours and 20 minutes with a standard deviation of 3 hours.
Test this assertion at the 5% significance test.
Question:4(one tail)
A cable manufacturer makes cable with breaking strength of 700 kg with a standard deviation of
13 kg. The manufacturer has invented a different technique that it claims results in stronger
cables. A random sample of 40 cables has a mean strength of 725 kg.
Question:5 (one -Tail)
Regular sugar tests for women and men in a city are provided in the table below
Gender Men Women
Sample 1600 1900
Sugar level on average 130 125
Standard deviation of 17.5 20
sample
Can we statistically verify that sugar level for women is lower than men?
Question:6
Height of men in two cities are provided in the table below
City Karachi Lahore
Sample 160 196
Height 170 cm 171 cm
Standard deviation 8 cm 7 cm

11 | P a g e
Testing of population proportion
Question: 7
The records of a certain hospital showed the birth of 723 males and 617 females in a certain
week. Do these figures conform to the hypotheses that the sexes are born in equal proportions.
Use 𝛼 = 0.02.
Solution:
Step: 1 we formulate our hypotheses as:
𝐻0 : 𝑃0 = 0.50 (Population proportion)
𝐻1 : 𝑃0 ≠ 0.50
Step: 2 Level of significance 𝛼 = 0.02.
Step: 3 Test statistics:
𝑃− 𝑃0
Z= 𝑃 (1− 𝑃0 )
√ 0
𝑛

Step: 4 Calculations:
n = 723 + 617
= 1340
X = 723(Male) Or : you can take X = 617
𝑋
P = 𝑛 (sample proportion)
723
= 1340
= 0.54
𝜋0 = 0.50
0.54−0.50
Z= (1−0.50)
√0.50
1340
0.04
= 0.0137 = 2.92
Step: 5 Critical Regions:
|Z|≥ 2.326
Step: 6 Conclusions:
Since our calculated value falls in the critical region so we reject 𝐻0.

Note: Always use population Proportion value in formula if not given use sample proportion
formula

Question: 8
A basket ball player has hit on 60% of his shots from the floor. If on the next 100 shots he makes
70 baskets would you say that his shooting has improved? (Use a 0.05 level of significance)

12 | P a g e
Solution:
Step: 1 we formulate our hypotheses as:
𝐻0 : P0 ≤ 0.60
𝐻1 : P0 > 0.60
Step: 2 level of significance 𝛼 = 0.05
Step: 3 Test statistic:
𝑃− 𝑃0
Z= 𝑃 (1− 𝑃0 )
√ 0
𝑛

Step: 4 Calculations:
n = 100
X = 70
𝑋
P=𝑛
70
= 100
= 0.70
𝑃0 = 0.60
0.70−0.60
Z= (1−0.60)
√0.60
100
0.10
= 0.0490
= 2.04
Step: 5 Critical Region:
Z < - 𝑍 0.05
Z < -1.645
Step: 6 Conclusion:
Since our calculated value falls in the critical region so we reject 𝐻0.

Question: 9
A cigarette manufacturing firm distributes two brands of cigarettes. It is found that 56 of 200
smokers prefer brand “A” and that 29 of 150 smokers prefer brand B. Test at 0.06 level of
significance that brand “A” outsell brand “B” by at least 10% against the alternative hypotheses
that the difference is less than 10%.
Solution:
Step: 1 We formulate our hypotheses as:
𝐻0 : 𝑃01 - 𝑃02 ≥ 0.10
𝐻2 : 𝑃01 - 𝑃02 < 0.10
Step: 2 level of significance 𝛼 = 0.06
Step: 3 Test statistic:

13 | P a g e
(𝑃1 − 𝑃2 )− (𝑃01 − 𝑃02 )
Z= 𝑃1 𝑞1 𝑃 𝑞
√ 𝑛 + 2 2
1 𝑛2

Step: 4 Conclusions:
Brand A Brand B
𝑛1 = 200 𝑛2 = 150
𝑋1 = 56 𝑋2 = 29
𝑋 𝑋
𝑃1 = 𝑛1 𝑃2 = 𝑛2
1 2
56 29
= 200 = 150
= 0.28 = 0.19

(0.28−0.19)−0.10
Z= 0.28 (1−0.28) 0.19 (1−0.19)
√ +
200 150
0.09−0.10
=
√0.0010+0.0010
−0.01
= 0.0447
= -0.22
Step: 5 Critical Region:
Z < -𝑍𝛼
Z < -𝑍0.06
Z < -1.555
Step: 6 Conclusions:
Since our calculated value falls in the acceptance region so we accept 𝐻0 .

Question: 10
A firm found with the help of a sample survey (size of sample 900) that 0.75 of the population
consumes things produced by them. The firm then advertised the goods in paper and on radio.
After one-year sample of size 1000 reveals that proportion of consumers of the goods produced
by the firm is now 0.8. is this rise significant indicating that the advertisement was effective?

Solution:
Step: 1 we formulate our hypotheses as:
𝐻0 : 𝑃02 - 𝑃01 ≤ 0
𝐻2 : 𝑃02 - 𝑃01 > 0
Step: 2 level of significance 𝛼 = 0.05
Step: 3 Test statistic:

14 | P a g e
(𝑃2 − 𝑃1 )−(𝑃02 − 𝑃01 )
Z= 𝑃1𝑞1 𝑃2𝑞2
√( 𝑛 + 𝑛 )
1 2

Step: 4 Calculations:
Sample I Sample II
𝑛1 = 900 𝑛2 = 1000
𝑃1 = 0.75 𝑃2 = 0.80

(0.80−0.75 )−0
Z= 0.75 𝑥 0.25 0.80 𝑥 0.20
√( + )
900 1000

= 2.60
Step: 5 Critical Region:
Z > 𝑍0.05
Z > 1.645
Step: 6 Conclusion:
Since our calculated value falls in the critical region so we reject 𝐻0.
Question:1
An election candidate claims that 60% of the voters support him. A random sample of 2500
voters showed that 1410 supported him (0.564 or 56.4%) Test the candidates claims 95%
confidence level?
Question:2 one tail)
An election candidate claims that at least 60% of the voters support him. A random sample of
2500 voters showed that 1410 supported him (0.564 or 56.4%). Test the candidates claims 95%
confidence level?
Question:3 (one tail)
A health official claims that citizens of Peshawar are fitter than citizen of Gujrat. 96 out of 200
(48%) citizens in Peshawar (selected at random) passed a standard fitness test. 84 out of 200
(42%) citizens in Gujrat (selected at random) passed the same test. Test the officials at the 95%
confidence level.
Question:4
A sample of 300 smokers and 70 non-smokers were taken for a study of likelihood of getting
cancer. Following table summarizes findings
Cancer Non- cancer Total
Smoker 270 30 300
Nonsmoker 8 62 70
Total 278 92 370

15 | P a g e
t-test
Question:5
An investigator suspects that the mean IQ level for a certain type of test is 110. He designs an
experiment where 5 volunteers each took the test and achieve the following scores. Do the scores
support his suspicion at the 95% confidence level?
Test A
Volunteer 1 135
Volunteer 2 103
Volunteer 3 129
Volunteer 4 96
Volunteer 5 121

Question:6 one tail


An investigator suspects that there is a difference between two ways of assessing IQ. He designs
an experiment where 5 volunteers each take both tests and achieve the following scores. Do the
scores support his suspicion at the 95% confidence level?
Test A Test B
Volunteer 1 135 125
Volunteer 2 103 102
Volunteer 3 129 117
Volunteer 4 96 94
Volunteer 5 121 121

16 | P a g e
Extra practice questions
Question:1
Heights in inches of ten individuals chosen at random from a normal population are as follows
63,63,66,67,68,69,70,70,71,71
In the light of above data discuss the suggestion that the mean height of the population at
significance level of α =0.005 is 66 inches.
Question:2
A television manufacturer claims that the mean life of the picture tubes of its classic TV brand is
8000 hours. A random sample of 18 picture tubes showed a mean life of 7850 hours with
standard deviation of 150 hours. Test the manufacture’s claim using a significance level of 0.01
Question:3
A researcher wants to test on the basis of a random sample of size n=7 whether the fat contents
of a certain kind of processed food does not exceed 30% what would he conclude at the 0.01
level of significance. If the sample values (in percentage) are 31.5 ,30.3,31.1,30.7,29.9,29.6 and
31.87?
Ho : µ ≤ 30% and H1 : µ >30%
Question:4(2011)
A manufacturer claims that after using an automatic bottling plant, the average quantity in each
bottle produced on its plant is 250ml. A random sample of 25 bottles showed a man quantity of
242 ml with standard deviation of 18 ml. Test the manufacturer’s claim. using a significance
level of 0.05
Question:5(2010)
Tara electronics claims that its energy saver bulbs have an average life of 6500 hours. A
consumer rights protection agency tested 15 such bulbs to check this claim. It found that the
mean life of 15 bulbs was 6300 hours with a standard deviation of 200 hours. At the 5%
significance level. Asses the claim of Tara Electronics. Assume that life of such bulbs has an
approximately normal distribution
Question:6(2008)
A random sample of size 16 has a mean of 53. The sum of square of deviations of values from
the mean is 150. Assuming the population values are approximately normally distributed. Test
the hypothesis that population mean is 56 with level of significance is 0.05
Question:7(2007)
When properly adjusted an automatic machine should produce parts that have a mean diameter
of 25 millimeters. Part diameters are normally distributed. the mean diameter of a sample of 10

17 | P a g e
parts is 25.02 mm with sample standard deviation =0.024. perform a hypothesis test at 5% level
of significance to evaluate whether the machine is working properly.
Question:8(1996)
An employment exchange office claims that the applications received in his office are processed
in an average of at most 5 working days. A sample of 8 applications are randomly taken and
these were processed in 7,6,7,9,3,3,8,5 days. Assuming that the processing time is normally
distributed. Test the validity of the officers claim at 0.01 level of significance
Hint: Ho : µ ≤ 5 and H1 : µ >5

(Z-TEST)
Question:1
A cigarette manufacturer claims that the amount of nicotine in each cigarette is 7.5mg. A random
sample of 40 cigarettes was tested and found to have a mean nicotine content of 7.67 mg with a
standard deviation of 0.6 mg. test the manufacturer’s claim at 5% level of significance
Ho : µ ≤ 5 and H 1 : µ >5

Question:2
A manufacturer claims that average life of a certain type of product is 270 hours with a standard
deviation of 25 hours. A sample of 12 units when tested showed a mean life of 255 hours. Using
a significant level of 0.05. evaluate the manufacturer’s claim
Question:3
It is claimed that by using a automatic filling machine. The average amount of chips in each bag
is 100 grams. A random sample of 36 bags of chips showed a mean weight of 101.25 grams with
a standard deviation of 2.5 grams. Is the machine operating properly at 0.05 level of significance
Question:4
A firm believes that the mean cost to process a sales order is Rs 132.50. the cost controller fears
that the average cost of processing is more than that. A random sample of 100 orders has a
sample mean of Rs 133.5 and standard deviation of Rs. 5 level of significance is 0.05
Required:
(i) State the null and alternative hypothesis
(ii) Which test distribution (z or t) will be used and why?
(iii) Whether the Ho will be rejected or not, give reason for your conclusion

18 | P a g e
Question:5
I.Q of students at a college are normally distributed with a mean=112 and a standard
deviation=12 professor Hameed believes that I.Q of students in his Statistics class do not have a
mean IQ of 112 to prove this point. A random sample of 5 students from statistics class is
chosen. Their IQ scores were found to be 115,124,131,129, and 120. Test the hypothesis that the
means IQ of the students of the class is the same as that of the population of students at the
college with level of significance is 0.05
Question:6(proportion)
Investigating the success of its interviewers. A firm finds that176 out of 225 interviewers
attempted by trained interviewers are successfully completed. Determine whether these data
provided sufficient evidence at the 5% level of significance to indicate a relationship between the
training status of interviewers and the outcome of interview attempted.
Question:7
A random sample of size 400 is drawn from a normal population. The sample means and
standard deviation are 5.2 and 3.7respectively
(i) Estimate standard error of mean
(ii) Test at 5% level of significance that population mean is not more than 5

Ho : µ ≤ 5 and H 1 : µ >5

Question:8
A machine is set to fill cereal boxes with a mean weight of 500 grams of cereal per box. The
standard deviation is known to be 125 grams. A random sample of 25 filled boxes is taken and
the mean weight of cereal per box is computed as 540 grams is there reason t believe that
machine is overfilling the boxes thereby increasing the costs. Used 0.05 level of significance
Ho : µ ≤ 5 and H 1 : µ >5

Question:9
A manufacturer claims that the average life of a certain type of a product is at least 150 hours. It
is known that the standard deviation of this type of product is 20 hours. A consumer wishes to
test the manufacturer’s claim and accordingly tests 16 products. The sample mean is 144 hours.
Perform a test to reject the manufacturer’s claim at 5% level of significance
Question:10
A manufacturer of bolts claims that the means length is 4500 inches with a standard deviation of
0.020 inches. A random sample of 16 bolts yields a mean of 4512 inches. Do these data provide
sufficient evidence to indicate that the true mean length is greater than manufacturer claim?
Assume that the dimensions are normally distributed. Use a 0.01 level of significance

19 | P a g e
HYPOTHESIS TESTING
14.1 The simplest form of inferential statistics, which uses known sample evidence (statistic)
to draw conclusions regarding unknown population characteristics (parameter), is known
as:
Test of hypothesis Estimation
Inferential statistics None of these
14.2 A numerical value assigned to the unknown population parameter is called:
Statistic Parameter
An estimate None of these
14.3 The statistic is referred to as________ of the unknown parameter:
Estimator Characteristic
Unbiased None of these
14.4 If the expected value of the estimator is equal to parameter being estimated, the estimator
is said to be:
Random Efficient
Unbiased None of these
14.5 if an estimator achieves improved reliability and precession as the sample size becomes
larger than the estimator is said to be:
Efficient Sufficient
Consistent None of these
14.6 Of all possible unbiased estimators of some parameter the one with the smallest variance
is said to be:
Most efficient More consistent
More random None of these
14.7 A single numerical quantity used to estimate the population parameter is called:
Point estimate Interval estimate
Unbiased estimate None of these
14.8 If it is desirable to determine an interval within which one would expect to find the value
of a parameter, such an interval is called:
Point estimate Interval estimate
Unbiased estimate None of these
14.9 The value obtained by subtracting the number of parameters to be estimated from the
number of independent values in a sample is called:
Type-I error Type-II error
Degrees of freedom None of these
14.10 The procedure of establishing a set of rules that lead to the acceptance or rejection of
some kind of statements about population parameters is called:
Statistical hypothesis Null hypothesis

20 | P a g e
Testing of hypothesis None of these
14.11 Any assumption or conjecture about one or more population is called:
Statistical hypothesis Null hypothesis
Composite of hypothesis None of these
14.12 The hypothesis against which we hope to gather evidence is called:
Statistical hypothesis Null hypothesis
Composite of hypothesis None of these
14.13 The hypothesis for which we wish to gather supporting evidence is called:
Null hypothesis Alternate hypothesis
Composite hypothesis None of these
14.14 All hypotheses testing results are important, but the test is statistically significant only
when the hypothesis is:
Accepted Rejected
Unbiased None of these
14.15 Rejection of the null hypothesis when it is true is called:
Type-I error Type-II error
Level of confidence None of these
14.16 The probability of committing a type-I error is called:
Type-II error Level of significance
Level of confidence None of these
14.17 Acceptance of null hypothesis when it is false is called a:
Type-I error Type-II error
Level of confidence None of these
14.18 The very first statement which is tested for possible rejection is called:
Null hypothesis Alternate hypothesis
Composite hypothesis None of these
14.19 To prove that one teaching method is superior to another, the null hypothesis would be;
that there is no difference in the two methods. Is the stated hypothesis is:
Correct Incorrect
Does not exist None of these
14.20 The procedure by which we verify an established hypothesis is known as:
Test of significance Test –statistic
One –tailed test None
14.21 A function obtained from sample data which provides the means of testing a statistical
hypothesis, is called:
Test of significance Test –statistic
One –tailed test None of these

21 | P a g e
14.22 The area specified for the values which are significantly different from null hypothesis
value is called:
Acceptance region Critical region
Level of significance None of these
14.23 If the alternate hypothesis does not specify the departure from 𝐻0 in a particular
direction, then the test is called:
Significant One –tailed
Two tailed None of these
14.24 If the alternate hypothesis specifies the departure from 𝐻0 in a particular direction, then
the test is called:
Not significant One-tailed
Two tailed None of these
14.25 given𝐻0 : student A and B are equal in ability. The given hypothesis was rejected and the
conclusion was drawn that student A is more capable than student B.
In the coming years it was found that the conclusion drawn was incorrect and there was
no difference in their capabilities. The type of error was committed by the statistician
was:
Type- I error Type –II error
Error in drawing conclusion None of these
14.26 If n is large i.e. n>30, the variance of population is unknown, and then the test-statistic
for testing mean of population must be:
t-test z- test
2
𝑋 test None of these
14.27 IF the sample size is small (i.e. <30) and variance of population is known, then to test the
population mean, the test statistic to be selected is:
z-test t-test
2
𝑋 test None of these
14.28 While testing the hypothesis about population mean, t-test is only selected if:
N < 30 𝜎 2 is unknown
If (a) and (b) both are valid None of these
14.29 While testing the hypothesis about population mean, if variance of population is known,
then:
Only z-test is valid Only t-test is valid
Either can be selected None of these
14.30 While testing the hypothesis about population mean, if variance of population is
unknown then the selection of test-statistic depends upon:
Size of population Size of sample
The value of X an 𝜇 None of these

22 | P a g e
14.31 While testing the hypothesis about population mean, if size of sample is large i.e.n>30,
then the selection of test-statistic depend upon:
Variance of population Variance of sample
Does not depend upon anything None of these
14.32 The sum of squares of a sequence of independent normal varieties with mean 𝜇 and
variance 𝜎 2 is said to be:
A chi-square variate A standard normal variate
A normal variate None of these
1. A tyre manufacture claim that the tyres produced by them have a mean life of 25,000 km.
Construct appropriate null and alternate hypothesis.

(a) Ho: μ ≥25,000 (b) Ho: μ ≥ 25,000


Ho: μ< 25,000 H₁: μ = 25,000
(b) Ho: μ ≥ 25,000 (c) Ho: μ< 25,000
H₁: μ > 25,000 H₁: μ > 25,000
2. A school claims that their students have a passing ratio of 75%. Construct appropriate null
and alternate hypothesis.
(a) Ho: μ ≥ 75% (b) Ho: μ ≥ 75%
H₁: μ< 75% Ho: μ =75%
(c)Ho: μ ≥ 75% (d) H₁: μ < 75%
H₁: μ >75% H₁: μ <75%
3. A jeans manufacturer claims that only 2% of its production has defects. Construct
appropriate null and alternate hypothesis.
(a) Ho: μ ≥ 2% (b) Ho: μ ≤ 2%
H₁: μ< 2% H1: μ > 2%
(c) Ho: μ = 2% Ho: μ ≥ 2%
H₁: μ ≠ 2% H₁: μ > 2%
4. Which of the following cannot be a possible value of significance level?

(a)1% (b)95%
(c)5% (d)101%
5. Which of the following cannot be a possible value of confidence level?
(a)1% (b)99%
(c)95% (d)105%
6. What is the sum of confidence level and significance level percentage?
(a)100% (b)105%
(c) Not a fixed value (d)Can take any value
7. If confidence level percentage is given as 95% what will be the value of significance level?
(a)5% (b)95%
(c)5% (d)0%
8. Identify the correct critical values for z test from following if significance level is 5% fora one
tail test.
(a) ±1.65 (b) ±2.33
(c) ±1.96 (d) ± 2.58

23 | P a g e
9. Identify the correct critical values for z test from following if significance level is 5% for a
two tail test.
(a) ±1.65 (b) ±2.33
(c) ±1.96 (d) ± 2.58
10. Identify the correct critical values for z test from following if significance level is 1% for a
two tail test.
(a) ±1.65 (b) ±2.33
(c) ±1.96 (d) ± 2.58
11. Identify the correct critical values for z test from following if significance level is 1% for a
one tail test.
(a) ±1.65 (b) ±2.33
(c) ±1.96 (d) ± 2.58
12. Perform hypothesis testing on following data and conclude whether to accept or reject null
hypothesis:
1.Claim: μ = 8
2.Population standard deviation = 0.2
3. Sample size = 49
4. Sample mean = 7.95
5. Significance level = 5%
(a) Accept (b) Reject
(c) Cannot be determined as confidence level (d) Cannot be determined as sample standard
is not given deviation is not given
13. Perform hypothesis testing on following data and conclude whether to accept or reject null
hypothesis:
1.Claim: μ = 8
2.Population standard deviation = 0.2
3. Sample size = 49
4. Sample mean = 7.95
5. Significance level = 1%
(a)Accept (b)Reject
(c) Cannot be determined as confidence level (d) Cannot be determined as sample standard
is not given deviation is not given
14. Perform hypothesis testing on following data and conclude whether to accept or reject null
hypothesis:
1.Claim: μ ≤ 8
2.Population standard deviation = 0.2
3. Sample size = 49
4. Sample mean = 7.95
5. Significance level = 1%
(a)Accept (b)Reject
(c) Cannot be determined as confidence level (d) Cannot be determined as sample standard
is not given deviation is not given
15. Perform hypothesis testing on following data and conclude whether to accept or reject null
hypothesis:
1.Claim: μ ≥8

24 | P a g e
2.Population standard deviation = 0.2
3. Sample size = 49
4. Sample mean = 7.95
5. Significance level = 1%
(a)Accept (b)Reject
(c) Cannot be determined as confidence level (d) Cannot be determined as sample
is not given standard deviation is not given
16. Perform hypothesis testing on following data and conclude whether to accept or reject null
hypothesis:
1. Claim: μ = 8
2.Population standard deviation = 0.2
3. Sample size = 49
4. Sample mean = 7.95
5. Significance level = 1%
(a)Accept (b)Reject
(c) Cannot be determined as confidence level (d) Cannot be determined as sample
is not given standard deviation is not given
17. If the sample size is 12, what will be the value of degrees of freedom in case of t-
distribution.
(a) 11 (b)12
(c)13 (d)10
18. What is the correct order of following steps for testing a hypothesis
1.Establish the hypothesis and its alternative.
2. Identify an appropriate sampling distribution.
3. Set the critical values.
4. Compute the test statistic.
5. Compare the test statistic with critical value and accept or reject the hypothesis
(a) 1, 2, 3, 4 and 5 (b) 1, 4, 2,3 and 5
(c) 5, 4, 3, 2 and 1 (d) 5, 3, 2, 1 and 4
19. The hypothesis which we want to test is called the ___ hypothesis and is denoted by H0
(a)Null (b) Alternate
(c)Significant (d)Confidence
20. In which of the following cases is the level of significance divided into two tails equally
(a) A manufacture claims that his products (b) A milk bottling plant is expected to fill
last for more than 24 hours 250ml of liquid in one bottle
(c) A manufacture claims that defects in its (d)A school claims that their passing
production are less than 5% ratio is 75% or more

25 | P a g e
1/1/2022 CHI-SQUARE
CHAPTER 15

Abdul Ahad
What is the chi-square test for? (one tail test)

The chi square test is used to determine whether there is a significant difference between observed
frequencies of results and expected frequencies of result.

• It is also called a "goodness of fit" statistic, because it measures how well the observed
distribution of data fits with the distribution that is expected if the variables are
independent.
• It is a test statistic for measuring independence.
• It asks whether the number of items in each category differs significantly from the
number of expected and whether any difference between the expected and observed is
due to chance or is it a real difference

Steps to calculate chi square:

State the Hypotheses

Suppose that Variable A has r levels, and Variable B has c levels. The null hypothesis states that
knowing the level of Variable A does not help you predict the level of Variable B. That is, the
variables are independent.

Ho: Variable A and Variable B are independent.

Ha: Variable A and Variable B are not independent.

The alternative hypothesis is that knowing the level of Variable A can help you predict the level
of Variable B.

Note: Support for the alternative hypothesis suggests that the variables are related; but the
relationship is not necessarily causal, in the sense that one variable "causes" the other.

Formulate an Analysis Plan

The analysis plan describes how to use sample data to accept or reject the null hypothesis. The
plan should specify the following elements.

▪ Significance level. Often, researchers choose significance levels equal to 0.01, 0.05, or
0.10; but any value between 0 and 1 can be used.
▪ Test method. Use the chi-square test for independence to determine whether there is a
significant relationship between two categorical variables.

1|Page
Analyze Sample Data

Using sample data, find the degrees of freedom, expected frequencies, test statistic, and the P-
value associated with the test statistic. The approach described in this section is illustrated in the
sample problem at the end of this lesson.

▪ Degrees of freedom. The degrees of freedom (DF) is equal to:

d.f = (r - 1) * (c - 1)

where r is the number of levels for one catagorical variable, and c is the number of levels
for the other categorical variable.

▪ Expected frequencies. The expected frequency counts are computed separately for each
level of one categorical variable at each level of the other categorical variable. Compute r
* c expected frequencies, according to the following formula.

Er,c = (nr * nc) / n

▪ where Er,c is the expected frequency count for level r of Variable A and level c of
Variable B, nr is the total number of sample observations at level r of Variable A, nc is the
total number of sample observations at level c of Variable B, and n is the total sample
size.
▪ Test statistic. The test statistic is a chi-square random variable (Χ2) defined by the
following equation.

Χ2 = Σ [ (O - E)2 / E ]

Interpret Results

If the sample findings are unlikely, given the null hypothesis, the researcher rejects the null
hypothesis. Typically, this involves comparing the P-value to the significance level, and rejecting
the null hypothesis when the P-value is less than the significance level.

2|Page
Points to Remember:

(i) Critical region is a region of rejection


(ii) The hypothesis which is to be test for possible rejection is null hypothesis
(iii) For a two tailed test of hypothesis at α =0.10 acceptance region is the entire region
between the two critical values
(iv) Hypothesis testing and estimation are both types of inferential statistics
(v) A test of hypothesis is always about population parameter
(vi) The equality signs always appear in Ho
(vii) A two tailed test is a test with two critical regions
(viii) If you wish to test the independence of two attributes, we will perform chi-square test
(ix) For goodness of fit test, we use chi-square distribution
(x) The range of chi-square distribution is 0 to ∞
(xi) The shape of chi –square distribution is +ve skewed
(xii) Chi-square distribution is a continuous distribution
(xiii) The sum of square of independence standard normal variable is chi-square variable
(xiv) Chi-square distribution is used to test the hypothesis concerning the independence of
two attributes and good ness of fit
(xv) Chi-square distribution is used to test population variance
(xvi) A table containing the data classified according to characteristics of population (a
attributes) is known as contingency table
(xvii) A technique by mean of which we test the hypothesis whether the sample distribution
is in agreement with the theoretical distribution is called goodness of fit
(xviii) A good of fit test can be applied by using chi-square distribution

3|Page
Goodness of fit
A test for comparing a theoretical distribution with the observed data from a sample data
Formula:
(O – E)2
χ 2=∑ E

Question-1 Goodness of fit


A car retailer sells a model of car with a choice of 5 colours. He expects demand this year to
follow demand last year in terms of choice of colours? The first 150 cars sold this year do not
seem to have followed this pattern and he is worried that tastes have changed.
Expected sales
(based on last year's
Actual sales (0) pattern)
Yellow 35 30
Red 50 45
Green 30 15
Blue 10 15
White 25 45

Perform a χ 2 at the 5% level of significance to determine whether this year the customers' tastes
as regard to the same as the last year.

Solution:
Step -1
Ho : (Null hypothesis) The two attributes are independent
H1: (Alternate hypothesis) The two attributes are not independent

Step -2 level of significance =5%

Step -3 Formula
(O – E)2
χ 2=∑ E

Step -4

Table to calculate χ 2
Observed (0) Expected (E) (O – E)2 (O – E)2
O-E E
35 30 5 25 0.83
50 45 5 25 0.56
30 15 15 225 15.00
10 15 -5 25 1.67
25 45 -20 400 8.89
150 150 26.95

4|Page
The chi-square (χ 2) value calculated (26.95) is more than this.
Step -5
Degrees of freedom 4

95% cut off value for 4 degrees of freedom is 9.488.

Step -6

Conclusion: The null hypothesis should be rejected. There is a significant difference between
customer taste this year compared to last year.

Test of association
A test which allows the comparison of two attributes in a sample of data to determine if there is
any relationship between them

Question-2 Test of association


A manufacturing company produces items at two different factories.
A survey is carried out to determine if there is an association between the quality of items and
the factory that they are made in. A sample of 100 items gave the following results:
Unsatisfactory Satisfactory Good
quality quality quality Total
Factory A 8 25 21 54
Factory B 4 27 15 46
12 52 36 100

Perform a χ 2 at the 1% level of significance to determine whether there is an association


between the quality of items and the factory that produced them.

Solution:
.
Step -1

Ho (Null hypothesis) The two attributes are


independent
H1 (Alternate) The two attributes are not
independent

Step -2 level of significance = 1%

Step -3
(O – E)2
χ 2=∑ E

5|Page
Step -4

Table to calculate χ 2:
Observed (0) Expected (E) O-E (O – E)2 (O – E)2
E
8 6.48 1.52 2.3104 0.357
4 5.52 -1.52 2.3104 0.419
25 28.08 -3.08 9.4864 0.338
27 23.92 3.08 9.4864 0.397
21 19.44 1.56 2.4336 0.125
15 16.56 -1.56 2.4336 0.147
100 100 1.783

Chi-square value calculated is less than table value


Step -5
Degrees of freedom 2
99% cut off value for 2 degrees of freedom is 9.210.

Step -6

Conclusion: There is no evidence that the null hypothesis should be rejected. There is no
evidence to link quality of items to the factory that they are made in.

FORMULA: CHI-SQUARE WITH YATES’ CONTINUITY CORRECTION

(|Actual− Expected value|−0.5)2 (|𝐎 − 𝐄|−𝟎.𝟓)𝟐


χ2 =∑ =∑
Expected value 𝐄

Where:
| | = Modulus (means ignore the sign)

The correction involves reducing all values of (O – E) by 0.5 regardless of the sign of (O – E).
|10| - 0.5 = 9.5
|−10| -0.5 = 9.5

Question-3 Test of association (if two row and two col are given use continuity correction)
A survey of 50 students at high school showed that they had made the following choices to study
a second European language. Carry out a x2 test at the 5% level of significance to determine
whether there is an association between gender and language chosen.
Unsatisfactory Satisfactory
quality quality Total
Male 14 6 20
Female 8 22 30
22 28 50

6|Page
Perform a χ 2 at the 5% level of significance to determine whether there is an association
between the quality of items and the factory that produced them.

Solution:

Step -1

Ho (Null hypothesis) The two attributes are independent.


H1 (Alternate hypothesis) The two attributes are not independent.

Step -2 level of significance =5%

Step -3
(|𝐎 − 𝐄|−𝟎.𝟓)𝟐

𝐄
Step -4
Table to calculate y2:
Observed (0) Expected (E) |O – E| – 0.5 (|O – E| − 0.5)2
E
14 8.8 4.7 2.510
8 13.2 4.7 1.673
6 11.2 4.7 1.972
22 16.8 4.7 1.315
50 50 7.47

Chi-square value calculated is more than this.

Step -5
Degrees of freedom 1
95% cut off value for 1 degrees of freedom is 3.841.

Step -6

Conclusion: The null hypothesis should be rejected.


There is evidence of association between gender and the language chosen.

Question-4-Test of variance
To judge certain safety features of car an engineer must know whether reaction time of drives to
a given emergency situation has a standard deviation of 0.010 second or whether it is greater
than 0.010 second. What can be concluded at 0.05 level of significance if he gets s=0.014 second
for a sample of size of n=15 Note: (0.010)2 = 0.0001

7|Page
Solution:
Step -1
Null Hypothesis 𝐻0 : 𝜎 2 ≤ 0.0001
Alternative Hypothesis 𝐻1 : 𝜎 2 > 0.0001
Step -2
Level of significance: 𝛼 = 0.05

Step -3
(𝒏−𝟏)𝒔𝟐
Test statistic χ 2 = 𝝈𝟐

Degrees of freedom n – 1 = 15 – 1 = 14

Step -4
(15−1)(0.000196)
Observed value χ 2 = = 27.44
0.0001

Since observed value at 14 degrees of freedom = 27.44 is greater than tabulated value of 23.68,

Step -5
we reject null hypotheses and conclude that there is a significant difference between the two
variances.
Question: 5
A dice was tossed 144 times and following outcomes were recorded:
Faces 1 2 3 4 5 6
Observed 22 23 27 25 26 21
Occurrence
Using chi-square test at 5% level of significance, assess the hypotheses that the dice is fair.
Solution:
Faces Observed occurrence O –E (𝑂 − 𝐸)2 /𝐸
1 22 -2 0.17
2 23 -1 0.04
3 27 3 0.38
4 25 1 0.04
5 26 2 0.17
6 21 -3 0.38
Total 144 1.18

𝟏𝟒𝟒
Average = = 24
𝟔

8|Page
Setp:1 Null Hypotheses 𝐻0 : the die is perfect
𝐻1 : The die is not perfect
Setp:2 level of significance =5%
(O – E)2
Setp:3 χ 2=∑ E

Setp:4 𝑋 2 = 1.18
Setp:5 The rejection region 2𝑋0.05,5 ≥ 1.145

Setp:6 Since the calculated value of 𝑋 2 = 1.18 is not falling in the rejection region we conclude
that die is a perfect die.
Question: 6
A group of people was surveyed about their favorite car. The following results were obtained:
Gender Frequency
Civic Corolla Liana
Male 27 37 11
Female 26 14 5
At 5% level of significance, test the hypotheses that the choice of favorite car is independent of
one’s gender.

Question: 7
The city government has conducted a survey for assessing the inclination of citizens towards
installing backup power arrangements in their homes. It has collected the following data:
Owners Tenants Total
Generator only UPS only 220 200 420
Generator + UPS No backup 160 170 330
140 110 250
180 220 400

700 700 1400

Using chi-square test at 5% level of significance, assess the hypotheses that installation of
backup power arrangement is independent of the types of residents i.e. owners or tenants.
Question: 8
The personal manager of a firm is concerned about ‘absenteeism’. He decided to sample the
records to determine if absenteeism is distributed evenly throughout the six days week. The
sample results are:

9|Page
Weekday Monday Tuesday Wednesday Thursday Friday Saturday
Number 12 9 11 10 9 9
Absent
(a) What is the expected frequency for each day?
(b) How many degrees of freedom are there?
(c) What is the chi-square value at 1% level of significance?
(d) Using the chi-square test of significance compute 𝑋 2
(e) Is the null hypothesis rejected at 1% level of significance?
Solution:
(a) The absenteeism is evenly distributed hence the probability of absenteeism for each
1
day in 6 days is
6

The total frequency = 12+9 +11+10+9+9= 60

1
expected frequency = 60*6 = 10

(b) Degrees of freedom


There are 6 days so degrees of freedom is

(c) n-1 = 6 -1 = 5

The value of 𝑥 2 𝛼0.01 at 5 degrees of freedom is 16.812


(d)
Days Observed Expected o-e (𝑜 − 𝑒)2 (𝑜 − 𝑒)2
value (0) value (e) 𝑒
Monday 12 10 2 4 0.4
Tuesday 9 10 -1 1 0.1
Wednesday 11 10 1 1 0.1
Thursday 10 10 0 0 0
Friday 9 10 -1 1 0.1
Saturday 9 10 -1 1 0.1
8 0.8
(𝑜−𝑒)2
𝑋2 = ∑ = 0.8
𝑒
(e) Since calculated value of 𝑋 2 = 0.8 against tabulated value of 𝑥 2 = 16.812 the null
hypotheses is accepted concluding that absenteeism is evenly distributed.
Question: 9
Given the following data, test at 0.05 level whether there Is any relationship between the level of
education and social activities of an individual?
Education level Social activity

10 | P a g e
Above average Average Below average
Graduation 20 10 10
Matriculation 30 50 80
Primary 10 60 130

11 | P a g e
CHI-SQUARE
15.1 For 𝛼 = 10% the value of χ 2 .05 and χ 2 .95 are:
3.84 and -3.84 3.84 and 0.004
-3.84 and 0.004 None of these
15.2 Chi-square values ranges from:
- ∞ to + ∞ -1 to +1
0 to ∞ None of these
15.3 If we want to test whether or not two samples are independent, the test-statistic to be
selected is:
t-test z-test
Chi-square test None of these
15.4 It is assumed (𝐻0 ) that the political affiliation of a person depends upon the level of
education. The hypothesis was tested using:
Normal distribution Chi square distribution
Binomial distribution None of these
15.5 A CA foundation course instructor believes that the grades of students in foundation
examination depend upon the college. The conjuncture will be tested by using:
Chi-square distribution t-distribution
Normal distribution None of these
15.6 Two populations’ proportions can be compared by using:
Binomial distribution Normal distribution
Chi-square distribution None of these
15.7 More than two (or several) population proportions can be tested simultaneously by using:
Binomial distribution Normal distribution
Chi-square distribution None of these
15.8 The expected frequencies are obtained by multiplying the row total with the column total
and:
Dividing by total number of observations Dividing by row total
Multiplying by total number of observation None of these
15.9 When we want to test the hypothesis concerning population variance the distribution to
be used is:
Normal distribution t-distribution
Chi-square distribution None of these
15.10 A table containing the data classified according to characteristics of population
(attributes) is known as:
Frequency table Contingency table
Correlation table None of these

12 | P a g e
15.11 A technique by means of which we test the hypothesis whether the sample distribution is
in agreement with the theoretical distribution is called:
Parametric test Non-parametric test
Goodness of-fit-test None of these
15.12 A goodness of-fit-test can be applied by using:
Normal distribution t-distribution
Chi-square distribution None of these

1. For a test association at 95% confidence interval, calculated chi-square value is 8.431 whereas
the table value is of 9.488. This means that
(a) Null hypothesis is rejected (b) Null hypothesis is accepted
(c) More information is needed (d)None of the above
2. 2 x 2 contingency table means:

(a) 2 values in each rows and column (b) 2 cells in each row
(c) 2 rows and 2 column (d) 2 cells in reach column
3. chi-square is __________

(a)a two-tailed distribution (b)not a tailed distribution


(c)a one-tailed distribution (d)none of the above
4. For the following set of data, compute the chi-square value:

(a) 0.35 (b)0.45


(c)0.55 (d)0.30
5. Chi-square distribution is a continuous probability distribution having range from ___ to
plus infinity.

(a)Zero (b) One


(c) Negative infinity (d)Negative one
6. A random variable with a Chi-square distribution is always____.

(a)Negative (b)non-negative
(c) positive (d)Infinity
7. Total area under the chi-square cure is_____.

(a) A.one (b) B. zero


(c) C. infinity (d) D. large
8. The Chi-square distribution is single peaked and has ____ skewness.

(a)Negative (b)Positive
(c)Small (d)Large
9. In goodness-of-fit test, the rejection region lies in the ____ tail of the distribution.

(a)Right (b)Left
(c)Right and left (d)Depends upon data

13 | P a g e
10. A ___ test is a technique by which we test whether the observed frequencies in a sampling
distribution are in agreement with the theoretical distribution.

(a) Test of association (b) Hyper geometric


(c) goodness-of-fit (d)chi-square curve
11. What will be the table value of chi-square for:

95% confidence level and 4 degrees of freedom

(a)0.711 (b)9.488
(c)11.070 (d)13.277
12. What will be the table value of chi-square for:

99% confidence level and 4 degrees of freedom

(a)0.711 (b)9.488
(c)11.070 (d)13.277
13. What will be the table value of chi-square for:

95% confidence level and 5 degrees of freedom

(a)0.711 (b)9.488
(c)11.070 (d)13.277
14. What will be the table value of chi-square for:

5% level of significance and 4 degrees of freedom

(a)0.711 (b)9.488
(c)11.070 (d)13.277
15. What will be the table value of chi-square for:

5% level of significance and 5 degrees of freedom

(a)0.711 (b)9.488
(c)11.070 (d)13.277
16. Which of the following cannot be a possible table value of Chi-square:

(a) -0.1 (b)0


(c)53 (d)54
17. What will be the value of degrees of freedom for a contingency table with five rows and
four columns?

(a) 13 (b)12
(c)8 (d)20
18. What will be the value of degrees of freedom for a contingency table with three rows and
two columns?

(a) 5 (b)6
(c)2 (d)1
19. What adjustment is made in the calculated value of chi-square in case of a 2x2
contingency table?

14 | P a g e
(a) Any numerical value for observed (b)Any numerical value for observed minus
minus expected value must be reduced expected value must be increased by 0.5
by 0.5 regardless of sign regardless of sign
(c)Any numerical value for observed vale (d) Nothing needs to be done
must be reduced by 0.5 regardless of sign
20. A ____ compares the occurrence of two attributes in a sample of data to determine if there
is any relationship between them.

(a) Test of association (b)goodness-of-fit


(c)chi square curve (d)frequency polygon
21. The null hypothesis is rejected if the chi-squares value___ for the data is greater than the
cut off value

(a) Calculated (b)observed


(c)expected (d)Judged
22. A dice was tossed 144 times and following outcomes were recorded:

Using chi-square test at 5% level of significance, assess the hypothesis that the dice is fair.

(a) Dice is fair (b)Dice is not fair


(c) Insufficient data (d)Impossible to compute even if more data is
available
23. A group of people was surveyed about their favourite car. The following result were obtained

At 5% level of significance, test the hypothesis that the choice of favourite car is independent of
one’s gender

(a) Favourite car is independent of one’s (b) Favourite car is dependent of one’s gender
gender
(c) Insufficient data (d) Impossible to compute even if more data is
available

15 | P a g e
EXTRA PRACTICE QUESTIONS
Chapter 13,14,15

Question:1 What would change in standard error, if the sample size is decreased from 75 to 40?

a) Increase by 36.93% b) increase by 29,73%


c)Increase by 32.81% d)Increase by 20.73%
Question :2 If a dice rolls 36 times the following observations recorded. Find the value of chi square

Face value of dice Observed Frequencies


1 8
2 6
3 10
4 5
5 4
6 3

a) +54.92 b) -54.92
c) Both (a) and (b) d) None of these
Question :3 level of significance is also referred as

a) Probability of Type I error b) Probability of type II Error


c) Probability of acceptance area d) Probability of other than rejection area
Question :4 Define hypothesis

An assumption about certain characteristics of a population. If it specifies values for every parameter of
a population, it is called a simple hypothesis; if not, a composite hypothesis. If it attempts to nullify the
difference between two sample means (by suggesting that the difference is of no statistical significance),
it is called a null hypothesis.

Question :5 Define inferential statistics

It makes inferences about populations using data drawn from the population. Instead of using the entire
population to gather the data, the statistician will collect a sample or samples from the millions of
residents and make inferences about the entire population using the sample.

Question :6 which of the following is used to draw conclusion about population on the basis of
sample

a) Null hypothesis b) Alternative hypothesis


c) Inferential statistics d) Normal distribution
Question:7 If mean is at center and frequency is distributed evenly throughout data, then data is

a) Positively skewed b) Negatively skewed


c) Normally distributed d) Balanced

16 | P a g e
Question :8 if peak of a histogram is at center and frequency is distributed evenly throughout data,
then data is

a) Positively skewed b) Negatively skewed


c) Normally distributed d) Balanced
Question 9 If the peak of the histogram is in the middle and the frequencies on either side are similar
to each other, then distribution is said to be

a) Normal b) Balance
c) Binomial d) Symmetric
Question :10-Recent bomb blasts in capital cities will decrease 10% votes of PMLN" The above
statement expresses

a) Null hypothesis b) Inferential statistics


c) Hypothesis testing d) None of these
Question: 11 Which distribution is more accurate while testing the difference of proportions in larger
samples

a) z-test b) chi-square
c) student's distribution d) probability distribution
Question: 12 A business researcher wanted to evaluate the eating habits of England residents from a
rural site, such mothers which have less than 3 babies. The sampling used for this purpose is called

a) Stratified b) Cluster
c) Systematic d) Quota sampling
Question: 13 A pharmaceutical company survey in rural area and brief their teams to collect the
sample by interviews from only those mothers who have less than 3 babies. The sampling used for
this purpose is called

a) Stratified b) Cluster
c) Systematic d) quota sampling
Question: 14 "the machine is working properly with the same average output level as given is last
year" The above statement represents:

a) statistical hypothesis b) null hypothesis


c) alternate hypothesis d) both (b) and (c)
Question: 15 A company has 250 employees and issue 200 forms to its employees to give feedback on
its products but only 180 employees gave feedback. What is sample size

a) 250 b) 200
c) 180 d) None of these
Question: 16 A company has 250 employees and issue 200 forms to its employees to share the views
about its new remuneration package, only 150 employees return forms and 80 support the package.
What is sample size of the company?

a) 250 b) 200
c) 180 d) None of these

17 | P a g e
Question: 17 A sample size is reduced from 75 to 40, standard error word:

a) Increase b) Decrease
c) Remain same d) None of these
Question: 18 Degree of freedom exist in

a) T test b) Goodness
c) Both A & B d) None of these
Question: 19 T-test depends on which of the following:

a) Population size b) Sample size


c) Population variance d) Population standard deviation
Question: 20 If a= 0.05 what will be the value of z from table in a two tails test

a) 2.33 b) 1.645
c) 1.96 d) 2.58
Question: 21 What will be the effect of change in sample size from 75 to 40 in standard error.

a) Increased by 36.93% b) decreased by 36.93%


c) increased by 10.5% d) decreased by 38.12%
Question: 22 A sample has been taken from normally distributed population and sample mean has been
found to be 62. The upper confidence limit of 95% for population mean is 84.60, population variance is
known to be 2401. What is sample size

a) 18 b) 20
c) 17 d) 25
Question: 24 In a Rice mill the bags of rice has mean weight of 5.05 kg and standard deviation of 0.02 kg.
if a bag is selected at random then find the probability that its weight is below 5 kg

a) 0.62 b) 0.0062
c) 0.4938 d) None of these
Question: 25 The probability of rejecting the true hypothesis is

a) Type I error b) Level of significance


c) Both a and b d) None of these
Question: 26 A sociologist was researching the question. Is there any relationship between the level of
education and social activities of an individual? She decided on three level of education attended or
completed university, attended Off completed college, and attended or completed secondary school or
less. Each individual kept a record of his or her social activities. The sociologist divided them into above-
average frequency, average frequency and below-average frequency. These data are shown in table;

Education Above Average Belove Average Average


University 10 10 20
College 30 50 80
Secondary School 20 60 120
Question :27 What is the name of this table?

a) Two ways tables b) Frequency table


c) Stem and leaf table d) Contingency table

18 | P a g e
Question :28

State null hypothesis

a) independent attributes b) Dependent attributes


c) Parallel attributes d) None of these
Question: 29

State alternative hypothesis

a) Independent attributes b) Dependent attributes


c) Parallel attributes d) None of these
Question :30 What is the tabulated value of test statistic?

a) 11.143 b) 9.488
c) 1.96 d) 1.645
Question:31

What is the calculated value of test statistic?

a) 9.49 b) 2.45
c) 1.35 d) 45
Question :32 Normal distribution is used when

a) Sample size is below 30 b) Sample size is above 30


c) SD of population is unknown d) None of these
Question: 33 If population standard deviation 12.5 and value of n-70 then standard mean is?

a) 1.494 b) 2.942
c) 5.945 d) 10
Question: 34 Find the tabulated value of 97%confidence interval

a) 2.33 b) 2.58
c) 2.17 d) 2.07
Question: 35 Find the tabulated value of 92% confidence interval

a) 2.33 b) 2.58
(c) 1.75 d) 2.07
Question: 36 Find the tabulated value of 88% confidence interval

a) 1.55 b) 1.56
c) 1.65 d) 1.555
Question: 37 With a sample of size 900, standard error is 3. What should be sample so that we could be
95% confident that population mean is within 4 units of sample mean

a) 1945 b) 2000
c) 1804 d) 1250
Question: 38 You are given X-80, -72, C1-95% n-50 Find 95% confidence interval of population mean

a) 78- - - - 82 b) 79- - - - 81
c) 76- - - - - 84 d) 75 - - - - - -85

19 | P a g e
Question: 39 You are given following data

∑𝑥 2 = 1850, 𝛴𝑥 = 160 𝑛 = 16
Find standard error

a) 1.02 b) 4.08
c) 1.04 d) 5.33
Question: 40 Intelligence of students between two cities can be tested by which of the following

a) Chi square-test b) Z-test


c) T-test d) None of these
Question: 41 N=300 male = 109

Find 95% confidence interval of male students in college

a) b)
c) d)
Question: 42 Properties of chi square distributions are?

a) Mean>Mode b) Mean=Mode
c) Mean <Mode d) None of these
Note: chi square distribution is positively skewed distribution so mean>Mode

Question: 43 A medicine company claims that average alcohol in their medicine is 0,767 ml. In order to
test the claim a sample of 45 units is selected having average alcohol content of 0.78mg and S.D=0.02
test the claim at a =0.05

Find calculates value

Find critical value

Whether Ho is accepted or rejected

One tail test or two tail tests

Question: 44 Yates correction is used in which of the following?

a) Binomial b) Poisson
c) Chi-square d) Normal
Question: 45 Which of the following is not expected to be normally distributed?

a) Height of men with age 20 years b) Age of students in a class


c) Age of entire population in country d) Both b and c
Question: 46 What are the properties of chi-square

Sol

(1) Positively skewed distribution

(2) Ranges from 0 to ∞

(3) Mean>Mode

20 | P a g e
(4) One sided test is conducted

(5) Used to test variance of population and independence of attributes

Question: 47 A college management wants to assess the intelligence of class students, for this purpose
which of the following distribution will be used?

a) T-test b) Z-test
c) X² test d) Anova test
Question: 48 To test similarity between two or different variables which of the following test is used

a) Z-test b) t-test
c) X² test d) 2 test and X² test
Question: 49 T-test is used regarding

a) Population Mean b) Sample mean


c) Population Standard deviation d) Sample Standard deviation
Question: 50 In a sample of 700 people in a city 313 are found men. You are required to

construct a 95% confidence interval of all the men in a city.

a) b)
c) d)
Question: 51 Sampling is used to test hypothesis about

a) Sample mean b) Population mean


c) Sample standard deviation d) None of these
Question: 52 A distribution with histogram at the center and similar values before and after it, is called

a) Symmetric distribution. b) Balanced distribution


c) Binomial distribution d) Normal distribution
Question: 53 A pharmaceutical company claims that amount of alcohol in a particular drug is 0.706mg. a
sample of size 38 is selected to test the claim and its mean= 0.705mg and standard deviation is 0.02mg.
level of significance is 0.05. which of the following statement is/a correct?

Statement 1: calculated value of test statistics is 0.308

Statement 2: tabulated value is 1.96

Statement 3: claim is accepted

a) 1,2 b) 2,3
c) 1,3 d) All are correct
Question: 54 Intelligence of students of two colleges can be compared by

a) Normal distribution b) Sampling distribution


c) Chi-square distribution d) T-distribution.
Question: 55 A sample size is increased from 10 to 100 standard error is decreased by

a) 68.38% b) 36.62%
c) 27.65% d) None of these

21 | P a g e
Question: 56 How many samples of size 3 can be drawn without replacement from a population of size
5?

a)12 b) 10
c)15 d)25
Question: 57 Which sampling provides separate estimates for population mean for different groups and
also an overall estimate?

(a) Simple random sampling b) Systematic sampling


c) Stratified random sampling d) Judgement sampling
Question: 58 The arithmetic mean of the upper and lower limits of the confidence interval for
population

a) Sample mean b) Population standard deviation


c) Population mean d) Sample standard deviation
Question: 59 If you want to test the claim that automobile is driven on the average more than

20,000 km per year. The null and alternative hypothesis will be:

a) μ =20,000 μ≠20,000 b) =20,000 μ >20,000


c) μ >20,000 μ≠20.000 d) μ =20,000 μ <20,000
Question: 60 If it is known that the 95% confidence limits to population mean are 48.04 and 51.96, what
is the value of the population variance when the sample size is 100?

a) 1 b) 100
c) 10 d) Can't be determined
Question: 61 The maximum speed limit on a busy road is 60 km/h. Congestion results in much slower
actual speeds. A random sample of 57 vehicles gave an average speed of 23.2 km/h with a standard
deviation of 0.3 km/h. What are the upper and lower limits of the confidence interval for the mean
speed, given a confidence level of 95%?

a) (23.12--23.28) b) (22.12----21.28)
c) (20.12----23.28) d) (23.12----20.28)
Question: 62 When population is heterogeneous, best suitable method for sampling is:

a) Cluster Sampling b) Quota Sampling


c) Stratified Sampling d) Systematic Sampling
Question: 63 The process of making an interval based on sample observations containing unknown value
of the population parameter with a known probability is called:

a) Point Estimation b) Interval Estimation


c) Stratified Sampling d) Random Sampling
Question: 64Cluster sampling is an example of:

a) Random sampling b) Probability Sampling


c) Non Random Sampling d) Non probability sampling
Question: 65In a binomial distribution

a) Mean = Variance b) Mean<Variance

22 | P a g e
c) Mean >= Variance d) Mean>Variance
Question: 66 The standard deviation of a binomial distribution depends on:

a) Probability of success b) No. of trials


c) Probability of failure d) All of the above
Question: 67 With a sample size of 900, the standard error is 3. What should be the sample size so that
we could be 95% confident that the population mean is within 4 of the sample mean?

a) 90 b) 1945
c) 1235 d) 1250
Question: 681f we take 1 person randomly from any group and then take 1 person after every 3 people
in the list, then what kind of probability sampling is this?

a) Systematic b) Stratified
c) Multiphase d) Cluster
Question: 69 A random sample of 100 units gave a mean of 74.8kg and S.D =8kg. find the upper limit of
95% confidence interval for population mean

a) 73.232 b) 76.368
c) 87.344 d) None of these
Question: 70 The maximum speed limit on a busy road is 60 km/h. Congestion results in much slower
actual speeds. A random sample of 57 vehicles gave an average speed of 23.2 km with a standard
deviation of 0.3 km/h. What are the upper and lower limits of the confidence interval for the mean
speed, given?

a confidence level of 95%

a) (23.12----23.28) b) (22.12--21.28)
(c) (20.12----23.28) d) (23 12----20.28)
Definition of cluster sampling, quota sampling & systematic sampling

Answer

Read study text chapter 13

Question: 71Cluster sampling is an example of:

a) Random sampling b) Non Random Sampling.


c) Probability Sampling d) Non probability sampling
Question: 72 Confidence interval for estimating population mean when population SD is unknown and
n<30 is?
𝑧𝛼 𝜎 𝑡𝛼 𝜎
a) 𝑥̅ ± b)𝑥̅ ± (𝜂 − 1)
2 √𝜂 2 √𝑛
𝑧𝛼 𝑆 𝑡𝛼(𝑛−1) 𝑆
c) 𝑥̅ ± d) 𝑥̅ ±
2 √𝑛 2 √𝑛
Question :74 A finite population of size 324 have mean 18, a sample of size 25 is selected, find mean of
sampling distribution

23 | P a g e
a)25 b)324
c)18 d)3.6
Question: 75 For a population consisting of heterogeneous groups, which sampling technique is more
reliable?

a) Simple random sampling b) Systematic Sampling


c) Stratified Random Sampling d) None of these
Question: 76 Chi Square distribution is used to test the hypothesis concerning:

a) The independence of two attributes b) Goodness of Fit


c) Population mean for large sample size d) (a)and (b) but not (c)
Question: 77 From an industrial area 70 companies were selected at random and 45 of them were
planning for expansion next year. Find 95% confidence limits for the proportion of companies planning
for expansion:

a) 0.35,0.57 b) 0.35, 0.75


c) 0.53, 0.75 d) 0.35, 0.77
Question: 78 Which sampling provides separate estimates for population mean for different groups and
also an overall estimate?

a) Simple random sampling b) Systematic sampling


c) Stratified random sampling d) Judgement sampling
Question: 79 A candidate for mayor in a large city believes that he appeals to at least 10 percent more of
the women voters than the men voters. He hires the services of a poll-taking organization, and they find
that 62 of 100 women interviewed support the candidate, and 69 of 150 men support him.

At the 0.05 significance level, is the hypothesis accepted or rejected?

a) accept null hypothesis H0 b) reject null hypothesis Ho


c) data is incomplete d) none of these
Question :80 A sample of 4 observations 2, 4, 6, 8 is drawn from a population. What is the standard
error?

a) 1.12 b) 1.29
c) 2.236 d) 1.25
Question: 82 Which of the following is continuous distribution?

a) Z-distribution b) T-distribution
c) Chi-square distribution d) All of these
Question: 83 The probability of rejecting a True hypothesis:

a) Level of significance b) Level of confidence


c) Type-1 error d) Both (a) and (c)
Question: 84 With a sample size of 900, the standard error is 3. What should be the sample size so that
we could be 95% confident that the population mean is within 4 of the sample mean?

(a) 90 b) 20
c) 1945 d) none of these

24 | P a g e
Question: 86 For a population consisting of heterogeneous groups, which sampling technique is more
reliable?

a) Simple random sampling b) Systematic Sampling


c) Stratified Random Sampling d) none of these
Question: 87Chi Square distribution is used to test the hypothesis concerning:

a) The independence of two attributes b) Goodness of Fit


c) Population mean for large sample size d) (a)and (b) but not (c)
Question: 88 From an industrial area 70 companies were selected at random and 45 of them were
planning for expansion next year. Find 95% confidence limits for the proportion of companies planning
for expansion:

a) 0.35, 0.57 b) 0.35, 0.75


c) 0.53, 0.75 d) 0.35, 0.77
Question: 89 Which sampling provides separate estimates for population mean for different groups
and also an overall estimate?

a) Simple random sampling b) Systematic sampling


c) Stratified random sampling d) Judgement sampling
Question: 90 A candidate for mayor in a large city believes that he appeals to at least 10 percent more of
the women voters than the men voters. He hires the services of a poll-taking organization, and they find
that 62 of 100 women interviewed support the candidate, and 69 of 150 men support him. At the 0.05

significance level, is the hypothesis accepted or rejected?

a) accept null hypothesis H0 b) reject null hypothesis Ho


c) data is incomplete d) none of these
Question: 91 If you want to test the claim that automobile is driven on the average more than 20,000
km per

year. The null and alternative hypothesis will be:

a) μ = 20,000 μ ≠ 20,000. b) μ >20,000 μ ≠ 20,000


c) μ =20,000 μ >20,000 d) μ =20,000 μ<20,000
Question: 92 If the hypothesis does not completely specify the value of all the parameters, then it is
called:

a) Composite Hypothesis b) Null Hypothesis


c) Alternative Hypothesis d) Simple Hypothesis
Question: 93The probability of rejecting a True hypothesis:

a) Level of significance b) Level of confidence


c) Type-1 error d) Both (a) and (c)
Question: 94 If we increase the level of significance, the range of acceptance region is:

a) Increased b) Decreased
c) No change d) None of these

25 | P a g e
Question: 95 A population has a mean of 75 and a standard deviation 20 are chosen and the sample
means recorded. What is the standard deviation of the sample means?

a) 10 b) 75
c) 2.24 d) none of these
Question: 96 Samples of size 81 are taken from a population of size 500 with mean 10 and standard
deviation

3. Find the standard deviation of the sample means.

a) 0.31 b) 0.33
c) 0.92 d) 3
Question: 97 As population size increases, the value of the finite population standard deviation of the
sample mean

a) Decreases b) Gets closer to the infinite population standard


deviation of the sample mean
c) Both a and b d) Increases
Question: 98 The admissions office at a college reported that this year's freshman class had an average
SAT score of 1103 with a standard deviation of 95. What is the probability that a sample of 40 students
from this year's freshman class had an average score greater than 1120?

a) 0.1292 b) 0.3708
c) 0.6292 d) 0.8707
Question: 99 40% of the employees at a large corporation are female. A sample of 50 employees is
taken and gender is recorded. What is the probability of a sample proportion of females in the sample
between 35 and 45?

a) 0.0693 b) 0.2649
c) 0.5284 d) 0.72
Question: 100 If the expected value of a point estimator is equal to the population parameter, then the
point estimation is known as consistent.

a) True b) False
c) Depends on circumstances d) No discrete conclusion be drawn
Question: 101 A study of voting chose 663 registered voters at random shortly after an election. Of
these, 72% said they had voted in the election. Election records show that only 56% of registered voters
voted in the election. Which of the following statements is true about the boldface numbers?

a) 72% is a parameter and 56% is a b) 72% is a sample and 56% is a population.


statistic.
c) 56% is a parameter and 72% is a statistic. d) 72% and 56% both are parameter
Question: 102 The Gallup Poll has decided to increase the size of its random sample of voters from
about 1500 people to about 4000 people right before an election. The Poll is designed to estimate the
proportion of voters who favor a new law banning smoking in public buildings. The effect of this increase
is to

a) Reduce the variability of the estimate. b) Have no effect since the population size is the
same.

26 | P a g e
c) Increase the variability of the estimate. d) Reduce the bias of the estimate.
Question: 103 A box contains 57 calculators out of which 36 are defective, if 4 calculators are selected
from the box then find probability that all are defective.

a) 0.8509 b) 0.01515
c) 0.1491 d) None of these
Question: 104 50% people use cellphone for writing notes, 40% people use paper, 20% people use both.
If a person is selected at random find the probability that he use neither for taking notes

a) 10% b) 30%
c) 20% d) 60%
Question: 105 Standard deviation of 10 values is 12, if A.M is 246 then find sum of squared observations

a) 606.6 b) 60660
c) 606600 d) 6.066
Question: 106 A population is normally distributed with mean 6000 and standard deviation of 400, if a
value is randomly selected find the probability that value is above 6800.

a) 0.0228 b) 1.0228
c) 6.028 d) None of these

27 | P a g e

You might also like